You are on page 1of 516

Internal Medicine

A 65 year old white female comes to your office with evidence of a fecal
impaction which you successfully treat. She relates a history of chronic
laxative use for most of her adult years. After proper preparation, you
perform sigmoidoscopy and note that the anal and rectal mucosa contain
scattered areas of bluish-black discoloration. Which one of the
following is the most likely explanation for the sigmoidoscopic findings‟

a) Endometriosis
b) Collagenous colitis
c) Melanosis coli
d) Metastatic malignant melanoma
e) Arteriovenous malformations
The correct answer is C

Explanation
This patient has typical findings of melanosis coli, the term used to
describe black or brown discoloration of the mucosa of the colon. It
results from the presence of dark pigment in large mononuclear cells or
macrophages in the lamina propria of the mucosa. The coloration is
usually most intense just inside the anal sphincter and is lighter
higher up in the sigmoid colon. The condition is thought to result from
fecal stasis and the use of anthracene cathartics such as cascara
sagrada, senna, and danthron. Ectopic endometrial tissue (endometriosis)
most commonly involves the serosal layer of those parts of the bowel
adjacent to the uterus and fallopian tubes, particularly the
rectosigmoid colon. Collagenous colitis does not cause mucosal
pigmentary changes. Melanoma rarely metastasizes multicentrically to the
bowel wall. Multiple arteriovenous malformations are more common in the
proximal bowel, and would not appear as described.

A 30-year-old female presents to your office with a clear nasal


discharge, sneezing, nasal congestion, and nasal itching. She notes that
these symptoms generally occur in the spring and fall.

The most effective drug for treatment and prevention is

a) Cetirizine (Zyrtec)
b) Cromolyn nasal spray (NasalCrom)
c) Ipratropium nasal spray (Atrovent)
d) Montelukast (Singulair)
e) Fluticasone nasal spray (Flonase)
The correct answer is E

Explanation
While all of the options listed are effective for allergic rhinitis,
topical intranasal steroids such as fluticasone nasal spray are the most
effective both for prevention and symptom relief.

Which of the following is not a treatment for an adult patient with


Wolff-Parkinson-White Syndrome (WPW)?
a) Cardioversion
b) Procainamide
c) Adenosine
d) Magnesium
The correct answer is D

Explanation
Wolff-Parkinson-White syndrome is a disorder in which an extra
electrical connection between the atria and the ventricles is present at
birth.

The treatment of choice is direct-current cardioversion. If


cardioversion is not possible, drugs that prolong the refractory period
of the accessory connection should be used. Examples are IV procainamide
and adenosine.

Magnesium is the treatment for torsade de pointes.


A 60-year-old male with a 10-year history of reasonably well-controlled
type 2 diabetes mellitus hyperlipidemia, and hypertension presents to
your office for follow-up. He complains of dyspnea on exertion and
swelling of his lower legs and feet. He has not had any chest pain. His
current medications include glyburide (Micronase, DiaBeta), atorvastatin
(Lipitor), hydrochlorothiazide, and daily aspirin.

He is moderately overweight, with a BMI of 29, and his blood pressure is


150/95 mm Hg. Examination reveals a few moist rales in both lung bases,
a soft S3 heart sound, and 3+ pretibial and pedal edema. A chest
radiograph reveals mild cardiomegaly and cephalization of the pulmonary
vasculature. Echocardiography reveals an ejection fraction of 25%, with
dyskinesia of the anterior left ventricular wall and mild mitral
insufficiency.

Therapeutic agents that have been shown to improve survival in such


cases include all of the following EXCEPT

a) Lisinopril (Prinivil, Zestril)


b) Valsartan (Diovan)
c) Digoxin
d) Metoprolol (Lopressor)
The correct answer is C

Explanation
This patient has systolic heart failure, most likely related to a silent
myocardial infarction, and exacerbated by hypertension. Digoxin improves
symptoms and decreases hospitalizations due to heart failure, but does
not prolong survival. ACE inhibitors, angiotensin receptor blockers, and
Beta-blockers have all been shown to improve survival in heart failure.

An 80 year old man who is taking warfarin because of a history of atrial


fibrillation, is found on routine blood work to have an INR of 7.2. He
reports no bleeding. Which one of the following is the best immediate
management of this problem?
a) Hold the warfarin for 3 days
b) Vitamin K 1 mg orally
c) Vitamin K 10 mg intravenously
d) 4 units of fresh frozen plasma in addition to 10mg of IV Vitamin K
e) Desmopressin (DDAVP)
The correct answer is A

Explanation
If the INR is between 5.0-9.0 and bleeding is absent, you should cease
warfarin therapy; consider reasons for elevated INR and patient-specific
factors. If bleeding risk is high, give vitamin K (1.0-2.0 mg orally or
0.5?1.0 mg intravenously). Measure INR within 24 hours, resume warfarin
at a reduced dose once INR is in therapeutic range.

However, if there is any clinically significant bleeding where


warfarin-induced coagulopathy is considered a contributing factor, you
should cease warfarin therapy, give 5.0-10.0 mg vitamin K intravenously,
as well as fresh frozen plasma (150-300 mL), assess patient continuously
until INR < 5.0, and bleeding stops.

An asymptomatic 38-year-old white male takes lisinopril (Prinivil,


Zestril) for hypertension. At a routine follow-up visit you order a
basic metabolic profile. The results are normal except for a serum
potassium level of 6.3 mEq/L (N 3.5-5.0).

Which one of the following should you do at this point?

a) Order another serum potassium level but make sure the patient
repeatedly clenches his
fist during the phlebotomy
b) Order another metabolic profile but make sure the sample stands for
several hours
before being centrifuged
c) Reassure the patient that the potassium elevation is likely due to a
laboratory error and
disregard the result
d) Temporarily discontinue the lisinopril and order a repeat potassium
level
The correct answer is D

Explanation
Pseudo-hyperkalemia is most commonly caused by lysis of red cells
released from platelets in clotted serum. Clenching the fist repeatedly
can aggravate red cell damage. As the patient is on an ACE inhibitor
that can raise potassium levels, reassurance without investigation would
not be appropriate. A plasma potassium level does not reflect potassium
released from cells damaged by phlebotomy.
A 19 year old female presents with pain and swelling of her right knee
and no other symptoms. X-ray shows joint effusion. She has been sexually
active since she was 15 and is seeing a gynecologist for follow-up of
cervical dysplasia. Rheumatoid factor was negative. Which one of the
following is the most likely diagnosis‟

a) Osteoarthritis
b) Gouty arthritis
c) Pseudogout
d) Gonococcal arthritis
e) Systemic lupus erythematosus
The correct answer is D

Explanation
Gonococcal arthritis is a more focal form of disseminated gonococcal
infection (DGI) that results in a frank septic arthritis with effusion.
Some patients have previous or coincident symptoms of DGI. Usually only
1 or 2 joints are involved, primarily the knees, ankles, wrists, and
elbows. Onset is often acute, with fever, severe pain, and limitation of
movement but may occur without constitutional symptoms. Infected joints
are swollen, and the overlying skin may be warm and red.

In 30 to 40% of patients with DGI, blood cultures are positive in the


1st week of illness. With septic arthritis, blood cultures are less
often positive, but joint fluids are more often positive. Isolated,
frank, acute arthritis in a sexually active patient requires joint
aspiration to diagnose gonococcal infection. Fluid is usually purulent
(WBCs > 20,000/mL). Cultures of joint fluid are positive in 40 to 50%,
but organisms are rarely visible on Gram stain. PCR testing may be more
sensitive but has not been evaluated.
An elderly female who has been your patient for several years is
discovered lying on the floor of her kitchen by a Meals-on-Wheels
volunteer. She is transported to the hospital in an unresponsive state.
After a thorough evaluation, you diagnose a massive cerebral infarct. On
several previous occasions the patient verbalized to you her desire to
not be subjected to life-prolonging treatments should she ever be
rendered incapacitated; however, she declined your suggestion that she
confirm this in writing. The patient is admitted to the hospital with
“do not resuscitate” (DNR) orders and supportive measures are
instituted. A neurology consultant evaluates her and agrees that her
condition is terminal and irreversible. The patient‟s nephew is angered
by the DNR status and, noting that she has insurance coverage, demands
every medical treatment that might prolong his aunt‟s life, including
resuscitation.

Which one of the following would be the best course of action in terms
of legality and ethics‟

a) Institution of aggressive medical therapies, including full


resuscitation in the event of
cardiac arrest and ventilator support in the event of respiratory arrest
b) Continuation of a supportive treatment plan, provided full
resuscitation is initiated in
the event of cardiac arrest and ventilator support is provided in the
event of respiratory
arrest
c) Performance of full CPR for cardiopulmonary arrest, but no
ventilator support for
respiratory failure
d) Continuation of the current treatment plan
The correct answer is D

Explanation
An adult patient or their legally authorized representative has the
right to refuse any medical treatment, regardless of its likelihood of
success; however, there is no legal right to receive any and all
treatment demanded. When patients have explicit advance directives in
writing, their wishes are clear. When no written document exists, but
the patient‟s desire is well known to his or her physician, the
physician is ethically bound to honor these wishes. This responsibility
must be balanced against the physician‟s ethical obligation not to
perform futile treatments of no benefit to the patient. A family member
acting as a medical decision-making proxy is obligated to represent what
they believe to be the patient‟s wishes, even in the face of conflict
with their personal beliefs.

This case highlights two commonly encountered issues: honoring the


patient‟s wishes when there is conflicting evidence of what their wishes
may be, and withholding medically futile treatment. In this case, the
futility of the patient‟s condition overrides any confusion as to her
wishes, and the admission treatment plan is the most appropriate.
Naturally, it would be best to discuss this with the nephew and attempt
to arrive at a consensus, with the patient‟s best interest being the
primary concern.
A 45 year old white male comes to your office with a 2-day history of
pain and swelling in the right testicle. He has no dysuria or urinary
frequency, and denies any sexual contact except with his wife. On
examination you note tenderness in the right posterior aspect of the
right testicle, along with some swelling and erythema of the overlying
scrotal skin. Which one of the following is true regarding this situation?

a) The most likely etiologic agent is Chlamydia trachomatis


b) Immediate surgical referral is indicated
c) Anaerobic bacteria are the infecting agents in most cases
d) The patient should be treated symptomatically until results from a
urethral culture are available
e) Antibiotic therapy should include coverage for coliform bacteria
The correct answer is E

Explanation
Epididymitis is an inflammation of the epididymis due to various
infectious agents or to local trauma. Pain may develop over a day or
two, or even more gradually. In men under the age of 35 whoa re sexually
active, the most common etiologic organisms are Chlamydia trachomatis
and Neisseia gonorrhoeae. In men over 35, infectious epididymitis is
usually nonspecific and is caused by coliform bacteria or Pseudomonas
species. The preferred treatment is ofloxacin, 400 mg orally twice a day
for 10 days. Alternative therapy is a single dose of ceftriaxone, 250
mg, plus doxycycline, 100 mg twice a day for 10 days.
Of the following, the best laboratory study to evaluate nutritional
status in an elderly patient is

a) Folate levels
b) Iron studies
c) Serum creatinine
d) Serum albumin
e) Serum electrolytes
The correct answer is D

Explanation
Serum albumin is sensitive to changes in nutritional status, although
its long half-life (17-21 days) means that return to normal is slow. A
lymphocyte count provides a quick estimate of immune status and low
counts may indicate a need for nutritional assessment. Iron studies are
more likely to be abnormal as a result of gastrointestinal blood loss.
Folate level mostly reflect generic nutritional deficiency, rather than
a specific deficiency. A low folate level is consistent with consumption
of a limited variety and quantity of food. Electrolyte levels do not
reflect nutrition.
A 30-year-old healthy female presents with a temperature of 38.3C
(101.0F), dyspnea, nonproductive cough, bronchial breath sounds, and
rales in the right mid-lung field. She has no allergies to medication or
associated medical problems.

This patient should receive

a) Empiric antibiotic treatment


b) Antibiotics only if a chest radiograph is abnormal
c) Antibiotics only if blood cultures are positive
d) Antibiotics only if a sputum Gram's stain shows pathologic organisms
e) No antibiotics
The correct answer is A

Explanation
This patient has findings typical of community-acquired pneumonia, and
thus should receive antibiotics, most likely empirically. Early in the
process, a radiograph may be negative (which would be a false-negative).
Sputum and blood cultures take several days for results, which would
delay treatment, and are positive only in a small percentage of cases.
An increase in the alveolar-arterial PO2 difference may occur in

a) Asthma
b) Acute hypoventilation
c) Anemia
d) Mild exercise
e) High altitude
The correct answer is A

Explanation
The difference between the calculated PAO2 and the measured PaO2 is
called the Alveolar-arterial PO2 difference or 'A-a Gradient' for short.
The A-a gradient answers the important question: Are the lungs
transferring oxygen properly from the atmosphere to the pulmonary
circulation If the A-a gradient is elevated, the answer is no. If the
A-a gradient is normal the answer is yes.

Causes of increased (A-a) difference:


Alveolar collapse(atelectasis), Intaalveolar filling(pneumonia,
pulmonary edema), Intracardiac shunt, Vascular shunt within lungs,
Asthma, COPD, ILD, Alveolar disease, Pulmonary vascular disease.

Hypoventilation alone can't produce increased (A-a) diff


Hemoglobin level has an ignorable effect on (A-a) diff
In high altitude the inspired O2 is decresed so the (A-a)diff has no
increase

A 45 year old man presents with acute pleuritic chest pain. He is


sitting up in bed leaned forward when you enter the examining room.
Physical exam reveals a friction rub on cardiac exam. Which of the
following is appropriate in treatment?
a) Antiarrhythmic
b) Beta-blocker
c) Albuterol
d) Naproxen
The correct answer is D

Explanation
Pericarditis is inflammation of the pericardium, often with fluid
accumulation. Pericarditis may be caused by many disorders (eg,
infection, MI, trauma, tumors, metabolic disorders) but is often
idiopathic. Symptoms include chest pain or tightness, often worsened by
deep breathing. Unlike ischemic chest pain, pain due to pericarditis is
usually aggravated by thoracic motion, cough, breathing, or swallowing
food; it may be relieved by sitting up and leaning forward.

Diagnosis is based on symptoms, a friction rub, ECG changes, and


evidence of pericardial fluid accumulation on x-ray or echocardiogram.
Finding the cause requires further evaluation. Treatment depends on the
cause, but general measures include analgesics (eg codeine),
anti-inflammatory drugs (eg naproxen), and sometimes surgery.
A middle-aged hairdresser presents with a complaint of soreness of the
proximal nail folds of several fingers on either hand, which has slowly
worsened over the last 6 months. The nails appear thickened and
distorted. Otherwise she is healthy and has no evidence of systemic
disease. Which one of the following would be the most effective initial
treatment?

a) Soaking in a dilute iodine solution twice daily to cleanse and


sterilize the nail beds
b) Oral amoxicillin/clavulanate (Augmentin) for up to 4?6 weeks
c) Topical betamethasone dipropionate (Diprolene) applied twice daily
to the nail folds for 3-4 weeks
d) Rheumatologic and autoimmune workups for HIV, hepatitis C,
psoriasis, and rheumatoid arthritis
The correct answer is C

Explanation
Chronic paronychia is a common condition in workers whose hands are
exposed to chemical irritants or are wet for long periods of time. This
patient is an otherwise healthy hairdresser, with frequent exposure to
irritants.

The patient should be advised to avoid exposure to harsh chemicals and


water. In addition, the use of strong topical corticosteroids over
several weeks can greatly reduce the inflammation, allowing the nail
folds to return to normal and helping the cuticles recover their natural
barrier to infection.

Soaking in iodine solution would kill bacteria, but would also


perpetuate the chronic irritation. Because the condition is related to
chemical and water irritation, a prolonged course of antibiotics should
not be the first treatment step, and could have serious side effects.
There is no need to explore less likely autoimmune causes for nail
changes at this time.
A 70 year old white male whom you are treating for hypertension has
several abnormal liver function tests. He is a nondrinker and is not
taking any prescription medications likely to cause hepatotoxicity.
However, during more extensive history taking, he tells you that he does
use some over-the-counter medications. Which one of these is most likely
responsible for the abnormal laboratory findings‟

a) Aspirin, used occasionally for headache


b) A fiber supplement taken to promote regular bowel habits
c) One long-acting niacin tablet per day
d) One 250-mg vitamin C tablet daily
e) Chewable simethicone after meals, almost daily
The correct answer is C

Explanation
Hepatotoxicity resulting from timed-release formulations of niacin has
been reported in elderly individuals. Patients may be taking this drug
without their physician‟s knowledge, feeling it is safe because it is a
vitamin.
Niacin can cause abnormal liver function tests, high blood sugar levels
and muscle pains. If a patient takes niacin, he/she should have blood
drawn for liver function tests every 6 to 8 weeks.

A 38 year old male presents with the following rash on his foot. It is
shiny, tender and has a well demarcated margin. He is also febrile. See
picture:

What is the most likely diagnosis‟

a) Osteomyelitis
b) Scalded skin syndrome
c) Acute cellulitis
d) Erysipelas
The correct answer is D

Explanation
Erysipelas is characterized clinically by shiny, raised, indurated, and
painful plaque-like lesions with distinct margins. There is also a
bullous form of erysipelas. Erysipelas is most often caused by group A
beta-hemolytic streptococci and occurs most frequently on the legs and
face. It is commonly accompanied by high fever, chills, and malaise.
Erysipelas may be recurrent and may result in chronic lymphedema.

Diagnosis is by characteristic appearance; blood culture is done in


toxic-appearing patients. Treatment of choice is penicillin or
dicloxacillin.
A 40 year old woman presents with painful swelling of the joints of her
fingers, excessive fatigue and a malar rash. Which one of the following
diseases is most likely?

a) Psoriasis
b) Pseudogout
c) Systemic lupus erythematosus (SLE)
d) Rheumatoid arthritis
e) Serum sickness
The correct answer is C

Explanation
Systemic lupus erythematosus is a chronic, multisystem, inflammatory
disorder of probable autoimmune etiology, occurring predominantly in
young women. Common manifestations include arthralgias and arthritis;
malar and other skin rashes; pleuritis or pericarditis; renal or CNS
involvement; and hematologic cytopenia.

Diagnosis requires clinical and serologic criteria. Treatment of severe


ongoing active disease requires corticosteroids, often
hydroxychloroquine, and sometimes immunosuppressants.
A 77-year-old white male complains of urinary incontinence of more than
one year‟s duration. The incontinence occurs with sudden urgency. No
association with coughing or positional change has been noted. There is
no history of fever or dysuria. One year ago he underwent transurethral
resection of the prostate (TURP) for benign prostatic hypertrophy and
says his urinary stream has improved. Rectal examination reveals a
smoothly enlarged prostate without nodularity, and normal sphincter
tone. No residual urine is found post-void catherterization.

Which one of the following is the most likely cause of this patient‟s
incontinence?

a) Detrusor instability
b) Urinary tract infection
c) Overflow
d) Fecal impaction
e) Recurrent bladder outlet obstruction
The correct answer is A

*Explanation*:
In elderly patients, detrusor instability is the most common cause of
urinary incontinence in both men and women. Incontinence may actually
become worse after surgical relief of obstructive prostatic hypertrophy.
Infection is unlikely as the cause of persistent incontinence in this
patient in the absence of fever or symptoms of urinary tract infection.
Overflow is unlikely in the absence of residual urine. Impaction is a
relatively rare cause of urinary incontinence, and associated findings
would be present on rectal examination. Normalization of the urinary
stream and the absence of residual urine reduce the likelihood of
recurrent obstruction. The prostate would be expected to remain enlarged
on rectal examination after transurethral resection of the prostate (TURP).
Which of the following radiographic features is most consistent with
osteoarthritis of the knee?

a) Marginal erosions
b) Juxta-articular osteopenia (demineralization)
c) Loss of articular cartilage with narrowing of the radiologic joint
space
d) Osteonecrosis (avascular necrosis) of the medial femoral condyle
e) Syndesmophyte formation
The correct answer is C

*Explanation*:
Osteoarthritis is a chronic arthropathy of an entire joint characterized
by disruption and potential loss of joint cartilage along with other
joint changes, including bone hypertrophy (osteophyte formation).
Symptoms include gradually developing pain aggravated or triggered by
activity, stiffness relieved < 30 min after activity, and occasional
joint swelling. Diagnosis is confirmed by x-rays. X-rays generally
reveal marginal osteophytes, narrowing of the joint space, increased
density of the subchondral bone, subchondral cyst formation, bony
remodeling, and joint effusions.

A 75 year old male complains of pain with defecation, and loose stools.
This problem seems to have developed gradually over the last several
months. His past medical history includes prostate cancer treated with
radiation therapy 5 years ago, hypertension, and osteoarthritis.
Medications include hydrochlorothiazide (HydroDIURIL), a Beta-blocker,
and acetaminophen. On colonoscopy, no polyps or cancer are found, but
the rectal and sigmoid areas show pallor with friability and
telangiectasias. The most likely diagnosis is

a) Familial angiodysplasia
b) Osler-Weber-Rendu syndrome
c) Radiation proctitis
d) Late-onset ulcerative colitis
e) Sensitivity to acetaminophen breakdown products
The correct answer is C

*Explanation*:
Chronic radiation proctitis develops months to years after radiation and
is characterized by pain with defecation, diarrhea, and sometimes rectal
bleeding. On colonoscopy, the mucosa is pale and friable with
telangiectases which are sometimes large, multiple, and serpiginous.
A 56 year old patient presents with polycythemia. He is found to have
low oxygen saturation, increased red blood cells mass, normal plasma
volume and increased erythropoietin levels. Which one of the following
is the most likely diagnosis‟
a) Polycythemia rubra vera
b) Chronic obstructive pulmonary disease (COPD)
c) Stress polycythemia
d) Renal adenocarcinoma
e) Congenital spherocytosis
The correct answer is B

*Explanation*:
Chronic obstructive pulmonary disease (COPD) is persistent obstruction
of the airways occurring with emphysema, chronic bronchitis, or both
disorders.

In the earlier stages of COPD, oxygen levels in the blood may be


decreased, but carbon dioxide levels remain normal. In the later stages,
carbon dioxide levels increase and oxygen levels fall.

The decrease in oxygen levels in the blood stimulates the bone marrow to
send more red blood cells into the bloodstream, a condition known as
secondary polycythemia. The decrease in oxygen levels in the blood also
increases the pressure in the artery through which blood flows from the
heart to the lungs (pulmonary artery).

A 58 year old man presents to your office with a history of having an


episode of sudden visual loss in his right eye. The patient describes
the loss of vision as similar to someone pulling a cover over his right
eye. Vision returned to the right eye after 10 minutes. This visual
field defect is secondary to which one of the following?

a) Scotoma
b) Amaurosis fugax
c) Strabismus
d) Esotropia
e) Night blindness
The correct answer is B

Explanation
Amaurosis Fugax occurs when emboli (blood clots) travel from the carotid
artery (near the heart) to the retinal arteries, thereby blocking the
blood supply to the part of the eye known as the retina, and resulting
in vision loss. In young patients, spasm of the retinal arteries blocks
the blood supply.

A "curtain" appears to pass down over the visual field, causing complete
loss of vision in the affected eye that lasts a few minutes.

Diagnosis is made by examination by an ophthalmologist. Tests such as an


ultrasound of carotid arteries may also be done.

Immediate evaluation by an ophthalmologist is needed. Treatment of


underlying causes which, if due to carotid artery disease, may include
anti-platelet medications (e.g., aspirin, Plavix) or carotid
endarterectomy (surgically removing cholesterol plaque in the carotid
artery).
Which one of the following drugs used to treat rheumatoid arthritis can
delay the progression of the disease?

a) Aspirin
b) Ibuprofen
c) Indomethacin (Indocin)
d) Capsaicin (Zostrix)
e) Hydroxychloroquine (Plaquenil)
The correct answer is E

Explanation
Hydroxychloroquine, originally developed as an antimalarial drug, is a
well-known disease-modifying agent that can slow the progression of
rheumatoid arthritis. Aspirin, indomethacin, and ibuprofen are
anti-inflammatory agents. They relieve pain and improve mobility, but do
not alter the progression of the disease. Capsaicin, a topical
substance-P depleter, can relieve pain symptoms.
Food with a higher glycemic index

a) releases carbohydrates into the bloodstream as glucose more rapidly


b) induces a slower insulin response
c) is less likely to lead to reactive hypoglycemia
d) is less likely to lead to metabolic syndrome
e) leads to lower levels of circulating insulin
The correct answer is A

Explanation
The higher the glycemic index of a food, the more rapidly its
carbohydrates are released into the bloodstream as glucose. Foods with a
high glycemic index induce a more rapid insulin response, are more
likely to lead to reactive hypoglycemia and a metabolic syndrome, and
lead to higher levels of circulating insulin.

A 28 year old female presents with chronic rhinorrhea. On physical exam


you find mucosal nasal atrophy. What is the most likely diagnosis‟

a) Sinusitis
b) Cocaine intake
c) Nasal polyposis
d) Allergic rhinitis
The correct answer is B

Explanation
Although most cocaine is snorted, smoking crack cocaine has become
widely publicized. The converted material is combusted and the resultant
smoke inhaled. Onset of effect is quicker, and intensity of the high is
magnified.

Severe toxic effects occur in the compulsive heavy user. Rarely,


repeated snorting causes nasal septal perforation due to local ischemia.
Repeatedly smoking volatile crack cocaine in high doses can have serious
toxic cardiovascular and behavioral consequences.

A 27 year old man is brought into the ER after a bicycling accident. A


car door suddenly opened in front of him, of which he smashed into and
was thrown 15 feet. On examination, he is drowsy and confused. He opens
his eyes when his name is called. He mumbles words that you understand
but the sentences do not make sense. He moves all four limbs but does
not respond to any commands. He is able to pull both hands away when
pinched and squirms when his sternum is rubbed, making no effort to stop
you. What is his Glasgow COMA Scale score?

a) 11
b) 10
c) 9
d) 8
e) 7
The correct answer is A

Explanation
The Glascow coma scale (GCS) is determined by assigning points to the
following three questions:
Eyes open: Spontaneously (4 points), to speech (3), to pain (2), none (1).
Best verbal response: Orientated (5 points), confused (4), inappropriate
words (3), incomprehensible sounds (2), none (1).
Best motor response: Obeys commands (6 points), localizes pain (5),
withdraws to pain (4), flexion (abnormal) to pain (3), extension to pain
(2), none (1).

This patient has scores of 3, 4 and 4 to the three questions


respectively, making his GCS score 11.

A 71 year old woman with a history of coronary artery disease presents


to her family physician for a routine check-up. The physician notices
that she has lost 10 kg since her last visit 6 months ago. When
questioned, she gives a history of intermittent periumbilical pain that
always begins 30 minutes after eating and lasts about 2 hours. She
claims that the pain is worse after large meals and so she has begun to
eat less out of fear of pain. Which one of the following is the most
likely diagnosis‟

a) Pancreatitis
b) Cholecystitis
c) Small bowel obstruction
d) Intestinal ischemia
e) Peptic ulcer disease
The correct answer is D

Explanation
A diminished blood flow to your small intestine or colon is called
intestinal ischemia.

Chronic intestinal ischemia, in which blood flow to your intestines is


reduced over time, is characterized by: Abdominal cramps or fullness,
beginning within 30 minutes after eating and lasting one to three hours.
Abdominal pain that gets progressively worse over weeks or months. Fear
of eating because of subsequent pain. Unintended weight loss.

An angiography involves guiding a catheter through an artery in your


groin up into the aorta and injecting a dye (radiographic contrast
medium). The dye allows better visualization of your intestinal
arteries. Although invasive, angiography has traditionally been
considered the gold standard in evaluating mesenteric ischemia. Its
advantage is that it allows for simultaneous removal of a blood clot
through the catheter or administration of medications to improve
intestinal blood flow.
Which murmur is best heard by having a patient lean forward in a sitting
position while holding his breath, preferably in expiration?

a) Aortic stenosis
b) Aortic insufficiency
c) Mitral stenosis
d) Mitral insufficiency
e) Tricuspid insufficiency
The correct answer is B

Explanation
To determine that a patient does not have a murmur of aortic
insufficiency, the physician must auscultate the heart with the patient
leaning forward in a sitting position while holding his breath,
preferably in expiration. In fact, auscultation should be performed in
various positions with all patients, depending on the type of
examination and the diagnostic findings anticipated.
A 62 year old man who is a cigar and pipe smoker develops a circular
lesion on the lower lip. See picture:

What is the most likely diagnosis‟

a) Basal cell carcinoma


b) Squamous cell carcinoma
c) Actinic keratoses
d) Malignant melanoma
The correct answer is B

Explanation
Squamous cell carcinoma of the oral cavity occurs mostly in smokers.
About 38% of all oral squamous cell carcinomas occur on the lower lip.
Fortunately for lower lip lesions, 5 year survival is 90%, and
metastases are rare. Surgery and radiation therapy are the treatments of
choice.

Basal cell carcinoma most often appears on sun-exposed areas such as the
face, scalp, ears, chest, back, and legs. The most common appearance of
basal cell cancer is that of a small dome-shaped bump that has a pearly
white color. Basal cell cancer can also appear as a pimple-like growth
that heals, only to come back again.
Three members of the same family present with a high fever and cough
that began abruptly yesterday. All three report having fevers over 40°C
(104°F), painful coughs, moderate sore throats, and prostration. They
have loss of appetite, but no vomiting or diarrhea. Two other family
members have similar symptoms. On examination the patients appear ill
and flushed. There is no cervical adenopathy, no visible pharyngeal
inflammation, and no significant findings on examination of the chest.

Which one of the following is the most likely diagnosis‟

a) Mycoplasma pneumonia
b) Influenza-like illness
c) Bacterial bronchitis
d) Upper respiratory infection
e) Legionnaires‟ disease
The correct answer is B

Explanation
Influenza has a very abrupt onset, and a fever with a nonproductive
cough is almost always present. Unconfirmed cases are referred to as
“influenza-like illness‟ (ILI) or „suspected influenza.” Patients with
confirmed cases tend to say they have never been so ill. Mycoplasma
pneumonia can spread among family members, but it is milder and has a
more indolent onset and a longer incubation period. Bacterial bronchitis
is an overdiagnosed, supposed complication of upper respiratory
infections, and is not contagious. While the phrase “cold and flu” is
often used, upper respiratory infections are not so febrile or
prostrating, and coryza is the dominant syndrome sooner or later.
Legionella can have point-source epidemics, but the incubation period is
longer, symptoms vary from mild illness to life-threatening pneumonia,
and diarrhea is prominent in many cases.
A 65 year old obese male complains of repeated attacks of strong and
sudden mid abdominal pain radiating to his left flank with pallor and
diaphoresis with asymptomatic periods in between. What is the most
likely diagnosis‟

a) Acute pancreatitis
b) Cholecystitis
c) Ureteral stone
d) Mesenteric Ischemia
e) Rupture of AAA
The correct answer is C

Explanation
A ureteral stone can cause severe abdominal pain, which will radiate to
the corresponding flank region. Diagnosis is made by imaging with a CT
or plain X-ray. Small stones may pass on their own with aggressive
hydration. Larger stones may need to be pulverized with lithotripsy.
Acute pancreatitis would presents as epigastric pain, radiating to the
back associated with nausea and vomiting. Cholecystitis would present as
right upper quadrant pain. Mesenteric ischemia is usually due to a clot
in one of the arteries that supply the intestines. Pain is out of
proportion to physical findings. Rupture of an abdominal aortic aorta
would involve radiation to the back.
Which one of the following medications is most effective for restless
legs syndrome?

a) Calcium carbonate
b) Phenytoin
c) Hydrochlorothiazide
d) Ibuprofen
e) Carbidopa/levodopa (Sinemet)
The correct answer is E

Explanation
Dopaminergic drugs are the first-line drugs for most cases of restless
legs syndrome (RLS). When dopaminergic agents are prescribed, patients
should be informed that although these medications are normally used to
treat Parkinson‟s disease, they also help relieve RLS symptoms.
A 50 year old man comes to the emergency room with a history of vomiting
for three days' duration. His past history reveals that for
approximately 20 years, he has been getting epigastric pain, lasting for
two to three weeks. He remembers getting relief from pain by taking milk
and antacids. Physical examination showed a fullness in the epigastric
area with visible peristalsis, absence of tenderness and normal active
bowel sounds. Which one of the following is the most likely diagnosis‟

a) Gastric outlet obstruction


b) Small bowel obstruction
c) Volvulus of the colon
d) Incarcerated umbilical hernia
e) Cholecystitis
The correct answer is A

Explanation
Intrinsic or extrinsic obstruction of the pyloric channel or duodenum is
the usual pathophysiology of gastric outlet obstruction (GOO). Nausea
and vomiting are the cardinal symptoms of GOO. Vomiting usually is
described as nonbilious, and it characteristically contains undigested
food particles. In the early stages of obstruction, vomiting may be
intermittent and usually occurs within 1 hour of a meal.

Early satiety and epigastric fullness are common. Weight loss is


frequent when the condition approaches chronicity and is most
significant in patients with malignant disease. Abdominal pain is not
frequent. Physical examination often demonstrates the presence of
chronic dehydration and malnutrition. A dilated stomach may be
appreciated as a tympanitic mass in the epigastric area and/or left
upper quadrant.

Plain abdominal radiographs, contrast upper GI studies (Gastrografin or


barium), and CT scans with oral contrast are helpful. Plain radiographs,
including the obstruction series (ie, supine abdomen, upright abdomen,
chest posteroanterior), can demonstrate the presence of gastric
dilatation and may be helpful in distinguishing the differential diagnosis.

Upper endoscopy can help visualize the gastric outlet and may provide a
tissue diagnosis when the obstruction is intraluminal. Treatment is
surgical.
A 73 year old white female complains of difficulty reading for the past
several months. Examination reveals 20/100 near vision bilaterally, and
normal visual fields by confrontation. An ophthalmoscopic examination
reveals only small yellow lesions cluttered in the retina. The most
likely diagnosis is

a) Macular degeneration
b) Glaucoma
c) Cataract
d) Central retinal vein occlusion
e) Retinal detachment
The correct answer is A

Explanation
Nonexudative age-related macular degeneration is characterised by
variable degrees of atrophy and degeneration of the outer retina,
retinal pigmentation epithelium, Bruch‟s membrane, and
Choriocapillaries. Of the ophthalmoscopically visible changes in the
retinal pigment epithelium and Bruch‟s membrane, drusen are the most
typical. Drusen are discrete, round, yellow-white deposits of variable
size beneath the pigment epithelium and are scattered throughout the
macula and posterior pole. Age-related macular degeneration is the
leading cause of permanent blindness in the elderly.
A 36-year-old female presents with the sudden onset of severe headache,
nausea, and photophobia. Her level of consciousness is progressively
diminishing. Which one of the following would be the most appropriate
next step?
a) Head CT without contrast
b) Head CT with contrast
c) Head MRI
d) Lumbar puncture
e) CT angiography
The correct answer is A

Explanation
The first study ordered in any patient with suspected subarachnoid
hemorrhage should be a head CT without contrast. It will reveal
subarachnoid hemorrhage in 100% of cases within 12 hours of the bleed,
and it is useful for identifying other sources for the headache, for
predicting the site of the aneurysm, and for predicting cerebral
vasospasm and poor outcome. As blood is cleared from the affected area,
CT sensitivity drops to 93% within 24 hours, and to 50% at 7 days.
Patients with a positive CT result for subarachnoid hemorrhage should
proceed directly to angiography and treatment. Patients with a suspected
subarachnoid hemorrhage who have negative or equivocal results on head
CT should have a lumbar puncture. MRI and CT with contrast are not used
for the diagnosis of acute subarachnoid hemorrhage.
For adults with obstructive sleep apnea, which one of the following is
the most effective treatment?

a) Modafinil (Provigil)
b) Weight reduction to achieve a body mass index (BMI) <30 kg/m^2
c) Mandible-positioning oral appliances
d) Continuous positive airway pressure (CPAP)
e) Uvulopalatal surgery
The correct answer is D

Explanation
Continuous positive airway pressure (CPAP) is the most effective
treatment for obstructive sleep apnea in adults. Weight reduction is
certainly beneficial in obese patients, but may not significantly
decrease apneic episodes. Results with devices that move the tongue or
mandible forward are variable and inconsistent. While modafinil has an
indication for treating sleep apnea, its efficacy is inconsistent.
Uvulopalatal surgery often reduces snoring but may not reduce the
frequency of apneic episodes during sleep.
A 65 year old man presents with a history of paroxysms of sweating,
palpitations, headaches and anxiety. This happens off and on. His blood
pressure on exam is 156/95. On further history he tells you that one of
his relatives had thyroid cancer. At this time what tests should be done
first?

a) 24 hour urinary catacholamines and metanephrines


b) Serum VMA
c) CT of adrenal
d) CT of head
e) CMP
The correct answer is A

Explanation
This patient needs to be screened for pheocromocytoma. Pheochromocytoma
is a tumor of the adrenal gland which causes very high levels of the
catecholamines (epinephrine and norepinephrine) to be secreted into the
bloodstream. This can lead to many sympathetic nervous system symptoms
like elevated blood pressure, palpitations, anxiety, diaphoresis,
headaches, weight loss. These symptoms happen is spurts or paroxysms.

Diagnoses is made by measuring the level of the catecholamines and their


breakdown products or metabolites which are called metanephrines in a 24
hour urine collection. Treatment involves medicines to control the blood
pressure and surgery to remove the tumor. (Metanephrine levels are
considered the most sensitive and specific test for a pheochromocytoma,
while vanillylmandelic acid is the least specific test and has a
false-positive rate greater than 15%.)

Recall the MEN syndromes. MEN (Multiple endocrine neoplasia) has three
types:

MEN I (Wermer syndrome) - tumors of the pancreas, pituitary and parathyroid.


MEN IIa (Sipple syndrome) - medullary thyroid carcinoma, pheochromcytoma
and tumor of the parathyroid.
MEN IIb - Medullary thyroid carcinoma, pheochromocytoma and neuromas.

This patient could have MEN II if you consider his family history.
A 25-year-old elementary school teacher presents with target or
iris-like lesions on her palms and soles. You should suspect

a) ß-streptococcal infection
b) erythema multiforme
c) urticaria
d) Lyme disease
e) discoid lupus erythematosus
The correct answer is B

Explanation
“Target” or “iris-like” lesions are found only in erythema multiforme.
Although these lesions are unique, they signify many and varied
etiologies that often require intervention. The other options listed
exhibit skin manifestations with much different appearances.

A 76 year old white male with a history of type 2 diabetes mellitus


presents with a well-demarcated ulcer on the lateral aspect of his right
ankle. Because of associated edema, you are unable to palpate pulses in
his feet.

The most appropriate next step in the care of this patient would be to

a) Begin compression therapy with elastic bandages


b) Prescribe aspirin, 300 mg daily
c) Prescribe pentoxifylline (Trental), 800 mg daily
d) Perform venography
e) Perform a noninvasive arterial evaluation
The correct answer is E

Explanation
Since palpation of the dorsalis pedis pulse on the affected limb is
difficult in this patient due to edema, further diagnostic evaluation to
rule out associated arterial insufficiency is warranted. Compression
therapy in patients with underlying arterial disease may cause limb
ischemia. Noninvasive diagnostic procedures, such as determining the
ankle-brachial index or performing transcutaneous oxygen measurement,
are simple methods for detecting arterial disease. Venography is usually
reserved for investigation before surgery, if indicated. Aspirin and
pentoxifylline may improve healing but would not be started until a
diagnosis has been established.
A patient presents with right ptosis and visual loss a few hours ago,
and later develops weakness of his left arm. What is the most
appropriate investigation?

a) CT of head
b) EEG
c) Lumbar spine MRI
d) Spinal tap with CSF analysis
The correct answer is A

Explanation
This is a case of Weber syndrome. This syndrome is due to damage to the
structures at the base of the midbrain including the third cranial nerve
and the corticospinal and corticobulbar fibers in the cerebral peduncle.
It is characterized by complete 3rd nerve palsy and associated with
contralateral hemiplegia.

Common causes of this syndrome are aneurysm in the posterior parts of


the Circle of Willis, tumor, and occlusion in vessels supplying the
anterior midbrain.

A CT of the head would be an appropriate first step in the work up of


this disease.
In a patient with Rocky Mountain spotted fever, which one of the
following laboratory abnormalities would indicate an increased risk of
death?

a) Marked leukocytosis
b) Thrombocytopenia
c) Elevated serum creatinine
d) Lymphocytic cerebrospinal fluid pleocytosis
e) Nonspecific ST-wave changes on an EKG
The correct answer is C

Explanation
A myriad of laboratory abnormalities may occur in Rocky Mountain spotted
fever (RMSF). These patients may have low, normal, or elevated WBC
counts, normal to low platelet counts, lymphocytic cerebrospinal fluid
pleocytosis, and nonspecific electrocardiographic findings. These
usually have little diagnostic value and often result in diagnostic
confusion for physicians caring for RMSF patients. Elevated serum
creatinine, however, has been shown to be associated with an increased
risk for a fatal outcome. Renal insufficiency of azotemia manifested by
this study is likely due to glomerular and tubular damage secondary to
widespread rickettsial vasculitis in the microcirculation of the kidney.

The metabolic syndrome responds best to


a) Thyroid hormone therapy
b) Antioxidants
c) Exercise and weight loss
d) Rest and relaxation techniques
e) An intensive insulin regimen
The correct answer is C

Explanation
The metabolic syndrome is a combination of obesity, lipid abnormalities,
hypertension, and hyperglycemia. It confers a high risk for
cardiovascular disease. Primary therapy includes exercise, weight loss,
and dietary changes. This improves all aspects of the metabolic
syndrome, mainly as a result of decreased insulin resistance.
Antioxidants have no proven benefit. Thyroid hormone has no effect on
metabolic syndrome unless a person is hypothyroid. Intensive insulin
regimens improve glycemic control, and have no effect on other aspects
of the syndrome, and may contribute to weight gain, which adversely
affects the syndrome. Rest and relaxation may make one feel better
psychologically, but are not a mainstay of treatment.

A 70-year-old female with type 2 diabetes mellitus is admitted to the


hospital with a 4-week history of fever, anorexia, and weight loss. Two
blood cultures are positive for Streptococcus bovis. In addition to
being treated for the infection, she should be evaluated for which one
of the following?

a) B-cell lymphoma
b) T-cell lymphoma
c) Multiple myeloma
d) Lung cancer
e) Colorectal cancer
The correct answer is E

Explanation
For unknown reasons, Streptococcus bovis bacteremia or endocarditis is
associated with a high incidence of occult colorectal malignancies. It
may also occur with upper gastrointestinal cancers. Radiography or
endoscopy is indicated.
The specificity of a screening test is best described as the proportion
of persons

a) with the condition who test positive


b) with the condition who test negative
c) with the condition who test positive, compared to the total number
screened
d) without the condition who test positive
e) without the condition who test negative
The correct answer is E

Explanation
A screening test‟s specificity is the proportion of persons without the
condition who test negative for that condition. In other words, it is a
measure of the test‟s ability to properly identify those who do not have
the disease. Conversely, the sensitivity of a screening test is the
proportion of those with the condition who test positive. The other
options listed describe false-negatives, false-positives, and prevalence.
A 52 year old male with a history of psychosis presents with muscle
stiffness and resting tremors, associated with difficulty in balance and
initiating movements.

Which of the following in considered a good choice as initial


treatment of early stages of this condition?

a) Levodopa
b) Bromocriptine
c) Chlorpromazine
d) Co-enzyme Q10
e) Selegiline
The correct answer is E

Explanation
Treatment of early Parkinson's disease include: DA agonists, amantadine,
MAOIs.

Dopamine agonists provide moderate symptomatic benefit and rarely cause


fluctuations and dyskinesias by themselves, but they have more side
effects than levodopa, including sleepiness and impulse control
disorders. However, these side effects resolve upon lowering the dose or
discontinuing the medication.

Amantadine inhibits the N -methyl-D-aspartic acid (NMDA)


receptor-mediated stimulation of acetylcholine release. It may also
enhance dopamine release, inhibit dopamine reuptake, stimulate
postsynaptic dopamine receptors, or enhance dopamine receptor
sensitivity. It provides some benefit for tremor, rigidity, and
bradykinesia.

MAO-B inhibitors provide mild symptomatic benefit, have excellent side


effect profiles, and may improve long-term outcomes. These
characteristics make MAO-B inhibitors a good choice as initial treatment
for many patients. When the MAO-B inhibitor alone is not sufficient to
provide good control of motor symptoms, another medication (eg, dopamine
agonist or levodopa) is added.

A patient who ate BBQ developed severe diarrhea. At the hospital his
blood count shows eosinophilia. The initial investigation to be done to
reach the diagnosis is

a) Muscle biopsy
b) Stool bacterial culture
c) Stool for ova and parasites
d) Stool WBC
e) Stool gram stain
The correct answer is C

Explanation
Eosinophilia is defined as a peripheral blood eosinophil count > 450/?L.
Causes are plenty but often represent an allergic reaction or parasitic
infection. Diagnosis involves selective testing directed at clinically
suspected underlying causes. A stool study for ova and parasites is most
appropriate at this time. Treatment is directed at the underlying cause.
A 35 year old man with a recent history of severe multiple trauma is
noted to have dyspnea and is diagnosed as having adult respiratory
distress syndrome (ARDS). All of the following statements about the
management of ARDS are correct, except
a) Management may include artificial ventilation with endotracheal
intubation and oxygen therapy
b) A sudden deterioration in a ventilated patient should raise the
suspicion of pneumothorax
c) Early management may include oxygen by mask and careful monitoring
of respiratory rate and arterial blood gases
d) Furosemide is routinely used to keep the lungs as dry as possible
e) Frequent arterial blood gas determinations will be required to
assess the adequacy of therapy in any ventilated patient
The correct answer is D

Explanation
People with ARDS are treated in an intensive care unit. Successful
treatment depends on correcting the underlying cause; oxygen therapy,
which is vital to correcting low oxygen levels, is combined with
treatment of the underlying cause.

If oxygen delivered by a face mask or nasal prongs does not correct the
low blood oxygen levels, or if excessively high doses of inhaled oxygen
are required, a ventilator must be used; this treatment is called
mechanical ventilation. A ventilator delivers oxygen-rich air under
pressure using a tube inserted through the mouth into the trachea. The
pressure supplied by the ventilator during and after a breath opens
collapsed (atelectatic) regions of the lung and allows oxygen to move
through the walls of the injured lungs into the blood.

The pressure and volume of air that the ventilator delivers to the lungs
with each breath must be adjusted to help keep the small airways and
alveoli open while avoiding rupturing the fragile air sacs, which can
lead to air accumulating around the lung and collapsing it (called
pneumothorax). Monitoring and adjusting the pressure also ensures that
the lungs do not receive an excessive concentration of oxygen, which can
damage the lungs and worsen ARDS.

In some cases, diuretic drugs may be needed to help remove fluid from
the lungs. Antibiotics are usually needed for people who develop
bacterial pneumonia. Some people may benefit from the use of intravenous
corticosteroids in the later stages of ARDS.
A 45 year old woman is brought to the health center by her husband
because of nausea, confusion, chills, fever, flank pain and cloudy
urine. She has a history of insulin-dependent diabetes mellitus, poorly
controlled hypertension and recurrent urinary tract infections. Her
vital signs are: temperature 40.0°C (104.0°F), pulse 120/min,
respirations 24/min, blood pressure 110/70 mm Hg. Funduscopic
examination shows diabetic retinopathy, which is unchanged from the
previous examination. Marked pain is present at the right costovertebral
angle. Several hemorrhagic bullous lesions are noted on the extremities.

Without prompt and aggressive treatment, this patient is most likely to


develop which of the following?

a) Diabetic ketoacidosis
b) Hyperosmolar coma
c) Meningitis
d) Pneumonia
e) Septic shock
The correct answer is E

Explanation
The patient is presenting with signs of septic shock from
pyelonephritis, including nausea, confusion, chills, fever, flank pain,
and cloudy urine. The hemorrhagic extremity lesions are most likely a
sign of septicemia and toxemia. Immediate treatment with intravenous
antibiotics is necessary to prevent cardiopulmonary collapse and death.

An asymptomatic 40-year-old male presents for a routine examination and


is found to have a total bilirubin level of 1.8 mg/dL (N < 1.0) and an
indirect bilirubin level of 1.3 mg/dL. He drinks 3-6 beers/week. An
examination and laboratory tests, including a CBC and serum liver
enzymes, are within normal limits. Which one of the following is true
regarding the diagnosis‟

a) The most likely diagnosis is alcoholic liver disease


b) The most likely diagnosis is Dubin-Johnson syndrome
c) The most likely diagnosis is Gilbert syndrome
d) Ultrasonography of the liver and gallbladder are necessary to make a
diagnosis
The correct answer is C

Explanation
Gilbert syndrome is a hereditary disease characterized by indirect
hyperbilirubinemia caused by impaired glucuronyl transferase activity.
The workup includes studies to exclude hemolysis (CBC, reticulocyte
count, and haptoglobin) and liver disease (AST, ALT, alkaline
phosphatase, and prothrombin time). Alcoholic liver disease is
associated with a greater elevation of AST than of ALT. Dubin-Johhnson
syndrome is a benign liver disease distinguished by direct or conjugated
hyperbilirubinemia. Imaging studies are not required to confirm Gilbert
syndrome; such studies are more useful for conditions involving
conjugated hyperbilirubinemia. Other causes of indirect
hyperbilirubinemia include hematoma, infection, cardiac disease,
rhabdomyolysis, living at high altitude, thyrotoxicosis, and some
medications.
When given during acute herpes zoster (varicella) infection, which one
of the following drugs has been shown to reduce the incidence of
postherpetic neuralgia?

a) Imipramine (Tofranil)
b) Capsaicin (Zostrix)
c) Amitriptyline
d) Acyclovir (Zovirax)
e) Oral prednisone
The correct answer is D

Explanation
A systematic review of 42 randomized, controlled trials showed that
acyclovir reduced the incidence of postherpetic neuralgia at 3 months
(number needed to treat = 3.2?8.0). Famciclovir was shown to reduce the
duration of postherpetic pain.

No conclusions could be drawn with regard to amitriptyline, narcotics,


capsaicin, anticonvulsants, or percutaneous nerve stimulation. Studies
examining these treatments were of fair to poor quality. Systemic
corticosteroids showed no additional benefit.
The lesions shown in here are typical of

a) Herpes zoster
b) Chickenpox
c) Herpes simplex
d) Molluscum contagiosum
e) Pyogenic granuloma
The correct answer is D

Explanation
Molluscum contagiosum is a viral infection of the skin characterized by
discrete, 2- to 5-mm, slightly imbilicated, flesh-colored, dome-shaped
papules. It spreads by autoinoculation, by scratching, or by touching a
lesion. Most lesions are self-limited and clear spontaneously in 6-9
months.
A 70 year old male with typical acute myocardial infarction symptoms
that have started about 2h ago. EKG shows ST segment elevation.
Performed coronary angiography revealed right coronary artery occlusion
by thrombi. Which of the following antithrombotic agents actively lyses
preformed clot?

a) Clopidogrel
b) Heparin
c) Aspirin
d) Nitroglycerin
e) Streptokinase
The correct answer is E

Explanation
Answer: E
Thrombolysis is generally preferred to PCI in cases where the time from
symptom onset is less than 3 hours
Streptokinase - Nonfibrin specific agent with a half-life of 23 min that
acts with plasminogen to convert plasminogen to plasmin. Plasmin
degrades fibrin clots as well as fibrinogen and other plasma proteins.
An increase in fibrinolytic activity that degrades fibrinogen levels for
24-36 h takes place with intravenous infusion of streptokinase.
A. Clopidogrel - Selectively inhibits adenosine diphosphate (ADP)
binding to platelet receptor and subsequent ADP-mediated activation of
glycoprotein GPIIb/IIIa complex, thereby inhibiting platelet aggregation.
B. Heparin - Augments activity of antithrombin III and prevents
conversion of fibrinogen to fibrin. Does not actively lyse preformed
clot, but it is able to inhibit further thrombogenesis after thrombolysis.
C. Aspirin - Inhibits cyclooxygenase, which produces thromboxane A2, a
potent platelet activator. Early administration has been shown to reduce
35-d mortality rate by 23%. An added mortality benefit exists when used
in combination with thrombolytics.
D. Nitroglycerin - Causes relaxation of vascular smooth muscle by
stimulating intracellular cyclic guanosine monophosphate production.
Result is decrease in blood pressure.

Compared to patients with permanent atrial fibrillation, patients with


paroxysmal atrial fibrillation have a risk of stroke that is

a) 50% less
b) 25% less
c) 25% greater
d) 50% greater
e) Similar
The correct answer is E

Explanation
Patients with paroxysmal atrial fibrillation (i.e., self-terminating)
and persisting atrial fibrillation (i.e., that lasts more than 7 days or
requires cardioversion) appear to have a risk of stroke that is similar
to that of patients with permanent atrial fibrillation.
A 20 year old college student presents to you with a history of
intermittent chest pain for about 6 months. He states that the pain can
occur at any time, but often occurs late in the day. The pain can be
dull, or aching and may last for hours. He is physically active and is
carrying a full academic load. The pain is unrelated to exercise and
does not restrict his activities. His father and paternal uncle have
both had myocardial infarctions within the past year. Physical
examination is normal, although the patient appears restless and has a
heart rate of 100/min. Which one of the following is the most likely
diagnosis‟

a) Reflux esophagitis
b) Generalized anxiety disorder
c) Transient myocardial ischemia
d) Myocardial infarction
e) Costochondritis
The correct answer is A

Explanation
Incompetence of the lower esophageal sphincter allows reflux of gastric
contents into the esophagus, causing burning pain. Prolonged reflux may
lead to esophagitis, stricture, and rarely metaplasia or malignancy.

The term reflux refers to the regurgitation of gastrointestinal contents


into the esophagus without associated retching and vomiting. The
contents may be gas, food, or gastrointestinal secretions. Abnormal
reflux may manifest itself by ease of provocation (e.g., the
regurgitation of gastric contents on stooping or lying down) or by
frequency (e.g., the occurrence of heartburn after each meal).
Gastroesophageal reflux disease refers to manifestations of esophageal,
laryngeal, or pulmonary injury related to the reflux of gastrointestinal
contents. The most common manifestation is reflux esophagitis.

The typical manifestations are heartburn, regurgitation, and dysphagia.


Heartburn (pyrosis) is defined as a retrosternal burning discomfort
located in the epigastric area that may radiate up toward the neck and
typically occurs in the post-prandial period, especially after a
high-fat or a large-volume meal.

Diagnosis is clinical, sometimes with endoscopy, with or without acid


testing. Treatment involves lifestyle modification, acid suppression
using proton pump inhibitors, and sometimes surgical repair.
A 25-year-old graduate student is visiting his parents during fall break
when he develops an acute headache, fever, and rash. When you see him in
your office, he has a widespread petechial rash and a stiff neck, and
his blood pressure is 78/40 mm Hg. You start intravenous fluids and
ceftriaxone (rocephin) and call for emergency transport. The patient
dies shortly after arrival at the hospital‟s emergency department. The
spinal fluid from a tap done just before his death reveals a large
number of polynuclear leukocytes and gram-negative diplococci.

Which one of the following is true regarding control measures for this
disease?

a) Throat and nasopharyngeal cultures of contracts should be performed,


and persons with positive cultures treated
b) Since this case likely represents a sporadic one, no contacts in
your community should receive prophylaxis
c) Since the attack rate of this disease in household contacts is only
slightly higher than that of the general population, these contacts
should not receive prophylaxis
d) Immunoprophylaxis with a seragroup-specific, multivalent vaccine is
the management of choice for those who need treatment
e) When chemoprophylaxis is indicated, ciprofloxacin (Cipro) is an
acceptable agent
The correct answer is E

Explanation
The patient died of presumed Neisseria meningitidis septicemia and
meningitis. Only high-risk contacts should receive prophylaxis. Throat
and nasopharyngeal cultures are of no value for deciding who should
receive prophylaxis. Close contacts of patients with invasive disease,
whether it is a sporadic case or part of an outbreak, are at high risk
and should receive prophylaxis within 24 hours of the diagnosis of the
primary case. The attack rate for household contacts is >300 times
higher than the rate in the general population. If the serogroup
identified is contained in the vaccine (A, C, Y, or W-135),
immunoprophylaxis may be useful as a secondary control measure. However,
chemoprophylaxis is considered the primary treatment option.
Ciprofloxacin is an acceptable form of chemoprophylaxis for N.
meningitidis in adults. It is not recommended for persons <18 years of
age or pregnant women.
You have just diagnosed pulmonary sarcoidosis in a 35 year old black
female. Which one of the following should be performed on the patient?

a) Slit lamp examination of the eyes


b) HLA serotype testing
c) Helper-to-suppressor T-cell ratio
d) Kveim skin test

The correct answer is A

Explanation
Patients with sarcoidosis should have a slit lamp examination to rule
out uveitis, pulmonary function testing, a serum calcium level, and
electrocardiography. In addition, serum angiotensin converting enzyme
and anergy skin testing might be considered. Certain HLA types have been
associated with sarcoidosis in Japan, but testing is not specific.
T-cell ratios may explain the reason for anergy, but are not specific or
recommended. The Kveim skin test is not used clinically now because of
the danger of injecting sarcoid antigen into another patient.
Features characteristic of acute tonsillitis include all of the
following, except

a) Odynophagia
b) Cough
c) Referred otalgia
d) Cervical lymphadenopathy
e) Dysphagia
The correct answer is B

Explanation
Tonsillitis is an inflammation of the tonsils and will often, but not
necessarily, cause a sore throat and fever. Tonsillitis is characterized
by red, swollen tonsils. White patches may or may not appear on the
tonsils. Other signs and symptoms of tonsillitis include a severe sore
throat, painful/difficult swallowing, headache, fever and chills,
enlarged and tender lymph nodes, pain in the tonsil area, ear pain, and
loss of voice.
In an HIV-positive patient, which one of the following would be the best
choice for treating seborrheic dermatitis‟

a) Erythromycin
b) Ketoconazole (Nizoral)
c) Trimethoprim/sulfamethoxazole (Bactrim, Septra)
d) Clindamycin (Celocin)
e) Dapsone
The correct answer is B

Explanation
Most patients with seborrheic dermatitis will respond to the usual
treatment of hygiene, shampoos, and mild topical corticosteroids.
However, experience with patients who are more difficult to treat,
especially those with HIV infection whose dermatitis is refractory to
standard topical treatment, has shown that ketoconazole in cream or
shampoo form is effective, along with antifungals. Seborrheic dermatitis
occurs in up to 50% of patients with HIV, and in some cases may be
complicated by concomitant infection with Pityrosporum, a yeast-like
fungus. The other antibiotics listed are of little value in treating
this type of dermatitis.
A 23 year old known drug addict is being treated with cloxacillin for
confirmed acute staphylococcal endocarditis involving the aortic valve.
His fever has persisted. The nursing staff calls you because he has
become quite distressed and is complaining of shortness of breath. On
examination, the first heart sound is barely audible and the previous
full-length diastolic murmur is now of very short duration, less than
1/2 diastole. There are bibasilar rales. Which one of the following
would be the most appropriate management?

a) Emergency aortic valve replacement


b) Continued observation with present therapy
c) Introduction of diuretic and afterload-reduction therapy
d) Antibiotic change to vancomycin
e) Repeat echocardiogram followed by surgery
The correct answer is E

Explanation
Infective endocarditis is infection of the endocardium, usually with
bacteria (commonly, streptococci and staphylococci) or fungi. It
produces fever, heart murmurs, petechiae, anemia, embolic phenomena, and
endocardial vegetations. Vegetations may result in valvular incompetence
or obstruction, myocardial abscess, or mycotic aneurysm. Diagnosis
requires demonstration of microorganisms in blood and usually
echocardiography. Treatment consists of prolonged antimicrobial
treatment and sometimes surgery.

Surgery (debridement, valve repair or replacement) is frequently


required for abscess, persistent infection despite antimicrobial therapy
(ie, persistent positive blood cultures or recurrent emboli), or severe
valvular regurgitation.

Timing of surgery requires experienced clinical judgment. If heart


failure caused by a correctable lesion is worsening (particularly when
the organism is S. aureus, a gram-negative bacillus, or a fungus),
surgery may be required after only 24 to 72 h of antimicrobial therapy.
In patients with prosthetic valves, surgery may be required when TEE
shows valve dehiscence on a paravalvular abscess, when valve dysfunction
precipitates heart failure, when recurrent emboli are detected, or when
the infection is caused by an antimicrobial-resistant organism.
A 58-year-old male who works with heavy machinery at a local factory
presents to your office for evaluation of hearing loss of several years‟
progression. He notes that the loss is mainly in the left ear and he
also has mild tinnitus. He has had no trauma to his head, and he has no
history of ear infections. Examination of the ears reveals normal
tympanic membranes and a neurologic examination is negative. When a
tuning fork is placed in the center of his forehead, he says the sound
is much louder on the right side (Weber test). Comparing sound in front
of the ear to the sound when the tuning fork is placed on the mastoid
(the Rinne test) reveals that air conduction is better than bone
conduction in the left ear.

Which one of the following is true regarding further evaluation and


management?

a) No treatment or further diagnostic studies are indicated


b) A hearing aid plus better hearing protection is all that is needed
c) Carotid ultrasonography should be ordered
d) A tympanogram is indicated
e) Audiometry is the best initial screening test
The correct answer is E

Explanation
Acoustic neuroma symptoms are due to cranial nerve involvement and
progression of tumor size. Hearing loss is present 95% of the time and
tinnitus is very common. The loss is usually chronic (over 3 years), and
as many as one-third of patients are unaware it has occurred. Vestibular
nerve involvement most often causes mild unsteadiness and rarely has
accompanying true vertigo. Trigeminal involvement can cause pain,
paresthesias, or numbness of the face. Facial paralysis occurs 6% of the
time.

The diagnosis of acoustic neuroma is based on asymmetric sensorineural


hearing loss or another cranial nerve deficit, with confirmation based
on MRI with gadolinium contrast or a CT scan. The best initial screening
laboratory test is audiometry, as only 5% of patients with acoustic
neuroma will have a normal test. Sensorineural loss is usually in the
higher frequencies. Brainstem-evoked response audiometry may be used as
a further screening measure when there are unexplained symmetrics and
standard audiometric testing.
Within minutes of eating some peanuts, a 23 year old woman develops
generalized pruritus and hyperemia of the skin, followed shortly by
swelling of the face and eyelids, dyspnea, and laryngeal edema. This
reaction is mediated by which one of the following?

a) Antigen-antibody complexes
b) Cytotoxic T cells
c) IgA antibodies
d) IgE antibodies
e) IgG antibodies
The correct answer is D

Explanation
An allergic reaction is when the immune system mistakes a harmless
substance for dangerous one and produces an antibody called
"immunoglobulin E", more commonly known as "IgE".

When an allergic person is exposed to an allergen, IgE is produced to


fight off the specific allergen. For example, if someone is allergic to
peanuts and eats some, then an "anti-peanut" IgE antibody is produced.

Each time IgE is produced, the IgE molecules attach themselves to mast
cells that are found in large numbers in the eyes, nose, lungs,
intestines, and immediately beneath the skin. These mast cells contain
many chemicals, including a substance called histamine which, when
released into the body, can cause runny nose, sneezing, watery eyes,
itching, hives, and wheezing. These effects are recognized as allergy
symptoms.
In some cases, reactions can occur in several places throughout the
body. Welts or hives may appear, spasm in the lungs may cause coughing
or wheezing, the throat and tongue may swell; if anaphylaxis (severe
allergic reaction) occurs, it may be fatal.
At what CD4 count are HIV patients at increased risk of developing PCP?

a) CD4 count >500x10^6


b) CD4 count 200-499x10^6
c) CD4 count <200x10^6
d) CD4 count < 500x10^6
The correct answer is C

Explanation
In HIV, if the CD4+ count falls below about 200 cells per microliter of
blood, the immune system becomes less able to fight certain infections
(for example, the fungal infection that causes Pneumocystis carinii
pneumonia [PCP]). These infections do not usually appear in people with
a healthy immune system and are called opportunistic infections.
A 67-year-old male presents with symptoms of decreased libido and
erectile dysfunction. An evaluation reveals that he has primary
testosterone deficiency, and you consider testosterone replacement therapy.

Which one of the following is true regarding this therapy?

a) Liver toxicity is common


b) It decreases the rate of bone fractures
c) The risk of prostate cancer is increased
d) HDL cholesterol levels typically increase
e) Erythrocytosis is a potential side effect
The correct answer is E

Explanation
When testosterone is taken at usual physiologic doses, liver toxicity is
unusual. Testosterone replacement therapy may stop bone loss and
increase bone density, but many studies have been equivocal, and none
have reported a decreased rate of fractures. Although prostate cancer is
thought to be stimulated by testosterone, no studies have shown
convincing evidence of an increased risk of prostate cancer associated
with testosterone therapy. Studies indicate that testosterone therapy
either has no effect on HDL cholesterol or reduces HDL cholesterol
levels. Testosterone stimulates erythropoiesis in 3%-18% of men
receiving testosterone therapy. Since polycythemia is a potential issue,
hemoglobin levels should be monitored.
Diagnosis of acute symptomatic pulmonary embolism can be excluded when
which of the following is normal?

a) Chest x-ray
b) Ventilation-perfusion lung scan
c) Bilateral leg venograms
d) PaO2 and A-a O2 gradient
The correct answer is B

Explanation
Pulmonary embolism is the occlusion of one or more pulmonary arteries by
thrombi that originate elsewhere, typically in the large veins of the
lower extremities or pelvis. Risk factors are conditions that impair
venous return, conditions that cause endothelial injury or dysfunction,
and underlying hypercoagulable states. Symptoms are nonspecific and
include dyspnea, pleuritic chest pain, cough, and, in severe cases,
syncope or cardiorespiratory arrest. Signs are also nonspecific and may
include tachypnea, tachycardia, hypotension, and a loud pulmonic
component of the 2nd heart sound. Diagnosis is based on a CT angiogram,
ventilation/perfusion scan, or a pulmonary arteriogram.
A 65-year-old female develops gram-negative septicemia from a urinary
tract infection. Despite the use of fluid resuscitation she remains
hypotensive, with a mean arterial pressure of 50 mm Hg. Which one of the
following would be the most appropriate treatment for this patient?

a) Vasopressin (Pitressin)
b) Phenylephrine (Neo-Synephrine)
c) Epinephrine
d) Norepinephrine (Levophed)
e) Low-dose dopamine
The correct answer is D

Explanation
In a patient with sepsis, vasopressors are indicated when fluid
resuscitation does not restore organ perfusion and blood pressure.
Norepinephrine and dopamine currently are the preferred pressor agents;
however, norepinephrine appears to be more effective and has a lower
mortality rate. Phenylephrine, epinephrine, or vasopressin should not be
used as first-line therapy. Vasopressin is employed after high-dose
conventional vasopressors have failed. The use of low-dose dopamine is
no longer recommended based on a clinical trial showing no benefit in
critically ill patients at risk for renal failure. If an agent is needed
to increase cardiac output, dobutamine is the agent of choice.
A 20 year old man returns to the office for a follow up visit 4 weeks
after being diagnosed with mononucleosis. At his initial visit, he had a
sore throat, nausea and a temperature of 39.4 C (103.0 F). Physical
examination at that time showed pharyngitis, submandibular and posterior
cervical lymphadenitis, and splenomegaly.

Complete blood count (CBC) done at that time showed a leukocyte count of
18,000 with an elevated number of monocytes, and a Monospot test was
positive. He asks you, "When can I play again?" Which of the following
is the most appropriate response to the patient regarding when he may
return to contact sports‟

a) Next season
b) When a Monospot test is negative
c) When he is no longer symptomatic
d) When his CBC is normal
e) When his physical examination is normal
The correct answer is E

Explanation
The concern with infectious mononucleosis, a disease caused by the
Epstein-Barr virus, is for splenomegaly. The issue of contact sports
after a course of mono relates solely to when the risk of splenic injury
has declined. No treatment other than rest is needed in the vast
majority of cases. The spleen, being part of the lymphatic system, will
generally regress in size in accord with the decline in physical signs,
such as swollen lymph nodes.

Waiting until next season is too long and, although likely to allow a
large margin of safety, will unnecessarily keep the student from playing
his sport. The Monospot test is the diagnostic test that detects the
antibody generated against the EBV (heterophil antibody) in the blood of
the patient. Since the immune response is last quite long, a negative
Monospot test may take years to become present.

Waiting for the patient to be symptom free is also too long as fatigue
may persist for months after the resolution of the lymph node
enlargement. The CBC may return to normal well before the lymph nodes
have regressed in size.
What is the chance of genetic transmission of Huntington‟s chorea from a
man to his son?

a) Zero
b) 25%
c) 50%
d) 100%
The correct answer is C

Explanation
Huntington chorea (HC), is an inherited disease characterized by
choreiform movements and progressive dementia. It is inherited as an
autosomal dominant disorder. There is a 50% chance of the man passing
the disease to his son.
A 40 year old woman develops recurrent papules and pustules in a
symmetrical pattern on her cheeks, nose, chin and forehead. See picture:
She blushes easily, especially when consuming hot liquids, alcohol, or
spicy foods. The most likely diagnosis is

a) Acne vulgaris
b) Perioral dermatitis
c) Acne rosacea
d) Seborrheic dermatitis
e) Carcinoid syndrome
The correct answer is C

Explanation
Rosacea (acne rosacea) is a chronic inflammatory disorder characterized
by facial flushing, telangiectasias, erythema, papules, pustules, and in
severe cases, rhinophyma. Diagnosis is based on characteristic
appearance. Rosacea manifests in 4 phases and is limited to the face and
scalp.

In the “pre-rosacea” phase, patients describe embarrassing flushing and


blushing, often accompanied by uncomfortable stinging. Common reported
triggers for these flares include sun exposure, emotional stress, cold
or hot weather, alcohol, spicy foods, exercise, wind, cosmetics, and hot
baths or hot drinks. These symptoms persist throughout other phases of
the disorder. Treatment is with accutane.
A 65-year-old white female develops a burning pain in the left lateral
thorax, followed 2 days later by a erythematous vesicular rash. Of the
following, the best treatment is

a) Topical corticosteroids
b) Oral corticosteroids
c) Topical acyclovir (Zovirax)
d) Oral valacyclovir (Valtrex)
e) Topical capsaicin (Zostrix)
The correct answer is D
Explanation
The rash described is typical of herpes zoster. This commonly occurs in
older individuals who have had chickenpox in childhood. The treatment of
choice for acute herpes zoster is oral antiviral agents. Acyclovir,
valacyclovir, and famiciclovir have all been shown to be efficacious
with 7 days of oral treatment. Studies suggest that valacyclovir may be
superior to acyclovir in decreasing both acute and postherpetic pain.
Famciclovir appears to be equal in efficacy to valacyclovir. Topical
acyclovir may be effective for more limited forms of herpes simplex, but
is usually not effective for herpes zoster. Topical and oral
corticosteroids may have some use for combating the inflammatory
process, and may decrease the incidence of postherpetic neuralgia in
certain individuals. Topic capsaicin may be useful in treating the pain
of acute herpes zoster infection, as well as postherpetic neuralgia.

A 79 year old white male with a previous history of prostate cancer has
a lumbar spine film suggesting osteopenia. Subsequent bone density
studies show a T score of 2.7. Which one of the following would be
appropriate therapy?

a) Testosterone
b) Calcitonin nasal spray (Micalcin)
c) Raloxifene (Evista)
d) Alendronate (Fosamax)
The correct answer is D

Explanation
The only approved treatments for male osteoporosis are alendronate and
recombinant parathyroid hormone. Several drugs have been tested in
clinical trials, and more pharmacologic treatments should become
available in the future as male osteoporosis is increasingly recognized.
Testosterone should not be used in this patient because of his history
of prostrate cancer.
An 85 year old female comes to your office for a routine annual visit.
While reviewing her medical history, you note that she began timolol
(Timoptic) eye drops 3 months ago for the treatment of glaucoma. Which
one of the following findings is most likely related to the eye drops‟

a) Bronchospasm
b) Elevated serum creatinine
c) Hirsuitism
d) Leukopenia
e) New-onset atrial fibrillation
The correct answer is A

Explanation
Ophthalmic Beta-adrenergic antagonists often have systemic side effects.
Beta2-Blockade can cause contraction of bronchial smooth muscle, leading
to bronchospasm; respiratory failure and death due to bronchospastic
complications have been reported, particularly in patients with asthma
or chronic obstructive pulmonary disease.

A 58-year-old white female who has always been in good health sees you
for a routine checkup. She has recently had two friends affected by
cardiovascular disease, and asks if she should be taking any drugs to
help her stay healthy. She does not smoke and her blood pressure is
130/85 mm Hg. Her total cholesterol level is 230 mg/dL, with an HDL
level of 70 mg/dL and an LDL level of 130 mg/dL. Her father had heart
problems beginning at age 65.

You recommend exercise, maintaining ideal weight, and staying on a


low-saturated fat diet. Which one of the following additional
recommendations would be most appropriate?

a) Hydrochlorothiazide to lower her blood pressure to <120/80 mm Hg,


plus aspirin,
81 mg/day, and no additional medications
b) Hydrochlorothiazide, a statin to lower her LDL to <130 mg/dL, and
aspirin, 81
mg/day
c) Aspirin, 81 mg/day, and no additional medications
d) Hydrochlorothiazide plus a statin, and no additional medications
e) No medications
The correct answer is E

Explanation
According to the Adult Treatment Panel III, the first step in deciding
treatment thresholds for cholesterol is to determine the number of risk
factors for a cardiovascular event. Risk factors to be considered
include HDL <35 mg/dL, smoking, hypertension, family history of a first
degree relative with premature atherosclerotic vascular disease (male
<45, female <55), and age (male >55, female >65). In addition, an
HDL-cholesterol level >60 mg/dL is considered a negative risk factor.

Given these thresholds, this patient has no risk factors for heart
disease and is at low risk for a cardiovascular event. Her father‟s
history of heart problems at age 65 is not a risk factor. Her blood
pressure is in the prehypertensive range, but she does not need
pharmacologic treatment. Her total cholesterol is mildly elevated, but
the HDL level is high and treatment is not indicated to lower her LDL
level since she is at low risk.

The Women‟s Health Initiative showed that given this patient‟s low
cardiac risk, aspirin is more likely to cause gastrointestinal bleeding
or serious bleeding following trauma than to protect her from a cardiac
event. Her main strategy for avoiding cardiovascular problems should be
to live a healthy lifestyle and stay off medications, unless her risk
factors change.
A 75 year old male is admitted to hospital with left lower quadrant
abdominal pain, nausea, a low-grade fever, and left lower quadrant
tenderness. Your diagnosis is acute diverticulitis. Treatment should
include all of the following, except

a) Broad spectrum antibiotics


b) Immediate laparotomy
c) Intravenous fluids
d) Meperidine
e) Nasogastric suction
The correct answer is B

Explanation
Diverticulitis is inflammation of a diverticulum, which can result in
phlegmon of the bowel wall, peritonitis, perforation, fistula, or
abscess. The primary symptom is abdominal pain. Diagnosis is by CT scan.

Patients with more severe symptoms (pain, fever, marked leukocytosis)


should be hospitalized, as should patients taking prednisone (who are at
higher risk for perforation and general peritonitis). Treatment is bed
rest, nothing by mouth, IV fluids, and IV antibiotics.

About 80% of patients can be treated successfully without surgery. An


abscess may respond to percutaneous drainage (CT guided). If response is
satisfactory, the patient remains hospitalized until symptoms are
relieved and a soft diet is resumed. A barium enema is performed > 2 wk
after symptoms have resolved.

Surgery is required for patients with free perforation or general


peritonitis and for patients with severe symptoms that do not respond to
nonsurgical treatment within 48 hours.
A 36-year-old male who recently immigrated from Rwanda presents with a
several-month history of cough accompanied by hemoptysis and weight
loss. He is afebrile, and a lung examination is normal. A chest
radiograph shows a cavitary lesion in his left upper lobe. He is
admitted to the hospital and placed in respiratory isolation. A
tuberculin test is positive, but three induced sputum smears are
negative for acid-fast bacillus. Cultures are still pending.

Which one of the following INITIAL treatment regimens is most appropriate?

a) No treatment until culture results are available


b) Isoniazid for 9 months
c) Rifampin and pyrazinamide for 8 weeks
d) Rifampin and/or isoniazid for 18 weeks
e) Rifampin, isoniazid, ethambutol, and pyrazinamide for 8 weeks
The correct answer is E

Explanation
Nearly one third of the world‟s population is infected with
Mycobacterium tuberculosis. The highest per capita incidence occurs in
sub-Saharan Africa. Although a positive acid-fast bacillus smear or
culture for M. tuberculosis is needed to confirm active disease,
treatment should begin without delay in patients for whom the clinical
suspicion is high, as in this case. Because of relatively high rates of
resistance to isoniazid, four drugs are necessary in the initial phase
to be maximally effective. Thus, the initial treatment regimen for all
adults with previously untreated tuberculosis should consist of 8 weeks
of therapy with rifampin, isoniazid, ethambutol, and pyrazinamide. Most
patients will require an 18-week continuation phase after initiation. If
the likelihood of active tuberculosis is low, treatment can be deferred
until the results of mycobacterial cultures are known.

In low-risk patients who are not initially treated, one of the other
treatment regimens for latent tuberculosis is acceptable if cultures are
negative, a tuberculin skin test is positive, and a chest radiograph is
unchanged after 2 months. However, the 8-week regimen of rifampin and
pyrazinamide should be used only in patients who are not likely to
complete a longer course of therapy and who can be monitored closely
because of an increased risk of hepatotoxicity.
A 52 year old white male has chronic musculoskeletal pain. He has been
using frequent doses of Extra-Strength Tylenol with good results. He is
concerned that he may be taking too much and asks what his maximum daily
dosage of acetaminophen should be. He weighs 70 kg (154 lb). You tell
him that the most he should take in 24 hours is

a) 2000 mg
b) 3000 mg
c) 4000 mg
d) 5000 mg
e) 6000 mg
The correct answer is C

Explanation
The maximum daily dosage for all acetaminophen preparations is 4000 mg.
Acetaminophen is used in more combination products than any other drug,
for a number of different indications. An FDA panel has recommended that
stronger warnings about hepatotoxicity be added to the label information
for acetaminophen. Because it is used so frequently and is present in so
many different preparations, care must be taken not to exceed the
maximum 24-hour dosage in order to avoid hepatotoxicity.
A 55-year-old male has heart failure. He becomes dyspneic with
significant exertion. His only medication is an ACE inhibitor. Which one
of the following additional medications has been shown to improve
longevity in this situation?

a) Digitalis
b) Warfarin (Coumadin)
c) Beta-Blockers
d) Amiodarone (Cordarone)
e) Nondihydropyridine calcium channel blockers
The correct answer is C

Explanation
-Blockers are recommended to reduce mortality in symptomatic patients
with heart failure (SOR A). Because polypharmacy can reduce compliance,
the role that digoxin will ultimately play in heart failure is unclear.
The Digitalis Investigation Group study revealed a trend toward
increased mortality among women with heart failure who were taking
digoxin, but digoxin levels were higher among women than men. There is
no evidence that warfarin decreases mortality in patients with heart
failure. There is also no evidence that amiodarone decreases mortality
from heart failure in patients with no history of atrial fibrillation.

Calcium channel blockers should be used with caution in patients with


heart failure because they can cause peripheral vasodilation, decreased
heart rate, decreased cardiac contractility, and decreased cardiac
conduction.

A 70 year old alcoholic male is recovering from a nontransmural


myocardial infarction. On the fourth hospital day, he describes a sudden
onset of excruciating abdominal pain, which is not significantly reduced
by large doses of morphine. He becomes nauseated, begins to vomit, and
has diarrhea.

The patient appears agitated and confused, and his heart rate increases.
He also becomes hypotensive. Physical examination of his abdomen reveals
minimal tenderness, decreased bowel sounds, and a moderately enlarged liver.

Laboratory Findings
WBCs................................................17,600/mm3 with a left shift (N 4300-10,800)
Hematocrit.........................................54% (N 37-49)
BUN...................................................40 mg/dL (N 8-25)
Creatinine...........................................1.0 (N 0.6-1.5)
Serum amylase...................................250 U/L (N 43-115)
Serum lipase.......................................100 U/L (N 0-160)
pH...................... ....................................7.14 (N 7.35-7.45)

The most likely diagnosis is

a) Alcohol withdrawal syndrome


b) Pulmonary embolism
c) Pancreatitis
d) Acute mesenteric artery embolism
The correct answer is D

Explanation
The hallmark of acute mesenteric artery ischemia is severe abdominal
pain that is out of proportion to physical findings. This is a
life-threatening event which often follows myocardial infarction when a
mural thrombus occludes a superior mesenteric artery. Patients rapidly
become acidotic and hypotensive, and experience a high mortality rate.

Alcoholic withdrawal syndrome has a much more insidious onset, with


tremors, agitation, and anxiety being the prominent features. Abdominal
pain is not a common prominent symptom or finding. Although pulmonary
embolism is possible in the setting described, dsypnea and chest pain
are the major symptoms. Abdominal pain and tenderness are not the usual
manifestation of this disease. A normal serum lipase level should point
to a non-pancreatic origin for this patient‟s problem. A mildly elevated
serum amylase level is not specific to the diagnosis and is commonly
seen in other pathologies.
A 60-year-old male who has a 5-year history of diabetes mellitus is
found to have an elevated creatinine level that has increased from 1.5
mg/dL to 2.0 mg/dL over the past year. His current medications include
lisinopril (Prinivil, Zestril) to control blood pressure and
proteinuria. He is aware that tight control of his blood glucose will
help prevent the progression of renal disease, but wants to know what
else he can do to preserve renal function. Which one of the following
would be the most appropriate recommendation for this patient?

a) Add an angiotensin receptor blocker (ARB)


b) Maintain blood pressure at 140/90 mm Hg or less
c) Follow a high-protein, low-fat diet
d) Use erythropoietin therapy to keep the hemoglobin level above 14 g/dL
e) Add a calcium channel blocker
The correct answer is A

Explanation
There are several strategies for slowing the progression of chronic
kidney disease to end-stage renal disease, which requires either a renal
transplant or dialysis. ACE inhibitors have been shown to be
renoprotective by preventing the progression of diabetic nephropathy.
There also is data showing that angiotensin receptor blockers probably
are as effective as ACE inhibitors. Combining the two can further retard
the progress of chronic kidney disease, but with the caveat that the two
medications can increase the risk of hyperkalemia in a patient with
impaired renal function, and the combination confers a higher mortality
risk than either agent alone.

In addition to tight glucose control, cessation of smoking, weight


control, and treatment of hyperlipidemia, patients should strive to
control hypertension. If proteinuria is less than 1 g/day, then the
blood pressure goal should be 130/80 mm Hg; however, if proteinuria is
more than 1 g/day, some authorities recommend a goal of 125/75 mm Hg.
While good control of blood pressure is ideal, calcium channel blockers
(especially the dihydropyridine class) should be used only if necessary,
because they have been associated with a more rapid decline of
glomerular filtration rate.

Recent studies have shown that patients on a low-protein diet have a


slower rate of deterioration in renal status. The recommended
restriction is 0.6-0.8 g/kg/day.

Anemia of chronic disease is a manifestation of chronic renal disorder.


Although erythropoietin therapy may not prevent progression, it does
improve survival and quality of life.

Which of the following is associated with biliary colic?

a) Epigastric pain
b) Rebound tenderness
c) Jaundice
d) Murphy sign
The correct answer is A

Explanation
About one third of patients with gallstones develop biliary colic or
other complications. Colic pain is precipitated by spasm of a dilated
cystic duct that is obstructed by gallstones. Attacks of biliary colic
are more common at night, possibly because the gallbladder shifts to a
horizontal position, facilitating the entry of stones into the cystic
duct. Biliary colic is characterized by the sudden onset of intense
right upper abdominal pain that may radiate to the shoulder. Sweating
and vomiting are common. The pain tends to be steady and lasts up to
three hours. Residual abdominal tenderness may occur.

Murphy sign, which is inspiratory arrest with palpation of right upper


quadrant is seen in cholecystitis.
A positive flexion abduction external rotation (FABER) test that elicits
posterior pain indicates involvement of which joint?

a) Sacroiliac
b) Shoulder
c) Ankle
d) Wrist
e) Knee
The correct answer is A

Explanation
When the flexion abduction external rotation (FABER) test elicits pain
posteriorly, it indicates sacroiliac involvement. Anterior pain
indicates hip involvement.
A 25 year old male presents to the ER complaining of palpitations of 1
hour duration. He denies any major medical illness. He does not smoke
but admits that he drinks caffeine excessively.

His vitals are within normal limits except for a pulse rate of 160/min.
The EKG showed a SVT which has been controlled by carotid massage. A
second EKG tracing is done and showed the following:

Which of the following is the most likely diagnosis of this patient?

a) Brugada syndrome
b) James preexcitation syndrome
c) Mahaim Preexcitation syndrome
d) Right bundle branch block
e) Wolff Parkinson White syndrome
The correct answer is E

Explanation
WPW is a congenital heart abnormality in which an accessory pathway
connects the atria to the ventricle leading to a preexcitation syndrome.
It is present in around 0.2-0.3% of the population. It is characterized
by its special EKG showing: the Delta wave (slow depolarization of the
QRS Complex), a wide QRS complex and a short PR interval.

Brugada syndrome is presented on the EKG by ST elevation with at least 2


mm J-point elevation a gradually descending ST segment and a negative
T-wave.

James and Mahaim syndromes are preexcitation syndromes without a Delta wave.

RBBB is characterized by a wide QRS complex and an ?M shape? of the QRS


complex in V1 and V2; moreover, it has no delta wave.

Do not get confused - forget about the relatively rare congenital heart
diseases such as Brugada, James and Mahaim syndromes. Instead, focus on
WPW and its management! This is a target for the licensing exams and
common condition in your clinical practice.
An elderly man presents with a skin lesion on his arm. He states that it
has been growing in the last few months. The lesion is asymmetric, has
no distinct border, varies in color and is greater than 6mm. What is the
most likely diagnosis‟

a) Basal cell carcinoma


b) Squamous cell carcinoma
c) Malignant melanoma
d) Pityriasis Rosea
e) Nevus
The correct answer is C

Explanation

The ABCD criteria is used to diagnose a malignant melanoma. ABCD is an


abbreviation for asymmetry, border, color and diameter.

Malignant melanoma arises from melanocytes in a pigmented area: skin,


mucous membranes, eyes, and CNS. Metastasis is correlated with depth of
dermal invasion. With spread, prognosis is poor.

Diagnosis is by biopsy. Wide surgical excision is the rule for operable


tumors. Metastatic disease requires chemotherapy but is difficult to cure.
A 26-year-old female presents with a 1-year history of recurring
abdominal pain associated with intermittent diarrhea, 5?7 days per
month. Her pain improves with defecation. There has been no blood in her
stool and no weight loss. Laboratory findings are normal, including a
CBC, chemistry profile, TSH level, and antibodies for celiac disease.
Which one of the following would be most appropriate at this point?

a) Colonoscopy
b) An upper GI series with small-bowel follow-through
c) Abdominal CT with contrast
d) A gluten-free diet
e) Loperamide (Imodium)
The correct answer is E

Explanation
This patient has classic symptoms of irritable bowel syndrome (IBS) and
meets the Rome criteria by having 3 days per month of abdominal pain for
the past 3 months, a change in the frequency of stool, and improvement
with defecation. According to current clinical guidelines IBS can be
diagnosed by history, physical examination, and routine laboratory
testing, as long as there are no warning signs. Warning signs include
rectal bleeding, anemia, weight loss, fever, a family history of colon
cancer, onset of symptoms after age 50, and a major change in symptoms.
Colonoscopy, CT, and GI contrast studies are not indicated. A
gluten-free diet would not be indicated since the antibody tests for
celiac disease are negative. Antidiarrheal agents such as loperamide are
generally safe and effective in the management of diarrheal symptoms in IBS.
A 31 year old woman with known HIV presents to her physician with a 3
month history of watery diarrhea, severe weakness, and a 22 pound weight
loss. Multiple stool tests for bacteria, ova, and parasites are
repeatedly negative. A colonoscopy is normal, as are biopsies of the
colon. Which of the following is the most likely explanation for her
diarrhea?

a) Cryptosporidiosis
b) Cytomegalovirus infection
c) Entamoeba histolytica
d) Enterotoxigenic Escherichia coli
e) Shigella dysenteriae
The correct answer is A

Explanation
This patient has a typical small bowel-type diarrhea, seen in HIV with
watery diarrhea, weakness, and weight loss. The most common etiology for
this syndrome is cryptosporidiosis infection of the small intestine,
where the spores can be seen on the tips of the villi on biopsy. This
organism can be demonstrated with special culture media. Other organisms
in the same family, such as microsporidia and Isospora belli, produce
identical syndromes.
A 46 year old woman has had rheumatoid arthritis for 8 years. Her hands
now show moderate ulnar deviation of the fingers and she says her wrists
and knees also hurt. She says, "I had several drop attacks during the
past 3 months." She characterizes these attacks as episodes of weakness
and loss of feeling in her legs for several minutes. During one of these
episodes, she became incontinent. On physical examination she has facial
plethora and swollen and painful metacarpophalangeal and knee joints,
bilaterally. The rest of the examination is normal. Which of the
following is the most likely cause of her "drop attacks‟"

a) Adrenal insufficiency
b) Anxiety
c) Atlanto-occipital instability
d) Cardiac arrhythmia
e) Cerebral ischemia
The correct answer is C

Explanation
Careful attention to her history however allows the diagnosis to
ascertained with near certainty. The drop attacks are characterized by
incontinence with weakness and loss of feeling in her legs. These
symptoms point to a neurologic defect that affects both the
parasympathetic and motor functions of the nervous system.
Atlanto-occipital joint subluxation is a common problem for patients
with advanced rheumatoid arthritis, as this patient clearly suffers
from. If the cervical spine were to suffer impingement from such
subluxation, it would be expected that motor loss below the level of the
impingement aswell as loss of bladder control from interruption of the
parasympathetic outflow (cranial component).
A 48 year old male is brought to the ED with CPR being administered. The
ECG shows electrical activity present but you cannot palpate a carotid
pulse. Which of the following is not on the differential for pulseless
electrical activity?

a) Hypotension
b) Alkalosis
c) Hypothermia
d) Tension pneumothorax
e) Cardiac tamponade
The correct answer is B

Explanation
These possible causes are remembered as the Hs and Ts, as follows:

Hypovolemia
Hypoxia
Hydrogen ions (Acidosis)
Hypothermia
Hyperkalemia or Hypokalemia
Hypoglycemia
Hypocalcaemia
Tablets or Toxins (Drug overdose)
Cardiac Tamponade
Tension pneumothorax
Thrombosis (Myocardial infarction)
Thrombosis (Pulmonary embolism)
Trauma (Hypovolemia from blood loss)
Which one of the following is the most common psychiatric complication
of insulin-dependent diabetes mellitus‟

a) Major affective disorder


b) Initial adjustment disorder with depressed mood
c) Suicide attempts with the emergence of diabetic retinopathy
d) Panic attacks
The correct answer is B

Explanation
Psychological morbidity is increased with diabetes, as it is in children
with other chronic illnesses. Initial adjustment to diabetes is
characterized by sadness, anxiety, withdrawal, and dependency, and 30%
of children develop a clinical adjustment disorder in the 3 months after
diagnosis. Such difficulties often resolve within the first year, but
poor adaptation in this initial phase places children at risk for later
psychological difficulties.

Mood disorders such as major depressive disorder and dysthymia are the
most frequently reported diagnoses in youth with type 1 diabetes, with a
cumulative probability of 27.5% by the 10th year of type 1 diabetes
duration.
One year after being diagnosed with early Alzheimer‟s disease, one of
your long-time patients develops symptomatic carotid stenosis. A
vascular surgeon has recommended surgical treatment, but the patient‟s
family is uncertain whether he should have the surgery or whether he is
capable of making the decision. The children are evenly split in their
opinion regarding the surgery, and they ask your opinion about this
decision.

Which one of the following is true regarding this situation?

a) The patient is incapable of making this decision because of his dementia


b) The Mini-Mental State Examination score determines competence
c) The patient should be evaluated by a psychiatrist
d) A judicial determination of competence should be obtained
e) The patient‟s decision-making capacity can be adequately assessed by
clinical
evaluation
The correct answer is E

Explanation
The primary care physician can assess a patient‟s decision-making
capacity based on ability to reason, communicate, understand the
proposed treatment, and grasp the consequences of accepting or declining
the suggested treatment.

Formal mental status testing and determination of capacity are different


functions. However, there is a certain level of cognitive impairment
where a patient simply lacks any ability to receive and process health
information. At somewhat higher levels of cognition a patient might lack
specific mental abilities, but still be able to satisfy the requirements
for making treatment decisions.

Accurate mental status testing is helpful for assessing the capacity to


choose; however, there is not a specific score that determines capacity.
Determination of capacity does not require legal intervention or
psychiatric expertise. There is no specific test for decision-making
capacity.

Competence is a legal term in this situation. Decisions regarding


competence are judicial determinations when evaluating the capacity of a
person to make nonmedical decisions such as financial decisions. Under
the law, adults are presumed to be competent until a specific action of
the appropriate court finds them otherwise.
Which of the following would not be expected in a right-sided
Brown-Séquard syndrome?

a) Right-sided hemi-paresis
b) Right-sided decreased proprioception
c) Left-sided decreased sensitivity to pinprick
d) Left-sided decreased vibration sense
The correct answer is D

Explanation
Brown-Sequard syndrome (BSS) is a rare neurological condition
characterized by a lesion in the spinal cord which results in weakness
or paralysis (hemiparaplegia) on one side of the body and a loss of
sensation (hemianesthesia) on the opposite side. In addition, loss of
proprioception (vibratory sense), spastic paralysis, and hyperactive
deep tendon reflexes develop ipsilaterally (same side).

BSS may be caused by a spinal cord tumor, trauma (such as a puncture


wound to the neck or back), ischemia (obstruction of a blood vessel), or
infectious or inflammatory diseases such as tuberculosis, or multiple
sclerosis.
In a patient who presents with symptoms of acute myocardial infarction,
which one of the following would be an indication for thrombolytic therapy?

a) New-onset ST-segment depression


b) New-onset left bundle branch block
c) New-onset first degree atrioventricular block
d) New-onset Wenckebach second degree heart block
e) Frequent unifocal ventricular ectopic beats
The correct answer is B

Explanation
In patients with ischemic chest pain, the EKG is important for
determining the need for fibrinolytic therapy. In addition to ST
elevation 1 mm in two or more contiguous standard limb leads and 2 mm in
two or more contiguous precordial leads, patients should also have new
left bundle branch block. In a patient with a presumed MI, left bundle
branch block suggests occlusion of the left anterior descending artery,
placing a significant portion of the left ventricle in jeopardy.
Thrombolytic therapy could be harmful in patients with ST-segment
depression only. Frequent unifocal ventricular ectopy may warrant
antiarrhythmic therapy, but not thrombolytic therapy.
Physiologic changes in the elderly lead to which one of the following?

a) Resistance to dehydration
b) A tendency toward hypernatremia due to diminished thirst response
c) Lower than usual concentrations of medications distributed in body
fluids
d) Greater than usual required dosages of medications
The correct answer is B

Explanation
There is no known resistance to dehydration in the elderly. There is a
smaller volume of dilution for drugs distributed in body fluids. This
may result in higher concentrations of medications, sometimes
approaching toxic levels, especially if the usual adult dosages are
given. Because elderly patients have a diminished thirst response, they
are prone to develop hypernatremia when water is not immediately available.

A 65 year old white female presents with weight loss and fatigue. On
examination, she has lymphadenopathy, hepatomegaly, and mild
splenomegaly. Her hemoglobin level is 9.0 g/dL (N 12.0-16.0), and a
chemistry panel reveals a serum protein level of 9.0 g/dL (N 6.0-8.0).
You order a chest radiograph, which shows clear lung fields and no
evidence of lytic lesionsin the thoracic spine. Serum protein
electrophoresis reveals a monoclinal (-globulin spike, which on
immunoelectrophoresis is found to be due to IgM 6-protein. Urine for
Bence-Jones protein is positive. A bone marrow biopsy from the iliac
crest demonstrates hypercellularity, with a large number of lymphocytes,
but normal-appearing plasma cells. Which one of the following is the
most likely diagnosis‟

a) Multiple myeloma
b) Waldenstrom‟s macroglobulinemia
c) Sarcoidosis
d) Monoclonal gammopathy of undetermined significance
e) Non-Hodgkin‟s lymphoma
The correct answer is B

Explanation
The patient has symptoms, signs, and laboratory findings consistent with
a diagnosis of Waldenström‟s macroglobulinemia. This illness is due to
an uncontrolled proliferation of lymphocytes and plasma cells, which
produce IgM proteins with ?-light chains. The average age at the time of
diagnosis is 65 years old. Weakness, fatigue, weight loss, bleeding, and
recurrent infections are common presenting symptoms. Physical findings
include pallor, hepatosplenomegaly, and lymphadenopathy. Typical
laboratory findings include moderate anemia and monoclonal IgM peaks on
serum electrophoresis. Bence-Jones protein is seen in 80% of cases, but
is typically absent in monoclonal gammopathy of undetermined
significance. Unlike in multiple myeloma, lytic bone lesions are not
seen, and marrow biopsy reveals mostly lymphocytes. Sarcoidosis usually
presents with hilar lymphadenopathy and a polyclonal gammopathy.
Non-Hodgkin‟s lymphoma presents with similar symptoms, lymphadenopathy,
and hepatosplenomegaly, but generally lacks a monoclonal gammopathy and
Bence-Jones proteinemia, and has distinctive malignant lymphocytes on
bone marrow biopsy.
Treatment of mild persistent asthma with a low-dose inhaled
corticosteroid alone would be the preferred treatment for a patient
whose asthma symptoms occur

a) Once daily
b) 3 times a week
c) Daily
d) Continuously
e) 2 times a week, but only with exercise
The correct answer is B

Explanation
Low-dose inhaled corticosteroids are preferred for patients with mild
persistent asthma, defined as the occurrence of symptoms more than twice
a week but less than once a day. Patients with severe persistent asthma,
defined as continual symptoms, should be treated with high-dose inhaled
corticosteroids and long-acting inhaled Beta2-agonists. Moderate asthma,
defined as daily symptoms, should be treated with low- to medium-dose
inhaled corticosteroids and long-acting inhaled Beta2-agonists. Mild
intermittent asthma, with symptoms 1 or 2 days per week, does not
require daily medication.

A 57 year old male with a history of heart failure, obesity, retention


of fluids, renal disease and shortness of breath is taking digitalis.
Which of the following describes the therapeutic effect produced by this
medication?

a) Decrease in the cardiac filling pressure and increase in cardiac output


b) Increase in the cardiac filling pressure and decrease in cardiac output
c) Increase in heart rate and decrease in cardiac output
d) Decrease in heart rate and decrease in cardiac output
e) Increase in force of contraction and decrease in cardiac output
The correct answer is A
Explanation
Digitalis reduces the cardiac filling pressure, increases the force of
contraction of myocardium with increase in the cardiac output but
decreases the heart rate. Digitalis also decreases venous and capillary
pressure with increased renal blood flow and decreased heart size. It
can also act by activation of neurohormonal system.

In a patient with acute renal failure, the diagnosis of acute


glomerulonephritis is most strongly supported by finding which one of
the following in the patient‟s urine?

a) RBC casts
b) WBC casts
c) Hyaline casts
d) Urine myoglobin
e) Urine eosinophils
The correct answer is A

Explanation
Acute renal failure (ARF) can develop over a period of days or weeks,
has multiple causes, and may be prerenal, intrinsic renal (subdivided
into vascular, glomerular, tubular, and interstitial), or postrenal. The
clinical history and presentation of the patient are obviously very
important in elucidating the cause, but the urinalysis provides valuable
clues as to the type of failure. With intrarenal failure, localization
to a vascular, glomerular, tubular, or interstitial focus is aided by
evaluation of the urine sediment. The presence of RBC casts indicates
glomerular disease, most commonly acute glomerular nephritis. This
condition is clinically manifested by hypertension, edema, and hematuria.

Hyaline casts are typical of a prerenal etiology, including hypovolemia


or decreased circulatory volume, as in heart failure. NSAID or ACE
inhibitor use may also result in prerenal ARF. WBC casts indicate acute
pyelonephritis or interstitial nephritis. The latter condition should be
suspected if eosinophils are found in the urine and may be due to an
allergic reaction to a drug. Rhabdomyolysis is most often related to
trauma or seizures, and results in circulating myoglobin which is
nephrotoxic and leads to renal failure due to acute tubular necrosis
(ATN). The urine in these patients will be heme-positive with few or no
red blood cells on microscopy.

A 59 year old male presents with unremitting fever of 4 weeks duration.

The fever has been associated with fatigue, malaise and night sweats.
The patient also admits to an unintentional 30 pounds weight loss in the
last two months. He drinks 3-4 beverages of alcohol every day since the
age of 20.

His vital signs are within normal limits except for a fever of 38C.
Physical exam is significant for a new onset murmur in the apical area
of the heart. Three sets of blood cultures were taken and empiric
antibiotics were started. Two sets of the cultures returned positive for
Streptococcus bovis. After proper management of the patient‟s presenting
disease, what would you do next?
a) Chest X-ray
b) Colonoscopy
c) Cystoscopy
d) Repeat the blood culture
e) Upper GI endoscopy
The correct answer is B

Explanation
When ever you find an unusual organism in an unusual site think of the
source. The patient is having Strep bovis endocarditis which is unusual
to be the cause of endocarditis. Strep bovis is usually present in the
colon, thus, you should think of a colon lesion such as colon cancer as
the source of the infection.

Upper GI endoscopy and cystoscopy are not useful as the organism is


normally present in the colon only.

Chest X-ray would be warranted but for other purposes (cardiomegaly,


septic emboli to the lungs‟)

Two positive culture sets out of three for an organism is highly


sensitive and specific; thus, no need to repeat the cultures.

>Strep bovis or Clostridium septicum endocarditis = Colonoscopy on the


licensing exams!
A woman is attacked by a bee. She comes to emergency room with
hypotension, tachycardia and visible allergic reaction with swelling.
What is the most appropriate initial management?

a) Antihistamine
b) Saline perfusion
c) Epinephrine injection
d) Intubation
e) IV steroid
The correct answer is C

Explanation
Anaphylactic reactions (anaphylaxis) are sudden, widespread, potentially
severe and life-threatening allergic reactions.

Anaphylactic reactions begin within 1 to 15 minutes of exposure to the


allergen. Rarely, reactions begin after 1 hour. The heart beats quickly.
The person may feel uneasy and become agitated. Blood pressure may fall,
causing fainting. Other symptoms include tingling (pins-and-needles)
sensations, itchy and flushed skin, throbbing in the ears, coughing,
sneezing, hives, and swelling (angioedema).

If an anaphylactic reaction occurs, an epinephrine injection should be


given immediately.
A 31-year-old male with type 1 diabetes mellitus is admitted to the
hospital with diabetic ketoacidosis and pneumonia. After initial
treatment in the emergency department with intravenous fluids and
insulin, laboratory tests reveal a serum phosphate level of 2.1 mg/dL (N
2.5?5.0). He is asymptomatic except for related pneumonia symptoms.

Which one of the following would be appropriate management of this


patient‟s low serum phosphate level?

a) No therapy
b) Oral phosphate replacement, 2.5?3.5 g/day in divided doses
c) Oral phosphate replacement, 2.5?3.5 g/day in divided doses, and oral
vitamin D
supplementation, 400?800 IU/day
d) Intravenous phosphate replacement, 0.08?0.16 mmol/kg over 6 hours
The correct answer is A

Explanation
Symptomatic hypophosphatemia rarely occurs unless serum phosphate levels
are below 2.0 mg/dL. Serious symptoms, including rhabdomyolysis, do not
occur until serum phosphate concentrations fall below 1.0 mg/dL. Thus,
treatment of hypophosphatemia with phosphate levels greater than or
equal to 2.0 mg/dL is targeted at an underlying etiology.

Hypophosphatemia in diabetic ketoacidosis cases is related to the


internal redistribution of phosphate from extracellular fluid during
treatment, and will resolve when normal dietary intake resumes.
Phosphate supplementation in this setting has not been shown to be
beneficial. Intravenous administration of phosphate can be dangerous,
resulting in the precipitation of calcium and producing the adverse
effects of hypocalcemia, renal failure, and possibly fatal arrhythmias.
A 72-year-old Asian female is found to have asymptomatic gallstones on
abdominal ultrasonography performed to evaluate an abdominal aortic
aneurysm. Which one of the following would be the most appropriate
management for the gallstones‟

a) Laparoscopic cholecystectomy
b) Open cholecystectomy
c) Lithotripsy
d) Treatment with ursodeoxycholic acid (Actigall)
e) Observation
The correct answer is E

Explanation
Gallstones are frequently discovered on a diagnostic workup for an
unrelated problem. Only 1%-2% of persons with asymptomatic gallstones
will require cholecystectomy in a given year, and two-thirds of patients
with asymptomatic gallstones will remain symptom free over a 20-year
period. The longer the patient remains asymptomatic, the more likely
that no symptoms will develop in the future. This patient may have had
gallstones for several years, and the best management would be to do
nothing unless symptoms develop.

A 54-year-old white female has been taking amoxicillin for 1 week for
sinusitis. She has developed diarrhea and has had 6?8 stools per day for
the past 2 days. Examination shows the patient to be well hydrated with
normal vital signs and a normal physical examination. The stool is
positive for occult blood, and a stool screen for Clostridium difficile
toxin is positive. The most appropriate treatment at this time would be

a) vancomycin (Vancocin) intravenously


b) metronidazole (Flagyl) orally
c) trimethoprim/sulfamethoxazole (Bactrim, Septra) orally
d) ciprofloxacin (Cipro) orally
The correct answer is B

Explanation
Many antibiotics can induce pseudomembranous colitis. Although oral
vancomycin was once the initial drug of choice for C. difficile, oral
metronidazole is now the first-line agent because of cost considerations
and because of concerns about the development of vancomycin-resistant
organisms. If the patient has refractory symptoms despite treatment with
oral metronidazole, then oral vancomycin would be appropriate.

Which of the following is most likely to be found in a patient with


acoustic neuroma in the affected ear?

a) Weber test showing tuning fork loudest in the affected ear


b) Air conduction equal to bone conduction
c) Air conduction greater than bone conduction
d) Bone conduction greater than air conduction
The correct answer is C

Explanation
An acoustic neuroma is a Schwann cell?derived tumor of the 8th cranial
nerve. Symptoms include unilateral hearing loss. Slowly progressive
unilateral sensorineural hearing loss is the hallmark symptom.

In the Weber test a tuning fork is struck and the stem of the fork is
placed on the top of the patient's skull - equal distance from the
patient's ears, in the middle of the forehead. The patient is asked to
report in which ear the sound is heard louder.
A patient with a unilateral sensorineural hearing loss would hear the
sound loudest in the unaffected ear.

A patient with a unilateral conductive hearing loss would hear the


tuning fork loudest in the affected ear.

The Rinne test is performed by placing a vibrating tuning fork initially


on the mastoid process (behind the ear) until sound is no longer heard,
the fork is then immediately placed just outside the ear. Normally, the
sound is audible at the ear.

In a normal ear, air conduction (AC) is better than bone conduction


(BC). AC > BC, and this is called a positive Rinne.

In conductive hearing loss, bone conduction is better than air: BC > AC,
a negative Rinne.

In sensorineural hearing loss, bone conduction and air conduction are


both equally depreciated, maintaining the relative difference of: AC >
BC, a positive Rinne.
An elderly woman develops a stasis ulcer on her lower extremity that
causes pigmentary changes. See picture:
The most appropriate intervention in the prevention and treatment of
this condition is

a) Pneumatic stockings
b) Stripping saphenous vein
c) Topical antibiotics
d) Anticoagulants
The correct answer is A

Explanation
Venous ulcers form when blood flow through the legs is reduced, causing
blood to pool in the leg veins. Then, pressure increases in the veins
and capillaries (tiny blood vessels that connect arteries and veins).
The increased pressure causes fluid to leak from the blood vessels into
surrounding tissue, and swelling develops.

If venous ulcers result from chronic venous insufficiency, the legs are
swollen, and the skin is dark reddish brown and very firm (a condition
called stasis dermatitis). The skin may itch, and the ulcers are usually
very painful.

The goals of prevention and treatment are to reduce leg swelling and
reduce the pressure in veins. If swelling is reduced enough, a venous
ulcer may never form or may heal on its own. Often, specially designed
bandages and pneumatic (compression) stockings help reduce swelling.

If cellulitis or signs of an infection of deeper tissues (such as pus)


are present, antibiotics are taken by mouth or given intravenously.
Drugs, such as nonprescription antibiotics, should not be applied
directly to the skin of the leg. These drugs can cause allergic
reactions that worsen the problem, particularly when the skin is cracked
or there is an open sore.
A patient with ascites is suspected to have secondary
hyperaldosteronism. Which of the following would be typical levels of
electrolytes in an aliquot specimen of urine?

a) Sodium 2 mEq/L, potassium 40 mEq/L


b) Sodium 5 mEq/L, potassium 0 mEq/L
c) Sodium 40 mEq/L, potassium 40 mEq/L
d) Sodium 80 mEq/L, potassium 2 mEq/L
e) Sodium 100 mEq/L, potassium 20 mEq/L
The correct answer is A

Explanation
Secondary hyperaldosteronism is characterized by sodium retention, and
thus decreased urinary sodium excretion, while potassium secretion is
normal to increased.
Which one of the following is preferred for chronic treatment of
congestive heart failure due to left ventricular systolic dysfunction?

a) Diuretics
b) Digoxin
c) Calcium channel blockers
d) ACE inhibitors
e) Hydralazine (Apresoline) plus isosorbide dinitrate (Isordil, orbitrate)
The correct answer is D

Explanation
ACE inhibitors are the preferred drugs for congestive heart failure due
to left ventricular systolic dysfunction, because they are associated
with the lower mortality. The combination of hydralazine/isosorbide
dinitrate is a reasonable alternative, and diuretics should be used
cautiously. It is not known whether digoxin affects mortality, although
it can help with symptoms.
Which of the following are not consistent with primary (spontaneous)
bacterial peritonitis‟

a) Abdominal discomfort and fever


b) Ascitic fluid neutrophil count of> 250x10^6 cells/L
c) Ascitic fluid WBC count of >500x10^6 cells /L
d) Multiple organisms on culture and sensitivity of ascitic fluid
The correct answer is D

Explanation
Spontaneous bacterial peritonitis (SBP) is infection of ascitic fluid
without an apparent source. SBP is particularly common in cirrhotic
ascites, especially in alcoholics. It can produce serious sequelae or
death. The most common bacteria causing SBP are the gram-negative
Escherichia coli and Klebsiella pneumoniae and the gram-positive
Streptococcus pneumoniae; usually only a single organism is involved.

An 83-year-old female presents to your office as a new patient. She


recently moved to the area to be closer to her family. A history reveals
that she has been in excellent health, has no complaints, and is on no
medication except occasional acetaminophen for knee pain. She has never
been in hospital and has not had any operations. She says that she feels
well.

The examination is normal, with expected age-related changes, except


that her blood pressure on three different readings averages 174/70 mm
Hg. These readings are confirmed on a subsequent follow-up visit.

In addition to lifestyle changes, which one of the following would be


most appropriate for the initial management of this patient‟s hypertension?

a) An alpha-blocker
b) An ACE inhibitor
c) A beta-blocker
d) An angiotensin receptor blocker
e) A thiazide diuretic
The correct answer is E

Explanation
Clinical trials support the treatment of systolic hypertension in the
older person with a systolic blood pressure of at least 160 mm Hg.
(Systolic hypertension is defined as systolic blood pressure of at least
140 mm Hg and a diastolic blood pressure of less than 90 mm Hg). The
studies most strongly support the use of thiazide diuretics and
long-acting calcium channel blockers as first-line therapy.
Alpha-Blockers are not recommended. ACE inhibitors, beta-blockers, and
angiotensin receptor blockers are used when certain compelling
indications are present, e.g. in a patient with diabetes or who has had
a myocardial infarction.
Which one of the following is true concerning eating disorders and bone
density?

a) Oral contraceptives will correct osteoporosis


b) Oligomenorrhea resulting from bulimia decreases bone density
c) Normalization of weight in anorexia is the most important factor in
regaining bone density
d) Osteoporosis is seen in patients in their late 20s or early 30s but
not in adolescents
The correct answer is C

Explanation
The patient with anorexia is at high risk for developing osteoporosis in
the adolescent years. Oral contraceptives may restore menses, but they
do not help increase bone density. Oligomenorrhea in patients with
bulimia does not affect bone density. In the adolescent with anorexia
nervosa, normalization of body weight is the single most important
factor in regaining bone density.
You are evaluating a patient with a tremor. On examination, you note
golden brown pigmented rings encircling the cornea.

You should suspect

a) Adult-onset idiopathic dystonia


b) Essential tremor
c) Enhanced physiologic tremor
d) Parkinson‟s disease
e) Wilson‟s disease
The correct answer is E

Explanation
If other clinical features suggestive of Wilson‟s disease are present,
such as dysarthia, dystonia, and parkinsonism, then a careful slit-lamp
examination of the eye should be performed by an experienced
ophthalmologist. Kayser-Fleischer rings on Descemet‟s membrane are
detectable in 99.3% of patients with Wilson‟s disease who have
neurologic abnormalities.
A chronic smoker presents with cough, dyspnea and swelling in the face
and neck. You suspect superior vena cava obstruction. What is your next
step?

a) Chest x-ray
b) Pulmonary function tests
c) V/Q Scan
d) Arteriogram
The correct answer is A

Explanation
Superior vena cava syndrome (SVCS) is a collection of symptoms caused by
the partial blockage or compression of the superior vena cava, the major
vein that carries blood from the head, neck, upper chest, and arms to
the heart. Nearly 95% of SVCS cases are caused by cancer.

The most common symptoms are these: problems breathing, coughing and
swelling in the face, neck, upper body, and arms.

Approximately 90% of cases are associated with a cancerous tumor that is


compressing the superior vena cava. A chest x-ray usually shows a
mediastinal mass (tumor) that is compressing the superior vena cava.
Which one of the following forms of glucocorticoid therapy is most
likely to result in hypothalamic-pituitary-adrenal axis suppression?

a) Split-dose oral therapy with a bedtime dose


b) Morning single-dose oral therapy
c) Alternate-day oral therapy
d) Topical therapy
e) Intra-articular injection therapy
The correct answer is A

Explanation
Late evening doses of glucocorticoid are more likely to suppress the
normal early morning ACTH surge. They should be avoided because of the
increased risk of suppressed adrenal secretion.

One-daily morning dosing is less likely to result in adrenal


suppression. If short-acting drugs such as hydrocortisone are used, the
last dose of the day should not be taken after 5 pm. Alternate-day
therapy is less likely to suppress the hypothalamic-pituitary-adrenal
axis. It is frequently less therapeutically effective. Topical and
intra-articular therapy rarely result in adrenal suppression.

The correct initial dosage and frequency for levodopa in the treatment
of Parkinson disease is

a) One 100mg tablet once a day


b) One 100mg tablet twice a day
c) One 100mg tablet three times a day
d) One 100mg tablet four times a day
The correct answer is C

Explanation
Sinemet (carbidopa/levadopa) dosage is best initiated with one
25mg/100mg tablet three times a day. This dosage schedule provides 75 mg
of carbidopa and 300 mg of levadopa per day.

Levodopa is the metabolic precursor of dopamine, crosses the blood-brain


barrier into the basal ganglia, where it is decarboxylated to form
dopamine. Coadministration of the peripheral decarboxylase inhibitor
carbidopa prevents levodopa catabolism, thus lowering the levodopa
dosage requirements and minimizing adverse effects.

Levodopa is most effective at relieving bradykinesia and rigidity,


although tremor is often substantially reduced. Mildly affected patients
who take levodopa may return to nearly normal, and bedbound patients may
become ambulatory.
A 35 year old black female has just returned home from a vacation in
Hawaii. She presents to your office with a swollen left lower extremity.
She has no previous history of similar problems. Homan‟s sign is
positive, and ultrasonography reveals a noncompressible vein in the left
popliteal fossa extending distally. Which one of the following is true
in this situation?

a) Monotherapy with an initial 10-mg loading dose of warfarin


(Coumadin) would be appropriate
b) Enoxaparin (Lovenox) should be administered at a dosage of 1 mg/k
subcutaneously twice a day
c) The incidence of thrombocytopenia is the same with
low-molecular-weight heparin as with unfractioned heparin
d) The dosage of warfarin should be adjusted to maintain the INR at 2.5-3.5
e) Anticoagulant therapy should be started as soon as possible and
maintained for 1 year to prevent deep vein thrombosis (DVT) recurrence
The correct answer is B

Explanation
The use of low-molecular-weight heparin allows patients with acute deep
vein thrombosis (DVT) to be managed as outpatients. The dosage is 1
mg/kg subcutaneously twice daily. Patient chosen for outpatient care
should have good cardiopulmonary reserve, normal renal function, and no
risk for excessive bleeding. Oral anticoagulation with warfarin can be
initiated on the first day of treatment after heparin loading is completed.

Monotherapy with warfarin is inappropriate. The incidence of


thrombocytopenia with low-molecular-weight heparin is lower than with
conventional heparin. The INR should be maintained at 2.0-3.0 in this
patient. The 2.5-3.5 range is used for patients with mechanical heart
valves. The therapeutic INR should be maintained for 3-6 month in a
patient with a first DVT related to travel.
A 24 year old man is diagnosed with gonorrhea. He is treated with
ceftriaxone. He returns a few days later and states that he still has
symptoms of dysuria and discharge. His urine microscopy shows no
organisms. What is the likely cause of his symptoms‟

a) Chlamydia
b) Herpes
c) Latent gonorrhea
d) Syphylis
The correct answer is A

Explanation
It is a standard guideline to always treat a patient with Gonorrhea for
both Gonorrhea and Chlamydia. Chlamydia is the most prevalent STD. It is
most common among people in their late teens and early twenties and can
coexist with gonorrhea.
Treatment options include for Gonorrhea: Ceftriaxone 125 mg IM single
dose, for Chlamydia: Doxycycline 100 mg Bid for 7 days or Azithromycin 1
gram orally as a single dose. In patient‟s that are pregnant, avoid
doxycycline. The patient should also be advised to have his sexual
partner treated.
Patients with acquired immunodeficiency syndrome (AIDS) can have
chorioretinitis with blindness, enteritis with intractable diarrhea,
interstitial pneumonitis, and adrenalitis, all caused by infection with

a) Cryptosporidium
b) Herpes zoster
c) Toxoplasma
d) Pneumocystis
e) Cytomegalovirus
The correct answer is E

Explanation
Anyone can become infected with CMV. Almost all people have been exposed
to CMV by the time they are adults, but the virus usually does not make
otherwise healthy people sick. However, some people are at increased
risk for active infection and serious complications: Babies born to
women who have a first-time CMV infection during pregnancy. Pregnant
women who work with infants and children. Persons with weakened immune
systems, including cancer patients on chemotherapy, organ transplant
recipients, and persons with HIV infection

Active infection in otherwise healthy children and adults can cause


prolonged high fever, chills, severe tiredness, a generally ill feeling,
headache, and an enlarged spleen.

Most infected newborns have no symptoms at birth, but, in some cases,


symptoms will appear over the next several years. These include mental
and developmental problems and vision or hearing problems. In rare
cases, a newborn can have a life-threatening infection at birth. Infants
and children who get CMV infection after birth have few, if any,
symptoms or complications. When symptoms do appear, they include lung
problems, poor weight gain, swollen glands, rash, liver problems, and
blood problems.

People with weakened immune systems can have more serious, potentially
life-threatening illnesses, with fever, pneumonia, liver infection, and
anemia. Illnesses can last for weeks or months and can be fatal. In
persons with HIV infection, CMV can infect the retina of the eye (CMV
retinitis) and cause blindness.
In a patient with vertigo, what sign or symptom would not lead to a
diagnosis of viral labyrinthitis‟

a) Otorrhea
b) Nystagmus
c) Hearing loss
d) Tinnitus
The correct answer is A

Explanation
Labyrinthitis is a balance disorder. It is an inflammatory process
affecting the labyrinths that house the vestibular system (which sense
changes in head position) of the inner ear.

In addition to balance control problems, a labyrinthitis patient may


encounter hearing loss and tinnitus. Labyrinthitis is caused by a virus,
but it can also arise from bacterial infection, head injury, an allergy
or as a reaction to a particular medicine. Both bacterial and viral
labyrinthitis can cause permanent hearing loss, although this is rare.
Labyrinthitis often follows an upper respiratory tract infection (URI).

When working properly, the vestibular system also relays information on


head movement to the eye muscle, forming the vestibulo-ocular reflex, in
order to retain continuous visual focus during motion. When the
vestibular system is affected by labyrinthitis, rapid, undesired eye
motion (nystagmus), often results from the improper indictations of
rotational motion.
Which of the following has extra human host?

a) B. fragilis
b) S. epidermidis
c) Spirochetes
d) Tapeworm
The correct answer is D

Explanation
All tapeworms (cestodes) cycle through 3 stage: eggs, larvae, and
adults. Adults inhabit the intestines of definitive hosts, mammalian
carnivores. Several of the adult tapeworms that infect humans are named
after their intermediate host: the fish tapeworm (Diphyllobothrium
latum), the beef tapeworm (Taenia saginata), and the pork tapeworm
(Taenia solium).

The other answer choices are found in humans. Bacteroides fragilis in


the large intestine. Staphylococcus epidermidis on the skin and
Spirochetes in the mouth.
An otherwise healthy 72 year old white male presents with pain on the
right side of his head, increasing in severity over the past 2 days.
Today he broke out in a rash consisting of grouped vesicles on an
erythematous base in the distribution of the first division of the fifth
cranial nerve. The right eyelid is involved, but the patient complains
of no pain in the eye itself, or of any visual disturbance. Appropriate
management at this time would include the administration of which one of
the following?

a) Intravenous acyclovir (Zovirax)


b) Oral famciclovir (Famvir)
c) Topical capsaicin (Zostrix)
d) Varicella-zoster immune globulin (VZIG)
e) Idoxuridine (Herplex) eye drops
The correct answer is B

Explanation
Herpes zoster, or shingles, is a common condition which results from the
reactivation of varicella virus acquired during an earlier episode of
chickenpox. The distribution of the characteristic rash typically
follows a single dermatome and does not cross the midline. The lesions
are typically painful, and postherpetic neuralgia can become a disabling
chronic problem. When any branch of the ophthalmic nerve is involved,
the condition is called herpes zoster ophthalmicus. Vesicles on the side
or tip of the nose (Hutchison‟s sign) that occur during an episode of
zoster suggest involvement of the nasociliary branch, and are associated
with the most serious ocular complications. Involvement of the other
sensory branches of the trigeminal nerve may affect the eyelid but
rarely involves the eye itself.

Treatment of uncomplicated herpes zoster ophthalmicus in an


immunocompetent patient includes oral acyclovir, famciclovir, or
valacyclovir. Corticosteroids can be used for acute pain but have no
effect on the development of postherpetic neuralgia.

Intravenous antiviral therapy is indicated for immunosuppressed patients


with extensive cutaneous disease, and those at high risk for ocular
complications. Treatment is most effective when started within 48 hours
of the onset of the disease.

Capsaicin cream is used in the treatment of postherpetic neuralgia, and


varicella zoster immunoglobulin (VZIG) is used in high-risk
immunocompromised patients to prevent varicella infection. Idoxuridine
is indicated for herpes simplex keratitis.
A 59 year old female with a history of hypertension, posttraumatic
stress disorder, and sarcoidosis, reports a 3-month history of hair loss
in the right temple area. She has been using an over-the-counter
corticosteroid cream without response. Her other medications include
ramipril (Altace), hydrochlorothiazide, clonazepam (Klonopin), and
bupropion (Wellbutrin). She reports increased stress from caring for her
chronically ill husband who has been hospitalized recently.

Examination of the skin and scalp is normal except for a 3x4-cm


irregularly shaped area of alopecia with a slightly red, raised anterior
edge located in the right temporal scalp. The affected skin is shiny,
with no scaling; no hairs or broken hairs are present.

The most likely cause of this patient‟s hair loss is

a) Drug-induced alopecia
b) Androgenic alopecia
c) Stress-induced alopecia areata
d) Sarcoidosis
e) Trichotillomania
The correct answer is D

Explanation
The condition of the skin in the affected area helps to differentiate
the cause of hair loss. In this case, the abnormal skin suggests a
scarring type of alopecia, which may be caused by infection (e.g.,
syphilis, tuberculosis, acquired immunodeficiency syndrome, herpes
zoster), autoimmune disease (discoid lupus erythematosus), sarcoidosis,
scalp trauma (e.g., injuries, burns), and radiation therapy. The skin is
normal in androgenic alopecia, alopecia areata, and drug-induced
alopecia. Trichotillomania, a compulsive plucking of one‟s hair, may
occur in the scalp. This can be differentiated from other forms of
scarring alopecia by an examination of the scalp showing decreased hair
density, as well as broken hairs of various lengths, rather than
complete baldness in the affected area. As the patient has known
sarcoidosis, this is the most likely cause of her scarring alopecia.

An 83-year-old male presents with type 2 diabetes mellitus, diabetic


nephropathy, hypertension, and congestive heart failure related to
ischemic cardiomyopathy. His ejection fraction is 25%, and his
creatinine clearance is 25 mL/min. You plan to add an ACE inihibitor to
his regimen, but are concerned about the development of hyperkalemia.

Which one of the following drugs would decrease his risk of hyperkalemia
if used concomitantly with an ACE inhibitor?

a) Furosemide (Lasix)
b) Naproxen (Naprosyn)
c) Celecoxib (Celebrex)
d) Spironolactone (Aldactone)
e) Propranolol (Inderal)
The correct answer is A

Explanation
The geriatric population is at increased risk of hyperkalemia associated
with the use of ACE inhibitors, especially in the presence of renal
disease. Drugs that increased this risk, if used concomitantly with an
ACE inhibitor, including ?-blockers, NSAIDs (including selective COX-2
inhibitors), and potassium-sparing diuretics. Other diuretics, such as
furosemide, enhance excretion of potassium by increasing delivery of
sodium to the collecting duct.
A 50 year old man with a history of hemochromatosis presents to the
emergency room vomiting up bright red blood. He had his most recent
phlebotomy yesterday. His blood pressure is 110/85 mm Hg, his pulse
115/min; his face is flushed, and he is diaphoretic. During the physical
examination splenomegaly and a venous pattern on his chest and abdomen
are noted. He seems somewhat drowsy and confused but has no focal
neurologic signs. What is the probable source of this patient's confusion?

a) Severe anemia
b) Hepatic encephalopathy
c) Subarachnoid hemorrhage
d) Vitamin B12 deficiency
e) Serum iron overload
The correct answer is B

Explanation
Hemochromatosis is the abnormal accumulation of iron in parenchymal
organs, leading to organ toxicity. It is the most common inherited liver
disease in whites and the most common autosomal recessive genetic
disorder. Hemochromatosis can cause liver damage and lead to hepatic
encephalopathy

Hepatic encephalopathy is a neuropsychiatric syndrome. It most often


results from high gut protein or acute metabolic stress (eg, GI
bleeding, infection, electrolyte abnormality) in a patient with
portal-systemic shunting. Symptoms are mainly neuropsychiatric (eg,
confusion, flapping tremor, coma). Diagnosis is based on clinical
findings. Treatment usually is correction of the acute cause,
restriction of dietary protein, and administration of oral lactulose.
A 35 year old male patient presents to your office complaining of
flushing episodes.

These episodes occur several times per day, without any apparent trigger
and last for a few minutes. During these episodes, his skin becomes
extremely red with burning sensation. The patient states that over the
past few weeks, he had watery diarrhea, few times a day, associated with
severe abdominal colic. The past medical and surgical history is
otherwise normal.

On physical examination, the patient appears comfortable. Vital signs:


Temp of 37C, BP is 130/85 mmHg, pulse is 80/min and respiratory rate is
14/min. Occasional wheezes are heard on lung auscultation. The abdomen
is soft and non tender with hyperactive bowel sounds.

Which of the following is the best initial test to order for this patient?

a) 24-hour urine 5-HIAA (5-hydroxyindolacetic acid)


b) 24-hour urine cortisol
c) 24-hour urine vanillymandelic acid and catecholamines levels
d) Blood serotonin level
e) Insulin, glucose and C-peptide serum level
The correct answer is A

Explanation
Carcinoid syndrome is the most likely diagnosis of the patient‟s
condition. Carcinoid syndrome is characterized by: flushing episodes,
diarrhea, wheezing and right sided cardiac valvular lesions caused by
the released hormones, especially serotonin. Any patient presenting with
an unexplained flushing and diarrhea should be tested for urine 5-HIAA
(5-hydroxyinolacetic acid). This is a very sensitive and specific test.

24-hour urine vanillymandelic acid and catecholamines levels would be


appropriate if pheochromocytoma is suspected.

Insulin, glucose and C-peptide serum level would be appropriate if


insulinoma is suspected.

Although it is highly specific, blood serotonin level is not a sensitive


as the blood level fluctuates constantly; thus, should not be ordered
initially.

24-hour urine cortisol would be appropriate if Cushing‟s syndrome is


suspected.

Which one of the following sleep disorders is in the general class of


circadian sleep disorders and may respond to bright-light therapy?

a) Shift-work insomnia
b) Alcohol-dependent sleep disorder
c) Inadequate sleep hygiene
d) Sleep-related myoclonus
The correct answer is A

Explanation
Shift-work insomnia is the only circadian sleep disorder listed. It may
respond to bright-light therapy. Alcoholism is a behavioral disorder
that may respond to gradual discontinuance. Inadequate sleep hygiene
(use of stimulants at night, sleeping other than at bedtime, etc.) may
respond to habit changes. Sleep-related myoclonus is an intrinsic sleep
disorder and can be treated with levodopa or clonazepam.

A 63 year old male patient is loaded with digitalis - 0.5 mg every 8


hours for three doses, followed by 0.20 mg per day. After several days
he started complaining about visual changes and gastrointestinal
disturbances. Today, cardiac arrhythmia is present. All of the following
should be included in management of this patient, *except*

a) Check serum digitalis and potassium levels


b) EKG
c) Administration of antiarrhythmic agents
d) Administration of digitalis antibodies
e) Correct electrolyte status
The correct answer is C

Explanation
In spite of its limited benefits and recognized hazards, digitalis is
still heavily used and toxicity is common. Therapy for toxicity
manifested as visual changes or gastrointestinal disturbances generally
requires no more than reducing the dose of the drug. If cardiac
arrhythmia is present and can be ascribed to digitalis, more vigorous
therapy may be necessary. Serum digitalis and potassium levels and the
electrocardiogram should always be monitored during therapy of
significant digitalis toxicity. Electrolyte status should be corrected
if abnormal.

In severe digitalis intoxication, serum potassium will already be


elevated at the time of diagnosis (because of potassium loss from the
intracellular compartment of skeletal muscle and other tissues).
Furthermore, automaticity is usually depressed, and antiarrhythmic
agents administered in this setting *may lead to cardiac arrest.* Such
patients are best treated with prompt insertion of a temporary cardiac
pacemaker catheter and administration of digitalis antibodies (digoxin
immune fab). These antibodies recognize digitoxin and cardiac glycosides
from many other plants in addition to digoxin. They are extremely useful
in reversing severe intoxication with most glycosides.

Digitalis-induced arrhythmias are frequently made worse by


cardioversion; this therapy should be reserved for ventricular
fibrillation if the arrhythmia is glycoside-induced.
A 47 year old male presents with a history of profound watery diarrhea
for 2 weeks. During this period, his weight has dropped from 157 lb to
148 lb. A Clostridium difficile assay is positive.

Which one of the following would be most appropriate in the management


of this patient?

a) Amoxicillin
b) Vancomycin (Vancocin)
c) Metronidazole (Flagyl)
d) Bacitracin methylene disalicylate
e) Saccharomyces cerevisiae (brewer‟s yeast)
The correct answer is C

Explanation
For Clostridium difficile-associated diarrhea, oral metronidazole and
oral vancomycin are equally effective treatments. As oral vancomycin is
significantly more expensive and may select for colonization with
vancomycin-resistant enterococci, oral metronidazole is recommended as
preferred therapy. In persons with recurrent infection, the addition of
the probiotic agent Saccharomyces cerevisiae reduces the risk of future
recurrences. Oral bacitracin methylene disalicylate was extensively
studied some 20 years ago, and while it was found to be as effective as
vancomycin in reducing clinical symptoms, it was less effective in
eradicating the organism or toxin from the stool and resulted in more
frequent relapses. Since C. difficile colonization is thought to be due
to the disruption of normal bowel flora by antibiotics, including
amoxicillin, this medication would be an inappropriate choice.
A 72-year-old patient has an acute onset of joint pain and swelling.
Aspiration reveals monosodium urate crystals. Compared with younger
patients, geriatric patients presenting for the first time with this
condition are more likely to

a) be male
b) have a monoarticular presentation
c) have involvement of joints of the upper extremity
d) have podagra (first metatarsophalangeal joint involvement)
e) be free of tophi
The correct answer is C

Explanation
The presentation of gout in the elderly is often different from the
classic presentation. It occurs more often in women, has a polyarticular
onset, and often involves small joints of the hands. Older patients also
tend to develop tophi early. Gout may appear in joints with Heberden‟s
nodes from osteoarthritis. Podagra is the classic presentation of gout
in the younger population.
A patient presents with a case of drug overdose. He has fever,
tachycardia, dilated pupils a blood pressure of 220/110. The most likely
cause is

a) Cocaine
b) LSD
c) Heroin
d) Barbiturates
The correct answer is A

Explanation
Because cocaine is a very short-acting drug, heavy users may inject it
or smoke it q 10 to 15 min. This repetition produces toxic effects, such
as tachycardia, hypertension, mydriasis, muscle twitching,
sleeplessness, and extreme nervousness.
A 34-year-old black female presents to you for preconception counselling
regarding the management of her chronic hypertension. Her blood pressure
has been well controlled on benazepril (Lotensin), 20 mg/day, without
any side effects. The patient‟s blood pressure was 145/95 mm hg prior to
beginning benazepril. She has been pregnant once before, and her
physician switched her to methyldopa (Aldomet) during that pregnancy,
but she suffered from drowsiness and a dry mouth during much of that
time. The pregnancy and delivery were otherwise uncomplicated. She has
no history of diabetes mellitus, renal insufficiency, or asthma. She is
a non-smoker.

Which one of the following would you do when she becomes pregnant?

a) Discontinue the benazepril and monitor closely throughout the


pregnancy for signs of preeclampsia or fetal growth restriction
b) Continue the benazepril through the pregnancy and delivery
c) Switch to atenolol (Tenormin) until after delivery
d) Switch to long-acting nifedipine (Procardia) until after delivery
The correct answer is A

Explanation
Most women with mild, uncomplicated essential hypertension are at
minimal risk for cardiac complications within the short time frame of
pregnancy. There is no evidence available that treatment of mild
essential hypertension during pregnancy provides any benefit to the
mother. Given the potential for short- and long-term risk to the fetus
from antihypertensive treatment, it is advisable to discontinue
antihypertensive treatment, monitor the mother for signs of
preeclampsia, and monitor fetal growth and development.

Medication is not necessary as long as the systolic blood pressure


remains below 160 mm Hg, the diastolic blood pressure remains below
105-110 mm Hg, and there are no signs of preeclampsia or fetal growth
restriction. Should the mother develop severe hypertension, treatment
can be initiated with long-acting nifedipine, labetalol, a thiazide
diuretic, or methylodopa. Atenolol has been associated with reduced
fetal growth, and ACE inhibitors are contraindicated in the second and
third trimesters.

A female patient presents with an acute onset of severe constant anal


pain that has started about 2 days ago. She states that It gets worse
during defecation, walking, and sitting. Physical examination and
anoscopy show a tender, blue swelling below the dentate line. What is
the most appropriate initial management?

a) Hot bandages
b) Sitz baths
c) Immediate excision
d) Systemic Antibiotics
e) Topical Antibiotics
The correct answer is C

Explanation
This patient has a thrombosed external hemorrhoid that requires
immediate incision and evacuation of the clot to provide symptomatic
relief. Pressure by compression is usually all that is needed to control
the bleeding. The typical presentation of a thrombosed external
hemorrhoid is an acute onset of very severe perianal pain, particularly
when walking and sitting. You know that it is an external hemorrhoid
because it is below the dentate line.
Acutely thrombosed external hemorrhoids may be safely excised in the
emergency department in patients who present within 48-72 hours of
symptom onset. In patients presenting after 72 hours from the start of
symptoms, conservative medical therapy is preferable.

A 30-year-old white male complains of several weeks of nasal stuffiness,


purulent nasal discharge, and facial pain. He does not respond to a
3-day course of trimethoprim/ sulfamethoxazole (Bactrim, Septra).
Follow-up treatment with 2 weeks of amoxicillin/clavulanate (Augmentin)
is similarly ineffective. Of the following diagnostic options, which one
is most appropriate at this time?

a) Pulmonary function testing


b) Coronal CT of the sinuses
c) Culture and sensitivity testing of the discharge
d) Erythrocyte sedimentation rate
The correct answer is B

Explanation
This patient has a clinical presentation consistent with acute
sinusitis. Failure to respond to adequate antibiotic therapy suggests
either a complication, progression to chronic sinusitis, or a different,
confounding diagnosis. The diagnostic procedure of choice in this
situation is coronal CT of the sinuses, due to its increased sensitivity
and competitive cost when compared with standard radiographs. Cultures
of the nasal discharge give unreliable results because of bacterial
contamination from the resident flora of the nose. The other options
listed do not contribute to the diagnosis and treatment of sinusitis.
A 35 year old woman with a history of multiple sclerosis comes to the
office because she had a sudden loss of vision in the right eye. She has
no history of diplopia. On examination, external ocular movements are
normal but funduscopic examination shows pallor of the optic disk. This
patient's condition is most likely due to demyelination of which of the
following?

a) Medial longitudinal fasciculus


b) Oculomotor nerve
c) Optic nerve
d) Trigeminal nerve
e) Visual cortex
The correct answer is C

Explanation
This patient has multiple sclerosis. This is a demyelinating disorder of
the central nervous system. The important information about localization
of this lesion comes from the history and physical examination. The
patient reports loss of vision and exam reveals normal extra-ocular
movements with pallor of the optic disk. This is consistent with loss of
cranial nerve two (the optic nerve) which conveys sensory information
from the retina to the brain.
While being investigated for long-standing hypertension, a 55 year old
woman is found to have the following serum laboratory test values:
normal creatinine, total protein, albumin, and globulin; increased
calcium and alkaline phosphatase, and decreased phosphorus. Which of the
following do these findings suggest the presence of?
a) Carcinoma metastatic to bone
b) Excessive dietary calcium intake
c) Multiple myeloma
d) Parathyroid adenoma
e) Sarcoidosis
The correct answer is D

Explanation
Primary hyperparathyroidism is usually the result of a single benign
adenoma. It is a cause of secondary hypertension.

Elevated parathyroid hormone (PTH) levels in the setting of


hypercalcemia establish the diagnosis of hyperparathyroidism. A
decreased serum phosphate level may be seen. Increased bone turnover may
be reflected in elevated levels of markers of bone formation (alkaline
phosphatase) and bone resorption (urinary pyridinoline).
A 40 year old woman develops a skin rash when she takes
trimethoprim/sulfamethoxazole for a urinary tract infection. Which one
of the following drugs would be most likely to cause a similar reaction
in this patient?

a) Celecoxib
b) Ranitidine
c) Spironolactone
d) Metformin
e) Propranolol
The correct answer is A

Explanation
Sulfonamides, commonly called sulfa drugs, contain SO2NH2, which is
present in numerous therapeutic agents, including the new Cox-2
inhibitors such as celecoxib.

Celecoxib is contra-indicated in patients with allergy to


sulfa-containing drugs (sulfonamides). Such drugs include the antibiotic
trimethoprim-sulfamethoxazole (Bactrim, Septra). Therefore, if you are
allergic to sulfa-containing drugs, do not take celecoxib.
A 50-year-old married male comes to your clinic complaining of right
testicular pain that developed quickly over the last 24 hours. He
reports no fever, dysuria, or urethral discharge. He is nauseous but has
no emesis or diarrhea. His abdominal examination is benign but he has an
enlarged, exquisitely tender right hemi-scrotum. A urinalysis is normal
and his WBC count is 13,000/mm3 (N 4300?10,800). Testicular
ultrasonography shows increased blood flow in the affected testicle and
epididymis.

Which one of the following would be the best treatment for this
patient‟s condition?

a) Ceftriaxone (Rocephin), 250 mg intramuscularly once, plus doxycycline, 100 mg orally


twice daily for 10 days, and treatment of all sexual partners
b) Clindamycin (Cleocin), 300 mg orally 4 times daily for 7 days
c) Famciclovir (Famvir), 250 mg orally 3 times daily for 7 days
d) Ofloxacin (Floxin), 300 mg orally twice daily for 10?14 days
e) Urgent urologic surgery consultation
The correct answer is D
Explanation
This patient has epididymo-orchitis. This condition is most often caused
by coliform bacteria in men over the age of 35 (and in homosexual
males). These organisms are effectively treated with ofloxacin. Those
less than age 35 with this condition have a higher chance of having a
sexually transmitted disease (STD) and would benefit from intramuscular
ceftriaxone and oral doxycycline. However, these medications would not
adequately cover coliform bacteria. Clindamycin and famciclovir would
also be ineffective. Testicular torsion is another possible cause of
testicular pain with a quick onset, and would require urgent urologic
surgery consultation. However, in cases of torsion the examination
usually reveals an elevated testicle and testicular ultrasonography
would show decreased or absent blood flow on the affected side.
An otherwise healthy 62-year-old male has been hospitalized with
community-acquired pneumonia for 2 days. He has remained bedridden. When
you see him while making rounds, he mentions that he has noticed
increased swelling and pain in his left lower extremity. Lower extremity
ultrasonography reveals a deep venous thrombosis (DVT) in his calf. He
has no previous history of blood clots.

Which one of the following is the best management of this patient‟s DVT?

a) Compression stockings and repeat ultrasonography in 2 days


b) Heparin therapy followed by oral anticoagulation for 3 months
c) Indefinite oral anticoagulation
d) Intravenous thrombolytic therapy
e) Insertion of an inferior vena cava filter
The correct answer is B

Explanation
Without anticoagulation, patients with an uncomplicated calf deep vein
thrombosis (DVT) have a significant risk of further clot extension,
acute pulmonary embolus, or recurrence of the thrombus. They are also at
risk for late complications such as the post-thrombotic syndrome or
chronic thromboembolic hypertension. For this reason, monitoring a DVT
by repeat ultrasonography is not a good option unless there are
contraindications to anticoagulation, such as recent surgery,
hemorrhagic stroke, active bleeding, or heparin-induced thrombocytopenia.

The recommended treatment for DVT is heparin (intravenous unfractionated


or subcutaneous low molecular weight) followed by oral anticoagulation
with warfarin once adequate anticoagulation is achieved. For a first
episode of DVT due to an idiopathic cause or transient risk factor, such
as short-term immobilization, the recommended length of treatment is 3?6
months. The benefit-to-risk ratio declines after 6 months unless the
patient has a recurrent DVT or a known chronic risk factor, such as a
thrombophilic condition or cancer.

Thrombolytic therapy with intravenous tissue plasminogen activator


(tPA), urokinase, or streptokinase typically is reserved for patients
with life-threatening pulmonary embolism. Inferior vena cava filters are
used in patients who have a contraindication to anticoagulant therapy,
recurrent venous thromboembolism despite adequate anticoagulation, or
such limited pulmonary vascular reserve that they may not survive
additional thromboemboli.
A 62 year old male is brought to the Emergency Department by family
members after he experienced a seizure. They report his recent
complaints of headache and weakness of his left hand. The most likely
diagnosis is

a) Migraine
b) Temporal arteritis
c) Brain neoplasm
d) Cluster headache
e) Idiopathic epilepsy
The correct answer is C

Explanation
In a brain neoplasm the cumulative effects of tumor invasion, peritumor
edema, and hydrocephalus may elevate the intracranial pressure (ICP) and
impair cerebral perfusion. The symptoms of brain tumors are numerous and
not specific to brain tumors, meaning they can be caused by many other
illnesses as well.

The symptoms are caused by the tumor pressing on or encroaching on other


parts of the brain and keeping them from functioning normally. Some
symptoms are caused by swelling in the brain caused by the tumor or
surrounding inflammation. The following symptoms are most common:
headache, weakness, clumsiness, difficulty walking and seizures.

Other nonspecific symptoms and signs include the following: altered


mental status: changes in concentration, memory, attention, or
alertness, nausea, vomiting: especially early in the morning,
abnormalities in vision, difficulty with speech, gradual changes in
intellectual or emotional capacity.
For 2 weeks, a 62 year old male with biopsy-documented cirrhosis and
ascites has had diffuse abdominal discomfort, fever, and night sweats.
His current medications are furosemide (Lasix) and spironolactone
(Aldactone). On examination, his temperature is 38.0°C (100.4°F), blood
pressure 100/60 mm Hg, heart rate 92 beats/min and regular. The heart
and lung examination is normal. The abdomen is soft with vague
tenderness in all quadrants. There is no rebound or guarding. The
presence of ascites is easily verified. Bowel sounds are quiet. The
rectal examination is normal, and the stool is negative for occult
blood. You perform diagnostic paracentesis and send a sample of fluid
for analysis. Which one of the following findings would best establish
the suspected diagnosis of spontaneous bacterial peritonitis‟

a) pH <7.2
b) Bloody appearance
c) Neutrophil count >300/mL
d) Positive cytology
e) Total protein > 1 g/DL
The correct answer is C

Explanation
Diagnostic paracentesis is recommended for patients with ascites of
recent onset, as well as for those with chronic ascites who present with
new clinical findings such as fever or abdominal pain. A neutrophil
count > 250/mL is diagnostic for peritonitis. Once peritonitis is
diagnosed, antibiotic therapy should be started immediately without
waiting for culture results. Bloody ascites with abnormal cytology may
be seen with hepatoma, but is not typical of peritonitis. The ascitic
fluid pH does not become abnormal until well after the neutrophil count
has risen, so it is a less reliable finding for treatment purposes. A
protein level > 1 g/dL is actually evidence against spontaneous
bacterial peritonitis.
A high-school football player reports a history of progressive inability
to participate over a month‟s time. Initially he had double vision
following a practice. He was better the following morning, but his
symptoms recurred during the next day‟s practice. Now he has double
vision most of the time, except right after sleeping. He says his
strength and agility are declining, and his coach thinks he is playing
too poorly to remain on the team.

On examination he has mild ptosis, which increases if he tries to


maintain an upward gaze. His arm strength is initially 5/5, but he tires
very rapidly. After a few minutes of isotonic exertion he cannot lift
his arms against gravity. His deep tendon reflexes are normal.

What is the most likely diagnosis‟

a) Guillain-Barré syndrome
b) Psychophysiologic weakness
c) Second impact syndrome
d) Myasthenia gravis
e) Myotonic dystrophy
The correct answer is D

Explanation
Myasthenia gravis is a disease of the motor end plate, most often
affecting the cranial nerves, especially the oculomotor nerves and the
eyelids. The disease begins with weakness and fatigue, but can progress
over days to months. Confirmatory testing consists of acetylcholine
receptor antibody testing or electromyography when the situation is not
urgent. When the diagnosis must be made promptly, the old-fashioned
Tensilon test gives immediate and dramatic results.

Deep tendon reflexes are lost in Guillain-Barré syndrome, which is an


autoimmune polyneuropathy affecting the longest nerves preferentially.
Myasthenia seems so odd that it can be mistaken for hysterical or
stress-related symptoms. Myotonic dystrophy becomes progressively
symptomatic in its victims, and is not always diagnosed before
adulthood, but it does not cause double vision, and affected individuals
would not be athletes at any point in their lives. They have general
weakness and a typical facial appearance. Second impact syndrome is a
potentially lethal response to repeated head injury.
A 35-year-old female sees you because she has lost her voice. She has
had no recent upper respiratory infection symptoms, cough, or heartburn,
and she has not done anything that would strain her voice. Examination
of the head and neck appear normal. A review of her chart shows this has
happened before, but an ear, nose, and throat evaluation found no
abnormalities. She also has been seen numerous times in the past few
years for headaches, chest pains, abdominal pains, rectal pressure, and
vaginal symptoms. Despite several workups and referrals, no definite
cause has been found and the symptoms persist. Which one of the
following would be the most reasonable plan of action?

a) Test for food allergies


b) Begin low-dose lorazepam (Ativan)
c) Begin a 6-week trial of a proton pump inhibitor
d) Schedule frequent office visits
The correct answer is D

Explanation
Somatization disorders should be considered in patients who have a
history of various complaints over a several-year period that involve
multiple organ systems. There is no test to confirm this diagnosis. It
is often intertwined in other psychiatric problems, including anxiety
disorder, personality disorder, and depression. Treatment includes
testing to make sure that there is nothing physically wrong, while
building a trusting relationship with the patient. Once this is
accomplished, it is reasonable to discuss the disorder with the patient.
Cognitive therapy has been shown to be of value, as well as regularly
scheduled office visits for monitoring and support. Medicines for
coexisting psychiatric problems also are of benefit. In addition,
referral for psychiatric consultation may be worthwhile.

Food allergies can cause a variety of symptoms, but usually not to the
extent seen with this patient, and testing for this might confuse the
issue. Lorazepam may help the symptoms if there is a coexisting anxiety
disorder, but it will not address the underlying problem. Laryngeal
esophageal reflux can cause hoarseness and will respond to proton pump
inhibitors, but given the repetitive nature of her symptoms and the
previous negative workups, it is not consistent with the whole picture.

A post menopausal woman comes to you to discuss hormone replacement


therapy (HRT). She has a history of migraines and a strong family
history of osteoporosis. What would be an appropriate choice?

a) Offer combination oral estrogen-progesterone HRT


b) Offer progesterone only
c) Offer biphosphonate only
d) Offer oral estrogen only
The correct answer is C

Explanation
Oral forms of estrogen and progesterone can cause several side effects,
one of which is migraines. For this patient the only medicine
appropriate is a bisphosphonate like Actonel or Fosamax to prevent her
from developing osteoporosis.

A 62-year-old white male has a history of coronary artery disease,


hypertension, and osteoarthritis. He is on lovastatin (Mevacor) for
hypercholesterolemia. You are choosing a medication to treat his
hypertension.

Which one of the following could increase this patient‟s risk of


rhabdomyolysis‟

a) Carvedilol (Coreg)
b) Quinapril (Accupril)
c) Losartan (Cozaar)
d) Atenolol (Tenormin)
e) Verapamil (Calan, Isoptin)
The correct answer is E

Explanation
Lovastatin, as well as other statin cholesterol-lowering medications, is
primarily metabolized through the cytochrome P-450 system. Medications
that inhibit the enzyme CYP 3A4, such as verapamil, increase serum
concentrations of selected statins and subsequently increase the risk of
rhabdomyolysis.
A 30 year old woman complains of episodic faintness, severe headaches,
tingling sensation in her hands, shortness of breath, sweating, tremor,
occasional palpitations, and severe anxiety. Thorough medical workup
reveals no pathologic condition. During an episode of these symptoms,
laboratory test results would probably reveal which one of the following?

a) Decreased hematocrit
b) Increased urea (BUN)
c) Decreased protein
d) Increased catecholamines in urine
e) Normal serum glucose level
The correct answer is D

Explanation
A pheochromocytoma is a catecholamine-secreting tumor of chromaffin
cells typically located in the adrenals. It causes persistent or
paroxysmal hypertension.

The classic history of a patient with a pheochromocytoma includes spells


characterized by headaches, palpitations, and diaphoresis in association
with severe hypertension. These 4 characteristics together are strongly
suggestive of a pheochromocytoma.

Characteristic Laboratory Test Findings:

1) increased catecholamines in urine (vanillymandelic acid and


metanephrine)
2) increased erythropoietin/hematocrit
3) increased sed rate
4) increased IL-6
5) increased amylase
6) hypoglycemia or hyperglycemia

Common symptoms and signs are paroxysms of tachycardia, diaphoresis,


postural hypotension, tachypnea, cold and clammy skin, severe headache,
angina, palpitations, nausea, vomiting, epigastric pain, visual
disturbances, dyspnea, paresthesias, constipation, and a sense of
impending doom.

Diagnosis is by measuring catecholamine products in blood or urine.


Imaging tests, especially CT or MRI, help localize tumors. Treatment
involves removal of the tumor when possible. Drug therapy for control of
BP includes alpha-blockade, possibly combined with beta-blockade.
Which one of the following types of hemorrhoids is particularly suitable
for treatment by elastic-band ligation?

a) External hemorrhoids with pruritus ani


b) External hemorrhoids with ulceration
c) External hemorrhoids with thrombosis
d) Internal hemorrhoids with bleeding
e) Internal hemorrhoids with fecal soiling
The correct answer is D

Explanation
Hemorrhoids are dilated veins of the hemorrhoidal plexus in the lower
rectum. Symptoms include irritation and bleeding. Thrombosed hemorrhoids
are painful. Diagnosis is by inspection or anoscopy.

Treatment is symptomatic or with endoscopic banding, injection


sclerotherapy, or sometimes surgery. Elastic/rubber band ligation is an
outpatient treatment for second-degree (those that bleed with
defecation) internal hemorrhoids.
A 35-year-old female presents with pain in her neck, back, and shoulder,
as well as a complaint of poor sleep and fatigue. For the last 6 months
she has experienced burning in her low back that radiates to her
buttocks, and she feels that her joints are swollen even though there is
no objective evidence of this. She also experiences morning stiffness
that improves as the day progresses. A physical examination is
unremarkable with the exception of palpation of 14 different tender
points above and below the diaphragm, including the upper border of the
trapezius muscle, the second rib space 3 cm lateral to the sternal
border bilaterally, both lateral epicondyles, the upper outer quadrants
of both gluteal muscles, and the radial fat pad of both knees.
Laboratory tests, including a CBC, a chemistry profile, thyroid function
tests, and an erythrocyte sedimentation rate, all are within normal limits.

Which one of the following would be the most helpful treatment for this
patient?

a) Massage therapy
b) NSAIDs
c) Trigger-point injections
d) Corticosteroids
e) Cognitive-behavior therapy
The correct answer is E

Explanation
The etiology of fibromyalgia remains unknown, but it is a common
condition that is underdiagnosed. It is much more common in women than
in men; additional risk factors include being divorced, having a low
income level, and not completing high school. The pathogenesis may be
related to central sensitization and dysregulation of the hypothalamic
(pituitary) adrenal axis.

Commonly associated symptoms include headache (often migraine in type),


anxiety, depression, and dizziness. The diagnosis is specific, and
requires the finding of at least 11 tender points in 18 possible
specific anatomic locations, with the pain occurring both above and
below the waist on both sides of the body for at least 3 months.

There is strong evidence for the effectiveness of both pharmacologic


therapies such as cyclobenzaprine and nonpharmacologic therapies such as
aerobic exercise, cognitive-behavioral therapy, and multidisciplinary
approaches that include patient education and exercise. The evidence of
effectiveness is weak for chiropractic therapy, electrotherapy, massage
therapy, and ultrasound. There is no known benefit from corticosteroids,
opioids, thyroid hormone, NSAIDs, melatonin, flexibility exercises, or
trigger-point injections.

Which one of the following is true regarding the use of oral vitamin B12
to treat vitamin B12 deficiencies‟

a) A daily maintenance dose >500 ?g is effective


b) It is not effective for pernicious anemia
c) It is not effective after total gastrectomy
d) It is not effective after terminal ileum resection
e) It should be used only if the stage I Schilling test is normal
The correct answer is A

Explanation
There is strong evidence for the existence of an alternative pathway to
the usual intrinsic factor/terminal ileum pathway for vitamin B12
absorption. Studies have shown that oral vitamin B12 at daily doses of
1000 microgram is effective for the treatment of vitamin B12 deficiency even in
the absence of intrinsic factor (pernicious anemia), an acidic
environment (gastrectomy), or terminal ileum resection (resection or
damage from Crohn‟s disease). This high oral dose is required because
doses of 500 microgram or less are only variably absorbed via the alternative
pathway. The Schilling test is no longer needed for determining the
route of therapy for vitamin B12 deficiency, as high-dose oral therapy
is effective even if the stage I Schilling test is abnormal.
Mr. Smith, a 55 year old male, has been your patient for 10 years. He is
a heavy smoker and has been diagnosed with COPD 6 years ago. He has been
admitted to the hospital multiple times for COPD exacerbations. Which of
the following interventions would you like to discuss with Mr. Smith
because of the evidence based fact that it prolongs survival in COPD
patients‟

a) Albutrol inhaler
b) Antibiotic prophylaxis
c) Home oxygen
d) Ipratropium inhaler
e) Steroids
The correct answer is C

Explanation
After quitting smoking, home oxygen therapy is the only modality known
to prolong survival in COPD. Home oxygen therapy is recommended when
PaO2 < 55 mm Hg or oxygen saturation < 88%. Hypoxia is the major cause
of mortality in COPD patients.

Survival issues are becoming hot topics for the licensing exams.

Bronchodilators control the symptoms of COPD patients but have not been
shown to improve survival.

Antibiotics and steroids are recommended for acute exacerbations. They


have no role for chronic COPD management.
A 68-year-old previously healthy male presents to the emergency
department with fever and rigors, chills, cough, fatigue, dyspnea, and
pleuritic chest pain. He has no chronic medical problems and does not
smoke. A chest film shows a lobar consolidation. The most likely
bacterial cause of this problem is
a) Staphylococcus aureus
b) Streptococcus pneumoniae
c) Legionella
d) Mycoplasma pneumoniae
e) Chlamydia
The correct answer is B

Explanation
This is a classic presentation for community-acquired pneumonia (CAP).
The classic symptom presentation includes cough, fever, pleuritic chest
pain, myalgias, dyspnea, malaise, and fatigue. Typical pneumonia is
usually caused by Streptococcus pneumoniae, with 60%?70% of typical
cases being due to this organism. Atypical pneumonia is usually caused
by Mycoplasma, Chlamydia, Legionella, adenovirus, or influenza virus.
Age is the best differentiating factor between typical and atypical
pneumonia, as older persons and the very young are more likely to have
typical etiologies and young adults are more likely to have atypical
etiologies.
A 68 year old man with a history of urolithiasis, suffered from a
seizure attack which involved loss of consciousness with tonic and
clonic muscular contractions. His tongue fell back into his throat and
he choked. He is treated with valproic acid. What are the most common
side effects caused by this medication?

a) Tremors
b) Acne
c) Dyspepsia and weight gain
d) Encephalopathy
e) Thrombocytopenia
The correct answer is C

Explanation
Common side effects of valproic acid are dyspepsia and/or weight gain.
Less common are fatigue, peripheral edema, acne, dizziness, drowsiness,
hair loss, headaches, nausea, sedation and tremors.
Valproic acid also causes hyperammonemia, which can lead to brain
damage. Valproate levels within the normal range are capable of causing
hyperammonemia and ensuing encephalopathy.
Rarely, valproic acid can cause blood dyscrasia, impaired liver
function, jaundice, thrombocytopenia, and prolonged coagulation times.
A patient is on carbamazepine for his new onset seizures. He has now had
3 attacks in last 2 weeks. His serum level of carbamazepine is normal.
What is the next step?

a) Obtain a CT scan and EEG


b) Give another agent (eg. Phenytoin)
c) Re-check carbamazepine level in 2 weeks
d) Check a CBC, CMP and TSH/FT4
The correct answer is B

Explanation
This patient has already been diagnosed with a seizure disorder. He has
been placed on carbamazepine and is currently at a therapeutic level.
But his seizures are not controlled. At this time another medication
should be added. Choices include phenytoin and valproic acid.
A repeat CT and EEG is not needed at this time. His carbamazepine level
is therapeutic yet he still has seizures so obtaining a repeat level is
not correct. Although lab tests may be necessary, it is not the best
choice at this time.
A 76-year-old white female comes to your office because of a skin rash
on her right upper back which has been present for the last 36 hours.
Examination reveals a vesicular eruption which is consistent with herpes
zoster.

Which one of the following would be appropriate?

a) Symptomatic treatment only


b) Oral acyclovir (Zovirax)
c) Acyclovir ointment applied 5 times a day
d) Amantadine (Symmetrel)
e) Prednisone
The correct answer is B

Explanation
Oral acyclovir in high doses (800 mg 5 times daily) is of some benefit
in treating cutaneous varicella zoster infections such as shingles. It
must be started within the first 48 hours, and some studies suggest a
beneficial effect of postherpetic neuralgia. Topical acyclovir does not
offer any significant benefit in the treatment of mucocutaneous herpes
infections. Amantadine is effective against influenza, but not against
herpesvirus. Prednisone has not been proven to decrease postherpetic
neuralgias.
A 40-year-old white male has elevated transaminase levels at a routine
visit, but your clinical evaluation shows no apparent cause. At a
follow-up visit 6 months later they remain elevated. He remains
asymptomatic and has a normal physical examination. You order further
tests, including serologic blood analyses.

Which one of the following imaging studies would be most appropriate?

a) Abdominal ultrasonography with Doppler


b) CT with contrast
c) MRI of the abdomen
d) Magnetic resonance angiography (MRA) of the abdomen
e) Positron emission tomography (PET) of the abdomen
The correct answer is A

Explanation
Ultrasonography with Doppler should be the first imaging study ordered
in the evaluation of suspected liver disease. It is widely available,
relatively inexpensive, involves no radiation or contrast exposure, and
provides helpful information regarding the appearance of the liver and
blood flow in the portal and hepatic veins. If hepatic nodules are found
on ultrasonography, CT is indicated to help differentiate between benign
and malignant lesions. MRI is best used as a follow-up study to
determine whether lesions have changed in appearance or size. MRA can
detect portal hypertension, portal flow volume and direction, and portal
vein thrombosis. PET scans can detect the presence of liver metastases
from certain cancers, but are not used as an initial test for evaluating
liver disease.
In a patient with recent onset of alcohol amnestic disorder, the mental
status test most likely to be done poorly is memory of

a) Political figures of 10 years ago


b) Digits (forward)
c) His own name
d) Digits (backward)
e) Three items for 5 minutes
The correct answer is E

Explanation
Korsakoff's psychosis (Korsakoff's/alcoholic amnestic syndrome) occurs
in 80% of untreated patients with Wernicke's encephalopathy. A severe or
repeated attack of postalcoholic delirium tremens can trigger
Korsakoff's psychosis whether or not a typical attack of Wernicke's
encephalopathy has occurred first.

Immediate memory is severely affected; retrograde and anterograde


amnesia occurs in varying degrees. Patients tend to draw on memory of
remote events, which appears to be less affected than memory of recent
events. Disorientation to time is common. Emotional changes are common;
they include apathy, blandness, or mild euphoria with little or no
response to events, even frightening ones. Spontaneity and initiative
may be decreased.

Confabulation is often a striking early feature; bewildered patients


unconsciously fabricate imaginary or confused accounts of events they
cannot recall; these fabrications may be so convincing that the
underlying disorder is not detected.
Which one of the following would be a major contraindication to using
bupropion (Wellbutrin) for smoking cessation?

a) A history of seizures
b) Bipolar depression
c) Hemolytic anemia
d) Mitral valve prolapse
e) Hypothyroidism
The correct answer is A

Explanation
Medications to help patients with tobacco cessation are not without
dangers. Bupropion has a good record of success in assisting patients to
stop smoking. However, major contraindications to its use include a
history of seizures, an eating disorder, or recent MAO inhibitor use.
The other conditions listed should always be considered, but are not
usually a major contraindication to bupropion use.
An 18 year old girl presents with thirst and polyuria in the order of
4-5 L per 24 hours. The urinalysis (urine microscopy) reveals no glucose
or albumin and a normal sediment. The urine specific gravity is 1.001,
and the serum sodium 137 mmol/L. After 8 hours with no fluid intake, the
urine specific gravity is 1.024 and the serum sodium 141 mmol/L. Which
one of the following is the most likely diagnosis‟

a) Primary polydipsia
b) Chronic renal failure
c) Neurogenic (central) diabetes insipidus
d) Nephrogenic diabetes insipidus
e) Syndrome of inappropriate antidiuretic hormone (ADH) secretion
The correct answer is A

Explanation
Primary polydipsia, also termed psychogenic diabetes insipidus, or
compulsive water drinking. It is a condition in which the subject
experiences a powerful need to ingest fluid and presents with polyuria
and polydipsia and a plasma osmolarity lower than the normal, perhaps as
low as or lower than 270 mOsm/L. The synthesis, secretion, nephron
sensitivity to ADH and the osmoreceptor response are all intact and
operative in these subjects.

The hallmark of this disease, is plasma hypo-osmolarity in the presence


of polyuria and the ability to form a maximally concentrated urine when
deprived of water or when ADH is administered. As noted above in central
or nephrogenic diabetes insipidus the polyuria precedes and is the cause
of the polydipsia. In the case of psychogenic diabetes insipidus the
polydipsia precedes and is the cause of the polyuria.
Which one of the following is the best choice for rapidly restoring
normal coagulation in a patient on warfarin (Coumadin) with serious
gastrointestinal bleeding?

a) Tissue plasminogen activator (tPA)


b) Methylprednisolone (Solu-Medrol)
c) Fresh frozen plasma
d) Vitamin K orally
e) Packed red blood cells
The correct answer is C

Explanation
With serious bleeding, immediate reversal of the anticoagulant effect of
warfarin is necessary. The only effective strategy is to give fresh
frozen plasma intravenously, restoring the vitamin K-dependent factors
VII, IX, and X which are depleted in patients taking warfarin. Oral
vitamin K is an effective method of reversing the effects of warfarin,
but takes 24 hours to reach full effect.

Packed red blood cells contain no clotting factors, and corticosteroids


have no effect on the synthesis of vitamin K-dependent factors. Tissue
plasminogen activator is a thrombolytic agent, and would make any
serious bleeding condition worse.
A 52 year old newspaper assistant editor, is seen in the office of his
family physician complaining of blood in the urine over the past four
days. He states that he has had this on two previous occasions, both in
relation to an upper respiratory tract infection. On both previous
occasions the urine cleared spontaneously over a period of five to seven
days.

At the time of the present visit he states that he has been feeling
lethargic with a sore throat for the past five days. He is a vegetarian
and smokes a pack of cigarettes each day. Blood pressure is 170/95 mmHg
in both arms, sitting and supine, on repeat testing. Urine analysis
shows specific gravity 1020, nitrites negative, + blood and + protein.
Urine microscopy reveals oxalate crystals, dysmorphic red blood cells
and red cell casts.
Which of the following secondary causes of hypertension does this
patient most likely have?

a) Coarctation of the aorta


b) Primary hyperaldosteronism
c) Renal artery stenosis, bilateral
d) Renal parenchymal disease
The correct answer is D

Explanation
About half of the people with acute glomerulonephritis have no symptoms.
If symptoms do occur, the first to appear are tissue swelling (edema)
due to fluid retention, low urine volume, and production of urine that
is dark because it contains blood. Edema may first appear as puffiness
of the face and eyelids but later is prominent in the legs. Blood
pressure increases as kidney function becomes impaired. In turn, high
blood pressure and swelling of the brain may produce headaches, visual
disturbances, and more serious disturbances of brain function (for
example, seizures or coma). In older people, nonspecific symptoms, such
as nausea and a general feeling of illness (malaise), are more common.
A 19 year old white male with a history of fever, fatigue, weight loss,
and mild diarrhea of 2 months‟ duration is found to have a palpable mass
in the right lower quadrant of the abdomen. The most likely diagnosis is

a) Crohn‟s disease (regional enteritis)


b) Ulcerative colitis
c) Amebic colitis
d) Diverticulitis
e) Lymphoma
The correct answer is A

Explanation
When Crohn‟s disease affects primarily the distal small intestine
(regional enteritis), a most characteristic clinical pattern emerges. A
young person, usually in the second or third decade, will present with a
period of episodic abdominal pain, largely postprandial and often
periumbilical, occasionally with low-grade fever and mild diarrhea.

Anorexia, nausea, and vomiting may also be present. Weight loss is


frequent. Some patients may be aware of tenderness in the right lower
quadrant and even of a palpable mass in that region.
In acute pyelonephritis, which of the following is most commonly
associated with bacteremic spread from a *distant focus*?

a) Escherichia coli
b) Proteus
c) Staphylococcus aureus
d) Serratia
e) Enterococcus
The correct answer is C

Explanation
Pyelonephritis is more common in women than in men. Escherichia coli, a
type of bacteria normally in the large intestine, causes about 90% of
cases of pyelonephritis among people who are not hospitalized or living
in a nursing home. Infections usually ascend from the genital area
through the urethra to the bladder, up the ureters, into the kidneys.
Infections can also be carried to the kidneys from another part of the
body through the bloodstream. For instance, a staphylococcal skin
infection can spread to the kidneys through the bloodstream.

In a person with a healthy urinary tract, an infection is usually


prevented from moving up the ureters into the kidneys by the flow of
urine washing organisms out and by closure of the ureters at their
entrance to the bladder. However, any physical obstruction to the flow
of urine, such as a structural abnormality, kidney stone, or an enlarged
prostate, or the backflow (reflux) of urine from the bladder into the
ureters increases the likelihood of pyelonephritis.
A patient presents after getting accidentally electrocuted at his job.
What is the least appropriate management?

a) Surgery
b) CPR
c) EKG
d) Sulfadiazine
e) IV fluids
The correct answer is A

Explanation
Electrical injury may result from contact with faulty electrical
appliances or machinery or inadvertent contact with household wiring or
electrical power lines. The most common entry point for electricity is
the hand; the second most common is the head.

Often, the main symptom of an electrical injury is a skin burn, although


not all electrical injuries cause external damage. High-voltage injuries
may cause massive internal burns.

A minor shock may cause muscle pain and may trigger mild muscle
contractions or startle a person, causing a fall. Severe shocks can
cause abnormal heart rhythms, ranging from inconsequential to
immediately fatal.

Once the person can be safely touched, the rescuer should check to see
if the person is breathing and has a pulse. If the person is not
breathing and has no pulse, cardiopulmonary resuscitation (CPR) should
be started immediately. People with rhabdomyolysis receive large amounts
of fluids containing sodium bicarbonate, which is given intravenously.

Skin burns are treated with burn cream (such as silver sulfadiazine or
bacitracin) and sterile dressings. If the injury is more severe, the
person is admitted to the hospital, ideally a burn center. An
electrocardiogram (ECG) is performed to monitor the heartbeat and to
look for heart damage.

A 65 year old male with back pain, nephrotic syndrome and anemia present
to the ER. Ultrasound shows normal kidney size. His creatinine is 500.
Which diagnosis best fits the scenario?

a) Polycystic kidney disease


b) Chronic glomerulonephritis
c) Multiple myeloma
d) Diabetic nephropathy
e) Analgesic abuse
The correct answer is C

Explanation
Multiple myeloma is a malignancy of plasma cells that produce monoclonal
immunoglobulin and invade and destroy adjacent bone tissue. Common
manifestations include bone pain, renal insufficiency, hypercalcemia,
anemia, and recurrent infections.

Persistent bone pain (especially in the back or thorax), renal failure,


and recurring bacterial infections are the most common presentations.
Pathologic fractures are common, and vertebral collapse may lead to
spinal cord compression and paraplegia. Symptoms of anemia predominate
or may be the sole reason for evaluation in some patients, and a few
have manifestations of hyperviscosity syndrome.

Diagnosis requires demonstration of M-protein (sometimes present in


urine and not serum) and either lytic bone lesions, light-chain
proteinuria, or excessive marrow plasma cells. A bone marrow biopsy is
usually needed.
A 35-year-old white female complains of unilateral frontotemporal
headaches. During these episodes, which occur every 2?3 weeks, she
becomes nauseated, sometimes to the point of vomiting. The headaches are
throbbing in character and last for 1?3 hours, often causing her to
leave work early. Relief is sometimes obtained with simple analgesics,
but more often with sleep or the passage of time. On the basis of this
history alone, the most likely diagnosis is

a) sinusitis
b) a brain tumor
c) muscle tension headache
d) cluster headache
e) migraine headache
The correct answer is E

Explanation
Migraine is the most likely diagnosis in this scenario, because the
patient is young and female; the headaches are unilateral, infrequent,
and throbbing; the headaches are associated with nausea and vomiting;
and sleep offers relief. Symptoms of sinusitis usually include fever,
facial pain, and a purulent nasal discharge. The pain of cerebral tumor
tends to occur daily and becomes more frequent and severe with time.
Furthermore, the prevalence of brain tumor is far less than that of
migraine. The pain of muscle tension headache is described as a pressure
or band-like tightening, often in a circumferential or cap distribution.
This headache also has a pattern of daily persistence, often continuing
day and night for long periods of time. Cluster headache is more common
in males, and presents as a constant, agonizing orbital pain, usually
beginning within 2 or 3 hours after falling asleep.
A 73-year-old Hispanic male presents to the emergency department with a
3-day history of abdominal and right flank pain. He is lethargic and
pale, and his skin is clammy. His blood pressure is 86/30 mm Hg, pulse
106 beats/min, and temperature 38.6° C (101.5° F). His chest is clear
and no murmurs are heard. He responds to painful stimuli. The abdomen is
soft with no guarding or rebound.
Immediate fluid resuscitation is begun and after an hour he has received
2 L of normal saline. In that hour he has had a urine output of only 30
cc. A chest film and an EKG are normal.
*
**Laboratory Findings*

WBCs. . . . . . . . . . . . . . . . . . 15,500/mm^3 (N 4300-10,800)


Platelets. . . . . . . . . . . . . . . . . 70,000/mm^3 (N 150,000-300,000)
Base deficit. . . . . . . . . . . . . . . 13 mEq/L
Serum pH. . . . . . . . . . . . . . . . 7.21 (N 7.35-7.45)
Urinalysis. . . . . . . . . . . . . . . . packed WBCs, 3+ bacteria

Which one of the following is most likely to enhance survival in this


patient?

a) Low-dose dopamine
b) Recombinant human activated protein C (Xigris)
c) Antithrombin
d) Bicarbonate
e) Erythropoietin
The correct answer is B

Explanation
This clinical scenario should lead one to think of septic shock. Recent
comprehensive investigations and reviews have demonstrated that the use
of recombinant activated protein C in patients with severe sepsis and a
high risk for death does improve survival. Low-dose dopamine should not
be used for maintenance or improvement of renal function. Antithrombin
administration is not recommended for the treatment of severe sepsis and
septic shock. As a specific treatment for anemia associated with severe
sepsis, erythropoietin has not been shown to be of benefit. There is no
evidence to support the use of bicarbonate in the treatment of
hypoperfusion-induced acidemia associated with sepsis.
A 78-year-old male comes to your office with a 3-day history of pain in
the right side of his chest. The pain is described as burning and
intense. Two days ago he noted a rash at that site. Examination reveals
groups of vesicles on an erythematous base in a T-5 dermatome
distribution on the right.

Which one of the following would be the most appropriate treatment to


minimize the chance of post-herpetic neuralgia?

a) Famiciclovir (Famvir)
b) Prednisone
c) Capsaicin (Zostrix)
d) Carbamazepine (Tegretol)
The correct answer is A

Explanation
The key indicator of postherpetic neuralgia is persistent pain 3 ? 6
months after an episode of herpes zoster. Studies show that patients who
present for treatment of herpes zoster within 72 hours will benefit from
antiviral therapy such as famciclovir to reduce the pain and decrease
the risk of postherpetic neuralgia. Treating zoster pain with tricyclic
antidepressants in low dosage (10 - 25 mg amitriptyline) may also
decrease risk. While steroids added to antiviral therapy may be of
benefit in short-term therapy, they do not reduce pain at 6 months.
A slender 22-year-old female is concerned about a recent weight loss of
10 lb, frequent mild abdominal pain, and significant diarrhea of 2
months‟ duration. Her physical examination is unremarkable, and
laboratory studies reveal only a moderate microcytic, hypochromic anemia.

Based on this presentation, which one of the following is the most


likely diagnosis‟

a) Irritable bowel syndrome


b) Villous adenoma
c) Infectious colitis
d) Celiac disease
e) Ulcerative colitis
The correct answer is D

Explanation
This constellation of symptoms strongly suggests celiac disease, a
surprisingly common disease with a prevalence of 1:13. Half the adults
with celiac disease or gluten-sensitive enteropathy present with anemia
or osteoporosis, without gastrointestinal symptoms. Individuals with
more significant mucosal involvement present with watery diarrhea,
weight loss, and vitamin and mineral deficiencies.
A 42-year-old female presents with a 2-day history of chest pain. She
describes the pain as sharp, located in the right upper chest, and
worsened by deep breathing or coughing. She also complains of shortness
of breath. She was previously healthy and has no recent history of
travel. Her vital signs are normal. A pleural friction rub is noted on
auscultation of the lungs. The remainder of the examination is normal.
An EKG, cardiac enzymes, oxygen saturation, and a D-dimer level are all
normal.

Which one of the following would be most appropriate at this point?

a) No further testing
b) A chest radiograph
c) An antinuclear antibody test
d) Echocardiography
e) Pulmonary angiography
The correct answer is B

Explanation
This patient has pleurisy. Patients presenting with pleuritic chest pain
may have life-threatening disorders, and pulmonary embolism, acute
myocardial infarction, and pneumothorax should be excluded. While 5%?20%
of patients with pulmonary embolism present with pleuritic chest pain,
this patient has no risks for pulmonary embolism and the normal D-dimer
level obviates the need for further evaluation. Moderate- to high-risk
patients may need a helical CT scan or other diagnostic testing.

An EKG and chest radiograph are recommended in the evaluation of


acute/subacute pleuritic chest pain. The chest radiograph will exclude
pneumothorax, pleural effusion, or pneumonia. An echocardiogram would
not be indicated if the cardiac examination and EKG are normal. An
antinuclear antibody level could be considered in recurrent pleurisy or
if other symptoms or signs of lupus were present, but it would not be
indicated in this patient.

Most cases of acute pleurisy are viral and should be treated with NSAIDs
unless the workup indicates another problem.

Which one of the following is the most likely cause of hypercalcemia in


a patient with hypercalcemia and hyperphosphatemia?

a) Vitamin D excess
b) Primary hyperparathyroidism
c) Humoral hypercalcemia of malignancy
d) Exogenous calcitonin administration
The correct answer is A

Explanation
The action of parathyroid hormone (PTH) causes phosphate loss through
the kidneys; consequently, hypercalcemia due to PTH excess is usually
associated with hypophosphatemia. Humoral hypercalcemia of malignancy is
associated either with osteolysis, in which case phosphate levels are
unaffected, or with elevated levels of a PTH-related peptide, which
would also be associated with hypophosphatemia. Calcitonin is a weak
inhibitor of PTH action and is not associated with hypercalcemia.
Vitamin D increases the gastrointestinal absorption of both calcium and
phosphorus; excess levels are therefore associated with elevated levels
of both calcium and phosphorus. This helps to distinguish vitamin D
excess from other causes of hypercalcemia.

A 73-year-old female began vomiting yesterday after undergoing


bronchoscopy. This morning she is still vomiting. Her husband brings her
to the emergency department because she is obviously uncomfortable and
can?t keep any fluid down. On examination she is delirious.

Laboratory Findings

Sodium----------------------------------140 mmol/L (N 135-145)

Potassium----------------------------------5.7 mmol/L (N 3.5-4.5)

BUN----------------------------------65 mg/dL (N 12-20)


Serum creatinine----------------------------------3.2 mg/dL (N 0.6-1.1)
Serum glucose----------------------------------650 mg/dL

The initial management of this patient should include which one of the
following?

a) Insulin infusion
b) Oral hypoglycemic agents and training in home glucose testing technique
c) Sodium polystyrene sulfonate (Kayexalate)
d) Insulin glargine (Lantus) and insulin lispro (Humalog)
The correct answer is A
Explanation
The patient is in a hyperosmolar, hyperglycemic state. She should be
treated with intravenous fluids and an insulin infusion in an
intensive-care setting, with hourly follow-up of blood glucose levels.
Although she has renal insufficiency and hyperkalemia, the hyperkalemia
will resolve with insulin and fluid infusion. In fact, potassium
supplementation will be needed within a few hours. The delirium will
usually clear with correction of the metabolic abnormalities. Oral
diabetic agents or a subcutaneous insulin protocol will be needed for
long-term control, but are not aggressive enough to manage this
patient‟s acute problem.
A 60-year-old black female has a history of hypertension that has been
well controlled with hydrochlorothiazide. However, she has developed an
allergy to the medication. Successful monotherapy for her hypertension
would be most likely with which one of the following?

a) Lisinopril (Prinivil, Zestril)


b) Hydralazine (Apresoline)
c) Clonidine (Catapres)
d) Atenolol (Tenormin)
e) Diltiazem (Cardizem)
The correct answer is E

Explanation
Monotherapy for hypertension in black patients is more likely to consist
of diuretics or calcium channel blockers than -blockers or ACE
inhibitors. It has been suggested that hypertension in blacks is not as
angiotensin II?dependent as it appears to be in Caucasians.
During cardiac resuscitation performed on an adult by two people, the
ratio of cardiac compressions to respirations recommended is

a) 5:1
b) 3:2
c) 10:1
d) 15:2
The correct answer is D

Explanation
While previous versions of the adult Basic Life Support guidelines
recommend a ratio of 15 compressions to 2 ventilations for one-rescuer
CPR and a ratio of 5 compressions to 1 ventilation for two-rescuer CPR,
current evidence suggests that coronary perfusion pressure is higher
after 15 uninterrupted chest compressions than it is after 5 chest
compressions. Thus the 15:2 ratio is now recommended for one or two
rescuers, and applies to adult Basic Life Support provided by both
laypersons and health care professionals.
Which of the following is associated with streptococcal pharyngitis‟

a) Cough
b) Cervical lymphadenopathy
c) Afebrile patient
d) Group B strep
The correct answer is B

Explanation
Streptococci are gram-positive aerobic organisms that cause many
disorders, including pharyngitis, pneumonia, wound and skin infections,
sepsis, and endocarditis. Symptoms vary with the organ infected.
Sequelae include rheumatic fever and glomerulonephritis. Clinical
diagnoses are confirmed by Gram stain and culture.

Strep throat (Streptococcal pharyngitis) is a form of Group A


streptococcal infection that affects the pharynx. Symptoms include:
severe sore throat, yellow and white patches in the throat, difficulty
swallowing, tender cervical lymphadenopathy, red and enlarged tonsils,
halitosis, fever of 38°C (101F) or greater, rash and absence of cough.

A 74 year old white male complains of pain in the right calf that recurs
on a regular basis. He smokes 1 pack of cigarettes per day and is
hypertensive. He has a history of previous heart attack but is otherwise
in fine health. Which one of the following findings would support a
diagnostic impression of peripheral vascular disease?

a) Pain during rest and exercise and the presence of swelling and
soreness behind the knee and in the calf
b) Pain that begins immediately upon walking and is unrelieved by rest
c) Doppler waveform analysis showing accentuated waveforms at a point
of decreased blood flow
d) Treadmill arterial flow studies showing a 20-mm Hg decrease in ankle
systolic blood pressure immediately following exercise
e) An ankle-brachial index of 1.15
The correct answer is D

Explanation
Peripheral vascular disease (PVD) is a clinical manifestation of
artherosclerotic disease and is caused by occlusion of the arteries to
the legs. Patients with significant arterial occlusive disease will have
a prominent decrease in the ankle-brachial index from baseline following
exercise, and usually a 20-mm Hg or greater decrease in systolic blood
pressure.

Pain during rest and exercise and the presence of swelling and soreness
behind the knee and in the calf is found in those with Baker‟s cysts.
Peripheral nerve pain commonly begins immediately upon walking and is
unrelieved by rest.
Doppler waveform analysis is useful in the diagnosis of PVD and will
reveal attenuated waveform at a point of decreased blood flow.
Employment of the ankle-brachial index is encouraged in daily practice
as a simple means to diagnose the presence of PVD. Generally,
ankle-brachial indices in the range of 0.91-1.30 are thought to be normal.

An 48 year old woman presents with thirst and polyuria in the order of
4-5 L per 24 hours. The urinalysis (urine microscopy) reveals no glucose
or albumin and a normal sediment. The urine specific gravity is 1.001,
and the serum sodium 137 mmol/L. After 8 hours of no fluid intake, the
urine specific gravity is 1.001 and the serum sodium 149 mmol/L. Which
one of the following is the most likely diagnosis‟

a) Primary polydipsia
b) Chronic renal failure
c) Neurogenic (central) diabetes insipidus
d) Syndrome of inappropriate antidiuretic hormone (ADH) secretion
The correct answer is C

Explanation
Diabetes insipidus (DI) results from a deficiency of ADH due to a
hypothalamic-pituitary disorder (central DI) or from resistance of the
kidney to ADH (nephrogenic DI). Polyuria and polydipsia develop.
Diagnosis is by water deprivation test showing failure to maximally
concentrate urine, ADH levels and response to exogenous ADH help
distinguish central DI from nephrogenic DI. Treatment is with intranasal
desmopressin or lypressin. Nonhormonal treatment includes use of
diuretics (mainly thiazides) and ADH-releasing drugs, such as
chlorpropamide.
A 60 year old man presents with a small lesion on his nose. On
examination of the lesion you note the following: the lesion is 3 mm in
diameter, slightly elevated with a waxy appearance, and has a slight
vascular pattern on top. This lesion is most characteristic of which one
of the following?

a) Basal cell carcinoma


b) Squamous cell carcinoma
c) Melanoma
d) Varicella-zoster lesion
e) Actinic keratosis
The correct answer is A

Explanation
Basal cell carcinoma is a superficial slowly growing papule or nodule
that derives from epidermal basal cells.

Most commonly, the carcinoma begins as a shiny papule, enlarges slowly,


and after a few months or years, shows a shiny, pearly border with
prominent engorged vessels (telangiectases) on the surface and a central
dell or ulcer.

Metastasis is rare, but local growth can be highly destructive.


Diagnosis is by biopsy. Treatment depends on the tumor's characteristics
and may involve curettage and electrodesiccation, surgical excision,
cryosurgery, or, occasionally, radiation therapy.
For several years, a hypertensive 65-year-old female has been treated
with hydrochlorothiazide, 25 mg/day; atenolol (Tenormin), 100 mg/day;
and hydralazine (Apresoline), 50 mg 4 times/day. Her blood pressure has
been well controlled on this regimen. Over the past 2 months she has
experienced malaise, along with diffuse joint pains that involve
symmetric sites in the fingers, hands, elbows, and knees. A pleural
friction rub is noted on examination. Laboratory testing shows that the
patient has mild anemia and leucopenia, with a negative rheumatoid
factor and a positive antinuclear antibody (ANA) titer of 1:640. Which
one of the following would be the most appropriate initial treatment?

a) Discontinue the thiazide diuretic and switch to a loop diuretic such as furosemide
(Lasix)
b) Discontinue the hydralazine
c) Begin treatment with prednisone, 40 mg/day orally
d) Treat with hydroxychloroquine (Plaquenil), 400 mg/day
e) Obtain renal function studies and anticipate that a renal biopsy will be needed
The correct answer is B

Explanation
There are many drugs that can induce a syndrome resembling systemic
lupus erythematosus, but the most common offender is procainamide,
followed by hydralazine. There is a genetic predisposition for this
drug-induced lupus, determined by drug acetylation rates. Polyarthritis
and pleuropericarditis occur in half of patients, but fortunately, CNS
and renal involvement are rare. While all patients with this condition
have positive ANAs and most have antibiodies to histones, antibodies to
double-stranded DNA and decreased complement levels are rare, which
distinguishes drug-induced lupus from idiopathic lupus.

The best initial management is to withdraw the drug, and most patients
improve in a few weeks. For those with severe symptoms, a short course
of corticosteroids is indicated. Once the offending drug is
discontinued, symptoms seldom last beyond 6 months.
A 68-year-old previously healthy male presents to the emergency
department with fever and rigors, chills, cough, fatigue, dyspnea, and
pleuritic chest pain. He has no chronic medical problems and does not
smoke. A chest film shows a lobar consolidation. The most likely
bacterial cause of this problem is

a) Staphylococcus aureus
b) Streptococcus pneumoniae
c) Legionella
d) Mycoplasma pneumoniae
e) Chlamydia
The correct answer is B

Explanation
This is a classic presentation for community-acquired pneumonia (CAP).
The classic symptom presentation includes cough, fever, pleuritic chest
pain, myalgias, dyspnea, malaise, and fatigue. Typical pneumonia is
usually caused by Streptococcus pneumoniae, with 60%-70% of typical
cases being due to this organism. Atypical pneumonia is usually caused
by Mycoplasma, Chlamydia, Legionella, adenovirus, or influenza virus.
Age is the best differentiating factor between typical and atypical
pneumonia, as older persons and the very young are more likely to have
typical etiologies and young adults are more likely to have atypical
etiologies.
Which one of the following is characteristic of osteoarthritis of the knee?

a) Greater frequency in men than in women


b) Increased pain with rest
c) A direct correlation between radiographic changes and pain severity
d) Reduction of pain with repair of associated meniscal tears
e) Reduction of pain with muscle strengthening
The correct answer is E

Explanation
Osteoarthritis of the knee is more common in women than in men. Rest
improves the pain of osteoarthritis, and increasing muscle strength
improves joint stability and reduces pain. Meniscal tears are extremely
common in advanced osteoarthritis, but repairing them fails to improve
the course of the disease. Radiographic changes correlate poorly with
pain severity in osteoarthritis.
A teenage girl was raped and she was given a prophylactic drug to
prevent gonorrhea and incubating syphilis. What is the most appropriate
treatment of choice?

a) Ceftriaxone
b) Doxycycline
c) Metronidazole
d) Azithromycin
The correct answer is A

Explanation
Treatment for suspected gonorrhea is with Ceftriazone, which can be
given 250mg IM. Chlamydia is usually treated with either doxycycline or
azithromycin. Metronidazole is a treatment for bacterial vaginosis and
trichomonis. Syphilis is treated with penicillin usually.

A 38 year old woman presents complaining of epigastric pain that is


exacerbated by eating. An EGD reveals the presence of a duodenal ulcer.
A test for Helicobacter pylori is positive for the H. pylori. Which of
the following is the most appropriate treatment?

a) Metronidazole
b) Lansoprazole
c) Clarithromycin and Amoxicillin
d) Metronidazole and amoxicillin
e) Lansoprazole and clarithromycin and amoxicillin
The correct answer is E

Explanation
Helicobacter pylori causes duodenal and gastric ulcers. The organism is
present in 95% to 100% of patients with duodenal ulcers and in 75% to
85% of those with gastric ulcers. Eradicating the organism generally
results in a cure for the disease.

H. pylori eradication requires multi-drug therapy, typically antibiotics


plus acid suppressants. Proton pump inhibitors suppress H. pylori, and
the increased gastric pH accompanying their use can enhance tissue
concentration and efficacy of antimicrobials, creating a hostile
environment for H. pylori.

Triple therapy is recommended. Oral omeprazole

20 mg bid or lansoprazole

30 mg bid, plus clarithromycin

500 mg bid, plus amoxicillin

1 g bid (or, for penicillin-allergic patients, metronidazole


500 mg bid) for 14 days, cures infection in > 95% of cases. This regimen
has excellent tolerability. Ranitidine

bismuth citrate 400 mg po bid may be substituted for the proton pump
inhibitor.
The appropriate initial management of deep venous thrombosis is to

a) Give heparin
b) Give warfarin
c) IVC filter
d) Pneumatic/compression stockings
The correct answer is A

Explanation
Deep venous thrombosis (DVT) is clotting of blood in a deep vein of an
extremity (usually calf or thigh) or the pelvis. DVT is the primary
cause of pulmonary embolism (PE). DVT results from conditions that
impair venous return, lead to endothelial injury or dysfunction, or
cause hypercoagulability.

DVT may be asymptomatic or cause pain and swelling in an extremity.


Diagnosis is by history, physical examination, and duplex
ultrasonography, with d-dimer or other testing as necessary.

All patients with DVT are given anticoagulants, initially an injectable


heparin (unfractionated or low mol weight), followed by warfarin.

An Inferior vena cava filter (IVCF) may help prevent PE in patients with
lower extremity DVT and contraindications to anticoagulants or with
recurrent DVT (or emboli) despite adequate anticoagulation. Patients at
higher risk of DVT should have compression devices or stockings placed
for prevention.
Schistocytes on blood film examination are unlikely to be seen in which
of the following?

a) Thrombotic thrombocytopenia purpura (TTP)


b) Thalassemia
c) Hemolytic anemia
d) Disseminated intravascular coagulation (DIC)
e) Glomerulonephritis
The correct answer is B

Explanation
Schistocytes are red cell fragments which are formed when fibrin strands
come in contact with circulating red cells. Schistocytes can be seen in
disseminated intravascular coagulation (DIC), micropathic hemolytic
anemia, glomerulonephritis, and hemolytic anemia resulting from
mechanical trauma to the red blood cells (such as severe burns).
In thalassemia the peripheral blood smear typically shows hypochromia,
microcytosis, and target cells.
Which one of the following is the most common risk factor for retinal
detachment?

a) Posterior detachment of the vitreous


b) Hyphema
c) Glaucoma
d) Cataract surgery
e) Diabetic retinopathy
The correct answer is A

Explanation
Vitreous detachment is very common after age 60 and occurs frequently in
younger persons with myopia. The separation of the posterior aspect of
the vitreous from the retina exerts traction on the retina, with the
attendant risks of a retinal tear and detachment. Symptoms of retinal
detachment may include light flashes (photopsia), a sudden appearance or
increase in “floaters”, or peripheral visual field loss, any of which
should prompt an ophthalmology referral. Cataract surgery can result in
premature shrinkage of the vitreous and thereby poses an increased risk,
but vitreous detachment resulting from other processes is more common.
Hyphema, glaucoma, and diabetic retinopathy are not specific risk
factors for retinal detachment.
A 60 year old man with a history of alcohol abuse presents with the new
onset of ascites. Which one of the following statements is false?

a) Salt restriction is the most important initial treatment measure


b) Abdominal paracentesis is indicated for diagnostic purposes
c) Spironolactone is the diuretic of choice
d) Bacterial peritonitis is commonly present in the absence of
abdominal pain, tenderness or fever
The correct answer is D

Explanation
Ascites is the condition in which there is free fluid in the peritoneal
cavity. The most common cause is portal hypertension. Symptoms usually
result from abdominal distention. Diagnosis is based on physical
examination, ultrasound, or CT. Treatments include bed rest, dietary Na
restriction, diuretics, and therapeutic paracentesis.

Ascitic fluid can become infected (spontaneous bacterial peritonitis),


often with pain and fever. Only 13% of patiens with bacterial
peritonitis are asymptomatic. Diagnosis of infection involves analysis
and culture of ascitic fluid. This infection is treated with antibiotics.

Bilevel positive airway pressure ventilation (BiPAP) has been shown to


reduce morbidity and mortality from which one of the following?

a) Acute respiratory distress syndrome


b) Respiratory failure associated with severe pneumonia
c) Sepsis
d) Pneumothorax
e) Exacerbation of COPD
The correct answer is E

Explanation
Bilevel positive airway pressure (BiPAP) is beneficial in the treatment
of patients with progressive respiratory acidosis, impending respiratory
failure, or progressive fatigue associated with increased work of
breathing. In patients with acute exacerbations of COPD, it not only
improves ventilation while decreasing pCO2 levels, but may well be the
key to avoiding intubation. BiPAP has been shown to be deleterious in
the treatment of respiratory failure associated with sepsis, pneumonia,
acute respiratory distress syndrome, or pneumothorax.
A 47 year old female presents with sudden strong pain in her right upper
quadrant. She has a fever of 38.5 degrees and she is vomiting. She is
not jaundiced. What is the most likely diagnosis‟

a) Cancer of pancreas
b) Acute cholecystitis
c) Acute hepatitis
d) Biliary lithiasis
The correct answer is B

Explanation
Acute cholecystitis is inflammation of the gallbladder that develops
over hours, usually as a result of cystic duct obstruction by a
gallstone. Symptoms include right upper quadrant pain and tenderness,
sometimes accompanied by fever, chills, nausea, and vomiting.

Most patients have had prior attacks of biliary colic or acute


cholecystitis. The pain of cholecystitis has a quality and location
similar to that of biliary colic but with longer duration (ie, > 6
hours) and greater severity. Vomiting is common, as is right subcostal
tenderness. Within a few hours, Murphy's sign (deep inspiration
exacerbates the pain during palpation of the right upper quadrant and
halts inspiration) develops with involuntary guarding of right-sided
abdominal muscles. Fever, usually low grade, is common. In the elderly,
fever may not develop, and the first or only symptoms may be systemic
and nonspecific (eg, anorexia, vomiting, malaise, weakness, fever).

Abdominal ultrasound detects the gallstone and sometimes the associated


inflammation. Treatment usually involves antibiotics and cholecystectomy.

A 36-year-old white female presents to the emergency department with


palpitations. Her pulse rate is 180 beats/min. An EKG reveals a regular
tachycardia with a narrow complex QRS and no apparent P waves. The
patient fails to respond to carotid massage or to two doses of
intravenous adenosine (Adenocard), 6 mg and 12 mg. The most appropriate
next step would be to administer intravenous

a) amiodarone (Cordarone)
b) digoxin (Lanoxin)
c) flecainide (Tambocor)
d) propafenone (Rhythmol)
e) verapamil (Calan)
The correct answer is E

Explanation
If supraventricular tachycardia is refractory to adenosine or rapidly
recurs, the tachycardia can usually be terminated by the administration
of intravenous verapamil or a beta-blocker. If that fails, intravenous
propafenone or flecainide may be necessary. It is also important to look
for and treat possible contributing causes such as hypovolemia, hypoxia,
or electrolyte disturbances. Electrical cardioversion may be necessary
if these measures fail to terminate the tachyarrhythmia.
Which one of the following is most appropriate for the initial treatment
of claudication?
a) Regular exercise
b) Chelation
c) Vasodilating agents
d) Warfarin (Coumadin)
The correct answer is A

Explanation
Claudication is exercise-induced lower-extremity pain that is caused by
ischemia and relieved by rest. It affects 10% of persons over 70 years
of age. However, up to 90% of patients with peripheral vascular disease
are asymptomatic.

Initial treatment should consist of vigorous risk factor modification


and exercise. Patients who follow an exercise regimen can increase their
walking time by 150%. A supervised program may produce better results.
Risk factors include diabetes mellitus, hypertension, smoking, and
hyperlipidemia. Unconventional treatments such as chelation have not
been shown to be effective. Vasodilating agents are of no benefit. There
is no evidence that anticoagulants such as aspirin have a role in the
treatment of claudication.

A 70 year old female presents with heart failure resulting in shortness


of breath, fatigue, exercise limitation and fluid retention in the form
of peripheral and pulmonary edema. What is the pathophysiology of the
disease?

a) Concentric cardiac hypertrophy


b) Increased venous perfusion
c) Increased contractility of the myocardium
d) Inactivation of sympathetic nervous system
e) Increase in cardiac output
The correct answer is A

Explanation
Cardiac hypertrophy occurs when terminally differentiated cardiac
myocytes respond to an increase in workload or injury (such as
hypertension or myocardial infarction) by increasing cell size to
maintain a normal cardiac output.

This decreases the contractility of the myocardium of the heart,


resulting in decreased output of the blood pumped by it.

This also decreases the size of the chambers of heart.


Once again hypertrophy is of two types pathological and physiological.
Pathological occurs due to abnormal stress and physiological occurs due
to pregnancy or exercise. Concentric hypertrophy is none other but
physiological hypertrophy.

A 25-year-old white female presents to your office with the following


complaints: sudden onset of intense apprehension, fear, terror
associated with impending doom, dyspnea, palpitations, and a feeling of
loss of control. Which one of the following is the most likely diagnosis‟

a) Pheochromocytoma
b) Panic attack
c) Hypochondriasis
d) Hypoglycemia
e) Hyperthyroidism
The correct answer is B

Explanation
Panic attacks generally begin between the ages of 17 and 30, and 80% of
those affected are women. Panic attacks are manifested by the sudden
onset of intense apprehension, fear, or terror, often associated with
thoughts of impending doom and at least four of the following somatic
symptoms: dyspnea (patients often hyperventilate); palpitations, chest
pain or discomfort; chocking or smothering sensations; dizziness; a
feeling of unreality; paresthesias; diaphoresis; faintness; trembling or
shaking; hot and cold flashes; and fears of dying, going crazy, or
losing control during an attack.

Hypochondriasis is a condition where the patient is preoccupied with


health and absorbed in his/her own physical ailments. Major depressive
episodes/depression are characterized by marked, sustained changes of
mood. In major depression the prevailing mood is low, being described as
“blue”, “down in the dumps” or apathetic. Part of the low mood consists
of a decreased ability to enjoy acitivites that usually are a source of
pleasure
You see a 49 year old man in the emergency room with a 20 year history
of alcohol abuse. He is agitated and floridly psychotic, with visual
hallucinations and persecutory delusions. On examination his blood
pressure, heart rate and respiratory rate are all increased. He is
disoriented, sweaty, and has abdominal cramps. What is the most likely
diagnosis‟

a) Subdural hematoma
b) Alcohol intoxication
c) Hepatic encephalopathy
d) Delirium tremens
e) Paranoid schizophrenia
The correct answer is D

Explanation
Delirium tremens (DTs), a very serious set of symptoms, may result if
alcohol withdrawal is left untreated. Delirium tremens usually does not
begin immediately; rather, it appears about 2 to 10 days after the
drinking stops. In delirium tremens, the person is initially anxious and
later develops increasing confusion, sleeplessness, nightmares,
excessive sweating, and profound depression. The pulse rate tends to
speed up. Fever typically develops.

The episode may escalate to include fleeting hallucinations, illusions


that arouse fear and restlessness, and disorientation with visual
hallucinations that may incite terror. Objects seen in dim light may be
particularly terrifying, and the person becomes extremely confused. The
floor may seem to move, the walls fall, or the room rotates. As the
delirium progresses, the hands develop a persistent tremor that
sometimes extends to the head and body, and most people become severely
uncoordinated. Delirium tremens can be fatal, particularly when untreated.
Mr. Johnson is a 90 year old man who presents to the Emergency
Department with severe abdominal pain.
He was watching TV when the pain began suddenly and became the worst
pain ever. After the onset of pain, he felt a sudden urge to empty his
bowel. He describes the pain as sharp and stabbing and nothing makes it
better. His past medical history is significant for 2 previous MI last
year and atrial fibrillation managed with atenolol and warfarin. He was
diagnosed with diabetes 10 years ago for which he is taking pioglitazone
and metformin. He admits that he is not a compliant patient.

Physical exam reveals an elderly male writhing on the bed in pain. Vital
signs examination reveals: T of 37 C, BP of 155/95 mm Hg, respiratory
rate of 15/min and an irregularly irregular pulse of 150/min. Abdominal
examination is normal except for mild tenderness to palpation in the
periumblical area. Which of the following is the most likely diagnosis
of this patient?

a) Acute appendicitis
b) Factitious disorder
c) Gastroenteritis
d) Mesenteric ischemia
e) Perforated peptic ulcer
The correct answer is D

Explanation
The classic picture of Acute Mesenteric ischemia (AMI) is an elderly
complaining of severe abdominal pain which is out of proportion to
physical findings. Given the patient‟s physical exam reveals an
irregularly irregular pulse ?Atrial Fibrillation?, AMI from a mesenteric
artery embolus is the most likely diagnosis.

Perforated peptic ulcer would present with peritoneal signs (board like
rigidity, rebound?).

Factitious disorder should never be the most likely diagnosis before


ruling out a potentially life threatening condition as mesenteric ischemia.

You should always think of acute appendicitis in every acute abdomen


case; however the hyper-acute symptoms and the absence of peritoneal
signs make appendicitis unlikely.

In the absence of vomiting, colicky pain and diarrhea, gastroenteritis


is unlikely.
Which of the following vitamins is produced endogenously?

a) Vitamin A
b) Vitamin B6
c) Vitamin C
d) Vitamin D
The correct answer is D

Explanation
Vitamins are substances that our body needs to grow and develop
normally. There are 13 vitamins we need. They are vitamins A, C, D, E, K
and the B vitamins (thiamine, riboflavin, niacin, pantothenic acid,
biotin, vitamin B-6, vitamin B-12 and folate). You can usually get all
your vitamins from the foods you eat. Your body can also make vitamins D
and K. People who eat a vegetarian diet may need to take a vitamin B12
supplement.
A 40-year-old white male was seen 4 weeks ago for a sudden onset of
cough and shortness of breath. At that visit his oxygen saturation was
92%, but his examination and a chest radiograph were normal. You
prescribed azithromycin (Zithromax) and an albuterol inhaler. Ten days
later he was feeling well and his oxygen saturation was 97%. Today, he
returns to the office with a dry cough and shortness of breath.

On examination he has rare inspiratory rales that clear with deep


breaths, and he has an oxygen saturation of 86%. A chest film and a
D-dimer test are normal. Pulmonary function tests show significant
restriction that improves only minimally with albuterol. He has not been
exposed to anyone with a similar illness, has no history of asthma, and
has no smoking history or occupational exposure. However, he reports
that 2 months ago his home was flooded after a heavy rain, and he has
been tearing out carpeting that was ruined by the flood.

Which one of the following is the most likely diagnosis‟

a) Persistent asthma with acute exacerbations


b) Legionnaires‟ disease
c) Pulmonary embolism, with lupus antibody as the most likely cause of
the negative
D-dimer test
d) Hypersensitivity pneumonitis
The correct answer is D

Explanation
Hypersensitivity pneumonitis can present in acute, subacute, or chronic
forms. The case described includes two episodes of the acute form. The
patient was exposed to mold antigens in his flooded home. Within 4?8
hours of exposure, chills, cough, and shortness of breath will be noted,
and at times will be dramatic. A chest film can be normal, even with
significant hypoxia. Pulmonary function tests will show restrictive
changes, as compared to the reversible obstructive changes of acute
asthma. Blood tests often show an elevated erythrocyte sedimentation
rate. Serum IgG tests for the probable antigen confirm the diagnosis.

Symptoms resolve over several days, but will suddenly and violently
recur with repeated exposure to the offending antigen. The subacute form
of hypersensitivity pneumonitis begins gradually over weeks or months,
causing a cough and increasing shortness of breath. The chronic form
develops over years of exposure, causing fibrotic changes to the lungs
that will be evident on radiographs, as well as chronic rales on
auscultation.

Asthma would be an unlikely diagnosis in this case, with the pulmonary


function tests showing restrictive changes rather than obstructive
changes, and little improvement with albuterol. Also, the lack of a
previous history of asthma makes it less likely. Legionnaires‟ disease
is always possible, but is unlikely in this case given the sudden onset,
quick recovery over several days, and sudden recurrence. Pulmonary
embolism is ruled out by the negative D-dimer test.

An obese middle-aged adult has acanthosis nigricans. Which one of the


following malignant diseases is the *least* likely associated illness‟
a) Hodgkin disease
b) Gastric adenocarcinoma
c) Pheochromocytoma
d) Endometrial cancer
e) Mycosis fungoides
The correct answer is B

Explanation
_

Which one of the following is true regarding supraventricular


tachycardia (SVT)?

a) Verapamil is useful for decreasing recurrences of SVT


b) Hyperparathyroidism is a common cause of SVT
c) Hyponatremia is a common cause of SVT
d) Structural heart disease is often associated with SVT
e) Excessive alcohol intake can suppress SVT
The correct answer is A

Explanation
The use of verapamil, propranolol, or digoxin reduces SVT episodes by
30%-50%. There is no evidence that one is superior to the others.
Hypokalemia and hyperthyroidism can cause SVT, but there is no
association with hyperparathyroid disease or hyponatremia. Structural
heart disease is a rare cause of supraventricular tachycardia, but still
should be ruled out with echocardiography. Excessive alcohol use can
precipitate SVT.

On her annual physical exam, a young white patient, known for refractory
hypertension is found to have right flank continuous bruit. You suspect
a secondary cause for her hypertension. Which of the following is most
likely?

a) Hyperaldosteronism
b) Cushing syndrome
c) Renal artery stenosis
d) Aortic coarctation
e) Pheochromocytoma
The correct answer is C

Explanation
Renovascular hypertension (RVH) is the most common form of curable high
blood pressure and is estimated to affect 1% of all hypertensive
individuals. It occurs when significant unilateral or bilateral renal
artery stenosis causes renal ischemia.

RVH develops as a result of significant stenosis (generally >60%) of one


main renal artery and occasionally arises from lesions of a distal
branch. Two thirds of renal artery stenoses result from atherosclerosis,
whereas the various forms of fibromuscular dysplasia (FMD) are
responsible for approximately one third. Atherosclerotic lesions are
typically proximal and found more frequently in older men with evidence
of extensive vascular involvement. Bilateral disease, azotemia, and
recurrent pulmonary edema are also more frequent in this group.
Conversely, patients with FMD tend to be young white women with no
family history of essential hypertension; lesions usually involve the
middle and distal segments of the renal artery. Smoking increases the
risk of renovascular hypertension in both groups.
Physical examination may show abdominal or flank continuous bruit and/or
advanced retinopathy (grades III, IV by KWB classification)
Which one of the following is an osmotic laxative?

a) Polycarbophil (e.g., FiberCon)


b) Polyethylene glycol (e.g., MiraLax)
c) Senna (e.g., Senokot)
d) Mineral oil (e.g., Agorol)
The correct answer is B

Explanation
Polyethylene glycol, a poorly absorbed sugar, is an osmotic laxative
and polycarbophil is a bulk-producing laxative. Senna and mineral oil
are stimulant laxatives.
A 22 year old male, working as a waiter in a nightclub, noticed a
gradual hearing loss. He admitted that the noise level in his job is
very high and that he was not advised to wear ear protection. Which one
of the following does he most likely suffer from?

a) Sensory hearing loss and vertigo


b) Both sensory and conductive hearing loss
c) Conductive hearing loss and ataxia
d) Sensory hearing loss and tinnitus
e) Conductive hearing loss and nystagmus
The correct answer is D

Explanation
When we are exposed to harmful noise, sounds that are too loud or loud
sounds that last a long time, sensitive structures in our inner ear can
be damaged, causing noise-induced hearing loss (NIHL).

Continuous exposure to loud noise also can damage the structure of hair
cells, resulting in hearing loss and tinnitus, although the process
occurs more gradually than for impulse noise.

NIHL is 100 percent preventable. All individuals should understand the


hazards of noise and how to practice good hearing health in everyday
life. To protect your hearing: Know which noises can cause damage (those
at or above 85 decibels). Wear earplugs or other hearing protective
devices when involved in a loud activity.
Which one of the following is true regarding acute bronchitis‟

a) The associated cough typically lasts more than 2 weeks


b) Fever, tachypnea, and rales are typical of acute bronchitis in young
patients
c) ß-Agonists are indicated for treatment
d) Antimicrobial agents are indicated for treatment
e) Purulent sputum implies a bacterial etiology
The correct answer is A

Explanation
The cough of acute bronchitis typically lasts 2 weeks and frequently up
to 20 days. Fever, tachypnea, and rales in young patients are typical of
pneumonia, not bronchitis. Both antimicrobials and ß-agonists are of
questionable benefit in patients with acute bronchitis. Viruses are the
most common cause of acute bronchitis, and purulent sputum reflects
desquamation of the airway, with the expectorated matter being mostly
epithelial cells.
A 70 year old female had a lumbar vertebral fracture 3 years ago. At
that time she had a dual-energy x-ray absorptiometry (DEXA) scan, with a
T score of ?2.6, and was placed on alendronate (Fosamax), calcium, and
vitamin D. She recently quit smoking. Her BMI is 19. A DEXA scan today
shows her bone mineral density to be ?2.1. Which one of the following
would be most appropriate in the management of this patient?

a) Replace alendronate with raloxifene (Evista)


b) Stop alendronate, but continue calcium and vitamin D
c) Add raloxifene to her regimen
d) Add teriparatide (Forteo) to her regimen
e) Make no change to her regimen

The correct answer is E

Explanation
Even though the patient‟s DEXA has improved and she is technically
osteopenic, she still has risk factors for osteoporosis, including
recent smoking, low BMI, and a prior fragility fracture. She should
continue her current regimen.
The 38-year-old female was referred to you by one of your regular
patients. She complains of a rash that appeared suddenly and spread over
her right trunk. She says that it “itches worse than anything I have
ever known”. On the first day or two of the rash, she felt slightly
feverish and had a temperature of 99.9F. Last week, she tried oatmeal
baths and lotion, without relief. She denies any other symptoms and has
no history of exposure to chemicals or new medications.

Which one of the following is the most likely diagnosis‟

a) Pityriasis rosea
b) Psoriasis
c) Lichen planus
d) Secondary syphilis
e) Tinea corporis
The correct answer is C
Explanation
Lichen planus usually appears suddenly, is often associated with a
low-grade fever, and is most common in middle age; women are affected
more often than men. The characteristic intensely pruritic lesions are
often described as purple, polygonal, and planar papules. Pityriasis
rosea begins in 50%-90% of patients with a primary lesion called the
“herald patch”, which is a scaly, oval patch a few centimeters in
diameter. If it appears, it is followed in a few days by the development
of scaly, erythematous patches on the trunk along cleavage planes of the
skin. It rarely occurs on the proximal extremities. The lesions are
usually asymptomatic with only mild itching. Secondary syphilis occurs
1-2 months after the primary lesion called the chancre. The rash usually
appears a erythematous macules, a maculopapular eruption, or circular
papules, but can take many forms. It tends to be widespread and includes
the palms and soles. Psoriasis is characterized by a rash described as
erythematous macules and plaques covered with silvery scales. Tinea
corporis presents with erythematous scaly patches with raised borders,
with occasional papules and sometimes pustules.
A patient who is in college in another state calls to report that he has
developed wheezing, oral itching, and a swollen lower lip after kissing
his girlfriend. The symptoms reminded him of an allergic reaction to
peanuts he had when he was a child, so he self-administered a dose of
adrenalin from his EpiPen kit 15 minutes ago. His itching and wheezing
have improved, and he asks what else he should do.

What advice should you provide?

a) He should take oral diphenhydramine (Benadryl) now and prednisone


for 3 days
b) He should go to the nearest emergency department
c) He should schedule a comprehensive reevaluation by an allergist
d) No further action is needed
The correct answer is B

Explanation
Patients with peanut allergy can have reactions to infinitesimal amounts
of peanut protein, including residue on the lips of other people. This
patient has successfully interrupted the course of anaphylaxis.
Diphenhydramine can help reduce subsequent symptoms, and prednisone is
generally given, although its value is unproven. However, the patient is
at risk of a biphasic reaction and should go to an emergency department
where additional adrenalin and resuscitation facilities are available.
The Academy of Pediatrics guideline recommends that all peanut-allergic
patients who require a dose of adrenalin be observed in an emergency
department for at least 4 hours.

Patients who have not already had a full allergy evaluation need to see
an allergist, but this patient‟s peanut allergy has been established.
Currently, desensitization is ineffective and has a high complication
rate, but in the future an agent that blocks IgE may be available.
Peanut-allergic patients tend to have accidental exposure about once
every 5 years in spite of efforts at avoidance.
A 47 year old male presents with a history of fatigue, arthralgias,
nonspecific abdominal pain, and erectile dysfunction. The initial
laboratory workup reveals a normal CBC and basic metabolic profile, but
slightly elevated transaminases.
Which one of the following is the most appropriate initial test to
evaluate for hereditary hemochromatosis‟

a) Serum ferritin testing


b) Serum transferrin saturation
c) Serum ceruloplasmin testing
d) Serum alpha-fetoprotein testing
e) HFE gene testing
The correct answer is B

Explanation
Hereditary hemochromatosis is the most common single-gene disorder in
Caucasians. Approximately 1 in 250-300 Caucasians are homozygous for the
mutation and 1 in 10 is a carrier. The classic description for this
disorder has been the triad of cutaneous hyperpigmentation, diabetes
mellitus, and cirrhosis; however, this represents very late
manifestations. If treatment is initiated early, life expectancy is
normal and many of the irreversible manifestations of the disease can be
avoided. Unfortunately, while it is estimated that the average primary
care physician sees one patient every 2 weeks with the disorder, most
will actually diagnose only a few cases in their careers.

The disease results from increased intestinal iron absorption with


subsequent deposition of iron in organs such as the heart, liver, skin,
pituitary gland, and genitalia. The differential diagnosis should
include iron overload from other sources such as chronic anemia,
prolonged iron supplementation, multiple transfusions, and chronic liver
disease. Alcohol use and hepatitis C may accelerate the expression of
the disease. Early symptomatology commonly includes arthralgias,
fatigue, and impotence, but approximately 75% of patients are
asymptomatic early on.

The serum transferrin saturation is the best initial screening test. It


is calculated by dividing the serum iron concentration by the total
iron-binding capacity and multiplying by 100. The normal range is
14%-50% with the range for hereditary hemochromatosis being 51%-100%.
However, it may be normal early in the course of the disease and can be
elevated in other states such as alcoholic liver disease and viral
hepatitis.

While serum ferritin is a sensitive assessment of iron overload, it is


an acute phase reactant and is often elevated in inflammatory and
infectious conditions. It is not the recommended screening test for
hereditary hemochromatosis. Serum ceruloplasmin is elevated in
neoplastic disorders, inflammatory states, systemic lupus erythematosus,
primary biliary cirrhosis and rheumatoid arthritis, and low in Wilson‟s
disease, advanced liver disease, total parenteral nutrition,
malabsorption, and nephritic syndrome. It is not useful in the
evaluation of hemochromatosis. Serum alpha-fetoprotein is useful in
screening for hepatocellular carcinoma in patients with cirrhosis, but
is not a screening test for hemochromatosis. Genetic testing for this
disorder is available and the gene, HFE, is located on the short arm of
chromosome 6. However, this would not be appropriate as an initial
screening test.

A 20 year old man presents with respiratory distress. You establish that
he has a severe acute asthmatic attack. In addition to oxygen, which one
of the following is the most important medication he requires‟

a) Morphine
b) Corticosteroids
c) Albuterol
d) Montelukast sodium
e) Sodium cromoglycate
The correct answer is C

Explanation
An asthma attack can be frightening, both to the person experiencing it
and to others around. Even when relatively mild, the symptoms provoke
anxiety and alarm. A severe asthma attack is a life-threatening
emergency that requires immediate, skilled, professional care. If not
treated adequately and quickly, a severe asthma attack can cause death.

Supplemental oxygen may be given during attacks. However, in severe


attacks, a doctor also needs to monitor carbon dioxide levels, and this
test requires a sample of blood from an artery.

Short-acting beta-adrenergic agonists (eg albuterol) are usually the


best drugs for relieving asthma attacks. Most beta-receptor agonists,
especially the inhaled ones, act within minutes, but the effects last
only 2 to 6 hours.

They are given by mouth in higher doses for people experiencing severe
attacks. Corticosteroids given by mouth are generally continued for at
least several days after a severe attack.
A 44-year-old black female reports diffuse aching, especially in her
upper legs and shoulders. The aching has increased, and she now has
trouble going up and down stairs because of weakness. She has no visual
symptoms, and a neurologic examination is normal except for proximal
muscle weakness. Laboratory tests reveal elevated levels of serum
creatine kinase and aldolase. Her symptoms improve significantly when
she is treated with corticosteroids. Which one of the following is the
most likely diagnosis‟

a) Duchenne‟s muscular dystrophy


b) Myasthenia gravis
c) Amyotrophic lateral sclerosis
d) Aseptic necrosis of the femoral head
e) Polymyositis
The correct answer is E

Explanation
The patient described has an inflammatory myopathy of the
polymyositis/dermatomyositis group. Proximal muscle involvement and
elevation of serum muscle enzymes such as creatine kinase and aldolase
are characteristic. Corticosteroids are the accepted treatment of choice.

It is extremely unlikely that Duchenne‟s muscular dystrophy would


present after age 30. In amyotrophic lateral sclerosis, an abnormal
neurologic examination with findings of upper motor neuron dysfunction
is characteristic. Patients with myasthenia gravis characteristically
have optic involvement, often presenting as diplopia. The predominant
symptom of aseptic necrosis of the femoral head is pain rather than
proximal muscle weakness. Elevated muscle enzymes are not characteristic.
A 51 year old man suffered a myocardial infarction 2 years ago. He has
been well since. His non-fasting cholesterol level is 6.4 mmol/L. Which
one of the following is the next step in his management?

a) No further action
b) Obtain a total cholesterol after a 12-hour fast
c) Obtain a lipid profile (cholesterol, triglycerides and HDL
cholesterol) after a 12-hour fast
d) Initiate a cholesterol-lowering diet
e) Obtain a lipoprotein electrophoresis
The correct answer is C

Explanation
A full lipid panel is a group of tests that are often ordered together
to determine risk of coronary heart disease. The tests that make up a
lipid profile are tests that have been shown to be good indicators of
whether someone is likely to have a heart attack or stroke caused by
blockage of blood vessels (hardening of the arteries).

The full lipid profile includes total cholesterol, HDL-cholesterol


(often called good cholesterol), LDL-cholesterol (often called bad
cholesterol), and triglycerides. Sometimes the report will include
additional calculated values such as the Cholesterol/HDL ratio or a risk
score based on lipid profile results, age, sex, and other risk factors.

A 51 year old woman presents to her physician for follow up of a fasting


serum cholesterol level of 243 mg/dL. She is post-menopausal since age
49, and has not been on hormone replacement therapy. She has a positive
family history for coronary artery disease and she has smoked one-half
pack of cigarettes per day for the past 15 years. Today she would like
to know the results of her full lipid panel. Which of the following
lipid panels would most strongly suggest the need for pharmacologic
therapy in this patient?

a) Total cholesterol 185 mg/dL, LDL 145 mg/dL


b) Total cholesterol 235 mg/dL, LDL 115 g/dL
c) Total cholesterol 245 mg/dL, LDL 165 mg/dL
d) Total cholesterol 295 mg/dL, LDL 125 mg/dL
The correct answer is C

Explanation
For those patients in whom a fasting panel has been obtained, a stepwise
approach to intervention based on the patient's LDL and risk factors may
be used. A patient with 2+ risk factors (this patient) and an LDL of
greater than 160 mg/dL warrants medical therapy.

A 60-year-old male with a right-sided pleural effusion undergoes


thoracentesis. Analysis of the pleural fluid reveals a protein level of
2.0 g/dL and an LDH level of 70 U/L. His serum protein level is 7.0 g/dL
(N 6.0-8.3) and his serum LDH level is 200 U/L (N 100-105). Based on
these findings, which one of the following is the most likely diagnosis‟

a) Heart failure
b) Pulmonary embolism
c) Tuberculous pleurisy
d) Malignancy
e) Bacterial pneumonia
The correct answer is A

Explanation
Pleural effusions may be exudates or transudates. The distinction is
important for an accurate diagnosis and to help determine what further
evaluations may be necessary. Light‟s criteria use ratios of fluid/serum
values for protein and LDH. Pleural fluid/serum ratios greater than 0.6
for LDH and 0.5 for protein are indicative of exudates. In the scenario
presented, both ratios are approximately 0.3; therefore, the fluid is a
transudate.

The list of causes for transudates is much shorter than for exudates.
The vast majority of transudates are due to heart failure, with
cirrhosis being the next most common cause. Once there is reasonable
certainty that the fluid is a transudate, additional studies usually are
not necessary. The other conditions listed result in exudative pleural
effusions.
What is the drug interaction commonly seen when a patient on warfarin is
given bactrim (Trimethoprim/ Sulfamethoxazole)?

a) Increases effect of warfarin, causing bleeding


b) Decreases effect of warfarin, potential for clot formation
c) Increases efficacy of bactrim
d) Decreases efficacy of bactrim
The correct answer is A

Explanation
Taking warfarin and bactrim may cause more of a chance for bleeding. It
has been reported that bactrim can prolong the prothrombin time in
patients who are receiving the anticoagulant warfarin.

Using warfarin together with sulfamethoxazole is usually not


recommended, but may be required in some cases. Potential interaction
can cause the patient to feel dizzy or lightheaded, have blood in the
urine, have bloody, black or sticky bowel movements, have unusual
bleeding in the vagina, have unusual bruising.
An elderly woman notes the gradual onset of fatigue and feeling tired.
She has gained a significant amount of weight over the last year. She
complains frequently that the house is too cold and is turning up the
thermostat. Constipation has become a problem. Which of the following is
true of this condition?

a) TSH is depressed
b) Anti-thyroglobulin antibody are elevated
c) Anti-thyroid microsomal antibodies are decreased
d) TRH is elevated
e) TSH is increased
The correct answer is E

Explanation
Hypothyroidism is thyroid hormone deficiency. It is diagnosed by
clinical features such as a typical facies, hoarse slow speech, and dry
skin, and by low levels of thyroid hormones. Symptoms may include cold
intolerance, constipation, forgetfulness, fatigue, personality changes
and weight gain.

Serum TSH is the most sensitive test. In primary hypothyroidism, there


is no feedback inhibition of the intact pituitary, and serum TSH is
always elevated, whereas serum free T4 is low. In secondary
hypothyroidism, free T4 and serum TSH are low.
During the physical exam of a patient‟s foot, you notice an inability to
dorsiflex. What nerve is most likely affected?

a) L1
b) L2
c) L3
d) L4
The correct answer is D

Explanation
“Foot Drop” describes a disorder that affects the patient's ability to
raise their foot at the ankle. It is further characterized by an
inability to point the toes toward the body (dorsiflexion) or move the
foot at the ankle inward or outward. Pain, weakness, and numbness may
accompany loss of function.

Drop foot is not a disease but a symptom of an underlying problem.


Depending on the cause, drop foot may be temporary or permanent. Often
drop foot is caused by injury to the peroneal nerve deep within the
lumbar and sacral spine. The peroneal nerve is a division of the sciatic
nerve. The peroneal nerve runs along the outside of the lower leg (below
the knee) and branches off into each ankle, foot, and first two toes. It
innervates or transmits signals to muscle groups responsible for ankle,
foot, and toe movement and sensation.

The peroneal nerve is susceptible to different types of injury. Some of


these include nerve compression from lumbar disc herniation (e.g. L4,
L5, S1).
You see a 55-year-old female for the first time. She has a 2-year
history of chronic daily cough; thick, malodorous sputum; and occasional
hemoptysis. She has been treated with antibiotics for recurrent
respiratory infections, but is frustrated with her continued symptoms.
She has never smoked. Her FEV_1 :FVC ratio is 60% and a CT scan shows
bronchial wall thickening and luminal dilation.

The most likely diagnosis is

a) emphysema
b) bronchiectasis
c) chronic bronchitis
d) bronchiolitis
e) asthma
The correct answer is B

Explanation
Bronchiectasis is an illness of the bronchi and bronchioles involving
obstructive and infectious processes that injure airways and cause
luminal dilation. In addition to daily viscid, often purulent sputum
production with occasional hemoptysis, wheezing and dyspnea occur in 75%
of patients. Emphysema and chronic 1 bronchitis, forms of COPD, also
cause a decreased FEV :FVC ratio, but the baseline sputum is generally
mucoid and luminal dilatation of bronchi is not characteristically
present. Bronchiolitis is usually secondary to respiratory syncytial
virus infection in young children. Asthma is not characterized by the
sputum and CT findings seen in this patient.
For which one of the following insulins does the onset of action occur
within 15 minutes, with peak action occurring within about 1 hour?

a) NPH
b) Regular insulin (Novolin R, Humulin R)
c) Lispro (Humalog)
d) Lente (insulin zinc suspension)
e) Glargine (Lantus)
The correct answer is C

Explanation
The traditional bolus or mealtime insulin is regular insulin that has an
onset of action within 30-60 minutes and peaks in 2-3 hours. As a
result, regular insulin must be injected 30-60 minutes before meals. In
order to avoid this problem, analogue insulins have been created by
substituting 1-3 amino acids in the human insulin protein, which allows
for altered absorption rates and more reliable absorption profiles. The
analogue forms of bolus or mealtime insulin include lispro and asparte.
Both of these begin acting within 15 minutes and peak in about 1 hour,
mimicking normal mealtime insulin release. NPH, lente, and glargine are
all basal insulins with a longer onset, peak, and duration of action.

A 72 year old man on physical examination is found to have expressive


dysphasia and mild right arm weakness. The most probable location of his
lesion is

a) Right parietal lobe


b) Left frontal lobe
c) Right frontal lobe
d) Left parietal lobe
e) Basal ganglia
The correct answer is B

*Explanation*:
The frontal lobes are essential for planning and executing learned and
purposeful behaviors; they are also the site of many inhibitory
functions. There are at least 4 functionally distinct areas in the
frontal lobes: the primary motor cortex in the precentral gyrus (the
most posterior part) and the medial, orbital, and lateral frontal areas
(termed the prefrontal areas).

Lesions of the inferolateral frontal area (Broca's area) cause


expressive aphasia (impaired comprehension or expression of words).
Lesions of the dorsolateral frontal area can impair the ability to
retain information and process it in real time (eg, to spell words
backwards, to alternate between letters and numbers sequentially).

In the primary motor cortex, all of the moving parts on one side of the
body are controlled by the contralateral side. Because 90% of motor
fibers from each hemisphere cross the midline in the brain stem, damage
to the motor cortex of one hemisphere causes weakness or paralysis on
the opposite side of the body.
A 36 year old member of the National Guard who has just returned from
Iraq consults you because of several ?boils‟ on the back of his neck
that have failed to heal over the last 6 months, despite two week-long
courses of cephalexin (Keflex). You observe three 1- to 2-cm raised
minimally tender lesions with central ulceration and crust formation. He
denies any fever or systemic symptoms. The most likely cause of these
lesions is

a) Pyrogenic granuloma
b) Leishmaniasis
c) Atypical mycobacterial infection
d) Squamous cell carcinoma
e) Epidermal inclusion cysts
Correct Answer:* b)

*Explanation*:
The most likely diagnosis is cutaneous leishmaniasis, caused by an
intracellular parasite transmitted by the bite of small sandflies.
Lesions develop gradually, and are often misdiagnosed as folliculitis or
as infected epidermal inclusion cysts, but they fail to respond to usual
skin antibiotics. Hundreds of cases have been diagnosed in troops
returning from Iraq, most likely due to Leishmania major.

Treatment is not always required, as most lesions will resolve over


several months; however, scarring is frequent. Military medical
facilities and the CDC are coordinating treatment when indicated with
sodium stibogluconate.
Which one of the following medications is most effective for treating
the arrhythmia shown here?

a) Atropine
b) Bretylium tosylate (Bretylol)
c) Lidocaine (Xylocaine)
d) Procainamide (Pronestyl)
e) Adenosine (Adenocard)
The correct answer is E

*Explanation*:
Adenosine, an expensive intravenous drug, is highly effective in
terminating many resultant supraventricular arrhythmias. Although it can
cause hypotension or transient atrial fibrillation, adenosine is
probably safer than verapamil because it disappears from the circulation
within seconds. Because of its safety, many cardiologist now prefer
adenosine over verapamil for treatment of hypotensive supraventricular
tachycardia. Bretylium tosylate, procainamide, and lidocaine are used to
treat ventricular arrhythmias. Atropine is indicated in the treatment
of sinus bradycardia.
A 52 year old man with a history of seizure attacks suffered from loss
of consciousness followed by tonic and clonic muscular contractions. His
tongue fell back into his throat and he choked. What is the best
diagnostic test for assessment of the presenting condition of this patient?

a) X-rays
b) CT-Scan
c) Electroencephalogram
d) Discography
e) Fluoroscopy
The correct answer is C

*Explanation*:
Answer: C ? Electroencephalogram is the appropriate diagnostic test to
be employed for the identification and assessment of grandmal seizures.
Electroencephalogram displays the electrical activity of the brain
through the electrodes connected. In this procedure plain and flat
electrodes are used to detect the electrical changes of the brain. Brain
cells communicate through electrical impulses, which would be detected
by the electrodes and recorded. It is the major diagnostic test for not
only grand mal seizure but also for other seizure disorders.

X ray or CT scan is not the right choice of diagnostic test for grandmal
seizure.
Discography is also not the right choice of diagnostic test for
identification and assessment of the grandmal seizure because it is done
for the patients going for lumbar surgery. This involves the insertion
of a thin needle into the lumbar disc by giving the anesthesia guided by
x-rays.
Fluoroscopy is not the appropriate choice of diagnostic test for
grandmal seizure because it is used to take the images of the body parts
in motion.

The earliest presenting symptom in most older patients with open-angle


glaucoma is

a) Unilateral eye pain


b) Unilateral eye redness
c) Unilateral visual loss
d) Tunnel vision
e) Double vision
The correct answer is D
*Explanation*:
About 3% of persons over age 55 have glaucoma, making it a leading cause
of vision impairment. Although it is usually asymptomatic, the most
common presenting symptom is tunnel vision, a gradual loss of peripheral
vision.
Radiographic features of osteoarthritis of the knee include which of the
following

a) Marginal erosions
b) Juxta-articular osteopenia (demineralization)
c) Loss of articular cartilage with narrowing of the radiologic joint space
d) Osteonecrosis (avascular necrosis) of the medial femoral condyle
e) High riding patella (patella alta)
The correct answer is C

*Explanation*:
Osteoarthritis (OA) is a chronic arthropathy of an entire joint
characterized by disruption and potential loss of joint cartilage along
with other joint changes, including bone hypertrophy (osteophyte
formation). Symptoms include gradually developing pain aggravated or
triggered by activity, stiffness relieved < 30 min after activity, and
occasional joint swelling. Diagnosis is confirmed by x-rays.

OA should be suspected in patients with gradual onset of symptoms and


signs, particularly in older adults. If OA is suspected, plain x-rays
should be obtained of the most symptomatic joints. X-rays generally
reveal marginal osteophytes, narrowing of the joint space, increased
density of the subchondral bone, subchondral cyst formation, bony
remodeling, and joint effusions. Standing x-rays of knees are more
sensitive to joint space narrowing.
A tuberculin-negative health science student is exposed to an infectious
tuberculous patient. If the student develops tuberculosis, which one of
the following forms of the disease is most likely to occur?

a) Pott disease (spinal tuberculosis)


b) Genitourinary tuberculosis
c) Miliary tuberculosis
d) Tuberculous glands in the neck
e) Tuberculous pneumonitis
The correct answer is E

*Explanation*:
Mycobacterium tuberculosis can live only in people, it cannot be carried
by animals, insects, soil, or other nonliving objects. A person can be
infected with tuberculosis only from another person who has active
disease. Touching someone who has the disease does not spread it,
because the bacteria are transmitted only through the air. Mycobacterium
bovis, a bacterium that can live in animals, is an exception. In
developing countries, children become infected with it by drinking
unpasteurized milk from infected cattle.

People with active tuberculosis in their lungs contaminate the air with
bacteria when they cough, sneeze, or even speak. These bacteria can stay
in the air for several hours. If another person breathes them in, that
person may become infected. People who have latent disease or
tuberculosis that is not in their lungs do not spread bacteria into the
air and cannot transmit the infection.

A 69-year-old male who has a history of hypertension and ischemic heart


disease presents with fatigue, dyspnea on exertion, and orthopnea. A
clinical examination demonstrates pulmonary rales, tachycardia, a third
heart sound, and mild pedal edema. A chest radiograph reveals
cardiomegaly and pulmonary venous congestion, and a two-dimensional
echocardiogram confirms heart failure.

Which one of the following is true regarding the management of this patient?

a) The serum brain natriuretic peptide (BNP) test can accurately


differentiate diastolic
from systolic heart failure
b) Optimal treatment for systolic heart failure is necessary to treat
diastolic heart failure
c) Drugs that inhibit the production of angiotensin or block
angiotensin II receptors are
contraindicated
d) Calcium channel blockers reduce mortality in patients with
isolated diastolic
dysfunction
e) ß-Blockers are indicated for treating diastolic dysfunction
The correct answer is E

*Explanation*:
The serum brain natriuretic peptide (BNP) test can accurately
differentiate heart failure from noncardiac conditions in a patient with
dyspnea, but it cannot distinguish diastolic from systolic heart
failure. Optimal treatment for systolic heart failure may exacerbate
diastolic heart failure. ACE inhibitors and angiotensin receptor
blockers (ARBs) directly affect myocardial relaxation and compliance by
inhibiting production of angiotensin or by blocking angiotensin II
receptors, thereby reducing interstitial collagen deposition and
fibrosis. The indirect benefits of optimizing hemodynamics include
improved left ventricular filling, reduced blood pressure, and improved
exercise capacity and quality of life.

Large randomized, controlled trials have not proved that calcium channel
blockers reduce mortality in patients with isolated diastolic
function. ß-Blockers control heart rate and prevent tachycardia, thereby
maximizing diastolic filling in patients with diastolic dysfunction.
They also reduce blood pressure and myocardial ischemia, promote
regression of left ventricular hypertrophy, and antagonize the excessive
adrenergic stimulation that occurs with heart failure. ß-Blockers also
have been independently associated with improved survival in patients
with diastolic heart failure.

A 45 year old obese female patient presents to the Emergency Room with a
history of right upper quadrant pain lasting two hours. She states that
this is her third attack of similar pain in the past two months. The
pain started suddenly and is described as sharp, constant and radiates
to her right shoulder. On examination she is afebrile and has mild right
upper quadrant tenderness. Which one of the following is the most likely
diagnosis‟
a) Duodenal ulcer
b) Myocardial infarction
c) Appendicitis
d) Diverticulitis
e) Biliary colic
The correct answer is E

*Explanation*:
Biliary colic is a condition characterized by extreme cramping pain in
the right upper abdomen. Gallstones in the gallbladder, or in the bile
ducts are the cause of the severe pain.

About one third of patients with gallstones develop biliary colic or


other complications. People who are overweight or have a high level of
blood cholesterol increase their risk of developing gallstones and
biliary colic. Pregnant women, or those on birth control pills or
estrogen replacement therapy also have a greater risk of developing
biliary colic. Other risk factors include rapid weight loss, diabetes,
certain gastrointestinal conditions and certain medications.

Biliary colic is caused by gallstones that form in the gallbladder or


bile ducts. Gallstones form when bile, a substance made in the liver to
help digest fats, contains too much cholesterol. A solid particle forms
and becomes a gallstone.

Biliary colic can cause extreme pain in the upper right abdomen and
nausea, especially after eating a meal high in fat. The condition can
also cause jaundice in the skin and eyes. The pain can last up to three
hours and sometimes spreads to the right shoulder or through the center
of the back. Fatty food intolerance, dyspepsia, indigestion, heartburn,
flatulence and eructation are other symptoms associated with gallstone
disease. Attacks of biliary colic are more common at night, possibly
because the gallbladder shifts to a horizontal position, making it
easier for stones to enter into the cystic duct.
A 60 year old man is seen in the emergency room and is diagnosed as
having acute pancreatitis. Which one of the following would suggest more
severe illness‟

a) Serum amylase level > 2000


b) Hyperglycemia and glycosuria
c) High urinary amylase level
d) Serum lipase level > 20,000
e) Decreased serum calcium level
The correct answer is B

*Explanation*:
Ranson‟s criteria for predicting the mortality of a patient involves
assessment on admission and then again at 48 hours, with the following
criteria:

On Admission:
Age > 55
WBC > 16 on admission
LDH > 350 on admission
SGOT(AST) > 250 on admission
Glucose > 200 on admission

At 48 hours After Admission:

Hct drop > 10%


BUN increase > 5 US (> 1.79 SI)
Ca < 8 (US) < 2 (SI)
Arterial pO2 < 60
Base deficit (24 - HCO3) < 4
Fluid needs > 6L

Of the answer choices, on admission a high glucose level would


contribute to this patients mortality.
A 49 year old woman presents to your office complaining of increasing
fatigue. She has always had heavy menstrual periods.

Her hemoglobin (Hb) is 85 g/L (Normal Hb 123-157 g/L), mean corpuscular


volume (MCV) is 75 fL (Normal 80-100), her ferritin is 10 µmol/L (Normal
23-862 µmol/L).

The next most appropriate step in the investigation of this patient is


to do which one of the following?

a) Prescribe ferrous gluconate and monitor response


b) Pelvic ultrasound
c) Bone marrow
d) Colonoscopy
e) Take a detailed dietary history
The correct answer is E

*Explanation*:
Iron deficiency anemia causes include blood loss, lack of iron in the
diet and inability to absorb iron. Women with heavy periods are at risk
of iron deficiency anemia because they lose a lot of blood during
menstruation. Your body regularly gets iron from the foods you eat. If
you consume too little iron, over time your body can become iron
deficient. Examples of iron-rich foods include meat, eggs, dairy
products or iron-fortified foods. For proper growth and development,
infants and children need iron from their diet, too.

An intestinal disorder, such as Crohn's disease or celiac disease, which


affects your intestine's ability to absorb nutrients from digested food,
can lead to iron deficiency anemia. If part of your small intestine has
been bypassed or removed surgically, that may affect your ability to
absorb iron and other nutrients. Some medications can interfere with
iron absorption.

To diagnose, it is important to check the hematocrit, the percentage of


your blood volume made up by red blood cells and hemoglobin. Normal
levels of hemoglobin range between 11.1 and 15.0 grams per deciliter
(g/dL). A lower than normal hemoglobin level indicates anemia. For women
a low level is less than 10 g/dL, and for men a low level is less than
12 g/dL.

In addition, blood tests for iron deficiency anemia typically include a


measurement of ferritin, a protein that helps store iron in your body. A
low level of ferritin usually indicates a low level of iron.
A 50 year old man presents to the emergency room with severe epigastric
pain, low-grade fever, tachycardia, and mild hypotension. The patient
relates a history of moderate to heavy social drinking. The chief
resident suspects acute pancreatitis. Which one of the following is the
single most important laboratory finding to confirm the diagnosis of
pancreatitis‟

a) Hyperlipidemia
b) Hyperbilirubinemia
c) Elevated serum amylase
d) Elevated serum phospholipase A
e) Elevated serum alkaline phosphatase
The correct answer is C

*Explanation*:
Acute pancreatitis is inflammation of the pancreas (and, sometimes,
adjacent tissues) caused by the release of activated pancreatic enzymes.
The most common triggers are biliary tract disease and chronic heavy
alcohol intake. The condition ranges from mild (abdominal pain and
vomiting) to severe (pancreatic necrosis and a systemic inflammatory
process with shock and multiorgan failure).

Diagnosis is based on clinical presentation and serum amylase and lipase


levels. Treatment is supportive, with IV fluids, analgesics, and fasting.
In a patient with iron deficiency anemia, the diagnosis can be confirmed
by giving the individual oral iron replacement, and then 1 week later

a) Repeating the hemoglobin level


b) Performing hemoglobin electrophoresis
c) Checking a serum iron level
d) Measuring the RBC indices
e) Performing a reticulocyte count
The correct answer is E

*Explanation*:
Once therapy for iron deficiency has begun, the patient may notice a
decrease in fatigue within the first week. Laboratory parameters will
also change in a predictable fashion. Reticulocytosis will peak in 7-10
days; however, the hemoglobin level will not begin to change for at
least 2 weeks, and can take 2 months to return to a normal level. Once
the ferritin level has returned to normal, iron replacement therapy can
be stopped.
A 47-year-old male who lives at sea level attempts to climb Mt. Rainier.
On the first day he ascends to 3400 m (11,000 ft). The next morning he
complains of headache, nausea, dizziness, and fatigue, but as he
continues the climb to the summit he becomes ataxic and confused. Which
one of the following is the treatment of choice?

a) Administration of oxygen and immediate descent


b) Dexamethasone, 8 mg intramuscularly
c) Acetazolamide (Diamox), 250 mg twice a day
d) Nifedipine (Procardia), 10 mg immediately, followed by 30 mg in 12
hours
e) Helicopter delivery of a portable hyperbaric chamber
The correct answer is A

*Explanation*:
The patient described initially showed signs of acute mountain sickness.
These include headache in an unacclimatized person who recently arrived
at an elevation >2500 m (8200 ft), plus the presence of one or more of
the following: anorexia, nausea, vomiting, insomnia, dizziness, or
fatigue. The patient‟s condition then deteriorated to high-altitude
cerebral edema, defined as the onset of ataxia and/or altered
consciousness in someone with acute mountain sickness. The management of
choice is a combination of descent and supplemental oxygen. Often, a
descent of only 500?1000 m (1600?3300 ft) will lead to resolution of
acute mountain sickness. Simulated descent with a portable hyperbaric
chamber also is effective, but descent should not be delayed while
awaiting helicopter delivery. If descent and/or administration of oxygen
is not possible, medical therapy with dexamethasone and/or acetazolamide
may reduce the severity of symptoms. Nifedipine has also been shown to
be helpful in cases of high-altitude pulmonary edema where descent
and/or supplemental oxygen is unavailable.
A 78 year old man complains of 3 months of fatigue and a 10 kg weight
loss. Physical examination reveals jaundice without abdominal tenderness
or organomegaly. Which one of the following is the most likely diagnosis‟

a) Cholecystitis
b) Pancreatitis
c) Hemochromatosis
d) Hepatic vein thrombosis
e) Pancreatic carcinoma
The correct answer is E

*Explanation*:
In pancreatic cancer, symptoms include weight loss, abdominal pain, and
jaundice. Symptoms occur late; by diagnosis, 90% of patients have
locally advanced tumors that have involved retroperitoneal structures,
spread to regional lymph nodes, or metastasized to the liver or lung.

Most patients have severe upper abdominal pain, which usually radiates
to the back. The pain may be relieved by bending forward or assuming the
fetal position. Weight loss is common. Adenocarcinomas of the head of
the pancreas produce obstructive jaundice (often causing pruritus) in 80
to 90% of patients.

Diagnosis is by CT. Treatment is surgical resection and adjuvant


chemotherapy and radiation therapy. Prognosis is poor because disease is
often advanced at the time of diagnosis.

When considering a diagnosis of pancreatitis, amylase levels

a) can help determine the severity of the disease


b) are less likely to be elevated in alcoholics
c) are more sensitive and specific than serum lipase levels
d) are less likely to be affected by nonpancreatic conditions such as
renal insufficiency
The correct answer is B

*Explanation*:
Amylase and lipase levels are used to help make the diagnosis of acute
pancreatitis. The serum lipase level is more specific and more sensitive
than the amylase level. Amylase elevations can be seen with other
abdominal illnesses, such as inflammation of the small bowel. Alcoholics
with recurrent pancreatitis may have normal serum amylase levels; in
such cases, serum lipase would be a better test.

There are several scoring systems for the severity of pancreatitis,


including the CT severity index, the APACHE II score, the Imrie Scoring
System, and Ranson‟s Criteria, but none of these use serum amylase in
their calculation. The elevation of serum amylase does not correspond
well with the severity of the pancreatitis.
A 73 year old white male who is otherwise in good health noted a sudden
shower of flashing lights before his left eye a few hours ago. He has
never had this symptom before. The most likely diagnosis is

a) Migraine syndrome
b) Retinal detachment
c) Vitreous floaters
d) Central retinal vein thrombosis
e) Temporal lobe tumor
The correct answer is B

*Explanation*:
While all of the conditions listed are associated with flashing light
before the eyes, a sudden shower of flashing lights is highly suggestive
of retinal detachment. Migraine syndrome rarely begins in the eighth
decade, vitreous floaters are more likely to be perceived as spots
rather than lights, and central retinal vein thrombosis would likely
cause more loss of vision. Emergency referral to an ophthalmologist is
mandatory in cases of probable retinal detachment.
A 20 year old white man was stabbed in the chest in a bar fight and
arrived at the emergency room within 30 minutes. You noticed that the
trachea is deviated away from the side of the chest that suffered the
puncture. Which one of the following would you find upon physical
examination of the traumatized side?

a) Increased fremitus
b) Increased breath sounds
c) Dullness to percussion
d) Hyperresonant percussion
e) Wheezing and stridor
The correct answer is D

*Explanation*:
A pneumothorax refers to a collection of gas in the pleural space
resulting in collapse of the lung on the affected side. A tension
pneumothorax is a life-threatening condition caused by air within the
pleural space that is under pressure; displacing mediastinal structures
and compromising cardiopulmonary function. A traumatic pneumothorax
results from blunt or penetrating injury that disrupts the parietal or
visceral pleura.

Physical findings classically consist of absent tactile fremitus,


hyperresonance to percussion, and decreased breath sounds on the side
with the pneumothorax. If the pneumothorax is large, the side with the
pneumothorax may be enlarged with the trachea visibly shifted to the
opposite side.

Diagnosis is made with an upright inspiratory chest x-ray. Treatment


involves needle thoracentesis to release the trapped air followed by
placement of a chest tube.
A 72 year old male with class III congestive heart failure (CHF) due to
systolic dysfunction asks if he can take ibuprofen for his ?aches and
pains‟. Appropriate counseling regarding NSAID use and heart failure
should include which one of the following?

a) NSAIDs are a good choice for pain relief as they decrease systemic
vascular resistance
b) NSAIDs are a good choice for pain relief as they augment the
effect of his diuretic
c) High-dose aspirin (325 mg/day) is preferable to other NSAIDs for
patients taking ACE inhibitors
d) NSAIDs, including high-dose aspirin, should be avoided in CHF
patients because they can cause fluid retention
The correct answer is D

*Explanation*:
If possible, NSAIDs should be avoided in patients with heart failure.
They cause sodium and water retention as well as an increase in systemic
vascular resistance which may lead to cardiac decompensation. Patients
with heart failure who take NSAIDs have a tenfold increased risk of
hospitalization for exacerbation of their CHF. NSAIDs alone in patients
with normal ventricular function have not been associated with initial
episodes of heart failure.

NSAIDs, including high-dose aspirin (325 mg/day) may decrease of negate


entirely the beneficial unloading effects of ACE inhibition. They have
been shown to have a negative impact on the long-term morbidity and
mortality benefits that ACE inhibitors provide. Sulindac and low-dose
aspirin (81 mg/day) are less likely to cause these negative effects.
A 68-year-old white female has a bone mineral density that is 2.7
standard deviations below the reference mean for healthy, young white
women. You prescribe alendronate (Fosamax), 70 mg orally once a week,
and counsel the patient that the medication should be taken

a) on an empty stomach
b) at bedtime
c) with her other medications
d) with no more than 4 oz of water
The correct answer is A

*Explanation*:
Absorption of alendronate can be reduced by food and certain minerals.
It should be taken with 6?8 oz of water at least 30 minutes before the
first food, drink (other than water), or medication of the day.
Furthermore, because of an increased risk of gastroesophageal
irritation, the patient should remain upright for at least 30 minutes
after taking alendronate.

A 75 year old female presents with a 1-month history of pain in her hips
and shoulders bilaterally, accompanied by marked stiffness in the
mornings. In addition, she reports a 4-lb weight loss and fatigue. She
denies fever, chills, sweats, nausea, vomiting, swallowing disturbances,
or changes in bowel habits. She specifically denies any visual symptoms
of headache. Her physical examination is unremarkable, except for
changes suggestive of osteoarthritis in the knees and hands. Laboratory
testing reveals a hemoglobin level of 11.8 g/dL (N 13.0-16.0), a
hematocrit of 36% (N 40-45), and an erythrocyte sedimentation rate of 84
mm/hr (N <20). What is the most appropriate management at this point?

a) Begin prednisone, 60 mg daily, and refer for a temporal artery


biopsy as soon as possible
b) Begin prednisone, 60 mg daily, and plan to refer for a temporal
artery biopsy if her symptoms do not respond
c) Begin prednisone, 15 mg daily, with no plans for a biopsy
d) Begin a COX-2 inhibitor such as rofecoxib (Vioxx), with no plans
for a biopsy
e) Refer for a temporal artery biopsy and wait for the results before
starting drug therapy
The correct answer is C

*Explanation*:
It is generally not considered necessary to refer patients with classic
polymyalgia rheumatica for a temporal artery biopsy in the absence of
symptoms or signs of giant cell arteritis (e.g., headache, visual
complaints, jaw claudication, fever, scalp tenderness, abnormal
funduscopic exam). Corticosteroids should be started at relatively low
doses (10-20 mg daily) and the patient followed for what should be a
rapid clinical response.

A young female presents with supraventricular tachycardia. She complains


of chest pain, palpitation and fainting. The best initial management is

a) IV Verapamil
b) Cardioversion
c) Increased vagal tone
d) Beta blocker
The correct answer is C

*Explanation*:
Episodes of paroxysmal supraventricular tachycardia often can be stopped
by one of several maneuvers that stimulate the vagus nerve and thus
decrease the heart rate. These maneuvers are usually conducted or
supervised by a doctor, but people who repeatedly experience the
arrhythmia often learn to perform the maneuvers themselves.

Maneuvers include straining as if having a difficult bowel movement,


rubbing the neck just below the angle of the jaw (which stimulates a
sensitive area on the carotid artery called the carotid sinus), and
plunging the face into a bowl of ice-cold water. These maneuvers are
most effective when they are used shortly after the arrhythmia starts.

If these maneuvers are not effective, if the arrhythmia produces severe


symptoms, or if the episode lasts more than 20 minutes, people are
advised to seek medical intervention to stop the episode. Doctors can
usually stop an episode promptly by giving an intravenous injection of a
drug, usually adenosine or verapamil.
Which of the following pairs of CNS lesions and corresponding visual
field defects is correct?

a) Temporal lobe tumour - central scotoma


b) Frontal lobe tumour - homonomous hemianopsia
c) Pituitary tumour - bitemporal hemianopsia
d) Occipital lobe tumour - altitudinal field defect
e) Multiple Sclerosis - superior quadrantanopia
The correct answer is C

*Explanation*:

A pituitary tumour can cause bitemporal hemianopsia. This is a condition


where one cannot see out of the periphery. Damage to the medial aspect
of the optic chiasm, as is often seen with a pituitary gland tumor, may
compromise the decussating fibers from both nasal hemiretinas. The loss
of peripheral vision in both eyes is called bitemporal hemianopia.
A 65 year old male smoker presents with stage I hypertension. The
patient has diabetes mellitus, obesity and renal disease. What is the
best medication of choice for treatment of this patient?
a) Hydrochlorothiazide
b) Propranolol
c) Nitroprusside
d) Captopril
e) Methyldopa
The correct answer is D

*Explanation*:
Captopril is the most appropriate medication for the
treatment of stage I hypertension. Captopril is an angiotensin
converting enzyme, which prevents the conversion of angiotensin I to
angiotensin II. Captopril is most widely used medication for
hypertension because of its efficient reduction of hypertension through
renin-angiotensin system. Since the patient already has renal disease
and diabetes, captopril is the ideal medication when compared to others.
It is given as 6.25 mg thrice a day initially and increased gradually
from 25-150 mg based on the requirement and dose response.

A. Borderline renal and/or hepatic insufficiency may be aggravated by


hydrochlorothiazide.
B. Propranolol is a non selective beta-blocking agent; can mask hypoglycemia.
C. Nitroprusside has decreased efficacy in renal failure. Toxic levels
of cyanide build up rapidly in patients with renal failure.
E. Methyldopa is not the ideal medication for the stage I hypertension
because it acts by reducing the total tissue peripheral and has direct
effect on the blood vessels, which is not desirable for the patient with
renal disease.
A patient with chronic renal failure is being treated with furosemide
for edema on a long term basis. Which one of the following is the most
common adverse effect?

a) Depression
b) Hypokalemia
c) Metabolic acidosis
d) Hypervolemia
e) Bradycardia
The correct answer is B

*Explanation*:
Furosemide is a powerful diuretic. Furosemide is used to treat excessive
fluid accumulation and swelling (edema) of the body caused by heart
failure, cirrhosis, chronic kidney failure, and nephrotic syndrome. It
is sometimes used in conjunction with other blood pressure pills to
treat high blood pressure.

Commonly observed side effects are low blood pressure, and water and
electrolyte depletion. Furosemide can cause lowering of blood potassium,
sodium, and magnesium levels. Low potassium and magnesium levels can
lead to heart rhythm abnormalities.
A 63 year old woman presents to the ER with palpitations, with a
pounding feeling in her chest and shortness of breath. An EKG done shows
the following:
This patient has

a) Atrial fibrillation
b) Atrial flutter
c) Ventricular fibrillation
d) Ventricular flutter
The correct answer is B

*Explanation*:
An EKG of Atrial flutter will show the following:

Atrial rate: Usually 300 beats/min, ranging between 220 and 350 beats/min
Rhythm: The atrial rhythm is regular
P waves: Flutter waves resemble a sawtooth or picket fence and are best
seen in leads II, III or aVF
PR interval: Usually the PR interval is regular, but it may vary
QRS interval: This pattern is usually normal

Treatment involves rate control with calcium channel blockers or beta


blockers. Conversion of rate with a type I antiarrhythmic agent, such as
quinidine or procainamide follows.
Which one of the following drug classes is preferred for treating
hypertension in patients who also have diabetes mellitus‟

a) Centrally-acting sympatholytics
b) Alpha-Blocking agents
c) Beta-Blocking agents
d) ACE inhibitors
e) Calcium channel blockers
The correct answer is D

*Explanation*:
ACE inhibitors have proven beneficial in patients who have either early
or established diabetic renal disease. They are the preferred therapy in
patients with diabetes and hypertension, according to guidelines from
the National Kidney Foundation, the World Health Organization.
A 40-year-old female presents with the gradual onset of weakness,
fatigue, anorexia, and nausea. She has lost 15 lb over the past 3 months.

A physical examination is unremarkable except for the patient‟s tired


appearance, a blood pressure of 90/60 mm Hg, and diffuse
hyperpigmentation of the skin and mucous membranes. Laboratory
evaluation includes a sodium level of 130 mmol/L (N 136-145) and a
potassium level of 5.8 mmol/L (N 3.5-5.1)
Which one of the following is the best test to further evaluate this
patient‟s condition?

a) A TSH level
b) A cosyntropin (Cortosyn) stimulation test
c) A plasma ACTH level
d) A dexamethasone suppression test
The correct answer is B

*Explanation*:
Primary adrenal insufficiency (Addison‟s disease) is an uncommon
condition in clinical practice, and typically has an autoimmune
etiology. Patients often experience the insidious onset of weakness,
fatigue, and gastrointestinal symptoms. Physical findings typically
include hypotension and diffuse hyperpigmentation of the patient‟s skin
and mucous membranes. Due to the loss of aldosterone, serum sodium
levels tend to be low, and hyperkalemia is common.

Cosyntropin is a synthetic analogue of ACTH. The cosyntropin stimulation


test is performed by administering 250 ?g of cosyntropin intravenously
or intramuscularly and measuring serum cortisol 30-60 minutes later. A
normal response is serum cortisol > 500 nmol/L. A lower level suggests
adrenal insufficiency.

In primary adrenal insufficiency, TSH and ACTH levels may be elevated,


but this is variable and ACTH continues to demonstrate diurnal
variation. The dexamethasone suppression test is used in the evaluation
of cortisol excess.

Osteoporotic bone loss can be caused or accelerated by prolonged use of


which one of the following medications‟

a) Hydrochlorothiazide
b) Phenytoin
c) Raloxifene (Evista)
d) Diazepam (Valium)
e) Fluoxetine (Prozac)
The correct answer is B

*Explanation*:
Secondary osteoporosis can result from a variety of endocrine,
nutritional, or genetic disorders, as well as from prolonged use of
certain medications. Anticonvulsants such as phenytoin increase the
hepatic metabolism of vitamin D, thereby reducing intestinal calcium
absorption. Other medications that adversely affect bone mineral density
include glucocorticoids, cyclosporine, phenobarbital, and heparin.
Thiazide diuretics reduce urinary calcium loss and are believed to
preserve bone density with long-term use. Benzodiazepines and SSRIs have
not been associated with increases in bone loss or in hip fractures.
Raloxifene, a selective estrogen receptor modulator, is indicated for
the prevention and treatment of osteoporosis in postmenopausal women.
A patient presents with complaints of weight loss, easy bruising and
greasy smelly stools. You also notice a dermatitis herpetiformis rash on
his arms. You suspect celiac sprue. What is the most appropriate method
for diagnosis‟

a) EGD
b) Bowel biopsy
c) Colonoscopy
d) CT of abdomen
The correct answer is B

*Explanation*:
Celiac sprue is an immunologically mediated disease in genetically
susceptible individuals caused by intolerance to gluten, resulting in
mucosal inflammation, which causes malabsorption. Symptoms usually
include diarrhea and abdominal discomfort.

Celiac sprue is a hereditary disorder caused by sensitivity to the


gliadin fraction of gluten, a protein found in wheat; similar proteins
occur in rye and barley. The inflammatory response produces
characteristic mucosal villous atrophy in the small bowel.

In adults, lassitude, weakness, and anorexia are most common. Mild and
intermittent diarrhea is sometimes the presenting symptom. Steatorrhea
ranges from mild to severe (7 to 50 g fat/day). Some patients have
weight loss, rarely enough to become underweight. Anemia, glossitis,
angular stomatitis, and aphthous ulcers are usually seen in these patients.

About 10% have dermatitis herpetiformis, an intensely pruritic


papulovesicular rash that is symmetrically distributed over the extensor
areas of the elbows, knees, buttocks, shoulders, and scalp. This rash
can be induced by a high-gluten diet.

Diagnosis is by small-bowel biopsies demonstrating characteristic though


not specific pathologic changes of villous atrophy that improve with a
strict gluten-free diet.
Treatment for Helicobacter pylori infection will reduce or improve which
one of the following?

a) The risk of peptic ulcer bleeding from chronic NSAID therapy


b) The risk of developing gastric cancer in asymptomatic patients
c) Symptoms of nonulcer dyspepsia
d) Symptoms of gastroesophageal reflux disease
The correct answer is A

*Explanation*:
Eradication of Helicobacter pylori significantly reduces the risk of
ulcer recurrence and rebleeding in patients with duodenal ulcer, and
reduces the risk of peptic ulcer development in patients on chronic
NSAID therapy.

Eradication has minimal or no effect on the symptoms of nonulcer


dyspepsia and gastroesophageal reflux disease. Although H. pylori
infection is associated with gastric cancer, no trials have shown that
eradication of H. pylori purely to prevent gastric cancer is beneficial.
A 59 year old man strikes the steering wheel of his car during a low
speed automobile accident. In the emergency department he is alert and
his vital signs are normal. He complains of mild tenderness on sternal
compression. Chest x-ray film shows a widened mediastinum. The most
appropriate first step is to order

a) CT scan of the chest with contrast


b) Bronchoscopy
c) Thoracentesis
d) Thoracic ultrasonography
The correct answer is A

*Explanation*:
A classic history such as this should always trigger a rule out
diagnosis for aortic arch dissection. Traumatic dissection is very
common in trauma situations and the classical description is a widened
mediastinum on plain chest films. However, this finding is highly
non-specific and therefore, CT imaging with contrast of the thorax is
now the preferred method for ruling out this diagnosis.

Bronchoscopy is used to assess the trachea and upper airways via direct
visualization. It has no role in the evaluation of possible aortic
injury. Thoracocentesis is used to evaluate fluid present in the pleural
space, a so called pleural effusion. A needle is inserted lateral or
inferior to the lung but within the pleural space to withdraw fluid for
analysis.

Thoracic ultrasonography could refer to imaging of the thorax or of the


heart. Regardless, trans-thoracic ultrasound has limited ability to
evaluate the aortic arch. A trans-esophageal ultrasound however is
exquisitely sensitive for detecting aortic arch
injury and allows complete inspection of the aortic root and valve, as
well as the coronary cusps and sinuses.
A 32-year-old black female presents with a 3-day history of fever,
cough, and shortness of breath. She has been healthy otherwise, except
for a sinus infection 2 months ago treated with amoxicillin. She does
not appear toxic. A chest radiograph reveals an infiltrate in the right
lower lobe, consistent with pneumonia. Which one of the following would
be the best choice for antibiotic treatment?

a) High-dose amoxicillin
b) Azithromycin (Zithromax)
c) Doxycycline
d) Levofloxacin (Levaquin)
e) Cefuroxime axetil (Ceftin)
The correct answer is D

*Explanation*:
For previously healthy patients with community-acquired pneumonia and no
risk factors for drug resistance, a macrolide such as azithromycin is
the preferred treatment (SOR A). Doxycycline is also acceptable (SOR C).

Patients who have been treated with antibiotics within the previous 3
months should be treated with a respiratory fluoroquinolone
(moxifloxacin, gemifloxacin, or levofloxacin) (SOR A). A -lactam plus a
macrolide is also an alternative (SOR A). The antibiotic chosen should
be from a different class than the one used for the previous infection.
These alternative treatments are also recommended for those with
comorbidities such as chronic heart, lung, liver, or renal disease;
diabetes mellitus; alcoholism; malignancies; asplenia; immunosuppressing
conditions or use of immunosuppressing drugs; or other risk factors for
drug-resistant Streptococcus pneumoniae infection (SOR A).
Which of the following scenarios best describes otosclerosis‟

a) Abnormal eardrum with conductive hearing loss


b) Normal eardrum with conductive hearing loss
c) Abnormal eardrum with sensorineural hearing loss
d) Normal eardrum with sensorineural hearing loss
The correct answer is B

*Explanation*:
Otosclerosis is a disease of the bone of the otic capsule that produces
an abnormal accumulation of new bone within the oval window.

In otosclerosis, the new bone traps and restricts the movement of the
stapes, causing conductive hearing loss. Otosclerosis also may produce a
sensorineural hearing loss, particularly when the foci of otosclerotic
bone are adjacent to the scala media. Half of all cases are inherited.
The measles virus may play an inciting role in patients with a genetic
predisposition for otosclerosis.

Although about 10% of white adults have some otosclerosis (compared with
1% of blacks), only about 10% of affected people develop conductive
hearing loss. Hearing loss from otosclerosis may manifest as early as
age 7 or 8, but most cases do not become evident until the late teen or
early adult years, when slowly progressive, asymmetric hearing loss is
diagnosed. Fixation of the stapes may progress rapidly during pregnancy.

A hearing aid may restore hearing. Alternatively, microsurgery to remove


some or all of the stapes and to replace it with a prosthesis may be
beneficial.

A 45 year old female suffered from upper respiratory tract infection.


Sputum culture test reveals /Streptococcus pneumoniae/. Her chart
contains "bronchial spasm to Penicillin" 4 years ago. What is the best
choice of medication for the treatment of this patient?

a) Ciprofloxacin
b) Amoxicillin
c) Erythromycin
d) Cefaclor
e) Sulfamethoxazole
The correct answer is C

*Explanation*:
Erythromycin is the best medicine for the upper respiratory
tract infections caused by streptococcus pneumoniae. Erythromycin is
characterized by a lactone ring to which sugars are attached. Its
mechanism of action is by binding with 50S ribosomal subunit and there
by inhibiting the bacteria protein synthesis. It is very effective in
showing bactericidal or bacteriostatic effect on streptococcus
pneumoniae and other strains. The strains those are resistant to
penicillin are susceptible to erythromycin and also erythromycin has few
side effects.

Ciprofloxacin does not cover Streptococcus.


Amoxicillin should not be used in this patient. (allergy)
Cefaclor is a cephalosporin derivative which should be avoided in the
patients with a history of hypersensitivity with penicillin.
TMP/SMZ alone does not cover Streptococcus
A 35-year-old African-Canadian male presents with a 3-day history of
increasing cough, sinus pressure, and low-grade fever. His past medical
history is positive only for hypertension.

Which one of the following would be most appropriate at this point?

a) Explaining that most cases of sinusitis resolve without treatment


b) Ordering sinus films to determine the most appropriate course of care
c) Prescribing high-dose pseudoephedrine and increased fluids
d) Prescribing a 7-day course of amoxicillin
e) Prescribing a 10-day course of levofloxacin (Levaquin)
The correct answer is A

*Explanation*:
In patients with a sinus infection, acute bacterial rhinosinusitis
should be diagnosed and treated with antibiotics only if symptoms have
not improved after 10 days or if they worsen after 5-7 days. First-line
therapy for acute bacterial sinusitis is amoxicillin. While increasing
fluid intake might be helpful, taking pseudoephedrine to reduce symptoms
would be a poor choice in a patient with hypertension.
A 55-year-old white male sees you for follow-up after a recent lipid
panel revealed no improvement in his hyperlipidemia. His total
cholesterol level is 275 mg/dL, with an LDL-cholesterol level of 180
mg/dL, an HDL-cholesterol level of 35 mg/dL, and a triglyceride level of
275 mg/dL. These numbers are similar to two previous lipid panels
obtained over the last several months, despite attempts at lifestyle
changes. He has adequately treated essential hypertension, with a blood
pressure of 125/83 mm Hg. There is no history of diabetes mellitus or
tobacco use, and no family history of premature coronary heart disease.

A physical examination is unremarkable except for a BMI of 33 kg/m and a


waist circumference of 107 cm (42 in). His fasting blood glucose level
is 107 mg/dL.

After discussion with the patient, you decide to start prescription drug
therapy. The initial target of this therapy should be to reach his goal
level of
a) LDL cholesterol
b) HDL cholesterol
c) non-HDL cholesterol
d) triglycerides
e) fasting blood glucose
The correct answer is A

*Explanation*:
This patient meets the criteria for metabolic syndrome. In addition to
lifestyle changes, pharmacologic treatment for his hyperlipidemia should
be considered. The initial goal of this therapy should be to reach his
LDL-cholesterol goal, usually using a statin. After achievement of this
goal, non-HDL cholesterol is the secondary target for therapy. Non-HDL
cholesterol is calculated by subtracting HDL cholesterol from total
cholesterol. The non-HDL cholesterol goal is 30 mg/dL higher than the
LDL-cholesterol goal.
A 68 year old white man comes to the office because of increasing
shortness of breath on exertion for the past 2 to 3 months. He has a
history of hypertension for which he takes hydrochlorothiazide. On
physical examination his pulse is 80/min. There is an apical lift
displaced to the left and a harsh diastolic murmur at the base. Further
physical examination is most likely to show

a) Bifid pulse
b) Low-amplitude pulse
c) Pulsus alternans
d) Pulsus paradoxus
e) Wide pulse pressure
The correct answer is E

*Explanation*:
This patient most likely has aortic insufficiency. One of the hallmark
physical findings of this valvular lesion is the presence of a wide
pulse pressure secondary to the diastolic run-off back into the
ventricle. Other signs such as Quincke‟s pulse or Musset‟s sign may also
be present.

A bifid pulse is seen with hypertrophic cardiomyopathy and is best


appreciated by palpation of the carotid artery. This bifid pulse occurs
as a result of no obstruction to blood flowing out from the left heart
chamber in the beginning, followed by an obstruction in the middle of
systole, and finally by a lessening of the obstruction at the end of
systole. Low amplitude pulse is seen with peripheral arteriosclerosis.

Pulsus alternans where one pulse feels large, the next pulse feels
small, is appreciated with severe congestive heart failure. Pulsus
paradoxus is an exaggeration of a normally present fall is systolic
blood pressure with inspiration. Normal decrease in systolic pressure
should be 10 mm Hg or less but with pulsus paradoxus, it can be 15-20 mm
Hg. This is most commonly seen with constrictive or restrictive diseases
of the heart or pericardium.
A 72 year old male with a serum creatinine level of 1.8 mg/dL (N
0.6-1.5) requires a contrast dye study. Which one of the following is
most effective when given prior to the administration of contrast to
reduce the risk for contrast-induced renal failure?
a) N-acetylcysteine
b) Mannitol
c) Furosemide (Lasix)
d) Methylprednisolone sodium succinate (Solu-Medrol)
e) Diphenhydramine (Benadryl)
The correct answer is A

*Explanation*:
Current methods for reducing the risk of renal failure induced by
contrast material include adequate hydration and the use of
N-acetylcysteine. Neither mannitol nor furosemide has been shown to
prevent contrast-induced renal failure. Corticosteroids and
antihistamines are useful for the prevention of idiosyncratic reactions
to contrast, but are not helpful in reducing the risk of renal failure.
A 42 year old woman has recently experienced fatigue, sleepiness, dry
skin, constipation, and a 4.5 kg weight gain. Her thyroid is firm and
twice the normal size. Which one of the following laboratory tests is
most likely to confirm the suspected diagnosis of hypothyroidism?

a) Serum thyroxine (T4)


b) Serum triiodothyronine (T3)
c) T3 resin uptake
d) Serum thyroid-stimulating hormone (TSH) measurement
e) Antithyroid antibodies
The correct answer is D

*Explanation*:
Hypothyroidism is thyroid hormone deficiency. It is diagnosed by
clinical features such as a typical facies, hoarse slow speech, and dry
skin, and by low levels of thyroid hormones.

Serum TSH is the most sensitive test. In primary hypothyroidism, there


is no feedback inhibition of the intact pituitary, and serum TSH is
always elevated, whereas serum free T4 is low. In secondary
hypothyroidism, free T4 and serum TSH are low (sometimes TSH is normal
but with decreased bioactivity). Management includes treatment of the
underlying cause and administration of thyroxine.

A 38 year old male who is a new patient reports mild intermittent


jaundice without other associated symptoms for the past several years.
His liver function tests are normal except for a total bilirubin of 1.3
mg/dL (N 0.3-1.0) and an indirect or unconjugated bilirubin of 1.0 mg/dL
(N 0.2-0.8). His CBC is normal. His past medical and surgical history is
unremarkable. Findings are similar on repeat laboratory testing. The
most likely cause of these findings is

a) Hepatitis C
b) Wilson‟s disease
c) Sickle cell anemia
d) Gilbert‟s syndrome
e) Drug toxicity
The correct answer is D

*Explanation*:
Gilbert‟s syndrome is the most common inherited disorder of bilirubin
metabolism. In patients with a normal CBC and liver function tests,
except for recurrent mildly elevated total and unconjugated
hyperbilirubinemia, the most likely diagnosis is Gilbert‟s syndrome.
Fasting, heavy physical exertion, sickle cell anemia, and drug toxicity
can also cause hyperbilirubinemia.

In achalasia of the esophagus

a) Early findings include a dilated esophagus with retained fluid and


an air-fluid level visible on a chest radiograph
b) Early symptoms are relieved by antireflux therapy
c) Acid reflux is commonly verified by pH monitoring
d) Endoscopy is required as part of the diagnostic workup
The correct answer is D

*Explanation*:
Achalasia of the esophagus is rare, and the consequences are subtle
during its course. The dilated esophagus with an air-fluid level and
retained food on a chest radiograph is a late finding. Dysphagia and
chest pain are the primary symptoms prior to esophageal dilatation. The
common occurrence of heartburn leads the clinician to suspect reflux
disease; however, antireflux therapy provides no relief. Despite the
reported heartburn, little reflux can be verified by pH monitoring.
Achalasia is best detected by functional studies, either fluoroscopy
during a barium swallow or manometry.

Manometry is the most sensitive diagnostic method. Endoscopy must be


done because of the possibility of a malignant tumor infiltrating the
gastroesophageal junction and creating a relative obstruction, as well
as the achalasia syndrome.
A 51 year old white woman with a history of hypertension and
Non insulin-dependent diabetes mellitus presents to the emergency
department with with short-term memory impairment and a blindness.
Patient exhibits Anton's syndrome secondary to occlusion of:

a) Left cerebral artery


b) Right cerebral artery
c) Left middle cerebral artery
d) Right middle cerebral artery
e) Posterior cerebral artery
The correct answer is E

*Explanation*: This patient has developed a stroke due to bilateral


occlusion of the posterior cerebral artery (PCA).
The PCA supplies parts of the midbrain, subthalamic nucleus, basal
nucleus, thalamus, mesial inferior temporal lobe, and occipital and
occipitoparietal cortices.
Bilateral infarction in the distal PCAs produces cortical blindness
(blindness with preserved pupillary light reaction). The patient is
often unaware of the blindness or may even deny it (Anton's syndrome - a
state in which they fervently believe they can see when they cannot).
Tiny islands of vision may persist, and the patient may report that
vision fluctuates as images are captured in the preserved portions.
Rarely, only peripheral vision is lost and central vision is spared,
resulting in "gun-barrel" vision.

A 45-year-old male was admitted to the hospital for nausea resulting


from chemotherapy for colon cancer. He has no other chronic diseases and
takes no routine medications. He was mildly dehydrated on admission and
has been receiving intravenous fluids (D5 ½-normal saline with potassium
chloride) at slightly higher than maintenance rates through an
indwelling port for the last 24 hours. The nausea is being controlled by
antiemetics, and his condition is improving. Results of routine blood
work at the time of admission and from the following morning are shown
below.

Admission
Following Morning

Glucose 109 mg/dL (N 65?110) 371 mg/dL


BUN 13 mg/dL (N 7?21) 9 mg/dL
Creatinine 0.9 mg/dL (N 0.6?1.6) 0.9 mg/dL
Sodium 143 mmol/L (N 136?144) 129 mmol/L
Potassium 3.7 mmol/L (N 3.6?5.1) 6.6 mmol/L
Chloride 110 mmol/L (N 101?111) 108 mmol/L
Total CO 220 mmol/L (N 22?32) 22 mmol/L

Which one of the following would be the most appropriate next step?
a) Start an intravenous insulin drip
b) Order blood work taken from a peripheral vein
c) Restrict the patient‟s free water intake
d) Switch from normal saline to hypertonic saline
e) Treat with diuretics
The correct answer is B

*Explanation*:
Physicians should avoid reacting to laboratory values without
considering the clinical scenario. This patient presented with mild
dehydration and normal laboratory values. Although he is improving
clinically, his laboratory values show multiple unexpected results. The
most noticeable is the severely elevated glucose, because he has no
history of diabetes mellitus or use of medications that could cause this
effect. Similarly, the elevated potassium and decreased sodium suggest
profound electrolyte abnormalities. Most likely, the laboratory
technician drew blood from the patient‟s indwelling port without
discarding the first several milliliters. Thus, the blood was
contaminated with intravenous fluids, resulting in the erroneous
results. A repeat blood test from a peripheral vein should give more
accurate results.

Which one of the following is more consistent with Parkinson‟s disease


than with other variant parkinsonian syndromes‟

a) Tremor at the onset of illness


b) Frequent falls at the onset of illness
c) Urinary urge incontinence at the onset of illness
d) Symmetric, bilateral motor manifestations at the onset of illness
The correct answer is A

*Explanation*:
Tremor is characteristically an early manifestation of Parkinson‟s
disease. It is less pronounced in other variant parkinsonian syndromes.
Examples of these syndromes include multiple system atrophy and
progressive supranuclear palsy. Initial motor findings, including
tremor, are often unilateral in Parkinson‟s disease.

Early falls, autonomic dysfunction, and symmetric motor findings are


more typical of variant parkinsonian syndromes. Compared to Parkinson‟s
disease, these syndromes respond poorly to levodopa.
A 72 year old man with heart failure has had high blood pressure treated
for long time. He was brought to ER. His vitals show a blood pressure of
80/50 and a heart rate of 100. What is the most appropriate next step?

a) Procaine
b) Lidocaine
c) Cardioversion
d) Digoxin
e) Furosemide
The correct answer is D

*Explanation*:
Congestive heart failure (CHF) is a condition in which the heart's
function as a pump to deliver oxygen rich blood to the body is
inadequate to meet the body's needs. Symptoms include dyspnea, fatigue,
and peripheral edema. Diagnosis is clinical and by chest x-ray and
echocardiography. Treatment is the same as for heart failure: ACE
inhibitors, beta-blockers, diuretics and digoxin.

This patient is likely in an acute congestive heart failure


exacerbation. Since the patient is hypotensive, use of either inotropic
therapies and/or mechanical circulatory support in addition to
continuous hemodynamic monitoring is indicated.
A 52 year old male with a history of seizure attacks suffered from loss
of consciousness followed by tonic and clonic muscular contractions. His
tongue fell back into the throat and he choked. What is the most likely
diagnosis of this presentation?

a) Petitmal seizure
b) Grandmal seizure
c) Myoclonic seizure
d) Atonic seizure
e) Convulsions
The correct answer is B

Explanation
Answer: B ? Granmal seizure (tonic-clonic seizures) is characterized by
sudden loss of the consciousness followed by tonic and clonic muscular
contractions. During this situation the tongue could fall back into the
throat and choke the patient. It can be further calssified as simple
partial seizure and complex partial seizure

A. Petitmal seizure is the wrong answer because it is not characterized


by tonic and clonic muscular movements.
C. Myoclonic seizure is also wrong answer because it involves just the
contraction of the muscles but there would be no loss of consciousness
or choking of the throat with saliva.
D. Atonic seizure is wrong answer because here the muscles loose their tone.
E. Convulsions are the wrong answer.

An elderly female with Alzheimer‟s disease is admitted to the hospital


for pneumonia. She has not been eating well for several weeks, so her
family requests that she be started on megestrol (Megace) for appetite
stimulation.

Which one of the following statements is true regarding this drug?

a) It stimulates the appetite center of the thalamus


b) It increases the release of growth hormone
c) It is associated with bleeding secondary to platelet dysfunction
d) It is a glucocorticoid derivative
e) It can result in adrenal suppression
The correct answer is E

Explanation
Megestrol acetate is a synthetic progestational agent that is used to
increase appetite in a variety of cachexia-inducing illnesses. It has
been associated with weight gain in well-designed, randomized trials in
populations of patients with a malignant disease or HIV infection. Its
mechanism of action is unknown. Megestrol acetate doses greater than 160
mg/day can suppress ACTH secretion, leading to secondary adrenal
insufficiency. For patients who are on chronic treatment or who are
being withdrawn from chronic treatment, consideration should be given to
empiric glucocorticoid treatment during times of stress, illness, or
surgery. Megestrol acetate is associated with thrombophlebitis and
pulmonary embolus, but not excessive bleeding.
Paraneoplastic syndromes due to ectopic hormone production are most
often associated with which one of the following cancers‟

a) Brain
b) Stomach
c) Cervical
d) Lung
e) Colon
The correct answer is D

Explanation
Ectopic ACTH production occurs most frequently with small cell
carcinomas of the lung, as does inappropriate antidiuretic hormone (ADH)
production. Hypercalcemia commonly occurs with metastatic lung and
breast cancer. Paraneoplastic syndromes secondary to ectopic secretion
of hormones is seen with a variety of tumors, most commonly lung
(especially small cell tumors), carcinoids, and pancreatic islet cell
tumors.
A 17 year old female has been a strict vegetarian for 3 years now. She
does not eat any meat whatsoever and watches the labels closely of
everything she buys at the grocery store. What vitamin deficiency is she
most likely to develop?

a) Folic acid
b) Vitamin B12
c) Vitamin A
d) Vitamin C
The correct answer is B

Explanation
Studies show that over 90% of vegans (those who ate the strictest
vegetarian diet, which shuns all animal products, including milk and
eggs) had vitamin B12 deficiency. Approximately 65% of vegetarians
(diets that included milk and eggs) were also deficient. Only 5% of
those who consumed meats had vitamin B12 deficiency.

B12 is not found in any plant food other than fortified cereals. It is,
however, abundant in many meats and fish, and in smaller amounts in milk
and eggs. This makes it difficult for people following a strict
vegetarian diet to get the necessary amount of vitamin B12.

Vitamin B12 deficiency leads to anemia. Symptoms of vitamin B12


deficiency, which usually come on gradually, include fatigue, weakness,
nausea, and constipation. Long-term and severe vitamin B12 deficiency
can lead to nerve changes such as numbness, tingling in the hands and
feet, balance and memory problems, and depression.

A 76-year-old male has fallen twice as a result of buckling of the left


knee during ambulation. Neither fall resulted in injury, and he is
advised to use an offset walking cane. The patient is left hand?dominant
and has normal strength in all four extremities. Crepitus is present in
both knees, but is much more pronounced in the left knee.

Which one of the following describes the best method for use of a cane
by this patient?

a) Place the cane in the left hand and advance it at the same time as the left leg
b) Place the cane in the left hand and advance it at the same time as the right leg
c) Place the cane in the right hand and advance it at the same time as the left leg
d) Place the cane in the right hand and advance it at the same time as the right leg
e) Switch the cane between hands at intervals of several hours to distribute the load
equally
The correct answer is C

Explanation
The standard walking cane generally is designed as a tool to aid in
balance and, to a lesser degree, reduce weight bearing on a specific
leg. The offset cane design results in the downward force vector being
placed directly over the shaft, making this choice ideal where improved
balance and reduction of weight bearing on a particular leg is desired.
Mechanical advantage produces maximum benefit when the cane is placed in
the hand opposite the most severely affected leg, and the movement of
the cane tracks the movement of the affected leg, consistent with normal
gait.
The definitive diagnosis of hypertrophic cardiomyopathy is made by

a) Physical examination
b) A chest radiograph
c) Electrocardiography
d) Echocardiography
The correct answer is D

Explanation
Hypertrophic cardiomyopathy may be suspected because of abnormalities
found on physical examination, on an EKG, or on a chest radiograph.
However, the definitive diagnosis is made by two-dimensional
echocardiography.
A 76-year-old male is brought to your office by his son. The patient
complains of dizziness that has slowly been worsening over the past
year. His description is vague, but he says that he notices the
dizziness when he tries to walk.

The review of systems is normal, except for increasing problems with his
?prostate? in the last 6 months, with dribbling and accidents at times.
The patient admits to ?going more often?, and sometimes ?without
warning?. The son states that his father seems more forgetful, slower of
speech, and not as ?full of life? as he used to be.

A neurologic examination reveals the patient to be oriented x 3, with a


somewhat flat affect and a wide-based, slow, shuffling gate. The
examination is otherwise normal. His Mini-Mental State Examination
(MMSE) score is 22 out of 30. No tremor is noted. A CBC, creatinine
level, TSH level, vitamin B12 level and VDRL are all normal.

Which one of the following should you do next?


a) Order a brain MRI
b) Prescribe sertraline (Zoloft)
c) Prescribe carbidopa/levodopa (Sinemet)
d) Order physical therapy
e) Order a brainstem auditory evoked potential
The correct answer is A

Explanation
This patient presents with the classic triad for normal pressure
hydrocephalus: ataxia, urinary incontinence, and dementia. MRI should be
the first step in the diagnosis of this condition, both to rule out
other conditions and to identify changes suggestive of normal pressure
hydrocephalus. These include enlargement of the ventricles with
preservation of cerebral parenchyma.

Brainstem auditory evoked response testing is used for evaluating


cochlear problems. While some other features of normal pressure
hydrocephalus overlap with those of depression and Parkinson‟s disease,
this patient‟s constellation of signs and symptoms are more typical for
normal pressure hydrocephalus. Physical therapy is not indicated prior
to making the diagnosis.

You are called to see an elderly hospitalized patient with mild


obstructive lung disease who was roused one morning with irritability,
restlessness and muscle twitching. Which of the following drug orders is
most likely associated with this episode?

a) Trimethoprim-sulfamethoxazole
b) Hydrochlorothiazide
c) Flurazepam
d) Theophylline
e) Senna
The correct answer is D

Explanation Manifestation of Theophylline Acute Toxicity:


Theophylline has a low therapeutic index. Theophylline toxicity is most
likely to occur when serum concentrations exceed 20 mcg/mL and becomes
progressively more severe at higher serum concentrations. Tachycardia,
in the absence of hypoxia, fever, or administration of sympathomimetic
drugs, may be an indication of theophylline toxicity. Anorexia, nausea
and occasional vomiting, diarrhea, insomnia, irritability, restlessness,
and headache commonly occur. The distinguishing symptoms of toxicity may
include agitated maniacal behavior, frequent vomiting, extreme thirst,
slight fever, tinnitus, palpitation, and arrhythmias. Patients may
experience delirium, muscle twitching, severe dehydration, albuminuria,
emesis of a "coffee ground" material, hyperthermia, and profuse
diaphoresis. Seizures may occur even without other preceding symptoms of
toxicity and often result in death.

Activated charcoal is ineffective for the treatment of acute ingestions


of which one of the following?

a) Acetaminophen
b) Enteric-coated aspirin
c) Ferrous sulfate
d) Amitriptyline
e) Digoxin
The correct answer is C
Explanation
Activated charcoal is widely used for gastrointestinal decontamination
following drug overdose. There is no evidence to support or exclude its
use more than 1 hour after the ingestion, however, and potential
complications such as aspiration should be weighed against benefits. It
is given orally or via nasogastric tube at a recommended dose of 1 g/kg
body weight. Charcoal absorbs >90% of most toxins in vitro if the amount
used is ten times the amount of the toxin. The charcoal-toxin complex is
later evacuated in the stool. However, charged (ionized) chemicals and
dissociated salts such as iron, lithium, fluoride, cyanide, mineral
acids, alkalis, and some other inorganic compounds are poorly absorbed
by charcoal. Even multiple-dose therapy is not effective in the
treatment of poisoning due to these agents. For toxic iron ingestions
useful treatments include gastric evacuation with ipecac or lavage,
whole bowel irrigation with polyethylene glycol, and chelation with
deferoxamine.
A gunshot wound to the upper arm causing shock-wave damage to the median
nerve may leave the patient with which one of the following?

a) Easily provoked pain in the hand


b) Weakness on wrist extension
c) Atrophy in the first dorsal interosseous muscle
d) Numbness over the fifth digit
e) Radial deviation of the hand
The correct answer is A

Explanation

Distal median nerve dysfunction is a form of peripheral neuropathy that


affects the movement of or sensation in the hands. A common type of
distal median nerve dysfunction is carpal tunnel syndrome.

Distal median nerve dysfunction occurs when the nerve is inflamed,


trapped, or injured by trauma. Trapping (entrapment) involves pressure
on the nerve where it passes through a narrow area. Wrist fractures may
injure the median nerve directly or may increase the risk for trapping a
nerve later on.

Sensation changes of the thumb and first 2 fingers, numbness, decreased


sensation, tingling, burning sensation, waking up at night with wrist or
hand pain (may be severe), weakness of the hand, dropping things and
difficulty grasping objects.
Major diagnostic criteria for polycythemia vera include increased red
cell mass, normal oxygen saturation, and

a) hypercapnia
b) thrombocytosis
c) thrombocytopenia
d) hepatomegaly
e) low erythropoietin
The correct answer is E

Explanation
Most consider a high hematocrit result to be an indication of
polycythemia vera. However, the diagnosis cannot be based solely on the
hematocrit result. Therefore, to help make the diagnosis, a test that
uses radioactively labeled red blood cells to determine the total number
of red blood cells in the body (red blood cell mass) is sometimes done.

Once the increased red blood cell mass (polycythemia) is discovered, one
must then determine whether it is polycythemia vera or polycythemia
caused by some other condition (secondary polycythemia).

Blood levels of erythropoietin, a hormone that stimulates the bone


marrow to produce red blood cells, also is measured. Levels of
erythropoietin are extremely low in polycythemia vera, but they are
often, normal or high in secondary polycythemia.
Which one of the following has been shown to reduce the risk of falls in
the elderly?

a) Vitamin D
b) Amitriptyline (Elavil)
c) Haloperidol (Haldol)
d) Lorazepam (Ativan)
e) Procainamide (Pronestyl)
The correct answer is A

Explanation
More than one-third of persons age 65 or older fall each year. Ten
percent of falls result in a serious injury. Arthritis; depression;
orthostasis; impaired cognition, vision, or gait; and the use of four or
more prescription medications increase the risk of falling. Muscle
strengthening and balance exercises and removal of physical hazards can
reduce the rate of falling. Vitamin D deficiency has been associated
with falling, and empiric supplementation with vitamin D reduces the
risk. Tricyclic antidepressants, neuroleptics, bezodiazepines, and type
IA antiarrhythmics are linked to an increased risk of falling.

A 43 year old male presents with fever, chills and weakness. On physical
exam he is found to have a heart murmur. You also notice multiple
splinter hemorrhages on his fingernails. The most appropriate diagnostic
test to obtain is

a) EKG
b) Urine culture
c) Blood culture
d) Chest X-ray
The correct answer is C

Explanation
Infective endocarditis is infection of the endocardium, usually with
bacteria (commonly, streptococci and staphylococci) or fungi. It
produces fever, heart murmurs, petechiae, anemia, embolic phenomena, and
endocardial vegetations. Vegetations may result in valvular incompetence
or obstruction, myocardial abscess, or mycotic aneurysm.

Retinal emboli can cause round or oval hemorrhagic retinal lesions with
small white centers (Roth's spots). Cutaneous manifestations include
petechiae (on the upper trunk, conjunctivae, mucous membranes, and
distal extremities), painful erythematous subcutaneous nodules on the
tips of digits (Osler's nodes), nontender hemorrhagic macules on the
palms or soles (Janeway lesions), and splinter hemorrhages under the nails.

Diagnosis requires demonstration of microorganisms in blood and usually


echocardiography. Treatment consists of prolonged antimicrobial
treatment and sometimes surgery.
A 45-year-old Hispanic female presents with diffuse, tender swelling in
her anterior neck. These symptoms were preceded by a prodrome of sore
throat, low-grade fever, myalgias, and fatigue. On examination she has a
tender, enlarged thyroid gland. Laboratory tests reveal elevated free
T_4 and low TSH.

Which one of the following is indicated at this time to determine


appropriate treatment?

a) Fine-needle aspiration of the thyroid gland


b) Radioactive iodine uptake (RAIU) at 24 hours
c) Immediate initiation of oral prednisone, 40?60 mg/day
d) Antibiotics that cover Streptococcus pyogenes, Streptococcus aureus, and
Streptococcus pneumoniae
e) A CT scan of the neck
The correct answer is B

Explanation
Subacute granulomatous thyroiditis is the most common cause of thyroid
pain. It is four times more common in women, most often occurs at 40?50
years of age, and is usually attributed to a viral infection. Symptoms
and signs of this disorder include a prodrome of myalgias, pharyngitis,
low-grade fever, and fatigue, followed by a tender, diffuse goiter and
neck pain that often radiates up to the ear. Hyperthyroidism is seen in
half of affected individuals, and must be differentiated from that found
with Graves‟ disease to determine appropriate treatment. In patients
with subacute thyroiditis the RAIU at 24 hours is low (<5%), whereas in
those with Graves‟ disease it is elevated. Treatment consists of
relieving the thyroid pain and tenderness with NSAIDs. If no improvement
occurs in a week, prednisone may be given in a dosage of 40?60 mg/day,
tapered off over 4?6 weeks.

Suppurative thyroiditis is an extremely rare form of thyroiditis caused


by bacterial, fungal, mycobacterial, or parasitic infection of the
thyroid gland. Predisposing factors include congenital abnormalities
such as persistent thyroglossal duct or piriform sinus fistula, greater
age, and immunosuppression. Patients commonly present with acute
unilateral anterior neck pain and erythema of the skin overlying an
exquisitely tender thyroid. Fever, dysphagia, and dysphonia are also
present. Thyroid function most often is normal, but hyper- or
hypothyroidism may be present. The erythrocyte sedimentation rate is
elevated, and the WBC count shows a marked increase with a left shift.
Fine-needle aspiration with a Gram stain and culture is the most useful
diagnostic test.
A 65 year old obese male patient complains of strong and sudden mid
abdominal pain radiating to his left flank. Physical exam does not show
any findings. What is the most likely diagnosis‟

a) Acute pancreatitis
b) Cholecystitis
c) Ureteral stone
d) Mesenteric ischemia
e) Rupture of abdominal aorta
The correct answer is D

Explanation
Acute mesenteric ischemia is interruption of intestinal blood flow by
embolism, thrombosis, or a low-flow state. It leads to mediator release,
inflammation, and ultimately infarction. Abdominal pain is out of
proportion to physical findings.

The early hallmark of mesenteric ischemia is severe pain but minimal


physical findings. The abdomen remains soft, with little or no
tenderness. Mild tachycardia may be present.

Early diagnosis is difficult, but angiography and exploratory laparotomy


have the most sensitivity; other imaging modalities often become
positive only late in the disease. Treatment is by embolectomy,
revascularization of viable segments, or resection; sometimes
vasodilator therapy is successful. Mortality is high.

Which one of the following clinical presentations is most consistent


with vitamin D deficiency in the aged?

a) Rapidly progressive osteoarthritis


b) Development of chronic bone pain and weakness in association with bone loss
c) Bleeding gums and fatigue
d) High-output cardiac failure with dermatitis and neuropathy
e) Diarrhea, dermatitis, and dementia
The correct answer is B

Explanation
Vitamin D deficiency is being recognized more frequently among the
elderly, especially in the nursing home or other settings where
inactivity, nutritional deficiency, and lack of sunlight exposure
combine to increase the propensity for deficiency. Vitamin D deficiency
has been associated with abnormalities in bone metabolism, primarily
osetomalacia. Clinically this can present as muscle weakness, limb pain,
and impaired physical function. Bleeding gums and fatigue may be
presenting symptoms of scurvy (vitamin C deficiency). High-output
cardiac failure with vasodilation, dermatitis, and neuropathy is the
clinical presentation of thiamine deficiency. The combination of
diarrhea, scaly dermatitis, and dementia is one of the clinical
presentations of niacin deficiency.
A 44-year-old black female has right upper quadrant abdominal pain that
began as a sharp stabbing pain the night before. It has continued as a
dull ache with brief periods of relief. She has experienced nausea the
last 3 days without any change in
bowel habits. There is no history of trauma. A medical and surgical
history reveals nothing significant.

On examination you note an obese female in mild distress. Her


temperature is 38.3 C (101.0 F), her heart rate 96 beats/min,
respirations 18/min, and blood pressure 122/84 mm Hg. The lungs are
clear, and the heart is normal. She is not jaundiced. There is mild
diffuse tenderness to palpation in the right upper quadrant with minimal
guarding and no rebound tenderness. Ultrasonography of the right upper
quadrant is nondiagnostic.

Which one of the following diagnostic tests would be most helpful?

a) Endoscopic retrograde cholangiopancreatography (ERCP)


b) Colonoscopy
c) A ventilation-perfusion lung scan
d) MRI of the abdomen
e) Cholescintigraphy (HIDA scan)
The correct answer is E

Explanation
This patient has signs and symptoms that are highly suggestive of
gallbladder disease, specifically acute cholecystitis. Cardiac, colonic,
pulmonary, and hepatic disorders can all produce a similar clinical
picture, but the case as presented favors a gallbladder problem. Nuclear
imaging is the most useful test for demonstrating an obstructed cystic
duct, the cause for acute cholecystitis in most patients. Right upper
quadrant abdominal ultrasonography may show the presence of gallstones
or a thickened gallbladder wall, but these findings can occur in chronic
cholecysitis and may not account for the current clinical screnario.
ERCP is not indicated until a diagnosis is established. MRI, colonoscopy
and a ventilation-perfusion lung scan would not be beneficial unless the
most likely diagnosis of gallstone cholecysitis is eliminated.
A 48 year old female presents suffering from left lower quadrant pain
with non-bloody stool. Tenderness is present over the area. She is
afebrile and has a normal rectal examination. Diverticulum is seen in a
barium enema study. What is the appropriate intervention?

a) Give antibiotic prophylaxis for 2 weeks


b) Try high fiber diet
c) Colonoscopy
d) Surgery
The correct answer is B

Explanation
Although this patient is young, she has early diverticulosis. A
diverticulum (plural - diverticula) is medical term for an outpouching
of a hollow (or a fluid filled) structure in the body, like an intestine.
The development of diverticula is caused by spasms of the muscular layer
of the intestine. The resulting pressure that these spasms exert on the
intestinal wall causes a part of the wall to bulge at a point of weakness.

Diverticulosis can cause painful cramps, diarrhea or other bowel


movement disturbances, and blood in the stool. The narrow opening of a
diverticulum can bleed, sometimes heavily, into the intestine and out
through the rectum. Stool that is trapped in a diverticulum may cause
not only bleeding but also inflammation and infection, resulting in
diverticulitis.

The diagnosis is confirmed by barium enema x-ray study or by


colonoscopy. However, if the person has severe abdominal pain, computed
tomography (CT) is performed instead, so as not to rupture the inflamed
intestine.

The goal of treatment is usually to reduce intestinal spasms, which is


best achieved by maintaining a high-fiber diet (which consists of
vegetables, fruits, and whole grains) and drinking plenty of fluids. An
increased bulk in the large intestine reduces spasms, which in turn
decreases the pressure on the walls of the large intestine.

Since this patient has no fever or signs of infection antibiotics,


colonoscopy and surgery are not necessary at this time.
An elderly patient with long standing rheumatoid arthritis who has been
taking medications for it, develops tinnitus. What is the most likely cause?

a) Aspirin toxicity
b) Meneire disease
c) Labyrinthitis
d) Acoustic neuroma
The correct answer is A

Explanation
Rheumatoid arthritis (RA) is a chronic autoimmune disease, producing
damage mediated by cytokines, chemokines, and metalloproteases.
Peripheral joints (eg, wrists, metacarpophalangeal joints) are
symmetrically inflamed, often resulting in progressive destruction of
articular structures, usually accompanied by systemic symptoms.
Diagnosis requires specific clinical, laboratory, and radiologic criteria.

If a patient takes aspirin daily (for RA, for example), it can cause or
worsen tinnitus. For this reason, aspirin is no longer used for RA, as
effective doses are often toxic.
An 18 year old male presents with a 2-week history of cough, headache,
malaise, and fever. Physical examination shows a patient in good general
condition with no abnormal findings on auscultation. Gram stain of the
sputum is negative and the white blood cell count is normal. Chest x-ray
shows bilateral pneumonia in the lower lobes without pleural effusion.
Which one of the following is the most likely cause?

a) Haemophilus influenzae
b) Staphylococcus aureus
c) Mycoplasma pneumoniae
d) Klebsiella pneumoniae
e) Streptococcus pneumoniae
The correct answer is C

Explanation
Walking pneumonia (a.k.a. mycoplasma pneumonia or atypical pneumonia) is
a term used to describe a mild case of pneumonia. It is a lung infection
that is caused by the mycoplasma pneumoniae organism. Walking pneumonia
is different from other types of pneumonia in that, even if untreated,
walking pneumonia would not cause a person to be confined to a bed or
require a hospital stay.

Signs and symptoms of walking pneumonia usually include a severe cough,


fever, abdominal pain, headaches, and chills. Mycoplasma pneumoniae, the
organism that causes walking pneumonia, can be contagious if contact
with the infected person is prolonged.

Walking pneumonia is transmitted through airborne droplets, which are


dispersed when a person talks, sneezes, coughs or laughs. The onset of
walking pneumonia takes approximately 1 to 3 weeks, beginning gradually,
with the earliest sign being a decrease in energy. Soon thereafter,
cold-like symptoms begin. You will be able to distinguish the difference
between walking pneumonia and a common cold because walking pneumonia
will get worse over 2 weeks.

Which is following statement about PSA is true?

a) Every man past the age of 50 should have an annual PSA test as a
primary screen for prostate cancer
b) PSA is best used to follow disease progression or recurrence
post-operatively
c) PSA is a useless test
d) Increased complex PSA to total PSA ratio favors BPH over prostate cancer
The correct answer is B

Explanation
Prostate-specific antigen (PSA), is a substance that is usually elevated
in men with prostate cancer. PSA levels can be misleading: they can be
normal when prostate cancer is present or elevated when prostate cancer
is absent. PSA levels normally increase with age, but cancer increases
the age-related change. Also, PSA levels can be slightly elevated in men
with disorders other than prostate cancer (such as BPH or prostatitis)
and in men who have undergone procedures involving the urinary tract
within the previous 2 days.
That is why a PSA level is not the best screening test, but is best used
to follow disease progression or recurrence post-operatively.
Which of the drugs cause amnesia as a side effect?

a) Triazolam
b) Diazepam
c) Phenytoin
d) Imipramine
The correct answer is A

Explanation
Triazolam is a member of the benzodiazepine family of drugs. Triazolam
is used for the short-term (generally seven to 10 days) treatment of
insomnia. Amnesia of varying severity and paradoxical reactions have
been reported following therapeutic doses of Triazolam.
A 63 year old male, with a history of arrhythmias, who is on
antiarrhythmic drug therapy, was identified with twisted QRS complex and
QT interval prolongation. What is the most preferred diagnostic test?

a) Magnetic Resonance Imaging


b) Electrocardiogram
c) Chest X-ray
d) Angiography
e) Electrophysiological testing
The correct answer is B

Explanation
Answer: B ? Electrocardiogram is the most preferred diagnostic test for
detection and evaluation of Torsades de Pointes. Electrocardiogram helps
in identification of the twist of QRS complex and prolongation of QT
segment. It has series of wave forms which denote the parts of heart and
their functionality. P wave reflects the atrial depolarization; PR wave
represents the spread of impulse through purkunjee fibers, QRS complex
reflects ventricular depolarization and QT interval widening represents
ventricular repolarization.

A. Magnetic Resonance Imaging is not correct because it gives two


dimensional image of the heart.
C. Chest radiographs should be performed to rule out structural heart
disease if any clinical suggestion is present (after EKG is done)
D. Angiography is used to assess the condition and functioning of blood
vessels but not the heart.
E. Electrophysiological testing is used to locate the ectopic foci and
to assess the therapeutic response to antiarrhythmiatic drugs.

A 81-year-old retired electrical engineer whose wife is a diabetic was


experimenting with his wife‟s glucose meter and found that his glucose
level was 198 mg/dL. He used her strips and lancets, and started his own
log. After a week, he brings the log to you. His premeal glucose levels
range from about 150 mg/dL to 250 mg/dL. A review of his medications
shows none that would be likely to increase his glucose level. A
physical examination does not suggest glucose intolerance secondary to a
process other than diabetes. His hemoglobin A1c is 9.0%.

Additional laboratory studies should be performed before prescribing


which one of the following for this patient?
a) Insulin NPH (Humulin N)
b) Metformin (Glucophage)
c) Glimepiride (Amaryl)
d) Miglitol (Glyset)
The correct answer is B

Explanation
The use of insulin therapy can be as appropriate in the older adult with
diabetes as in younger individuals. In patients with reduced muscle
mass, such as the elderly (especially those older than 80 years of age),
using serum creatinine concentration to estimate the glomeruler
filtration rate may be misleading, and creatinine clearance should be
determined. If creatinine clearance is < 70 mL/min, metformin should not
be prescribed. The other two oral antidiabetic agents are safe to use in
an elderly patient without other initial laboratory data.

A 60 year old male indicates that he occasionally brings up what appears


to be undigested food long after his meal. He also admits that he
sometimes chokes on food, and that his wife says he has bad breath. The
most likely diagnosis is

a) Achalasia
b) Esophageal reflux
c) Cancer of esophagus
d) Zenker‟s diverticulum
e) Large cervical bone spur
The correct answer is D

Explanation
The combination of halitosis, late regurgitation of undigested food, and
choking suggests Zenker‟s diverticulum. Patients may also have dysphagia
and weight loss. The diagnosis is usually made with a barium swallow.
The treatment is surgical.

A 60 year old man presents to your office complaining of low back pain.

Over the past 2 years, the patient has been having progressively
worsening pain while walking. On walking, he begins to experience low
back pain as well as numbness in his thighs and calves. These symptoms
persist until the patient sits down, bends his trunk and rests for a few
minutes. He has no history of trauma. His past medical history is
significant for essential hypertension diagnosed 10 years ago for which
he takes losartan and hydrochlorothiazide.

On examination, Temp is 36.60C, BP is 130/75 mmHg, pulse is 80/min and


respiratory rate is 14/min. Neurologic exam reveals no weakness of his
lower limbs, but there is decreased sensation in the S1 distribution
with diminished ankle reflexes bilaterally. The straight leg raise test
is negative bilaterally. Lower extremities pulsations are strong and
regular.

Which of the following is the best diagnostic test to confirm his


suspected diagnosis‟
a) Ankle Brachial Index (ABI) measurement
b) CT Scan of the lumbar spine
c) Duplex ultrasound of the lower limbs
d) MRI of the lumbar spine
e) X-Ray of the lumbar spine
The correct answer is D

Explanation
The most likely diagnosis for the patient‟s condition is neurogenic
claudication due to degenerative spinal stenosis. It is important to
take a detailed history from the patient as the claudication that is
abolished when the patient stops walking is characteristic of
vasculogenic claudication; however when it is relieved upon sitting and
bending over this is almost pathognomonic for lumbar spinal stenosis.
With normal peripheral pulses, this patient is unlikely to have
peripheral vascular disease significant enough to cause his presenting
symptoms.

The most sensitive and specific test to confirm lumbar spinal stenosis
is MRI of the lumbar spine.

CT scan can suggest lumbar spinal stenosis; however, MRI is the gold
standard for diagnosis; it can delineate the spinal cord impingement in
the stenotic vertebral canal clearly.

X-Ray of the spine is the most appropriate initial imaging to exclude


other vertebral pathologies.

ABI and Duplex of the lower limbs would be the best diagnostic testing
for suspected peripheral vascular disease.

A 65-year-old white female presents with symptoms and signs of


intermittent claudication. She has a history of increasing angina
pectoris leading to coronary artery bypass surgery 3 years ago. She has
generally done well since the surgery, and you have provided appropriate
medical management for postoperative coronary artery disease. She has
reduced her cigarette smoking, but still smokes and leads a sedentary
lifestyle.

In addition to emphasizing smoking cessation and recommending an


exercise program, which one of the following would be most effective for
managing this patient‟s intermittent claudication?

a) Cilostazol (Pletal)
b) Clopidogrel (Plavix)
c) Pentoxifylline (Trental)
d) Ginkgo biloba
e) Aspirin
The correct answer is A

Explanation
Of the medications currently available for the treatment of intermittent
claudication, cilostazol is the most effective. A meta-analysis of eight
randomized, placebo-controlled trials of cilostazol for moderate to
severe claudication found that 100 mg of the drug twice daily increased
maximal and pain-free walking distances by 50% and 67%, respectively.
Clopidogrel may be marginally more effective than aspirin as an
antiplatelet agent, but is more expensive. Neither pentoxifylline nor
ginkgo biloba, a non?FDA-approved dietary supplement with antiplatelet,
vasodilating, and antioxidant activity, has been shown to be more
effective than placebo. At best, they would be a weak intervention for
peripheral artery disease.
A 70-year-old retired farmer presents with an angulated right knee and a
painful hip. He asks you about the possibility of ?getting a new knee,?
although he is not eager to do so.

You would advise him that the major indication for knee replacement is

a) severe joint pain at rest


b) marked joint space narrowing seen on radiologic studies
c) destruction and loss of motion of the contralateral joint
d) an acutely infected joint
The correct answer is A

Explanation
The major indication for joint replacement is severe joint pain, usually
pain at rest. Loss of joint function and radiographic evidence of severe
destruction of the joint may also be considered in the decision. The
appearance of the joint and the status of the contralateral joint may be
minor considerations. Surgical insertion of a foreign body into an
infected joint is contraindicated.
A 31-year-old healthy female is admitted to the hospital from the
emergency department after presenting with aching in her right shoulder
and swelling in the ipsilateral forearm and hand. The only precipitating
event that she can recall is digging strenuously in the back yard to put
in a new garden. Ultrasonography is remarkable for a thrombus in the
axillosubclavian vein. She has no prior history of clotting, takes no
medications, and has no previous history of medical or surgical
procedures involving this extremity.

The most likely etiology for this patient‟s condition is

a) a hypercoagulable state
b) a compressive anomaly in the thoracic outlet
c) use of injection drugs
d) Budd-Chiari syndrome
The correct answer is B

Explanation
Thrombosis of the upper extremity accounts for about 10% of all venous
thromboembolism (VTE) cases. However, axillosubclavian vein thrombosis
(ASVT) is becoming more frequent with the increased use of indwelling
subclavian vein catheters. Spontaneous ASVT (not catheter related) is
seen most commonly in young, healthy individuals. The most common
associated etiologic factor is the presence of a compressive anomaly in
the thoracic outlet. These anomalies are often bilateral, and the other
upper extremity at similar risk for thrombosis.

While a hypercoagulable state also may contribute to the thrombosis, it


is much less common. Budd-Chiari syndrome refers to thrombosis in the
intrahepatic, suprahepatic, or hepatic veins. It is not commonly
associated with spontaneous upper-extremity thrombosis.

A 33-year-old Native Canadian female who lives on a Navajo reservation


is hospitalized with a history of fever, myalgia, and increasing
tachypnea progressing to respiratory failure. Laboratory evaluation
reveals thrombocytopenia, leukocytosis with a left shift, circulating
immunoblasts, and hemoconcentration.

This patient has an infectious disease spread by

a) Fleas
b) Humans
c) Ticks
d) Rodents
e) Lice
The correct answer is D

Explanation
The tetrad of thrombocytopenia, left-shifted leukocytosis, circulating
immunoblasts, and hemoconcentration is seldom seen in viral infections
other than hantavirus pulmonary syndrome. It is spread by rodent excreta
and saliva. There is no human-to-human transmission.

A 23-year-old female sees you with a complaint of intermittent


“irregular” heartbeats that occur once every week or two, but do not
cause her to feel lightheaded or fatigued. They last only a few seconds
and resolve spontaneously. She has never passed out, had chest pain, or
had difficulty with exertion. She is otherwise healthy, and a physical
examination is normal. Which one of the following cardiac studies should
be ordered initially?

a) 24-hour ambulatory EKG monitoring (Holter monitor)


b) 30-day continuous closed-loop event recording
c) Echocardiography
d) An EKG
e) Electrophysiologic studies
The correct answer is D

Explanation
The symptom of an increased or abnormal sensation of one‟s heartbeat is
referred to as ?palpitations.? This condition is common to primary care,
but is often benign. Commonly, these sensations have their basis in
anxiety or panic. However, about 50% of those who complain of
palpitations will be found to have a diagnosable cardiac condition. It
is recommended to start the evaluation for cardiac causes with an EKG,
which will assess the baseline rhythm and screen for signs of chamber
enlargement, previous myocardial infarction, conduction disturbances,
and a prolonged QT interval.
A 28 year old white female complains of fatigue, stiffness, and a
diffuse aching in her neck, shoulders, and back for several months. She
is not sleeping well. A thorough physical examination is unremarkable
except for tenderness over the upper trapezius, second costochondral
junctions, lateral epicondyles, and medial knees. A CBC, erythrocyte
sedimentation rate, and rheumatoid factor are unremarkable. Which one of
the following is the most appropriate management at this time?

a) Prescribe amitriptyline (Elavil), 10-25 mg every night at bedtime,


along with exercise aimed at improving her overall level of fitness
b) Prescribe a 2-week course of a short-acting, sedative-hypnotic such
as triazolam (Halcion), 0.25 mg every night at bedtime
c) Schedule electromyography and nerve conduction studies to rule out
neuropathies
d) Refer the patient to a rheumatologist for suspected seronegative
rheumatoid arthritis
The correct answer is A

Explanation
This patient has a classic presentation of fibrositis fibromyalgia
syndrome. In addition to small bedtime doses of amitriptyline or
cyclobenzaprine, symptoms are improved by an increase in physical
fitness, stress reduction, regulation of sleep schedules, and
reassurance. Opiate analgesics and sedative-hypnotics are not
recommended for long-term treatment of this disorder.
A 24 year old white female presents to the office with a 6 month history
of abdominal pain. A physical examination, including pelvic and rectal
examinations, is normal. Which one of the following would indicate a
need for further evaluation?

a) Relief of symptoms with defecation


b) Changes in stool consistency from loose and watery to constipation
c) Passage of mucus with bowel movements
d) Abdominal bloating
e) Worsening symptoms at night
The correct answer is E

Explanation
Irritable bowel syndrome (IBS) is a benign, chronic symptom complex of
altered bowel habits and abdominal pain. It is the most common
functional disorder of the gastrointestinal tract. The presence of
nocturnal symptoms is a red flag which should alert the physician to an
alternate diagnosis and may require further evaluation. The other
symptoms listed are Rome I and II criteria for diagnosing irritable
bowel syndrome.
A 66 year old female patient complains of right upper quadrant pain. She
is febrile. Physical exam shows slight abdominal tenderness, what
investigation is appropriate to confirm the diagnosis‟

a) Biopsy
b) Barium study
c) Peritoneal Aspiration
d) Rectosigmoidoscopy
e) Ultrasound
The correct answer is E

Explanation
Cholelithiasis is the presence of one or more calculi (gallstones) in
the gallbladder. 20% of people > 65 years have gallstones, and most
disorders of the extrahepatic biliary tract arise from gallstones.
Gallstones may be asymptomatic or cause biliary colic.

Pain may occur in the right upper quadrant but is often poorly localized
or occurs elsewhere in the abdomen, particularly among diabetics and the
elderly. Mild right upper quadrant or epigastric tenderness may be
present, but peritoneal findings are absent, and laboratory tests are
normal.

Diagnosis is usually based on ultrasound. If cholelithiasis causes


symptoms or complications, cholecystectomy becomes necessary.
67 year old smoker, with a 40 pack year history of smoking cigarettes,
comes to your office for an annual check up. He is known to have
uncontrolled hypertension, with a BP today of 159/89mmhg. Today he tells
you that he is becoming short of breath, more so than usual. He also can
manage about 2 blocks before he needs to sit at a bench, before he can
resume. He tells you that his calf muscles begin to ache at the end of
the two blocks. You take an ankle-brachial pressure index (ABPI)
measurement. What do you expect his ABPI value to be between?

a) >1.5

b) 1

c) 1.0-0.7

d) 0.7-0.4

e) <0.35
The correct answer is D

Explanation
This individual is experiencing claudicating pain, which is pain that
begins upon exertion and ends with rest. A normal ABPI is anything
greater than 0.9. You should expect the ABPI in someone with
claudicating pain to lie between 0.7-0.4. When the ABPI is below 0.4,
rest pain is a key sign to be aware of, and would indicate urgent
vascular surgery to alleviate the ischemia.
Which of the following is not given in an acute attack of asthma?

a) Ipratropium bromide
b) Sodium cromolyn
c) Theophylline
d) Salmeterol
e) Steroid
The correct answer is B

Explanation
Asthma is a disease of diffuse airway inflammation caused by a variety
of triggering stimuli resulting in partially or completely reversible
bronchoconstriction. Symptoms and signs include dyspnea, chest
tightness, and wheezing.

The diagnosis is based on history, physical examination, and pulmonary


function tests. Treatment involves controlling triggering factors and
drug therapy, most commonly with inhaled ?-agonists and inhaled
corticosteroids. Anticholinergic medications, such as ipratropium
bromide provide addition benefit when used in combination with
beta-agonists in those with moderate or severe symptoms. Cromolyn is
used more in the prophylaxis of asthma.
A 49 year old white female comes to your office complaining of painful,
cold finger tips which turn white when she hanging out her laundry.
While there is no approved treatment for this condition at this time,
which one of the following drugs has been shown to be useful?
a) Propranolol (Inderal)
b) Nifedipine (Procardia)
c) Ergotamine/caffeine (Cafergot)
d) Methysergide (Sansert)
The correct answer is B

Explanation
At present there is no approved treatment for Raynaud‟s disease.
However, patients with this disorder reportedly experience subjective
symptomatic improvement with calcium channel antagonists. Nifedipine is
the calcium channel blocker of choice in patients with Raynaud‟s
disease. Beta-Blockers can produce arterial insufficiency of the Raynaud
type, so propranolol and atenolol would be contraindicated. Drugs such
as ergotamine preparations and methysergide can produce cold
sensitivity, and should therefore be avoided in patients with Raynaud‟s
disease.
A 70 year old man with a history of coronary heart disease and
hypertension, presents with a 3 week history of increasing shortness of
breath. A chest x-ray shows a moderate sized right pleural effusion.
Thoracentesis shows the protein in the pleura/serum ratio to be <0.5,
consistent with a transudate. These results are compatible with a
pleural effusion caused by which one of the following?

a) Nephrotic syndrome
b) Congestive heart failure
c) Adenocarcinoma of the lung
d) Hepatic vein thrombosis
e) Cirrhosis of the liver
The correct answer is B

Explanation
Pleural effusions are accumulations of fluid within the pleural space.
They have multiple causes and usually are classified as transudates or
exudates. Detection is by physical examination and chest x-ray;
thoracentesis and pleural fluid analysis are often required to determine
cause. Asymptomatic transudates require no treatment. Symptomatic
transudates and almost all exudates require thoracentesis, chest tube
drainage, pleurodesis, and/or pleurectomy.

Transudative effusions are caused by some combination of increased


hydrostatic pressure and decreased oncotic pressure in the pulmonary or
systemic circulation. Heart failure is the most common cause, followed
by cirrhosis with ascites and hypoalbuminemia, usually from the
nephrotic syndrome.

Exudative effusions are caused by local processes leading to increased


capillary permeability resulting in exudation of fluid, protein, cells,
and other serum constituents. Causes are numerous, the most common being
pneumonia, malignancy, pulmonary embolism, viral infection, and TB.
A 43 year old black male is hospitalized for treatment of a deep venous
thrombosis of the right leg extending to the mid-thigh. His baseline
platelet count on admission in 250,000/mm3 (N 150,000-450,000). On the
day of admission he is started on heparin and warfarin (Coumadin).

He shows steady clinical improvement, and discharge is planned for the


fifth hospital day. On the morning he is to be discharged, laboratory
results show therapeutic values for both prothrombin time (PT) and
partial thromboplastin time (PTT), but his platelet count is 45,000/mm3.
Which one of the following would be most appropriate at this time?

a) Stop the heparin but continue the warfarin


b) Stop the warfarin but continue the heparin
c) Stop the heparin and warfarin and substitute a low-molecular-weight heparin
d) Stop the heparin and warfarin and start a non-heparin anticoagulant
e) Discharge the patient, as clinical improvement has occurred
The correct answer is D

Explanation
Heparin-induced thrombocytopenia (HIT) results from the formation of
heparin-dependent IgG antibody. Characteristically developing 5-10 days
after the initiation of heparin by any route, it is manifested by a fall
in the platelet count to less than 50% of the baseline or to an absolute
level of < 50,000/mm3, mandating discontinuation of heparin. HIT is best
envisioned as a state of excessive and unrestrained thrombin generation,
and in this prothrombotic mileu warfarin treatment may intensify this
state and must also be stopped. Direct thrombin inhibitors have been
approved by the FDA for use as anticoagulants in patients with HIT.

The primary treatment of HIT is to stop heparin and warfarin and


substitute a non-heparin anticoagulant. A hypercoagulable state exists
for several days after heparin is stopped, mandating continuation of
monitoring and further antithrombotic measures. Although
low-molecular-weight heparins appear less likely to stimulate HIT-IgG
antibody formation, they are also contraindicated, since HIT-IgG
crossreacts with all low-molecular-weight heparins.
A 35-year-old male who works as an EMT has recently developed deep,
severe, continuous headaches behind his left eye. The headaches occur
daily around 10:00 a.m. and last 1?2 hours. He notes that the headaches
bother him less when he remains active. When the headaches occur he also
has tearing from the left eye and a nasal discharge from the left naris.
He has found that using oxygen gives him relief. A physical examination
reveals no significant findings.

Which one of the following would be most appropriate at this time?

a) MRI
b) Carbamazepine (Tegretol)
c) Neurologic consultation
d) Avoiding triptan medications such as sumatriptan (Imitrex)
e) Daily therapy with verapamil (Isoptin)
The correct answer is E

Explanation
Cluster headaches are repetitive headaches that occur for weeks or
months at a time. They are more common in men between the ages of 30 and
50. The headaches begin without warning and reach a crescendo within
minutes. Most times the headaches are located behind the eye or at the
temple, and are associated with ipsilateral lacrimation, redness of the
eye, nasal stuffiness, sweating, pallor, and Horner‟s syndrome. Patients
note that they are sensitive to alcohol. They often prefer to remain
active rather than rest in a dark, quiet room as someone with a migraine
would likely do. A typical attack will last from 30 minutes to 3 hours.
It is common for the pain to recur each day at the same time. These
attacks occur 1?3 times a day over a 4- to 8-week period. The patient
will often remain pain-free for 6 months to a year.

A cluster headache can be distinguished from a migraine, a tension-type


headache, and a brain tumor by its presentation. Headaches associated
with a brain tumor are often accompanied by nausea, vomiting, and
worsening of the pain (which is usually bifrontal) with change in body
position. There are usually positive findings on neurologic examination,
as well as a change from the previous headache pattern. Migraines are
more common in women and may be associated with an aura. They are
throbbing in nature and accompanied by nausea, photophobia, and
phonophobia. These patients are usually much more comfortable in the
dark. Both cluster and migraine headaches will respond to abortive
therapy with triptan medications.

Prophylaxis for migraine is normally begun if there are four or more


attacks within a month. In contrast, once the diagnosis of cluster
headache is made, it is important to initiate both abortive and
prophylactic therapy right away. Abortive therapy can be with 100%
oxygen, triptans, octreotide, or dihydroergotamine. A number of
prophylactic medications are available, including verapamil, lithium,
prednisone (short-term use only), ergotamine, cyproheptadine, and
indomethacin.
A 33-year-old white female presents with tremor and a history of weight
loss. On examination she is found to have mild, regular tachycardia and
exophthalmos. Laboratory tests confirm hyperthyroidism. Which one of the
following treatments has been found to potentially worsen ophthalmopathy?

a) Radioactive iodine
b) Propylthiouracil
c) Methimazole (Tapazole)
d) Thyroid hormone replacement plus propylthiouracil
e) Thyroidectomy
The correct answer is A

Explanation
The ophthalmopathy of Graves‟ disease may initially flare and worsen
when treated with radioactive iodine. Antithyroid drugs, including
carbimazole, propylthiouracil, and methimazole, are not associated with
this problem. The addition of thyroid hormone to these drugs at
suppressive doses has not shown any clear benefit over titration of the
antithyroid drug, and relapse rates are similar. Thyroid surgery in the
controlled patient has not been significantly associated with this problem.
You are performing pulmonary function tests on a 80 year old male with
persistent cough. Which one of the following would occur normally with age?

a) Fall in vital capacity


b) Fall in FEV1/FVC
c) Fall in arterial p02
d) Increase in arterial pCO2
e) Decrease in arterial pH
The correct answer is B

Explanation
Pulmonary function tests (PFT‟s) provide measures of flow rates, lung
volumes, gas exchange, and respiratory muscle function. Basic pulmonary
function tests available in the ambulatory setting include spirometry
and pulse oximetry; these tests provide physiologic measures of
pulmonary function and can be used to quickly narrow a differential
diagnosis and suggest a subsequent strategy of additional testing or
therapy.

FEV1, FVC, FEF25-75% and PEF increase and FEV1/FVC% decrease with age
until about 20 years old in females and 25 years in males. After this,
all indices gradually fall, although the precise rate of decline is
probably masked due to the complex interrelationship between age and
height. The fall in FEV1/FVC% with age in adults is due to the greater
decline in FEV1 than FVC.
A 68 year old male with supra-ventricular tachycardia was treated with
Quinidine. Which of the following medications can cause life-threatening
arrhythmias, when it is given concomitantly with quinidine?

a) Phenytoin
b) Phenobarbital
c) Amiodarone
d) Atenolol
e) Ramipril
The correct answer is C

Explanation
With the exact mechanism not known, amiodarone (Cordarone),
another type of antiarrhythmic drug, may decrease removal of quinidine
by the kidneys or liver giving rise to elevated quinidine blood levels,
which may result in life-threatening arrhythmias, including torsades de
pointes. It is important, therefore, to decrease quinidine doses when it
is given concomitantly with amiodarone.

A. and B. Removal of quinidine by the liver is accelerated by


phenobarbital, phenytoin (Dilantin), and rifampin (Rifamate), requiring
an increase in quinidine dose.
D. Atenolol is a beta blocking antihypertensive, which does not cause
any toxic effects if given concomitantly with quinidine.
E. Ramipril is an anti-hypertensive, which produces its action by
blocking Angiotensin converting enzyme given in combination with
quinidine does not produce any toxic effect.

A 60-year-old female presents with a 2-day history of nausea and


diarrhea. She now experiences dizziness when standing. She reports that
she has been unable to hold down more than sips of fluid but has been
able to take her chronic medications, which include glipizide
(Glucotrol), hydrochlorothiazide, lisinopril (Prinivil, Zestril),
atorvastatin (Lipitor), atenolol (Tenormin), and long-acting diltiazem
(Cardiazem LA). She has diabetes mellitus, essential hypertension,
hypercholesterolemia, and chronic renal insufficiency (baseline
creatinine 1.5 mg/dL).

Laboratory results are significant for a potassium level of 3.4 mmol/L


(N 3.5-5.1), a bicarbonate level of 19 mEq/L (N 22-26), a BUN of 78
mg/dL (N 8-25), and a serum creatinine level of 3.9 mg/dL (N 0.6-1.5).
Physical examination reveals her to be awake but groggy, with a blood
pressure of 88/50 mm Hg, a pulse rate of 58 beats/min, flat neck veins,
clear lung fields, regular heart sounds, and no peripheral edema.

You decide to admit the patient and temporarily stop all of her oral
medications. A Foley catheter is placed and a small amount of clear
urine with a high specific gravity obtained.

Which one of the following is the most appropriate initial treatment to


correct this patient‟s acute renal failure?

a) Furosemide (Lasix), 80 mg intravenously in one dose


b) Nitroglycerin intravenously, titrated as necessary
c) Normal saline intravenously
d) Hemodialysis
e) Percutaneous catheterization of the renal pelvis
The correct answer is C

Explanation
The patient presents with acute renal failure, probably caused by a
prerenal mechanism. It is likely that she became dehydrated and was
unable to maintain an adequate cardiac output because of the multiple
antihypertensive medications she takes.

Renal parenchymal ischemia leading to intrinsic renal compromise is also


possible.

The findings from the Foley catheterization would rule out most causes
of postrenal obstruction.

Dialysis would be indicated only for chronic uremic symptoms, severe


hyperkalemia, fluid overload, or more severe acidosis. In this patient,
reestablishing intravascular volume with a crystalloid fluid is
imperative and is usually very effective for reversing renal
abnormalities. There are no indications for the other therapies listed.

Which one of the following tests is most useful for the initial workup
of suspected Alzheimer‟s disease?

a) Carotid Doppler ultrasonography


b) An erythrocyte sedimentation rate
c) A cardiac stress test
d) A TSH level
e) A lumbar puncture with measurement of cerebrospinal fluid pressure
The correct answer is D

Explanation
Occult hyper- or hypothyroidism can imitate the symptoms of Alzheimer‟s
disease, especially in the elderly, and thus should be considered in the
initial workup.
Cardiac stress testing, carotid Doppler ultrasonography, and an
erythrocyte sedimentation rate are part of the routine evaluation if
normal pressure hydrocephalus is suspected.
Neither the cerebral angiogram nor the cardiac stress test is useful for
the initial evaluation of Alzheimer‟s disease.
A 55-year-old male who has a long history of marginally-controlled
hypertension presents with gradually increasing shortness of breath and
reduced exercise tolerance. His physical examination is normal except
for a blood pressure of 140/90 mm Hg, bilateral basilar rales, and trace
pitting edema. Which one of the following ancillary studies would be the
preferred diagnostic tool for evaluating this patient?

a) 12-lead electrocardiography
b) Posteroanterior and lateral chest radiographs
c) 2-dimensional echocardiography with Doppler
d) Radionuclide ventriculography
e) Cardiac MRI
The correct answer is C

Explanation
The most useful diagnostic tool for evaluating patients with heart
failure is two-dimensional echocardiography with Doppler to assess left
ventricular ejection fraction (LVEF), left ventricular size, ventricular
compliance, wall thickness, and valve function. The test should be
performed during the initial evaluation. Radionuclide ventriculography
can be used to assess LVEF and volumes, and MRI or CT also may provide
information in selected patients. Chest radiography (posteroanterior and
lateral) and 12-lead electrocardiography should be performed in all
patients presenting with heart failure, but should not be used as the
primary basis for determining which abnormalities are responsible for
the heart failure.
A 66-year-old female presents for a preoperative evaluation prior to
elective podiatric surgery. She has no complaints other than her foot
problem, and says she feels well. On examination she has an irregularly
irregular heart rate with a 2/6 holosystolic murmur. An EKG reveals
atrial fibrillation with a rate of 110 beats/min. Echocardiography shows
mild to moderate mitral regurgitation and a dilated left atrium, but is
otherwise normal. Which one of the following is the most appropriate
initial treatment for this patient?

a) Digoxin, 0.125 mg/day


b) Quinidine gluconate, 324 mg 3 times daily
c) Atenolol (Tenormin), 25 mg/day
d) Sustained-release nifedipine (Adalat, Procardia), 180 mg/day
e) Unfractionated heparin sodium, 5000 units subcutaneously 3 times daily
The correct answer is C

Explanation
The primary goals of atrial fibrillation treatment are rate control and
prevention of thromboembolism. Guidelines recommend rate control with
atenolol, metoprolol, diltiazem, or verapamil (SOR A).

Digoxin does not control the heart rate with stress. Quinidine is
proarrhythmic and does not control the heart rate. Nifedipine does not
control the heart rate, and heparin does not provide adequate
anticoagulation or control the heart rate.

A 26 year old man is admitted to the hospital for lethargy and


confusion. The patient has a 5 year history of severe hypertension that
has been refractory to medical therapy. Over the past few days, the man
has become increasingly lethargic and confused. On the day of admission,
he was found in his home, lying on the floor and non-responsive. His
roommate called the ambulance, and the patient was brought to the ER.

Laboratory values are as follows:

Serum sodium 112 mmol/L, Normal (135?147 mmol/L)


Serum potassium 5.1 mmol/L, N (3.5?5 mmol/L)
Serum osmolality 230 mmol/kg, N (280?300 mmol/kg)

Which of the following substances is most likely mediating the


hyponatremia?

a) Aldosterone
b) Antidiuretic hormone (ADH)
c) Cortisol
d) Somatostatin
e) Thyroid stimulating hormone (TSH)
The correct answer is B

Explanation
Hyponatremia is the most common electrolyte abnormality encountered in
the hospital. This patient is clearly hyponatremic. ADH works at the
collecting ducts of the nephron in the kidney to bring back water from
the urine into the bloodstream. This causes both the serum osmolality
and serum sodium to drop as the serum becomes more dilute.

Recall that aldosterone brings back sodium from the urine into the
bloodstream and kicks out serum potassium into the urine. Therefore,
high levels of aldosterone would present as hypokalemia with hypernatremia.
Which one of the following is considered a contraindication to the use
of Beta-blockers for congestive heart failure?

a) Mild asthma
b) Symptomatic heart block
c) New York Heart Association (NYHA) Class III heart failure
d) NYHA Class I heart failure in a patient with a history of a previous
myocardial infarction
e) An ejection fraction <30%
The correct answer is B

Explanation
According to several randomized, controlled trials, mortality rates are
improved in patients with heart failure, who receive Beta-blockers in
addition to diuretics, ACE inhibitors, and occasionally, digoxin.
Contraindications to Beta-blocker use include hemodynamic instability,
heart block, bradycardia, and severe asthma. beta-blockers may be tried
in patients with mild asthma or COPD as long as they are monitored for
potential exacerbations. Beta-blocker-use has been shown to be effective
in patients with NYHA Class II or III heart failure. There is no
absolute threshold ejection fraction. Beta-blockers have also been shown
to decrease mortality in patients with a previous history of myocardial
infarction, regardless of their NYHA classification.
A 25 year old woman presents with intense itching due to a rash that
started yesterday. She denies any insect bites or recent exposure to
anything new. You believe this is a self-limited case of pruritus and
decide to treat symptomatically. Which of the following is an
appropriate choice?

a) Permethrin
b) Lindane
c) Dexamethasone
d) Hydroxyzine
e) Prednisone
The correct answer is D

Explanation
Pruritus (itching) can be a symptom of primary skin diseases or of
systemic disease. Skin diseases notorious for causing intense pruritus
include scabies, pediculosis, insect bites, urticaria, atopic and
contact dermatitis, lichen planus, miliaria, and dermatitis herpetiformis.

Topical agents may help localized pruritus. Options include


camphor/menthol lotions and doxepin. Topical diphenhydramine and doxepin
should be avoided because they may sensitize the skin. Systemic agents
are indicated for generalized pruritus or local pruritus resistant to
topical agents. Antihistamines, most notably hydroxyzine are effective
and most commonly used.

Permethrin and lindane are used in the treatment of scabies. Strong


steroids such as prednisone and betamethasone are not required in a
self-limited case of pruritus.
Which medication works well to discourage people with alcohol abuse from
drinking?

a) Abilify
b) Fluoxetine
c) Disulfiram
d) Clozapine
The correct answer is C

Explanation
In the treatment of alcohol abuse, a rehabilitation program is often the
best approach. Rehabilitation programs combine medical supervision and
psychotherapy, including one-on-one and group therapy.

Alcoholics Anonymous (AA) has benefited alcoholics more than any other
approach. The patient must find an AA group in which he is comfortable.
AA provides the patient with nondrinking friends who are always
available and a nondrinking environment in which to socialize.

Drug therapy should be used in combination with psychotherapy.


Disulfiram interferes with the metabolism of acetaldehyde (an
intermediary product in the oxidation of alcohol) so that acetaldehyde
accumulates. Drinking alcohol within 12 hours of taking disulfiram
produces facial flushing in 5 to 15 minutes, then intense vasodilation
of the face and neck with suffusion of the conjunctivae, throbbing
headache, tachycardia, hyperpnea, and sweating.

With high doses of alcohol, nausea and vomiting may follow in 30 to 60


minutes and may lead to hypotension, dizziness, and sometimes fainting
and collapse. The reaction can last up to 3 hours. Few patients risk
ingesting alcohol while taking disulfiram because of the intense discomfort.
A 72 year old man with a history of chronic hypertension presents with a
complaint of chest pain radiating to the left arm along with shortness
of breath, nausea, vomiting, palpitations and sweating. In regards to
this patient's presentation, what condition is correctly matched with
its serum 'marker'?

a) Myocardial infarction and troponin levels


b) Angina pectoris and streptokinase levels
c) Lymphoma and lactic dehydrogenase levels
d) Arrhythmias and myoglobin levels
e) Myocardial infarction and amylase levels
The correct answer is A

Explanation
Answer: A ? This patient is clearly suffering from myocaridal infarction
(MI). Current clinical practice is to measure two different cardiac
enzymes when an MI is suspected: creatine kinase (CK), and troponin (T).
CK is released into the bloodstream 4 to 6 hours after heart cell damage
occurs, and peak blood levels of CK are seen after 24 hours. Elevated CK
levels usually, but not always, indicate heart muscle damage. T is
released into the bloodstream 2 to 6 hours after heart cell damage, and
blood levels peak in 12 to 26 hours. Troponin is the preferred biomarker
for diagnosis. Troponins have the greatest sensitivity and specificity
in detecting MI. The test result is both diagnostic as well as
prognostic of outcome.

B. Streptokinase lis a thrombolytic. It's a nonfibrin specific agent


with a half-life of 23 min that acts with plasminogen to convert
plasminogen to plasmin. Besides, this is not a typical presentation of
angina pectoris
C. This presentation is not typical for lymphoma.
D. Myoglobin, a low-molecular-weight heme protein found in cardiac and
skeletal muscle, is released more rapidly from infarcted myocardium than
troponin and CK-MB and may be detected as early as 2 hours after MI.
Myoglobin levels rise early in the course of MI. (not connected to
arrhythmias)
E. Amylase is is elevated in pancreas diseases but also
in amylase-producing small-cell lung cancer.

Which one of the following is true concerning community-acquired pneumonia?

a) The absence of a pulmonary infiltrate on a chest film rules out


pneumonia
b) Intravenous antibiotics are always more effective than oral
antibiotics in treating
uncomplicated pneumonia
c) In patients with uncomplicated pneumonia, the risk of relapse is
greatest on the day
prior to planned discharge
d) Hypoxemia is rarely an indication for hospitalization
e) It takes several weeks for all symptoms of pneumonia to resolve
The correct answer is E

Explanation
One week after their initial presentation with pneumonia, 80% of
patients still have fatigue and cough. It usually takes several weeks
for all manifestations to resolve and the patient to return to normal.

According to the latest Thoracic Society guidelines for the diagnosis


and treatment of adults with community-acquired pneumonia (CAP), all
patients with suspected CAP should have a chest radiograph to establish
the diagnosis and identify complications (pleural effusion, multilobar
disease). Chest radiography performed early in the course of the disease
may be negative, however. Intravenous antibiotics are frequently no
better than oral antibiotics in patients with uncomplicated pneumonia.
The risk of relapse or worsening is greatest on the day of admission and
goes down progressively after that. All patients with hypoxemia should
be hospitalized.
A 25-year-old male presents 5 hours after the onset of the ?worst
headache of my life?. His temperature is 37.0?C (98.6?F), blood pressure
140/90 mm Hg, respiratory rate 20/min, and pulse rate 90 beats/min. The
patient is lethargic but oriented; there are no focal neurologic
findings, but neck stiffness is present. Unenhanced CT of the head is
negative. You elect to perform a lumbar puncture.

At this time, which one of the following findings in bloody spinal fluid
would indicate a diagnosis of subarachnoid hemorrhage?

a) Pink discoloration of the supernate of centrifuged spinal fluid


b) A WBC:RBC ratio of 1/1000
c) A protein (mg/dL) to RBC (count/dL) of 1/1000
d) A glucose level below 40 mg/dL
The correct answer is A

Explanation
Red blood cells lyse within a few hours in cerebrospinal fluid. This
imparts a pink color to the supernate of cerebrospinal fluid after
centrifugation. The fluid would be clear on a traumatic tap if the
spinal fluid were immediately centrifuged. CT scans will miss a small
percentage of subarachnoid hemorrhages.

Xanthochromia of the supernate of bloody cerebrospinal fluid requires a


period of about 10 hours to develop. It would be unlikely in a patient
presenting this early. It is a useful factor in distinguishing
subarachnoid hemorrhage from traumatic spinal tap when the patient
presents after 10 horus.

The WBC:RBC ratio is more useful in diagnosing meningitis with traumatic


taps. The ratio could be similar in subarachnoid hemorrhage or traumatic
spinal tap. The 1:1000 ratio of mg of protein/dL to RBC/dL is a correct
approximation but would not be very useful in distinguishing
subarachnoid hemorrage from traumatic tap.

Glucose in cerebrospinal fluid will drop with chemical meningitis caused


by blood. This reduction is less than with most infections. It does not
drop below a level of 40 mg/dL.
Which of the following is not a common infectious cause of acute diarrhea?

a) Escherichia coli
b) Shigella
c) Norwalk virus
d) Vibrio cholerae
e) Helicobacter pylori
The correct answer is E

Explanation
Acute diarrhea can be caused by Escherichia coli, shigella, norwalk
virus, vibrio cholerae. H. pylori is a common gastric pathogen that
causes gastritis, peptic ulcer disease, gastric adenocarcinoma, and
low-grade gastric lymphoma.
Which one of the following is characteristic of benign positional vertigo?

a) A duration of 10-15 minutes


b) Associated hearing loss and tinnitus
c) Associated diplopia and facial numbness
d) Being triggered by turning the head
e) Being preceded or accompanied by headache
The correct answer is D

Explanation
Benign positional vertigo is provoked by a change in position and lasts
only 10-20 seconds. Meniere‟s disease involves vertigo lasting for
hours, and is accompanied by unilateral hearing loss and tinnitus.
Vertebrobasilar insufficiency can cause vertigo, diplopia, and facial
numbness. Migraine-associated vertigo is preceded or accompanied by
headache.
A 67 year old female is concerned about osteoporosis. The study of
choice for determining whether or not she has osteoporosis is

a) Dual-energy x-ray absorptiometry (DEXA) of the hip and spine


b) A quantititative CT scan of the hip and spine
c) Plain radiographs of the hip and spine
d) Ultrasonography of the heel
e) A DEXA scan of the middle phalanx of the non-dominant hand
The correct answer is A

Explanation
Dual-energy x-ray absorptiometry (DEXA) of the hip and spine is the
method of choice for assessment of bone mineral density. Quantitative CT
is the most sensitive method, but results in substantially greater
radiation exposure. Plain radiographs are no sensitive enough to
diagnose osteoporosis until total bone density has decreased by 50%. The
predictive value of DEXA scans and ultrasound examinations of peripheral
bones in assessing fracture risk at the hip of verterbrae is not clear.
A 67 year old male presents with complaint of anginal chest discomfort
that comes and goes. He states it always happens when he is lying in his
bed or watching TV lying on his couch and is unrelated to his exercise.
Identify the type of angina patient is experiencing.

a) Prinzmetal‟s angina
b) Unstable angina
c) Stable angina
d) Angina decibutus
e) Variant angina
The correct answer is D

Explanation
Answer: D ? Angina decubitus is angina that occurs when a person is
lying down (not necessarily only at night) without any apparent cause.
Angina decubitus occurs because gravity redistributes fluids in the
body. This redistribution makes the heart work harder.

A. Prinzmetal‟s angina is induced by coronary artery spasm which is


reflected by either permanent blockade or blockade due to plaque. This
usually occurs at rest.
B. Unstable angina occurs at rest in patients with significant coronary
artery disease. Angina occurs frequently for extended periods of more
than 20 minutes with a week‟s presence.
C. Stable angina occurs due to exertion or stress. This type of angina
is relieved by rest or by nitrates.
E. Variant angina is just another name for Prinzmetal‟s angina.

What is the most important test for diagnose acute hepatitis A infection?

a) Hepatitis A IgG antibody


b) Hepatitis A IgM antibody
c) PT/INR
d) Increased liver function tests
The correct answer is B

Explanation

The above graphic illustrates the diagnostic algorithm in a patient


suspected of having acute hepatitis.

Hepatitis A virus is present in serum only during acute infection and


cannot be detected by clinically available tests. IgM antibody typically
develops early in the infection and peaks about 1 to 2 weeks after the
development of jaundice. It diminishes within several weeks, followed by
the development of protective IgG antibody (IgG anti-HAV), which
persists usually for life.

Thus, IgM antibody is a marker of acute infection, whereas IgG anti-HAV


merely indicates previous exposure to HAV and immunity to recurrent
infection.
A 25-year-old male comes to the emergency department with the sudden
onset of moderate to severe right-sided chest pain and mild dyspnea.
Vital signs are normal. A chest film shows a loss of markings along the
right lung margins, involving about 10%?15% of the lung space. The
mediastinum has not shifted. The best INITIAL treatment would be

a) strict bed rest


b) oxygen supplementation and close observation
c) decompression of the chest by insertion of a large-bore intravenous
catheter into the right second intercostal space at the midclavicular line
d) immediate chest tube insertion using a water seal
e) thoracotomy for wedge resection of pulmonary blebs
The correct answer is B

Explanation
A small spontaneous pneumothorax involving less than 15%?20% of lung
volume can be managed by administering oxygen and observing the patient.
The pneumothorax will usually resorb in about 10 days if no ongoing air
leak is present. Oxygen lowers the pressure gradient for nitrogen and
favors transfer of gas from the pleural space to the capillaries.
Decompression with anterior placement of an intravenous catheter is
usually reserved for tension pneumothorax. Chest tube placement is used
if observation is not successful or for larger pneumothoraces. Strict
bed rest is not indicated.
A 35 year old female neglected the torn skin near the finger nail which
developed into Staphylococcal cellulitis. She complains of sweating and
chills. Her temperature is 39 degrees Celsius. What is the appropriate
outpatient oral therapy for this complication?

a) Dicloxacillin
b) Vancomycin
c) Methicillin
d) Cefazolin
e) Penicillin
The correct answer is A

Explanation
Answer: A ? Dicloxacillin is the appropriate medication for
Staphylococcus cellulitis. Dicloxacillin is penicillin derivate active
against staphylococcus aureus, which is resistant to other penicillin
derivatives. Mechanism of action of dicloxacillin is by inhibiting the
synthesis of bacterial cell walls. It weakens the peptidoglycan chain
structure of the cell wall of the bacteria there by inhibiting the
synthesis of it. Since dicloxacillin is acid stable it can be given
orally. It is very effective for the treatment of cellulitis.

B. Vancomycin is not the correct choice of the medication for


staphylococcus cellulitis because of its poor bioavailability through
oral route.
C. Methicillin is also wrong choice because of its unfavorable side
effects, though Staphylococcus aureus is sensitive to it.
D. Cefazolin is not the correct choice of medication for cellulitis
because of its adverse effects.
E. Penicillin is not preferred as the best choice of medication because
of the resistance from the majority of strains of bacteria.

All of the following dementias can benefit from specific treatments, except

a) Normal pressure hydrocephalus (NPH)


b) Alzheimer‟s disease
c) Sarcoidosis
d) Creutzfeldt Jakob disease
e) B12 deficiency
The correct answer is D
Explanation
The dementias associated with NPH, alzheimers, sarcoidosis and B12
deficiency can be either cured or benefit from the following treatments:
Shunting of the ventricles in NPH, Acetylcholinesterase inhibitors such
as donepezil, rivastigmine and tacrine for Alzhiemers. Vitamin B12
replacement for B12 deficiency.

Currently, CJD cannot be cured, and its progress cannot be slowed. The
disease is fatal, usually within months or a few years.

The funduscopic findings shown here are most likely to be found in which
one of the following patients‟

a) A 20-year-old male who was hit in the left eye in a fight and
complains of a “black eye”but no visual symptoms
b) A 43-year-old hypertensive male who presents with a chief complain
of gradually increasing difficulty reading the newspaper, except when
held at arm‟s length
c) A 45-year-old female with metastatic melanoma and no visual
symptoms, who is undergoing evaluation to determine the primary site of
her tumor
d) A 64-year-old male who complains of “floaters” in his left eye
e) A 70-year-old diabetic female who has sudden painless impairment of
vision in her left eye
The correct answer is E

Explanation
The 70-year-old diabetic woman with a sudden painless impairment of
vision in her left eye is most likely to have the funduscopic findings
shown. There is a classic boat-shaped preretinal hemorrhage, probably
secondary to diabetic retinopathy-induced neovascularization. The flat
top of the lesion has resulted from the effects of gravity, confirming
the lesion‟s location in the potential space between the retina and the
vitreous. It is located directly over the macula, and thus would cause
significant visual symptoms. Preretinal hemorrhages can result from
trauma, such as a blow to the eye, but this one would be symptomatic
because of its location. “Floaters” are located in the vitreous.
Melanoma has a different appearance and would not cause visual symptoms
in this location. Difficulty reading a newspaper at a normal distance is
most likely due to presbyopia.
An asymptomatic 41-year-old male is having a life insurance physical. He
is reasonably active and has no chest pain or palpitations. An EKG, his
first, is shown in Figure 1. There is no family history of vascular
disease. He has never had any health problems other than a couple of
sports-related injuries. His blood pressure is 118/60 mm Hg, height 178
cm (70 in), and weight 70 kg (154 lb). A chest radiograph is normal.

Which one of the following would be most appropriate at this time?

a) An exercise stress test


b) Holter monitoring
c) Electrophysiologic mapping
d) Cardiac catheterization
e) No further evaluation
The correct answer is E

Explanation
The EKG demonstrates unifocal premature contractions. Prospective
studies of large populations of ambulatory men have not shown increased
mortality on long-term follow up. Regardless of the presence and
persistence of ventricular ectopy, the patients have a prognosis no
different than that of other healthy people, and above all, there is no
increased risk of sudden death. In the absence of angina, a history of
heart failure, cardiomegaly on a chest radiograph, left ventricular
hypertrophy, bundle branch block, or hypertension, further diagnostic
evaluation is unwarranted.
A 36-year-old male with a history of a seizure disorder is brought to
the emergency department with generalized tonic-clonic activity.
Emergency medical personnel report this has been ongoing for 15 minutes.

After initial resuscitative measures, the preferred medication in this


situation is

a) phenytoin (Dilantin)
b) fosphenytoin (Cerebyx)
c) diazepam (Valium)
d) lorazepam (Ativan)
e) valproate sodium (Depacon)
The correct answer is D
Explanation
This patient presents in status epilepticus. Although diazepam has long
been an effective treatment, lorazepam has emerged as the preferred
agent because of pharmacologic properties that should give it a longer
duration of action than diazepam. Phenytoin and fosphenytoin are limited
by potentially serious adverse effects, such as hypotension. Valproic
acid is not FDA approved for status epilepticus.
A 56-year-old white male has New York Heart Association (NYHA) class III
chronic heart failure. Despite conventional therapy with appropriate
dosages of a diuretic, an ACE inhibitor, and a ß-adrenergic blocker, his
left ventricular ejection fraction hovers around 35%, and he continues
to have dyspnea on exertion. At a recent office visit his electrolytes
were within normal limits. After you obtain an EKG during an office
visit today, you consider adding digoxin to his treatment regimen.

Which one of the following is true regarding digoxin therapy in this


situation?

a) It is the treatment of choice if the patient‟s EKG shows


atrioventricular block
b) It is not likely to improve the ejection fraction
c) A loading dose will be necessary
d) Serial drug levels are generally not necessary
e) A reasonable dosage is 0.50 mg/day orally
The correct answer is D

Explanation
Digoxin remains a useful drug in the management of chronic heart failure
caused by systolic dysfunction. Randomized, controlled studies have
convincingly confirmed its efficacy in improving ejection fraction and
submaximal exercise capacity. Current consensus guidelines recommend its
use for patients in whom NYHA class II?IV symptoms persist despite
conventional therapy with diuretics, ACE inhibitors, and ß-adrenergic
blockers. Digoxin should not be administered to a patient who has an
atrioventricular block, unless it is first treated with a permanent
pacemaker. Loading doses of digoxin are not necessary during initiation
of therapy for patients with chronic heart failure, and serial
assessments of serum digoxin levels are not necessary in most patients.

The radioimmunoassay was developed to assist in the evaluation of


toxicity, rather than the efficacy of the drug. There appears to be
little relationship between serum digoxin concentration and the drug‟s
therapeutic effect. The dosage of digoxin should be 0.125?0.25 mg daily
in the majority of patients. Higher doses (e.g., 0.375?0.50 mg) are
rarely needed, especially as initial therapy for chronic heart failure.
A 72-year-old male is brought to your office by a friend because of
increasing confusion, irritability, and difficulty walking. This began
shortly after the patient‟s car broke down and he had to walk a mile to
get to a phone and call the friend. The temperature outdoors has been
near 100?F.

On examination you note a rectal temperature of 39.5?C (103.1?F). His


pulse rate is 110 beats/min and his blood pressure is 100/60 mm Hg.

Which one of this patient‟s findings indicates that he has heatstroke


rather than heat exhaustion?
a) Confusion
b) Sweating
c) His temperature
d) His heart rate
e) His blood pressure
The correct answer is A

Explanation
Heat exhaustion and heatstroke are both on the continuum of heat-related
illness. Heatstroke is a much more severe condition than heat
exhaustion. Evidence of central nervous system dysfunction is evidence
of heatstroke rather than heat exhaustion, even if other symptoms are
not severe and point to heat exhaustion. Heatstroke is a medical emergency.

A 56 year old black female with past medical history of diabetes


mellitus type 2 presents with a rash on the region of her axilla. See
picture:

This rash is known as what?

a) Malignant Melanoma
b) Hodgkin disease
c) Keratosis
d) Acanthosis Nigricans
The correct answer is D

Explanation
Acanthosis nigricans is most commonly caused by insulin resistance,
usually from type 2 diabetes mellitus.

Other causes are familial, obesity, drug-induced, malignant (gastric


cancer), idiopathic, and Polycystic ovary syndrome. In the context of a
malignant disease, Acanthosis nigricans is a paraneoplastic syndrome and
is then commonly referred to as Acanthosis nigricans maligna. Acanthosis
nigricans should make you suspicious for a malignancy.

People with acanthosis nigricans should be screened for diabetes and,


although rare, cancer. Controlling blood glucose levels through exercise
and diet often improves symptoms.
Which of the following is the best test in the long term follow up of a
diabetic patient?

a) HbA1C
b) C-Peptide
c) Fasting blood glucose
d) Random blood glucose
e) Diary of daily blood glucose values
The correct answer is A

Explanation HbA1C is a test that measures the amount of glycosylated


hemoglobin in your blood. The test gives a good estimate of how well
diabetes is being managed over time. In particular over the past 3
months, since that is the life span of the red blood cell that contains
the hemoglobin molecule.

This test measures blood sugar control over an extended period in people
with diabetes. In general, the higher your HbA1C value, the higher the
risk that you will develop complications from diabetes (eye disease,
kidney disease, nerve damage, heart disease, and stroke). This is
especially true if your HbA1C remains elevated on more than one occasion.

Most physicians will consider a HbA1C less than 7 as an indicator of


good diabetic control.

Unlike a diary, the HbA1C value does not lie.

A 35-year-old black male has a blood pressure of 142/88 mm Hg, confirmed


on repeat measurements. A complete metabolic panel and urinalysis reveal
a serum creatinine level of 1.9 mg/dL (N 0.6?1.5) and 2+ protein in the
urine. Which one of the following would be the most appropriate initial
treatment?

a) ACE inhibitors
b) Aldosterone antagonists
c) Beta - Blockers
d) Calcium channel blockers
e) Diuretics
The correct answer is A

Explanation
Although JNC-7 guidelines recommend a diuretic as the initial
pharmacologic agent for most patients with hypertension, the presence of
?compelling indications‟ may indicate the need for treatment with
antihypertensive agents that demonstrate a particular benefit in primary
or secondary prevention. JNC-7 guidelines recommend ACE inhibitors
(ACEIs) or angiotensin receptor blockers (ARBs) for hypertensive
patients with chronic kidney disease (SOR A).

First-line therapy for proteinuric kidney disease includes an ACEI or an


ARB. Because these drugs can cause elevations in creatinine and
potassium, these levels should be monitored. A serum creatinine level as
much as 35% above baseline is acceptable in patients taking these agents
and is not a reason to withhold treatment unless hyperkalemia develops.
If an ACEI or an ARB does not control the hypertension, the addition of
a diuretic or a calcium channel blocker may be required. The combination
of ACEIs and diuretics may be used to control hypertension in patients
with diabetes mellitus, heart failure, or high coronary disease risk, as
well as post myocardial infarction. Calcium channel blockers are
recommended for managing hypertension in patients with diabetes or high
coronary disease risk.

-Blockers are useful as part of combination therapy in patients with


hypertension and heart failure, or post myocardial infarction.

A 73-year-old female who is a patient in the skilled-care wing of an


extended-care facility is nearly ready for discharge. Her extended
rehabilitation stay was a result of surgery for a hip fracture, so she
asks what she can do to prevent further fractures. A dual-energy x-ray
absorptiometry (DEXA) scan reveals her T score to be ?1.4 in both the
hip and the spine.

Which one of the following statements is true regarding treatment of


this patient‟s condition?

a) No treatment is indicated
b) Raloxifene (Evista) can help prevent further hip fractures
c) Calcitonin is the most effective drug available for the treatment of
osteoporosis
d) Bisphosphonates can reduce fracture risk even if bone density is not
substantially
increased
The correct answer is D

Explanation
Anyone with established disease, such as osteoporotic hip fracture, is a
candidate for osteoporosis treatment, regardless of their T score.
Raloxifene is effective for prevention of vertebral fractures in
patients with osteoporosis, but not for other fractures. Other agents
are more effective than calcitonin for the treatment of osteoporosis.
Increases in bone mineral density during treatment with bisphosphonates
contribute only a small part to fracture prevention.

A 68 year old white female is hospitalized for pneumonia. She appears


acutely ill and slightly lethargic. Her examination is consistent with
right lower lobe pneumonia but is otherwise normal. Her pulse rate is 90
beats/min and regular. Her weight is normal for height. Her TSH level is
9.0 microU/mL (N 1.0-5.0) The most appropriate initial step for managing
her thyroid abnormality is

a) Follow-up testing after discharge


b) Free T4 and rT3 levels now
c) Thyroid antimicrosomal antibody levels now
d) Levothyroxine (synthroid), 0.025 mg every day
e) Levothyroxine, 0.1 mg every day
The correct answer is A

Explanation
The likelihood that this patient has significant thyroid disease is very
low given the minimal elevation of TSH, normal clinical examination, and
concomitant pneumonia. The elevated TSH level is likely due to her
illness rather than to any underlying thyroid condition. Even if she had
a palpable thyroid, her risk of hypothyroidism would be on the order of
5%. A TSH level > 20 U/mL in an acutely ill patient reflects true
hypothyroidism only about 40% of the time. It is likely that this
patient has sick euthyroid syndrome and that follow-up thyroid testing
after discharge when she has recovered is appropriate and is very likely
to be normal. Free T4, rT3, and TSH levels would be appropriate for
subsequent evaluation if the patient‟s laboratory values did not return
to normal after resolution of the pneumonia.
You are reviewing laboratory findings in a 64-year-old male hospitalized
with acute renal failure. The patient is not on any long-term
medications. His renal function has previously been normal, but
currently his serum creatinine level is 2.8 mg/dL (N 0.6?1.5), BUN 60
mg/dL (N 8?25), and fractional excretion of sodium (FENa) 0.75%. His
urine specific gravity is 1.025, and the urine sediment shows only
hyaline casts.

Based on these findings, which one of the following conditions is most


likely?

a) Hypervolemia
b) Acute pyelonephritis
c) Interstitial nephritis
d) Obstruction due to benign prostatic hypertrophy
e) Hypovolemia
The correct answer is E

Explanation
Acute renal failure (ARF) has prerenal, renal, and postrenal causes. The
approach to patients with ARF must be systematic, requiring a thorough
history, medication review (including all nonprescription drugs), and
careful physical examination with attention to volume status. Besides
creatinine, BUN, and electrolytes, the urinalysis (including
microscopic) and the fractional excretion of sodium (FENa) are essential
to the initial evaluation. The FENa is very useful in separating
prerenal causes from renal or postrenal causes in patients who have not
been on chronic diuretic therapy. A FENa <1.0%, as in this patient,
indicates a prerenal cause. Of the possibilities listed, hypovolemia is
the only condition that could result in prerenal ARF. Furthermore, a
BUN:creatinine ratio >20, a urine specific gravity >1.020, and hyaline
casts in the sediment all support the diagnosis of hypovolemia.

Acute pyelonephritis may or may not result in ARF, and these patients
usually have WBCs or WBC casts in the urine. Neither hypervolemia (due
to fluid overload) nor benign prostatic hypertrophy would typically be
associated with the laboratory findings given. Interstitial nephritis, a
renal cause of ARF, would most likely result in a FENa greater than 1.0%.
Total parenteral nutrition is most appropriate for patients

a) With poorly functioning gastrointestinal tracts who cannot tolerate


enteral feeding
b) Who cannot swallow because of an esophageal motility problem
c) Who refuse to eat
d) In whom maintenance nutrition is desired for a short period
following recovery from surgery
The correct answer is A

Explanation
Total parenteral nutrition (TPN) is indicated for patients with poorly
functioning gastrointestinal tracts who cannot tolerate other means of
nutritional support and for those with high caloric requirements that
cannot otherwise be met. Patients who cannot swallow because of an
esophageal motility problem and those who are resistant to feeding can
be managed with tube feedings. Peripheral alimentation, which provides
fewer calories than TPN or liquid tube feedings, would be more
appropriate over the short term in patients recovering from surgery.
A 25-year-old clinically healthy black female is involved in a minor
auto accident. Chest radiographs obtained after the wreck reveal
bilateral hilar lymphadenopathy. She has no history of environmental
exposures and has no symptoms. A physical examination is completely
normal. Your initial workup includes a normal comprehensive metabolic
panel, CBC, and urinalysis; a negative tuberculin skin test; a normal
EKG; and normal pulmonary function tests. A transbronchial lung biopsy
specimen reveals a noncaseating epithelioid granuloma.

Which one of the following would be the most appropriate treatment at


this time?

a) Long-term high-dose systemic corticosteroids


b) Pulsed doses of systemic corticosteroids
c) Inhaled corticosteroids
d) Oral methotrexate weekly
e) Observation only
The correct answer is E

Explanation
Sarcoidosis is a disease of unknown cause characterized by the presence
of noncaseating epithelioid granulomas; it involves many different organ
systems. The lungs are commonly involved, with bilateral hilar
lymphadenopathy being commonly present and pulmonary infiltrates and
fibrosis somewhat typical. Other frequent areas of involvement include
the skin, central nervous system, eyes, liver, heart, salivary glands,
kidneys, muscles, and bones. When the disease is limited to asymptomatic
hilar adenopathy, it is termed stage I and no treatment has been shown
to be beneficial. The most appropriate management of stage I patients is
routine follow-up.
A 35-year-old pharmacist has a positive response to the CAGE
questionnaire. He admits to drinking at least a full case of beer (24
12-oz cans) each week. The history reveals evidence of tolerance and
withdrawal. He smokes cigarettes, but denies other risky behaviors. He
tells you that his marriage is failing.

The patient‟s general health appears good otherwise. The only positive
findings on the physical examination are a mild tremor, sweaty palms,
and mild tachycardia.

In this situation, which one of the following would be most helpful in


choosing to recommend an outpatient versus inpatient detoxification program?
a) A urine toxicology screen
b) A serum magnesium level
c) An EKG
d) An EEG
e) Posteroanterior and lateral chest radiographs
The correct answer is A

Explanation
Because polysubstance abuse is common in patients with alcohol
dependence, physicians may wish to perform a urine toxicology screen in
these patients. Coexisting use of other addictive substances constitutes
a relative contraindication for selecting patients for alcohol
withdrawal outpatient treatment. In this case, a pharmacist would have
relatively easy access to other addicting substances, even though he
denies using an. An EKG, EEG or chest radiograph should not be ordered
unless there is a clinical indication, such as a seizure history
indicating the need for an EEG. Magnesium deficiency, if present, should
be corrected during alcohol withdrawal syndrome, but it is not necessary
as a screening test for detoxification.

An elevated level of hemoglobin A2 in a patient with mild microcytic


anemia suggests the diagnosis of

a) Alpha-thalassemia
b) Beta-thalassemia
c) Sickle trait
d) Hereditary spherocytosis
e) Hereditary persistence of fetal hemoglobin
The correct answer is B

Explanation
Thalassemias are a group of inherited microcytic, hemolytic anemias
characterized by defective Hb synthesis. They are particularly common in
people of Mediterranean, African, and Southeast Asian ancestry. Symptoms
and signs result from anemia, hemolysis, splenomegaly, bone marrow
hyperplasia, and, if there have been multiple transfusions, iron overload.

Beta-thalassemia results from decreased production of beta-polypeptide


chains. Alpha-thalassemia, which results from decreased production of
alpha-polypeptide chains. Diagnosis is based on quantitative Hb analysis
(hemoglobin electrophoresis). Treatment for severe forms may include
transfusion, splenectomy, chelation, and stem cell transplantation.
A 70 year old man complains of increasing urinary frequency and
dribbling. On physical examination, a digital rectal exam reveals a
normal-sized prostate. The prostate specific antigen level is elevated
at 14 ng/mL. Ultrasonography reveals a small hypoechoic area on the
prostate measuring 5 × 9 mm. Which of the following is the most
appropriate next step?

a) Finasteride
b) Biopsy of prostate
c) Repeat PSA in 3 months
d) Alfuzosin
The correct answer is B

Explanation
Measurement of serum levels of prostate-specific antigen (PSA) can be
used to screen for prostate cancer. However, this substance can also be
elevated in prostate hypertrophy. A transrectal ultrasound can identify
lesions not palpable on rectal examination, and the area can be biopsied
under ultrasound guidance.
Which one of the following is the most accurate and cost-effective
screening procedure for thyroid disease in geriatric patients‟

a) Serum T_3
b) Serum free T_4
c) Radioactive iodine (RAIU) uptake
d) Serum TSH
e) Radioallergosorbent test (RAST)
The correct answer is D

Explanation
Physicians should consider periodic screening for biochemical
abnormalities in elderly patients, because a clinical presentation of
thyroid disease is not reliable in this age group. The most effective
way to screen for thyroid dysfunction is to order a TSH level with
third-generation assays using an immunoradiometric assay (IRMA). The
RAST test is used for allergy testing. Notably, the College of
Physicians recommends periodic screening only for women age 50 and
older. Other organizations have varying guidelines.
A 65 year old female presents to your office complaining of fever,
facial pain and nasal discharge tinged with blood.

The condition started a week ago and is worsening progressively. Her


past medical history is significant for diabetes mellitus diagnosed 15
years ago. She is taking insulin to control her blood sugar and admits
that her diabetes has been poorly controlled in the last few years.

Her vital signs are within normal limits except for a temp of 390C.
HEENT exam shows congestion of the left nasal turbinates with several
black lesions that seem to be necrotic. Severe pain is elicited upon
maxillary pressure. What would you recommend?

a) Amphotericin B
b) Amphotericin B + fluconazole
c) Ketoconazole
d) Surgical debridement + Amphotericin B
e) Surgical debridement only
The correct answer is D

Explanation
Mucormycosis is a serious infection that is usually seen in uncontrolled
diabetes. It is caused by a fungus called Rhizopus. The treatment of
mucormycosis requires surgical debridement and IV Amphotericin B.

Amphotericin alone is not effective; it should be combined with


aggressive surgical debridement in order to allow for the antifungal
medication to reach the necrotic areas.

The “Azoles” family of drugs is not effective against Rhizopus.


Amphotericin is the only effective drug against it.
A patient with delirium tremens manifests all of the following, except

a) Clouded consciousness
b) Hypothermia
c) Coarse tremor
d) Tachypnea
e) Hyperhidrosis
The correct answer is B

Explanation
Delirium tremens commonly begins two to three days after the last
alcohol drink, but it may be delayed more than a week. Its peak
intensity is usually four to five days after the last drink. This
condition causes dangerous shifts in your breathing, your circulation
and your temperature control. It can cause your heart to race
dangerously or can cause your blood pressure to increase dramatically,
and it can cause dangerous dehydration. Delirium tremens also can
temporarily reduce the amount of blood flow to your brain.

Symptoms can include confusion, disorientation, stupor or loss of


consciousness, nervous or angry behavior, irrational beliefs, soaking
sweats, hyperthermia, tremor, tachypnea, sleep disturbances and
hallucinations.
A recurrent painful swelling of a submandibular salivary gland is most
likely to be due to

a) Virus infection
b) Calculus
c) Sarcoidosis
d) Mixed tumor
e) AIDS
The correct answer is B

Explanation
Sialolithiasis is the result of tiny stones, called sialoliths, forming
inside the salivary glands. Symptoms of sialolithiasis may include:
swollen glands around the neck, mouth pain, mouth tenderness, redness,
swelling around the mouth, neck pain or swelling.

Risk factors for developing sialolithiasis include: dehydration, which


thickens the saliva, not eating a lot, which decreases the demand for
saliva. Medications that decrease saliva production, including certain
antihistamines, blood pressure drugs and psychiatric medications. If the
stones cause a blockage within the salivary glands, it can lead to a
painful, infection called sialadenitis.
A 56 year old man develops pain and swelling in his right lower leg and
ankle. He states it started yesterday. There is some discoloration on
the ankle. He states that he accidentally banged his ankle also. What is
the most appropriate treatment?

a) Steroids
b) Narcotics
c) Anticoagulants
d) Antibiotics
The correct answer is C

Explanation
Deep venous thrombosis (DVT) is clotting of blood in a deep vein of an
extremity (usually calf or thigh) or the pelvis. DVT is the primary
cause of pulmonary embolism. DVT results from conditions that impair
venous return, lead to endothelial injury or dysfunction, or cause
hypercoagulability.

Three main factors (known as Virchow's triad) can contribute to deep


vein thrombosis: injury to the vein's lining (trauma, surgery), an
increased tendency for blood to clot (inherited hypercoagulability), and
slowing of blood flow (stasis).

DVT may be asymptomatic or cause pain and swelling in an extremity.


Diagnosis is by history, physical examination, and duplex
ultrasonography, with d-dimer or other testing as necessary. Treatment
is with anticoagulants. Prognosis is generally good with prompt,
adequate treatment.
A 48 year old unemployed house painter presents to the emergency
department with a gradual onset of lethargy and weakness. A physical
examination is remarkable for 4+ pitting edema of the lower extremities
and a prominent abdomen.

Laboratory Findings

Serum sodium---------------------------------122 mEq/L (N 135-145)


Serum osmolality-----------------------------260 mOsm/kg H2O (N 280-296)
Urine sodium-----------------------------------5 mEq/L
Urine osmolality-------------------------------250 mOsm/kg H2O

The most likely diagnosis in this case is

a) Syndrome of inappropriate antidiuretic hormone (SIADH)


b) Primary polydipsia
c) Adrenal insufficiency
d) Cirrhosis
e) Salt-wasting nephropathy
The correct answer is D

Explanation
The individual described in this case has symptomatic hyponatremia.
Headache, mental confusion, nausea, and malaise are common. Seizures,
stupor, and coma generally do not occur until sodium concentrations fall
below 120 mEq/L. The presence of significant peripheral edema in this
patient indicates extracellular fluid volume expansion, and his serum
osmolality is low. In this situation, hyponatremia is usually a
manifestation of an edematous state, such as hepatic cirrhosis,
congestive heart failure, or the nephritic syndrome. Although these
patients have increase extracellular fluid, their intravascular fluid is
depleted, and their body‟s attempt to conserve sodium at the level of
the kidney produces urine with a sodium concentration < 20 mEq/L. They
have approximately increased arginine vasopressin (AVP) levels,
resulting in a urine osmolality that is less than maximally dilute and
often > 100 mOsm/kg H2O.

Patients with the syndrome of inappropriate antidiuretic hormone (SIADH)


have normal volume status and urine sodium levels which are typically >
20 mEq/L. Patients with primary polydipsia often have an underlying
psychiatric disorder. They have normal volume status, and produce large
volumes of very dilute urine (< 50 mOsm/kg H2O). Patients with adrenal
insufficiency typically have normal volume status, but may be
dehydrated. Patients with salt-wasting nephropathy are typically
dehydrated despite producing urine with a sodium concentrate > 20 mEq/L.

A 45-year-old female with asthma uses her albuterol (Proventil,


Ventolin) inhaler ?once or twice a week? to relieve her wheezing. What
daily medication should now be added?

a) No daily medication
b) Low-dose inhaled corticosteroids
c) Oral beta_2 -agonists
d) Long-acting inhaled beta_2 -agonists
e) Low-dose inhaled corticosteroids plus long-acting inhaled beta_2 -agonists
The correct answer is A

Explanation
The patient has mild intermittent asthma and requires no daily
medication. Current guidelines recommend the following treatments based
on asthma classifications:

Mild intermittent asthma (symptoms 2 days per week or less, nighttime


symptoms 2 nights or less per month) - no daily medication needed.

Mild persistent asthma (symptoms more than 2 days per week but less than
one time per day. nighttime symptoms more than 2 nights per month) ?
daily low-dose inhaled corticosteroids.

Moderate persistent asthma (symptoms every day, nighttime more than once
weekly) / daily low- to medium-dose inhaled corticosteroids and
long-acting beta_2 -agonists.

Severe persistent asthma (symptoms continual during the day, frequently


at night) / daily high-dose inhaled corticosteroids and long-acting beta_2
-agonists.

A 65 year old male comes to the office complaining of excruciating pain


in his left ear that began last night.

The pain is 10/10 in intensity and radiates to his mandible. He also


complains of ear discharge along with the pain. His condition is
worsening progressively despite the use of Tobradex? ear drops
(Tobramycin + dexamethasone). Review of systems is otherwise normal.

His vitals signs are within normal limits except for a temp of 38C.
HEENT exam shows the presence of granulation tissue in the lower part of
his external auditory canal at the junction between the cartilaginous
and bony parts of the canal. When the patient is asked to smile, some
weakness of the left facial nerve is noticed.

Which of the following pathogens is most likely to be responsible for


this patient‟s condition?

a) Aspergillus nigrican
b) Escherichia coli
c) Pseudomonas aeruginosa
d) Streptococcus pneumonia
e) Streptococcus pyogenes
The correct answer is C

Explanation
Malignant otitis externa (MOE), skull base osteomyelitis, is the most
likely diagnosis of this patient. MOE is usually seen in diabetics and
immunosuppressed patients and often presents with a very intense ear
pain and ear discharge. Fever is often present and the pain may radiate
to the TMJ. Resistance to topical antibiotics along with the history of
diabetes should be a useful clue. The most common cause of malignant
otitis externa is Pseudomonas aeruginosa.

Strep pyogenes and Staph aureus may be superadded; however very rarely
to be the responsible agents.

Strep pneumoniae is the most common cause of otitis media and not externa.

Escherichia coli is the most common cause of UTI and not MOE.

Aspergillus species especially the fumigatus type may be occasionally


the cause of MOE.

Know the infections associated with diabetes well! Malignant otitis


externa, erysipelas, cellulitis, mucormycosis, fournier‟s gangrene and
emphysematous cholecystitis are frequently asked!
Regarding fibroadenoma of the breast, all of the following are true,
*except*

a) The most common benign tumor


b) When aspirated - no fluid comes out

c) Malignant tumor
d) Tumor is encapsulated
e) Tumor is mobile
The correct answer is C

Explanation
Fibroadenoma is the most common benign tumor of the breast and the most
common breast tumor in women under age 30. They are typically painless
lumps that feel like small, slippery marbles. They usually develop in
young women, often in teenagers, and may be mistaken for cancer although
they are benign and tend to be more circumscribed and mobile. Simple
fibroadenoma does not appear to increase risk of breast cancer; complex
fibroadenoma may increase risk slightly.

The tumor is not fixed to the adjacent skin, muscle, or lymph nodes, so
they are mobile within the breast on palpation. It is commonly found
immediately adjacent to the areola, though rarely directly behind the
nipple.

Macroscopically the tumor is round or ovoid, elastic, nodular, and has a


well demarcated capsule; on cut surface it is grey-white. When aspirated
- no fluid comes out.
All of the following will be seen in alcohol withdrawal, except

a) Course tremor
b) Palpitations
c) Hallucinations
d) Urinary incontinence
The correct answer is D

Explanation
Alcohol withdrawal refers to a group of symptoms that may occur from
suddenly stopping the use of alcohol after chronic or prolonged ingestion.

Mild to moderate physical symptoms include: headache, sweating,


especially the palms of the hands or the face, nausea, vomiting, loss of
appetite, insomnia, sleeping difficulty, paleness, rapid heart rate
(palpitations), pupils different size (enlarged, dilated pupils), clammy
skin, abnormal movements, tremor of the hands, involuntary and abnormal
movements of the eyelids.

Severe symptoms include: A state of confusion and hallucinations


(visual) known as delirium tremens, agitation, fever, convulsions and
black outs.
Which one of the following medications should be discontinued in a
patient with diabetic gastroparesis‟

a) Exenatide (Byetta)
b) Benazepril (Lotensin)
c) Metformin (Glucophage)
d) Hydrochlorothiazide
e) Prochlorperazine maleate
The correct answer is A

Explanation
Delayed gastric emptying may be caused or exacerbated by medications for
diabetes, including amylin analogues (e.g., pramlintide) and
glucagon-like peptide 1 (e.g., exenatide). Delayed gastric emptying has
a direct effect on glucose metabolism, in addition to being a means of
reducing the severity of postprandial hyperglycemia. In a clinical trial
of exenatide, nausea occurred in 57% of patients and vomiting occurred
in 19%, which led to the cessation of treatment in about one-third of
patients. The other medications listed do not cause delayed gastric
emptying.
A child is noted to develop easy bruising and excessive bleeding from
small skin cuts. Tests show bleeding time is increased and a deficiency
of von Willebrand's factor. What is the appropriate treatment?

a) Factor 7
b) Factor 9
c) Platelet transfusion
d) DDAVP
The correct answer is D

Explanation
Von Willebrand's disease is a hereditary deficiency of von Willebrand's
factor (VWF), which causes platelet dysfunction. Bleeding tendency is
usually mild. Screening tests show a prolonged bleeding time, normal
platelet count, and, possibly, a slightly prolonged PTT.

Diagnosis is based on low levels of VWF antigen and abnormal ristocetin


cofactor activity. Treatment involves control of bleeding with
replacement therapy (cryoprecipitate or pasteurized intermediate-purity
factor VIII concentrate) or desmopressin (DDAVP).
Factors 8 and 9 are given in hemophilia.
A 60 year old man presents with back and leg pain and trouble urinating.
On examination, he has decreased sensation over the buttocks, normal
motor power, and absent ankle jerks bilaterally. How would this patient
best be investigated?

a) Plain films of the lumbar spine


b) CT of lumbar spine
c) MRI of spine
d) Investigate only if no improvement after 6 weeks of symptomatic treatment
e) Gallium scan of the spine
The correct answer is C

Explanation
Sciatica is pain along the sciatic nerve. It usually results from
compression of nerve roots in the lower back. Common causes include disk
disease, osteophytes, and narrowing of the spinal canal (spinal
stenosis). Symptoms include pain radiating from the buttocks down the
leg. Diagnosis is by MRI or CT. Electromyography and nerve conduction
studies help confirm the affected level. Treatment includes symptomatic
measures and sometimes surgery, particularly if there is a neurologic
deficit.
A 65 year old male, who has been recently retired from a cement
manufacturing company in Manitoba, complains of fever, night sweats and
a chronic progressive cough that is resistant to antibiotic treatment.
An x-ray shows multiple small coin-like lesions in the upper lobe of the
right lung. The TB skin test was negative twice. Which one of the
following is the most likely cause?

a) Legionellosis
b) Tuberculosis
c) Silicosis
d) Dimorphic mycosis
e) Atypical mycobacteria
The correct answer is C

Explanation
Silicosis is caused by inhalation of crystalline-free silica dust and is
characterized by nodular pulmonary fibrosis. Chronic silicosis initially
causes no symptoms or only mild dyspnea but over years can advance to
involve most of the lung and cause dyspnea, hypoxemia, pulmonary
hypertension, and respiratory impairment. Diagnosis is based on history
and chest x-ray. No effective treatment exists except supportive care
and, for severe cases, lung transplantation.
A 38 year old nurse comes to the emergency department after leaving work
early because of a "horrible headache." She has had a "cold" with sinus
congestion for the past week, and yesterday she began taking an
over-the-counter combination of diphenhydramine and pseudoephedrine. She
tells you she has a history of "migraines," multiple allergies,
premenstrual syndrome and depression, for which she takes phenelzine (a
monoamine oxidase inhibitor). Vital signs are: temperature 37.2°C
(99.0°F), pulse 90/min, respirations 16/min and blood pressure 210/118
mm Hg. Which of the following is the most appropriate action?

a) Administer meperidine, intramuscularly


b) Administer phentolamine, intravenously
c) Order CT scan of the head
d) Order transillumination of the sinuses
e) Prescribe oral oxycodone and nasal corticosteroids
The correct answer is B

Explanation
This patient has hypertension secondary to the effects of the
pseudoephedrine and her monoamine oxidase inhibitor. The inhibition of
MAO-A causes the rise of norepinephrine, dopamine and serotonin in the
synaptic cleft, of MAOB only of dopamine. Pseudoephedrine displaces
norepinephrine from synaptic terminals. For this reasons, nasal sprays
such as the one this patient is taking are to be avoided when MAOIs are
used. Phentolamine is a reversible alpha-adrenergic receptor blocker
that will blunt the effects of the excess catecholamines on arteriolar
vasoconstriction.
A 42-year-old male presents with a 48-hour history of crampy abdominal
pain and diarrhea. He has a history of mild aortic insufficiency. Two
weeks ago he received 2 g of amoxicillin as prophylaxis when having his
teeth cleaned. He has not traveled recently. A stool smear reveals 4+ WBCs.

The organisms most likely responsible for this patient‟s illness is

a) Campylobacter jejuni
b) Clostriudium difficile
c) Salmonella typhosa
d) Shigella sonnei
e) Rotavirus
Explanation
Clostridium difficile is a major cause of nosocomial colitis,
generally occurring after antibiotic-induced alterations of bowel flora.
The other choices listed are not associated with antibiotic-induced
diarrhea.
A heavy-set 62 year old man with normocytic, normochromic anemia
presents with an acute backache after sitting down on the toilet. He is
unable to pass his urine and unable to stand up because of weakness in
his legs. Which one of the following is the most appropriate next
investigation?

a) Serum calcium level


b) Renal function assessment
c) Urine test for Bence Jones protein
d) Protein electrophoresis
e) MRI of the spine
The correct answer is A

Explanation
Multiple myeloma is a malignancy of plasma cells that produce monoclonal
immunoglobulin and invade and destroy adjacent bone tissue. Common
manifestations include bone pain, renal insufficiency, hypercalcemia,
anemia, and recurrent infections. (Hypercalcemia remains the most
frequent metabolic complication of myeloma in patients, and excessive
osteolysis plays a major contributory role in its pathogenesis.)

Diagnosis requires demonstration of M-protein (sometimes present in


urine and not serum) and either lytic bone lesions, light-chain
proteinuria, or excessive marrow plasma cells. A bone marrow biopsy is
usually needed. Specific treatment includes conventional chemotherapy.

Workup:
Laboratory Studies
* Perform a complete blood cell (CBC) count
* Obtain a comprehensive metabolic panel to assess levels of total
protein, albumin and globulin, blood urea nitrogen (BUN), creatinine,
and uric acid (uric acid will be high if the patient has high cell
turnover or is dehydrated).
* Obtain serum protein electrophoresis, urine protein electrophoresis,
and immunofixation (to determine the type of each protein present; to
identify the presence of the Bence Jones protein in urine; to identify
the subtype of protein)
* Obtain a 24-hour urine collection for quantification of the Bence
Jones protein (ie, lambda light chains), protein, and creatinine clearance.
* Quantitative immunoglobulin levels (ie, IgG, IgA, IgM)
* Beta-2 microglobulin
* C-reactive protein (CRP)
* Check the serum viscosity in patients with central nervous system
(CNS) symptoms, nosebleeds, or very high M protein levels.

Imaging Studies
* Skeletal series
o Plain radiography remains the gold standard imaging procedure for
staging newly diagnosed and relapsed myeloma patients
o Perform a complete skeletal series at multiple myeloma diagnosis,
including the skull (a very common site of bone lesions in persons with
multiple myeloma), the long bones (to look for impending fractures), and
the spine.
* Magnetic resonance imaging (MRI)
o Findings from MRIs of the vertebrae are often positive when plain
radiographs are not.
o For this reason, evaluate symptomatic patients with MRI to obtain a
clear view of the spinal column and to assess the integrity of the
spinal cord.

Procedures
* Obtain bone marrow aspirate and biopsy samples
* Cytogenetic analysis of the bone marrow
Prolonged vomiting is associated with what electrolyte abnormality?

a) Hypochloremic hypokalemic metabolic acidosis


b) Hypochloremic hypokalemic metabolic alkalosis
c) Hyperchloremic metabolic acidosis
d) Hyperkalemia
The correct answer is B

Explanation
During prolonged vomiting, the patient will vomit out protons (H),
potassium ions (K) and chloride ions (Cl). This will eventually lead to
a blood work showing low potassium level, low chloride level and a shift
toward alkalosis (since protons contribute to acid state).

A 30 year old black female sees you because her hair has been falling
out over the past 2 months. She tells you that she straightened, waved,
and lightened her hair about 3 months ago. Examination shows most of the
hair loss to be in the midline of the scalp, with an area of dark hairs
about 3 cm in length in the center region. There is no evidence of
balding or scarring. A potassium hydroxide (KOH) scraping is negative.
Which one of the following is most appropriate management?

a) Psoralen plus ultraviolent light (PUVA)


b) Advising the patient to avoid hair treatment products
c) Oral itraconazole (Sporanox)
d) Topical minoxidil (Rogaine)
e) Intralesional corticosteroids
The correct answer is B

Explanation
The fact that this patient‟s hair is breaking off rather than coming out
by the roots strongly suggests that her hair loss is the result of
improper use of hair cosmetics. Findings that lend additional support to
this diagnosis include the lack of scarring, the absence of bald areas,
and the normal KOH examination for fungal elements. Bleaching and
permanent straightening products contain hydrogen peroxide, which can
reconfigure and weaken hair fiber bonds. Eventually, light to moderate
brushing will cause the hair to break off and fall out. Treatment is
preventive.
An 87-year-old female is hospitalized for suspected aspiration
pneumonia. She has recently been having difficulty swallowing, causing
her to choke and gag on both solids and liquids. The speech therapist at
the hospital requests a modified barium swallow. Prior to ordering this
you review her medications.

Which one of these medications, listed below, is a possible cause of her


dysphagia?

a) Hydrochlorothiazide
b) Citalopram (Celexa)
c) Quinapril (Accupril)
d) Digoxin (Lanoxin)
e) Risperidone (Risperidal)
The correct answer is E

Explanation
Dysphagia is associated with Parkinson‟s disease and can also occur as a
consequence of drug-induced parkinsonism. Antipsychotics are potent
antidopaminergic agents and can induce symptoms that closely mimic
Parkinson‟s disease, including dysphagia. Atypical antipsychotics such
as risperidone can also cause parkinsonism and dysphagia.

A 35 year old man is seen in the Emergency Department after attempting


suicide by taking an overdose of warfarin. Which one of the following is
used as a treatment for this circumstance?

a) Heparin
b) Allopurinol
c) Coumarin
d) Vitamin E
e) Vitamin K
The correct answer is E

Explanation
Warfarin overdose is evident mainly as excessive bleeding, which may
first be noticed as bleeding gums with brushing or easy bruising.
Vitamin K is used to reduce INR in cases of warfarin overdose.

A 55 year old woman presents with ptosis and diplopia. On examination of


the eye, the pupil is widely dilated and non-reactive to light. Which
one of the following is likely injured?

a) Oculomotor nerve
b) Trochlear nerve
c) Trigeminal nerve
d) Abducent nerve
e) Optic nerve
The correct answer is A

Explanation
The oculomotor nerve is the third of twelve paired cranial nerves. It
controls most of the eye's movements, constriction of the pupil, and
maintains an open eyelid. Since the oculomotor nerve controls most of
the eye muscles, it may be easier to detect damage to it. Damage to this
nerve, termed oculomotor nerve palsy is also known by the down n' out
symptoms, because of the position of the affected eye.

The oculomotor nerve also controls the constriction of the pupils. This
can be tested in two main ways. By moving a finger towards a person's
face to induce accommodation, as well as them going cross-eyed, their
pupils should constrict. Shining a light into their eyes should also
make their pupils constrict. Both pupils should constrict at the same
time, independent of what eye the light is actually shone on.
A 56-year-old female presents for a routine visit. An otherwise normal
physical examination reveals a 2-cm right-sided thyroid nodule. Her TSH
levels are normal. She has no history of neck irradiation, and there is
no family history of thyroid cancer.

You recommend

a) A fine-needle aspiration biopsy


b) Suppression of the nodule with levothhyroxine (Synthroid)
c) Removal of the nodule
d) A serum calcitonin level
e) A radioactive iodine uptake test and a thyroid scan
The correct answer is A

Explanation
In the absence of risk factors for cancer, a patient with a normal TSH
level who is found to have a thyroid nodule on physical examination
should have a fine-needle aspiration biopsy. Independent of morphology,
fine-needle aspiration provides the most direct and specific information
about a thyroid nodule.
A 48-year-old sedentary white male who is in excellent health presents
for a routine evaluation. He takes no medications. He smokes 1 pack of
cigarettes a day and does not drink alcohol. His laboratory work is
normal except for a urinalysis that reveals 3?5 RBCs/hpf. Results are
similar on repeat urinalysis 2 weeks later. A CT scan of the abdomen and
pelvis finds the kidneys to be normal with no evidence of stones.

Which one of the following would be the most appropriate next step?

a) A repeat urinalysis in 1 month


b) A urine culture, and if negative, a repeat urinalysis in 3 months
c) Referral to urology for cystoscopy
d) Urine cytology
e) CT of the urinary bladder
The correct answer is C

Explanation
The Urological Association recommends that all patients over 40 years of
age and those who have risk factors for bladder cancer undergo
cystoscopy to complete the evaluation of microscopic hematuria.
Cystoscopy is the only reliable method of detecting transitional cell
carcinoma of the bladder and the urethra.

Note: Voided urinary cytology may be helpful if results are positive,


but a negative cytology result cannot be considered definitive. Urinary
cytology for routine screening is controversial.

A 67 year old white male sees you for a new patient visit. He is
asymptomatic and has not seen a doctor in 10 years. He does not smoke or
drink and takes no medication. He says he has a history of “mild high
blood pressure” but has never been treated for this. His blood pressure
today is 180/90 mm Hg. He has a decreased arteriovenous ratio on
fundoscopic examination, his point of maximal intensity is displaced
laterally, and he has decreased pedal pulses. The most appropriate
management at this point would be to

a) Perform two blood pressure measurements 1 week apart to establish


the diagnosis of hypertension
b) Order ambulatory blood pressure monitoring
c) Order a laboratory workup to rule out causes of secondary hypertension
d) Order an exercise stress test
e) Prescribe a diuretic
The correct answer is E

Explanation
Elevated blood pressure along with physical findings of cardiovascular
disease establishes the diagnosis of hypertension in this patient, so it
is not necessary to take follow-up blood pressure readings prior to
starting treatment. Since he has no symptoms or physical findings
suggestive of secondary hypertension it is also not necessary to perform
a laboratory workup prior to treatment. Because he has Stage 3
hypertension with evidence of end-organ disease, treatment with
antihypertensives is indicated at this point.

At least three large clinical trials, including the European Working


Party on High Blood Pressure in the Elderly (EWPHE) trial, have shown
that diuretics are the most effective single agents for hypertension in
the elderly. A low-sodium diet can be added, as can a ?-blocker if the
hypertension fails to respond to diuretics alone.
A 52 year old woman, recently diagnosed with hypertension, presents to
her family physician complaining of hirsutism and acne. Laboratory
examination reveals: cortisol 1500 nmol/L (N: 200 - 660 nmol/L)
aldosterone 200 pmol/L (N: 140 - 415 pmol/L) ACTH 4 pmol/L (N: 4 - 22
pmol/L) Which one of the following is the most likely diagnosis‟

a) Addison's disease
b) Cushing's syndrome
c) Bartter's syndrome
d) Renal artery stenosis
e) Conn's syndrome
The correct answer is B

Explanation
In Cushing's syndrome, the level of corticosteroids is excessive,
usually from overproduction by the adrenal glands. Corticosteroids alter
the amount and distribution of body fat. Excessive fat develops
throughout the torso and may be particularly noticeable at the top of
the back. A person with Cushing's syndrome usually has a large, round
face (moon face). The arms and legs are usually slender in proportion to
the thickened trunk. Muscles lose their bulk, leading to weakness.

High corticosteroid levels over time raise the blood pressure, weaken
bones (osteoporosis), and diminish resistance to infections. The risk of
developing kidney stones and diabetes is increased, and mental
disturbances, including depression and hallucinations, may occur. Women
usually have an irregular menstrual cycle. Children with Cushing's
syndrome grow slowly and remain short. In some people, the adrenal
glands also produce large amounts of androgens (testosterone and similar
hormones), leading to increased facial and body hair in women and balding.

When doctors suspect Cushing's syndrome, they measure the level of


cortisol, the main corticosteroid hormone, in the blood. Normally,
cortisol levels are high in the morning and lower late in the day. In
people who have Cushing's syndrome, cortisol levels are very high
throughout the day.
A 28 year old male comes to the ER complaining of intermittent cramps in
his lower abdominal pain, rectal urgency and diarrhea with occasional
blood in the stools of 3 weeks duration. His condition has been
worsening progressively for the last 3 days. He has vomited several
times and feels chilly. He denies the intake of any antibiotics recently.

Vitals signs show: pulse is PR: 102/min, BP is 125/75mmHg, Temp is 39C


and RR is 16/min.

Physical exam reveals a distended and tender to palpation abdomen


without any evidence of peritoneal signs. Bowel sounds are absent.
Rectal exam shows blood and mucus.

Upright X-ray film of the abdomen discloses a hugely distended


transverse colon filled with gas.

Which of the following diseases would be the most likely precipitant of


the patient‟s condition?

a) Clostridium difficile colitis


b) Colon cancer
c) Crohn‟s disease
d) Typhoid enteritis
e) Ulcerative colitis
The correct answer is E

Explanation
This patient has classic presentation of toxic megacolon. The most
common cause of toxic colon is inflammatory bowel disease (IBD),
especially ulcerative colitis. The three week history of diarrhea in a
young person that has worsened dramatically ending in bowel obstruction
with systemic toxicity is a classic scenario of ulcerative colitis
complicated with toxic megacolon. Toxic megacolon is an emergent life
threatening condition that should be managed immediately with IV fluids,
steroids and NG tube.
Although Crohn‟s disease can also lead to a fulminant colitis, it is
very rare.

Colon cancer is very unlikely to present as toxic megacolon; moreover,


the patient‟s young age favors against this diagnosis.

Intestinal perforation, not fulminant colitis, is the classic


complication described in typhoid fever.

In the absence of antibiotics intake, pseudomembrenous colitis becomes


unlikely.
Which one of the following would be most appropriate to treat a dental
infection requiring antibiotic therapy?

a) Erythromycin
b) Penicillin
c) Dicloxacillin (Dynapen)
d) Tetracycline
e) Cefixime (Suprax)
The correct answer is B

Explanation
Many dental conditions causing inflammation do not require antibiotic
therapy. Dental caries, reversible pulpitis, gingivitis, periodontitis,
and periapical abscesses usually are treated with local procedures
without antibiotics. Cellulitis, however, requires either outpatient
antibiotics or inpatient antibiotic treatment if the cellulites spreads
to the deeper spaces of the head and neck. The antibiotic of choice,
especially for outpatient treatment, is oral penicillin G, 500 mg 3
times daily.
An elderly patient presents for treatment of head trauma. He develops
changes in behavior with frequent vomiting and headache. All of the
following are indicated initially, except

a) Head CT
b) Catheter
c) IV Fluids
d) Lumbar puncture
The correct answer is D

Explanation
CT scanning of the head is done before lumbar puncture because the most
worrisome contraindication to lumbar puncture is the suspicion of
increased ICP due to a cerebral mass lesion. Performing a lumbar
puncture in these patients may lead to either trans-tentorial or uncal
herniation and acute neurological deterioration.
A 23-year-old male with Down syndrome is brought to your office by his
parents. The patient has had a low grade fever (approximately 100?F) for
7 days. The fever is not associated with rhinorrhea, sore throat, cough,
dysuria, or shortness of breath. The parents note that their son has
complained of aches in various places, and has not been eating well for
several weeks. The physical examination is remarkable for swollen lymph
glands in multiple places, pale conjunctivae, and an 8-lb weight loss
since his last visit 1 month ago.

Which one of the following is most consistent with this patient‟s symptoms‟

a) Acute myeloid leukemia


b) Atypical pneumonia
c) Hodgkin‟s lymphoma
d) Myocarditis
e) Urinary tract infection
The correct answer is A

Explanation
This patient‟s symptoms are typical of a sub-acute, systemic illness
with signs of anemia, a presentation associated with acute myeloid
leukemia. This disease is more common in persons with Down syndrome.
There are no urinary symptoms, and the time course would not suggest a
urinary tract infection. Chest pain or shortness of breath would be
expected with myocarditis.

Other malignancies that are more common in those with Down syndrome are
acute lymphomatic leukemia, testicular cancer, and liver cancer.

In which one of the following clinical situations would it be most


appropriate to use a beta-blocker that has intrinsic sympathomimetic
activity, such as acebutolol or pindolol?

a) As a cardioprotective agent post myocardial infarction


b) In a hypertensive patient with symptomatic bradycardia while taking
metoprolol (Lopressor)
c) In a hypertensive patient with diabetes mellitus
d) In a hypertensive patient with asthma
e) To maintain sinus rhythm in a patient with chronic atrial fibrillation
The correct answer is B

Explanation
Beta-blockers with intrinsic sympathomimetic activity (ISA) are less
beneficial in reducing mortality post myocardial infarction, and for
this reason are not recommended for ischemic heart disease. They have a
potential advantage in only one clinical situation. Since they tend to
lower heart rates less, they may be beneficial in patients with
symptomatic bradycardia while taking other beta-blockers. All
beta-blockers should be used cautiously in patients with diabetes or
asthma. Only sotalol, which delays ventricular depolarization, has been
shown to be effective for maintenance of sinus rhythm in patients with
chronic atrial fibrillation.
Patients with Wolff-Parkinson-White syndrome who have episodic
symptomatic supraventricular tachycardia or atrial fibrillation benefit
most from

a) Episodic intravenous digoxin


b) Long-term oral digitalis
c) Episodic Beta-blockers
d) Radiofrequency catheter ablation of bypass tracts
The correct answer is D

Explanation
Radiofrequency catheter ablation of bypass tracts is possible in over
90% of patients and is safer and more cost effective than surgery, with
a similar success rate. Intravenous and oral digoxin can shorten the
refractory period of the accessory pathway, and increase the ventricular
rate, causing ventricular fibrillation. Beta-blockers will not control
the ventricular response during atrial fibrillation when conduction
proceeds over the bypass tract.
A 30 year old white male presents with a polymorphous skin rash
consisting of grouped vesicles urticarial wheals, and papular lesions
distributed symmetrically over the elbows, knees, and buttocks. A skin
biopsy shows IgA deposition and a diagnosis of dermatitis herpetiformis
is made.

The mainstay of therapy is

a) Dapsone
b) Prednisone
c) Cephalosporins
d) Methotrexate
e) Tetracycline
The correct answer is A

Explanation
The patient presents with a polymorphous skin rash and a biopsy
consistent with dermatiti, herpetiformis. The mainstay of treatment is
either the sulfones or sulfonamides.
A 60 year old man is admitted to the hospital because of acute
pancreatitis. Laboratory studies show:

Amylase 1,000 U/L Hematocrit 42%


Calcium 8.4 mg/dL WBC 14,000/mm3
BUN 5 mg/dL

Results of serum liver chemistry profile are normal. After 48 hours of


fluid therapy and observation, a poor prognosis would be indicated by
which of the following laboratory studies‟

a) Serum alanine aminotransferase (ALT) of 106 U/L


b) Serum amylase of 2,000 U/L
c) Serum bilirubin of 4.2 mg/dL
d) Serum calcium of 6.6 mg/dL
e) Serum glucose of 200 mg/dL
The correct answer is D
Explanation
Of the listed laboratory abnormalities (elevated ALT amylase, bilirubin,
and glucose) the one that is most suggestive of severe pancreatic
inflammation, and even necrosis, is hypocalcemia. This is presumably due
to the results of saponification of calcium by released fatty acids.
Which one of the following is most effective for reducing the risk of
both vertebral and nonvertebral fractures in women with postmenopausal
osteoporosis‟

a) Salmon calcitonin (Miacalcin)


b) Recombinant paratyphoid hormone, e.g., teriparatide (Forteo)
c) Selective estrogen receptor modulators (SERMs), e.g., raloxifene
(Evista)
d) Biphophonates, e.g., risedronate (Actonel) or alendronate (Fosamax)
e) Estrogen preparations
The correct answer is D

Explanation
Of the approved agents, biphosphonates are the most effective in
reducing the risk of vertebral and nonvertebral fractures in the setting
of osteoporosis. Salmon calcitonin is recommended for use in women with
osteoporosis who are at least 5 years post menopause and cannot take
other agents. Teriparatide is approved for a maximum of 2 years of use
in patients with severe osteoporosis who are at high risk for fractures.
It must be administrated parenterally, and the long-term effects are not
known. Raloxifene has demonstrated risk reduction for vertebral
fractures, but not for nonvertebral fractures. Estrogen preparations are
approved for the prevention, but not the treatment of osteoporosis.
Hormone therapy, however, is associated with increased risk of breast
cancer, heart attack, stroke, and blood clots in the lung, risks that
now appear to outweigh the benefits.
A female patient complains of a severe headache and jaw pain when she
chews. She also complains of shoulder pain and bilateral wrist weakness.
Initial lab tests show an ESR of 75 (normal is less than 30). What is
the most appropriate diagnosis‟

a) Rheumatoid Arthritis
b) Temporal Arteritis
c) Polymyositis
d) Sarcoidosis
The correct answer is B

Explanation
Temporal arteritis is a chronic inflammatory disease of large blood
vessels, particularly the carotid artery and its branches, occurring
primarily in older adults. Simultaneous polymyalgia rheumatica is
common. Focal symptoms and signs may include headaches, visual
disturbances, temporal artery tenderness, and pain in the jaw muscles
while chewing. Fever, weight loss, malaise, and fatigue are also common.
ESR is typically very high.

Diagnosis is most often by biopsy. Treatment with high-dose


corticosteroids is usually highly effective and prevents visual loss.
Which of the following would not be part of your plan for the treatment
of acute ventricular fibrillation?

a) Electrical defibrillation
b) Lidocaine
c) Epinephrine
d) Bretylium
e) Manganese
The correct answer is E

Explanation
Ventricular fibrillation (V-Fib) must be treated as an extreme
emergency. Cardiopulmonary resuscitation (CPR) must be started as soon
as possible, within a few minutes. It must be followed by cardioversion,
or defibrillation (an electrical shock delivered to the chest), as soon
as the defibrillator is available. Antiarrhythmic drugs may then be
given to help maintain the normal heart rhythm.

Medications involved include lidocaine, epinephrine and bretylium.


Manganese is not part of the treatment of V-Fib.
Hemolytic anemia is characterized by all of the following,*except*

a) Increased LDH
b) Increased reticulocytosis
c) Increased unconjugated bilirubin
d) Increased haptoglobin
The correct answer is D

Explanation
Hemolysis presents as acute or chronic anemia, reticulocytosis, or
jaundice. The diagnosis is established by reticulocytosis, increased
unconjugated bilirubin and lactate dehydrogenase, decreased haptoglobin,
and peripheral blood smear findings. Premature destruction of
erythrocytes occurs intravascularly or extravascularly.

The etiologies of hemolysis often are categorized as acquired or


hereditary. Common acquired causes of hemolytic anemia are autoimmunity,
microangiopathy, and infection. Immune-mediated hemolysis, caused by
antierythrocyte antibodies, can be secondary to malignancies, autoimmune
disorders, drugs, and transfusion reactions.
Which one of the following electrolyte disorders may be associated with
a prolonged QT interval on an EKG?

a) Hyperkalemia
b) Hypermagnesemia
c) Hyponatremia
d) Hypocalcemia
The correct answer is D

Explanation
In addition to specific cardiac diagnoses, the EKG also provides clues
to other significant underlying problems. Hypokalemia, hypocalcemia, and
hypomagnesemia can all cause a prolongation of the QT interval, as can
type 1A antiarrhythmics (e.g., quinidine, procainamide) or severe CNS
abnormalities (i.e., subarachnoid hemorrhage). The potentially lethal
arrhythmia torsades de pointes (polymorphic ventricular tachycardia) is
associated with the prolonged QT interval resulting from these
electrolyte disorders. Hyperkalemia, hyponatremia, and hypermagnesemia
are not associated with prolongation of the QT interval.
A 35 year old woman presents with ataxia, weakness of the left side of
her body, dysphagia and visual problems. Physical reveals diplopia and
nystagmus. What is the most likely diagnosis‟

a) Multiple sclerosis
b) Fibromyalgia
c) Dermatomyositis
d) Systemic lupus erythematosus
e) Amyotrophic lateral sclerosis
The correct answer is A

Explanation
Multiple sclerosis is characterized by disseminated patches of
demyelination in the brain and spinal cord. Common symptoms include
visual and oculomotor abnormalities, paresthesias, weakness, spasticity,
urinary dysfunction, and mild cognitive impairment. Typically,
neurologic deficits are multiple, with remissions and exacerbations
gradually producing disability.

Diagnosis is by history of remissions and exacerbations plus clinical


signs, test results, lesions seen on MRI, or other criteria (depending
on symptoms) to objectively demonstrate ? 2 separate neurologic
abnormalities. Treatment includes corticosteroids for acute
exacerbations, immunomodulatory drugs to prevent exacerbations, and
supportive measures.
Which of the following medications may cause amnesia as a side effect?

a) Hydroxyzine
b) Fluoxetine
c) Triazolam
d) Clonidine
The correct answer is C

Explanation
Triazolam is a benzodiazepine medications used for sleep disorders. It
can be addictive if used improperly. This medication can cause sedation
and, because of this, use of triazolam by seniors can contribute to
falls and accidents and cause confusion and amnesia.
By definition, patients with diastolic heart failure have an ejection
fraction greater than

a) 10%
b) 20%
c) 30%
d) 40%
e) 50%
The correct answer is E

Explanation
Epidemiologic studies have established that 40%-50% of patients with
heart failure have a normal ejection fracture (>50%) without primary
valve disease, a clinical syndrome commonly referred to as diastolic
heart failure.
A patient has the following values of Na 140 mmol/L, K 4.0 mmol/L, Cl
100 mmol/L, HCO3 24 mmol/L. Calculate the anion gap.

a) 122
b) 40
c) 20
d) 16
The correct answer is C

Explanation
Anion gap is calculated by the sum of cations minus anions as follows:
Anion gap = (Na+K)-(CL+HCO3). In this question the calculation would be:
(140+4) - (100+24) = 20.
An elderly woman who has hypertension and diabetes would benefit most
from which of the following medications‟

a) Metoprolol
b) Amlodipine
c) Hydrochlorothiazide
d) Benazepril
The correct answer is D

Explanation
Angiotensin-converting enzyme (ACE) inhibitors lower blood pressure in
part by dilating arterioles. They dilate arterioles by preventing the
formation of angiotensin II, which causes arterioles to constrict.
Specifically, these inhibitors block the action of
angiotensin-converting enzyme, which converts angiotensin I to
angiotensin II.

These drugs are particularly useful for people with coronary artery
disease or heart failure, whites, young people, people with protein in
their urine because of chronic kidney disease or diabetic kidney
disease, and men who develop sexual dysfunction as a side effect of
another antihypertensive drug.
Which one of the following is contraindicated in the treatment of
patients with cocaine-induced arrhythmias‟

a) Lidocaine
b) Sodium bicarbonate
c) Lorazepam (Ativan)
d) Verapamil (Calan, Isoptin)
e) Metoprolol (Lopressor)
The correct answer is E

Explanation
When treating arrhythmias related to cocaine toxicity, hypertonic sodium
bicarbonate and benzodiazepines may be given when the distinction
between sodium channel blockade-induced QRS-complex widening and
ischemia-induced ventricular tachycardia is unclear. Lidocaine may
subsequently be utilized if necessary. Verapamil has been shown to
reverse cocaine-induced coronary vasospasm. Beta-Adrenergic blocking
drugs have been shown to exacerbate coronary vasospasm by resulting in
unopposed alpha-adrenergic activity. Beta-Blockers are therefore
contraindicated in the treatment of cocaine-induced cardiac problems.
A 25 year old male, diagnosed with WPW 2 years ago, presents to the ER
complaining of palpitations of 1 hour duration. He denies any major
medical illness. He does not smoke but admits that he drinks caffeine
excessively.

His vitals are within normal limits except for a pulse rate of 160/min.
The EKG showed a SVT which failed to be controlled by carotid massage.
Which of the following is the most appropriate next step of management
of this patient?

a) Adenosine
b) Digoxin
c) Procainamide
d) Propranolol
e) Verapamil
The correct answer is C

Explanation
Never give a nodal blocking agent (such as Adenosine, beta blockers,
digoxin and verapamil) to a WPW patient presenting with tachyarrythmia!
You may kill him! AV nodal blocking agents will block the AV node and
allow the accessory pathway conduction to increase which may lead to a V
tach or V fib and death.

Procainamide is appropriate in the stable patient because of its useful


anti-arrhythmic property.
Which one of the following statements is true about celiac disease
(gluten-sensitivity enteropathy) in adults‟

a) It is more common among blacks


b) Symptoms are limited to gastrointestinal complaints
c) Type 2 diabetes are at increased risk for the disease
d) Serum antibody tests are sensitive and specific
e) Colonoscopy with mucosal biopsy is required to make the diagnosis
The correct answer is D

Explanation
Celiac disease is thought to be greatly underdiagnosed in Canada.
Antibody tests indicate that the prevalence is approximately 1:250 among
adult Canadians of European ancestry. Approximately 7% of type 1
diabetes have celiac disease. A number of other autoimmune syndromes
have been associated with celiac disease, including thyroid disease and
rheumatoid arthritis. There is no reported association with type 2 diabetes.

Gastointestinal involvement may manifest as diarrhea, constipation, or


other symptoms of malabsorption, such as bloating, flatus, or belching.
Fatigue, depression, fibromyalgia-like symptoms, aphthous stomatitis,
bone pain, dyspepsia, gastroesophageal reflux, and other nonspecific
symptoms may be present and can make the diagnosis quite challenging.
Dermatitis herpetiformis is seen in 10% of patients with celiac disease.

Serum antibody testing, especially IgA antiendomysial antibody, is


highly sensitive and specific and readily available at a cost of about
$100 to $200. Definitive diagnosis generally requires
esophagogastroduodenoscopy with a biopsy of the distal duodenum to
detect characteristic villous flattening.

A 30 year old woman complains to her doctor of swollen lymph nodes in


her neck and painful nodules on her lower legs. A chest x-ray shows
bilateral hilar and right paratracheal lymphadenopathy. Which one of the
following is the most likely diagnosis

a) Lymphoma
b) Metastatic thyroid carcinoma
c) Teratoma
d) Wegener's granulomatosis
e) Sarcoidosis
The correct answer is E

Explanation
Sarcoidosis is characterized by noncaseating granulomas in one or more
organs and tissues; etiology is unknown. The lungs and lymphatic system
are most often affected, but sarcoidosis may affect any organ. Pulmonary
symptoms range from none (limited disease).

Diagnosis usually is first suspected because of pulmonary involvement


and is confirmed by chest x-ray, biopsy, and exclusion of other causes
of granulomatous inflammation. Chest x-ray, or high-resolution chest CT
will show hilar and mediastinal lymphadenopathy. First-line treatment is
corticosteroids. Prognosis is excellent for limited disease but poor for
more advanced disease.
A 65-year-old male presents with a 1-month history of problems passing
urine. He says that his bladder will feel full when he needs to urinate,
but the urine stream is weak and his bladder does not feel as if it has
emptied completely. The symptoms have become increasingly severe over
the past week. Other symptoms include upper respiratory congestion for 3
days which he has treated with an over-the-counter decongestant with
some relief, constipation with no passage of stool in the past 9 days,
and increasing low back pain incompletely relieved with ibuprofen, with
associated weakness in both legs.

Examination shows a healthy-appearing male who is moderately overweight.


He is afebrile and vital signs are normal. There is no abdominal
tenderness and no masses are detected. A rectal examination reveals a
large amount of hard stool in the rectum; a markedly enlarged (4+),
boggy, tender prostate gland; laxity of the anal sphincter; and numbness
in the perianal area. Urinalysis shows trace protein and 10?20 WBCs/hpf.
Ultrasonography shows a post-void residual volume of 250 mL (normal for
age <100).

Which one of the following must be done urgently in this patient?

a) Foley catheterization
b) Hospitalization for intravenous antibiotics
c) Digital disimpaction of the rectum, and Fleet enemas until clear
d) MRI of the lumbosacral spine
The correct answer is D

Explanation
The differential diagnosis of urinary retention in the elderly is broad.
While most causes are benign and readily treated, the physician must be
vigilant in looking for conditions that require urgent intervention.

This patient presents with many possible causes of urinary retention,


with the most common being benign prostatic hyperplasia. Acute
prostatitis, especially in a male with an enlarged prostate, is another
relatively common reason for obstructive symptoms. This patient‟s
physical examination and abnormal urinalysis support this diagnosis, but
his normal vital signs and lack of fever suggest he can be treated with
an oral fluroquinolone and does not require hospital admission for
intravenous therapy.

Medications such as oral decongestants can contribute to urinary


retention in men with enlarged prostate glands, and should be used with
caution and discontinued if obstructive symptoms occur. Obstipation
(severe constipation caused by intestinal obstruction) with stool
impaction is another relatively common reason for urinary retention in
the elderly and can be treated with manual disimpaction and enemas.

In this patient, the presence of increasing low back pain and leg
weakness, and the findings of anal sphincter laxity and numbness in the
perianal area on examination, suggest the presence of a serious
neurologic etiology such as cauda equina syndrome. Urgent diagnosis and
treatment are necessary to reduce morbidity, and MRI should be performed
immediately.

The presence of a mildly elevated post-void residual is not an


indication for urgent decompression with a Foley catheter.
Dizziness is most likely to have a serious etiology when it

a) is associated with diplopia


b) is associated with intense nausea and vomiting
c) occurs when the patient rolls over in bed
d) occurs when the patient first arises in the morning
e) occurs after 2 minutes of hyperventilation
The correct answer is A

Explanation
Diplopia, along with other neurologic symptoms such as weakness or
difficulty with speech, suggests a central cause of vertigo and requires
a complete workup. Dizziness on first arising, dizziness with rolling
over in bed, and dizziness with nausea and vomiting are consistent with
peripheral causes of vertigo, such as benign positional vertigo.
Dizziness that occurs after a couple of minutes of hyperventilation
suggests a psychogenic cause.
A 16 year old male presents with history of weight loss, abdominal pain
and bloody diarrhea. Investigation shows a normal upper endoscopy and
colonoscopy. What is the most appropriate next step?

a) Upper GI series
b) CT of abdomen
c) Stool studies
d) CBC and Retic count
The correct answer is A

Explanation
Crohn's disease is a chronic transmural inflammatory disease that
usually affects the distal ileum and colon but may occur in any part of
the GI tract. Symptoms include diarrhea and abdominal pain. Abscesses,
internal and external fistulas, and bowel obstruction may arise.
Extraintestinal symptoms, particularly arthritis, may occur.
Diagnosis is by colonoscopy and barium contrast studies. If initial
presentation is less acute, an upper GI series with small-bowel
follow-through and spot films of the terminal ileum is recommended.
Treatment is with 5-aminosalicylic acid, corticosteroids,
immunomodulators, anticytokines, antibiotics, and often surgery.
A 43 year old woman with severe abdominal pain that radiates to her back
is found to have high amylase and lipase levels. Which of the following
is not part of the treatment of this illness‟

a) Analgesics
b) Antibiotics
c) NPO status
d) IV fluids
The correct answer is B

Explanation
Acute pancreatitis is inflammation of the pancreas (and, sometimes,
adjacent tissues) caused by the release of activated pancreatic enzymes.
The most common triggers are biliary tract disease and chronic heavy
alcohol intake.

The condition ranges from mild (abdominal pain and vomiting) to severe
(pancreatic necrosis and a systemic inflammatory process with shock and
multiorgan failure).

Diagnosis is based on clinical presentation and serum amylase and lipase


levels. Treatment is supportive, with IV fluids, analgesics (morphine),
and fasting (NPO status).

Antibiotics are not used in the treatment of acute pancreatitis.

A 57 year old white male with coronary artery disease suffered a cardiac
arrest while jogging 10 days ago. He was resuscitated after 45 minutes
but has remained unresponsive and on a ventilator since then. Which one
of the following is required to diagnose brain death in this patient?

a) Hypothermia
b) Continuous mechanical ventilation
c) The absence of spontaneous body movements
d) Electroencephalographic confirmation
e) Normal corneal reflexes
The correct answer is D

Explanation
An electroencephalogram (EEG) is one of the most well-validated tests
for confirming brain death. The presence of hypothermia will confound
any attempt to determine brain death; the patient must be warm to ensure
the diagnosis. The mechanical ventilator may give false readings and
must be discontinued to get a real picture of the patient‟s ability to
breathe. Spontaneous body movements can occur even hours after actual
brain death. Corneal reflexes never occur after death.
A patient admitted to the hospital has a basic metabolic panel (BMP)
drawn and sent to the lab. The serum sodium, calcium, and bicarbonate
are all normal except the serum potassium was 7.4. The patient is
asymptomatic. What is the most appropriate next step?

a) EKG
b) Repeat the BMP
c) Calcium gluconate
d) Insulin
The correct answer is B

Explanation
Very high levels of potassium in an asymptomatic patient may be due to
lab error that can occur when the blood sample undergoes hemolysis. A
repeat draw and measurement of the serum potassium is indicated in this
patient.
A 15 year old presents with blood pressure of 155/90 mmHg on routine
physical exam. He denies any symptoms. What is the most appropriate
management?

a) Low sodium diet


b) Start a diuretic
c) Repeat measurement on two more occasions
d) Order an EKG
The correct answer is C

Explanation
In the diagnosis of hypertension it is important to have at least three
high blood pressure readings before the start of drug treatment.
A 60-year-old male who has mild COPD comes to your office for evaluation
of cough, fever, fatigue, mild but increasing shortness of breath, and
occasional pain with deep inspiration. Symptoms have been present for
3-4 days, and his cough is becoming more productive.

On examination of the chest you note crackles and rales, dullness to


percussion, and tactile fremitus over the right posterior chest. His
respiratory rate is 18/min, and his temperature is 38.1° C (100.5° F).
You suspect community-acquired pneumonia.

Which one of the following is true regarding this patient‟s condition?

a) A chest radiograph is indicated


b) Sputum samples have a high diagnostic yield for bacterial pneumonia
c) Blood cultures are indicated prior to starting antibacterial drug
therapy
d) The preferred antibiotic for initial empiric management is
amoxicillin or trimethoprim/
sulfamethoxazole (Bactrim, Septra)
e) Treatment should be started in a hospital rather than an ambulatory
setting
The correct answer is A

Explanation
The Thoracic Society guidelines for managing community-acquired
pneumonia (CAP) recommend that ?all patients suspected of CAP should
receive chest radiography to establish the diagnosis and identify
complications (pleural effusion, multilobar disease).? Sputum samples
are adequate in only 52.3% of patients with CAP, and only 44% of those
samples contain pathogens. In a study of CAP in 19 Canadian hospitals
over a 6-month period, positive blood cultures were obtained in only
5.2%?6.2% of patients, including those with the most severe disease.
Based on these findings, other researchers concluded that a positive
blood culture had no correlation with the severity of illness or
outcome. ATS guidelines recommend two sets of blood cultures for
hospitalized patients, but this is not necessary for outpatient diagnosis.

Initial treatment of CAP is empiric, and macrolides, fluoroquinolones,


or doxycycline should be used in most patients, although
fluoroquinolones should be used sparingly because of increasing
resistance concerns. Amoxicillin/clavulanate and ß-lactams are
alternatives. The Pneumonia Severity Index was developed to assist
physicians in identifying patients who are at higher risk of
complications and are more likely to benefit from hospitalization. Based
on these guidelines, this patient falls into a low-risk category and can
be treated as an outpatient.
A 54-year-old African-Canadian female is admitted to the hospital for
intravenous heparin therapy for a deep-vein thrombosis of her leg. She
had previously been very healthy. Shortly after admission she begins to
experience profound orthostatic hypotension and gastrointestinal
distress. Serial hemoglobin tests are stable, but serum sodium and
bicarbonate levels are low. Her potassium level is slightly elevated.

Which one of the following tests would most quickly determine if she has
suffered from acute adrenal cortex insufficiency (Addison‟s disease)?

a) A cosyntropin stimulation test


b) A dexamethasone suppression test
c) A renin:aldosterone ratio
d) A serum adrenocorticotropic hormone (ACTH) level
e) Measurement of 24-hour urinary excretion of catecholamines
The correct answer is A

Explanation
Destruction of the adrenal cortex by acute causes (infarction,
hemorrhage, infection) or chronic ones (granulomatous disease, immune
destruction) results in a loss of circulating levels of cortisol and
aldosterone. The effects of this deficiency can include electrolyte
disturbances such as those seen in this patient, gastrointestinal
distress (including ?acute abdomen,? diarrhea, nausea, etc.),
orthostatic hypotension, and patient fatigue. If the process is gradual,
ACTH levels will increase, resulting in a darkened skin tone. Measuring
serum levels of ACTH may be helpful in determining the cause of
decreased cortisol levels, but is not a good first test of adrenal
cortex function. The dexamethasone suppresion test is used to evaluate
glucocorticoid excess.

The cosyntropin stimulation test does assess adrenal cortex function.


This test involves injecting cosyntropin, 250 µg intramuscularly or
intravenously, and measuring cortisol levels 60 minutes later. If the
result shows an inadequate response (cortisol level <495 nmol/L),
adrenal cortical function is proven to be at least 90% diminished.
A 26-year-old white female with regular periods presents to your office
with complaints of excessive hair growth. An examination reveals
moderately increased facial and body hair.

Which one of the following studies would you order initially to evaluate
androgen excess‟
a) Dehydroepiandrosterone
b) FSH
c) Free testosterone
d) Prolactin
e) 17 alpha-hydroxyprogesterone
The correct answer is C

Explanation
With mild hirsutism, regular menses, and no features to suggest a
secondary cause, it is reasonable to forgo laboratory evaluation given
the very high likelihood that the hirsutism is idiopathic. About half of
women with moderate or severe hirsutism have androgen excess. The
initial step in the workup should be measurement of plasma free
testosterone in the early morning, ideally between days 4 and 10 of the
menstrual cycle. If free testosterone is abnormal, further workup would
include an evaluation for an underlying cause.

The 2002 ACOG guidelines on hirsutism state that dehydroepiandrosterone


sulfate levels should be measured only in cases of rapid virilization.
If testosterone is elevated, ACOG recommends measurement of thyrotropin,
prolactin, and early-morning levels of 17 alpha-hydroxyprogesterone to
rule out other androgen-excess disorders. ACOG suggests considering
evaluation for Cushing‟s syndrome and other rare disorders only with
suggestive symptoms or signs.

If this woman with hirsutism had reported irregular menses consistent


with anovulation, polycystic ovary syndrome (PCOS) should be suspected
as the cause of her androgen excess and hirsutism. The workup for PCOS
should include evaluation for glucose intolerance and metabolic syndrome.
A 21 year old woman is seen in the Emergency Department with a history
of acetaminophen overdose 4 hours previously. She is drowsy with a
respiration rate of 24/minute. Treatment might include all of the
following, except

a) Acetylcysteine therapy
b) Intubation
c) Enteric charcoal
d) I.V. fluids
The correct answer is B

Explanation
Activated charcoal is given if acetaminophen is likely to still remain
in the GI tract. N-Acetylcysteine is an antidote for acetaminophen
poisoning. This drug is a glutathione precursor that decreases
acetaminophen toxicity by increasing hepatic glutathione stores and
possibly via other mechanisms.

For acute poisoning, N-acetylcysteine is given if hepatotoxicity is


likely based on acetaminophen dose or serum level. The drug is most
effective if given within 8 h of acetaminophen ingestion.

N-Acetylcysteine is equally effective given IV or po. IV therapy is


given as a continuous infusion. A loading dose of 150 mg/kg in 200 mL of
5% D/W given over 15 min is followed by maintenance doses of 50 mg/kg in
500 mL of 5% D/W given over 4 h, then 100 mg/kg in 1000 mL of 5% D/W
given over 16 h.
A 20-year-old college student is concerned about an itchy area over his
right eye that has been present for 2 days. He reports increased tearing
in this eye.

Which one of the following is the most likely diagnosis‟

a) Orf
b) Herpes ophthalmicus
c) Erysipelas
d) Contact dermatitis
e) Atopic dermatitis
The correct answer is D

Explanation
Many areas contain poison ivy/oak plants with the potential for
inflammatory local and generalized rhus dermatitis, especially in
patient‟s exposed areas. This patient has the characteristic vesicles
and sharply demarcated transverse lesions caused by brushing against the
plant. The patient must be examined thoroughly to determine whether
topical or systemic therapy should be used. The lesions pictured do not
resemble those of herpes ophthalmicus, erysipelas, or atopic dermatitis.
Orf is a viral disease with bulla formation usually found on the hands
of persons working with sheep.
In persons with recurrent pulmonary embolism, which one of the following
is the most common abnormality?

a) Antithrombin III deficiency


b) Protein C deficiency
c) Protein S deficiency
d) The presence of the factor V Leiden mutation
e) The presence of antiphospholipid antibodies
The correct answer is D

Explanation
The routine laboratory workup for a hypercoagulable state in patients
with pulmonary embolism used to include assays of antithrombin III,
protein C, and protein S. However, the levels of all three of these
coagulation-inhibiting proteins can be depressed during an acute
thrombotic state. In addition, heparin depresses antithrombin III
levels, and warfarin depresses protein C and protein S levels.

Furthermore, pregnancy and the use of oral contraceptive agents cause


protein S levels to decline. These proteins should not be measured
routinely, because the results may be misleading and because
deficiencies of antithrombin III, protein C, and protein S rarely occur.
Patients with these protein deficiencies have frequent recurrences of
venous thromboembolism during the first 1-3 years after the cessation of
anticoagulant therapy.

It is best to test for the factor V Leiden mutation because it is


responsible for the most common hypercoagulable state,
hyperhomocysteinemia, which can readily be treated with vitamins. It is
also prudent to test for the lupus anticoagulant; if it is present,
particularly intensive anticoagulation may be required. Many patients
with antiphospholipid antibodies or the lupus anticoagulant do not have
systemic lupus erythematosus.

A patient who has a chronic peptic ulcer asks for the best method for
curing this disease. Your answer is

a) Helicobacter pylori eradication


b) Proton pump inhibitors
c) H2 blockers
d) Nissen fundoplication
The correct answer is A

Explanation
H. pylori and NSAIDs disrupt normal mucosal defense and repair, making
the mucosa more susceptible to acid. H. pylori infection is present in
50 to 70% of patients with duodenal ulcers and 30 to 50% of patients
with gastric ulcers. If H. pylori is eradicated, only 10% of patients
have recurrence of peptic ulcer disease, compared with 70% recurrence in
patients treated with acid suppression alone.

Symptoms include pain often localized to the epigastrium and relieved by


food or antacids. The pain is described as burning or gnawing, or
sometimes as a sensation of hunger. The course is usually chronic and
recurrent.

Diagnosis of peptic ulcer is suggested by patient history and confirmed


by endoscopy (EGD). Treatment of gastric and duodenal ulcers requires
eradication of H. pylori when present.

Methods of decreasing acidity include a number of drugs including proton


pump inhibitos and H2 blockers.
Nissen fundoplication is a surgical treatment for chronic GERD.
An 84 year old male with advanced dementia develops a fever, mental
status changes, and bacteremia. No other history is available. What is
the most likely source of his infection?

a) A urinary tract infection


b) Pneumonia
c) Cholecystitis
d) Diverticulitis
e) Cellulitis
The correct answer is A

Explanation
Many signs and symptoms of infection that are common in younger adults
present less frequently or not at all in older adults. This patient has
no obvious source of infection, and must be treated empirically. The
other conditions listed must be considered, but the most common source
of bacteremia in older adults is a urinary tract infection.

Sleep apnea is known to be associated with

a) Hypertension
b) Respiratory muscle dysfunction
c) Carpal tunnel syndrome
d) Hypercalcemia
e) Previous tonsillectomy
The correct answer is A

Explanation
Sleep apnea is associated with hypertension, and may in fact be a cause
of hypertension. Improvement of sleep apnea can be associated with
improved blood pressure.
What is the first sign in impending foot gangrene?

a) Rest pain
b) Shiny and dry skin
c) Loss of hair
d) Loss of sensation
The correct answer is A

Explanation
Chronic critical limb ischemia is manifested by pain at rest, nonhealing
wounds and gangrene. Ischemic rest pain is typically described as a
burning pain in the arch or distal foot that occurs while the patient is
recumbent but is relieved when the patient returns to a position in
which the feet are dependent.

Intervention may include conservative therapy, revascularization or


amputation. Progressive gangrene, rapidly enlarging wounds or continuous
ischemic rest pain can signify a threat to the limb and suggest the need
for revascularization in patients without prohibitive operative risks.
Bypass grafts are usually required because of the multilevel and distal
nature of the arterial narrowing in critical limb ischemia.

Patients with diabetes are more likely than other patients to have
distal disease that is less amenable to bypass grafting. Compared with
amputation, revascularization is more cost-effective and is associated
with better perioperative morbidity and mortality. Limb preservation
should be the goal in most patients with critical limb ischemia.
In an 80 year old homebound female with constipation not adequately
responding to increased fluid and psyllium (Metamucil) supplementation,
the safest stimulant laxative to add to her regimen is

a) Senna
b) Phenolphthalein
c) Castor oil
d) Bisacodyl
e) Milk of Magnesia
The correct answer is A

Explanation
Senna has been shown to be safe, free of significant intestinal side
effects, and beneficial over the long term. Phenolphthalein and castor
oil can cause malabsorption, dehydration, lipoid pneumonia, and
cathartic colon. Bisacodyl suppositories used daily can cause rectal
burning, and oral bisacodyl can cause hypokalemia, abdominal cramps, and
vomiting. Milk of Magnesia is a saline cathartic that can cause elevated
magnesium levels and dehydration, watery stools, and fecal incontinence.
You are treating a 50 year old white male for diabetes mellitus and
hyperlipidemia. At the time of his initial presentation 1 year ago, his
hemoglobin A1C was 8.0% (N 3.8-6.4), LDL 130 mg/dL, HDL 28 mg/dL, and
triglycerides 450 mg/dL. After treatment with metformin (Glucophage) and
high-dose simvastatin (Zocor), his most recent laboratory evaluation
revealed a hemoglobin A1C of 6.2%, LDL 95 mg/dL, HDL 32 mg/dL, and
triglycerides 300 mg/dL.

The patient has not had any documented coronary or peripheral vascular
disease. His family history is positive for a myocardial infarction in
his father at age 55. He is a nonsmoker. He has a body mass index (BMI)
of 28 and has been unable to lose weight. His blood pressure is well
controlled on enalapril (Vasotec). What is the most appropriate
management of his elevated triglycerides‟

a) No specific treatment
b) Switch from metformin to a sulfonylurea such as glyburide
(Micronase, DiaBeta) or glipizide (Glucotrol)
c) Addition of a fibrate such as gemfibrozil (Lopid) or fenofibrate
(Tricor)
d) Addition of cholestyramine (Questran)
The correct answer is C

Explanation
Although the significance of elevated triglycerides and a low HDL in
low-risk patient is somewhat uncertain, in a high-risk patient such as a
diabetic, improvement in these results will lower the risk of subsequent
cardiac events. In diabetics, metformin and thiazolidinediones (e.g.,
rosiglitazone) are more likely to improve lipid levels than are
sulfonylureas. Nucotinic acid is problematic in diabetics, as it tends
to cause deterioration in glucose control. Fibrates are good choices for
this patient because they will lower the triglyceride level and raise
the HDL level. Exercise and weight loss are likely to be helpful as
well. Cholestyramine will raise triglyceride levels.
A 76-year-old female is hospitalized for fever and weakness of several
days‟ duration. Her history and physical findings are otherwise
unremarkable except for a temperature of 37.1° C (100.2° F), a pulse
rate of 100 beats/min, and a blood pressure of 95/55 mm Hg. A urinalysis
reveals 10-15 WBCs/hpf and a urine culture reveals methicillin-sensitive
Staphylococcus aureus. The most appropriate action at this point is to

a) reculture the urine, as the bacteria on the first urine culture is


most likely a skin
contaminant
b) obtain a blood culture and examine the patient for a portal of entry
c) obtain a blood culture and start the patient on intravenous
vancomycin (Vancocin)
d) start the patient on oral cephalexin (Keflex)
The correct answer is B

Explanation
Staphylococcus aureus is an unusual genitourinary pathogen; when found
in the urine, it should be assumed to have migrated from a primary
location. The patient should be examined carefully for a portal of entry
such as a skin ulcer, intravenous site, or area of dermatitis. An
echocardiogram is often required to rule out endocarditis.
Methicillin-sensitive S. aureus can be treated with a
penicillinase-resistant penicillin or a first-generation cephalosporin.
Vancomycin should be reserved for treating methicillin-resistant S.
aureus. Although oral cephalexin can be used to treat
methicillin-sensitive S. aureus, this particular patient is too ill and
needs to be evaluated for bacteremia.
Which one of the following benzodiazepines has the shortest elimination
half-life in the body?

a) Triazolam
b) Clorazepate
c) Chlordiazepoxide
d) Nitrazepam
e) Diazepam
The correct answer is A

Explanation
Triazolam has the shortest half-life of any benzodiazepine at 3 hours.
Alprazolam (Xanax) has a half life of 12 hours. Lorazepam (Ativan) has a
half-life 14 hours. Diazepam (Valium) has a half life of 43 hours.

A 66 year old diabetic man presents with constriction of the pupil,


drooping of the upper lid and enophthalmos on the left. Which one of the
following nerves is most likely involved?

a) Oculomotor
b) Sympathetic
c) Trochlear
d) Trigeminal
e) Abducent
The correct answer is B

Explanation
This patient has symptoms of miosis (constriction of the pupil), ptosis
(drooping of the upper lid) and enophthalmos (is recession of the
eyeball within the orbit). These are symptoms of Horner syndrome. Which
is caused by the interruption of the sympathetic nerve fibers. This can
occur centrally (ie, between the hypothalamus and the fibers' point of
exit from the spinal cord [C8 to T2]) or peripherally (ie, cervical
sympathetic chain, superior cervical ganglion, along the carotid artery).
A 25 year old male patient is experiencing inability to swallow and has
speech difficulty. The patient was in perfect health prior to the
consumption of home-canned green beans. Which one of the following is
the most appropriate treatment?

a) Administration of antitoxin
b) Penicillin administration
c) Administration of tetanus toxoid
d) Placement of the patient in a hyperbaric oxygen chamber
The correct answer is A

Explanation
Botulism is neuromuscular poisoning from Clostridium botulinum toxin.
Home-canned foods are the most common sources, but commercially prepared
foods have been implicated in about 10% of outbreaks. Vegetables, fish,
fruits, and condiments are the most common vehicles, but beef, milk
products, pork, poultry, and other foods have been involved.

Symptoms of bulbar paresis (eg, dysarthria, dysphagia, dysphonia, a


flaccid facial expression) develop. If diagnosed early, food-borne and
wound botulism can be treated with an antitoxin that blocks the action
of neurotoxin circulating in the blood. The trivalent antitoxin is
effective against three neurotoxins: A, B, and E.
A 67 year old smoker with a history of pulmonary tuberculosis at 22
years of age presents with a 6 month history of increasing shortness of
breath. On office spirometry his FVC is 60% of predicted, his FEV1 is
80% of predicted, and his FEV1 / FVC ratio is 0.8.

What is the most appropriate next step?

a) Refer to a pulmonary laboratory for static lung volume measurement


and diffusion studies
b) Perform a bronchodilator challenge test
c) Investigate for nonpulmonary causes for the patient‟s symptoms
d) Order a PPD skin test
e) Prescribe albuterol
The correct answer is A

Explanation
Pulmonary function tests are usually classified as normal, compatible
with a restrictive defect, or consistent with obstructive airway
disease. In restrictive ventilatory processes, the FVC is decreased, the
FEV1 is decreased or normal, and the absolute FEV1 /FVC is >0.7. In
obstructive airway problems, findings include a normal or decreased FVC,
a decreased FEV1 , and an absolute FEV1 /FVC <0.7.

When simple spirometry suggests a restrictive ventilatory problem, the


patient should undergo full pulmonary function testing for static lung
volume measurements and diffusing capacity of the lung for carbon
monoxide. If spirometry suggests an obstructive problem, it should be
repeated after administering an inhaled bronchodilator. Because this
patient has a history of pulmonary tuberculosis, a TB skin test is
inappropriate and would not provide any useful information.
Which one of the following is a known risk factor for necrotizing
soft-tissue infections‟

a) Age <25 years


b) Direct inguinal hernia
c) Hyperalbuminemia
d) Diabetes mellitus
The correct answer is D

Explanation
Known risk factors for necrotizing soft-tissue infections include age
>50 years, atherosclerosis, burns, cancer or other immunocompromised
states, chronic alcoholism, corticosteroid use, diabetes mellitus,
hypoalbuminemia, intravenous drug abuse, malnutrition, obesity, occult
diverticulitis, peripheral vascular disease, postoperative infection,
strangulated femoral hernia with content extravasation, and trauma.
A 42-year-old previously healthy white female presents to your office
with her third episode of abdominal pain. This episode began 2 hours
ago, and the pain is improving. She describes colicky right upper
quadrant and epigastric pain. On examination you note mild right upper
quadrant tenderness, with otherwise unremarkable findings. Renal
function tests are normal. Which one of the following would be most
appropriate at this point?

a) KUB films
b) Ultrasonography of the right upper quadrant
c) Abdominal CT with intravenous contrast
d) Abdominal CT with intravenous and oral contrast
e) MRI of the abdomen
The correct answer is B

Explanation
Ultrasonography of the right upper quadrant is recommended as the
initial imaging study for right upper quadrant pain (SOR C). KUB films
can detect kidney stones but may miss gallstones. CT also may miss
gallstones, and is more invasive than ultrasonography. Abdominal CT with
intravenous contrast is the preferred test for right lower quadrant
pain, and abdominal CT with intravenous and oral contrast is preferred
for left lower quadrant pain. MRI is preferred for detecting tumors, and
is inappropriate as the initial imaging study for right upper quadrant pain.

Tension pneumothorax is best diagnosed with

a) Stat CT scan
b) Chest x-ray
c) Watch and wait
d) Clinical exam
The correct answer is D

Explanation
Tension pneumothorax is a pneumothorax causing progressive rise in
intrapleural pressure to levels that become positive throughout the
respiratory cycle and collapse the lung, shift the mediastinum, and
impair venous return to the heart. Air continues to get into the pleural
space but cannot exit. Without proper treatment, the impaired venous
return can cause systemic hypotension and respiratory and cardiac arrest
within minutes.

Tension pneumothorax most commonly occurs in patients receiving


positive-pressure mechanical ventilation (particularly during
resuscitation). It is rarely a complication of traumatic pneumothorax,
when a chest wound acts as a one-way valve that traps increasing volumes
of air in the pleural space with inspiration. Diagnosis is based on
physical examination and chest x-ray. Most pneumothoraces require needle
thoracentesis or tube thoracostomy.
An 84-year-old black female is brought to your office by her daughter,
who is concerned that the mother has memory problems and is neglecting
to pay her monthly bills. The mother also is forgetting appointments and
asks the same questions repeatedly. This problem has been steadily
worsening over the last 1-2 years.

The patient has very little insight into her problems, scores 24 out of
a possible 30 points on the Mini-Mental State Examination, and has
difficulty with short-term recall and visuospatial tasks. Her physical
examination and a thorough laboratory workup are normal. A CT scan of
the brain reveals diffuse atrophy.

Which one of the following is the most likely etiology for the patient‟s
memory problem?

a) Alzheimer‟s disease
b) Dementia resulting from depression
c) Lewy body dementia
d) Multi-infarct dementia
e) Normal aging
The correct answer is A

Explanation
The patient shows classic symptoms of early Alzheimer‟s disease, with
difficulties in at least two cognitive domains that are severe enough to
influence daily living. Normal aging changes can decrease one‟s ability
to retrieve information but do not influence daily living and are
usually noticed more by the patient than by family members. Depression
was previously thought to cause “pseudodementia” with significant
regularity. However, several recent studies have shown that treating
depressive symptoms does not result in significantly improved cognitive
performance. It is now believed that progressive memory loss frequently
results in depressive symptoms, rather than the converse. Lewy body
dementia is associated with physical findings of parkinsonism and often
the presence of visual hallucinations, both of which are absent in this
patient. There are no signs of multiple infarcts on brain imaging,
effectively ruling out this diagnosis.
A 24 year old woman is seen because of high fever, prostration
(exhaustion), vomiting, and diarrhea. Her pulse is rapid and thready,
and her blood pressure is recorded at 60/40 mmHg. A diffuse generalized
macular rash is noted. Following stabilization with fluid and vasoactive
medications, culture of which of the following specimens will most
likely lead to the correct diagnosis‟

a) Sputum
b) Urine
c) Stool
d) Cervicovaginal secretions
e) Cerebrospinal fluid
The correct answer is D

Explanation
Toxic shock syndrome is caused by staphylococcal or streptococcal
exotoxins. Onset is sudden, with fever (39° to 40.5°C, which remains
elevated), hypotension, a diffuse macular erythroderma, and involvement
of at least 2 other organ systems.

The staphylococcal TSS rash is more likely to desquamate, particularly


on the palms and soles, between 3 and 7 days after onset.

Diagnosis is made clinically and by isolating the organism. Specimens


for culture should be taken from any lesions, the nose (for
staphylococci), throat (for streptococci), vagina (for both), and blood.
Treatment includes antibiotics, intensive support, and immunoglobulin.
Which of the following is true about nasopharyngeal carcinoma

a) Is most common in people from south-east Africa


b) Presents early with nasal pain
c) Is treated by wide surgical excision
d) Is monitored by measurement of Epstein-Barr virus antibodies
e) Is curable in 90% of patients
The correct answer is D

Explanation
Nasopharyngeal cancers are rare in the US but common in the South China
Sea region. Epstein-Barr virus is a significant risk factor, and there
is hereditary predisposition. Symptoms develop late, including
unilateral bloody nasal discharge, obstruction, facial swelling, and
numbness. Diagnosis is based on inspection and biopsy, with MRI to
evaluate extent. Treatment is with radiation, chemotherapy, and, rarely,
surgery.
A 60-year-old male was seen in your clinic 2 days ago for heart failure
and hypertension. You initiated an ACE inhibitor to address both of
these conditions. His current medications include lisinopril (Prinivil,
Zestril), metoprolol (Toprol), and low-dose aspirin. He called the
clinic today with some difficulty swallowing and a swollen upper lip. He
denies any hives.

The patient‟s complaints are most likely due to

a) Ludwig‟s angina
b) cholinergic urticaria
c) dependent edema
d) angina pectoris
e) angioedema
The correct answer is E

Explanation
Angioedema is a subcutaneous or deep tissue edema from mast cell release
(as in an allergic reaction) or an increase in vascular permeability
from bradykinin release (as seen with ACE inhibitors) or complement
system abnormalities. Ludwig‟s angina is a cellulitis in the
submandibular or sublingual areas. Cholinergic urticaria is manifested
by hives associated with sweating or hot showers. Dependent edema is
swelling in the lower or dependent portions of the body and can be
associated with calcium channel blockers. Angina pectoris is not a
well-documented cause of angioedema.
Which one of the following drugs or groups of drugs is contraindicated
in the management of patients with moderately severe ulcerative colitis‟

a) Antibiotics
b) Hydrophilic bulking agents
c) Corticosteroids
d) Narcotic antidiarrheal agents
e) Oral sulfasalazine
The correct answer is D

Explanation
Ulcerative colitis (UC) is a chronic disease in which the large
intestine becomes inflamed and ulcerated (pitted or eroded), leading to
flare-ups (bouts or attacks) of bloody diarrhea, abdominal cramps, and
fever. The long-term risk of colon cancer is increased.

Patients taking antidiarrheal drugs must be closely monitored to avoid


precipitating toxic megacolon.

Toxic megacolon can be spontaneous in people with particularly severe


UC. In some cases it can result from the overuse of certain drugs
including narcotics, drugs used for pain relief, anticholinergics, drugs
used for depression, anxiety, and nervousness; and antidiarrheals, such
as loperamide.
In a 70 year old female with a history on angina tells you about
yesterday's ?gripping? and ?pressure like? chest pain that went away on
its own after 15mins. Her resting ECG is normal. What is the most
appropriate diagnostic test for the above patient?

a) EECP
b) Angiography
c) Stress testing
d) Thallium imaging
e) Chest X-ray
The correct answer is C

Explanation
Answer: C ? Graded exercise stress testing is the most widely used test
for the evaluation of patients presenting with chest pain. In patients
with established stable angina pectoris, it also can provide prognostic
information about the extent of disease.
Exercise stress testing can be performed alone and in conjunction with
echocardiography or myocardial perfusion scintigraphy tests. Stress
echocardiography has an overall sensitivity of 78% and specificity of
86%; myocardial perfusion scintigraphy has an overall sensitivity of 83%
and specificity of 77%.

A. EECP is for therapeutic use. In patients whose angina is refractory


to medical therapy who are not suitable candidates for either
percutaneous or surgical revascularization, enhanced external
counterpulsation (EECP) is a safe and noninvasive alternative therapy.
B. Consider coronary angiography in symptomatic patients with
inconclusive noninvasive study results, in survivors of sudden cardiac
death, in those who are considered to have a poor prognosis based on the
results of noninvasive studies, in those with occupational requirements
for a definite diagnosis (eg, pilots), or in patients with coronary
artery disease who are severely symptomatic despite maximal medical therapy.
D. Thallium Tl 201 and technetium Tc 99m sestamibi are the most
frequently used myocardial perfusion scintigraphy tests. These tests are
especially useful in patients with baseline ECG abnormalities, to
localize the region of ischemia, and as prognostic indicators.
E. Chest radiograph findings are usually normal in patients with angina
pectoris

Contraindications to thrombolytic therapy in treatment of pulmonary


embolism include which one of the following?

a) Age > 80
b) Intracranial neoplasms
c) Blood pressure of 140/85
d) GI bleeding 6 months ago
The correct answer is B

Explanation
Contraindications to thrombolytic therapy include eye or central nervous
system surgery within the preceding 2 weeks, intracranial neoplasms or
vascular abnormalities, stroke within the preceding 2 months, active
bleeding, severe hypertension, and allergy to thrombolytic agents. Age
is not a consideration.
A 23 year old female presents with a fibroadenoma of the left breast in
the lower medial quadrant. All of the following are possible options, except

a) Ultrasound
b) Fine needle aspiration
c) Mammography
d) Observe for a few cycles
The correct answer is C

Explanation
Fibroadenomas are typically painless lumps that feel like small,
slippery marbles. They usually develop in young women, often in
teenagers, and may be mistaken for cancer although they are benign and
tend to be more circumscribed and mobile. Simple fibroadenoma does not
appear to increase risk of breast cancer, complex fibroadenoma may
increase risk slightly.

Below age 30, breasts tend to be "radiographically dense," which means


the breasts contain a large amount of glandular tissue which is
difficult to image in fine detail. Mammograms for this age group is
usually not done. An ultrasound of the breasts is usually done instead.
A patient with flank pain has an intravenous urography (IVU), which show
the calyces of the kidney are seen blunted. Dye excretion is delayed in
the affected kidney. A 2-3mm stone is seen in the pelvis in the lower
1/3rd of the ureter. What is the appropriate management?

a) Analgesics and hydration


b) Lithotripsy
c) Percutaneous stone removal
d) Remove the stone by retrograde cystoscopy
The correct answer is A

Explanation
A ureteral stone can cause severe abdominal pain, which will radiate to
the corresponding flank region. Diagnosis is made by imaging with a CT
or Plain x-ray or as in this case, an IVU. Small stones (less than 5mm)
may pass on their own with aggressive hydration. Larger stones (greater
than 5 mm) may need to be pulverized with lithotripsy or removal.
A 27 year old man with advanced HIV is sent for evaluation of his
nephrotic syndrome. His blood pressure is 142/84 mm Hg. He has 3+ edema
in both legs. His risk factor for AIDS is his IV heroin use. His
creatinine is 2.1 mg/dL, and his urine reveals +3 protein, no blood. A
kidney biopsy would most likely reveal which of the following?

a) Diabetic nephropathy
b) Focal glomerular sclerosis
c) IgA nephropathy
d) Membranous nephropathy
e) Minimal change disease
The correct answer is B

Explanation
Focal glomerular sclerosis is the type of nephropathy most commonly seen
in IV drug users with AIDS. It is likely to lead to a very rapid loss of
renal function.

There is no clinical evidence to indicate that this person has diabetes,


making diabetic nephropathy unlikely. Minimal change disease and IgA
nephropathy and membranous nephropathy are only very rarely associated
with AIDS.
A 75-year-old male is brought to your office by his wife, who states
that he has had mental difficulties in recent months, such as not being
able to balance their checkbook or plan for his annual visit with the
accountant. He was able to capably perform these activities in the past.
She also tells you that he has reported seeing animals in the room with
him that he can describe vividly. He naps for 3 or more hours each day,
and stares blankly for long periods of time. He seems almost normal at
times, but appears very confused at other times. This confusion seems to
come and go randomly. He also has been dreaming a lot, and has fallen
more than once recently. His only medication is aspirin, 81 mg/day.

On examination the patient walks slowly with a somewhat stooped posture


and almost falls when turning around. He has only minimal facial
expressiveness. No tremor is noted, and the remainder of the examination
is normal. He is able to recall three words out of three, but clock
drawing is abnormal. Laboratory studies are normal, and a CT of the
brain shows changes of aging.

What type of dementia does this patient most likely have?

a) Dementia with Lewy bodies


b) Dementia of Parkinson‟s disease
c) Alzheimer‟s disease
d) Frontotemporal dementia
e) Vascular dementia
The correct answer is A
Explanation
This patient has dementia with Lewy bodies, which is the second most
common histopathologic type of dementia after Alzheimer‟s disease. He
demonstrates typical symptoms and signs of dementia with Lewy bodies,
including well-formed hallucinations, vivid dreams, fluctuating
cognition, sleep disorder with periods of daytime sleeping, frequent
falls, deficits in visuospatial ability (abnormal clock drawing), and
REM sleep disorder (vivid dreams). In Alzheimer‟s disease the
predominant early symptom is memory impairment, without the other
symptoms found in this patient. In dementia of Parkinson‟s disease,
extrapyramidal symptoms such as tremor, bradykinesia, and rigidity
precede the onset of memory impairment by more than a year. Patients
with vascular dementia have risk factors and symptoms of stroke.
Frontotemporal dementia presents with behavioral changes, including
disinhibition, or language problems such as various types of aphasia.
An S_4 is UNLIKELY to be present in patients with

a) Aortic stenosis
b) Significant systemic hypertension
c) Idiopathic hypertrophic subaortic stenosis
d) Pulmonary hypertension
e) Atrial fibrillation
The correct answer is E

Explanation
An important diagnostic clue in a patient with suspected congestive
heart failure is an S4 gallop, usually associated with decreased
ventricular compliance. All of the listed clinical situations can
exhibit an S4 gallop except atrial fibrillation, since no atrial ?kick?
is present to produce the gallop sound.
A 68 year old man presents to the emergency room with a four day history
of worsening headaches and confusion. He is on no medications. Physical
examination confirms the confusion, but is otherwise normal. His plasma
sodium is found to be 117 mmol/L (normal 135 - 145 mmol/L) and plasma
osmolality is 248 mmol/kg H20 (normal 285 - 295mmol/kg H20). Which one
of the following is the most likely diagnosis‟

a) Hypoadrenalism
b) Syndrome of inappropriate antidiuretic hormone secretion
c) Congestive heart failure
d) Pseudohyponatremia
e) Hyperthyroidism
The correct answer is B

Explanation
A cause of hyponatremia is the syndrome of inappropriate secretion of
antidiuretic hormone (SIADH). People who have SIADH have a low sodium
level because the pituitary gland secretes too much antidiuretic
hormone. Hyponatremia also occurs in people who have underactive adrenal
glands and who thus excrete too much sodium in the urine. This can
result in a low plasma osmolality due to the excess water being brought
back into the bloodstream by the excess ADH.

When the sodium level in the blood falls quickly, symptoms tend to
develop rapidly and be more severe. The brain is particularly sensitive
to changes in the sodium level in the blood. Therefore, symptoms such as
lethargy and confusion occur first. As hyponatremia becomes more severe,
muscle twitching and seizures may occur; stupor, coma, and death may
follow. The diagnosis of hyponatremia is made by measuring the sodium
level in the blood.
A 78 year old man develops prostate cancer and is also found to have
metastasis. Which of the following is not an appropriate treatment?

a) Androgen blockade
b) Castration
c) Prostatectomy
d) Watchful waiting
The correct answer is D

Explanation
Patients with a locally advanced tumor or metastases may benefit from
androgen deprivation by castration, either surgically with bilateral
orchiectomy or medically with luteinizing hormone-releasing hormone
(LHRH) agonists, such as leuprolide.

Radical prostatectomy (removal of prostate with adnexal structures and


regional lymph nodes) is probably best for patients < 70 with a tumor
confined to the prostate.

Most patients, regardless of age, prefer definitive therapy. However,


watchful waiting may be appropriate for asymptomatic patients > 70 with
localized prostate cancer.
Which one of the following is associated with a group A ß-streptococcal
infection?

a) Guttate psoriasis
b) Lichen planus
c) Pityriasis rosea
d) Bullous impetigo
e) Roseola
The correct answer is A

Explanation
In guttate psoriasis there is an acute onset of small, widely scattered,
uniform lesions, often following a streptococcal infection. Bullous
impetigo is caused by a dermatologic toxin produced at the site of an
infection with Staphylococcus aureus. Exanthem subitum (roseola, sixth
disease) is caused by human herpesvirus 6. Lichen planus is a
papulosquamous disorder with an unknown etiology, but cutaneous
eruptions clinically resembling lichen planus have been observed after
administration of numerous drugs, including diuretics, gold,
antimalarials, penicillamine, and phenothiazines, and in patients with
skin lesions of chronic graft-versus-host-disease. Additionally, lichen
planus associated with abnormal liver function has been correlated with
viral hepatitis, particularly hepatitis C infection. Pityriasis rosea is
a papulosquamous eruption of unknown etiology that occurs more commonly
in the spring and fall. Its first manifestation is the development of a
2- to 6-cm annular lesion (the herald patch). There is some evidence
that it has a viral origin.
A frail 83-year-old male with a 10-year history of diabetes mellitus is
admitted to a nursing home. His blood glucose level, which he rarely
checks, is typically over 200 mg/dL. His serum creatinine level is 1.9
mg/dL. He also has had several episodes of heart failure. His current
medications include glipizide (Glucotrol), lisinopril (Prinivil,
Zestril), and furosemide (Lasix). Which one of the following would be
most appropriate to add to this patient‟s regimen to treat his diabetes
mellitus‟

a) The diabetic 1800-calorie/day diet


b) Metformin (Glucophage)
c) Pioglitazone (Actos)
d) Exenatide (Byetta)
e) Insulin glargine (Lantus)
The correct answer is E

Explanation
For geriatric patients in long-term care facilities, the predictable
glucose control of glargine is the best approach to consider initially.
The current guidelines do not recommend a strict diet for frail diabetic
patients in nursing homes. Exenatide is not recommended for the frail
elderly because of concerns about weight loss and nausea. Heart failure
precludes the use of pioglitazone, and renal failure precludes the use
of metformin.
An 82-year-old white female visits your office. She reports a 2-week
history of shoulder stiffness that is severe in the morning, and a 3-day
history of pain on the left side of the face while chewing food. She
denies dental pain or sensitivity. Her family history is negative for
rheumatologic disorders.

Your evaluation reveals a 5-lb weight loss and evidence of mild


depression. Her temperature is 38.0°C (100.4°F), she has no dental decay
or carotid bruit, and her left temporal scalp is tender. Her lungs are
clear to auscultation, there are no abnormal heart sounds, and her
abdomen is nontender. She has no joint swelling or warmth, but
experiences discomfort with shoulder and hip range of motion.

Laboratory analysis revelas a hemoglobin level of 11.0 g/dL (N 11.7 ?


16.1), an erythrocyte sedimentation rate of 80 mm/hr (N 0 ? 30), a serum
glucose level of 120 mg/dL, and a BUN level of 24 mg/dL (N 8 ? 23).

Which one of the following is the greatest immediate risk for this patient?

a) Monocular vision loss


b) Hemiparesis
c) Seizure
d) Sudden death
e) Adhesive capsulitis of the shoulder
The correct answer is A

Explanation
The most dreaded acute complication for this patient would be monocular
visual loss. The patient has findings suggestive of polymyalgia
rheumatica with probable associated temporal arteritis. These conditions
may be different manifestations of the same disease process. Shoulder
and thigh pain are typical, and morning stiffness, low-grade fever, and
weight loss are common. Jaw claudication and depression may be
associated findings, and paralysis and stroke develop less frequently.
Intracranial arteries are usually spared, but the verterbral basilar
arteries may be involved. Sudden death with arteritis of the coronary
arteries is extremely rare. Seizures are not characteristic. Adhesive
capsulitis can develop as a secondary manifestation of any shoulder
disorder that limits mobility. This tends to develop gradually rather
than acutely, and can usually be avoided with appropriate therapy.

A previously healthy 42-year-old male who is not on any medications has


blood drawn for a routine preemployment evaluation. Laboratory studies
are normal except for a potassium level of 5.4 mmol/L (N 3.5-5.0).

Which one of the following would be the most appropriate next step?

a) Obtain an EKG to rule out T-wave abnormalities


b) Repeat the blood sample collection
c) Prescribe oral sodium polystyrene sulfonate (Kayexalate) and recheck
the potassium level in 2 days
d) Administer intravenous calcium
e) Administer intravenous insulin following by an inhaled ?_2 -agonist
The correct answer is B

Explanation
Given the absence of underlying renal problems or medication use, the
most likely cause of this patient‟s elevated potassium level is
pseudohyperkalemia. Pseudohyperkalemia occurs when the laboratory value
does not accurately reflect actual serum values. The most common cause
is lysis of red blood cells during phlebotomy collection. It can be
excluded by repeating the blood sample collection as nontraumatically as
possible.

Potassium values less than 6mEq/L are not usually clinically significant
and do not cause EKG changes. Treatment with sodium polystyrene
sulfonate, intravenous calcium, intravenous insulin, and/or an inhaled
beta_2 -agonist are necessary only if pseudohyperkalemia has been ruled out
and/or there are clinical indicators such as EKG changes, a rapid rise
of serum potassium, decreased renal function, and/or significant acidosis.
A 68 year old white female presents to your office and reports that
yesterday she had a 20-minute episode of difficulty speaking and
weakness of the right side of the face and right arm.

She has never experienced any episodes similar to this in the past and
reports her overall health to be excellent. In fact, she tells you that
she has not seen a physician since her hysterectomy for fibroids 20
years ago. Her only medication is occasional acetaminophen or ibuprofen
for knee pain. Physical examination reveals a blood pressure of 160/90
mm Hg, an irregularly irregular heartbeat with a rate of 90/min, and
otherwise normal cardiovascular examination, and a completely normal
neurologic examination. Her EKG confirms atrial fibrillation with
evidence for left ventricular hypertrophy but no Q waves or ST
elevation. You are able to obtain an emergent CT scan of the brain
without contrast, which is negative. Which one of the following is the
most appropriate immediate management?

a) Lowering blood pressure


b) Antiplatelet therapy with clopidogrel (Plavix)
c) Anticoagulation with heparin.
d) Electrical or chemical cardioversion
e) An MRI scan of the brain with contrast
The correct answer is C

Explanation
The patient described presents with a history most consistent with a
recent, resolved transient ischemic attack (TIA). This was most likely
due to an embolus related to her atrial fibrillation. Her risk for a
recurrent neurologic event (TIA or stroke) is high. Long-term
anticoagulation with warfarin reduces this risk. The use of antiplatelet
agents such as clopidogrel to reduce TIAs has not been studied. Lowering
blood pressure and lipid levels can reduce risks over the long term, but
do not require immediate intervention.

Cardioversion for patients with atrial fibrillation of uncertain or long


duration may be appropriate but should not be attempted before several
weeks of anticoagulation in the stable patient. Because cardioversion
can lead to systemic emboli, heparin should be given before medical
cardioversion is attempted. Anticoagulation with warfarin (Coumadin)
should be continued for four weeks after cardioversion.
Promoting good sleep hygiene is basic in the treatment of insomnia.
Which one of the following measures will aid in promoting healthy sleep
habits‟

a) Vigorous evening exercise


b) Taking an enjoyable book or magazine to bed to read
c) Drinking a glass of wine as a sedative before retiring
d) Eating the heaviest meal of the day close to bedtime
e) Maintaining a regular sleep/wake schedule
The correct answer is E

Explanation
Maintaining a regular sleep/wake schedule helps prevent insomnia. While
a light snack before bed may be sleep inducing, heavy meals close to
bedtime may be counterproductive. Alcohol should be avoided as a
sedative, to prevent midsleep awakenings. Hours spent reading or
watching television in bed can lead to long awakenings in the middle of
the night.
A 20 year old presents with fever and non productive cough, but is
clinically well. Chest X-ray shows bilateral basal Infiltration. What is
the most appropriate treatment for suspected mycoplasma pneumonia?

a) Erythromycin
b) Doxycycline
c) Levaquin
d) Penicillin
The correct answer is A

Explanation
Community-acquired pneumonia develops in people with limited or no
contact with medical institutions or settings. The most commonly
identified pathogens are Streptococcus pneumoniae, Haemophilus
influenzae, and atypical organisms (ie, Chlamydia pneumoniae, Mycoplasma
pneumoniae, Legionella).

Symptoms and signs are fever, cough, dyspnea, tachypnea, and


tachycardia. Diagnosis is based on clinical presentation and chest
x-ray. Treatment is with empirically chosen antibiotics. Prognosis is
excellent for relatively young and/or healthy patients, but many
pneumonias, especially when caused by S. pneumoniae and influenza virus,
are fatal in older, sicker patients.

Treatment for mycoplasma pneumonia is mostly with a macrolide such as


erythromycin.
Which one of the following tests is most specific for diagnosing
rheumatoid arthritis‟

a) Anti-cyclic citrullinated peptide (anti-CCP) antibody


b) Antinuclear antibody
c) Erythrocyte sedimentation rate
d) Serum complement levels
e) Anti-Sm antibody
The correct answer is A

Explanation
Rheumatoid arthritis is primarily a clinical diagnosis and no single
laboratory test is considered definitively diagnostic. Anti-cyclic
citrullinated peptide (anti-CCP) antibody is recommended by
rheumatologists to improve the specificity of testing for rheumatoid
arthritis. Anti-CCP is more specific than rheumatoid factor, and may
predict erosive disease more accurately.

Antinuclear antibody has limited usefulness for the diagnosis of


rheumatoid arthritis. Sm antigen is useful to help diagnose systemic
lupus erythematosus. Nonspecific changes in complement levels are seen
in many rheumatologic disorders. The erythrocyte sedimentation rate is
useful in monitoring disease activity and the course of rheumatoid
arthritis, but is not specific.
Low to normal follicle-stimulating hormone levels (FSH) are found in
patients with

a) Resistant ovary syndrome


b) Previous pelvic radiotherapy for cervical cancer
c) Anorexia nervosa
d) Pure gonadal dysgenesis
e) Premature ovarian failure
The correct answer is C

Explanation
A low caloric intake not only interferes with the nutritional needs of a
young and growing organism, but also interferes with the homeostatic
mechanisms necessary to maintain functioning. Undernutrition and weight
loss associated with anorexia nervosa either reverse pubertal changes
and prevent menarche from occurring (primary amenorrhea) or regress the
hyperthalamo-pituitary-gonadal axis to prepubertal regulation, resulting
in low LH, low FSH and minimal estrogen and progesterone plasma levels
(secondary amenorhhea).
A 50-year-old accountant presents with the recent onset of right frontal
headaches. The patient has an eruption on his forehead. He reports that
he developed both the headache and the skin lesions 6 days ago. He also
has noticed mild malaise and arthralgias.
In addition to your evaluation, you should refer this patient to

a) An allergist
b) An ophthalmologist
c) A dermatologist
d) A neurologist
e) A general internist
The correct answer is B

Explanation
This case of herpes zoster ophthalmicus requires antiviral therapy,
family physician follow-up, and consultation with an ophthalmologist due
to the possibility of herpes simplex corneal infection, often manifested
by herpetic involvement of the nasociliary branch of the ophthalmic nerve.

A 45-year-old male comes to your office for follow-up of his diabetes


mellitus, which was diagnosed 3 months ago. He wanted to try lifestyle
modification, including diet, exercise, and weight loss, before trying
medications. He has lost 3 kg (7 lb), but still is 14 kg (31 lb)
overweight. His examination is unremarkable, as is his chemistry
profile. His hemoglobin A_1c level is 8.3%.

Which one of the following would be most appropriate at this point?

a) Exenatide (Byetta)
b) Glyburide (Micronase, DiaBeta)
c) Insulin
d) Metformin (Glucophage)
e) Rosiglitazone (Avandia)
The correct answer is D

Explanation
The Diabetes Association recommends lifestyle intervention along with
metformin as initial therapy for type 2 diabetes. If the follow-up
hemoglobin A_1c is more than 7.0%, then insulin (most effective), a
sulfonylurea (least expensive), or a glitazone (no risk of hypoglycemia)
can be added. The hemoglobin A_1c should be checked every 3 months until
it is less than 7.0%, and every 6 months thereafter.

A middle-aged woman is admitted unconscious to the Emergency Department


following a head injury in a car accident. There is bruising over the
upper abdomen. Blood pressure is 80 mm Hg systolic, pulse 120/minute.
What is the most important initial step in the management of this patient?

a) X-ray (three views) of abdomen


b) Immediate laparotomy
c) Draw blood sample and start IV fluids
d) Perform emergency burrholes
e) Diagnostic peritoneal lavage
The correct answer is C

Explanation
Almost all circulatory shock states require large-volume IV fluid
replacement, as does severe intravascular volume depletion (eg, from
diarrhea or heat stroke). Intravascular volume deficiency is acutely
compensated by vasoconstriction, followed over hours by migration of
fluid from the extravascular compartment to the intravascular,
maintaining circulating volume at the expense of total body water.

A 56 year old man, whose father died of prostate cancer, is found to


have on digital rectal exam (DRE) a small nodule on the right lobe of
his prostate. His PSA level is 2.2. The appropriate management is

a) Follow up in 3 months with DRE and PSA


b) Follow up in 6 months with DRE and PSA
c) Follow up in 12 months with DRE and PSA
d) Ultrasound guided biopsy now
The correct answer is D

Explanation
An annual digital rectal examination (DRE) and PSA beginning at age 50
year for the early detection of both rectal and prostate cancer is,
recommended. A PSA level of less than 4 ng/ml is normal. A level between
4 and 10 is the ?grey? zone and above 10 is suspicious for malignancy.

During a DRE, a nodule must be taken seriously since 50% of prostate


nodules are cancerous. Especially in this patient who has positive
family history. The presence or absence of cancer in a prostate nodule
is ordinarily determined by the results of a transrectal needle
aspiration biopsy.
A 67 year old woman develops delirium 3 days after an acute myocardial
infarction. She is markedly agitated. Which one of the following is the
most useful treatment?
a) Diazepam
b) Morphine
c) Nitroglycerine
d) Chlorpromazine
e) Haloperidol
The correct answer is E

Explanation
Delirium is an acute, transient, usually reversible, fluctuating
disturbance in attention, cognition, and consciousness level. Causes
include almost any disorder, intoxication, or drug. Diagnosis is
clinical, with laboratory and usually imaging tests to identify the
cause. Treatment is correction of the cause and supportive measures.

Drugs given to treat delirium typically include low-dose haloperidol,


may lessen agitation or psychotic symptoms; occasionally, much higher
doses are necessary. However, drugs do not correct the underlying
problem and may prolong or exacerbate delirium.

Second-generation (atypical) antipsychotics (eg, risperidone) may be


used instead because they have fewer extrapyramidal adverse effects;
however, long-term use in patients with dementia may increase risk of
stroke and death.
A 58-year-old white male has a negative screening colonoscopy. He has no
symptoms and no family history of colon carcinoma. His next screening
colonoscopy should be scheduled in

a) 1 year
b) 3 years
c) 5 years
d) 10 years
The correct answer is D

Explanation
The evidence supports a 10-year interval for colonoscopy in patients
less than 80 years old. For patients with a family history of colon
cancer a 5-year interval is recommended, or 3 years if benign polyps are
found. Screening in patients over 80 years old is controversial.

Which one of the following signs is consistent with an overdose of morphine?

a) Tremulousness
b) Nystagmus
c) Respiratory depression
d) Mydriasis
e) Hypertension
The correct answer is C

Explanation
Respiratory depression is the chief hazard of all morphine preparations.
Respiratory depression occurs more frequently in elderly or debilitated
patients and in those suffering from conditions accompanied by hypoxia,
hypercapnia, or upper airway obstruction, in whom even moderate
therapeutic doses may significantly decrease pulmonary ventilation.
Acute overdosage with morphine is manifested by respiratory depression,
somnolence progressing to stupor or coma, skeletal muscle flaccidity,
cold and clammy skin and constricted pupils.
A 42-year-old African-Canadian male recently traveled to the Caribbean
for a scuba diving trip. Since his return he has noted brief
intermittent episodes of vertigo not associated with nausea or vomiting.
He is concerned, however, because these episodes occurred after sneezing
or coughing and then a couple of times after straining while lifting
something. He has had no hearing loss, and no vertigo with positional
changes such as bending over or turning over in bed.

The most likely cause of this patient‟s vertigo is

a) vestibular neuronitis
b) Meniere‟s disease
c) benign paroxysmal positional vertigo
d) a perilymphatic fistula
e) multiple sclerosis triggered by a rapid change in climate
The correct answer is D

Explanation
A perilymphatic fistula between the middle and inner ear may be caused
by barotrauma from scuba diving, as well as by direct blows, heavy
weight bearing, and excessive straining (e.g., with sneezing or bowel
movements.) This patient‟s recent trip involved two of these potential
factors. Vestibular neuronitis is a more sudden, unremitting syndrome.
Meniere‟s disease is manifested by episodes of vertigo, associated with
hearing loss and often with nausea and vomiting. Benign paroxysmal
positional vertigo is more likely in older individuals, and is
associated with postural change. Multiple sclerosis requires symptoms in
multiple areas and is not thought to be provoked by climate change.

A 62-year-old white female complains of bone and muscle pain in both


legs for several years. She denies any joint pain or swelling. Her only
medication is occasional acetaminophen for pain. She denies taking
vitamins or supplements, although she eats a serving of yogurt every day.

On examination the patient weighs 61 kg (134 lb) and is 157 cm (62 in)
tall. Her height has not changed in the past 2 years. The remainder of
the examination is unremarkable. Laboratory evaluation reveals that her
CBC, erythrocyte sedimentation rate, and creatinine, protein, albumin,
alkaline phosphatase, and TSH levels are all within normal limits. Her
serum calcium level is 9.1 mg/dL (N 8.5-10.5) and her 25-OH vitamin D
level is 16 ng/mL (N 25?80).

What is the best initial treatment for this patient?

a) Calcium carbonate, 500 mg twice daily, and vitamin D_2 , 400 IU/day
b) Calcium carbonate, 500 mg 3 times daily, and vitamin D_2 , 800 IU/day
c) Calcium gluconate, 600 mg/day, and vitamin D_2 , 800 IU/day
d) Vitamin D_2 , 800 IU/day
e) Vitamin D_2 , 50,000 IU once a week for 8-12 weeks
The correct answer is E

Explanation
This patient has vitamin D deficiency and requires replacement doses.
The amount of vitamin D needed to correct a deficiency is higher than
the dosage recommended for prevention of osteoporosis (800 IU/day) or
the recommended daily allowance (200 IU/day). It is typically given as a
weekly dose of 50,000 IU of vitamin D_2 for 8?12 weeks (evidence level
C). After this, a 25-OH vitamin D_2 level should be repeated. Daily
doses of vitamin D up to 10,000 IU are also considered to be effective
and safe for treatment of vitamin D deficiency.
A 55 year old man presents to the hospital with the complaint of severe
intermittent pain in his right lower back that radiates around his trunk
into his lower quadrant and upper right thigh. Which one of the
following is the most likely diagnosis‟

a) Hepatitis
b) Appendicitis
c) A ureteral stone
d) Pyelonephritis
e) Biliary obstruction
The correct answer is C

Explanation
Stones, especially tiny ones, may not cause any symptoms. Stones in the
bladder may cause pain in the lower abdomen. Stones that obstruct the
ureter or renal pelvis or any of the kidney's drainage tubes may cause
back pain or renal colic. Renal colic is characterized by an
excruciating intermittent pain, usually in the flank (the area between
the ribs and hip), that spreads across the abdomen, often to the genital
area and inner thigh. The pain tends to come in waves, gradually
increasing to a peak intensity, then fading, over about 20 to 60
minutes. The pain may radiate down the abdomen toward the groin or
testicle or vulva.

Other symptoms include nausea and vomiting, restlessness, sweating, and


blood in the urine. A person may have an urge to urinate frequently,
particularly as a stone passes down the ureter. Chills, fever, and
abdominal distention sometimes occur.
You are measuring the blood pressure in an adult patient with suspected
hypertension. Which one of the following is recommended for an accurate
measurement?

a) Use a mercury manometer with the column at atrial level


b) Choose a cuff with bladder width about 40% of arm circumference
c) Increase cuff pressure rapidly until Korotkoff sounds disappear then
release for measurement
d) The rate of drop of mercury should be 2 mm per second
e) Record the pressure at which the radial artery pulse first appears
(systolic pressure)
The correct answer is D

Explanation
When measuring the blood pressure the cuff is inflated until the blood
flow through the brachial artery stops. With sufficient compression, the
cuff cuts off blood flow through the artery, and no sound is heard in
the stethoscope. The pressure in the cuff is increased rapidly to 30
millimeters of mercury above the point that no blood flow is taking
place through the cuff when no sound can be heard in the stethoscope or
when a pulse can no longer be felt in the wrist.

Then valve is loosened in the bulb and to lessen the air pressure.
Pressure is then decreased so that the rate of drop is 2 millimeters per
second. When the pressure falls to the point that blood begins to flow
through the artery again, the number that the column of mercury has
risen to at the first sound heard in the stethoscope is the systolic
blood pressure (SBP), the first number in the blood pressure reading

Look at the column of mercury to see the number at that pressure point.
When the cuff decompresses to the point that blood flows freely in the
artery, the sound is no longer heard in the stethoscope. The number next
to the top of the column of mercury when the sound ceases is the
diastolic blood pressure (DBP), the second number in the blood pressure
reading. Again, look at the column of mercury to see the number at that
pressure point. Record the SBP and the DBP numbers immediately (don't
depend on memory), and note the arm (right or left) used for taking the
measurement is noted.
The best initial test for hyperaldosteronism (conn disease) in a patient
who you suspect to have a secondary cause of hypertension is

a) Serum renin level


b) Basic metabolic panel
c) Serum aldosterone level
d) CT of abdomen
The correct answer is B

Explanation
Hyperaldosteronism can be caused by a tumor (usually a noncancerous
adenoma) in the adrenal gland (a condition called Conn's syndrome),
although sometimes both glands are involved and are overactive.
Sometimes hyperaldosteronism is a response to certain diseases, such as
very high blood pressure (hypertension) or narrowing of one of the
arteries to the kidneys.

A doctor who suspects hyperaldosteronism first tests the levels of


sodium and potassium in the blood by obtaining a basic metabolic panel
(BMP). The doctor may also measure aldosterone levels. If they are high,
spironolactone, a drug that blocks the action of aldosterone, may be
given to see if the levels of sodium and potassium return to normal. In
Conn's syndrome, the levels of renin are also very low.

When too much aldosterone is being produced, doctors examine the adrenal
glands for a noncancerous tumor (adenoma). Computed tomography (CT) or
magnetic resonance imaging (MRI) can be helpful.
A 45 year old woman smoker, on steroids for seropositive rheumatoid
arthritis, presents with cough, hemoptysis, and a perihilar lung nodule
on chest x-ray. The best way to establish the diagnosis is

a) Tomography
b) Computed tomography scan of chest
c) Tuberculin skin test
d) Pulmonary function test
e) Bronchoscopy
The correct answer is C

Explanation
Tuberculosis (TB) infection usually occurs initially in the upper part
(lobe) of the lungs. The body's immune system, however, can stop the
bacteria from continuing to reproduce. Thus, the immune system can make
the lung infection inactive (dormant). On the other hand, if the body's
immune system cannot contain the TB bacteria, the bacteria will
reproduce (become active or reactivate) in the lungs and spread
elsewhere in the body.

It may take many months from the time the infection initially gets into
the lungs until symptoms develop. The usual symptoms that occur with an
active TB infection are a generalized tiredness or weakness, weight
loss, fever, and night sweats. If the infection in the lung worsens,
then further symptoms can include coughing, chest pain, coughing up of
sputum (material from the lungs) and/or blood, and shortness of breath.
If the infection spreads beyond the lungs, the symptoms will depend upon
the organs involved.

TB can be diagnosed in several different ways, including chest x-rays,


analysis of sputum, and skin tests. Sometimes, the chest x-rays can
reveal evidence of active tuberculosis pneumonia. Other times, the
x-rays may show scarring (fibrosis) or hardening (calcification) in the
lungs, suggesting that the TB is contained and inactive.

Several types of skin tests are used to screen for TB infection. These
so-called tuberculin skin tests include the Tine test and the Mantoux
test, also known as the PPD (purified protein derivative) test. In each
of these tests, a small amount of purified extract from dead
tuberculosis bacteria is injected under the skin. If a person is
infected with tuberculosis, a raised and reddened area will occur around
the site of the test injection.
A 35-year-old male with a previous history of kidney stones presents
with symptoms consistent with a recurrence of this problem. The initial
workup reveals elevated serum calcium. Which one of the following tests
would be most appropriate at this point?

a) Serum calcitonin
b) 24-hour urine for calcium and phosphate
c) Serum phosphate and magnesium
d) Serum parathyroid hormone
e) Spot urine for microalbumin
The correct answer is D

Explanation
A patient with a recurrent kidney stone and an elevated serum calcium
level most likely has hyperparathyroidism, and a parathyroid hormone
(PTH) level would be appropriate. Elevated PTH is caused by a single
parathyroid adenoma in approximately 80% of cases. The resultant
hypercalcemia is often discovered in asymptomatic persons having
laboratory work for other reasons. An elevated PTH by immunoassay
confirms the diagnosis. In the past, tests based on renal responses to
elevated PTH were used to make the diagnosis. These included blood
phosphate, chloride, and magnesium, as well as urinary or nephrogenous
cyclic adenosine monophosphate. These tests are not specific for this
problem, however, and are therefore not cost-effective. Serum calcitonin
levels have no practical clinical use.
A 34 year old woman presents with a 3 day history of increasing fever
with chills, sweats, nausea, and 2 episodes of vomiting. On examination,
she has costovertebral angle tenderness and urine reveals gross pyuria.
Initial management should include
a) Abdominal ultrasound
b) Abdominal CT
c) Empirical treatment with antibiotics; image only if she fails to
respond to therapy
d) Treat with antibiotics and perform IVP 4-6 weeks after resolution of
illness
e) Immediate DTPA renal scintigraphy
The correct answer is C

Explanation
Pyelonephritis is a bacterial infection of one or both kidneys.
Infection can spread up the urinary tract to the kidneys, or the kidneys
may become infected through bacteria in the bloodstream. Chills, fever,
back pain, nausea, and vomiting can occur. Symptoms of pyelonephritis
often begin suddenly with chills, fever, pain in the lower part of the
back on either side, nausea, and vomiting. Urine and sometimes blood
tests are done to diagnose pyelonephritis. Antibiotics are given to
treat the infection.
A 35 year old white female comes to your office with a 3-month history
of the gradual onset of pain and tenderness in the wrists and hands. She
also complains of 1 hour of morning stiffness. She denies rash, fever,
or skin changes. On physical examination she has symmetric swelling of
the proximal interphalangeal joints and metacarpophalangeal joints.
Motion of these joints is painful. She has no rash or mouth ulcers.
Radiographs of the hands and wrists are negative, and a chest film is
unremarkable. Her CBC is normal, but the erythrocyte sedimentation rate
is elevated at 40 mm/hr. Latex fixation for rheumatoid factor is
negative, and an antinuclear antibody (ANA) test is negative. The most
likely diagnosis in this patient is

a) Rheumatoid arthritis
b) Systemic lupus erythematosus
c) Sarcoidosis
d) Lyme disease
e) Calcium pyrophosphate desposition disease
The correct answer is A

Explanation
The patient has rheumatoid arthritis (RA) by symptoms and physical
findings. A positive latex fixation test for rheumatoid factor is not
necessary for the diagnosis. A negative rheumatoid factor does not
exclude RA, and a positive rheumatoid factor is not specific. Rheumatoid
factor is found in the serum of approximately 85% of adult patients with
RA; in subjects without RA, the incidence of positive rheumatoid factor
is 1%-5% and increases with age.

The ANA test is positive in at least 95% of patients with systemic lupus
erythematosus (SLE), but in only about 35% of patients with RA.
Elevation of the erythrocyte sedimentation rate is seen in many patients
with RA, and the degree of elevation roughly parallels disease activity.
A mean of 6 months after the onset of Lyme disease, 60% of patients in
the U.S. have brief attacks of asymmetric, oligoarticular arthritis,
primarily in the large joints and especially in the knee.
A 20 year old woman presents with a 3 week history of bloody diarrhea
and weight loss. On examination, she is febrile, dehydrated and has a
distended abdomen with left lower quadrant tenderness. You suspect that
she has ulcerative colitis. All of the following are appropriate initial
investigations, except

a) Plain abdominal x-ray


b) Serum electrolytes
c) Stool cultures
d) Barium enema
e) Sigmoidoscopy
The correct answer is D

Explanation
Ulcerative colitis is a chronic inflammatory and ulcerative disease
arising in the colonic mucosa, characterized most often by bloody
diarrhea. Extraintestinal symptoms, particularly arthritis, may occur.
Long-term risk of colon cancer is high.

Laboratory tests should be done to screen for anemia, hypoalbuminemia,


and electrolyte abnormalities. Plain x-rays of the abdomen may show
mucosal edema, loss of haustration, and absence of formed stool in the
diseased bowel. Barium enema shows similar changes, albeit more clearly,
and may also demonstrate ulcerations, but the enema should not be
performed during an acute presentation because of the risk of inducing
perforation.
A healthy 24 year old male presents with a sore throat of 2 days‟
duration. He reports mild congestion and a dry cough. On examination,
his temperature is 37.2°C (99.0°F). His pharynx is red without exudates,
and there are no anterior cervical nodes. His tympanic membranes are
normal, and his chest is clear. You would do which one of the following?

a) Treat with analgesics and supportive care


b) Treat with azithromycin (Zithromax)
c) Perform a throat culture and begin treatment with penicillin
d) Perform a rapid strep test
The correct answer is A

Explanation
The Centers for Disease Control and Prevention (CDC) assembled a panel
of national health experts to develop evidence-based guidelines for
evaluating and treating adults with acute respiratory disease. In
clinical screening, the most reliable predictors of streptococcal
pharyngitis are the Centor criteria. These include tonsillar exudates,
tender anterior cervical lymphadenopathy, absence of cough, and history
of fever.

Patients with four positive criteria should be treated with antibiotics,


those with three positive criteria should be tested and treated if
positive, and those with 0-1 positive criteria should be treated with
analgesics and supportive care only. This patient has only one of the
Centor criteria, and according to the panel should not be tested or
treated with antibiotics.
A 40 year old male developed Adult Respiratory Distress Syndrome (ARDS)
after a severe attack of pancreatitis. He was admitted to the ICU,
intubated and ventilated with PEEP (Positive end-expiratory pressure) of
14 cm water, and FiO2 of 70%.

Suddenly, his pulse increases from 90 to 150/min; systolic Blood


Pressure drops from 120 to 60 mm Hg; Central Venous Pressure increases
from 20 to 50cm water.

Physical exam shows tracheal deviation to the left side and absent
breath sounds on the right side. JVD is noted.

What is the most appropriate next step in management?

a) Chest tube
b) Chest X ray
c) EKG, CPK MB and Troponin
d) IV heparin
e) Needle thoracostomy
The correct answer is E

Explanation
Tension pneumothorax is not an uncommon complication of mechanical
ventilation when high PEEP is used. This occurs due to barotrauma.

Tracheal deviation + absent breath sounds + hypotension and JVD =


Tension pneumothorax.

The patient is dying. There is no time for diagnostic tests. Needle


thoracentesis is life saving.

A chest tube placement is appropriate after needle decompression of the


pleural space.
A 52 year old male with a history of psychosis presents with muscle
stiffness and resting tremors, associated with difficulty in balance and
initiating movements. What is the best diagnostic method for detection
of the above condition?

a) Serum dopamine levels


b) Computed tomography scan
c) Positron emission tomography scan
d) Magnetic resonance imaging
e) Serum ceruloplasmin concentration
The correct answer is C

Explanation
The major neuropathologic findings in Parkinson disease are a loss of
pigmented dopaminergic neurons in the substantia nigra and the presence
of Lewy bodies.

The characteristic symptoms of moderate Parkinson's disease can be


remembered with the acronym TRAP:
T
Tremor Involuntary trembling of the limbs
R
Rigidity Stiffness of the muscles
A
Akinesia Lack of movement or slowness in initiating and maintaining
movement
P
Postural instability Characteristic bending or flexion of the body, associated with
difficulty in balance and disturbances in gait
Positron emission tomography (PET) and single photon emission CT (SPECT)
are useful diagnostic imaging studies.
The scan uses 18-flurodopa to measure dopamine neurons in the substantia
nigra area of the brain. The symptoms of Parkinson‟s are caused by loss
of the dopamine neuron. 18-flurodopa is injected into the patient and
then the brain is scanned. The scan is very sensitive to the dopamine
cells and many people with Parkinson‟s will have an abnormal PET scan.

Magnetic resonance imaging (MRI) and computed tomography (CT) scan are
unremarkable in Parkinson disease.
No laboratory biomarkers exist for Parkinson disease. Serum
ceruloplasmin concentration is obtained as a screening test for Wilson
disease.
A 20 year old tall thin man presents to the ER with difficulty breathing
and chest pain when he tries to breath. A STAT Chest X-ray shows the
following:

What is the most appropriate immediate intervention?

a) Antibiotics
b) Needle thoracentesis
c) Chest tube
d) CT of chest
The correct answer is B

Explanation
Pneumothorax is air in the pleural space causing partial or complete
lung collapse. Pneumothorax can occur spontaneously or from underlying
pulmonary disease, trauma, or medical procedures.

Symptoms include dyspnea, pleuritic chest pain, and anxiety. Dyspnea may
be sudden or gradual in onset depending on the rate of development and
size of the pneumothorax. Physical findings classically consist of
absent tactile fremitus, hyperresonance to percussion, and decreased
breath sounds on the side with the pneumothorax.

Diagnosis is made with chest x-ray. Radiolucent air and the absence of
lung markings juxtaposed between a shrunken lobe or lung and the
parietal pleura are diagnostic of pneumothorax. Tracheal deviation and
mediastinal shift occur with large pneumothoraces.

Tension pneumothorax is a medical emergency. It should be treated


immediately by inserting a 14- or 16-gauge needle with catheter through
the chest wall in the 2nd intercostal space at the mid-clavicular line.
The sound of high-pressure air escaping confirms diagnosis. Emergency
decompression must be followed immediately by chest tube thoracostomy,
after which the catheter is removed.
Which one of the following is a significant side effect of varenicline
(Chantix)?

a) Facial hirsutism
b) Paroxysmal dysrhythmias
c) Pleurisy-like symptoms
d) Suicidal ideation
e) Insomnia
The correct answer is D

Explanation
Education, support, and medications are all valuable tools in assisting
patients with a smoking habit. Varenicline, a clinically effective
smoking-cessation product, has been associated with patient mood changes
following the initiation of therapy, including suicidal thoughts and
aggressive and erratic behavior. The other problems listed have not been
associated with varenicline use.
A 68 year old man presents with cardiac arrest due to ventricular
rupture. Which one of the following myocardial changes is the most
frequent cause?

a) Abscess formation and tissue destruction due to infective endocarditis


b) Fatty change due to interaction of diphtheria exotoxin and carnitine
c) Aschoff bodies associated with rheumatic fever
d) Inflammation due to coxsackie B infection
e) Necrosis due to coronary artery obstruction
The correct answer is E

Explanation
The most common cause of myocardial infarction is narrowing of the
coronary blood vessels due to atheromatous plaques. Plaque rupture with
subsequent exposure of the basement membrane results in platelet
aggregation, thrombus formation, fibrin accumulation, hemorrhage into
the plaque, and varying degrees of vasospasm.

This can result in partial or complete occlusion of the vessel and


subsequent myocardial ischemia. Total occlusion of the vessel for more
than 4-6 hours results in irreversible myocardial necrosis, but
reperfusion within this period can salvage the myocardium and reduce
morbidity and mortality.
A 30 year old man has a history of recurrent pneumonias and chronic
cough dating from early childhood. The cough, which is worse in the
morning and on lying down, is productive of foul-smelling purulent
sputum which is occasionally bloody-tinged. The patient is chronically
ill and has clubbed fingers. Rales are heard over the posterior lung
bases. Which one of the following is the most likely diagnosis‟

a) Chronic bronchitis
b) Pulmonary aspergillosis
c) Pulmonary neoplasm
d) Chronic obstructive emphysema
e) Bronchiectasis
The correct answer is E

Explanation
Bronchiectasis is dilation and destruction of larger bronchi caused by
chronic infection and inflammation. Common causes are cystic fibrosis,
immune defects, and infections, though some cases appear to be
idiopathic. Symptoms are chronic cough and purulent sputum
expectoration; some patients may also have fever and dyspnea.

Diagnosis is based on history and imaging, usually involving


high-resolution CT, though standard chest x-rays may be diagnostic.
Treatment and prevention of acute exacerbations are with antibiotics,
drainage of secretions, and management of complications, such as
superinfection and hemoptysis. Treatment of underlying causes is
important whenever possible.
Which one of the following should be avoided when treating pain in the
elderly?

a) Fentanyl (Sublimaze)
b) Hydrocodone
c) Meperidine (Demerol)
d) Morphine
e) Oxycodone (OxyContin)
The correct answer is C

Explanation
According to the Beers criteria, a list of drugs that should generally
be avoided in older adults, meperidine should not be used in the elderly
because its metabolite can accumulate and cause seizures. The other
medications are not listed in the Beers criteria and are not
contraindicated in the elderly.
An elderly man complains of lower back pain. X-ray shows decreased joint
space and osteophytes. What is the most likely diagnosis‟

a) Degenerative joint disease (osteoarthritis)


b) Gout
c) Pseudogout
d) Rhuematoid arthritis
The correct answer is A

Explanation
Osteoarthritis is a chronic arthropathy of an entire joint characterized
by disruption and potential loss of joint cartilage along with other
joint changes, including bone hypertrophy (osteophyte formation).
Symptoms include gradually developing pain aggravated or triggered by
activity, stiffness relieved < 30 min after activity, and occasional
joint swelling.

Diagnosis is confirmed by x-rays. Plain x-rays should be obtained of the


most symptomatic joints. X-rays generally reveal marginal osteophytes,
narrowing of the joint space, increased density of the subchondral bone,
subchondral cyst formation, bony remodeling, and joint effusions.
Standing x-rays of knees are more sensitive to joint space narrowing.

Treatment involves physical measures (including rehabilitation), drugs,


and surgery.
A 21 year old woman comes to the emergency department because, she says,
"I'm burning up." She is known to staff as an intravenous drug user. On
physical examination a systolic heart murmur is detected over the
precordium. An expected physical finding will be which of the following?

a) Decreased intensity of S1
b) Increased intensity of the murmur with deep inspiration
c) Increased intensity of the murmur with forced expiration
d) Positive Kussmaul sign (rise in jugular venous pulse with inspiration)
e) Right-sided gallop
The correct answer is B

Explanation
An intravenous drug abuser with high fevers and a cardiac murmur should
be considered to have acute bacterial endocarditis with staphylococcus
until proven otherwise. The valvular lesion most commonly seen in these
patients is tricuspid regurgitation. This is a systolic murmur and since
it is located on the right side of the heart, it will increase in
intensity with inspiration, which increases right heart filling.
When evaluating a patient on chronic phenytoin (Dilantin) therapy for a
seizure disorder, which one of the following is a sign of toxicity?

a) Peripheral neuropathy
b) Ataxia
c) Clonus
d) Ballistic movements
e) Photophobia
The correct answer is B

Explanation
Physicians often see patients with seizure disorders well controlled on
phenytoin. However, due to its many side effects and associated
illnesses, careful monitoring is required. Screening for ataxia, which
is often subtle, must be performed at each visit, even when following
blood levels at regular intervals.
In a patient with HIV infection, the threshold for initiating treatment
for tuberculosis after PPD screening is induration greater than or equal to

a) 2.5 mm
b) 5.0 mm
c) 7.5 mm
d) 10 mm
e) 15 mm
The correct answer is B

Explanation Treatment should be initiated when a PPD causes


induration > 5 mm in a patient with HIV infection.

Which one of the following can be used to treat torsades de pointes‟

a) Atropine
b) Procainamide (Procanbid, Pronestyl)
c) Magnesium
d) Disopyramide (Norpace)
e) Quinidine
The correct answer is C
Explanation
Torsades de pointes, a special form of ventricular tachycardia (VT),
causes a gradual alteration in the amplitude and direction of electrical
activity. Torsades requires different treatment than the other VTs.
Recent reports have demonstrated that magnesium sulfate can effectively
abolish runs of torsades de pointes. Quinidine and other drugs that
prolong repolarization (procainamide, phenothiazines, tricyclic
antidepressants, disopyramide) are contraindicated because they can
exacerbate torsades. Atropine is indicated for treatment of symptomatic
bradycardia.
A 75 year old Asian female comes to your office with a 6 week history of
the gradual onset of localized low back pain. The pain is increased with
walking or standing and relieved by sitting and lying. Pushing a
shopping cart dramatically relieves the pain. Based on the history,
which one of the following is the most likely diagnosis‟

a) Spinal cord tumor


b) Epidural abscess
c) Osteoporotic vertebral fracture
d) Spinal stenosis
e) Sciatica
The correct answer is D

Explanation
Spinal stenosis pain is often referred to as pseudoclaudication because
the pain is worsened by walking. Any movement that flexes the spine,
such as pushing a shopping cart, relieves the pain. Pain from tumor of
infection is persistent. Osteoporotic vertebral fractures have a sudden
onset. Sciatica does not have the typical findings of spinal stenosis.
You see a patient with a serum sodium level of 122 mEq/L (N 135?145) and
a serum osmolality of 255 mOsm/kg H O (N 280?295). Which one of the
following would best correlate with a diagnosis of syndrome of
inappropriate antidiuresis‟

a) Fractional excretion of sodium <1%


b) Elevated urine osmolality
c) Elevated serum glucose
d) Elevated BUN
e) Low plasma arginine vasopressin
The correct answer is B

Explanation
The syndrome of inappropriate antidiuresis (SIAD, formerly SIADH) is
related to a variety of pulmonary and central nervous system disorders
in which hyponatremia and hypo-osmolality are paradoxically associated
with an inappropriately concentrated urine. Most, but not all, cases are
associated with increased levels of the antidiuretic hormone arginine
vasopressin (AVP). For a diagnosis of SIAD to be made, the patient must
be euvolemic and not on diuretics (within 24?48 hours), and the urine
osmolality must be high in conjunction with both low serum sodium and
low osmolality. The BUN is normal or low and the fractional excretion of
sodium is >1%.

Fluid restriction (<800 cc/24 hours) over several days will correct the
hyponatremia/hypo-osmolality, but definitive treatment requires
eliminating the underlying cause, if possible. In the case of severe,
acute hyponatremia with symptoms (e.g., confusion, obtundation,
seizures), hypertonic (3%) saline can be slowly infused intravenously
but can have dangerous neurologic side effects.

Elevated serum glucose levels give rise to a factitious hyponatremia,


but not SIAD.

A 45 year old man develops fever, cough with sputum production, night
sweats, anorexia and weight loss. The expectorated sputum
characteristically is foul smelling and bad tasting. Imaging shows he
has developed an abscess in his lung. He is HIV negative. All of the
following organisms cause lung abscess in this patient, except

a) Pneumocystis carinii
b) Staph aureus
c) Klebsiella pneumonia
d) Pseudomonas
The correct answer is A

Explanation
Lung abscess' can be caused by a bacterial infection that reaches the
lungs in several ways. The most common is aspiration of oropharyngeal
contents.

Patients at the highest risk for developing lung abscess have the
following risk factors: poor dentition, seizure disorder and alcohol abuse.

In addition the following infectious etiologies of pneumonia may


progress to parenchymal necrosis and lung abscess formation: pseudomonas
aeruginosa, klebsiella pneumoniae, staphylococcus aureus, streptococcal
pneumonia and nocardia species.

Pneumocystis jiroveci (formerly Pneumocystis carinii) is an


opportunistic bug that can cause lung abscess in HIV/AIDS patients by
taking advantage of the patients weakened immune system.
Thiazide diuretics are the first choice for which of the following patients

a) Hypertensive alcoholic with malnutrition


b) Hypertensive with diabetes
c) Hypertensive with gout
d) Hypertensive elderly patient
e) Hypertensive with left ventricular hypertrophy
The correct answer is D

Explanation
In most instances, low-dose diuretic therapy should be used as initial
antihypertensive therapy in the elderly. A thiazide diuretic is often
the first drug given to treat high blood pressure. Diuretics also help
the kidneys eliminate salt and water, decreasing fluid volume throughout
the body and thus lowering blood pressure. Diuretics are particularly
useful for blacks, older people, obese people, and people with heart
failure or chronic kidney failure.

Thiazide diuretics are contraindicated in patients with gout. Thiamine,


ACEI and beta-blocker are the treatments of choice for hypertensive
patients that are alcoholics, diabetics and those with LVH, respectively.
A 47-year-old male with chronic kidney disease is being treated with
epoetin alfa (Procrit). His hemoglobin level is 11.3 g/dL (N 13.0?18.0).
Which one of the following would be most appropriate with regard to his
epoetin alfa regimen?

a) Increase the dosage until the hemoglobin level is >12.0 g/dL


b) Increase the frequency of injections, using the same dose
c) Decrease the frequency of injections, using the same dose
d) Continue the current regimen
The correct answer is D

Explanation
In patients with renal failure, the risk for death and serious
cardiovascular events is increased with higher hemoglobin levels ( 13.5
g/dL), and it is therefore recommended that levels be maintained at
10?12 g/dL. Studies have also demonstrated less mortality and morbidity
when the dosage of epoetin alfa is set to achieve a target hemoglobin of
<12 g/dL.
Homosexual males who engage in anal receptive intercourse are at risk
for developing anal carcinoma associated with which one of the following
infections‟

a) Syphilis
b) Human papillomavirus
c) Chlamydia
d) Gonorrhea
e) Calymmatobacterium granulomatis (granuloma inguinale)
The correct answer is B

Explanation
Human papillomavirus types 16 and 18 are associated with anal and
genital warts and the development of anal carcinomas. Anal Papanicolaou
smears are recommended yearly for HIV-positive patients because of their
increased risk for HPV-related carcinoma. The other STDs listed are not
associated with anal carcinoma.

A 26 year old dental hygienist presents with severe pain associated with
a blister which is filled with purulent-appearing material on the palmar
aspect of the distal phalanx of her right index finger. She has a
low-grade fever, and there is marked redness around the blister. You
note tender right axillary adenopathy. She tells you that there were
several smaller blisters in the area of the larger lesion a few days ago.

Which one of the following is the most likely diagnosis‟

a) Herpetic whitlow
b) Paronychia
c) Felon
d) Pyogenic flexor tenosynovitis
e) Brown recluse spider bite
The correct answer is A

Explanation
Herpetic whitlow results from inoculation of type 1 or type 2 herpes
simplex virus into broken skin. Health care workers exposed to oral
secretions may be susceptible. An abrupt onset of redness and
significant localized pain associated with the development of small,
clear vesicles is typical. The vesicles often coalesce over the next
several days, and the fluid may become cloudy. Fever with epitrochlear
and axillary lymphadenopathy also may occur. Herpetic whitlow may mimic
other hand infections and injuries, but the history and examination
usually lead to an accurate diagnosis, which may be confirmed by viral
culture.
A young woman presents a feature of cold intolerance, fatigue, anorexia,
weight gain and dry skin. Her TSH is increased and FreeT4 is low. What
is the most appropriate treatment?

a) Beta-blocker
b) Levothyroxine
c) Methimazole
d) Propylthiouracil
The correct answer is B

Explanation
Hypothyroidism is thyroid hormone deficiency. It is diagnosed by
clinical features such as a typical facies, hoarse slow speech, and dry
skin, and by low levels of thyroid hormones. Management includes
treatment of the underlying cause and administration of thyroxine.
A 73-year-old male with COPD presents to the emergency department with
increasing dyspnea. Examination reveals no sign of jugular venous
distention. A chest examination reveals decreased breath sounds and
scattered rhonchi, and the heart sounds are very distant but not gallop
or murmur is noted. There is +1 edema of the lower extremities. Chest
radiographs reveal cardiomegaly but no pleural effusion. The patient‟s
B-type natriuretic peptide level is 850 pg/mL (N < 100) and his serum
creatinine level is 0.8 mg/dL (N 0.6 - 1.5).

Which one of the following would be the most appropriate initial management:

a) Intravenous heparin
b) Tiotropium (Spiriva)
c) Levalbuterol (Xopenex) via nebulizer
d) Prednisone, 20 mg twice daily for 1 week
e) Furosemide (Lasix), 40 mg intravenously
The correct answer is E

Explanation
B-type natriuretic peptide (BNP) is secreted in the ventricles and is
sensitive to changes in left ventricular function. Concentrations
correlate with end-diastolic pressure, which in turn correlates with
dyspnea and congestive heart failure. BNP levels can be useful when
trying to determine whether dyspnea is due to cardiac, pulmonary, or
deconditioning etiologies. A value of less than 100 pg/mL excludes
congestive heart failure as the cause for dyspnea. If it is greater than
400 pg/mL, the likelihood of congestive heart failure is 95%. Patients
with values of 100 - 400 pg/mL need further investigation.

There are some pulmonary problems that may elevate BNP, such as lung
cancer, cor pulmonale, and pulmonary embolus. However, these patients do
not have the same extent of elevation that those with acute left
ventricular dysfunction will have. If these problems can be ruled out,
then individuals with levels between 100 - 400 pg/mL most likely have
congestive heart failure. Initial therapy should be a loop diuretic. It
should be noted that BNP is partially excreted by the kidneys, so levels
are inversely proportional to creatinine clearance.
Which one of the following is the most common cause of primary
hyperparathyroidism?

a) Lithium therapy
b) Parathyroid cancer
c) Multiple endocrine neoplasia
d) An adenoma of a single parathyroid gland
The correct answer is D

Explanation
An adenoma of a single parathyroid gland is the cause of primary
hyperparathyroidism in 85% of cases. Other fairly frequent causes are
multiple adenomas and hypertrophy of all four glands. Cancer and
multiple endocrine neoplasias are rare. Lithium therapy may cause
hyperparathyroidism, but this is not considered primary hyperparathyroidism.

A 75-year-old male presents to the emergency department with a


several-hour history of back pain in the interscapular region. His
medical history includes a previous myocardial infarction (MI) several
years ago, a history of cigarette smoking until the time of the MI, and
hypertension that is well controlled with hydrochlorothiazide and
lisinopril (Prinivil, Zestril). The patient appears anxious, but all
pulses are intact. His blood pressure is 170/110 mm Hg and his pulse
rate is 110 beats/min. An EKG shows evidence of an old inferior wall MI
but no acute changes. A chest radiograph shows a widened mediastinum and
a normal aortic arch, and CT of the chest shows a dissecting aneurysm of
the descending aorta that is distal to the proximal abdominal aorta but
does not involve the renal arteries. Which one of the following would be
the most appropriate next step in the management of this patient?

a) Immediate surgical intervention


b) Arteriography of the aorta
c) Intravenous nitroprusside (Nitropress)
d) A nitroglycerin drip
e) Intravenous labetalol (Trandate)
The correct answer is E

Explanation
Patients with thoracic aneurysms often present without symptoms. With
dissecting aneurysms, however, the presenting symptom depends on the
location of the aneurysm. Aneurysms can compress or distort nearby
structures, resulting in branch vessel compression or embolization of
peripheral arteries from a thrombus within the aneurysm. Leakage of the
aneurysm will cause pain, and rupture can occur with catastrophic
results, including severe pain, hypotension, shock, and death. Aneurysms
in the ascending aorta may present with acute heart failure brought
about by aortic regurgitation from aortic root dilatation and distortion
of the annulus. Other presenting findings may include hoarseness,
myocardial ischemia, paralysis of a hemidiaphragm, wheezing, coughing,
hemoptysis, dyspnea, dysphagia, or superior vena cava syndrome.
This diagnosis should be suspected in individuals in their sixties and
seventies with the same risk factors as those for coronary artery
disease, particularly smokers. A chest radiograph may show widening of
the mediastinum, enlargement of the aortic knob, or tracheal
displacement. Transesophageal echocardiography can be very useful when
dissection is suspected. CT with intravenous contrast is very accurate
for showing the size, extent of disease, pressure of leakage, and nearby
pathology. Angiography is the preferred method for evaluation and is
best for evaluation of branch vessel pathology. MR angiography provides
noninvasive multiplanar image reconstruction, but does have limited
availability and lower resolution than traditional contrast angiography.

Acute dissection of the ascending aorta is a surgical emergency, but


dissections confined to the descending aorta are managed medically
unless the patient demonstrates progression or continued hemorrhage into
the retroperitoneal space or pleura. Initial management should reduce
the systolic blood pressure to 100-120 mm Hg or to the lowest level
tolerated.

The use of a beta-blocker such as propranolol or labetalol to get the


heart rate below 60 beats/min should be first-line therapy. If the
systolic blood pressure remains over 100 mm Hg, intravenous
nitroprusside should be added. Vasodilation will induce reflex
activation of the sympathetic nervous system, causing increased
ventricular contraction and increased shear stress on the aorta.

For descending dissections, surgery is indicated only for complications


such as occlusion of a major aortic branch, continued extension or
expansion of the dissection, or rupture (which may be manifested by
persistent or recurrent pain).

A mildly hypertensive 54-year-old male with type 2 diabetes asks about


the benefit of adding fish oil capsules to his daily medication regimen
of atorvastatin (Lipitor), metformin (Glucophage), and aspirin.

You advise that fish oil supplements have been shown to

a) increase the risk of rhabdomyolysis


b) lower triglycerides
c) decrease cardiovascular risks in the general population
d) have antiarrhythmic properties
e) improve glycemic control
The correct answer is B

Explanation
The omega-3 polyunsaturated fatty acids found in fish oil have been
shown to lower plasma triglycerides and reduce the risk of stroke after
a myocardial infarction. Unlike niacin and gemfibrozil, they do not
increase the risk of rhabdomyolysis in statin patients. However, they
may worsen glycemic control in diabetics and have a proarrhythmic effect
in coronary artery disease patients. There is no evidence that fish oil
supplements prevent coronary disease in the general population.
A 35 year old woman is brought to the ER in severe distress. She has had
the "flu" for three days. Past history reveals a six month history of
fatigue and malaise. Physical exam reveals BP 120/65 mm Hg supine, 90/58
mm Hg standing, and darkened areas of skin on the knuckles, creases of
the palm, elbows and an abdominal scar. Laboratory values are Na 122
mmol/L, K 5.8 mmol/L, Cl 95 mmol/L, CO2 18 mmol/L. Which one of the
following laboratory tests is most likely to assist in confirming your
diagnosis‟

a) Plasma follicle-stimulating hormone (FSH) and luteinizing hormone (LH)


b) 24 hour urine metanephrines and vanillylmandelic acid (VMA)
c) Plasma catecholamines, total and fractionated
d) Plasma cortisol and adrenocorticotropic hormone (ACTH)
e) Plasma 17OH progesterone
The correct answer is D

Explanation
Addison's disease (adrenal insufficiency) is an insidious, usually
progressive hypofunctioning of the adrenal cortex. It produces various
symptoms, including hypotension and hyperpigmentation, and can lead to
adrenal crisis with cardiovascular collapse.

Diagnosis is clinical and is made by finding elevated plasma ACTH with


low plasma cortisol. Treatment depends on the cause but generally
includes hydrocortisone and sometimes other hormones.
What is the commonest cause of pseudomembranous colitis‟

a) Clostridium botulinum
b) Clostridium difficile
c) Clostridium perfringens
d) Clostridium tetani
The correct answer is B

Explanation
Toxins produced by Clostridium difficile strains in the GI tract cause
pseudomembranous colitis, typically after antibiotic use. Symptoms are
diarrhea, sometimes bloody, rarely progressing to sepsis and acute
abdomen. Diagnosis is by identifying C. difficile toxin in stool.
Treatment is oral metronidazole or vancomycin.
You are asked to see a 48 year old male patient because of persistent
hypertension despite taking metoprolol, enalapril and nifedipine. There
is no history of palpitations or flushing, and the patient does not use
any nonsteroidal anti-inflammatory drugs. Blood pressure in your office
was found to be elevated at 160/110 mmHg. Recent laboratory testing
shows sodium: 146 mmol/L; potassium: 2.4 mmol/L and creatinine: 85
µmol/L. There is no proteinuria on urinalysis. Which one of the
following is the most appropriate test to arrange at this time?

a) Random plasma aldosterone/plasma renin activity ratio


b) 24-hour urine collection for catecholamines and creatinine
c) 24-hour ambulatory blood pressure monitor
d) 24-hour urine collection for cortisol
e) Nuclear medicine captopril renal scan
The correct answer is A

Explanation
Aldosterone, a hormone produced and secreted by the adrenal glands,
signals the kidneys to excrete less sodium and more potassium.
Hyperaldosteronism can be caused by a tumor (usually a noncancerous
adenoma) in the adrenal gland (a condition called Conn's syndrome),
although sometimes both glands are involved and are overactive.
Sometimes hyperaldosteronism is a response to certain diseases, such as
very high blood pressure (hypertension) or narrowing of one of the
arteries to the kidneys.

The amount of salt in the diet and medications, such as over-the-counter


pain relievers of the non-steroid class (such as Motrin and Advil),
diuretics (water pills), beta blockers, steroids, angiotensin-converting
enzyme (ACE) inhibitors, and oral contraceptives can affect the test
results. You should ask your patient to stop taking some of his
medicines for 2 weeks before the test.

Screening test:
Random plasma aldosterone/plasma renin activity (PRA) ratio - Because
this ratio is fairly constant over many physiologic conditions, it can
be used as a screening test. Normal values are less than 270 when
aldosterone concentration is expressed in pmol/L, or are less than 10
when aldosterone concentration is expressed in ng/dL.

Confirmatory test:
The 24-hour urinary aldosterone (U-Aldo) excretion test is one of the
most useful confirmatory diagnostic tools.

Which one of the following agents is associated with the greatest risk
for rebound hypertension if discontinued abruptly?

a) Clonidine (Catapres)
b) Captopril (Capoten)
c) Hydrochlorothiazide
d) Losartan (Cozaar)
e) Diltiazem (Cardizem)
The correct answer is A

Explanation
Clonidine is the classic example of an antihypertensive agent associated
with rebound hypertension on withdrawal. Guanfacine, another a_2
-adrenergic agonist, has also been associated with rebound hypertension
on abrupt withdrawal.

Although blood pressure will often gradually rise to pretreatment levels


when treatment with diuretics, calcium channel blockers, ACE inhibitors,
and angiotensin receptor blockers is stopped, a rebound increase in
blood pressure would not be expected.
A male patient has decided to quit smoking and is gone cigarette free
for a few weeks now. If he undergoes smoking withdrawal, which of the
following is the most likely symptom he will have?

a) Insomnia
b) Tachycardia
c) Anorexia
d) Tremor
The correct answer is A

Explanation
The first few weeks after quitting smoking are usually the most
difficult and it's safe to say that it normally takes at least 8-12
weeks before a person starts to feel comfortable with their new
lifestyle change of being an ex-smoker. The most common symptoms of
nicotine withdrawal are impaired concentration, irritability, tension,
disturbed sleep or drowsiness, intense longing for a cigarette/nicotine,
headaches, and an increased appetite leading to weight gain.
A 49-year-old female who takes multiple medications has a chemistry
profile as part of her routine monitoring. She is found to have an
elevated calcium level. All other values on the profile are normal, and
the patient is not currently symptomatic. Follow-up testing reveals a
serum calcium level of 11.2 mg/dL (N 8.4?10.2) and an intact parathyroid
hormone level of 80 pg/mL (N 10?65). Which one of the following should
be discontinued for 3 months before repeat laboratory evaluation and
treatment?

a) Lithium
b) Furosemide (Lasix)
c) Raloxifene (Evista)
d) Calcium carbonate
e) Vitamin D
The correct answer is A

Explanation
Lithium therapy can elevate calcium levels by elevating parathyroid
hormone secretion from the thyroid gland. This duplicates the laboratory
findings seen with mild primary hyperparathyroidism. If possible,
lithium should be discontinued for 3 months before reevaluation (SOR C).
This is most important for avoiding unnecessary parathyroid surgery.

Vitamin D and calcium supplementation could contribute to hypercalcemia


in rare instances, but they would not cause elevation of parathyroid
hormone. Raloxifene has actually been shown to mildly reduce elevated
calcium levels, and furosemide is used with saline infusions to lower
significantly elevated calcium levels.

Cardiogenic shock is a major, and frequently fatal, complication of a


variety of acute and chronic disorders that impair the ability of the
heart to maintain adequate tissue perfusion. These patients demonstrate
clinical signs of low cardiac output, with adequate intravascular
volume. Which of the following is usually found on physical examination?

a) Normal capillary refill.


b) Normal peripheral pulses
c) Bradycardia
d) Narrow pulse pressure
e) High urine output.
The correct answer is D

Explanation
Answer: D
Cardiogenic shock characterized by primary myocardial dysfunction causes
the heart to be unable to maintain adequate cardiac output. These
patients demonstrate clinical signs of low cardiac output, with adequate
intravascular volume. The patients have cool and clammy extremities,
poor capillary refill, tachycardia, narrow pulse pressure, and low urine
output. Peripheral pulses are rapid and faint and may be irregular if
arrhythmias are present.
Based on the etiology and pathophysiology, cardiogenic shock can be
divided into systolic dysfunction, diastolic dysfunction, valvular
dysfunction, cardiac arrhythmias, coronary artery disease, and
mechanical complications.
In Cushing syndrome you find all, except

a) Low BP
b) Obesity
c) Striation of skin
d) Trunk obesity
The correct answer is A

Explanation
Cushings syndrome is one of the secondary causes of hypertension.
Therefore the blood pressure would be high. Along with Cushings, the
other causes include Hyperaldosteronism, Aortic coarctation,
Pheochromocytoma and Stenosis of renal artery. Remember the mnemonic
C.H.A.P.S. for these 5 causes of secondary hypertension.
In which of the following diseases would you see a ?gloves and stocking?
neuropathy?

a) Diabetes mellitus
b) Lupus
c) Multiple sclerosis
d) ALS
The correct answer is A

Explanation
In diabetes mellitus leg examination may reveal ulcers and peripheral
vascular disease, diabetic foot disease and trophic skin changes and
skin infections. Patients may also have the classic 'glove and stocking'
neuropathy which refers to reduced sensation in the distal portions of
your limbs (i.e. where gloves and stockings are usually positioned).

The neuropathy starts distally and progresses further up the limb as the
condition worsens. The sensations of vibration (tested with a tuning
fork placed on the bone) and proprioception (recognition of joint space
position, tested by wiggling your toe or finger with your eyes closed)
tend to be affected first. In advanced cases patients may have severe
pain and impaired motor function.
A 70 year old man presented with ptosis, myosis and anhydrosis on the
left side. Which one of the following is the most likely cause of this
condition?

a) Tumour induced exophthalmos


b) Fourth cranial nerve palsy
c) Apical pulmonary carcinoma
d) Enlarged thyroid gland
e) Thoracocervical venous dilatation
The correct answer is C

Explanation
Horner's syndrome results when the cervical sympathetic pathway running
from the hypothalamus to the eye is disrupted. The syndrome may be
central, preganglionic, or postganglionic in origin; it may be primary
or secondary to another disorder.
An apical bronchogenic carcinoma (eg Pancoast tumor) is the most common
cause of Horner syndrome.

Central lesions include brain stem ischemia, syringomyelia, and brain


tumor; peripheral lesions include Pancoast tumor, thyrocervical venous
dilatation, cervical adenopathy, neck and skull injuries, aortic or
carotid dissection, and thoracic aortic aneurysm. A congenital form exists.

Symptoms include ptosis, miosis, anhidrosis, and hyperemia of the


affected side. In the congenital form, the iris does not become
pigmented and remains blue-gray. Liquid cocaine 10% can be applied to
the affected eye; poor pupillary dilation after 30 min indicates
Horner's syndrome. If results are positive, 1% hydroxyamphetamine
solution or 5% n-methyl hydroxyamphetamine can be applied to the eye 48
h later to determine whether the lesion is preganglionic (if the pupil
dilates) or postganglionic (if the pupil does not dilate).
A 43 year old woman has an elevated blood pressure that was first
detected during a routine screening at health fair in a local shopping
mall. When she came to see you, a week after that, her blood pressure
was 145/95 mm Hg. Today, 1 year later, she is asymptomatic and her blood
pressure is 140/85 mm Hg. She is continuing to take the prescribed
antihypertensive medication. Which of the following is the most
appropriate management at this time?

a) Continue the antihypertensive therapy


b) Discontinue the antihypertensive therapy
c) Order a chest x-ray film and electrocardiography
d) Order complete blood count, serum electrolyte concentrations and
liver chemistry profile
e) Reduce the dosage of the antihypertensive medication by 50%
The correct answer is A

Explanation
She has fortunately remained asymptomatic with a blood pressure that has
declined to the normal range. She most likely suffers from essential
hypertension and, since this is a lifelong disorder, she will most
likely require long-term antihypertensive therapy and therefore
discontinuing it and reducing the dose are inappropriate at this time.

A 60-year-old male presents with an acute onset of pain and swelling in


the right big toe. He can recall no mechanism of injury. He has
hypertension which is well controlled with hydrochlorothiazide. On
examination the area around the toe is slightly warm. Which one of the
following should be AVOIDED in this patient at this time?

a) Allopurinol (Zyloprim)
b) Colchicine
c) NSAIDs
d) Prednisone
e) Aspiration of the joint
The correct answer is A

Explanation
This patient likely has gout. Aspiration should be attempted to get a
specific diagnosis. The initial treatment for gout is NSAIDs,
colchicine, or cortisone injections (SOR B). Allopurinol should be
avoided until the episode of gout is controlled, because it may
precipitate worsening. In addition to medication, recommended management
includes addressing risk factors such as obesity, diuretic use,
high-purine diet, and alcohol intake (SOR B).

A 25 year old right-handed woman has recurrent spells during which she
seems to be "out of contact" for about 30 seconds. These are associated
with a feeling of fear and recollection of memories from the past. For a
few minutes following an attack, she has trouble finding words to
express herself. The most likely site of origin for these attacks is

a) The right temporal lobe


b) The left parietal lobe
c) The left frontal lobe
d) The left temporal lobe
e) Both frontal lobes
The correct answer is C

Explanation
Aphasia is a language disorder that results from damage to portions of
the brain that are responsible for language. For most people, these are
parts of the left side (hemisphere) of the brain. Aphasia usually occurs
suddenly, often as the result of a stroke or head injury, but it may
also develop slowly, as in the case of a brain tumor.

In expressive (motor, or Broca's) aphasia, comprehension and ability to


conceptualize are relatively preserved, but the ability to create words
is impaired.

Aphasia is caused by damage to one or more of the language areas of the


brain. Many times, the cause of the brain injury is a stroke. A stroke
occurs when, for some reason, blood is unable to reach a part of the
brain. Brain cells die when they do not receive their normal supply of
blood, which carries oxygen and important nutrients. Other causes of
brain injury are severe blows to the head, brain tumors, brain
infections, and other conditions of the brain.

Individuals with Broca's aphasia have damage to the frontal lobe of the
brain. These individuals frequently speak in short, meaningful phrases
that are produced with great effort. Broca's aphasia is thus
characterized as a nonfluent aphasia.
A 46 year old man comes to the emergency department because of
excruciating left flank pain that radiates to his left testicle. On
physical examination he is extremely restless and is in obvious pain.
Genitalia are normal. Abdominal examination is normal except for
intermittent guarding with spasms of pain. Plain x-ray film of the
abdomen is normal. Urinalysis and urinary sediment are: pH 6.5, Specific
gravity 1.025, Glucose negative, Protein negative. The most appropriate
diagnostic study is

a) CT scan of the kidney


b) Culture of the urine
c) Determination of serum uric acid concentration
d) CT scan of the abdomen
e) Measurement of 24 hour urinary calcium excretion
The correct answer is D

Explanation
The patient‟s clinical description is consistent with acute renal colic
from nephrolithiasis. In addition, the picture clearly shows crystals.
The stone is more likely to be found in the ureter. A CT scan of the
abdomen is the most appropriate of the tests listed to find a ureteral
stone.

CT scanning of the kidney is not going to be the most helpful because


the calculi are within the urinary tract, not necessarily within the
kidneys. There is no description of dysuria, frequency, or burning and
there is no fever described. This would go against the need for culture.
Although evaluation for causes of crystalluria and kidney stones may be
indicated in patients with nephrolithiasis, this is premature since we
have not even found the stone yet to confirm the diagnosis.
A 55 year old male needs treatment for venous thromboembolic disease.
What is the loading dose of unfractioned heparin to be administered?

a) 4000 unit bolus given through IV


b) 5000 unit bolus given through IV
c) 3000 unit bolus given through IV
d) 8000 unit bolus given through IV
e) 6000 unit bolus given through IV
The correct answer is D

Explanation
Answer: D
Initial DVT treatment with unfractionated heparin (UFH):
-requires bolus (7500-10,000 IU) followed by continuous IV infusion
(1000-1500 IU/h)
-weight-based heparin nomograms help to achieve proper dosing
-advantages: rapidly reversible by protamine in case of bleeding
-disadvantages: must monitor aPTT with adjustment of dose to reach
therapeutic level (~2x control value); monitor platelet counts for
development of thrombocytopenia.
An HIV patient presents with fever and weight loss. You suspect an
infection with an opportunistic organism. The patient may have all of
the following, except

a) Mycobacterium avium complex


b) Candida
c) Cytomegalovirus (CMV)
d) Pneumocystis carinii
e) Mycoplasma pneumonia
The correct answer is E

Explanation
There are a number of opportunistic organisms (bacterial, fungal,
protozoal and viral) that can cause infection in an immunocompromised
patient such as those with HIV. Mycoplasma pneumoniae is not one of
those opportunistic organisms.

When used to lower total serum cholesterol, nicotinic acid tends to

a) Increase serum triglycerides


b) Increase serum glucose
c) Increase LDL cholesterol
d) Increase the LDL/HDL cholesterol ratio
e) Decrease HDL cholesterol
The correct answer is B

Explanation
Nicotinic acid lowers total serum cholesterol, LDL cholesterol, serum
triglycerides, and the LDL/HDL ratio. It increase HDL cholesterol. It
worsens glucose control, and is therefore not a good choice for use in
diabetics.
A 78 year old man presents with a 2 month history of progressive
hoarseness. He has been a smoker for over 20 years. He denies any
history of dyspnea, dysphagia or hemoptysis. He denies any fever, night
sweats or weight loss. On physical exam he has no lymphadenopathy. What
is the first thing to be done?

a) Chest X-ray
b) Laryngoscopy
c) Throat culture
d) CT
e) Reassurance
The correct answer is B

Explanation Laryngoscopy should be done on this patient first,


usually by an ENT specialist. See picture:

In this patient laryngoscopy confirmed that a large, firm, smooth


swelling had involved the supraglottic larynx (voice-box) on the right.
A biopsy in this patient actually revealed a lymphoma. A computed
tomography (CT) of the neck usually follows to further evaluate the
extent of the tumor. Despite its relative rarity, the consequences of a
missed diagnosis warrant vigilance for this type of laryngeal tumor.

Although he is a smoker, he denies any hemoptysis or weight loss making


lung cancer a little lower on your differential.
Which one of the following clinical signs or symptoms is most sensitive
for bacterial rhinosinusitis, and is associated with the highest
probability of sinusitis‟
a) Purulent secretions
b) Maxillary pain
c) Maxillary tenderness
d) Maxillary toothache
e) Worsening symptoms after initial improvement
The correct answer is E

Explanation
The sensitivity and probability of sinusitis for each of the listed
clinical findings are as follows:

Purulent secretions: sensitivity = 32%-62%, probability unknown


Maxillary pain: sensitivity = 51%, probability 19%
Maxillary tenderness: sensitivity = 49%, probability 21%
Maxillary toothache: sensitivity = 18%-36%, probability unknown
Worsening symptoms after initial improvement: sensitivity = 72%,
probability = 27%
A 62-year-old white male complains of fatigue and proximal extremity
discomfort without any localized joint pain. Which one of the following
conditions is associated with a consistently normal creatine kinase
enzyme level at all phases of disease?

a) Polymyalgia rheumatica
b) Polymyositis
c) Dermatomyositis
d) Drug-induced myopathy
e) Hypothyroid endocrinopathy
The correct answer is A

Explanation
Polymyalgia rheumatica is a disease of the middle-aged and elderly.
Discomfort is common in the neck, shoulders, and hip girdle areas. There
is an absence of objective joint swelling, and findings tend to be
symmetric. Characteristically, the erythrocyte sedimentation rate and
C-reactive protein levels are significantly elevated; however, these
tests are nonspecific. Occasionally there are mild elevations of liver
enzymes, but muscle enzymes, including creatine kinase, are not elevated
in this disorder. Elevation of muscle enzymes strongly suggests another
diagnosis.

Polymyositis and dermatomyositis are associated with variable levels of


muscle enzyme elevations during the active phases of the disease.

Drug-induced myopathies such as those seen with the statin family of


cholesterol-lowering medications tend to produce some elevation of
muscle enzymes during the course of the disorder.

Hypothyroidism is associated with creatine kinase elevation. It should


be strongly considered in the patient with unexplained, otherwise
asymptomatic creatine kinase elevation found on a routine chemistry
profile. Hyperthyroidism may cause muscle disease and loss of muscle,
but it is not associated with creatine kinase elevation.
A 60-year-old white male comes to your office for evaluation of a
chronic cough productive of large amounts of sputum, accompanied by
dyspnea on exertion. He has smoked 2 packs of cigarettes a day for the
past 40 years.

The best diagnostic test for evaluating this problem is


a) arterial blood gases
b) alpha_1 -antitrypsin
c) brain natriuretic peptide (BNP)
d) CT of the chest
e) spirometry
The correct answer is E

Explanation
In patients with suspected COPD, the best diagnostic test is office
spirometry. If the FEV_1 /FVC ratio is <70% and the FEV_1 is <80% of
predicted, the patient has COPD. This generally occurs in mid- to late
life. While cigarette smoking is the largest single risk factor, only
20% of smokers develop clinically 1 significant COPD. The second most
common risk factor is alpha_1 -antitrypsin deficiency, which causes 1%
of cases. These patients present with cough, sputum production, and
dyspnea on exertion. They often experience orthopnea soon after
reclining, whereas patients with heart failure typically experience
orthopnea several hours after reclining, when fluid mobilizes from the
lower extremities.
Having all patients over age 50 take low-dose (81 mg) aspirin daily
would result in

a) a decrease in cardiovascular mortality in men and women


b) a decrease in hemorrhagic stroke in women
c) a decrease in myocardial infarction in men
d) a decrease in strokes in men
e) no increase in major bleeding episodes
The correct answer is C

Explanation
A meta-analysis of six well controlled clinical trials of aspirin
prophylaxis showed a 32% decrease in myocardial infarctions in men
taking aspirin. There was no decrease in cardiovascular mortality or
all-cause mortality in either sex, and there was a trend toward
increased risk of stroke, primarily hemorrhagic stroke. There was a 24%
decrease in ischemic stroke in women, however. The risk of major
bleeding disorders was around 76% higher in aspirin users. The analysis
suggests that aspirin may do more harm than good in healthy persons
without cardiovascular risk factors.
Which one of the following most increases insulin sensitivity in an
overweight patient with diabetes mellitus‟

a) Metformin (Glucophage)
b) Acarbose (Precose)
c) Glyburide (DiaBeta, Micronase)
d) NPH insulin
The correct answer is A

Explanation
Metformin increases insulin sensitivity much more than sulfonylureas or
insulin. This means lower insulin levels achieve the same level of
glycemic control, and may be one reason that weight changes are less
likely to be seen in diabetic patients on metformin. Acarbose is an
-glucosidase inhibitor that delays glucose absorption.
What is the most characteristic lab finding in a patient with hemolytic
anemia?
a) Reticulocytosis
b) Increased MCHC
c) MCV < 80
d) Abnormal Hb electrophoresis
e) MCV > 80
The correct answer is A

Explanation
Hemolysis is the premature destruction of erythrocytes, and it leads to
hemolytic anemia when bone marrow activity cannot compensate for the
erythrocyte loss. Clinical presentation depends on whether the onset of
hemolysis is gradual or abrupt and on the severity of erythrocyte
destruction. A patient with mild hemolysis may be asymptomatic. In more
serious cases, the anemia can be life threatening, and patients can
present with angina and cardiopulmonary decompensation.

An increased reticulocyte count is a criterion for hemolysis but is not


specific for hemolysis.
A strict vegan, develops symptoms of fatigue, dizziness and dyspnea. She
comes in for evaluation and her CBC shows anemia with a MCV of 104. What
is the most appropriate next step in diagnosis‟

a) Iron level
b) Hemoglobin electrophoresis
c) Vitamin B12 level
d) RBC scan
The correct answer is C

Explanation
In vitamin B12 deficiency anemia develops, causing paleness, weakness,
fatigue, and, if severe, shortness of breath and dizziness. A severe
deficiency may cause tingling or loss of sensation in the hands and
feet, muscle weakness, loss of reflexes, difficulty walking, confusion,
and dementia.

The diagnosis is based on blood tests. A CBC will show macrocytic


anemia. When high doses of vitamin B12 supplements are taken, most
symptoms resolve. Symptoms due to nerve damage, such as neuropathy or
dementia in older people, may persist.

Vitamin B12 deficiency develops in people who do not consume any animal
products (vegans) unless they take supplements. If a vegan mother
breastfeeds her infant, the infant is at risk of vitamin B12 deficiency.

An iron level and Hb electrophoresis would be useful of the anemia was


microcytic (MCV < 80). An RBC scan would be useful if bleeding is suspected.
A 54 year old man presents with acute onset of painful great right toe.
It is swollen, red and tender. This is his first episode. You decide to
order a serum uric acid and a 24 urine for uric acid. You then prescribe
for him which of the following medicines

a) Colchicine
b) Indomethacin
c) Allopurinol
d) Probenacid
The correct answer is B
Explanation
Gout is a disease that can occur when purine metabolism gives rise to
uric acid. Uric acid is normally excreted in the urine. But in certain
people this is not done. These patients are called undersecretors. While
other patients can excrete the uric acid normally but their body
produces larger than normal amounts. These patients are called
overproducers.

The treatment of acute gout involves NSAIDS such as indomethacin.


Colchicine is also prescribed but is not as well tolerated due to side
effects.

The treatment of chronic gout involves allopurinol for overproducers and


probenacid for undersecretors. The tests ordered (serum uric acid and 24
urine for uric acid) should help classifying which category this patient
is in.

A 75-year-old black female is diagnosed with macular degeneration. She


is being treated for type 2 diabetes mellitus, hypothyroidism,
hypertension, hypercholesterolemia, and gout.

Which one of her conditions has the strongest association with macular
degeneration?

a) Type 2 diabetes mellitus


b) Hypothyroidism
c) Hypertension
d) Gout
e) Alcohol
The correct answer is C

Explanation
Age-related macular degeneration is the most common cause of blindness
in the older population. It occurs more frequently in light-skinned
individuals. Risk factors include smoking and hypertension.
A 32 year old woman complains of polyarticular symmetric arthritis and a
diffuse maculopapular rash that flares up with exposure to sun light.
She is afebrile. Physical examination reveals tenosynovitis of her
wrists. Which one of the following is the most likely diagnosis‟

a) Gonococcal arthritis
b) Reiter's syndrome
c) Rheumatoid arthritis
d) Systemic lupus erythematosus
e) Meningococcemia
The correct answer is D

Explanation
Systemic lupus erythematosus is a chronic, multisystem, inflammatory
disorder of probable autoimmune etiology, occurring predominantly in
young women. Common manifestations include arthralgias and arthritis
(polyarticular, symmetric); malar and other skin rashes
(photosensitivity rash); pleuritis or pericarditis; renal or CNS
involvement; and hematologic cytopenia. Diagnosis requires clinical and
serologic criteria. Treatment of severe ongoing active disease requires
corticosteroids, often hydroxychloroquine, and sometimes immunosuppressants.
A 45-year-old white male consults you because of a painless, circular,
1-cm white spot inside his mouth, which he noticed 3 days ago. You are
treating him with propranolol (Inderal) for hypertension, and you know
him to be a heavy alcohol user. After a careful physical examination,
your tentative diagnosis is leukoplakia of the buccal mucosa. You elect
to observe the lesion for 2 weeks. On the patient‟s return, the lesion
is still present and unchanged in appearance. The best course of
management at this time is to

a) reassure the patient and continue to observe


b) discontinue propranolol
c) treat with oral nystatin
d) order a fluorescent antinuclear antibody test
e) perform a biopsy of the lesion
The correct answer is E

Explanation
Leukoplakia is a white keratotic lesion seen on mucous membranes.
Irritation from various mechanical and chemical stimuli, including
alcohol, favors development of the lesion. Leukoplakia can occur in any
area of the mouth and usually exhibits benign hyperkeratosis on biopsy.
On long-term follow-up, 2%?6% of these lesions will have undergone
malignant transformation into squamous cell carcinoma. Oral nystatin
would not be appropriate treatment, as this lesion is not typical of
oral candidiasis. Candidal lesions are usually multiple and spread
quickly when left untreated. A fluorescent antinuclear antibody test is
also not indicated, as the oral lesions of lupus erythematosus are
typically irregular, erosive, and necrotic. An idiosyncratic reaction to
propranolol is unlikely in this patient.
A 55-year-old white male sees you for a routine annual visit. His
fasting blood glucose level is 187 mg/dL. Repeat testing 1 week later
reveals a fasting glucose level of 155 mg/dL and an HbA1c of 9.4%. His
BMI is 30 kg/m . He does not seem to have any symptoms of diabetes mellitus.

In addition to lifestyle changes, which one of the following would you


prescribe initially?

a) Metformin (Glucophage)
b) Glyburide (DiaBeta, Micronase)
c) Rosiglitazone (Avandia)
d) Bedtime long-acting insulin (Lantus)
e) Bedtime long-acting insulin and rapid-acting insulin (Novolog) with
each meal
The correct answer is A

Explanation
Metformin is widely accepted as the first-line drug for type 2 diabetes
mellitus. It is relatively effective, safe, and inexpensive, and has
been used widely for many years. Unlike other oral hypoglycemics and
insulin, it does not cause weight gain. It should be started at the same
time as lifestyle modifications, rather than waiting to see if a diet
and exercise regimen alone will work.

If metformin is not effective, a sulfonylurea, a thiazolidinedione, or


insulin can be added, with the choice based on the severity of the
hyperglycemia.
You are evaluating a 45-year-old male construction worker with regard to
his skin and sun exposure history. Which one of the following lesions
should be considered premalignant?

a) Sebaceous hyperplasia
b) Actinic keratosis
c) Seborrheic keratosis
d) A de Morgan spot
e) A halo nevus
The correct answer is B

Explanation
Physicians should advise patients of the dangers of sun exposure,
especially those with a fair complexion who work outdoors. Although
malignant melanoma is the most serious condition of those listed,
actinic keratosis may lead to squamous cell carcinoma with significant
morbidity.
Which of the following is the most important step in the management of
peripheral arterial disease?

a) Diabetic control
b) Blood pressure control
c) Exercise
d) Stop smoking
The correct answer is D

Explanation
Peripheral arterial disease (PAD) is a common circulatory problem in
which narrowed arteries reduce the blood flow to the limbs.

Of perhaps greater concern is that peripheral arterial disease is likely


to be a sign of widespread accumulation of fatty deposits in your
arteries (atherosclerosis). This condition may be reducing blood flow to
the heart and brain as well.

Strictly speaking, peripheral arterial disease refers to a problem with


any of the arteries outside, or peripheral to, the heart, but the term
is commonly used to describe circulatory problems in the limbs or pelvis.

Most people with PAD can be treated with lifestyle changes, medications
or both. Lifestyle changes to lower your risk include: stop smoking
(smokers have a particularly strong risk of PAD), control diabetes,
control blood pressure, be physically active (including a supervised
exercise program), eat a low-saturated-fat, low-cholesterol diet.
A 43 year old man who is being treated with hydrochlorothiazide (HCTZ)
for control of mild edema presents complaining of malaise, fatigue,
muscular weakness, and muscle cramps. Which one of the following is the
most likely cause of the patient's symptoms‟

a) Hyperglycemia
b) Hyperuricemia
c) Drug hypersensitivity
d) Hyperkalemia
e) Hypokalemia
The correct answer is E

Explanation
HyperGLUC is a mnemonic used to remember the side effects of HCTZ. GLUC
stands for hyperglycemia, hyperlipidemia, hyperuricemia, and hypercalcemia.

Thiazide diuretics increase the excretion of water by inhibiting the


reabsorption of sodium and chloride ions at the distal renal tubule. The
natriuretic effects are accompanied by a secondary loss of potassium and
bicarbonate which can cause a mild hypokalemic, hypochloremic, metabolic
alkalosis.
A 35 year old male presents with cough, breathlessness, wheeze and
tightness in his chest. He states that all this has started after 15mins
of his afternoon jogging. What is the most likely diagnosis of his
condition?

a) Emphysema
b) Allergies
c) Asthma
d) Bronchitis
e) Unstable Angina
The correct answer is C

Explanation
Answer: C ? Asthma is characterized by cough, breathlessness, wheezing
and tightness of chest. Since the patient suffered from the symptoms
after exercise, it is known as exercise induced asthma. Asthma is a
chronic inflammatory disorder of the airways due to inflammatory
responses or airway obstruction or hyperresponsiveness or air way
remodeling. Factors which precipitate the asthma are allergens, chemical
irritants, flour, wood, textile chemicals, viral infections, exercise,
emotions and few drugs. Allergens like pollen are the most common
allergen responsible for the cause of asthma. Chemical irritants like
industrial pollutants are also responsible for precipitating the asthma.
Depression, anxiety and excess laughing also trigger asthma.

A. Emphysema is one of the chronic obstructive pulmonary diseases marked


by lysis of the alveoli resulting in formation of sac like structures.
B. Allergies present differently (teary eyes, runny nose...usually not
with sudden onset)
D. Bronchitis is unlikely due to presentation. (not exercise induced,
usually accompanied by fever, sore throat etc)
E. Unstable Angina usually occurs at rest or with minimal exercise and
does not present with cough or wheezing.
A 75 year old otherwise healthy white female states that she has passed
out three time in the last month while walking briskly during her daily
walk with the local senior citizens mall walkers‟ club. This history
would suggest which one of the following as the etiology of her syncope?

a) Vasovagal syncope
b) Transient ischemic attack
c) Orthostatis hypotension
d) Atrial myxoma
e) Aortic stenosis
The correct answer is E

Explanation
Syncope with exercise is a manifestation of organic heart disease in
which cardiac output is fixed and does not rise (or even fall) with
exertion. Syncope, commonly on exertion, is reported in up to 42% of
patients with severe aortic stenosis. Vasovagal syncope is associated
with unpleasant stimuli or physiologic conditions, including sights,
sounds, smells, sudden pain, sustained upright posture, heart, hunger,
and acute blood loss. Transient ischemic attacks are not related to
exertion. Orthostatic hypotension is associated with changing from a
sitting or lying position to an upright position. Atrial myxoma is
associated with syncope related to changes in position, such as bending,
changing from sitting to lying, or turning over in bed.
A 40-year-old white female complains of discomfort in her anterior neck.
She also gives a history of malaise, low-grade fever, and a tender
thyroid gland.

Laboratory Findings

WBCs---------------------------------12,100/mm3 (N 4300-10.800)

Granulocytes---------------------------------30% (N 42-75)

Monocytes---------------------------------4% (N 2-9)

Lymphocytes---------------------------------66% (N 20-51)
Free thyroxine (FT4---------------------------------2.8 ng/dL (N 0.8-2.4)
Erythrocyte sedimendation rate (Westergren---------------------------------60 mm/hr (N 0-20)

Which one of the following would most likely be seen on a thyroid


nuclear medicine study?

a) Slightly increased uptake


b) Normal uptake
c) Markedly decreased radioactive iodine uptake
d) A single hot node
e) Multiple cold areas
The correct answer is C

Explanation
This patient has a clinical presentation typical of subacute
thyroiditis. An elevated erythrocyte sedimentation rate is an almost
certain feature, and slight leukocytosis may also be seen, as well as a
modest degree of thyrototoxicosis and a slightly elevated serum T4
level. A small thyroid gland would be expected on a thyroid scan.
Because the disease interferes with iodine metabolism, radioiodine
uptake is decreased. Hot, toxic nodules or a multinodular goiter would
not be expected. Generalized thyroid enlargement and increased
radioactive iodine uptake would be expected with Graves‟ disease.
Of the following, which is the most frequent cause of seizures in the
elderly?

a) Alcohol withdrawal
b) Stroke
c) Head trauma
d) Hypoglycemia
e) Dementia
The correct answer is B
Explanation
The conditions listed are all causes of seizures. Of course, there are
many other causes of seizures in the elderly, including primary and
metastatic neoplasias (e.g., electrolyte disorders). However, in the
geriatric population, cerebrovascular disease is the most common cause
of seizures, with about 10% of stroke victims developing epileptic
seizures. Seizures are more common following hemorrhagic strokes
compared to nonhemorrhagic strokes.
Nitroglycerin administered sublingually may contribute to the relief of
myocardial ischemic pain by each of the following mechanisms, except

a) Coronary vasodilation
b) Decreased venous pooling resulting in increased cardiac preload
c) Reduced systemic vascular resistance
d) Reduced ventricular volume
The correct answer is B

Explanation
Nitroglycerin, is used to reduce cardiac workload in selected patients.
Nitroglycerin dilates veins, arteries, and arterioles, reducing LV
preload and afterload. As a result, myocardial O2 demand is reduced,
lessening ischemia. IV nitroglycerin is recommended during the first 24
to 48 h for patients with heart failure, large anterior MI, persistent
chest discomfort, or hypertension. BP can be reduced by 10 to 20 mm Hg
but not to < 80 to 90 mm Hg systolic. Longer use may benefit patients
with recurrent chest pain or persistent pulmonary congestion. In
high-risk patients, nitroglycerin given in the first few hours reduces
infarct size and short-term and possibly long-term mortality risk.
Nitroglycerin is not routinely given to low-risk patients with
uncomplicated MI.
A 17-year-old white male presents with facial swelling and difficulty
chewing food. He tells you the symptoms resulted from a fight. On
examination you note trismus and left periorbital swelling.

Which one of the following fractures is most likely?

a) Zygomatic arch
b) Inferior orbital floor
c) Medial orbital floor
d) Basal skull
e) Nasal septum
The correct answer is A

Explanation
Fractures involving the zygomatic arch frequently cause trismus. Many of
these fractures are tripod-type fractures. Dental malocclusion and
temporomandibular joint pain are common. Inferior orbital fractures may
be associated with numbness of the cheek if the infraorbital nerve is
involved. They are also sometimes associated with limited upward gaze if
the infraorbital muscle is involved. The medial orbital wall may be
involved in blowout fractures. Limitation of abduction of the involved
eye may result. Basal skull fractures may result in bilateral
periorbital bruising or “raccoon eyes”. The most serious complication of
nasal fractures is a nasal septal hematoma. This can cause necrosis of
the septal cartilage if not promptly treated.
A 79-year-old male is admitted to the hospital because of a sudden
inability to ambulate. He has a past history of gout. On examination his
temperature is 38.2°C (100.8°F) and he has bilateral knee effusions. His
WBC count is 14,000/mm3 with 82% segs. His serum uric acid level is 8.5
mg/dL (N <6.5). Which one of the following would be most appropriate at
this point?

a) 24-hour urine collection for uric acid


b) Arthrocentesis
c) Initiation of allopurinol
d) Initiation of antibiotics
e) Initiation of furosemide (Lasix)
The correct answer is B

Explanation
Polyarticular arthritis often presents with fever, knee and other joint
effusions, and leukocytosis. A 24-hour urine collection is not routine,
is difficult for the patient, and typically does not change therapy.
Especially in cases where a joint effusion is accompanied by fever,
diagnostic arthrocentesis should be performed to help guide therapy.
Allopurinol should not be initiated during an acute gouty attack, but
may be started after a patient has recovered. Diuretics increase uric
acid levels.
A 75-year-old white female presents with hyponatremia, with a serum
level of 118 mEq/L, a urine osmolality >100 mOsm/kg H_2 O, and a serum
osmolality of 242 mOsm/kg H_2 O. She complains of some fatigue, but is
alert and oriented. Her blood pressure is 136/82 mm Hg. She has normal
thyroid, adrenal, cardiac, hepatic, and renal function. You admit her to
the hospital for treatment and observation.

Which one of the following is the most appropriate initial treatment?

a) Administration of 3% normal saline


b) Administration of normal saline
c) Free water restriction
d) Demeclocycline (Declomycin)
The correct answer is C

Explanation
This patient probably has the syndrome of inappropriate secretion of
antidiuretic hormone (SIADH). SIADH can be caused by CNS tumors, various
infections such as meningitis, and pneumonia. Several drugs can cause
this condition, including amiodarone, carbamazepine, SSRIs, and
chlorpromazine. In this fairly asymptomatic patient, initial management
should be free water restriction. As she is hemodynamically stable, she
does not need normal saline. Moreover, administration of normal saline
may exacerbate the hyponatremia, as the sodium may be rapidly excreted
while the water is retained. If she had a rapid onset and neurologic
symptoms such as seizures, hypertonic saline could be given.

Correction should be slow, with a goal of no more than a 1?2 mmol/L/hr


increase in the sodium level; a normal sodium level should not be
reached within the first 48 hours of treatment. Demeclocycline is
appropriate for patients who cannot adhere to the requirement for fluid
restriction, or who have recalcitrant hyponatremia despite restriction.
The most significant risk factor for spontaneous intracerebral
hemorrhage is

a) Atrial fibrillation
b) Cigarette smoking
c) Excessive alcohol use
d) Hyperlipidemia
e) Hypertension
The correct answer is E

Explanation
Intracranial hemorrhage (ICH) causes 10%-15% of all strokes, and is
associated with the highest mortality. The rate of ICH is expected to
double in the next 50 years as our population ages, as the incidence of
ICH increases after age 55. The incidence in blacks is twice that of
whites. Other risk factors include excessive alcohol use, low serum
cholesterol, and low education levels. However, hypertension is the most
common cause of ICH, and its most important risk factor. In patients
with hypertension, the risk for ICH increases further if the patient
also smokes, or is nonadherent to a medication regimen. Having
hypertension also increases the risk of ICH in people younger than 55
years. While atrial fibrillation is a risk factor for stroke, it is not
an independent risk factor for ICH.

The primary reason for treating uncomplicated streptococcal pharyngitis


is to

a) Reduce the severity of acute symptoms


b) Reduce the incidence of subsequent rheumatic fever
c) Reduce the risk of poststreptococcal glomerulonephritis
d) Prevent peritonsillar or retropharyngeal abscess
e) Shorten the duration of illness
The correct answer is B

Explanation
In the setting of streptococcal pharyngitis, antibiotics have been shown
to reduce the severity of acute symptoms, including peritonsillar or
retropharyngeal abscess, and to shorten the duration of illness by about
one day. However, it is unclear if antibiotic treatment reduces the risk
of glomerulonephritis. Thus, the primary reason for treating
uncomplicated streptococcal pharyngitis is to markedly reduce the
subsequent rheumatic fever.

A 40-year-old female comes to your office for a routine examination. She


has been in good health and has no complaints other than obesity. Her
mother is diabetic and the patient has had a child that weighed 9 lb at
birth. Her examination is negative except for her obesity. A fasting
glucose level is 128 mg/dL, and when repeated 2 days later it is 135
mg/dL. Which one of the following would be most appropriate at this point?

a) Diagnose type 2 diabetes mellitus and begin diet and exercise therapy
b) Begin an oral hypoglycemic agent
c) Order a glucose tolerance test
d) Tell the patient that she has impaired glucose homeostasis but is
not diabetic
The correct answer is A

Explanation
The criteria for diagnosing diabetes mellitus include any one of the
following: symptoms of diabetes (polyuria, polydipsia, weight loss) plus
a casual glucose level 200 mg/dL; a fasting plasma glucose level 126
mg/dL; or a 2-hour postprandial glucose level 200 mg/dL after a 75-gram
glucose load. In the absence of unequivocal hyperglycemia the test must
be repeated on a different day.

The criteria for impaired glucose homeostasis include either a fasting


glucose level of 100?125 mg/dL (impaired fasting glucose) or a 2-hour
glucose level of 140?199 mg/dL on an oral glucose tolerance test. Normal
values are now considered <100 mg/dL for fasting glucose and <140 mg/dL
for the 2-hour glucose level on an oral glucose tolerance test.

Which one of the following would be appropriate for pain control in a


critically ill patient in the intensive-care unit?

a) Meperidine (Demerol)
b) Ketorolac (Toradol)
c) Butorphanol (Stadol)
d) Hydromorphone (Dilaudid)
e) Methadone (Dolophine)
The correct answer is D

Explanation
Hydromorphone, morphine sulfate, and fentanyl are the three most
commonly used opioid analgesics for pain control in the intensive-care
unit (ICU). The other drugs listed are considered inappropriate for
analgesia in the ICU. Meperidine is not recommended because its
analgesia is not superior to that of morphine. Also, its metabolite
normeperidine is eliminated through the kidneys and has neurotoxic
effects, including seizures and delirium. NSAIDs such as ketorolac are
not recommended, because they are not superior to opiates. In addition,
they cause platelet inhibition and have an antiprostaglandin effect that
increases the risk of stress ulcers and renal injury.

Opiate agonist-antagonists such as butorphanol may precipitate


withdrawal in chronic opiate users or in ICU patients who have received
opiates for a prolonged period of time. They also may reverse opiate
analgesic effects. Methadone has an extended duration of activity that
increases the risk of accumulation and adverse effects with repeated dosing.
A 40 year old woman has episodic headaches that last 30 minutes and are
associated with sweating, palpitations, and feelings of apprehension.
She has lost 6.8 kg over the past three months. Physical examination
reveals a thin woman with a pulse of 112 beats/minute and a blood
pressure of 150/100 mmHg lying and 130/80 mmHg standing. Which one of
the following is the most likely diagnosis‟

a) Pheochromocytoma
b) Hyperaldosteronism
c) Posterior fossa tumor
d) Renal artery stenosis
e) Carcinoid syndrome
The correct answer is A

Explanation
A pheochromocytoma is a catecholamine-secreting tumor of chromaffin
cells typically located in the adrenals. It causes persistent or
paroxysmal hypertension. Diagnosis is by measuring catecholamine
products in blood or urine. Imaging tests, especially CT or MRI, help
localize tumors. Treatment involves removal of the tumor when possible.
Drug therapy for control of BP includes beta-blockade, possibly combined
with beta-blockade.
A 30 year old female comes to your clinic with the complaints of
palpitations.

The symptoms started 3 weeks ago. She also complains of weight loss,
increased appetite and heat intolerance. Her vital signs are within
normal limits except for a pulse of 100/min. Physical exam shows that
her thyroid gland is diffusely enlarged.

You order thyroid function tests which show low TSH, high T4 and T3.
After proper counseling, the patient refuses surgery or radioactive
iodine ablation. You prescribed propylthiouracil for her. Which of the
following is the most common potentially life threatening side effect of
this medication?

a) Hypocalcemia
b) Liver failure
c) Renal failure
d) Serious infection
e) Stomach cancer
The correct answer is D

Explanation
No blood! No Drug! Patients taking propylthiouracil for a long time
should be followed routinely with blood tests ?CBC with differential? to
guard against the potentially life threatening agranulocytosis which
could put the patient at risk for dissiminated infections.

Liver necrosis and failure is a possible side effect for


propylthiouracil; however it is very rare to occur. Always read the
statement of the question carefully; we are asking for the most common
one. Do not let them fool you on the exam!

Hypocalcemia is a common side effect following thyroidectomy.

Renal failure and stomach cancer are not associated with propylthiouracil.

A long-time user of smokeless tobacco noticed several whitish,


thickened, painless patches on the lining of his cheeks. Which of the
following is the most probable diagnosis‟

a) Aphthous ulcers
b) Adenocarcinoma
c) Keloids
d) Oral leukoplakia
e) Epidermolysis bullosa
The correct answer is D

Explanation
Leukoplakia is a condition in which thickened, white patches form on
your gums, on the inside of your cheeks and sometimes on your tongue.
These patches can't easily be scraped off.

Leukoplakia is the most common of all chronic mouth lesions. Although


anyone can develop leukoplakia, it's most common in older men. People
with compromised immune systems sometimes develop an unusual form of the
disorder called hairy leukoplakia.
The cause of leukoplakia is unknown. Many possible causes have been
linked to leukoplakia, including tobacco, long-term alcohol use and
other chronic irritants. Though mechanical irritants, such as rough
fillings or dentures, were once felt to be a cause of leukoplakia,
they're no longer believed to lead to this condition.

Tobacco use appears to be responsible for most cases of leukoplakia. The


vast majority of people who develop leukoplakia are smokers, and most
leukoplakic patches either improve or disappear within a year after
stopping smoking. Chewing tobacco and snuff also play a key role, as
many as three out of four regular users of "smokeless tobacco" products
eventually develop leukoplakia where they hold the tobacco against their
cheeks.
A 34 year old white male letter carrier has developed progressively
worsening dysphagia for liquids and solids over the past 3 months. He
says that he has lost about 30 lb during that time. On examination, you
note that he is emaciated and appears ill. His pulse rate is 98
beats/min, temperature 37.8°C (100.2°F), respiratory rate 24/min, and
blood pressure 95/60 mm Hg. His weight is 45 kg (99 lb) and his height
is 170 cm (67 in). His dentition is poor, and there is evidence of oral
thrush. His mucous membranes are dry.

You palpate small posterior cervical and axillary nodes. The heart,
lung, and abdominal examinations are normal. You promptly consult a
gastroenterologist, who performs upper endoscopy which reveals numerous
small ulcers scattered throughout the esophagus with otherwise normal
mucosa. As you continue to investigate, you take a more detailed
history. Which one of the following findings is most likely to be
related to the patient‟s problem?

a) Intravenous drug use


b) Easy bruisability
c) A family history of esophageal cancer
d) Chest pain relieved by nitroglycerin
e) Recent travel to Russia
The correct answer is A

Explanation
A young man with weight loss, oral thrush, lymphadenopathy, and
ulcerative esophagitis is likely to have HIV infection. Intravenuous
drug use is responsible for over a quarter of HIV infections in Canada.
Esophageal disease develops in more than half of all patients with
advanced infection during the course of their illness. The most common
pathogens causing esophageal ulceration in HIV-positive patients include
Candida, herpes simplex virus, and cytomegalovirus. Identifying the
causative agent through culture or tissue sampling is important for
providing prompt and specific therapy.

Which one of the following diseases is most commonly associated with


atrial fibrillation?

a) Crohn‟s disease (regional enteritis)


b) Tuberculosis
c) Hypothyroidism
d) Systemic hypertension
e) Rheumatoid arthritis
The correct answer is D

Explanation
Atrial fibrillation can be a cardiac sequela of primary noncardiac
diseases. Systemic hypertension is present in 45% of patients with
atrial fibrillation. Atrial fibrillation is also associated with
diabetes mellitus, pulmonary diseases (specifically COPD, primary
pulmonary hypertension, and acute pulmonary embolism), and acute ethanol
ingestion. Atrial fibrillation is not commonly associated with Crohn‟s
disease, tuberculosis, hypothyroidism (2% of patients), or rheumatoid
arthritis.
A 29 year old has her blood pressure at 180/105 mmHg. She admits to
stress at work and states that her father was diagnosed as hypertensive
at the age of 60 but, thus far, has not required treatment.

The results of investigations are given below:

Na 146 mmol/L
K 2.8 mmol/L
Urea 4.2 mmol/L
Creatinine 92 mmol/L

Which of the following secondary causes of hypertension does this


patient most likely have?

a) Coarctation of the aorta


b) Primary hyperaldosteronism
c) Renal artery stenosis, bilateral
d) Renal parenchymal disease
The correct answer is B

Explanation
In hyperaldosteronism, overproduction of aldosterone leads to fluid
retention and increased blood pressure. When too much aldosterone is
being produced, doctors examine the adrenal glands for a noncancerous
tumor (adenoma).
A 74 year old female is brought to the emergency room. She is
unresponsive and restless. Her breathing is laboured. Her family
indicates that she has a history of confusion and dementia. They also
indicate that she ingested "anti-freeze" from the garage. Blood gas
results are: pH 7.09, PCO2 7 mmHg, PO2 133 mmHg, HCO3 2 mmol/L, O2
saturation 97 %. The interpretation of the blood gas status is

a) Primary metabolic acidosis with respiratory compensation


b) Primary metabolic acidosis with no compensation
c) Primary respiratory acidosis with no metabolic compensation
d) Primary respiratory acidosis with metabolic compensation
e) Primary metabolic alkalosis with respiratory compensation
The correct answer is A

Explanation
The pH is less than 7.4. Therefore this patient is in acidosis. The PCO2
would be expected to be high (greater than 40) if the acidosis was due
to a respiratory cause (eg hypoventilation). Since this patients PCO2 is
quite low, she is in metabolic acidosis.
The following table describes the various acid/base states and the
corresponding pH, CO2 and HCO3 (bicarb) values:

PH PCO2 HCO3 Interpretation


< 7.4 > 40 > 24 Respiratory acidosis with compensated
< 7.4 > 40 Normal Respiratory acidosis pure
< 7.4 < 40 < 24 Metabolic acidosis with compensated
> 7.4 < 40 < 24 Respiratory alkalosis with compensated
> 7.4 < 40 Normal Respiratory alkalosis pure
> 7.4 > 40 > 24 Metabolic alkalosis with compensated

In this patient: pH = 7.09, PCO2 = 7 mmHg, Bicarb = 2

Overall change is acid.


Respiratory change is alkaline - therefore NOT contributing to the
acidosis.
Metabolic change is acid - therefore responsible for the acidosis.
The components are pulling in opposite directions. So it is primarily a
metabolic problem with some respiratory compensation.
Interpretation: metabolic acidosis with respiratory compensation.

A 74 year old, right-handed man presents with a past medical history of


hypertension and dyslipidemia for 30 years. He is a retired banker who
recently has had trouble calculating his restaurant bill. He also
notices that his writing has deteriorated. On physical exam, he has
difficulty naming his fingers and is confused with distinguishing left
from right. The lesion is most likely in which part of the brain?

a) Occipital
b) Parietal
c) Temporal
d) Frontal
The correct answer is B

Explanation
Damage to the front part of the parietal lobe on one side causes
numbness and impairs sensation on the opposite side of the body.
Affected people have difficulty identifying a sensation's location and
type (pain, heat, cold, or vibration). Damage to the back part of the
parietal lobe causes right-left disorientation and problems with
calculations and drawing. Damage to the right parietal lobe can cause
apraxia, the inability to perform simple skilled tasks, such as brushing
the hair or dressing. Sudden damage to the parietal lobe can cause
people to ignore the serious nature of their disorder and even neglect
or deny the existence of the side of the body opposite the injury. Such
people may become confused or delirious and unable to dress themselves
or to perform other ordinary tasks.
You note a skin lesion on the nose of a 70-year-old male painter during
a visit for a routine upper respiratory infection. He tells you that the
lesion “sometimes bleeds a little”. It is a raised, smooth, pale,
pearly, shiny papule with prominent telangiectasia evidence across its
surface.

This lesion is most likely


a) A spider angioma
b) A basal cell carcinoma
c) An atypical melanoma
d) Actinic keratosis
e) Sebaceous hyperplasia
The correct answer is B

Explanation
The lesion described is a basal cell carcinoma, which is an indication
to biopsy the lesion and/or refer the patient to a dermatologist.

Which one of the following features is of most specific value in


differentiating optic neuritis from papilledema?

a) Loss of vision
b) Blurring of the disc margins
c) Enlargement of the blind spot
d) Swelling of the disc
e) Distension of retinal veins
The correct answer is A

Explanation
Papilledema is swelling of the optic disk due to increased intracranial
pressure. All other causes of optic disk swelling, such as that due to
malignant hypertension or thrombosis of the central retinal vein, do not
involve increased intracranial pressure and therefore are not causes of
papilledema. Papilledema requires an immediate search for the cause.
There are no early symptoms, although transiently diminished vision
lasting only seconds can occur. Diagnosis is by ophthalmoscopy with
further tests, usually brain imaging, to determine cause. Treatment is
directed at the underlying condition.

Optic neuritis is inflammation of the optic nerve. Symptoms are usually


unilateral, with eye pain and partial or complete vision loss. Diagnosis
is primarily clinical. Treatment is directed at the underlying
condition; most cases resolve spontaneously.
Which one of the following is the most common type of anemia in the
geriatric age group?

a) Iron deficiency anemia from blood loss


b) Nutritional anemia from vitamin B_12 deficiency
c) Anemia of chronic disease
d) Hemolytic anemia
e) Myelodysplastic anemia
The correct answer is C

Explanation
The most common cause of anemia in the geriatric age group is chronic
disease (35%?40%), which is often asymptomatic, followed by iron
deficiency anemia (8%?15%), chronic renal disease (6%?8%), blood loss
(7%), myelodysplasia (5%), and vitamin B_12 deficiency (5%). Renal
insufficiency accounts for the greatest percentage of patients with
anemia of chronic disease.
You are counseling a 24-year-old female about treatment for her acne.
Despite conventional treatment with topical agents and systemic
antibiotics, she continues to experience flares of inflammatory acne and
believes that her acne is severely limiting her social relationships and
her ability to make a favorable impression during job interviews. She
asks you about using isotretinoin (Accutane).

Which one of the following would be accurate advice?

a) Isotretinoin is most useful in treating comedonal acne


b) Concomitant therapy with topical keratolytic agents is usually well
tolerated
c) Teratogenicity is the most devastating adverse effect
d) Elevations in liver enzymes often necessitate discontinuing treatment
e) Osteoporosis and osteophyte formation are common late complications
of treatment
The correct answer is C

Explanation
Isotretinoin is FDA-labeled only for treatment of severe, recalcitrant,
nodular acne. Because of documented clinical experience with the drug,
however, in addition to additional published evidence, there is
international consensus that isotretinoin may be appropriate in other
situations. These include an inadequate response to appropriate
conventional therapy for less severe acne, scarring inflammatory acne,
and acne that causes severe psychologic distress. When isotretinoin is
used, adjuvant therapy with topical keratolytics and drying agents
should be discontinued because concomitant use may lead to excessive
dryness.

The most devastating adverse effect of isotretinoin is teratogenicity.


Major malformations may occur in 25%?30% of fetuses exposed to the drug.
Liver enzyme levels should be monitored periodically, although
elevations beyond the reference range are rare. If elevations do occur,
it may be necessary to reduce the dosage, or in rare instances to
discontinue therapy. Long-term retinoid therapy may be complicated by
skeletal changes, including osteoporosis and osteophyte formation. No
studies have reported notable bony changes associated with short-term
therapy of the duration typical for treatment of acne (usually 5 months).
A 30 year old man has been on life support systems for the past 48 hours
following blunt head trauma. Brain death cannot be established in this
patient if there is the presence of which of the following?

a) Carotid blood flow


b) Cremasteric reflexes
c) Elevated serum aminoglycoside concentrations
d) Hypothermia
e) Marked cerebral edema
The correct answer is D

Explanation
Hypothermia should be excluded before making a diagnosis of brain death.
If present, it can be the cause of a flat EEG. Carotid blood flow
implies the presence of cerebral circulation, and hence the presence of
some viable tissue. The absence of carotid blood flow does not help
diagnosis either due to the presence of the vertebral system.
Cremasteric reflexes are spinal reflexes (and not brainstem reflexes),
and their presence does not exclude the diagnosis. Elevated serum
aminoglycoside concentration is nonspecific, and has little bearing.
Cerebral edema does not rule out brain death.
The long term management of atrial fibrillation includes

a) Procainamide
b) Calcium channel blocker
c) ACEI
d) Warfarin
The correct answer is D

Explanation
Atrial fibrillation is a rapid, irregularly irregular atrial rhythm.
Symptoms include palpitations and sometimes weakness, dyspnea, and
presyncope. Atrial thrombi often form, causing a significant risk of
embolic stroke. Diagnosis is by ECG.

Treatment involves ventricular rate control with beta-blockers, calcium


channel blockers or digoxin.

Measures to prevent thromboembolism are necessary. Warfarin titrated to


an INR of 2 to 3 should be used. Anticoagulants should be continued
indefinitely for patients with recurrent paroxysmal, persistent, or
permanent AF in the presence of risk factors for thromboembolism.
Healthy patients with a single episode of lone AF are anticoagulated for
4 weeks.

Aspirin is less effective than warfarin but is used for patients with no
risk factors for thromboembolism or those with contraindications to
warfarin.
Which one of the following is more typical of "fainting" as a conversion
symptom than of a syncopal attack due to orthostatic hypotension?

a) Bradycardia
b) Muscle twitching
c) Absence of pallor or sweating
d) Urinary incontinence
e) Rapid recovery
The correct answer is C

Explanation
Symptoms of conversion disorder often develop abruptly, and onset can
usually be linked to a very stressful event. Symptoms are limited to
those that affect voluntary motor or sensory function and suggest a
neurologic or general medical condition (eg, impaired coordination or
balance, weakness, or paralysis of an arm or a leg or loss of sensation
in a body part). Other symptoms can include seizures, blindness, double
vision, deafness, aphonia, difficulty in swallowing, sensation of a lump
in the throat, and urinary retention.
A 57-year-old male has laboratory screening performed prior to his
annual examination. One of his six fecal occult blood tests is positive.
A CBC, chemistry panel, lipid panel, and prostate-specific antigen level
are all normal.

Which one of the following would be the most appropriate course of action?

a) Repeat the fecal occult blood testing in 3 months


b) Perform a rectal examination in the office, and if a stool guaiac is
negative repeat the
fecal occult blood testing in 3 months
c) Refer for colonoscopy
d) Refer for flexible sigmoidoscopy
e) Refer for pre- and post-contrast CT of the abdomen and pelvis
The correct answer is C

Explanation
Asymptomatic persons between 50 and 80 years of age should be screened
for colon cancer. A single fecal occult blood test (FOBT) performed in
the office after a digital rectal examination is an inadequate screen,
with significant false-negative and false-positive rates. Annual
six-sample FOBT (two samples from three different bowel movements)
improves colon cancer detection but can still have false-negative
results. If any of the six samples tests positive for occult blood,
colonoscopy is recommended.
A 49 year old male with a history of hypertension presents with a 4-week
history of a swollen, painful knee. There is no history of injury. He
was started on an antihypertensive medication regimen 2 months before
the knee problem developed. You suspect gout, and laboratory evaluation
reveals a serum uric acid level of 10.5 mg/dL (N 3.6-8.5).

The antihypertensive agent most likely to cause this problem is

a) Metoprolol (Lopressor)
b) Enalapril (Vasotec)
c) Losartan (Cozaar)
d) Hydrochlorothiazide
e) Terazosin (Hytrin)
The correct answer is D

Explanation
While hypokalemia is the most common side effect of the thiazide-type
diuretics, they also reduce uric acid excretion by the kidney, which can
result in hyperuricemia and clinical gout. Patients on thiazides who
develop a single-joint arthropathy should be investigated for this
possibility.
A 75-year-old patient with underlying chronic renal failure requires
cardiac catheterization. Which one of the following interventions is
most likely to help prevent acute renal failure due to contrast-induced
nephropathy?

a) Hydration with normal saline and mannitol


b) Hydration with sodium bicarbonate
c) Hydration plus a loop diuretic
d) Administering fenoldopam (Corlopam) prior to the procedure
e) Infusion of natriuretic peptides prior to the procedure
The correct answer is B
Explanation
Several studies have demonstrated that hydration with sodium bicarbonate
reduces the risk of contrast-induced nephropathy in those undergoing
cardiac catheterization. Studies of interventions to prevent renal
failure in patients at high risk have shown that mannitol plus hydration
does not reduce acute renal failure compared to hydration alone.
Randomized, controlled trials have shown that fenoldopam does not
decrease the need for dialysis or improve survival. One systematic
review found that low-osmolality contrast media reduced nephrotoxicity
in persons with underlying renal failure requiring studies using contrast.

One systematic review and one subsequent randomized, controlled trial


found that adding loop diuretics to fluids was not effective and may
actually increase the possibility of acute renal failure compared to
fluids alone. A large randomized, controlled trial found no significant
difference between natriuretic peptides and placebo in preventing acute
renal failure induced by contrast media.

You have hospitalized a 67 year old obese white female for urosepsis.
She has completed a course of intravenous antibiotics. She has
hypertension, diabetes mellitus, and congestive heart failure. In
addition, she has renal failure which has worsened, and she has been on
hemodialysis for 1 week. The chart lists her medications as enalapril
(Vasotec), furosemide (Lasix), labetalol (Trandate, Mormodyne), insulin,
and heparin for flushing intravenous lines. For the past 2 days she has
had nosebleeds. A CBC is normal except for mild anemia and marked
thrombocytopenia of 28,000/mL. Which of the following is most likely the
cause of her thrombocytopenia?

a) Enalapril
b) Furosemide
c) Labetalol
d) Insulin
e) Heparin
The correct answer is E

Explanation
A number of medications can cause thrombocytopenia, but heparin is a
more likely cause than enalapril, furosemide, labetalol, or insulin.
Even the small doses of heparin used to flush intravenous lines can be a
source of thrombocytopenia.
A 45-year-old white female is diagnosed with hepatitis C. Which one of
the following would be most useful to determine the stage of the disease?

a) Liver enzymes (ALT/AST)


b) A prothrombin time
c) A hepatitis C viral assay
d) A CT scan of the liver
e) A liver biopsy
The correct answer is E

Explanation
Approximately 2% of the U.S. population is positive for hepatitis C.
Intravenous drug users and those who received blood transfusions before
1990 are at risk. Screening is done through antibody screening and
confirmed by viral assay. Liver enzyme testing is not reliable. The best
way to determine the stage of the disease is with a liver biopsy.
A 68 year old man presents with a chief complaint that solid food gets
stuck in the middle of his chest. In addition, he admits to a 11 kg
weight loss over the last 3 months. Which one of the following is the
most likely diagnosis‟

a) Esophagitis
b) Lower esophageal ring
c) Esophageal carcinoma
d) Cerebrovascular accident
e) Myocardial infarction
The correct answer is C

Explanation
The most common malignant esophageal tumor is squamous cell carcinoma,
followed by adenocarcinoma. Symptoms are progressive dysphagia and
weight loss.

Diagnosis is by endoscopy, followed by CT and endoscopic ultrasound for


staging. Treatment varies with stage and generally includes surgery with
or without chemotherapy and radiation. Long-term survival is poor except
for those with local disease.
A woman complains of early morning headache. She also has vomited a few
times and has nystagmus. This has been going on for 6 months. What is
the next step in your investigation?

a) Referral to eye specialist


b) CT of head
c) EEG
d) Temporal artery biopsy
The correct answer is B

Explanation
Many symptoms result from increased intracranial pressure. The most
common is headache. Headache may be most intense when patients awake
from deep non-REM sleep (usually several hours after falling asleep)
because hypoventilation, which increases cerebral blood flow and thus
intracranial pressure, is usually maximal during non-REM sleep.

When intracranial pressure is very high, the headache may be accompanied


by vomiting, which may occur with little preceding nausea. Papilledema
develops in about 25% of patients with a brain tumor but may be absent
even if intracranial pressure is increased. In infants and very young
children, increased intracranial pressure may enlarge the head. If
intracranial pressure increases sufficiently, brain herniation occurs.

If an intracranial mass is suspected, a CT or MRI of the head/brain


should be done.
Compared with younger adults, healthy older adults

a) Fall asleep more quickly at bedtime


b) Awaken less frequently during the night
c) Spend less time awake in bed
d) Spend fewer hours in stages of deep sleep
The correct answer is D
Explanation
Normal older adults require less total sleep time. This change begins by
the age of 50, and gradually increases with time. By age 75, total sleep
time is reduce to 6 or 7 hours per night, and by age 85, 5 to 6 hours of
total sleep time is biologically and physiologically normal. Older
adults take longer to fall asleep, awaken more frequently at night,
spend more time awake in bed, and spend far less time in stages of deep
sleep.
A 40 year old man complains of chronic fatigue for the past year. His
wife states he snores loudly and at times she thinks he seems to stop
breathing. Which one of the following is the most likely diagnosis‟

a) Narcolepsy syndrome
b) Catalepsy syndrome
c) Sleep apnea syndrome
d) Kleine-Levin syndrome
e) Hypersomnia disorder
The correct answer is C

Explanation
Obstructive sleep apnea consists of episodes of partial or complete
closure of the upper airway that occur during sleep and lead to
breathing cessation (defined as a period of apnea > 10 sec). Symptoms
include restlessness, snoring, recurrent awakening, morning headache,
and excessive daytime sleepiness.

Diagnosis is based on sleep history, and polysomnography. Treatment is


with nasal continuous positive airway pressure, oral appliances, and, in
refractory cases, surgery. Prognosis is good with treatment. Most cases
remain undiagnosed and untreated and are often associated with
hypertension, heart failure, and injury or death from motor vehicle
crashes and other accidents resulting from hypersomnolence.
Hydronephrosis is associated with all of the following conditions, except

a) Post-ureteral valve
b) Obstructive uropathy
c) BPH
d) Nephrotic syndrome
The correct answer is D

Explanation
Hydronephrosis is distention (dilation) of the kidney with urine, caused
by backward pressure on the kidney when the flow of urine is obstructed.
Hydronephrosis commonly results from an obstruction located at the
junction of the ureter and renal pelvis (ureteropelvic junction). Causes
of this type of obstruction include the following:

Structural abnormalities, for example, a birth defect in which the


insertion of the ureter into the renal pelvis is too high or there is
inadequate development of the ureteral muscles (congenital ureteropelvic
junction obstruction).

Hydronephrosis can also result from an obstruction below the


ureteropelvic junction or from backflow (reflux) of urine from the
bladder. Causes of this type of obstruction include the following:

Obstruction in the ureter and an obstruction that prevents urine flow


from the bladder to the urethra, resulting from prostate enlargement
(most often caused by a condition called benign prostatic hyperplasia, BPH).
A 90 year old Asian female who lives in a nursing home is noted to have
> 100,000 Escherichia coli on a urinalysis performed because her urine
„smelled strong?. She is afebrile and is asymptomatic. Which one of the
following is most appropriate?

a) Antibiotic treatment for 3 days


b) Antibiotic treatment for 10 days
c) A repeat culture and treatment if positive
d) Foley catheter insertion
e) No treatment
The correct answer is E

Explanation
Antibiotic treatment of nursing-home patients with asymptomatic
bacteriuria is not beneficial. Chronic incontinence is not improved,
subsequent episodes of symptomatic urinary tract infection are not
reduced, and there is no decrease in overall mortality.

Ehrlichiosis may result from exposure to

a) Freshwater snails
b) Fleas
c) Macaques
d) Rats
e) Ticks
The correct answer is E

Explanation
Human ehrlichiosis is an illness from infection with Ehrlichia
chaffeensis. Monocytotropic ehrlichiosis is transmitted by the bite of
the lone star tick, Amblyomma americanum, which feeds primarily on deer.
Granulocytotropic ehrlichiosis is transmitted by the bite of Ixodes
ticks. Freshwater snails are vectors for schistosomiasis, and fleas are
vectors for plague and murine typhus. Macaque bites can transmit B
virus, which results in CNS disease, and rat bites may result in rat
bite fever.
According to the Seventh Report of the Joint National Committee on
Prevention, Detection, Evaluation, and Treatment of High Blood Pressure
(JNC-7), which one of the following should be first-line treatment for
otherwise healthy older adults with hypertension?

a) ACE inhibitors
b) ß-Blockers
c) Calcium channel blockers
d) Thiazide diuretics
e) Alpha-Blockers
The correct answer is D

Explanation
JNC-7 recommendations for treating hypertension are similar in the
general population and in older persons. The key points include the
recommendation to treat isolated systolic blood pressure, and that
thiazide diuretics should be first-line treatment.
You have been a family doctor of Mr. Smith, a 50 year old hypertensive
man, for 10 years.
2 weeks, you have diagnosed Mr. Smith with diabetes mellitus. After
appropriate counseling and management of his condition, what is the best
screening test for nephropathy that you should order?

a) Abdominal CT scan
b) Creatinine clearance
c) Dipstick testing
d) HbA1C
e) Urine for albumin/creatinine ratio
The correct answer is E

Explanation
The patient is at high risk for developing nephropathy because he has
both diabetes and hypertension. The best and most convenient screening
test for nephropathy is spot urine testing for albumin/creatinine ratio
to detect microalbuminuria which is the earliest manifestation of
diabetic nephropathy. A ratio less than 30 is normal while a ratio
between 30 and 300 suggests microalbuminuria.

Creatinine clearance is useful for assessment of advanced renal disease.

Dipstick detects macroalbuminuria and not microalbuminuria which is a


late manifestation of the nephropathy.

CT scan of the abdomen is appropriate to delineate gross lesions of the


kidney and is never appropriate for nephropathy screening.

HbA1C is a screening test for diabetes control in the last three months
and does not reflect the kidney condition.
A CBC obtained as part of a life insurance examination of a healthy
33-year-old African-Canadian male reveals ?clumped platelets.? Which one
of the following should you do next?

a) Repeat the platelet count using a specimen collection tube with an


alternative
anticoagulant
b) Refer the patient to a hematologist for bone marrow evaluation
c) Hospitalize the patient for aggressive antiplatelet therapy and
parenteral
anticoagulation
d) Treat the result as a laboratory error and take no further action

The correct answer is A

Explanation
Clumped platelets usually are an artifact secondary to antibodies to the
anticoagulant used to obtain blood for a CBC. An alternative
anticoagulant, usually citrate, should be used, and the test repeated to
determine an accurate count before undertaking any additional measures.
A 55-year-old male presents with a hemoglobin level of 19.0 g/dL (N
13.0-18.0) and a hematocrit of 56.0% (N 37.0-49.0) on multiple
occasions. He is a nonsmoker with normal oxygen saturations. An RBC mass
study is elevated and an erythropoietin level is low.

Which one of the following is the most likely diagnosis‟


a) COPD
b) Polycythemia vera
c) Renal artery stenosis
d) Renal cell carcinoma
e) Obstructive sleep apnea
The correct answer is B

Explanation
Polycythemia vera is the most common chronic myeloproliferative
disorder. It is most often brought to light by an elevated hemoglobin or
hematocrit. Hematocrit levels greater than 50% in men and 45% in women
may be abnormal. An elevated RBC mass study (the first test done to
confirm polycythemia) associated with a low erythropoietin (EPO) level
is diagnostic of polycythemia vera. COPD, renal artery, stenosis, renal
cell carcinoma, and obstructive sleep apnea may cause secondary forms of
polycythemia and are associated with elevated EPO levels.
You are the attending physician at a long-term care facility. A new
resident, an 85-year-old female, presents for an initial visit. Upon
reviewing her history, you find that she is on 18 different medications.
While attempting to obtain additional history and medical information,
you decide to stop or decrease some of her medications and monitor her
response.

Which one of the following would be most appropriate to stop or decrease


initially?

a) Sertraline (Zoloft), 25 mg daily


b) Acetaminophen/diphenhydramine (Tylenol PM HS), 500 mg/25 mg dailyB)
c) Dipyridamole/aspirin (Aggrenox), 200 mg/25 mg
d) Digoxin, 0.125 mg every other day
e) Omeprazole (Prilsosec), 20 mg daily
The correct answer is B

Explanation
Polypharmacy is a term commonly associated with physicians‟ prescribing
tendencies for the elderly population. Although the term carries a
negative connotation, the use of numerous medications is necessary in
some elderly patients. However, some medications have been identified as
having a considerably higher potential to cause problems when prescribed
to elderly patients.

The older antihistamines, primarily diphenhydramine, cause many adverse


CNS effects such as cognitive slowing and delirium in older patients.
This effect is more pronounced in elderly patients with some degree of
preexisting cognitive impairment. The anticholinergic properties of
older antihistamines produce effects such as dry mouth, constipation,
blurred vision, and drowsiness. Thus, the use of antihistamines as
sleeping agents or for symptomatic relief often leads to problems in
elderly patients. The sedative effect of older antihistamines also
increases older patients‟ risks of falls. Hip fracture and subsequent
death have been reported in patients who use older antihistamines such
as diphenhydramine. Thus, the acetaminophen/diphenhydramine would be an
appropriate medication to stop initially.

Sertraline is an SSRI, a preferred class for the treatment of depression


in the elderly as compared to the tricyclic antidepressants, which are
associated with several side effects. Therefore, until further
information is obtained, it is appropriate to continue the
dipyridamole/aspirin in this patient. When used in elderly patients with
congestive heart failure, digoxin should be given in a dosage no greater
than 0.125 mg daily; the lose dose used in this individual is not
inappropriate until the reason for its use is clarified. Fexofenadine is
one of the newer nonsedating antihistamines that is recommended for
treating allergy symptoms in the elderly.
A 47 year old female with jaundice has a history of nausea and vomiting,
anorexia and depression. She is on rifampin, INH and pyridoxine.
Physical exam shows hepatomegaly and abdominal tenderness. Labs reveal
her SGOT (AST) and SGPT (ALT) are greatly increased, alkaline
phosphatase is slightly increased. What is the most likely diagnosis‟

a) Drug induced hepatitis


b) Viral hepatitis
c) Acute pancreatitis
d) Liver cancer
The correct answer is A

Explanation
INH stands for isoniazid. But INH could also stand for ?Injures Neurons
and Hepatocytes‟.

Because of the risk of hepatitis, patients taking INH should have their
blood liver tests monitored monthly and should notify their physicians
immediately if symptoms or signs of hepatitis arise. These symptoms and
signs include unexplained loss of appetite, nausea, vomiting, dark
urine, yellow skin or a yellowish tinge to the whites of the eyes,
persistent fatigue, weakness or fever of greater than 3 days duration,
or abdominal tenderness or discomfort, especially in the right upper
part of the abdomen.

Damage to nerves (peripheral neuropathy) may occur with INH and cause
numbness and tingling of the hands or feet. Other rare side effects of
the nervous system include encephalopathy (inflammation of the brain),
optic neuritis (inflammation of the nerve coming from the eye), atrophy
(degeneration) of the nerve coming from the eye, seizures, impaired
memory, psychosis. Pyridoxine (vitamin B6), taken daily, decreases the
risk of neural side effects.
The Mini-Mental State Examination (MMSE) tests for

a) Mood
b) Behavior
c) Intelligence quotient
d) Cognitive function
e) Functional impairment
The correct answer is D

Explanation
The MMSE is most commonly used in clinical settings. It is considered
valuable because it assesses a broad range of cognitive abilities (i.e.,
memory, language, spatial ability, set shifting) in a simple and
straightforward manner. In addition, the wide use of the MMSE in
epidemiologic studies has yielded cutoff scores that facilitate the
identification of patient with cognitive dysfunction.
A 62 year old woman who is a regular patient calls the office because
she has developed severe muscle weakness, muscle cramps and polyuria.
She began treatment 6 weeks ago with 50 mg of chlorthalidone daily for
mild-to-moderate essential hypertension. The most likely explanation for
her symptoms is the development of which of the following?

a) Diabetes mellitus
b) Hypokalemia
c) Hypomagnesemia
d) Hyponatremia
e) Metabolic acidosis
The correct answer is B

Explanation
Hypokalemia is a very common side effect of nonpotassium-sparing
diuretics (e.g., chlorthalidone). This is often more pronounced in the
older age group. Patients usually complain of muscle weakness, fatigue,
and cramps. Constipation and ileus characterize the smooth muscle
involvement, whereas hyporeflexia, flaccid paralysis, and tetany are
signs of severe hypokalemia.
The manager of a local chicken processing plant asks about arranging
screening tests for his 100 employees. Several are smokers, and one
individual was recently found to have lung cancer. He asks what the best
and most cost-effective way to screen for this would be. Based on
randomized, controlled trials and recent guidelines, you would advise

a) annual chest radiographs


b) annual sputum cytology
c) both sputum cytology and chest radiographs annually
d) annual spiral CT
e) no screening for asymptomatic individuals
The correct answer is E

Explanation
There is insufficient evidence to recommend either for or against
screening for cancer of the lung. To date, screening has not been shown
to decrease the number of deaths from lung cancer. Case control studies
done in Japan suggest improved mortality with annual chest radiographs,
and a large randomized, controlled trial is now under way. While
screening CT in high-risk groups would identify a high percentage of
stage 1 lung cancers, there is no data available at this time from
randomized studies to show that this is worthwhile. Studies of this
issue are also currently under way, however.
What is the mechanism of action in organophosphate poisoning?

a) Cholinesterase inhibition
b) Cholinesterase activation
c) Catecholamine inhibition
d) Catecholamine activation
The correct answer is A

Explanation
Organophosphate compounds are a diverse group of chemicals used in both
domestic and industrial settings. Examples of organophosphates include:
insecticides, nerve gases and herbicides. Nerve agents have also been
used in battle, notably in Iraq in the 1980s. Additionally, chemical
weapons still pose a very real concern in this age of terrorist activity.
The primary mechanism of action of organophosphate pesticides is
inhibition of acetylcholinesterase (AChE). AChE is an enzyme that
degrades the neurotransmitter acetylcholine (ACh) into choline and
acetic acid. ACh is found in the central and peripheral nervous system,
neuromuscular junctions, and red blood cells (RBCs).

Organophosphates inactivate AChE by phosphorylating the serine hydroxyl


group located at the active site of AChE. This leads to an increase in
the amount of Acetylcholine in the body and a wide variety of reactions.

A 70 year old female patient comes to your office complaining of nausea


and early satiety of two months duration.

Every time she eats, she feels bloated rapidly, she nauseates and
sometimes vomits. She denies any pain in the abdomen or any heartburn
symptoms. Her past medical history is significant for diabetes mellitus
diagnosed 20 years ago. She stopped the oral hypoglycemic drugs 2 years
ago and began taking insulin regularly.

Her vital signs are within normal limits and the physical exam is
unremarkable. What is the most appropriate next step in the management
of this patient?

a) Cisapride
b) Domperidone
c) Metoclopramide
d) Omeprazole
e) Triple antibiotics for H. pylori
The correct answer is C

Explanation
The patient is most likely suffering from autonomic neuropathy causing
gastroparesis which most commonly presents with anorexia, nausea,
vomiting, abdominal bloating and early satiety. The best management for
this condition is to take small frequent meals, improve glycemic control
and take metoclopramide.

Domperidone is a prokinetic and antiemetic drug that may be used for


gastroparesis; however, it is much less effective than metoclopromide.

Omeprazole and H. pylori would be appropriate for peptic ulcer disease


not gastroparesis.

Cisapride is a serotonin agonist drug that is very effective for


gastroparesis but it is not recommended initially. It is advised in
resistant cases because it can increase the risk of cardiac arrhythmias.
A 49 year old man who smokes two packs of cigarettes a day presents with
a lung mass on x-ray and recent weight gain. Laboratory examination
shows hyponatremia with hyperosmolar urine. The patient probably has
which one of the following diagnoses‟

a) Renal failure
b) Pituitary failure
c) Conn's syndrome
d) Cardiac failure
e) Inappropriate ADH (secretion)
The correct answer is E

Explanation
SIADH (syndrome of inappropriate secretion of anti-diuretic hormone) is
a common paraneoplastic syndrome that affects the endocrine system. This
syndrome is most often associated with small-cell lung cancer; however,
other cancers such as brain tumors, leukemia, lymphoma, colon, prostate,
and head and neck cancers can lead to SIADH. SIADH is caused by the
inappropriate production and secretion of arginine vasopressin or
antidiuretic hormone (ADH) by tumor cells.

Patients with SIADH may not have symptoms, especially in the early
stages. When symptoms do occur they are usually related to hyponatremia,
which leads to central nervous system toxicity if left untreated. Lab
values will show concentrated urine in the presence of plasma
hypo-osmolality (hyponatremia).

Signs and symptoms associated with hyponatremia include fatigue,


anorexia, headache and mild alteration in mental status in early stages.
If SIADH remains untreated, symptoms can progress to confusion,
delirium, seizures, coma, and death. Treatment approaches for SIADH are
to treat the underlying tumor and restriction of fluids. More severe
cases may require the administration of medications.
During the morning rounds, a second year resident presents a patient as
"a 58 year old female who suffers from rigidity and tremors. She has
been diagnosed with a neurodegenerative disease involving the
depigmentation of substantia nigra and loss of dopaminergic input to
basal ganglia."
Identify the medication that would worsen rather than improve this
patient's condition?

a) Levodopa
b) Bromocriptine
c) Chlorpromazine
d) Pergoilde
e) Selegiline
The correct answer is C

Explanation
Chlorpromazine is a phenothiazide derivative effective in
the treatment of positive symptoms of schizophrenia. One of the side
effects of phenothiazide derivatives is induction of pseudoparkinsonism.
Since the patient is already suffering from the symptoms of parkinson‟s
disease, administration of chlorpromazine would further worsen the
condition.

All remaining options are Antiparkinson medications and could be used in


the management of this patient.
An elderly male with mild dementia is involved in a motor vehicle
accident, and his son is concerned that it may no longer be safe for him
to drive. Which one of the following has the legal authority to revoke
or restrict this patient‟s driver‟s license?

a) The patient‟s son


b) A psychiatric consultant
c) The person designated as having power of attorney
d) The family physician
e) A representative of the state department of motor vehicles
The correct answer is E

Explanation
The family physician or a consulting psychiatrist can make
recommendations regarding driving, and the patient‟s family or the
person designated as having power of attorney can withhold access to a
vehicle, but the state motor vehicle department reserves final judgement
in these situations, and may require a driving test.
Which one of the following is the first-line antibiotic treatment for
uncomplicated acute otitis media?

a) Ceftriaxone (Rocephin)
b) Amoxicillin
c) Azithromycin (Zithromax)
d) Cefuroxime (Ceftin)
e) Trimethoprim/sulfamethoxazole (Bactrim, Septra)
The correct answer is B

Explanation
Amoxicillin remains the recommended first-line treatment for
uncomplicated acute otitis media. Various other antimicrobial agents
have not proved to be more efficacious, and are associated with more
frequent side effects.
Which of the following statements about aspiration of a peanut into the
tracheobronchial tree is *false*?

a) The peanut is more likely to be in the left lower lobe bronchus


b) Expiratory wheeze is the most likely finding on physical exam
c) The peanut should be removed with the patient under general
anesthesia through an open bronchoscope with forceps designed to grasp
peanuts
d) A chest x-ray may show atelectasis distal to the blocked bronchus
e) Pneumonia may be a complication
The correct answer is A

Explanation
In the above diagram one can see that the right bronchus is more
vertical and therefore a peanut is much more likely to go down the right
bronchus than the more horizontal left bronchus.
Which one of the following skin infections should initially be treated
with oral antifungal therapy?

a) Tinea capitis
b) Tinea corporis
c) Tinea cruris
d) Erythrasma
e) Mycosis fungoides
The correct answer is A

Explanation
Most tinea infections respond to topical therapy, but oral therapy is
required for tinea capitis so that the drug will penetrate the hair
shafts (SOR B). Tinea corporis may require oral therapy in severe cases,
but usually responds to topical therapy (SOR A). Oral therapy has a
higher likelihood of side effects.

Erythrasma and mycosis fungoides are not fungal diseases.

Mr. William is a 35 year old businessman that presents to your clinic


complaining of intermittent abdominal cramps and diarrhea of two months
duration.

The episodes of diarrhea have increased in frequency recently and have


been associated with blood per stool in the last few days. After a
detailed history and physical exam, you ordered several blood tests
which were significant for a microcytic anemia and elevated ESR.
Colonoscopy reveals extensive disease from terminal ileum to the rectum
with multiple ulcerations. Biopsies of the lesions reveal the presence
of non-caseating granulomas.
Which of the following findings would suggest Crohn‟s disease rather
than ulcerative colitis‟

a) Crypt abscess
b) Elevated ESR
c) Hyperplastic polyps
d) Non caseating granulomas
e) Pseudomembranes
The correct answer is D

Explanation
Ulcerative colitis and Crohn's disease are the two major types of
inflammatory bowel diseases. Non-caseating granulomas are pathognomonic
of CD, and not seen in UC. However, it should be noted that they are
present in most and not all patients with CD.

Elevated ESR is a very non-specific marker of inflammation; it can be


elevated in inflammatory, neoplastic and rheumatic diseases.

Crypt abscesses may be seen in both UC and CD.

Pseudomembranes are the hallmarks of Clostridium difficile colitis.

Hyperplastic polyps are inflammatory polyps that denote chronic


inflammatory diseases; they are non specific lesions.

A 63-year-old female presents for a routine evaluation. She asks if she


and her husband, age 69, would benefit from the herpes zoster vaccine.
You advise her that

a) the vaccine is more than 90% effective in preventing herpes zoster


b) postherpetic pain reduction is the greatest benefit of the vaccine
c) the efficacy of the vaccine decreases after age 70
d) the vaccine is more effective for reducing occurrences than for
reducing postherpetic
neuralgia
The correct answer is B

Explanation
The live attenuated varicella-zoster vaccine has been shown to reduce
outbreaks of herpes zoster by 51% in vaccinated individuals. The
greatest benefit is the reduction in the severity and duration of
postherpetic neuralgia. Efficacy does not decrease after age 70, the age
group at greatest risk for postherpetic neuralgia.
A 65 year old woman with aortic sclerosis is admitted with chest pain.
An infarct is ruled out by cardiac enzymes, but the patient has
recurrent symptoms when weaned off heparin. On hospitalization day 2,
she has right arm pain, absent brachial pulse on the right, and a cold
distal right arm. Her hematocrit is 34%, and platelets are 30,000/mm3.
Her partial thromboplastin time is 64 sec. Which of the following is the
most likely cause of this patient's absent brachial pulse?

a) Embolization from aortic sclerosis


b) Heparin-induced thrombocytopenia
c) Paradoxical embolus
d) Hypercoagulable state from immobilization
e) Vasospasm of the brachial artery
The correct answer is B

Explanation
Heparin-induced thrombocytopenia (HIT) is the result of platelet
aggregation caused by heparin-induced antibodies. It is seen in 1% to 5%
of patients on heparin. Therapy is discontinuation of the heparin and
use of another anticoagulant, such as lepirudin. When the platelet count
falls below 50,000/mm3 the heparin should be stopped. HIT can lead to
limb-threatening thromboses, as in this patient, and constitutes a
medical emergency. Arterial thrombosis is a manifestation of the HIT
syndrome.
An 18 year old woman previously in good health seeks help at an
emergency room for lightheadedness, headaches and nausea. She appears
anxious and is tremulous, sweating and breathing heavily. While waiting
to see a physician, she begins to complain of tingling around her mouth
and in her fingers. Which one of the following would be your first
management step for this patient?

a) Ask her to breathe into a paper bag


b) Order immediate intravenous infusion of a glucose solution
c) Order a urine drug screen
d) Give supplemental oxygen by mask
e) Administer 2 mg of sublingual lorazepam (Ativan)
The correct answer is B

Explanation
The symptoms of hypoglycemia rarely develop until the level of sugar in
the blood falls below 60 milligrams per deciliter of blood. Some people
develop symptoms at slightly higher levels, especially when blood sugar
levels fall quickly, and some do not develop symptoms until the sugar
levels in their blood are much lower.

The body first responds to a fall in the level of sugar in the blood by
releasing epinephrine from the adrenal glands. Epinephrine stimulates
the release of sugar from body stores but also causes symptoms similar
to those of an anxiety attack: sweating, nervousness, shaking,
faintness, palpitations, tingling sensations around the mouth and hunger.

More severe hypoglycemia reduces the sugar supply to the brain, causing
dizziness, fatigue, weakness, headaches, inability to concentrate,
confusion, inappropriate behavior that can be mistaken for drunkenness,
slurred speech, blurred vision, seizures, and coma. Severe and prolonged
hypoglycemia may permanently damage the brain. Symptoms can begin slowly
or suddenly, progressing from mild discomfort to severe confusion or
panic within minutes.
A patient of yours has a karyotype done shown below:
What will the patient most likely be?

a) Genotypically male
b) Phenotypically male
c) Phenotypically female
d) Mosaic
The correct answer is C

Explanation
This is the karytope of a patient with Turner syndrome. Turner syndrome
only occurs in females. A normal female will be 46XX and a normal male
is 46XY. Turner syndrome patients are 45XO.

Genotype is the internally coded, inheritable information carried by all


living organisms. This stored information is used as a "blueprint" or
set of instructions for building and maintaining a living creature.
Since a male has one Y chromosome, this patient is not genotypically
male but is genotypically female.

Phenotype is the outward, physical manifestation of the organism. These


are the physical parts, for example gender, hair color or anything that
is part of the observable structure, function or behavior of a living
organism. This patient has only one X chromosome and therefore is
phenotypically female.

Mosaicism is a condition in which tissues of genetically different types


occur in the same organism. For example in a person with mosaic Turner
Syndrome, some cells have the normal number of 46 chromosomes, but other
cells are missing one X or there are structural defects in the second X.
The mosaic karyotypes would be 45,X/46,XX or 45,X/46,XY.
A 55-year-old black male with osteoarthritis of the knees asks for
advice on improving the function of his knees and controlling arthritis
pain. Which one of the following would be appropriate advice?

a) Topical capsaicin (Zostrix) applied twice daily will improve both


pain and function
b) Glucosamine will improve both pain and function
c) A therapeutic exercise program will improve both pain and function
d) An intra-articular corticosteroid injection will provide at least 6
months of pain relief
e) NSAIDs will slow the progression of the disease
The correct answer is C

Explanation
A therapeutic exercise program will reduce both pain and disability in
patients with osteoarthritis of the knee (SOR A). There is no evidence
to support the use of capsaicin cream, but NSAIDs will reduce pain and
there are proven therapies that will improve function of the patient‟s
knee. While intra-articular corticosteroids are effective in relieving
pain in the short term (up to 4 weeks), there is no evidence for
long-term efficacy. There is not good evidence to support the use of
glucosamine for treating osteoarthritis of the knee. One systematic
review found it no more effective than placebo.
Which class of antibiotics is most likely to induce esophagitis in the
elderly?

a) Cephalosporins
b) Penicillins
c) Quinolones
d) Sulfonamides
e) Tetracyclines
The correct answer is E

Explanation
Decreased esophageal peristaltic clearance, which is common among older
persons, may be associated with pill retention. Esophageal injury may
occur as a result of prolonged contact of the caustic contents of the
medication with the esophageal mucosa. Tetracyclines, particularly
doxycycline, are the most common antibiotics associated with
esophagitis, but aspirin and all NSAIDs can also damage the esophagus.
Other offenders include potassium chloride, quinidine, iron, and
alendronate.
A 16 year old girl is brought to hospital by her frantic parents after a
bee sting. Vitals sings are BP 70/40, RR 30 and laboured, HR 140, T
37.5. Which of the following would *not* be an option in her management?

a) Epinephrine
b) Diphenhydramine
c) Methylprednisolone
d) Salbutomol
e) Atropine
The correct answer is E

Explanation
Anaphylactic reactions (anaphylaxis) are sudden, widespread, potentially
severe and life-threatening allergic reactions. If an anaphylactic
reaction occurs, an epinephrine injection should be given immediately.
People who have these reactions should always carry a self-injecting
syringe of epinephrine and antihistamine tablets for prompt treatment.
Atropine would not be appropriate, since it would raise her heart rate,
which is already tachycardic.
Which one of the following most accurately describes the murmur of
aortic regurgitation?
a) Systolic ejection murmur, loudest at the right upper sternal border
b) Diastolic decrescendo murmur, loudest at the lower left sternal border
c) Diastolic rumbling murmur, loudest at the left anterior axillary line
d) Holosystolic murmur, loudest at the left upper sternal border
The correct answer is B

Explanation
Aortic regurgitation (AR) is the result of incompetent aortic valve
leaflets or dilatation of the aortic root. The most common etiology of
valvular disease is rheumatic fever, but AR may also occur in patients
with a congenital bicuspid aortic valve. Patients may remain
asymptomatic for many years, until symptoms of heart failure develop due
to left ventricular hypertrophy. The murmur is a diastolic decrescendo,
“blowing” quality murmur, beginning right after the second heart sound.
It is best heard at the left lower sternal border with the patient
upright and leaning forward, placing the heart closest to the anterior chest
wall.
Which one of the following is the most common cause of blindness in the
elderly?

a) Diabetic retinopathy
b) Macular degeneration
c) Glaucoma
d) Atherosclerosis
e) Retinal detachment
The correct answer is B

Explanation
Macular degeneration leads the list of causes of blindness in persons
over the age of 55. As the name implies, there is eventual loss of
central, high-acute vision. Peripheral vision is spared. In the early
form, the maculae exhibit pigment by-products called drusen. The late
disease is further divided into two categories: the more common ?dry? or
atrophic macular degeneration and the exudates or ?wet? macular
degeneration. Although less common, the exudative type is more severe,
and various therapies, including laser photocoagulation of neovascular
vessels and photodynamic therapy, are effective in only a fraction of
affected patients.

The other ophthalmologic conditions listed are certainly more prevalent


in the elderly, but are less common causes of blindness than macular
degeneration.
What is the chance of genetic transmission of Duchene Muscular Dystrophy
(DMD) from parents to offspring, if the father is affected and the
mother is not?

a) Zero
b) 25%
c) 33%
d) 50%
e) 100%
The correct answer is A

Explanation
Duchenne muscular dystrophy is inherited in an X-linked recessive
pattern. The following table illustrates the outcomes for an affected
male with a normal female and an affected female with a normal male:

X X Xa Xa

Xa XaX XaX X XXa XXa


Y YX YX Y YXa YXa

In first scenario (concerning this question), the affected male mates


with a normal female. The female offspring will be carriers and the male
offspring will be normal. No offspring will be *affected*.

In the second scenario, a normal male mates with an affected female.


Resulting in male offspring that are affected and female offspring that
are carriers.
Guillain-Barré syndrome is not associated with which of the following?

a) Proximal muscle weakness


b) Areflexia
c) Infection with C. jejuni
d) Treatment with antibiotics
The correct answer is D

Explanation
Guillain-Barré syndrome is an acute, usually rapidly progressive
inflammatory polyneuropathy characterized by muscular weakness and mild
distal sensory loss. Cause is thought to be autoimmune.

In about 2?3 of patients, the syndrome begins 5 days to 3 weeks after an


infectious disorder, surgery, or vaccination. Infection is the trigger
in > 50% of patients; common pathogens include Campylobacter jejuni,
enteric viruses, herpesviruses (including cytomegalovirus and those
causing infectious mononucleosis), and Mycoplasma.

Flaccid weakness predominates in most patients; it is always more


prominent than sensory abnormalities and may be most prominent
proximally. Relatively symmetric weakness with paresthesias usually
begins in the legs and progresses to the arms, but it occasionally
begins in the arms or head. In 90% of patients, weakness is maximal at 3
weeks. Deep tendon reflexes are lost. Sphincters are usually spared.
Facial and oropharyngeal muscles are weak in > 50% of patients with
severe disease. Respiratory paralysis severe enough to require
endotracheal intubation and mechanical ventilation occurs in 5 to 10%.

Diagnosis is clinical. Treatment includes plasmapheresis, ?-globulin,


and, for severe cases, mechanical ventilation.
A 38-year-old male is admitted to the hospital because of weakness and
is found to have severe hypokalemia. His urinary electrolytes show
significant potassium wasting.

This combination of findings suggests the patient also has a significant


deficiency of

a) aldosterone
b) magnesium
c) sodium
d) hydrogen ions
e) calcium
The correct answer is B

Explanation
Approximately half of hospitalized patients in whom serum electrolytes
are ordered have unrecognized hypomagnesemia. Many of these patients
have hypokalemia and/or hypocalcemia. Hypomagnesemia also causes renal
potassium wasting, which is refractory to potassium replacement until
magnesium is replaced.
A man develops swelling in his knee. It is red and warm. An
arthrocentesis is performed and comes back showing rhomboid positively
bifringent crystals. What is your treatment?

a) Allopurinol
b) Probenacid
c) Colchicine
d) Indomethacin
The correct answer is D

Explanation
Pseudogout (CPPD - calcium pyrophosphate dihydrate crystal deposition
disease) is a disorder caused by deposits of calcium pyrophosphate
crystals in the cartilage and then in the fluid of the joints, leading
to intermittent attacks of painful joint inflammation.

Symptoms vary widely. Some people have attacks of painful joint


inflammation, usually in the knees, wrists, or other relatively large
joints.

Diagnosis is by taking fluid from an inflamed joint through a needle


(joint aspiration). Positively bifringent calcium pyrophosphate crystals
(rather than urate crystals) are found in the joint fluid.

Treatment is with nonsteroidal anti-inflammatory drugs (NSAIDs) such as


indomethacin.
You are asked to see a 60 year old man with a history of recurrent
depression who has failed to respond to several second-generation
antidepressants. You are considering a trial of a tricyclic
antidepressant. What is the most relevant investigation to perform prior
to initiating this treatment?

a) Complete blood count


b) Transaminases
c) Electrocardiogram
d) Chest x-ray
e) Creatinine
The correct answer is C

Explanation
When using a tricyclic antidepressant, it is important to consider that
the central nervous system and heart are the two main systems that are
affected in toxicity. Initial or mild symptoms include drowsiness, a dry
mouth, nausea, and vomiting. More severe complications, include
hypotension, cardiac rhythm disturbances, hallucinations, and seizures.
Electrocardiogram (ECG) abnormalities are frequent and a wide variety of
cardiac dysrhythmias can occur, the most common being sinus tachycardia
and intraventricular conduction delay (QRS prolongation). Seizures and
cardiac dysrhythmias are the most important life threatening complications.
A 30-year-old nurse was vaccinated more than 10 years ago for hepatitis
B. When attempting to donate blood recently, he was found to have mildly
elevated liver transaminases and now is concerned about the possibility
of viral hepatitis.

Which one of the following serologic results would be consistent with


acute hepatitis B virus infection?

a) Negative hepatitis B surface antigen (HBsAg)


b) Positive hepatitis B surface antibody (anti-HBs)
c) Positive IgM hepatitis B core antibody (IgM anti-HBc)
d) Positive hepatitis B e-antigen antibody (anti-HBe)
e) Positive IgG hepatitis B core antibody (IgG anti-HBc)
The correct answer is C

Explanation
HBV has at least 3 distinct antigen-antibody systems that can be tested:
HBsAg, hepatitis B core antigen (HBcAg), and HBeAg. HBV-DNA can also be
tested.
In a woman under 40 years of age, which one of the following breast
abnormalities would have the highest predictive value for malignancy?

a) Painful, moveable mass


b) Painless, moveable mass
c) Blood nipple discharge
d) Clear nipple discharge
e) Breast skin edema with dimpling
The correct answer is E

Explanation
Most breast cancers are discovered as a lump by the patient or during
routine physical examination or mammography. Less commonly, the
presenting symptom is breast pain or enlargement or a nondescript
thickening in the breast.

A common finding during physical examination is a dominant mass, a lump


distinctly different from the surrounding breast tissue. Diffuse
fibrotic changes in a quadrant of the breast, usually the upper outer
quadrant, are more characteristic of benign disorders; a slightly firmer
thickening in one breast but not the other may be a sign of cancer.

More advanced breast cancers are characterized by fixation of the lump


to the chest wall or to overlying skin, by satellite nodules or ulcers
in the skin, or by exaggeration of the usual skin markings resulting
from lymphedema (so-called peau d'orange).
A 45 year old man presents with swelling in the right scrotum. You
notice a tender mass above the testis. The skin of his scrotum is red
and inflamed. On rectal exam his prostate is enlarged but not tender. He
has pyuria (pus in his urine). What is the diagnosis‟

a) Epididymitis
b) Epididymitis and prostatitis
c) Gonococcal urethritis
d) Nephrolithiasis
The correct answer is A

Explanation
Epididymitis presents with scrotal pain and swelling. Prehn's sign is
positive (Prehn's sign, the physical lifting of the testicles relieves
the pain of epididymitis but not pain caused by testicular torsion).

Tests include U/A and Urine culture. The cause is usually gonorrhea or
chlamydia. Treatment is with antibiotics.
Which one of the following is most characteristic of noncardiogenic
pulmonary edema?

a) Paroxysmal nocturnal dyspnea


b) Poor left ventricular function
c) Brain natriuretic peptide levels >500 pg/mL
d) An S_3 gallop
e) A history of sepsis, trauma, or aspiration
The correct answer is E

Explanation
Increased membrane permeability in the lungs causes noncardiogenic
pulmonary edema, flooding the lungs with fluid from the vascular space.
Consequently, it is associated with damage to the lungs from trauma,
aspiration, or sepsis.

Paroxysmal nocturnal dyspnea, an S3 gallop, and poor left ventricular


function are associated with cardiac-induced pulmonary edema. Elevated
brain natriuretic peptide levels mean that the cardiac walls are
stretched and that the heart is likely the cause of the pulmonary edema.

An 80-year-old male presents to your office for preoperative clearance.


He denies any complaints other than some right knee pain resulting from
a longstanding injury.

His EKG reveals

a) Right bundle branch block


b) Left bundle branch block
c) Wolff-Parkinson-White syndrome type A
d) Right ventricular hypertrophy
e) Posterior myocardial infarction
The correct answer is A

Explanation
This EKG shows the classic pattern of right bundle branch block (RBBB).
There is a prolonged QRS, an rSR?, downsloping ST- and T-wave inversions
in V_1 and V_2 , and a wide, slurred S wave in lead I. Left bundle
branch block also causes a prolonged QRS, but very different additional
findings. Wolff-Parkinson-White (WPW) syndrome type A causes a prolonged
QRS but there is a characteristic initial slurring of the QRS (a ? wave)
not seen on this EKG. A shortened PR interval would also be likely.
Right ventricular hypertrophy will produce an R wave in V_1 , but there
will not be QRS widening. A true posterior myocardial infarction would
also produce an R wave in V_1 , as well as the ST segment depression
found in RBBB, but the T waves would be upright; in addition, the
patient would likely be quite ill.
A 45 year old man enters the hospital because of an episode of acute
flank pain and hematuria. Abdominal radiograph reveals a stone in the
right kidney. An intravenous urogram shows the stone to be
nonobstructing. Which type of kidney stone is most likely to have caused
this patient's condition?

a) Calcium oxalate
b) Uric acid
c) Xanthine
d) Struvite
e) Cystine
The correct answer is A

Explanation
About 80% of kidney stones are composed of calcium, and the remainder
are composed of various substances, including uric acid, cystine,
xanthine and struvite. Struvite stones, a mixture of magnesium,
ammonium, and phosphate, are also called infection stones, because they
form only in infected urine.

Most stones will show up on a plain abdominal x-ray. But cystine calculi
are relatively radiolucent on plain radiography, therefore will not be seen.

What is the diagnostic test of choice for the diagnosis of myasthenia


gravis‟

a) Tensilon test

b) EEG
c) MRI of brain
d) CSF analysis
The correct answer is A

Explanation
Myasthenia gravis is an autoimmune disorder of episodic muscle weakness
and easy fatigability caused by antibody- and cell-mediated destruction
of acetylcholine receptors. It is more common among young women and
older men but may occur at any age. Symptoms worsen with muscle activity
and lessen with rest.

Diagnosis is by IV edrophonium (tensilon) challenge, which briefly


lessens the weakness. Treatment includes anticholinesterase drugs,
immunosuppressants, corticosteroids, thymectomy, and plasmapheresis.
A 63 year old male with uncontrolled hypertension comes to see you with
complaints of weight gain and feeling tired. Lab results show low
hemoglobin and low ferritin levels. What is the possible reason for his
hypertension?

a) Hyposecretion of aldosterone
b) Normochromic anemia
c) Increased blood supply to renal arteries
d) Decreased blood supply to renal arteries
e) Decreased tissue peripheral resistance
The correct answer is D

Explanation
Answer: D ? Decreased blood supply to renal arteries is one of the
reasons for the hypertension. Decrease in the blood supply to renal
arteries stimulates the renin-angiotensin system. Renin is then
converted to angiotensin I and finally to angiotensin II in the presence
of angiotensin converting enzyme. Angiotensin II is a powerful
vasoconstrictor, which results in increase of blood pressure.
Angiotensin also stimulates the adrenal cortex for the release of
aldosterone. Aldosterone decreases the reabsorption of the sodium from
the blood and increases the retention of it in blood. Increased
retention of the sodium in blood would increase the blood volume and
further increase the blood pressure.

A. Hyposecretion of aldosterone would result in hypotension.


B. He has hypochromic anemia.
C. Increased blood supply to renal arteries does not stimulate the
renin-angiotensin system.
E. Decreased tissue peripheral resistance leads to hypotension.

A 46 year old female presents to your office complaining of wheezing for


the past 2 weeks. She denies a history of asthma or any major medical
illness. She is a non-smoker but drinks alcohol on the weekends. She
admits to having intermittent hoarseness of voice for the past few weeks.

Her vitals are within normal limits. Examination of the upper


respiratory tract reveals a red and inflamed larynx. Chest is clear to
auscultation and percussion.

Based on the history and physical exam, what is the most appropriate
initial treatment for this patient?

a) A trial of antibiotics
b) Omeprazole daily
c) Oral steroids
d) Sulbutamol + Ipratropium inhaler
e) Sulbutamol inhaler
The correct answer is B

Explanation When it is unclear whether the patient has nocturnal


asthma or gastroesophageal reflux disease, a trial of proton pump
inhibitors is both diagnostic and therapeutic.

There is no evidence of bacterial infectious process; thus empiric


antibiotics are inappropriate.

Oral steroids would worsen the GERD and even if this were nocturnal
asthma, they are never appropriate as an initial approach.

Sulbutamol and/or Ipratropium would be appropriate for asthma or COPD


control.

Which one of the following reverses the effect of warfarin (Coumadin)?

a) Vitamin A
b) Vitamin C
c) Vitamin D
d) Vitamin E
e) Vitamin K
The correct answer is E

Explanation
In patients whose INR becomes elevated while taking warfarin, the INR
can be lowered either by withholding warfarin or by oral or parenteral
administration of vitamin K.
A patient presents with pulmonary edema and has a blood pressure of
180/95. What is the most appropriate initial treatment?

a) Furosemide
b) Metoprolol
c) Morphine
d) Enalapril
The correct answer is A

Explanation
Sometimes increased pressure in the blood vessels in the lungs forces
fluid into the air sacs, preventing them from absorbing oxygen, a
condition called pulmonary edema.

The treatment of pulmonary edema is as follows:

Preload reducers. Diuretics, such as furosemide (Lasix) are used to


treat pulmonary edema. These medications dilate the veins in the lungs
and elsewhere in the body, which decreases fluid pressure going into the
heart and lungs.

Morphine, a narcotic, is a mainstay in treating cardiac pulmonary edema,


may be used to relieve shortness of breath and associated anxiety.

Afterload reducers. These drugs dilate the peripheral vessels and take a
pressure load off the left ventricle. Some examples of afterload reducer
medications include nitroprusside and enalapril.
Black cohosh (Cimicifuga racemosa) is an herbal remedy used by patients for

a) Dementia
b) Menopausal symptoms
c) Nausea and vomiting
d) Benign prostatic hypertrophy
e) Osteoarthritis
The correct answer is B

Explanation
The dried up root of black cohosh is approved by the German Commission
for painful menstruation and the symptoms of menopause.

A 68 year old man with a history of urolithiasis, suffered from a


seizure attack which involved loss of consciousness with tonic and
clonic muscular contractions. His tongue fell back into his throat and
he choked.
Which of the following medications is contraindicated in this patient's
treatment?

a) Levetiracetam
b) Phenytoin
c) Topiramate
d) Carbamazepine
e) Valproic acid
The correct answer is C

Explanation
This patient suffers from granmal seizure (tonic-clonic seizures), which
is characterized by sudden loss of the consciousness followed by tonic
and clonic muscular contractions.

Patients with a history of kidney stones or those taking high-dose


vitamin C or calcium supplementation should not take Topiramate because
of the increased risk of kidney stones.

Valproic acid is considered the agent of first choice for the treatment
of grandmal (tonic-clonic) seizures.
Phenytoin and carbamazepine are reasonable second options among the
older group of medications, but the newer medications tend to work
equally well if not better and have better side effect profiles,
especially long-term side effect.
Levetiracetam is indicated for primary generalized tonic-clonic seizures
in adults and children aged 6 years or older, as well as for use in
juvenile myoclonic epilepsy and for partial seizures.
A 70 year old man presents to the ED with a severe shortness of breath
and diaphoresis.

His past medical history is significant for Diabetes and two previous
MIs. He is taking metformin, pioglitazone, aspirin, losartan and
rosuvastatin. His vitals are significant for a pulse rate of 110/min and
a RR of 30/min. Physical exam shows JVD, lower limb pitting edema and
bilateral basilar lung crackles.

Which of the following medications is the most appropriate at this time?

a) Beta blockers
b) Digoxin
c) Lidocaine
d) Mannitol
e) Morphine
The correct answer is E

Explanation
The best initial therapy for Acute cardiogenic pulmonary edema includes
*L*oop diuretics, *M*orphine, *N*itrates and *O*xygen ?*LMNO*?. Morphine
promotes venodilation thus decreases the preload; moreover, morphine
alleviates the severe anxiety of acute pulmonary edema patients.

Beta blockers are excellent drugs for congestive heart failure and
decrease mortality; however they should be avoided in the acute
exacerbation because of its negative inotropism.
Osmotic diuretics such as mannitol are contraindicated because they
increase the blood volume thus the afterload.

Digoxin would be appropriate for long term management.

In the absence of a ventricular tachycardia or other arrhythmia,


lidocaine is not recommended.
A 60-year-old male presents to the urgent care center with a fever and a
productive cough. He has a 40 pack/year history of cigarette smoking. In
addition to lobar pneumonia on a chest radiograph, there is an
incidental finding of bilateral pleural plaques. Which one of the
following is the most likely cause of this finding?

a) Coal dust
b) Silicon dust
c) Asbestos
d) Vinyl chloride
e) Radon
The correct answer is C

Explanation
Development of pleural plaques is the most common pathologic pulmonary
response to asbestos inhalation. Over time, collagen is deposited in the
pleura and may calcify. Most plaques are asymptomatic, and there is no
evidence that plaques transform into malignant lesions. Plaques occur in
approximately 50% of persons with heavy and prolonged exposure to
asbestos and, therefore, are a marker of asbestos exposure. This should
alert the physician to follow the patient for development of more
serious asbestos-related diseases (e.g., lung cancer and mesothelioma).
Although the other substances listed are associated with pulmonary
diseases (coal dust and silicon dust with pneumoconiosis, and vinyl
chloride and radon with lung cancer), none is associated with pleural
plaques as found in this patient.
A 35-year-old male complains of 2 months of right shoulder pain. He does
not recall an injury, but says it is painful to lie on his right side or
to work with his right hand above his head. On examination, the shoulder
appears normal and there is no pain with external rotation of the
shoulder, bringing the arm across the body (scarf test), or attempted
external and internal rotation of the shoulder against resistance.
Lowering the arm from full abduction (painful arc), attempted abduction
above 45° against resistance, and elevating the internally rotated arm
above 90° against resistance are all painful.

The most likely diagnosis is

a) subdeltoid bursitis
b) adhesive capsulitis
c) impingement syndrome
d) glenohumeral osteoarthritis
e) acromioclavicular osteoarthritis
The correct answer is C

Explanation
The combination of a painful arc and pain on use of the supraspinatus
muscle indicates impingement syndrome, which is due to irritation of the
rotator cuff under the coracoacromial arch. It is by far the most common
cause of shoulder pain seen by family physicians. Subdeltoid bursitis is
a much more acute problem, and impairs shoulder mobility in all
directions. Adhesive capsulitis produces loss of external rotation.
Glenohumeral arthritis produces pain with external rotation, and
variable amounts of impaired mobility, depending on progression of the
problem over time. Acromioclavicular joint arthritis produces a positive
scarf sign, and often a visible bump over the joint, since it lies so
close to the skin surface.
Adult ankylosing spondylitis

a) commonly begins after age 45


b) is more common in women than in men
c) is most commonly initially manifested in knees and hands
d) typically causes early morning pain and stiffness that improves with activity
The correct answer is D

Explanation
Symptoms of ankylosing spondylitis usually begin in late adolescence or
early adulthood. Only about 5% of cases begin after age 40. Back
complaints are the initial complaint in 75% of patients with adult-onset
ankylosing spondylitis. This disease is three to five times more common
in men than in women. Backache and stiffness tend to be present after
periods of inactivity and tend to improve with activity.

An asymptomatic 32 year old African-Canadian female presents with a


hemoglobin concentration of 10.2 g/dL and a mean corpuscular volume
(MCV) of 68 µm^3 . Iron studies are normal.

In which one of the following would a normal pattern on hemoglobin


electrophoresis be expected?

a) Alpha-Thalassemia minor (trait)


b) ß-Thalassemia minor (trait)
c) ß-Thalassemia major
d) Hemoglobin SC disease
e) Hemoglobin H disease
The correct answer is A

Explanation
Thalassemia is an inherited hematologic disorder that affects the
balanced production of globin chains of the hemoglobin molecule. Normal
adult hemoglobin contains two alpha chains and two ß chains that are
produced in equal amounts. Thalassemia minor results in a partial
reduction in the production of either the alpha- or ß-chains. Patients
with this problem usually have hypochromic, microcytic red blood cells
with a normal or elevated RBC count and mildly reduced hemoglobin.

In ß-thalassemia minor, hemoglobin electrophoresis is abnormal and shows


a increased concentration of HbA2 and an decrease in HbA . In
alpha-thalassemia minor alpha-chain production is reduced but there is
no increase in other abnormal hemoglobins and electrophoresis is normal.
If the production of alpha-chains is severely reduced it results in the
production of hemoglobins with all ß-chains, termed Hb H, or all
gamma-chains, called Hb Barts.

ß-Thalassemia major is a severe anemic syndrome without the production


of any HbA. An abnormal hemoglobin consisting of four alpha-chains is
found on electrophoresis. Hemoglobin SC is an abnormal hemoglobin with
amino acid substitution in the globin chain that is abnormal on
electrophoresis.
In addition to calcium and vitamin D supplementation, patients who are
beginning long-term treatment with prednisone (> 3 months at a dosage >
5 mg/day) should also receive

a) bisphosphonate therapy
b) calcitonin
c) estrogen replacement therapy
d) recombinant human parathyroid hormone
e) raloxifene (Evista)
The correct answer is A

Explanation
The current guidelines recommend that patients who are beginning
long-term treatment with prednisone ( 3 months at a dosage 5 mg/day), or
an equivalent, receive bisphosphonate therapy in addition to calcium and
vitamin D supplementation, regardless of their T score. The other
treatments are not recommended for prevention of glucocorticoid-induced
osteoporosis.
A 58 year old woman is scheduled for an abdominal operation. She has
hypothyroidism that is controlled with thyroid replacement medication
and will be unable to eat or drink for 4 days following the procedure.
She is concerned about receiving her thyroid medication. She should be
advised of which of the following?

a) Although she will have symptoms of hypothyroidism, she will not be


given the medication
b) Although she will not receive the medication, she will have no
adverse effects
c) The medication will be administered daily while the nasogastric tube
is clamped
d) The medication will be discontinued temporarily before the operation
e) She will receive the medication intravenously
The correct answer is B

Explanation
Thyroxine is the hormone of choice for replacement therapy. It has a
half-life of seven days, and any alteration in dose is not reflected for
four to six weeks. Therefore, it is very unlikely that she will develop
signs and symptoms of hypothyroidism. She will not be given the
medication either through the nasogastric tube or intravenously, nor
does she require a preoperative loading dose.

Hemoglobin A1C assays are inaccurate in patients with

a) Secondary hypertension
b) Morbid obesity
c) Cor pulmonale
d) Sickle cell disease
e) Hypothyroidism
The correct answer is D

Explanation
The glycosylated hemoglobin assay is rendered inaccurate by conditions
affecting red blood cell survival, such as sickle cell disease or the
presence of hemoglobin C.
A healthy 40 year old female presents for her annual gynecologic
examination. She tells you that she also needs a tuberculin screening
test for her anticipated volunteer work at the local hospital. She has
had no significant illness or exposures and has been your patient for
nearly 20 years. You administer a PPD test which shows 10 mm of
induration on the second day. The most appropriate next step in her
evaluation would be

a) A repeat PPD in 2 weeks


b) A chest radiograph (two views)
c) Screening liver function tests
d) Isoniazid (INH) for 9 months for treatment of her latent tuberculosis
The correct answer is A

Explanation
The Thoracic Society and the Centers for Disease Control and Prevention
(CDC) advocated a shift in focus from screening the general population
to testing only patients at increased risk for developing tuberculosis.
In some persons PPD reactivity wanes with time but can be recalled by a
second skin test administered 1 week or more after the first (i.e.,
two-step testing). For persons undergoing PPD skin testing, such as
health-care workers, initial two-step testing may preclude
misclassification of persons with boosted reactions as PPD converters.
In those at low risk, such as this patient, a tuberculin skin test is
now considered positive only if induration is at least 15 mm. Thus, this
hospital volunteer would pose little risk to the hospital population
since her 10-mm reaction falls within the guidelines of a negative test.
She does not require diagnostic evaluation at this time, and isoniazid
therapy is not indicated.
A 38 year old man returns to the office for follow-up of an abnormal
liver chemistry profile ordered 3 weeks ago during a routine
examination. At that time, his physical examination was normal, but he
had a serum AST concentration of 72 U/L. His serum bilirubin and
alkaline phosphatase concentrations were normal. History includes an
episode of hepatitis A at age 22 years. He has no history of
transfusions or intravenous drug use. He drinks two to three beers
daily. Today's follow-up test results show:

Anti-HAV Positive
Anti-HBs Negative
HBsAg Positive
HBcAg Positive

Which of the following is the most appropriate next step?

a) Begin interferon-alfa therapy


b) Begin corticosteroid therapy
c) Have him cease all alcohol consumption and retest him in 2 months
d) Order hepatitis B virus DNA polymerase study
e) Schedule liver biopsy
The correct answer is C

Explanation
The indication for extensive testing in this patient was the abnormal
liver enzymes. The results of this testing reveals a positive HAV
antibody which is indicative of prior exposure to the hepatitis A virus.
Hepatitis B virus, a DNA virus, is transmitted through parenteral or
mucosal exposure to infectious blood or body fluids and can lead to
chronic as well as acute infection, cirrhosis, and primary
hepatocellular carcinoma. Only approximately 10% of all acute hepatitis
B infections progress to chronic infection in adults. The presence of a
positive HBsAg (surface antigen to hepatitis B) and HBcAg (core antigen)
indicates acute, active hepatitis B infection. After infection and 1 to
6 weeks before symptoms occur HBsAg appears. The relevant question
concerns the actual effect on liver enzymes. For this reasons, the
patient should refrain from alcohol consumption, which also elevates
liver enzymes.
You are treating an 18 year old white male college freshman for allergic
rhinitis. It is September and he tells you that he has severe symptoms
every autumn which impair his academic performance. He has a strongly
positive family history of atopic dermatitis. Which one of the following
medications is considered optimal treatment for this condition?

a) Intranasal glucocorticoids
b) Intranasal cromolyn sodium
c) Intranasal decongestants
d) Intranasal antihistamine
The correct answer is A

Explanation
Topical intranasal glucocorticoids are currently believed to be the most
efficacious medications for the treatment of allergic rhinitis. They are
far superior to oral preparations in terms of safety. Cromolyn sodium is
also an effective topical agent for allergic rhinitis; however, it is
more effective if started prior to the season of peak symptoms. Because
of the high risk of rhinitis medicamentosa with chronic use of topical
decongestants, these agents have limited usefulness in the treatment of
allergic rhinitis.

Some of the newer oral antihistamines have been found to be comparable


in efficacy to intranasal steroids, but their use slightly increases the
incidence of adverse effects and drug interactions. They are not as
useful for congestion as for sneezing, pruritis, and rhinorrhea. Newer
agents are relatively free of sedation. Overall, they are not as
effective as topical glucocorticoids. Azelastine, an intranasal
antihistamine, is effective in controlling symptoms but can cause
somnolence and a bitter taste.
The most common cause of monocular vision loss in children and young
adults is

a) amblyopia
b) congenital cataract
c) type 1 diabetes mellitus
d) trauma
The correct answer is A

Explanation
Amblyopia due to strabismus is the leading cause of monocular vision
loss in children and young adults, and is a major health problem. It is
crucial to make the diagnosis early, so screening during infancy and
childhood is necessary. Early screening results in better outcomes.
However, studies suggest that only 20% of school-age children are
routinely screened.

Congenital cataracts, ptosis, and corneal lesions are less common causes
of amblyopia, which act by deprivation of visual information. Neither
type of diabetes mellitus is a major cause of monocular vision loss.
Trauma is much less common than amblyopia.
A 48 year old woman receiving chemotherapy for widespread metastatic
breast cancer is admitted to hospital with increasing drowsiness,
confusion and constipation. She is found to be hypercalcemic. Which one
of the following treatment strategies represents the best initial
management of her hypercalcemia?

a) Change the chemotherapy regimen


b) Administer a bisphosphonate drug intravenously
c) Give normal saline solution and a diuretic together
d) Give normal saline solution
e) Give dextrose 5% in water solution
The correct answer is D

Explanation
Hypercalcemia is total serum Ca > 12 mg/dL (3 mmol/L), or ionized Ca > 6
mg/dL (1.5 mmol/L). The most common cause is iatrogenic. GI signs may
occur (anorexia, vomiting, constipation) and sometimes lethargy or
seizures.
*Treatment*: give IV fluids. Then, once the patient is well hydrated,
give furosemide (i.e. a loop diuretic) to cause calcium diuresis.
Thiazides are contraindicated because they increase serum calcium
levels. Other treatments include phosphorus administration (use oral
phosphorus; inravenous administration can be dangerous), calcitonin,
diphosphonates, plicamycin, or prednisone. Correction of the underlying
cause of hypercalcemia is the ultimate goal.
Which one of the following is characteristic of
fibromyositis/fibromyalgia syndrome?

a) Onset after age 50


b) Sleep disturbance
c) Proximal muscle weakness
d) An elevated erythrocyte sedimentation rate
e) A positive muscle biopsy
The correct answer is B

Explanation
The fibromyositis/fibromyalgia syndrome refers to musculoskeletal pain
with the broad spectrum of nonarticular rheumatism. It occurs
predominantly in women of childbearing age. The pain causes sleep
disturbances, but there is a lack of objective findings compared to the
plethora of symptoms. There are no characteristic laboratory
abnormalities; the erythrocyte sedimentation rate is usually normal.
Muscle biopsies are nonspecific.
Which one of the following statements is true regarding the use of
exenatide (Byetta) in the treatment of diabetes‟

a) It may be used for either type 1 or type 2 diabetes


b) It may be used as monotherapy for type 2 diabetes, in addition to
diet and exercise
c) Hypoglycemia is a common side effect
d) It may help with weight reduction
The correct answer is D

Explanation
Exenatide is used in type 2 diabetes as a supplement to oral drugs. It
rarely causes hypoglycemia, except when used with sulfonylureas. It
frequently causes weight loss.
A man presents with a thickened lip with vesicles. See picture:

What is the most likely diagnosis‟

a) Urticaria
b) Angioedema
c) Herpes simplex virus
d) Impetigo
The correct answer is C

Explanation
Herpes simplex viruses (human herpes viruses 1 and 2) commonly cause
recurrent infection affecting the skin, mouth, lips, eyes, and genitals.
Common severe infections include encephalitis, meningitis, neonatal
herpes, and, in immunocompromised patients, disseminated infection.
Mucocutaneous infections cause clusters of small painful vesicles on an
erythematous base.

Diagnosis is clinical; laboratory confirmation by culture, PCR, direct


immunofluorescence, or serology can be performed. Treatment is
symptomatic; antiviral therapy with acyclovir, valacyclovir, or
famciclovir is helpful for severe infections and, if begun early, in
recurrent or primary infections.
Patients with symptomatic congestive heart failure associated with a
reduced systolic ejection fraction or left ventricular remodeling should
be initially treated with which one of the following agents‟

a) An ACE inhibitor
b) Hydralazine (Apresoline)
c) Warfarin (Coumadin)
d) Amiodarone (Cordarone)
e) Verapamil (Calan, Isoptin)
The correct answer is A

Explanation
It has been shown that congestive heart failure (CHF) patients treated
with ACE inhibitors survive longer, and all such patients should take
these agents if tolerated. Warfarin and /or antiarrhythmic drugs should
be given only to selected CHF patients. Verapamil may adversely affect
cardiac function and should be avoided in patients with CHF. Hydralazine
can be used, but because of its side effects profile would be a
second-line agent.

A 23 year old woman reports that she has been troubled by episodes
during which she feels apprehensive and which usually occur in the
morning. Her heart rate increases. She sweats excessively. Agitation and
restlessness are prominent. Which one of the following laboratory tests
should be ordered?

a) Thyroid function tests


b) Toxicologic screen
c) Measurement of serum sodium level
d) Measurement of serum ALT level
e) Measurement of serum ammonia level
The correct answer is A

Explanation
Hyperthyroidism is characterized by hypermetabolism and elevated serum
levels of free thyroid hormones. Many common symptoms and signs of
hyperthyroidism are similar to those of adrenergic excess, such as
nervousness, palpitations, hyperactivity, increased sweating, heat
hypersensitivity, fatigue, increased appetite, weight loss, insomnia,
weakness, and frequent bowel movements (occasionally diarrhea).
Hypomenorrhea may be present. Signs may include warm, moist skin;
tremor; tachycardia; widened pulse pressure; atrial fibrillation; and
palpitations.

Diagnosis is based on history, physical examination, and thyroid


function tests. Serum TSH is the best test, because TSH is suppressed in
hyperthyroid patients except when the etiology is a TSH-secreting
pituitary adenoma or pituitary resistance to thyroid hormone. Free T4 is
increased.
A 56-year-old white male presents with a 2-week history of intermittent
pain in his left leg. The pain usually occurs while he is walking and is
primarily in the calf muscle or Achilles region. Sometimes he will
awaken at night with ?cramps‟ in the affected leg. He has no known risk
factors for atherosclerosis. Which one of the following would be the
best initial test for peripheral vascular occlusive disease?

a) Ankle-brachial index
b) Arterial Doppler ultrasonography
c) Arteriography
d) Magnetic resonance angiography (MRA)
e) Venous ultrasonography
The correct answer is A

Explanation
The ankle-brachial index (ABI) is an inexpensive, sensitive screening
tool and is the most appropriate first test for peripheral vascular
occlusive disease (PVOD) in this patient. The ABI is the ratio of
systolic blood pressure measured in the ankle to systolic pressure using
the standard brachial measurement. A ratio of 0.9?1.2 is considered
normal. Severe disease is defined as a ratio <0.50.

More invasive and expensive testing using Doppler ultrasonography,


arteriography, or magnetic resonance angiography may be useful if the
ABI suggests an abnormality. Venous ultrasonography would not detect
PVOD, but it could rule out deep venous thrombosis, which is another
common etiology for calf pain.
A 24 year old male presents with a painless lump in the right testis
that was recently discovered by his sexual partner. The most useful
initial diagnostic procedure is

a) Ultrasonography of the scrotum


b) A trans-scrotal biopsy of the lesion performed under local
anesthesia in your office
c) A therapeutic trial of a quinolone antibiotic for 3 weeks to shrink
the lesion
d) A Beta-hCG level
e) Reassurance that benign lesions will generally regress, and
reexamination in 4-6 weeks
The correct answer is A

Explanation
Ultrasonography of the scrotum will distinguish intra- from
extratesticular activity nearly 100% of the time. A trans-scrotal biopsy
is absolutely contraindicated, since it could lead to inguinal node
metastatic spread. Antibiotic trials are not indicated for
intratesticular masses. Beta-hCG is produced by only a small number of
testicular tumors. Watchful waiting is not appropriate for testicular
masses, which are usually malignant. Doubling time of testicular cancers
is thought to be 10-30 days.
Which one of the following is true regarding the treatment of acute gout
attacks‟

a) If a uric-acid lowering drug is not already being used, one should


be started immediately
b) Intravenous colchicines is the treatment of choice for patients with
a history of peptic ulcer disease
c) Generic NSAIDs are rapidly effective in most patients
d) Untreated hyperuricemia is a contraindication to the use of COX-2
inhibitors
The correct answer is C

Explanation
The preferred and most cost-effective treatment of an acute gout attack
is the use of a generic NSAID such as ibuprofen or indomethacin.
Colchicine, particularly the intravenous form, has the potential for
serious side effects, including bone marrow suppression. Uric
acid-lowering drugs do not have a role in the treatment of acute attacks.

The most appropriate and commonly used treatment for acute pericarditis is

a) Acetaminophen
b) Dipyridamole (Persantine)
c) Ibuprofen
d) Prednisone
e) Erythromycin
The correct answer is C

Explanation
Appropriate therapy for acute pericarditis includes NSAIDs, mainly
ibuprofen, for 1-3 weeks. Other NSAIDs could be used, but
corticosteroids should be avoided since they may cause a stubborn
corticosteroid-dependent syndrome. A combination of ibuprofen and
colchicines may be the ideal therapy.
All of the following symptoms favor the diagnosis of achalasia, except

a) Dysphagia for solids


b) Dysphagia for liquids
c) Painful swallowing
d) Regurgitation
e) Nocturnal regurgitation of undigested food
The correct answer is C

Explanation
Achalasia is a neurogenic esophageal motility disorder characterized by
impaired esophageal peristalsis, a lack of lower esophageal sphincter
relaxation during swallowing, and an elevation of lower esophageal
sphincter resting pressure. Symptoms are slowly progressive dysphagia,
usually to both liquids and solids, and regurgitation of undigested
food. Evaluation typically includes barium swallow, endoscopy, and
sometimes manometry. Treatments include dilation, chemical denervation,
and surgical myotomy.

Odynophagia (pain during swallowing) is usually not associated with


achalasia.
A 54-year-old white male presents with drooping of his right eyelid for
3 weeks. On examination, he has ptosis of the right upper lid, miosis of
the right pupil, and decreased sweating on the right side of his face.
Extraocular muscle movements are intact. In addition to a complete
history and physical examination, which one of the following would be
most appropriate at this point?

a) A chest radiograph
b) MRI of the brain and orbits
c) 131 Iodine thyroid scanning
d) A fasting blood glucose level
e) An acetylcholine receptor antibody level
The correct answer is A

Explanation
The clinical triad of Horner‟s syndrome?ipsilateral ptosis, miosis, and
decreased facial sweating‟suggests decreased sympathetic innervation due
to involvement of the stellate ganglion, a complication of Pancoast‟s
superior sulcus tumors of the lung. Radiographs or MRI of the pulmonary
apices and paracervical area is indicated.

Horner‟s syndrome may accompany intracranial pathology, such as the


lateral medullary syndrome (Wallenberg‟s syndrome), but is associated
with multiple other neurologic symptoms, so MRI of the brain is not
indicated at this point.
The acetylcholine receptor antibody level is a test for myasthenia
gravis, which can also present with ptosis, but not with full-blown
Horner‟s syndrome. Diabetes mellitus and thyroid disease do not commonly
present with Horner‟s syndrome.

A woman presents with chief complaint of headaches that started before


her menstrual cycle and end with the cycle. What is the diagnosis‟

a) Migraine headache
b) Premenstrual headache
c) Tension headache
d) Cluster headache
e) Sinus headache
The correct answer is B

Explanation This is a classic and straight forward presentation of a


premenstrual headache.

The other types of headaces are outlined in the graphic above. Common
treatments for sinus, cluster, tension and migraine headaches are
antibiotics, oxygen, NSAIDS and sumatriptan respectively.
Which of the following statements about estrogen therapy in
postmenopausal women is correct?

a) It is a major risk factor for breast cancer


b) It decreases hepatic triglyceride production
c) It produces hypertension
d) It protects against vertebral compression fractures
e) It directly stimulates the proliferation of vascular endothelium
The correct answer is D

Explanation
Postmenopausal osteoporosis results from increased osteoclast activity
and affects primarily trabecular bone. It occurs between ages 51 and 75
and is 6 times more common in women than in men. Estrogen loss may
elevate levels of cytokines, which are thought to increase recruitment
and activity of osteoclasts in trabecular bone, resulting in increased
bone resorption. Late menarche, early menopause, and nulliparity
increase the risk. Although calcitonin levels are decreased in women
compared with men, calcitonin deficiency does not appear to be important.

Estrogen can preserve bone density and prevent fractures. Most effective
if started within 4 to 6 years of menopause, estrogen may slow bone loss
and possibly reduce fractures even when started much later.
A 25 year old white male exhibits a syndrome of fever, headache,
myalgia, and aching joints 3 days after a summer camping trip. He
subsequently develops a generalized maculopapular eruption that
initially involves the palms and soles and then becomes hemorrhagic.
Splenomegaly is noted. A protracted 3-week course of the illness occurs
in spite of antibiotic therapy and is followed by gradual resolution of
the illness. Which one of the following is the most likely diagnosis‟

a) Lyme arthritis
b) Systemic lupus erythematosus in exacerbation
c) Subacute immune complex disease
d) Gonococcemia
e) Rocky Mountain spotted fever
The correct answer is E

Explanation
Fever, headache, and myalgia must be recognized as the classic triad of
Rocky Mountain spotted fever in all patients who might have been bitten
by a tick, as commonly occurs in campers. This patient exhibitis a
classic picture of the disease, which often has a virulent 3-week course
even when treated with tetracycline or chloramphenicol. A 10% mortality
rate has been noted for this illness. Lyme disease is also the result of
tick transmission, but is otherwise unlike Rocky Mountain spotted fever;
gonococcemia is associated with nomoarticualr pain and a popular rash;
systemic lupus erythematosus and immune complex disease do not produce
the clinical picture seen in this patient.
A 30 year old Eric presents to the ER complaining of palpitations that
started about an hour ago.

The patient is concerned because ?nothing like this has ever happened?
to him before. He denies any past medical problems. He admits that, due
to stress at work, he smoked too much and drank five cups of coffee a
few hours ago.

His vital signs are: BP is 125/80, temp is 37C, pulse is 160/min and
respiratory rate is 22/min. a STAT EKG shows the tracing below (Lead II):
You start carotid massage immediately; however the heart tracing is
still the same. What would you do next?

a) Adenosine
b) Asynchronous cardioversion
c) Magnesium sulfate
d) Propranolol
e) Synchronized cardioversion
The correct answer is A

Explanation
The patient is having supraventricular tachycardia ? this EKG shows:
narrow and regular QRS complexes with absent P wave. The patient is
clinically stable; thus, the goal is to convert him back to sinus rhythm
through a brief episode of AV block. Adenosine is the drug of choice for
short-term termination of SVT.

Synchronized cardioversion would be appropriate if the patient was unstable.

Asynchronous cardioversion is the treatment of choice for ventricular


fibrillation.

Although propranolol is effective, it is slow in action and much less


effective than adenosine in the treatment of SVT.

Magnesium sulfate is the drug of choice for Torsade de point.

Don?t be afraid of EKGs on the exam! There are a few EKGs that you
should memorize, and that‟s it! Look at A fib, A flutter, SVT, V tach, V
fib, AV block, Torsade de point and WPW EKGs.
Which one of the following is associated with testosterone
supplementation in men with hypogonadism?

a) Muscle wasting
b) Polycythemia
c) Osteoporosis
d) An increased risk of benign prostatic hypertrophy
The correct answer is B

Explanation
Testosterone increases hematocrit and can cause polycythemia. In
patients receiving testosterone supplementation, hematocrit should be
monitored every 6 months for the first 18 months, then annually.
Testosterone should be discontinued if there is more than a 50% rise in
hematocrit. Testosterone also causes an increase in lean body mass, and
may increase bone density.
A 30 year old patient with asthma complains of daily wheezing and
occasional waking at night with cough and chest tightness for three
weeks. His usual medication is salbutamol two puffs tid-qid. The next
step in management is

a) Add long-term theophylline


b) Increase salbutamol to two puffs q4h
c) Add ipratropium bromide two puffs qid
d) Add beclomethasone two puffs qid
e) Discontinue salbutamol and begin prednisone 50 mg and taper over 2 weeks
The correct answer is D
Explanation
Stage one of asthma is when a patient has symptoms 2 or less times per
week. For these patients the use of a short acting beta agonist (eg
albuterol, salbutamol) on an as needed basis is recommended.

Stage two of asthma is when a patient has symptoms greater than twice a
week. At this point you must add as inhaled steroid (eg fluticasone,
beclomethasone) to the patients treatment. Some physicians will also add
a nighttime dose of singulair.
A 75 year old Asian female comes to your office with a 6 week history of
the gradual onset of localized low back pain. The pain is increased with
walking or standing and relieved by sitting and lying. Pushing a
shopping cart dramatically relieves the pain. Based on the history,
which one of the following is the most likely diagnosis‟

a) Spinal cord tumor


b) Epidural abscess
c) Osteoporotic vertebral fracture
d) Spinal stenosis
e) Sciatica
The correct answer is D

Explanation
Spinal stenosis pain is often referred to as pseudoclaudication because
the pain is worsened by walking. Any movement that flexes the spine,
such as pushing a shopping cart, relieves the pain. Pain from tumor or
infection is persistent. Osteoporotic vertebral fractures have a sudden
onset. Sciatica does not have the typical findings of spinal stenosis.
Oral contraceptive pills have been shown to protect from which of the
following cancers‟

a) Cervical
b) Breast
c) Uterine
d) Colorectal
The correct answer is C

Explanation
It has been observed by several researchers that oral contraceptive
pills (OCP‟s) use decreases the risk of uterine cancer. Oral
contraceptives thin the lining of the uterus, resulting in a decreased
monthly menstrual flow for women, as well as a decreased rate of mitosis
(division) for a woman's endometrial cells. This results in an overall
decreased risk of uterine cancer.
A 60 year old man has an ulcerating carcinoma of the lateral side of the
tongue. Which one of the following features has the most important
effect upon his management?

a) He smokes 30 cigarettes daily


b) He has a hard cervical lymph node 1.5 cm in diameter
c) The tumor measures 2 cm in diameter
d) The tumor is a squamous cell carcinoma
e) He has severe dental caries
The correct answer is B
Explanation
Oral squamous cell carcinoma affects about 30,000 Americans each year.
Ninety percent are smokers. Alcohol is also a risk factor. Early,
curable lesions are rarely symptomatic; thus, preventing fatal disease
requires early detection by screening. Treatment is with surgery,
radiation, or both. The overall 5-yr survival rate is 52%.

If carcinoma of the tongue is localized (no lymph node involvement),


5-yr survival is about 50%. For localized carcinoma of the floor of the
mouth, 5-yr survival is 65%. With lymph node metastasis, the 5-yr
survival is 20%.

For tongue lesions, radiation therapy is often the treatment of choice


because surgery is extensive, disfiguring, and associated with poor
quality of life. In addition, patients with cancer of the floor of the
mouth are often medically compromised and are not good candidates for
surgery.
A 35 year old woman comes to the physician because of palpitations,
restlessness, sweating, weight loss, and a tremor for the past 6 weeks.
Examination shows a fine tremor, lid lag and stare, and pretibial
myxedema. The thyroid gland is diffusely enlarged, asymmetric, and
lobular. A bruit is present over the gland. Laboratory studies show an
undetectable level of thyroid-stimulating hormone, an increased level of
thyroid hormones, and an increased radioactive iodine uptake. The
diagnosis of Graves' disease is made and the treatment options are
discussed. The patient selects radioactive iodine therapy. This patient
is at greatest risk for which of the following conditions‟

a) Granulocytopenia
b) Hypothyroidism
c) Recurrent laryngeal nerve damage
d) Thyroid carcinoma
The correct answer is B

Explanation
Hypothyroidism is the main complication of radioactive iodine therapy,
affecting up to 70% of patients in 10 years. Radioactive iodine therapy
is a safe and effective treatment for Graves' disease because it can
provide the same ablative effects of surgery without the surgical
complications. There is no evidence that this treatment increases the
risk for carcinoma.
A 65-year-old male smoker complains of dyspepsia, weight loss, early
satiety, and occasional nausea and vomiting. Which one of the following
would be the initial diagnostic method of choice?

a) Upper gastrointestinal endoscopy


b) CT of the upper abdomen
c) A single contrast upper GI barium swallow
d) Endoscopic ultrasonography
The correct answer is A

Explanation
The evaluation of suspected gastric cancer should initially include
either a double-contrast barium swallow or upper gastrointestinal
endoscopy. CT may be useful in staging once the diagnosis is made.
Endoscopic ultrasonography may also be used as a secondary evaluation to
determine the extent of local disease.

A 75 year old male has not seen a physician in 25 years and presents
with advanced Parkinson‟s disease. The best initial treatment would be

a) Referral to a neurosurgeon for thalamotomy


b) Amantadine (Symmetrel)
c) Benztropine (Cogentin)
d) Pramipexole (Mirapex)
e) Carbidopa/levodopa (Sinemet)
The correct answer is E

Explanation
While anticholinergics such as benztropine and amantadine may provide
some improvement of symptoms, these effects wane within a few months.
Such medications are not a good option in this patient with advanced
disease.

Dopamine agonists provide some improvements in motor complications, but


are mainly used to delay the introduction of levodopa in younger
patients, to avoid levodopa-related adverse reactions.
Carbidopa/levodopa is better for initial therapy in older patients, and
those who present with more severe symptoms. Slow-release versions of
this combination may decrease motor fluctuations.

Stereotactic thalamotomy is used to ameliorate tremors that have become


disabling. This procedure has been replaced by other surgical options
such as pallidotomy and high-frequency, deep-brain stimulation of
specific nuclei.
A 70 year old woman returns to the office because of aching and weakness
in her arms to the point where she cannot lift her arm to brush her
hair. Physical examination shows no muscle tenderness or other evidence
of joint disease in both arms. The aching improves when she takes the
prescribed nonsteroidal anti-inflammatory drug (NSAID). She also
describes tenderness over the right temporal area of her scalp. Physical
examination of the scalp shows no lesions. Which of the following is the
most appropriate next step?

a) Increase the dose of the NSAID


b) Order determination of erythrocyte sedimentation rate
c) Order determination of serum rheumatoid factor
d) Order x-ray films of the cervical spine
e) Refer her for psychiatric counseling
The correct answer is B

Explanation
The tenderness over the right temporal area of the patient‟s scalp is
worrisome for temporal arteritis, which is often associated with an
elevated erythrocyte sedimentation rate. Prompt treatment with steroids
is often indicated to prevent blindness. The diagnosis can be
established with a temporal artery biopsy.

Increasing her dose of NSAIDs is incorrect. The most common treatments


for pain are the non-steroidal anti-inflammatory drugs, or NSAIDs. If
this is ineffective, oral steroids such as prednisone should be started.
Since this patient does not have signs or symptoms consistent with any
type of arthritis (no joint disease; arthritis is a disease of joints),
a serum RF is not useful.

A radiograph of the cervical spine is not indicated here as this patient


does not describe radicular symptoms that could be attributed to
compression of single nerve root. Rather, her symptoms are of
generalized weakness. This is not a symptom that a cervical disc could
be responsible for.
Propranolol can be used in all of the following, except

a) Lithium induced tremor


b) Panic attack
c) Hypoglycemia
d) Hypertension
e) Angina
The correct answer is C

Explanation
Propranolol is a beta-blocker used to treat chest pain (angina), high
blood pressure, irregular heartbeats, migraine headaches, tremors and
anxiety.

Beta-blockers can cause hypoglycemia or hyperglycemia. Also,


beta-blockers can blunt some of the body's responses to hypoglycemia
such as rapid heart rate, thus making it difficult for patients to
recognize (and treat) hypoglycemic reactions.
Each of the following is a correct statement about COPD, except

a) The type of emphysema associated with smoking is usually centriacinar


b) Clubbing is a typical clinical feature of COPD
c) Inhaled corticosteroids are not recommended as monotherapy for COPD
d) Smoking cessation is the single most effective therapy for the
majority of COPD patients
e) Long-term oxygen therapy improves survival in hypoxemic patients
with COPD
The correct answer is B

Explanation
Chronic obstructive pulmonary disease is partially reversible airflow
obstruction caused by an inflammatory response to inhaled toxins, often
cigarette smoke. ?1-Antitrypsin deficiency and various of occupational
exposures are less common causes in nonsmokers. Symptoms are productive
cough and dyspnea that develop over years; common signs include
decreased breath sounds, prolonged expiratory phase of respiration, and
wheezing.

Long-term O2 therapy prolongs life in COPD patients whose Pao2 is


chronically < 55 mm Hg. Continual 24-h use is more effective than a 12-h
nocturnal regimen. O2 therapy brings Hct toward normal levels;
moderately improves neuropsychologic factors, possibly by facilitating
sleep; and ameliorates pulmonary hemodynamic abnormalities. O2 therapy
also increases exercise tolerance in many patients. Long-term oxygen
therapy improves survival by 2-fold or more in hypoxemic patients with COPD

notes:
- centriacinar emphysema begins in the respiratory bronchioles and
spreads peripherally. Also termed centrilobular emphysema, this form is
associated with long-standing cigarette smoking and predominantly
involves the upper half of the lungs.
- clubbing is an enlargement of the tips of the fingers or toes and a
change in the angle where the nails emerge. Clubbing seems to occur with
some lung disorders (lung cancer, lung abscess, bronchiectasis), but not
with others (pneumonia, asthma, chronic obstructive pulmonary disease).
Clubbing also occurs with some congenital heart disorders and liver
disorders, or in some cases, it may be inherited and not indicate any
disorder. Clubbing itself does not need treatment.
A young patient in the ER has a ventricular fibrillation arrhythmia. He
is treated with procainamide. His blood pressure then decreases to
80/60. You try another time with procainamide, his blood pressure still
decreases. What is the appropriate step at this time?

a) Benztropine
b) Saline perfusion
c) Digitalis
d) Defibrillation
e) Intubation
The correct answer is D

Explanation
Ventricular fibrillation produces uncoordinated quivering of the
ventricle with no useful contractions. It causes immediate syncope and
death within minutes. Treatment is with cardiopulmonary resuscitation,
including immediate defibrillation.

An 86 year old generally debilitated man in an extended care facility


has a chronic /Escherichia coli/ bacteriuria. About once every 6 months,
he develops mild fever, frequency of micturition and some urinary
incontinence. On several of these exacerbations, his urine cultures grow
/Escherichia coli/ with colony counts greater than 100,000/mL. Which one
of the following would be the most appropriate management?

a) Catheterize and irrigate the bladder daily


b) Determine the cause of the infection by cystoscopy and intravenous
pyelogram
c) Do not give specific treatment but maintain a high fluid intake
d) Place him on continuous low-dose antibiotic therapy
e) Treat only the acute episodes of infection with appropriate antibiotic
The correct answer is E

Explanation
It is recommended that for asymptomatic bacteriuria in elderly patients,
short-course antimicrobial therapy with follow-up can eliminate
bacteriuria in most of these patients for at least a six-month period.
A patient on clozapine has to be concerned with which of the following
side effects‟

a) Dystonia
b) Tardive dyskinesia
c) Decreased white blood cell count
d) Parkinson
The correct answer is C
Explanation
Clozapine is used in the management of psychotic disorders including
schizophrenia. Because of concern for the side effect of
agranulocytosis, clozapine should be reserved for patients who have
failed to respond to other standard medications or who are at risk for
recurring suicidal behavior.

Clozapine may cause a severe reduction in white blood cell count, a


condition known as agranulocytosis, in approximately 1 in 100 patients
who take it for at least one year. White blood cells fight infections,
and a severe reduction in white blood cells can result in severe
infections. If not caught early, agranulocytosis can be fatal.
Therefore, the white blood cell count should be measured (with a blood
test) prior to starting treatment and regularly (weekly) while patients
receive this medication, and for 4 weeks after it is stopped.
A 25 year old woman complains of oral ulcers that have been bothering
her. Upon further questioning, she points out that she also experiences
recurrent abdominal cramps, diarrhea and that she has ?lost weight?. A
colonoscopy has demonstrated skipped lesions throughout intestines. What
is the most likely cause of this patient‟s symptoms‟

a) Crohn‟s disease
b) Ulcerative colitis
c) Irritable bowel syndrome
d) Herpes simplex 1
e) Celiac disease
The correct answer is A

Explanation
Crohn‟s disease is a chronic inflammatory disorder that can affect the
entire GI tract. It is more common amongst Caucasians, Jews and Asians
and peaks in people‟s 20‟s. It often presents with weight loss,
recurrent abdominal cramping and diarrhea. Ulcers can be visualized
plainly in the oral cavity or lower along the GI tract with endoscopy
(inflammatory patterns within the bowel are not continuous, termed „skip
lesions‟). Endoscopy with biopsy is diagnostic. Anal fissures and
abscesses are common, and fistulae can develop between the GI tract, the
skin, vagina or bladder.
It is treated comprehensively with modifications to diet and drugs
(sulfasalazine, prednisone for acute flare ups, immunosuppressives,
antibiotics, antidiarrheals and/or new biologicals such as infliximab).
In certain cases, surgery is warranted (fistulae, perforations or
obstructions).

Ulcerative colitis, b), always involves inflammation to the rectum, a


point of which will be alluded to in some aspect in a questions‟ stem;
furthermore, it does not involve lesions in the oral cavity.
Inflammatory patterns within the bowel are continuous.

Option c), IBS, is a diagnosis of exclusion, and often presents with


alternating patterns of diarrhea and constipation, changes in the
frequency of bowel movements and relief upon defecation. Patients with
IBS do not experience weight loss.

Herpes simplex 1, d), was placed in the question as a distractor related


to the patients oral ulcers and would not present with the other listed
symptoms.
Finally, celiac disease, e), may present similarly (including the oral
manifestations) and may also include anemia and complaints of bloating
and gas. It will not show a pattern of skip lesions (rather it will show
blunting of villi & crypt hyperplasia with lymphocytic infiltration).

Pearl: Crohn‟s can affect the entire GI system (?it skips from gums to
bums‟).
An 80 year old man, who is taking warfarin because of a history of
atrial fibrillation, presents to the emergency room with a syncopal
episode. His stool has been black. His hemoglobin is 65 g/L and INR 7.2.
Which one of the following is the best immediate approach to the
management of raised INR?

a) Vitamin K, 1 mg orally
b) Vitamin K, 10 mg intravenously
c) Hold warfarin for 3 days
d) Desmopressin (DDAVP)
e) 4 units of fresh frozen plasma in addition to 10mg of IV Vitamin K
The correct answer is E

Explanation
If the INR is between 5.0-9.0 and bleeding is absent, you should cease
warfarin therapy; consider reasons for elevated INR and patient-specific
factors. If bleeding risk is high, give vitamin K (1.0-2.0 mg orally or
0.5-1.0 mg intravenously). Measure INR within 24 hours, resume warfarin
at a reduced dose once INR is in therapeutic range.

However, if there is any clinically significant bleeding where


warfarin-induced coagulopathy is considered a contributing factor, you
should cease warfarin therapy, give 5.0-10.0 mg vitamin K intravenously,
as well as fresh frozen plasma (150-300 mL), assess patient continuously
until INR < 5.0, and bleeding stops.
A patient with known type II diabetes mellitus who is treated with
Clorpropamide, presents to ER unconscious. His blood sugar is very low.
You give the patient 1 ampule of D5 and the patient wakes up promptly.
What is the next appropriate management?

a) Give another ampule then discharge


b) Give 2 ampules then discharge
c) Give another ampule then observe the patient for 6 hours in the ER
d) Give one more ampule, admit the patient and start IV glucose
infusion with frequent ongoing glucose measurement
The correct answer is D

Explanation
This patient should be admitted. Hypoglycemia causes sweating, nausea,
warmth, anxiety, tremulousness, palpitations, and possibly hunger and
paresthesias. Insufficient glucose supply to the brain causes headache,
blurred or double vision, confusion, difficulty speaking, seizures, and
coma. In controlled settings, autonomic symptoms begin at or beneath a
plasma glucose level of about 60 mg/dL (3.33 mmol/L), whereas CNS
symptoms occur at or below a glucose level of about 50 mg/dL (2.78 mmol/L).

With severe hypoglycemia patients should be given 50% dextrose 50 to 100


mL IV bolus, with or without a continuous infusion of 5 to 10% dextrose
solution sufficient to resolve symptoms. Careful monitoring in the
inpatient setting is recommended.
A 28 year old married mother of one presents to her family physician
with a history of sudden onset of palpitations, feelings she can not get
enough air and that she is going to die. She has had several of these
episodes in the last two months. She has been to the emergency
department on three occasions and was told that her blood pressure, ECG,
and chest x-ray are normal. Which one of the following is the most
likely diagnosis‟

a) Hyperthyroidism
b) Premature atrial contractions
c) Pheochromocytoma
d) Recurrent major depression
e) Panic disorder
The correct answer is E

Explanation
A panic attack is the sudden onset of a discrete, brief period of
intense discomfort, anxiety, or fear accompanied by somatic or cognitive
symptoms. Panic disorder is occurrence of repeated panic attacks
typically accompanied by fears about future attacks or changes in
behavior to avoid situations that might predispose to attacks.

Diagnosis is clinical. Isolated panic attacks may not require treatment.


Panic disorder is treated with drug therapy, psychotherapy (eg, exposure
therapy, cognitive-behavioral therapy), or both.
A man has a semen analysis done. It shows abnormal motility and shape,
and a total sperm count of 950,000 sperm (normal sperm count is greater
than 2,000,000). What do you advise him about his future fertility?

a) Chance of fertility is 10%


b) Chance of fertility is 50%
c) He is impotent
d) He could have a child with in-vitro fertilization (IVF)
The correct answer is D

Explanation
Abnormal morphology (shape) and motility can prevent the sperm from
reaching the egg. The sperm need motility to be able to swim well and
survive for a number of hours in the female reproductive tract. If they
do meet, abnormal-looking sperm might be incapable of fertilization.

The motility of this man‟s sperm is poor and therefore he is unlikely to


be able to be fertile. In addition, infertility specialists have stated
that 1 million motile sperm is the minimum amount of sperm associated
with a reasonable chance of pregnancy success at intrauterine insemination.

However in-vitro fertilization (IVF) can be used to treat infertility


due to his oligospermia. The procedure would involve controlled ovarian
hyperstimulation, oocyte retrieval, fertilization with sperm, embryo
culture, and embryo transfer.

Impotence is defined as an inability to achieve and/or maintain an erection.


A 54 year old man presents with extreme pain and swelling of the left
middle finger. Four days earlier, he suffered a small puncture wound to
the volar aspect of the finger at the level of the distal flexor crease.
Passive extension of the proximal interphalangeal (PIP) and distal
interphalangeal (DIP) joints severely aggravates his symptoms. Which one
of the following is the most likely diagnosis‟

a) Pulp space infection


b) Cellulitis
c) Mid-palmar space infection
d) Septic tenosynovitis
e) Septic arthritis distal interphalangeal (DIP) joint
The correct answer is D

Explanation
These conditions are degeneration (tendinopathy) and associated
inflammation (tendinitis) of a tendon or inflammation of the tendon
sheath lining (tenosynovitis). Symptoms usually include pain with motion
and tenderness on palpation. Chronic tendon deterioration or
inflammation can cause scars that restrict motion.

Volar flexor tenosynovitis is one of the most common musculoskeletal


entities and is often overlooked. Pain occurs in the palm on the volar
aspect of the thumb or other digits and may radiate distally. The
diagnosis is made by eliciting tenderness on palpation of the tendon and
sheath and by noting swelling and possibly a nodule. In later stages,
triggering or snapping of the digit occurs on flexion (trigger finger).

Diagnosis is clinical. Treatment includes rest, NSAIDs, and sometimes


corticosteroid injections.
A 39 year old automobile mechanic presents because of the new onset of
wheezing. His meds are propranolol, enalapril, hydrochlorothiazide,
ranitidine and occasional pseudoephedrine for symptoms of allergic
rhinitis. On physical examination, he appears comfortable. His blood
pressure is 134/88 mm Hg, pulse is 68/min, and respirations are 18/min.
On lung examination, soft expiratory wheezes are heard throughout both
lung fields. Which of the following medications is most likely
contributing to his wheezing?

a) Enalapril
b) Hydrochlorothiazide
c) Propranolol
d) Pseudoephedrine
e) Ranitidine
The correct answer is C

Explanation
Propranolol, like other nonspecific beta blockers, may cause
bronchospasm by blocking the beta receptors in the bronchial tree. Beta
stimulation in the lungs produces bronchodilation, and its blockade
leads to bronchoconstriction. In fact, propranolol is contraindicated in
patients with known asthma or chronic obstructive pulmonary disease (COPD).
After a long history of multiple myeloma, a 67 year old male is noted to
have abundant acellular eosinophilic deposits around the pulmonary
microvasculature at autopsy. A congo red special stain demonstrates
apple green birefringence. Which one of the following conditions is the
most likely explanation?
a) Sarcoidosis
b) Histoplasmosis
c) Amyloidosis
d) Mesothelioma
e) Silicosis
The correct answer is C

Explanation
Amyloidosis is any of a group of disparate conditions characterized by
extracellular deposition of various insoluble proteins. These proteins
may accumulate locally, causing relatively few symptoms, or widely,
involving multiple organs and producing severe multiorgan failure.
Amyloidosis can be primary or be secondary to various infectious,
inflammatory, or malignant conditions. Rarely, it results from any of
several inherited metabolic defects. Diagnosis is by biopsy of affected
tissue.

Amyloid deposits may be formed from at least 18 different proteins,


including immunoglobulin fragments. Amyloid deposits are metabolically
inert but interfere physically with organ structure and function. All
stain positive with Congo red dye, stain pink with hematoxylin and
eosin, and have apple-green birefringence under polarized light after
Congo red staining. Amyloid deposits have a fibrillar, usually rigid,
and nonbranching ultrastructure.
Warfarin is used for prophylaxis and treatment of venous thrombosis and
its extension, prophylaxis and treatment of pulmonary embolism,
prophylaxis and treatment of thromboembolic complications associated
with atrial fibrillation and/or cardiac valve replacement, and as an
adjunct in the treatment of coronary occlusion. What is its mechanism of
action?

a) It alters the synthesis of blood coagulation factors I and VIII.


b) It antagonizing the vitamin D production
c) It acts as the Vitamin K agonist
d) It inhibits epoxide reductase, thereby diminishing available Vitamin
K stores.
e) It alters the synthesis of blood coagulation factors III, IV and V.
The correct answer is D

Explanation
Warfarin is a coumarin-derivative anticoagulant that alters the
synthesis of blood coagulation factors II (prothrombin), VII, IX, and X
in the liver by interfering with the action of vitamin K. Warfarin
inhibits epoxide reductase, thereby diminishing available vitamin K
stores and inhibiting production of functioning coagulation factors.

A 34-year-old black female has had frequent asthma exacerbations for the
past several months. She uses her albuterol (Proventil, Ventolin)
inhaler 4 times per week with a good response.

Which one of the following is preferred for the prevention of future


asthma exacerbation?

a) Long-acting ?_2 -agonists


b) Leukotriene modifiers
c) Inhaled corticosteroids
d) Theophylline
e) No additional therapy
The correct answer is C

Explanation
This patient has mild persistent asthma and a controller medicine is
indicated. The preferred treatment for patients with mild persistent
asthma is low-dose inhaled corticosteroids. Alternate second-line
treatments include cromolyn, leukotriene modifiers, nedocromil, and
sustained-release theophylline. Long-acting ?_2 -agonists should not be
used alone for the treatment of persistent asthma. They can be used in
combination with inhaled corticosteroids for initial treatment of
moderate persistent asthma, but are not recommended for initial
treatment of mild persistent asthma. Short-acting bronchodilators, such
as albuterol, are effective for treatment of acute asthma symptoms, but
should not be used as preventive treatment of patients with persistent
asthma.
A 25-year-old runner complains of nonfocal knee pain. She does not
remember any specific injury. You suspect patellofemoral pain syndrome.

Which one of the following would be most consistent with this diagnosis‟

a) Pain with prolonged sitting


b) Swelling
c) Locking
d) Giving way
The correct answer is A

Explanation
Patellofemoral pain syndrome causes nonfocal or anterior knee pain, and
is often seen in runners. Common symptoms include stiffness, pain with
prolonged sitting, and pain with climbing or descending stairs. Rarely
is there swelling, locking, or giving way; these symptoms are more
likely to be associated with more profound problems such as a ligament
or cartilage tear.
A 24 year old man presents for evaluation of a 4 month history of
postprandial diarrhea, weight loss of 9 pounds, and lower abdominal
pain. He denies recent travel or antibiotic use. On physical
examination, his temperature is 38.0 C (100.4), and he has several oral
aphthous ulcers. On abdominal examination, there is tenderness and mild
voluntary guarding in the right lower quadrant. A rectal examination
reveals brown stool that is strongly guaiac positive. Which of the
following is most likely causing this patient's symptoms‟

a) A gram-negative organism
b) Folate deficiency
c) Mucosal ulceration with no transmural involvement in the ascending colon
d) Toxin-producing organism
e) Transmural inflammation in the region of the terminal ileum
The correct answer is E

Explanation
This patient, with postprandial diarrhea, weight loss, low-grade fever,
and right lower quadrant findings on physical examination, has the
typical presentation of Crohn disease, which most commonly involves the
terminal ileum. Inflammation in this disease is transmural, as opposed
to the inflammation in ulcerative colitis that is limited to the mucosa
of the large intestine.
What would be the best test (hight sensitivity and specificity) to
confirm the diagnosis of hereditary spherocytosis‟

a) Coombs test
b) Bone marrow aspiration
c) Cryohemolysis test
d) Red blood cell enzyme studies
e) Human leukocyte antigens (HLA) studies
The correct answer is C

Explanation
Hereditary spherocytosis and hereditary elliptocytosis are congenital
RBC membrane disorders. Symptoms, generally milder in hereditary
elliptocytosis, include variable degrees of anemia, jaundice, and
splenomegaly.

The red cell count and hemoglobin are moderately reduced. Some of the
asymptomatic patients detected by family surveys have normal red cell
counts when first seen. The red cells are usually normocytic, but
microcytosis may occur. Macrocytosis may present during periods of
marked reticulocytosis. Spherocytes in varying numbers, sizes, and
shapes are seen on a Wright-stained smear. The reticulocyte count is
increased to 5-20%.

The indirect serum bilirubin and stool urobilinogen are usually


elevated, and serum haptoglobin is usually decreased to absent. The
Coombs test is negative. Osmotic fragility is increased; hemolysis of
5-10% of cells may be observed at saline concentrations of 0.6%.

A more accurate reflector of fragility is the cryohemolysis test, which


has a sensitivity and specificity of almost 95% for spherocytosis.
Metformin (Glucophage), which is normally used in the management of
diabetes mellitus, has also been shown to have a beneficial effect in

a) Osteoporosis
b) Hyperthyroidism
c) Polycycstic ovary syndrome
d) Right ventricular hypertrophy
e) Morbid truncal obesity
The correct answer is C

Explanation
Recent data suggests that insulin resistance and hyperinsulinemia are
important in the pathogenesis of polycystic ovary syndrome (POS).
Treatment with drugs that reduce insulin levels, such as metformin, has
been shown to correct many of the metabolic abnormalities associated
with POS. Such correction results in resumption of ovulation, decreased
insulin resistance, and improved beta-cell function; it also produces
improvement in cardiovascular risk factors such as dyslipidemia and
impaired fibrinolysis.

A 20 year old man is referred for assessment of proteinuria found on an


insurance examination. The history is unrevealing. Physical examination
is normal. Urinalysis (urine microscopy) shows 1-2+ proteinuria by
"dipstick" and is otherwise unremarkable. Urea nitrogen is 5.4 mmol/L
(Normal 2.5-8.0 mmol/L) and serum creatinine is 88.4 µmol/L (Normal
70-120 µmol/L). Which one of the following would be the correct management?

a) Determination of 24-hour urine protein excretion, fractionated into


supine and upright samples
b) Reassurance and reassessment in 6 months
c) Intravenous pyelogram and cystoscopic examination
d) Admission to hospital for percutaneous renal biopsy
The correct answer is A

Explanation
Proteinuria is protein, usually albumin, in urine. In many renal
disorders, it occurs with other urinary abnormalities (eg, hematuria).
Isolated proteinuria is urinary protein without other symptoms or
abnormalities. Causes may be categorized as glomerular, tubular,
overflow, or physiologic. Exercise proteinuria, sometimes with
hematuria, hemoglobinuria, or myoglobinuria, is proteinuria of unknown
cause that may occur in runners, boxers, and other people engaged in
vigorous exercise.

High concentrations of protein cause frothy or sudsy urine; patients,


especially men, occasionally report this symptom. But diagnosis of
proteinuria requires urinalysis; it is most often detected incidentally.
History and physical examination occasionally give clues to cause.

If urine dipstick assay detects proteinuria, it should be repeated 1 to


2 times; resolution suggests an initial false-positive result or a
transient cause. Persistent proteinuria should be quantified with a 24-h
urine collection, although calculation of a protein:creatinine ratio in
a random sample may be sufficient. Nephrotic syndrome is defined as
proteinuria > 3.5 g/day.

Microscopic analysis should be performed to detect casts, lipiduria, and


other signs of glomerular disorders. Orthostatic proteinuria is
diagnosed with a 24-h urine collection split between a 16-h upright
daytime specimen (in which protein is present) and an 8-h recumbent
nighttime specimen (in which protein should be < 50 mg).
Which one of the following studies is indicated for patients with
restless legs syndrome (RLS)?

a) Antinuclear antibody (ANA) testing


b) A creatine phosphokinase level
c) A CBC
d) A serum ferritin level
e) Electromyography
The correct answer is D

Explanation
A high prevalence of iron deficiency has been found among patients with
restless legs syndrome (RLS), and treatment of the deficiency has been
reported to improve or resolve symptoms. A low serum ferritin level (<50
mg/L) has been associated with greater severity of RLS and with a
reduction in the quantity of sleep as determined by polysomnography. In
patients with a serum ferritin level <18 mg/L, treatment with oral iron
supplements resulted in improvements in the severity of the symptoms of
RLS and, in some patients, complete resolution of symptoms.
Because RLS may be a symptom of iron deficiency and because iron
deficiency is frequently present in the absence of anemia, iron status
(serum ferritin and iron saturation) should be assessed. No other
laboratory tests are routinely indicated.
A 70 year old woman presents with acute knee arthritis. Radiographs show
meniscal calcification (chondrocalcinosis). Analysis of the synovial
fluid reveals weakly positive birefringent rhomboid-shaped crystals. The
crystals are most likely

a) Monosodium urate
b) Calcium hydroxyapatite
c) Cholesterol
d) Calcium pyrophosphate dihydrate
The correct answer is D

Explanation
Calcium pyrophosphate dihydrate (CPPD) crystal deposition disease
involves intra-articular and/or extra-articular deposition of CPPD
crystals. Manifestations are protean and may be minimal or include
intermittent attacks of acute arthritis and a degenerative arthropathy
that is often severe. Diagnosis requires identification of CPPD crystals
in synovial fluid. Diagnosis is established by identifying rhomboid or
rod-shaped, weakly positively birefringent crystals on polarized light
microscopy of synovial fluid.
Which one of the following antihypertensive agents may help preserve
bone mineral density?

a) Atenolol (Tenormin)
b) Doxazosin (Cardura)
c) Enalapril (Vasotec)
d) Hydrochlorothiazide
e) Nifedipine (Procarda, Adalat)
The correct answer is D

Explanation
In healthy older adults, low-dose hydrochlorothiazide preserves bone
mineral density of the hip and spine. The modest effects observed over 3
years, if accumulated over 10-20 years, may explain the one-third
reduction in risk for hip fracture associated with thiazide use in many
epidemiologic studies. There are no such benefits reported for the other
medications listed.
Five days ago you prescribed amoxicillin, 500 mg 3 times daily for 14
days, to a 34-year-old female with purulent nasal drainage and
tenderness over her maxillary sinuses. She returns today with worsening
symptoms.

Which one of the following would be the best choice for continued therapy?

a) Change to a high-dose amoxicillin regimen


b) Complete the prescribed regimen of amoxicillin
c) Discontinue amoxicillin and initiate antihistamines
d) Discontinue amoxicillin and start azithromycin (Zithromax)
e) Discontinue amoxicillin and start levofloxacin (Levaquin)
The correct answer is E

Explanation
Guidelines recommend that if symptoms of rhinosinusitis have not
improved after 3-5 days, antibiotic resistance should be considered and
broadening the spectrum of coverage is appropriate. Levofloxacin is
recommended in this situation. Given the symptoms of an infectious
etiology, antihistamines alone are not appropriate. While high-dose
amoxicillin is recommended as a first-line therapy, it is not
recommended for treatment failures. Resistance to azithromycin is fairly
high after an initial antibiotic failure.
You see a patient for the first time who has AIDS and chronic hepatitis
B. He is losing weight, and in spite of adequate antiretroviral therapy,
is becoming weaker, to the point of being virtually bedridden. Because
of ascites, low serum albumin, and elevated liver enzymes, you suspect
chronic hepatitis as the cause of his decline. Which one of the
following would be most likely to improve this patient‟s condition?

a) Antiviral drugs for hepatitis B


b) Appetite stimulation with topical androgens
c) Appetite stimulation with dronabinol (Marinol)
d) Liver transplantation
e) No treatment, with palliative care being the only appropriate management
The correct answer is A

Explanation
Effective oral antiviral drugs are now available for chronic hepatitis B
(at a cost of about $20 per day) and can be added to highly active
antiretrovirals. The recent trend in the treatment of newly diagnosed
patients with AIDS and hepatitis B is to treat both problems initially,
selecting AIDS drugs that are also active against hepatitis B. Various
agents to stimulate appetite are used in declining HIV patients, but
have little benefit. Liver transplantation has been done in a few cases
of coinfection with hepatitis B and HIV, but the hepatitis B viremia has
to be suppressed first.
An 85 year old white male is brought to you for the first time by his
son. The father has recently seen a neurologist who performed a workup
for dementia and diagnosed moderate Alzheimer‟s disease. Which one of
the following is true regarding the use of a cholinesterase inhibitor in
this patient?

a) It is too late to initiate cholinesterase therapy


b) Agitation is often intensified by these agents
c) Memory is likely to improve significantly
d) If the patient has a vascular dementia rather than Alzheimer‟s
dementia the drug will not be useful
e) Nursing-home placement may be delayed a year or longer
The correct answer is E

Explanation
Medications for dementia should be prescribed with caution, and the
patient watched closely for side effects. Currently available
cholinesterase inhibitors are at least as effective for vascular
dementia as they are for Alzheimer‟s dementia, although they are not
approved for this use by the FDA. These agents include donepezil,
rivastigmine, and galantamine, and they are often helpful in Alzheimer‟s
disease patients with agitation. Anticholinesterase therapy is
considered the standard of care for Alzheimer‟s disease, and therapy can
begin at any stage in the disease process, although early therapy is the
most beneficial. While the medication will not restore memory, it does
prevent the rapid loss of more memory. Long-term studies on
effectiveness are still in progress, but most evidence at present
indicates that nursing-home placement can be delayed by a year and
possibly longer.
A 62-year-old white female presents to your office with moderately
severe knee pain. She has a history of osteoarthritis and is not aware
of any recent injury. The pain bothers her both during the day and at night.

Examination reveals a moderately obese female with a normal knee


examination except for tenderness in the medial tibial plateau region,
approximately 3 cm (1½ in) below the medial joint line of the knee. The
area of tenderness is about the size of a quarter. All ligaments of the
knee are intact on examination. There is no knee effusion. A radiograph
is negative except for minimal degenerative changes.

Which one of the following should you suspect?

a) De Quervain‟s tendinitis
b) Prepatellar bursitis
c) Bursitis of the medial collateral ligament
d) Anserine bursitis
e) Medial meniscus tear
The correct answer is D

Explanation
Anserine bursitis is characterized by pain, particularly at night, that
occurs in the medial knee region over the upper tibia. It is located
about 2-3 cm below the medial joint line. It can be bilateral. A
diagnosis of anserine bursitis requires local tenderness confined to a
quarter-sized area of the medial tibial plateau, approximately 3 cm
below the medial joint line; a negative valgus stress maneuver, which
indicates an intact medial collateral ligament; and a normal radiograph
of the tibia indicating no underlying pathology. De Quervain‟s
tendinitis is located in the wrist region, not the knee. Prepatellar
bursitis is characterized by knee swelling and pain over the front of
the knee. Bursitis that occurs adjacent to the medial collateral
ligament typically presents with tenderness over the medial aspect of
the knee. Medial joint line pain is characteristic of osteoarthritis,
second and third degree medial collateral ligament injuries, medial
meniscal tears, and fractures of the tibial plateau.
Osmotic demyelination can result when which one of the following is
corrected too rapidly?

a) Hypocalcemia
b) Hypoglycemia
c) Hypomagnesemia
d) Hypokalemia
e) Hyponatremia
The correct answer is E

Explanation
The adaptation that permits survival in chronic hyponatremia also makes
the brain vulnerable to injury from overzealous therapy. When
hyponatremia is corrected too rapidly, outpacing the brain‟s ability to
recapture lost organic osmolytes, osmotic demyelination can result.
Osmotic demyelination syndrome can usually be avoided by limiting
correction of chronic hyponatremia to <10?12 mmol/L in 24 hours and to
<18 mmol/L in 48 hours.
A patient with rheumatoid arthritis who has been taking aspirin present
with a normocytic normochromic anemia. All of the following are true
about his anemia, except

a) It will respond to treatment with vitamin B12


b) Could be because of blood loss
c) MCV of 91
d) Elevated reticulocyte count
The correct answer is A

Explanation
When a normocytic normochromic anemia is encountered, the anemia can
have 3 possible etiologies (ie, blood loss, hemolysis, decreased
production). In most anemias, one of these causes is the dominant factor.

Hemolysis will show an increased reticulocyte count. Normocytic anemias


have an MCV value between 80 and 100.

Vitamin B12 is given for anemia due to vitamin B12 deficiency. This type
of anemia would be macrocytic.
A 72 year old male with a history of previous inferior myocardial
infarction sees you prior to surgery for symptomatic gallstones. He
denies chest pain or dyspnea. His current medications include aspirin,
81 mg daily; ramipril (Altace), 10 mg daily; and pravastatin
(Pravachol), 40 mg daily. He is in good health otherwise and has no
other health complaints. He has been cleared for surgery by his
cardiologist. Which one of the following should be considered before and
after surgery, assuming no contraindications‟

a) Atenolol (Tenormin)
b) Verapamil (Calan, Isoptin)
c) Digoxin
d) Transdermal nitroglycerin
e) Intravenous nitroglycerin
The correct answer is A

Explanation
A recent development in the prophylaxis of surgery-related cardiac
complications is the use of Beta-blockers perioperatively for patients
with cardiac risk factors. In a randomized, double-blind,
placebo-controlled trial involving 200 patients who were undergoing
elective noncardiac surgery that required general anesthesia, the effect
of atenolol on perioperative cardiac complications was evaluated.

Patients were eligible for Beta-blocker therapy if they had known


coronary artery disease or two or more risk factors. Atenolol was not
used if the resting heart rate was < 55 beats/min, systolic blood
pressure was < 100 mm Hg, or there was evidence of congestive heart
failure, third degree heart block, or bronchospasm. A 5-mg dose of
intravenous atenolol was given 30 minutes before surgery and then again
immediately after surgery. Oral atenolol, 50-100 mg, was given until
hospital discharge or 7 days postoperatively. The results of the study
showed that mortality from cardiac causes was 65% lower in the patients
receiving atenolol. Another study showed similar perioperative benefit
using the Beta-blocker bisoprolol.
A 48 year old woman with a history of deep venous thrombosis is taking
warfarin (Coumadin), 10 mg once daily, and is maintaining an
International Normalized Ratio (INR) of 2.75. She started taking high
doses of garlic and would like to know if she can continue to take it.
She has blood in her nares and multiple bruises on her extremities that
began after she started taking the garlic. Which one of the following
statements is correct?

a) She can continue to take the garlic because her INR is appropriate
b) interaction with garlic should be suspected despite an appropriate INR
c) Garlic does not pose a risk of drug interactions and she can
continue taking it
d) Garlic in combination with warfarin poses serious risk of cerebral
hemorrhage and should be stopped immediately
The correct answer is D

Explanation
Garlic is thought to provide several cardiovascular benefits, such as
blood pressure lowering, serum lipid lowering, and antithrombotic
activity. Garlic oil has been reported to interrupt thromboxane
synthesis, thereby inhibiting platelet function. There have been reports
of spontaneous epidural hematoma after ingesting approximately 2000 mg
of garlic daily (equivalent to about four cloves) for an undetermined
period.

The available information suggests that a serious interaction is


possible. Patients taking warfarin should be advised to avoid garlic
supplements. However, they should also be aware that regular ingestion
of food products containing small amounts of garlic should not pose a
problem. If excessive garlic consumption and warfarin use occur
concomitantly, the patient's INR should be closely monitored.
The drug of choice for an obese diabetic type 2 patient is

a) Acarbose
b) Metformin
c) Rosiglitazone
d) Glyburide
The correct answer is B

Explanation
Metformin has been clearly established as the drug of choice in obese
patients with diabetes mellitus. Metformin is as effective as
sulphonylureas at reducing HbA1c, and most importantly has a beneficial
effect on overall mortality in obese patients.

Metformin monotherapy is unlikely to be effective in patients who fail


to respond to sulphonylureas, but in patients who are secondary failures
to sulphonylureas, the addition of metformin causes substantial blood
glucose lowering. Metformin should be avoided in patients with renal
dysfunction. Otherwise its effects on bodyweight, serum lipids and its
lack of hypoglycemia effect make it an excellent first line agent.

The most effective treatment for osteoporosis is

a) Exercise
b) Analgesics
c) Vitamins
d) Bisphosphonates
The correct answer is D

Explanation
Osteoporosis is a progressive metabolic bone disease that decreases bone
density (bone mass per unit volume), with deterioration of bone
structure. Skeletal weakness leads to fractures with minor or inapparent
trauma, particularly in the thoracic and lumbar spine, wrist, and hip.
Acute or chronic back pain is common. Diagnosis is by dual-energy x-ray
absorptiometry.

Bisphosphonates are first-line drug therapy. By inhibiting bone


resorption, bisphosphonates preserve bone mass and can decrease
vertebral and hip fractures by 50%. Estrogen can preserve bone density
and prevent fractures.

Prevention and treatment involve Ca and vitamin D supplements, exercises


to maximize bone and muscle strength and minimize the risk of falls, and
drug therapy to preserve bone mass or stimulate new bone formation.
A 53-year-old male accountant comes to your office with progressive
facial weakness on the left side that began yesterday. He also reports
pain behind the left ear and decreased lacrimation from the left eye. He
has been in good health and had his yearly physical examination 1 week
ago, which was normal. His lipid levels, chemistry profile, and CBC were
all normal. He has not been involved in any outdoor activities, nor does
he engage in any high-risk sexual behavior.

On examination, the forehead is smooth bilaterally and there is


flattening of the left nasolabial fold and decreased ability to close
the left eye. The mouth appears to be drawn to the right. The remainder
of his general examination and neurologic examination are normal.

Which one of the following would be the most appropriate management at


this time?

a) Carotid ultrasonography
b) High-resolution CT
c) MRI with gadolinium enhancement
d) Aspirin and observation
e) Prednisone and valacyclovir (Valtrex)
The correct answer is E

Explanation
Acute partial or complete paralysis of the peripheral facial nerves is
called Bell‟s palsy. The etiology is still unknown, but it could be
genetic, metabolic, autoimmune, vascular, entrapment, or infectious.
There is reasonable evidence indicating that the condition may be due to
reactivation of herpes simplex virus, resulting in a viral-induced neuritis.

Associated infections may be viral (herpes simplex, herpes zoster, HIV,


mumps, adenovirus, coxsackievirus, polio, Epstein-Barr virus, influenza)
or bacterial (otitis media, Lyme disease, syphilis, leprosy). Women who
are pregnant have a risk three times higher than that of nonpregnant
women. Sarcoidosis, multiple sclerosis, and post-infectious
demyelination are other possibilities. Hypertension, diabetes mellitus,
and hypothyroidism may be risk factors, but are probably not etiologic
agents.
The key diagnostic point is determining the time of onset. If the onset
occurs over a day or two and maximal paralysis is reached in 3 weeks or
less, it is likely a Bell‟s palsy. A prolonged, slowly progressive, or
relapsing course suggests tumor, especially if there is no recovery.
Examination for middle ear disease and checking for parotid masses
should be part of the evaluation.

Tests may be necessary if the etiologies noted above have been ruled
out. There is some evidence to show that treatment with prednisone and
an antiviral agent such as valacyclovir is beneficial.
A passenger on a bus sustains an injury to his eye with the newspaper of
a neighboring person. His vision becomes blurry and he develops redness
and pain in his eye. What is the likely diagnosis‟

a) Corneal abrasion
b) Conjunctivitis
c) Iritis
d) Keratitis
The correct answer is A

Explanation
Corneal abrasion is the most common type of injury involving the cornea.
Contact with dust, dirt, sand, wood shavings, metal particles or even an
edge of a piece of paper can scratch or cut the cornea. Because the
cornea is extremely sensitive, abrasions can be painful. In case of such
an injury, prompt medical care should be obtained.
An elderly male had difficulty voiding and required catheterization
followed by cystoscopy the next day. Later, on the day of the
cystoscopy, he became confused and lethargic, and he developed
tachycardia, hyperventilation and fever. The most likely cause of this
clinical picture is

a) Analgesic overdose
b) Urinary retention
c) Respiratory infection
d) Early septic shock
e) Pulmonary embolism
The correct answer is D

Explanation
Sepsis, severe sepsis, and septic shock are inflammatory states
resulting from systemic bacterial infection. In severe sepsis and septic
shock, there is critical reduction in tissue perfusion. Common causes
include gram-negative organisms, staphylococci, and meningococci.
Symptoms often begin with shaking chills and include fever, hypotension,
oliguria, and confusion.

Acute failure of multiple organs can occur, including the lungs,


kidneys, and liver. Treatment is aggressive fluid resuscitation,
antibiotics, supportive care, and sometimes intensive control of blood
glucose and administration of corticosteroids and activated protein C.

Which one of the following is correct regarding nonalcoholic fatty liver


disease?
a) It is considered a benign condition
b) It is not associated with cirrhosis
c) It is associated with type 2 diabetes mellitus
d) Isolated alkaline phosphatase elevation is the most common
associated laboratory
abnormality
e) The diagnosis is usually confirmed by CT
The correct answer is C

Explanation
Nonalcoholic fatty liver disease is associated with type 2 diabetes
mellitus. It is not always a benign condition; in fact, it is now
thought to be the most common cause of cryptogenic cirrhosis,
particularly in overweight or obese individuals. It is associated with
aminotransferase elevation, but can occur with normal or only slightly
elevated levels. Imaging studies have limited utility in diagnosing
nonalcoholic fatty liver disease, and cannot be routinely recommended.
A 27-year-old female complains of palpitations and heat intolerance. She
had similar symptoms before being diagnosed with hyperthyroidism and
treated with propylthiouracil. The propylthiouracil was discontinued 6
weeks ago by an emergency department physician because the patient
developed a rash. The rash resolved and the patient did not keep a
recommended follow-up appointment with you.

On physical examination, the patient‟s pulse is 134 beats/min and


regular, her blood pressure is 136/58 mm Hg, and her temperature is
36.9C (98.4F). The thyroid is diffusely enlarged without nodularity.
The skin is moist and a fine tremor is present. The examination is
otherwise unremarkable.

Which one of the following is most likely to be effective for initial


symptom control?

a) Propranolol (Inderal)
b) Potassium iodide (SSKI)
c) Verapamil (Calan, Isoptin)
d) Adenosine (Adenocard)
e) Diltiazem (Cardizem)
The correct answer is A

Explanation
Symptoms of hyperthyroidism including palpitations, heat intolerance,
tachycardia, and tremor are largely mediated by increased adrenergic
activity. Propranolol is useful for treating the symptoms of mild
hyperthyroidism. It is effective in decreasing tachycardia and tremor
and also has a role in the treatment of thyroid storm. At high doses it
blocks conversion of T4 to T3, in addition to providing ?-blockade.

Antithyroid agents such as propylthiouracil are useful, but require an


extended period for symptom control. Potassium iodide should not be used
until antithyroid therapy with an agent such as propylthiouracil has
been instituted. It is most frequently used as part of the treatment of
thyroid storm.

Calcium channel blockers such as diltiazem and verapamil may be useful


in slowing supraventricular arrhythmias by blocking conduction at the AV
node. They are not useful for the adrenergically triggered sinus
tachycardia of hyperthyroidism. Adenosine has similar but transient
effects on the AV node.
A 45-year-old male was recently diagnosed with type 2 diabetes mellitus.
He presents to your office today to begin oral antihyperglycemic
therapy. Results of a fasting serum metabolic panel include the following:

Laboratory Findings

Sodium---------------------------------136mmol/L (N 136-145)

Potassium---------------------------------3.7 mmol/L (N 3.5-5.1)

Chloride---------------------------------102 mmol/L (N 98-107)


BUN---------------------------------15mg/dL (N 7-18)

Creatinine---------------------------------1.7 mg/dL (N 0.6-1.3)

Glucose---------------------------------183 mg/dL

Which one of the following medications would be contraindicated in this


patient?

a) Glipizide (Glucotrol)
b) Acarbose (Precose)
c) Metformin (Glucophage)
d) Pioglitazone (Actos)
The correct answer is C

Explanation
Maintaining good blood glucose control is important for preventing the
microvascular complications of diabetes mellitus. A number of oral
antihyperglycemic agents are available, but each drug class has unique
adverse effects that affect their appropriateness for individual
patients. In this case, the patient has evidence of renal impairment
(serum creatinine ?1.5 mg/dL for men and ?1.4 mg/dL for women). This
leads to an increased risk of lactic acidosis when a biguanide such as
metformin is used. Other contraindications to biguanide use include
hepatic dysfunction, congestive heart failure, metabolic acidosis,
dehydration, and alcoholism. The sulfonylureas include chlorpropamide
(1st generation) and glipizide (2nd generation). They are associated
with weight gain and hypoglycemia. ?-Glucosidase inhibitors (e.g.,
acarbose) are less effective than other drug classes as monotherapy and
can be associated with gastrointestinal side effects. Thiazolidinediones
(e.g., pioglitazone) are associated with weight gain, edema, and
potential liver injury.
A 62-year-old white male presents with symptoms of intermittent
claudication. The patient smokes 1 pack of cigarettes per day and says
he has been told in the past that he has diabetes mellitus, high
cholesterol, and high blood pressure. He has not followed up on these
problems with a physician because he has been ?too busy?.

On examination his blood pressure is 140/105 mm Hg, and he has findings


consistent with peripheral artery disease. A laboratory workup reveals
moderate hyperglycemia and elevated total and LDL cholesterol.
Which one of the following would have the most significant effect on
improving his symptoms of intermittent claudication?

a) Control of his diabetes mellitus


b) Control of his hypertension
c) Treatment of his dyslipidemia
d) Smoking cessation
The correct answer is D

Explanation
Cigarette smoking is the most important risk factor for intermittent
claudication, which is the most common symptom of peripheral vascular
disease (PVD). Intermittent claudication is described as cramping pain
in the muscles of the legs with exercise, and is experienced by 2% of
adults over 65 years of age. In one multi-site study of nearly 7,000
adults over 50 years of age, 29% had PVD, defined by an ankle-brachial
index (ABI) of ? 0.9. As many as 90% of those with PVD may be asymptomatic.

Cigarette smoking increases the odds for PVD by 1.4 for every 10
cigarettes smoked/day. In addition to smoking, risk factors include
diabetes mellitus, hypertension, and hyperlipidemia. Lifestyle
modification is important in the management of PVD, with smoking
cessation as the single most important intervention. Exercise training
can increase functional capacity by increasing walking time up to 150%.
Medical management of comorbid conditions is necessary to optimize
control and decrease morbidity and mortality from PVD and its
complications, e.g., leg ulcers, osteomyelitis, acute limb ischemia, and
amputation.

The Heart Outcome Prevention Evaluation (HOPE) trial demonstrated that


use of ramipril reduced cardiovascular morbidity and mortality in PVD
patients by 25%, regardless of the need for control of hypertension.
However, data are insufficient to support the use of an ACE inhibitor in
patients with PVD who are not hypertensive.
The lupus patient you are following suddenly develops a fever and notes
that her urine is dark in color. Analysis of her urine reveals the
presence of red cells. She has had some joint aching and been feeling
unwell for a few days. Her malar rash is about the same as usual for
her. Which one of the following suggests she is experiencing an
exacerbation of her disease?

a) C3 and C4 are both normal


b) C3 is low and C4 is normal
c) CH50, C3 and C4 are normal
d) C3 and C4 are low
The correct answer is D

Explanation
Serum complement levels (C3 and C4) indicate consumption of immune
complexes. Elevated or normal serum complement levels indicate that
immune complexes are being cleared. During a lupus exacerbation, immune
complexes are not cleared from the body. Therefore, decreasing serum
complement levels are consistent with exacerbation of the disease (lupus
flare).
A 58 year old male with a history of seizure attacks suffered from a
30min loss of consciousness with repetitive seizures with no recovery of
consciousness between attacks. What is the disorder this patient is
suffering from?

a) Tonic-clonic seizure
b) Absence seizure
c) Atonic seizure
d) Myoclonic seizure
e) Status epilepticus
The correct answer is E

Explanation
Answer: E
Status epilepticus is prolonged, repetitive seizure activity that lasts
more than 20 to 30 minutes, during time which the patient is
unconscious. Status epilepticus is a medical emergency with a
significantly poor outcome; it can result in death if not treated
aggressively.

Tonic-clonic seizure - specific movements of the arms and legs and/or


the face may occur with loss of consciousness. The seizure usually lasts
5 to 20 minutes.
Absence seizures cause a short loss of consciousness (just a few
seconds) with few or no symptoms
Atonic and myoclonic seizures are not characterized by loss of
consciousness.

A 72-year-old female with longstanding diabetes mellitus presents to


your office. During the review of systems, she complains of difficulty
voiding and frequent ?dribbling?. A urinalysis is negative for infection
and her post-void residual volume is 250 mL.

Which one of the following is the most likely cause of this patient‟s
urinary incontinence?

a) Excess urine output due to hyperglycemia


b) Atrophic vaginitis
c) A grade II cystocele
d) Asymptomatic bacteriuria
e) Autonomic neuropathy
The correct answer is E

Explanation
Dribbling and increased post-void residual volume (> 100 mL) are signs
of overflow incontinence. Overflow incontinence can be cause by outflow
obstruction (e.g. prostate hypertrophy, urethral constriction, fecal
impaction) or, as in this case, by detrusor muscles denervation caused
by diabetic or other neuropathies. Excess urine output from
hyperglycemia would result in frequent urination, but not urinary
retention. Atrophic vaginitis and cystoceles are usually associated with
stress incontinence. Asymptomatic bacteriuria is unlikely because the
patient does not have any evidence of infection.
A 26-year-old female nurse has had recent contact with patients with
AIDS and tuberculosis (TB), and now has a positive tuberculin skin test.
Her test was negative a year ago. She has no other medical complaints.
Testing to exclude which one of the following is most important before
starting latent TB therapy with isoniazid (INH)?

a) Active TB
b) Previous hepatitis
c) Diabetes mellitus
d) Neuropathy
e) Pregnancy
The correct answer is A

Explanation
It is essential to exclude active tuberculosis infection through a
screening chest radiograph before instituting therapy for latent
tuberculosis. Failure to do so increases the risk of developing drug
resistance. Patients who have had hepatitis in the past require
monitoring of liver function but can usually be treated. Diabetics, who
have a higher risk of developing tuberculosis, should be treated.
Pyridoxine (vitamin B) is often given to patients with neuropathy.
Pregnancy is not a contraindication for INH prophylaxis when there is
documentation of a valid indication such as conversion to a positive PPD
within the past 2 years. In addition to INH, pregnant patients should be
treated with pyridoxine. In otherwise healthy individuals, pyridoxine
supplementation is unnecessary.
A patient with end-stage renal failure is found to be anemic. You
suspect a vitamin B 12 deficiency, but testing reveals that her vitamin
B12 level is “low normal”. Which one of the following laboratory test
results would confirm a diagnosis of vitamin B 12 deficiency?

a) Low epogen
b) Low folic acid
c) High methylmalonic acid (MMA)
d) Low homocysteine
e) A normal peripheral smear
The correct answer is C

Explanation
Patients with renal failure often have normal vitamin B12 levels despite
an actual deficiency. In this situation, the clinician can order a
methylmalonic acid (MMA) level to confirm the diagnosis. Vitamin B12 is
the necessary coenzyme in the metabolism of MMA to succinylcholine.
Thus, in the absence of vitamin B 12, MMA levels increase. Additionally,
homocysteine levels would be elevated in the presence of vitamin B 12
deficiency (SOR A).
Three days after coming back from a camping trip a 25 year old man
develops abdominal pain and diarrhea that is liquid containing mucus and
blood. Stool studies show shigella. The most appropriate (non-optional)
initial treatment for this is

a) Trimethoprim-sulfamethoxazole
b) Metronidazole
c) Fluid and electrolyte replacement
d) Loperamide
e) Ceftriaxone
The correct answer is C

Explanation
Shigellosis is an acute infection of the intestine caused by Shigella.
Symptoms include fever, nausea, vomiting, and diarrhea that is usually
bloody. Diagnosis is clinical and confirmed by stool culture. Treatment
is supportive, mostly with rehydration; antibiotics (eg, ampicillin or
trimethoprim-sulfamethoxazole) are optional.

If an ampicillin and TMP-SMX resistant strain is isolated or if


susceptibility is unknown, parenteral ceftriaxone sodium,
fluoroquinolone (eg, ciprofloxacin, ofloxacin), azithromycin dihydrate
(off-label indication), or cefixime are the drugs of choice.

A 28 year old woman who is known to be HIV positive comes to the


emergency department because of increasing headaches, right-sided
weakness and disorientation for the past week. A major motor (grand mal)
seizure occurs shortly after admission. On physical examination
following the seizure, vital signs are normal. There is no nuchal
rigidity. Funduscopic examination shows papilledema. There is also right
hemiparesis and aphasia. Which of the following is the most likely
diagnosis‟

a) Meningioma
b) Meningococcal meningitis
c) Neurosyphilis
d) Toxoplasmosis
e) Tuberculous meningitis
The correct answer is D

Explanation
The patient has CNS symptoms, and is HIV-positive. Some 15% of such
patients have toxoplasmosis, making this the most likely diagnosis. The
neurological symptoms of CNS toxoplasmosis will vary depending upon the
site and severity of the infection. A glioblastoma, neurosyphilis,
meningococcal, or tubercular meningitis could cause some or all of the
symptoms, but given the patient‟s history, are much less likely than
toxoplasmosis. Obviously the next step in management would be to confirm
the diagnosis.
A healthy 25-year-old female runner presents with a complaint of right
heel pain for 2 months. The pain is most pronounced with the first steps
of the day or after periods of rest, and is located around the medial
calcaneal tuberosity.

Which one of the following is NOT recommended for acute treatment?

a) Extracorporeal shock wave therapy


b) Prefabricated insoles (heel pad)
c) Night splints
d) Corticosteroid iontophoresis
e) NSAIDs
The correct answer is A

Explanation
These findings are classic for plantar fasciitis. Treatments in the
acute phase include insoles, night splints, corticosteroid
iontophoresis, and NSAIDs. Based on current evidence, extracorporeal
shock wave therapy is recommended only after 12 months of symptoms.

In prescribing an exercise program for elderly, community-dwelling


patients, it is important to note that

a) Graded exercise stress testing should be done before beginning the


program
b) Target heart rates should be 80% of the predicted maximum
c) The initial routines can be as short as 6 minutes repeated
throughout the day and still be beneficial
d) Treadmill walking is especially beneficial to patients with neuropathy
The correct answer is C

Explanation
Initial exercise routines for the elderly can be as short as 6 minutes
in duration. Even 30 minutes per week of exercise has been shown to be
beneficial. Graded exercise testing need not be done, especially if
low-level exercise is planned. A target heart rate of 60%-75% of the
predicted maximum should be set as a ceiling. Patients with peripheral
neuropathy should not perform treadmill walking or step aerobics because
of the risk of damage to their feet.
A 19 year old female runner has a 1-week history of constant groin pain.
There is limited hip motion on flexion and internal rotation of the
right hip. Radiographs of the hip and pelvis are normal. Which one of
the following is the most likely diagnosis‟

a) Iliotibial band syndrome


b) Stress fracture of the right femoral neck
c) Osteitis pubis
d) Pelvic inflammatory disease
The correct answer is B

Explanation
Stress fractures of the femoral neck are most commonly seen in military
recruits and runners. They present with persistent groin pain, and
limited hip flexion and internal rotation. Radiographs may be normal
early. Iliotibial band syndrome also occurs in runners and presents with
stinging pain over the lateral femoral epicondyle. Osteitis pubis occurs
in distance runners and presents with pain in the anterior pelvic area
and tenderness over the symphysis pubis. Pelvic inflammatory disease is
associated with abdominal pain and fever.
A patient who has chronic peptic ulcer asks for the best method for
curing this disease. Your answer is

a) Helicobacter pylori eradication


b) Proton pump inhibitors
c) H2 blockers
d) Nissen fundoplication
The correct answer is A

Explanation
H. pylori and NSAIDs disrupt normal mucosal defense and repair, making
the mucosa more susceptible to acid. H. pylori infection is present in
50 to 70% of patients with duodenal ulcers and 30 to 50% of patients
with gastric ulcers. If H. pylori is eradicated, only 10% of patients
have recurrence of peptic ulcer disease, compared with 70% recurrence in
patients treated with acid suppression alone.

Symptoms include pain often localized to the epigastrium and relieved by


food or antacids. The pain is described as burning or gnawing, or
sometimes as a sensation of hunger. The course is usually chronic and
recurrent.

Diagnosis of peptic ulcer is suggested by patient history and confirmed


by endoscopy (EGD). Treatment of gastric and duodenal ulcers requires
eradication of H. pylori when present.

Methods of decreasing acidity include a number of drugs including proton


pump inhibitos and H2 blockers.

Nissen fundoplication is a surgical treatment for chronic GERD.


You are counseling a 45-year-old obese male regarding weight loss. The
patient has elevated triglyceride levels, low HDL levels, and stage 1
hypertension. He does not currently take medications and would like to
avoid taking medications in the future. The patient has heard good
things about low-carbohydrate diets and asks your opinion about
beginning such a plan.

A low-carbohydrate diet in a patient such as this is most likely to


result in

a) Increased LDL levels


b) Increased triglyceride levels
c) Increased blood pressure
d) Development of the metabolic syndrome
e) Better short-term weight loss than with traditional low-fat diets
The correct answer is E

Explanation
The recent public interest in low-carbohydrate diets has led to research
into the potential health benefits or risks. Emerging data are mostly
encouraging, in that these diets do not seem to cause the expected
increases in blood pressure, LDL levels, or triglyceride levels that the
medical community had first assumed. In fact, these diets seem to
consistently result in more weight loss than low-fat diets, and lead to
a more significant reduction in triglyceride levels. This patient has
symptoms of the metabolic syndrome and has a higher risk of glucose
intolerance or diabetes. Low-carbohydrate diets have been shown to
reduce insulin resistance at least as well as, if not better than,
traditional diet plans. More research will likely elicit any long-term
effects of these diets that are not currently known.

A 28 year old man with chest pain presents to the ER with increase rate
of respiration. On physical exam he has decreased breath sounds on the
right side and his trachea is slightly deviated to the left. What is the
most appropriate initial management?

a) Needle thoracentesis
b) Chest tube
c) Albuterol
d) Epinephrine
The correct answer is A
Explanation
Pneumothorax is air in the pleural space causing partial or complete
lung collapse. Pneumothorax can occur spontaneously or from underlying
pulmonary disease, trauma, or medical procedures.

Symptoms include dyspnea, pleuritic chest pain, and anxiety. Dyspnea may
be sudden or gradual in onset depending on the rate of development and
size of the pneumothorax. Pain can simulate cardiac ischemia,
musculoskeletal injury (when referred to the shoulder), or an
intra-abdominal process (when referred to the abdomen).

Physical findings classically consist of absent tactile fremitus,


hyperresonance to percussion, and decreased breath sounds on the side
with the pneumothorax. If the pneumothorax is large, the side with the
pneumothorax may be enlarged with the trachea visibly shifted to the
opposite side.

Tension pneumothorax is a medical emergency. It should be treated


immediately with needle thoracentesis, followed by placement of chest tube.
A 24-year-old female with a past history of asthma presents to the
emergency department with an asthma exacerbation. Treatment with an
inhaled bronchodilator and ipratropium (Atrovent) does not lead to
significant improvement, and she is admitted to the hospital for ongoing
management. On examination she is afebrile, her respiratory rate is
24/min, her pulse rate is 92 beats/min, and oxygen saturation is 92% on
room air. She has diffuse bilateral inspiratory and expiratory wheezes
with mild intercostal retractions. Which one of the following should be
considered in the acute management of this patient?

a) Chest physical therapy


b) Inhaled fluticasone/salmeterol (Advair)
c) Oral azithromycin (Zithromax)
d) Oral prednisone
e) Oral theophylline
The correct answer is D

Explanation
Hospital management of acute exacerbations of asthma should include
inhaled short-acting bronchodilators in all patients. Systemic
corticosteroids are recommended for all patients admitted to the
hospital. The efficacy of oral prednisone has been shown to be
equivalent to that of intravenous methylprednisolone (SOR A). Oxygen
should also be considered in most patients. Antibiotics are not
recommended in the treatment of asthma exacerbations unless there is a
comorbid infection. Inhaled ipratropium bromide is recommended for
treatment in the emergency department, but not in the hospital (SOR A).
Chest physical therapy and methylxanthines are not recommended in the
treatment of acute asthma exacerbations.
A 45-year-old obese female presents for follow-up of type 2 diabetes
mellitus and hypertension. Her medications include aspirin, lisinopril
(Prinivil, Zestril), and metformin (Glucophage). She is asymptomatic.

Laboratory results include a hemoglobin A_1c of 8.1%, a triglyceride


level of 400 mg/dL, an AST (SGOT) level of 50 U/L (N 9-25), and an ALT
(SGPT) level of 62 U/L (N 7-30).

Which one of the following is most accurate regarding this patient?


a) She most likely has alcoholic hepatitis
b) She has metformin-related liver dysfunction
c) She has nonalcoholic fatty liver disease
d) She has a totally benign hypertransaminasemia
The correct answer is C

Explanation
Nonalcoholic fatty liver disease is frequently found in obese patients
with type 2 diabetes. Although generally benign, it can progress to
cirrhosis in 25% of cases. The AST:ALT ratio is usually less than 1 in
nonalcoholic fatty liver disease and greater than 2 in alcoholic liver
disease. Metformin can be useful in correcting some of the metabolic
problems in this condition and does not cause liver dysfunction.

Which one of the following is true regarding placement of a percutaneous


endoscopic gastrostomy (PEG) tube in severely demented patients‟

a) Survival is prolonged
b) Aspiration risk is reduced
c) Pressure ulcer risk is increased
d) Patient comfort is enhanced
e) Nutritional status is improved
The correct answer is C

Explanation
The number of cases of dementia is expected to quadruple in the next 50
years. Percutaneous endoscopic gastrostomy (PEG) tubes are increasingly
placed, despite a paucity of evidence for their benefit. Bedfast,
incontinent, demented patients with PEG tubes are more likely to be
restrained, while at the same time producing more urine, thereby
increasing the risk for pressure ulcers. Neither survival nor
nutritional status has been found to be improved through the use of PEG
tubes. Since stomach contents and saliva cannot be kept out of the
airways after PEG placement, aspiration risk is not reduced.

Tube-fed patients may be denied the pleasure of eating and/or made


uncomfortable by the tube or by frequent repositioning or restraining.
No studies suggest that tube feeding makes demented patients more
comfortable.
A patient presents with pneumonia. On physical exam you find the rash
Kaposi‟s sarcoma. What is the most likely organism involved in his
pneumonia?

a) Mycobacterium Avium Complex


b) Coccidioidomycosis
c) Pneumocystis Carinii
d) Cytomegalovirus
The correct answer is C

Explanation
Kaposi‟s sarcoma is found in patients that are severely
immunocompromised (eg HIV/AIDS). In such patients who develop pneumonia,
the most likely organism is Pneumocystis Carinii.
Pneumocystis jiroveci (formerly P. carinii) pneumonia is the most
common, serious, opportunistic infection in HIV-infected patients and
has high mortality.
A 48-year-old female presents with a new onset of polyuria and
polydypsiaa. A random finger-stick blood glucose level is 320 mg/dL.

Which one of her medications listed below is most likely to be linked to


her new onset of diabetes‟

a) Atenolol (Tenormin)
b) Trazodone (Desyrel)
c) Paroxetine (Paxil)
d) Risperidone (Risperidal)
The correct answer is D

Explanation
Recent data suggests a link between atypical antipsychotics and
diabetes. In particular, studies have suggested an increased risk of
diabetes for patients on risperidone, quetiapine, clozapine, and
olanzapine. This association has been noted even in patients who do not
gain weight on the antipsychotic. The other medications listed have not
been implicated in diabetes.

Some experts recommend checking a fasting glucose level before and soon
after initiating an atypical antipsychotic. In addition, monitoring
blood glucose every 3-6 months should be considered, especially in
patients with other risk factors for developing diabetes. Patients with
preexisting diabetes may suffer worsened glycemic control when started
on an atypical antipsychotic.
A 65 year old male with known metastatic lung cancer is hospitalized
because of decreased appetite, lethargy, and confusion of 2 weeks‟
duration. Laboratory evaluation reveals the following:

Serum calcium------------------15.8 mg/dL (N 8.4-10.0)


Serum phosphorus------------------3.9 mg/dL (N 2.6-4.2)
Serum creatinine------------------1.1 mg/dL (N 0.7-1.3)
Total serum protein------------------7.3 g/dL (N 6.0-8.0)
Albumin------------------4.1 g/dL (N 3.7-4.8)

Which one of the following is the most appropriate initial management?

a) Calcitonin (Calcimar) subcutaneously


b) Pamidronate disodium (Aredia) by intravenous infusion
c) Normal saline intravenously
d) Plicamycin (Mithramycin) intravenously
e) Furosemide (Lasix) intravenously
The correct answer is C

Explanation
Initial management of hypercalcemia of malignancy calls for fluid
replacement with normal saline to correct the volume depletion that is
invariably present and to enhance renal calcium excretion. The use of
loop diuretics such as furosemide should be restricted to patients in
danger of fluid overload, since these drugs can aggravate volume
depletion and are not very effective alone in prompting renal calcium
excretion. Although intravenous pamidronate has become the mainstay for
the hypercalcemia of malignancy, it is considered only after the
hypercalcemic patient has been rendered euvolemic by saline repletion.
The same is true for the other calcium-lowering agents listed.
A 25 year old, African-Canadian female presents to your clinic with the
complaints of shortness of breath for the past 3 months.

She said that the shortness of breath was mild at the beginning but has
worsened progressively over the last 3 months. One month ago, she had
painful reddish lesions on both legs that resolved spontaneously. Her
review of systems reveals intermittent joint stiffness. She denies any
weight loss, night sweats, recent infection, travel or exposure to ill
contacts. She does not smoke but drinks alcohol on social occasions. Her
only medication is a daily multi-vitamin formula.

Vital signs are within normal limits. Breath sounds are clear to
auscultation bilaterally. The heart exam is regular in rate and rhythm
with normal S1 and S2 and no murmurs, rubs or gallops.

You order a Chest X-ray which reveals symmetric bilateral hilar


lymphadenopathy and diffuse interstitial infiltrates. Labs, including
CBC, ESR and BMP, are normal. Which of the following is the most likely
diagnosis‟

a) Atypical pneumonia
b) Fibrosing alveolitis
c) Lymphoma
d) Sarcoidosis
e) Tuberculosis
The correct answer is D

Explanation
Sarcoidosis is a non-caseating granulomatous multisystem disorder of
unknown etiology. Bilateral symmetric hilar lymphadenopathy in a young
African person is a classic presentation. It can also involve any system
in the body; pulmonary interstitial infiltrates, liver involvement, skin
or eye disease (uveitis). The patient‟s history of painful reddish
lesions on her legs is characteristic of erythema nodosum associated
with sarcoidosis.

Lymphoma is a big differential and should be ruled out with a


transbronchial biopsy; however, the African race, the symmetric pattern
of hilar lymphadenopathy and the absence of systemic symptoms make
lymphoma less likely.

In the absence of risk factors, cough, weight loss, night sweats and
fever, TB is unlikely.

Fibrosing alveolitis (idiopathic pulmonary fibrosis - IPF) is less


likely because of the presence of extrapulmonary manifestations (joint
stiffness) and the bilateral lymphadenopathy. Moreover, IPF usually
presents in older ages.

Atypical pneumonia would give fever, crackles on physical exam without


the hilar lymph nodes involvement.
You are handed a copy of this EKG to interpret:
What is your answer?

a) Left Bundle branch block & Left axis deviation


b) Left Bundle branch block & Right axis deviation
c) Right Bundle branch block & Left axis deviation
d) Right Bundle branch block & Right axis deviation
The correct answer is C

Explanation This EKG clearly shows a right bundle branch block, look
at lead V1. The axis is calculated as follows:

Look at the net amplitude in leads I and aVF, then find which quadrant
this falls in.

Both I and aVF positive = normal axis


Both I and aVF negative = axis in the Northwest Territory
Lead I negative and aVF positive = right axis deviation
Lead I positive and aVF negative = left axis deviation

Causes of a Northwest axis (no man's land): emphysema, hyperkalemia,


lead transposition, artificial cardiac pacing, ventricular tachycardia.

Causes of right axis deviation: normal finding in children and tall thin
adults, right ventricular hypertrophy, COPD, anterolateral MI, left
posterior hemiblock, pulmonary embolus, Wolff-Parkinson-White syndrome,
ASD, VSD.

Causes of left axis deviation: left anterior hemiblock, Q waves of


inferior myocardial infarction, artificial cardiac pacing, hyperkalemia
and tricuspid atresia.
A 38 year old epileptic man, brought in unconscious in the middle of the
night, has a shallow respiration at a rate > 40/minute and is cyanotic.
His pulse is 130/minute and his blood pressure is 80/50 mm Hg. The first
thing you should do is
a) Call a close relative and inquire about any new drug intake
b) Install a nasogastric tube
c) Administer 35-40 % FiO2 by nasal canula
d) Perform endotracheal intubation
e) Order EKG and chest x-ray and reassess
The correct answer is D

Explanation
ABC is a mnemonic for memorizing essential steps in dealing with an
unconscious or unresponsive patient. It stands for Airway, Breathing and
Circulation.

If the patient's airway is blocked, oxygen cannot reach the lungs and so
cannot be transported round the body in the blood. Ensuring a clear
airway is the first step in treating any patient. Common problems with
the airway involve blockage by the tongue or vomit.

Initial opening of the airway is often achieved by a "head tilt ? chin


lift" or jaw thrust technique, although further maneuvers such as
intubation may be necessary.

The patient is next assessed for breathing. Common findings during an


assessment of breathing may include normal breathing, noisy breathing,
gasping or coughing. The rescuer proceeds to act on these based on
his/her training. Generally at this point it will become clear whether
or not the casualty needs supportive care (such as the recovery
position) or rescue breathing.

Once oxygen can be delivered to the lungs by a clear airway and


efficient breathing, there needs to be a circulation to deliver it to
the rest of the body. This can be assessed in a number of ways,
including a pulse check, ECG analysis, or capillary refill time.

A 60-year-old female is admitted to the hospital with pneumonia 1 week


after her discharge following elective colorectal surgery. Her initial
stay was 5 days and she had no complications. She had no signs of
infection until 2 days ago when she developed a temperature of 39.1°C
(102.4°F), a cough with yellow sputum, and hypoxia. She has no abdominal
pain or diarrhea. Her pulse rate is slightly elevated to 96 beats/min,
and her blood pressure is unchanged from baseline. A chest radiograph
confirms a left lingular infiltrate. Methicillin-resistant
Staphylococcus pneumonia is rare in this institution. Of the following
antibiotic regimens, which one would be the best initial treatment for
this patient?

a) Ampicillin/sulbactam (Unasyn)
b) Ceftazidime sodium (Fortaz) and gentamicin
c) Ceftriaxone (Rocephin) and azithromycin (Zithromax)
d) Clarithromycin (Biaxin)
e) Levofloxacin (Levaquin)
The correct answer is B

Explanation
This patient has a significant pneumonia that requires the initiation of
empiric antibiotics. It is important to remember that because this
patient was recently in the hospital, the usual coverage for
community-acquired pneumonia is not adequate. Health care?associated
pneumonia is more likely to involve severe pathogens such as Pseudomonas
aeruginosa, Klebsiella pneumoniae, and Acinetobacter species.
Methicillin-resistant Staphylococcus aureus also is a consideration,
depending on local prevalence. Of the antibiotic regimens listed,
ceftazidime and gentamicin is the only choice that covers these organisms.
A 25 year old white male who has a poorly controlled major seizure
disorder and a 6-week history of recurrent fever, anorexia, and
persistent, productive coughing visits your office. On physical
examination he is noted to have a temperature of 38.3°C (101.0°F), a
respiratory rate of 16/min, gingival hyperplasia, and a fetid odor to
his breath. Auscultation of the lungs reveals rales in the mid-portion
of the right lung posteriorly. Which one of the following is most likely
to be found on a chest radiograph?

a) Sarcoidosis
b) Miliary calcifications
c) A lung abscess
d) A right hilar mass
e) A right pleural effusion
The correct answer is C

Explanation
Anaerobic lung abscesses are most often found in a person predisposed to
aspiration who complains of a productive cough associated with fever,
anorexia, and weakness. Physical examination usually reveals poor dental
hygiene, a fetid odor to the breath and sputum, rales, and pulmonary
findings consistent with consolidation. Patients who have sarcoidosis
usually do not have a productive cough and have bilateral physical
findings. A persistent productive cough is not a striking finding in
disseminated tuberculosis, which would be suggested by military
calcifications on a chest film. The clinical presentation and physical
findings are not consistent with a simple mass in the right hilum nor
with a right pleural effusion.
A patient, who ate a lot of meat at a barbeque, develops diarrhea and
muscle pain. His symptoms have been present for two weeks now. His
initial complete blood count shows eosinophilia. The best investigation
to be done to reach the diagnosis‟

a) Muscle biopsy
b) Stool culture
c) Stool gram stain
d) Stool ova and parasites
The correct answer is A

Explanation
Trichinosis is infection with Trichinella spiralis or related
Trichinella species. Symptoms include initial GI irritation followed by
periorbital edema, muscle pain, fever, and eosinophilia.

Humans become infected by eating raw, undercooked, or processed meat


from infected animals, most commonly pigs, wild boar, or bear.

Diagnosis is clinical and with serologic tests. Muscle biopsy may be


diagnostic but is seldom necessary. Treatment is with mebendazole or
albendazole plus prednisone if symptoms are severe.
No specific tests to diagnose the intestinal stage are available. After
the 2nd weeks of infection, a muscle biopsy may disclose larvae and
cysts but is seldom necessary. Diffuse inflammation in muscle tissue
indicates recent infection.
In the course of DKA, serum potassium levels

a) Remain unaffected
b) Can appear normal but total body potassium may actually be low
c) Can appear normal but total body potassium may actually be high
d) Will naturally be corrected by insulin administration
The correct answer is B

Explanation
In the course of DKA, serum potassium levels can appear normal but total
body potassium may actually be low. The reason is that in states of
acidosis, protons exchange for K ions in the cells. As the protons enter
the cells, K ions will leave the cells and enter the bloodstream. This
will make it seem like the serum body potassium is normal.
For a 52 year old sedentary, normotensive, non-smoking white male who is
not overweight, drinks 60 mL of whiskey per day and plays golf
occasionally, which one of the following conditions is most likely to
cause death within the next 10 years‟

a) Motor vehicle accident


b) Cerebrovascular disease
c) Suicide
d) Cirrhosis of the liver
e) Ischemic heart disease
The correct answer is E

Explanation
In a male over the age of 50 with low risk factors, the most common
cause of morbidity is due to ischemic heart disease.

Ischemic heart disease (IHD), or myocardial ischemia, is a disease


characterized by reduced blood supply to the heart muscle, usually due
to coronary artery disease (atherosclerosis of the coronary arteries).
Its risk increases with age, smoking, hypercholesterolemia (high
cholesterol levels), diabetes, hypertension (high blood pressure) and is
more common in men and those who have close relatives with ischemic
heart disease.

It is the most common cause of death in most Western countries, and a


major cause of hospital admissions. There is limited evidence for
population screening, but prevention (with a healthy diet and sometimes
medication for diabetes, cholesterol and high blood pressure) is used
both to prevent IHD and to decrease the risk of complications.

Exposure to lead can cause all of the following, except

a) Anemia
b) Cirrhosis of the liver
c) Paresis
d) Abdominal pain
e) Porphyrinuria
The correct answer is B
Explanation
Chronic lead poisoning in children may cause mental retardation, seizure
disorders, aggressive behavior disorders, developmental regression,
chronic abdominal pain, and anemia. Cirrhosis of the liver is not
associated with lead.
A positive Lachman test indicates injury to the

a) medial collateral ligament


b) posterior cruciate ligament
c) medial meniscus
d) anterior cruciate ligament
e) lateral collateral ligament
The correct answer is D

Explanation
The Lachman test is performed with the knee flexed to 25°?30° while the
examiner grasps the distal femur in one hand and the proximal tibia in
the other. While the femur is held stationary, the tibia is pulled
anteriorly, using a „shucking? action. If a distinct end point is
reached, as if a piece of loose rope suddenly becomes taut, the test is
negative or normal. A soft or indistinct end point, as if stretching an
elastic band, is a positive or abnormal test that indicates a ruptured
anterior cruciate ligament. In this case, the anterior drawer test would
also be positive, but it is not as specific as the Lachman test.
Injuries to the other structures listed are diagnosed using other
maneuvers, and are not associated with a positive Lachman test.
The peripheral blood smear of a severely anemic patient reveals oval
macrocytes, hypersegmented neutrophils, and decreased platelets. The
most likely cause of the anemia is

a) A red cell membrane protein defect


b) Vitamin B12 or folate deficiency
c) An amino acid substitution in the globin chain
d) Marrow hypoplasia
e) Iron deficiency
The correct answer is B

Explanation
Hypersegmented neutrophils and macro-ovalocytes strongly suggest
megaloblastic anemia. Megaloblastic anemia is an anemia (of macrocytic
classification) which results from inhibition of DNA synthesis in red
blood cell production. It is often due to deficiency of vitamin B12
and/or folic acid.
Oral preparations of testosterone available (e.g. methyltestosterone,
fluoxymesterone) are infrequently prescribed because of their
association with substantial

a) Cardiotoxicity
b) Gastrointestinal toxicity
c) Hepatotoxicity
d) Nephrotoxicity
e) Neurotoxicity
The correct answer is C
Explanation
Oral preparations of testosterone are infrequently prescribed because of
their association with substantial hepatotoxicity, including the
development of benign and malignant neoplasms.

A 67 year old woman with a history of heart failure comes to see you
today. Her brother is a physician and he told her to ask you to put her
on a medication which will reduce both preload and afterload. Which of
the following would you choose if you followed her brother's request?

a) Hydralazine
b) Isosorbide dinitrate
c) Furosemide
d) Dobutamine
e) Prazosin
The correct answer is E

Explanation
Drugs that reduce both prelod and afterload include nitroprusside,
prazosin, ACE inhibitors and angiotensin II receptor blockers. These
agents reduce ventricular filling pressures by facilitating venous
pooling and improve cardiac output by reducing total peripheral resistance.

Hydralazine and minoxidil reduce afterload by relaxing arteriolar smooth


muscle.
The venodilators and diuretics are used to decrease preload. The
venodilators include nitroglycerine and isosorbide dinitrite.
Diuretics like furosemide, bumetanide and hydrochlorothiazide are used
to reduce blood volume, thus reducing preload.
Dobutamine increases myocardial contractility with no substantial
alteration of total peripheral resistance.
A 35 year old male with epigastric pain and positive serology for H.
pylori was treated with triple therapy. Several months later, he was
asymptomatic but his serology was positive. Which one of the following
is the best management strategy?

a) Repeat the triple therapy


b) Perform endoscopy
c) Repeat serology in three months
d) Test his spouse for H. pylori
e) Reassure and counsel
The correct answer is E

Explanation
H. pylori is a common gastric pathogen that causes gastritis, peptic
ulcer disease, gastric adenocarcinoma, and low-grade gastric lymphoma.
Infection may be asymptomatic or result in varying degrees of dyspepsia.
Diagnosis is by urea breath test and testing of endoscopic biopsy samples.

Laboratory and office-based serologic assays for antibodies to H. pylori


have sensitivity and specificity of > 85% and are considered the
noninvasive tests of choice for initial documentation of H. pylori
infection. However, because qualitative assays remain positive for up to
3 years after successful treatment and because quantitative antibody
levels do not decline significantly for 6 to 12 months after treatment,
serologic assays are not usually used to assess cure.
The most common cause of chronic cough in adults is

a) gastroesophageal reflux disease


b) asthma
c) upper airway cough syndrome (postnasal drip)
d) nonasthmatic eosinophilic bronchitis
e) ACE inhibitor use
The correct answer is C

Explanation
Causes of chronic cough in the adult patient are often somewhat obscure,
as the onset may be vague and there are no consistently reliable
defining characteristics. Further complicating this diagnostic challenge
is the possibility that the cough may be due to multiple concurrent
conditions that must each be addressed before the cough will resolve.
Each of the options listed may cause chronic cough and must be considered.

In order, the three most common causes of chronic cough are upper airway
cough syndrome (UACS), asthma, and gastroesophageal reflux disease.
Nonasthmatic eosinophilic bronchitis is frequent enough to warrant
consideration as well. A 2001 decision analysis provides support for the
strategy of treating chronic cough empirically. Since the most common
cause for chronic cough is UACS (with an assumed prevalence of 44%) due
to rhinosinus conditions, an empiric trial of a first-generation
antihistamine/decongestant is a reasonable option. Further treatment
choices are based on the response to this regimen.
A 30 year old woman complains of episodic headache, sweating,
palpitations, tingling sensation in her hands, shortness of breath, and
severe anxiety. Thorough medical workup reveals no pathologic condition.
During an episode of these symptoms, chemical analysis of the serum will
probably reveal which one of the following?

a) Decreased chloride
b) Increased urea (BUN)
c) Decreased protein
d) Increased serum amylase
e) Increased pH
The correct answer is D

Explanation
A pheochromocytoma is a catecholamine-secreting tumor of chromaffin
cells typically located in the adrenals. It causes persistent or
paroxysmal hypertension.

The classic triad of symptoms in patients with a pheochromocytoma


consists of episodic headache, diaphoresis, and tachycardia. Other
common symptoms and signs include postural hypotension, cold and clammy
skin, severe headache, angina, palpitations, nausea, vomiting,
epigastric pain, visual disturbances, dyspnea, paresthesias,
constipation, and a sense of impending doom.

Diagnosis is by measuring catecholamine products in blood or urine.


Imaging tests, especially CT or MRI, help localize tumors. Treatment
involves removal of the tumor when possible. Drug therapy for control of
BP includes a-blockade, possibly combined with ß-blockade.
Ectopic amylase production by lung, ovary, pancreas, and colon
malignancies; pheochromocytoma; thymoma; multiple myeloma (increased
salivary amylase); and breast cancer (increased pancreatic amylase) are
miscellaneous causes of hyperamylasemia.
A 60 year old diabetic male presents to the ER with fever and painful
swelling of his right leg that began few hours ago.

He is taking metformin and glyburide for his diabetes since he was


diagnosed 5 years ago and admits to medication non-compliance.

Vital signs are within normal limits except for a fever of 39 C.


Examination of the leg shows raised borders of a well-demarcated area of
erythema that is warm and extremely painful to touch. Cracks in the skin
of the toe webs are noted on the examination of his feet.

Which of the following is the most likely causative organism for the
patient‟s condition?

a) Actinomyces israeili
b) Clostridium perfringens
c) Dermatophytes
d) Staphylococcus aureus
e) Streptococcus pyogenes (groupA)
The correct answer is E

Explanation
Erysipelas, most commonly caused by group A- beta-hemolytic
streptococci, is the most likely diagnosis of this patient. The
characteristic finding of erysipelas is a sharply demarcated,
erythematous, edematous tender skin lesion with raised borders. It is
frequently seen in diabetics where a tenia pedis infection is the usual
portal of entry of the infection.

Staphylococcus aureus is a rare cause of erysipelas. Remember that staph


aureus are coagulase positive organisms that secrete coagulase enzyme
which limits the spread of the infection in the dermis.

Dermatophytes are the cause of the toe web infection and not the erysipelas.

Actinomyces israelii is the causative agent of cervicofacial infections


characterized by the sinuses discharging sulphur granules.

Clostridium perfringens is the causative agent of gas gangrene.


A 55 year old male smoker undergoes a physical examination and
laboratory workup for a life insurance application. History and physical
examination, including rectal examination, are normal. Blood count is
also normal, but urinalysis reveals the presence of blood in the urine
(> 20 RBC/HPF). Cystoscopy is negative. Abdominal ultrasound reveals a 5
cm solid mass in the upper pole of the left kidney, confirmed by
computerized tomography scan. Which one of the following is the most
appropriate management?

a) Computerized tomography scan guided percutaneous biopsy


b) Retrograde pyelogram
c) Repeat computerized tomography scan in 6 months
d) Intravenous pyelogram
e) Surgical resection
The correct answer is E

Explanation
In addition to taking a complete medical history and performing a
physical exam, blood and urine tests a CT and biopsy is likely needed in
the diagnosis. Computerized tomography (CT) or magnetic resonance
imaging (MRI) scan. CT scans use computers to create more detailed
images than those produced by conventional X-rays. MRI scans use
magnetic fields and radio waves to generate cross-sectional pictures of
your body.

Biopsies are commonly performed on tumors that develop in a ureter or in


the kidney pelvis, the area at the center of the kidneys where urine
collects. During a biopsy, a sample of tissue is removed and examined
under a microscope. It's the only way to confirm the presence of cancer.
A solid kidney tumor, on the other hand, is often removed without a
biopsy because these tumors are almost always cancerous and because a
needle biopsy may spread cancer cells outside the biopsied kidney.

A 65 year old man presents with low back pain. His labs show anemia and
hypercalcemia. You suspect multiple myeloma. What is the most
appropriate test to confirm the diagnosis‟

a) WBC count
b) Bone marrow aspiration
c) Platelet count
d) X-rays
The correct answer is B

Explanation
Multiple myeloma is a malignancy of plasma cells that produce monoclonal
immunoglobulin and invade and destroy adjacent bone tissue. Common
manifestations include bone pain, renal insufficiency, hypercalcemia,
anemia, and recurrent infections.

Diagnosis requires demonstration of M-protein (sometimes present in


urine and not serum) and either lytic bone lesions, light-chain
proteinuria, or excessive marrow plasma cells. A bone marrow biopsy is
usually needed. Specific treatment includes conventional chemotherapy.
An 80 year old woman is brought to the office for the first time by her
son because she has signs of mildly decreasing mental function. She is
having increasing trouble reading, writing and watching television. She
has mild, stable angina pectoris and she had an uncomplicated myocardial
infarction 8 years ago. Physical examination now is normal except for
corrected visual acuity of 20/200 O.U., which appears to be caused by
cataracts. Mini-mental state test score is 29 out of 30. Which of the
following is the most correct statement about this patient's condition?

a) Her daily activities would probably improve if she had cataract


extraction with lens implantation
b) Her diminished mental status is a contraindication for a cataract
operation
c) Her history of cardiac disease and advanced age are
contraindications for a cataract operation
d) Her mental status should be reevaluated in 1 year
e) You need more information to decide whether she would be helped by a
cataract operation
The correct answer is A

Explanation
This question clearly describes the patient who has stable medical
problems, which would not be a contraindication to cataract repair under
local anesthesia. Her degree of visual impairment would certainly
interfere with daily activities, and suggesting surgical correction
clearly indicates that the physician is aware of the importance of the
patient‟s quality of life. Her possibly diminished mental status or
history of cardiac disease are not contraindications to the surgery. And
there is no additional information needed to decide if she would benefit
from the surgery. While re-evaluating her mental status in one year may
be appropriate, it does nothing to improve her quality of life at the
present time.
Which one of the following causes of anemia is associated with a normal
red cell distribution width?

a) Vitamin B 12 deficiency
b) Iron deficiency
c) Beta-Thalassemia trait
d) Sideroblastic anemia
e) Myelofibrosis
The correct answer is C

Explanation
Red cell distribution width (RDW) is particularly useful in
distinguishing anemic disorders, especially iron deficiency anemia (high
RDW, normal to low mean corpuscular volume) and uncomplicated
heterozygous thalassemia (normal RDW, low mean corpuscular volume).
A 58 year old male with a history of seizure attacks suffered from a
30min loss of consciousness with repetitive seizures with no recovery of
consciousness between attacks. What is the first choice treatment for
his condition?

a) Diazepam
b) Phenytoin
c) Phenobarbital
d) Carbamazepine
e) Valproic acid
The correct answer is A

Explanation
Status epilepticus (SE) is prolonged, repetitive seizure activity that
lasts more than 20 to 30 minutes, during time which the patient is
unconscious. Status epilepticus is a medical emergency with a
significantly poor outcome; it can result in death if not treated
aggressively.

A useful algorithm for the treatment of SE is as follows:


>Start an IV line, administer a 50-mL bolus of 50% dextrose IV and 100
mg of thiamine, then start the anticonvulsant. In some settings where
drug intoxication might be likely, consider also adding naloxone at
0.4-2.0 mg IV to the dextrose bag.
>Administer diazepam (0.15 mg/kg) or lorazepam (0.1 mg/kg) IV over 5
minutes, followed preferably by fosphenytoin (15-20 mg phenytoin
equivalents PE/kg at a rate not to exceed 150 mg PE/min) or phenytoin
(18-20 mg/kg at a rate not to exceed 50 mg/min). Never mix phenytoin
with a 5% dextrose solution; put it in a normal saline solution to
minimize the risk of crystal precipitation.
>Intubate if necessary, and control hyperthermia.
>If seizures continue after 20 minutes, give additional fosphenytoin (10
mg PE/kg IV) or phenytoin (10 mg/kg IV). Aim for a total serum phenytoin
level of about 22-25 microg/mL.
>If seizures continue after 20 minutes, give phenobarbital (15 mg/kg
IV). Several other alternatives to phenobarbital, such as valproate,
levetiracetam, propofol, midazolam, or pentobarbital, are shorter acting
than phenobarbital and allow for periodic neurologic assessments.
>If seizures continue, consider administering general anesthesia with
medications such as propofol, midazolam, or pentobarbital titrated by IV
drip to a burst-suppression pattern in the EEG trace.

A 45 year old woman presents with pruritus and progressive jaundice,


with increased liver function tests and total bilurubin. What is the
next investigation you should do?

a) Ultrasound
b) Serum lead level
c) ERCP
d) Anti-mitochondrial antibodies
e) CT of Abdomen
The correct answer is D

Explanation
Primary biliary cirrhosis (PBC) is inflammation with progressive
scarring of the bile ducts in the liver. Eventually, the ducts are
blocked, the liver becomes scarred, and liver failure develops. PBC is
most common among women aged 35 to 70, although it can occur in men and
women of any age.

An autoimmune reaction is the likely cause of primary biliary cirrhosis.


Itchiness, fatigue, a dry mouth and eyes, and jaundice are common. A
blood test to measure antimitochondrial antibodies is highly accurate
for the diagnosis. Treatment focuses on relieving symptoms, slowing
liver damage, and treating complications.

[Note: If you think about the differential diagnosis for this patient,
it would certainly include PBS, autoimmune hepatitis, PSC (or
cholangiocarcinoma). First, you want to do the easiest/fastest tests
that would give you more info and help you come up with the correct
diagnosis: this would be a simple blood test. You should order liver
enzymes and antibodies. If this does not establish the diagnosis then
you order an ultrasound followed by ERCP (as being the most invasive
diagnostic test among the ones mentioned above)]
A 60 year old female presents to the emergency department complaining of
shortness of breath.

She arrived from Australia yesterday after a long visit at her


daughter‟s. She has developed swelling of her left leg yesterday evening
followed by shortness of breath that began today. Her past medical
history is unremarkable.
Vital signs are significant for pulse of 120/min and a RR of 24/min.
Physical exam is non contributory except for the swollen tender left
leg. Chest x-ray is normal. You ordered the arterial blood gases, which
of the following findings would suggest pulmonary embolism?

a) Decreased A-a gradient


b) Decreased PCO2 level
c) Increased PCO2 level
d) Normal PO2
e) PH less than 7.38
The correct answer is B

Explanation
The patient is most likely having pulmonary embolism. The shortness of
breath that follows the leg swelling with the associated tachycardia and
tachypnea suggests the diagnosis of PE. A low PCO2 (less than 40mmHg) is
usually seen due to the associated tachypnea.

The A-a gradient is usually increased due to the ventilation/perfusion


mismatch.

PO2 is almost always decreased in PE because of the low perfusion i.e.


less oxygenation.

Respiratory alkalosis and not acidosis is usually seen in PE.


A 45 year old woman presents with decreased visual acuity, no pain and
no tears. What is the initial exam to do?

a) Tonometry
b) Refraction testing
c) Angiography
d) Neurologic test
e) Rheumatoid factor screening
The correct answer is A

Explanation
Tonometry is the procedure performed to determine the intraocular
pressure (IOP), the fluid pressure inside the eye. It is an important
test in the evaluation of ocular conditions such as glaucoma as well as
conditions such as phthisis bulbi, and iritis.
This patient could be suffering from cataract or error of refraction but
open angle glaucoma is also painless and given the seriousness of this
condition - it should be excluded first.

Which one of the following is an indicator of low risk for streptococcal


infection in a patient with acute pharyngitis‟

a) Anterior cervical lymphadenopathy


b) Cough
c) Fever
d) Headache
e) Soft palate petechiae
The correct answer is B

Explanation
Indicators of a low risk streptococcal infections include the absence of
fever in patients not using antipyretic agents, the absence of
pharyngeal erythema, and the presence of obvious manifestations of the
common cold. Uncharacteristic symptoms include coryza, hoarseness, cough
and diarrhea.
A 32-year-old white male teacher is seen for a paroxysmal cough of 5
days‟ duration. He tells you that a student in his class was diagnosed
with pertussis 3 weeks ago. Which one of the following would be the best
treatment?

a) Amoxicillin
b) Azithromycin (Zithromax)
c) Cephalexin (Keflex)
d) Ciprofloxacin (Cipro)
e) Doxycycline
The correct answer is B

Explanation
Macrolides are considered first-line therapy for Bordetella pertussis
infection. Trimethoprim/sulfamethoxazole is considered second-line therapy.

An asthmatic patient presents to the ER with dyspnea. He used his


inhaled salbutamol with no improvement. Physical exam shows dyspnea with
rhonchi in the upper chest. What is the most appropriate management?

a) IV fluids and aminophylline


b) Oral Corticosteroids
c) Endotracheal intubation
d) IV methylprednisolone
e) Ipratropium
The correct answer is B

Explanation
In the treatment of an asthma exacerbation systemic corticosteroids
(prednisone, prednisolone, methylprednisolone) should be given for all
acute exacerbation; they are unnecessary for patients whose PEF
normalizes after 1 or 2 bronchodilator doses.

IV methylprednisolone can be given if an IV line is already in place and


can be switched to oral dosing whenever necessary or convenient. The
bioavailability of orally and parenterally administered steroids is the
same, and numerous randomized double-blind trials have demonstrated this
equivalence. A primary reason to use intravenous corticosteroids is the
adage to avoid medications by mouth when intubation is imminent.
However, for most ED patients with acute asthma, the use of oral
corticosteroids obviates placement of an intravenous line.

A 52 year old man with a long history of alcohol use comes to the
emergency room of a general hospital. He is confused and on examination
is noted to have palsies of conjugate gaze, horizontal nystagmus, and
ataxia. Which one of the following is the most likely diagnosis‟

a) Conversion reaction
b) Alcohol intoxication
c) Alcohol hallucinosis
d) Delirium tremens
e) Wernicke's encephalopathy
The correct answer is E

Explanation
Wernicke's encephalopathy is a disorder characterized by acute onset of
confusion, nystagmus, partial ophthalmoplegia, and ataxia due to thiamin
deficiency. Diagnosis is primarily clinical. The disorder may remit with
treatment, persist, or degenerate into Korsakoff's psychosis. Treatment
consists of thiamine and supportive measures.

Wernicke's encephalopathy results from inadequate intake or absorption


of thiamine plus continued carbohydrate ingestion. Severe alcoholism is
a common underlying condition. Excessive alcohol intake interferes with
thiamine absorption from the GI tract and hepatic storage of thiamine;
the poor nutrition associated with alcoholism often precludes adequate
thiamine intake.

Wernicke's encephalopathy may also result from other conditions that


cause prolonged undernutrition or vitamin deficiency (eg, recurrent
dialysis, hyperemesis, starvation, gastric plication, cancer, AIDS).
A 48 year old with a 20 year history of severe Crohn's disease requiring
in-patient hospitalization, complains of back pain and stiffness
especially in the morning, that usually gets better after a warm shower.
Which one of the following is the most likely cause of the pain?

a) Osteomalacia
b) Ankylosis spondylitis
c) Osteoporosis
d) Avascular necrosis
e) Osteoarthritis
The correct answer is B

Explanation
The extra-intestinal manifestations of inflammatory bowel disease are
numerous. Crohn's disease is associated with a type of rheumatologic
disease known as seronegative spondyloarthropathy. This group of
diseases is characterized by inflammation of one or more joints
(arthritis) or muscle insertions (enthesitis). The arthritis can affect
larger joints such as the knee or shoulder or may exclusively involve
the small joints of the hand and feet.
The arthritis may also involve the spine, leading to ankylosis
spondylitis if the entire spine is involved or simply sacroiliitis if
only the lower spine is involved
Which one of the following studies is the diagnostic procedure of choice
to detect giant-cell arteritis‟

a) Biopsy of the temporalis muscle


b) Biopsy of the temporal artery
c) CT scan of the temporal artery
d) MRI of the temporal artery
e) An erythrocyte sedimentation rate
The correct answer is B

Explanation

A definite diagnosis of giant-cell arteritis requires histomorphologic


examination of arterial tissue. Considering the lack of specificity of
the clinical and laboratory markers and the important therapeutic
implications of prolonged corticosteroid therapy, histologic
confirmation of the diagnosis should be sought whenever possible. Thus,
biopsy of the temporal artery would be most efficacious in diagnosing
giant-cell arteritis.
A woman presents with complaints of muscle weakness. Physical exam shows
the following rash which you recognize as goittrons papules. See picture:

What is the most likely diagnosis‟

a) Dermatomyositis
b) Mixed Connective Tissue disease
c) Scleroderma
d) Polymyalgia rheumatica
The correct answer is A

Explanation
In dermatomyositis rashes tend to appear at the same time as muscle
weakness and other symptoms. A shadowy-red or purplish rash (heliotrope
rash) can appear on the face. A reddish-purple swelling around the eyes
is characteristic. Another rash (goittrons papules), which may be scaly,
smooth, or raised, may appear almost anywhere on the body but is
especially common on the knuckles and sides of the hands. The nail beds
may redden. When the rashes fade, brownish pigmentation, scarring,
shriveling, or pale depigmented patches may develop on the skin.

Criteria used to make the diagnosis of dermatomyositis includes: muscle


weakness at the shoulders or hips, a characteristic rash, increased
blood levels of certain muscle enzymes (especially creatine kinase),
characteristic changes in muscle tissue obtained by biopsy and observed
under a microscope, and abnormalities in the electrical activity of
muscles measured by electromyography

Generally, a corticosteroid, usually prednisone, given by mouth in high


doses slowly improves strength and relieves pain and swelling,
controlling the disease.
A 20 year old student consults you because of diarrhea of 10 weeks
duration. His illness started in February, two weeks after a skiing trip
in the Rocky Mountains, with nausea, anorexia and abdominal discomfort.
Shortly thereafter, his stools became frequent and watery. Physical
examination is unremarkable. Complete blood count, including
differential, is normal. Stool examination shows absence of fecal
leukocytes. Which one of the following is the most likely cause of his
condition?

a) Giardia lamblia
b) Salmonella
c) Yersinia enterocolitica
d) Enterotoxigenic Escherichia coli
e) Campylobacter jejuni
The correct answer is A

Explanation
Giardiasis is infection with the flagellated protozoan Giardia lamblia.
Infection can be asymptomatic or cause symptoms ranging from
intermittent flatulence to chronic malabsorption. Waterborne
transmission is the major source of giardiasis. Common in campers.

Diagnosis is by identifying the organism in fresh stool or duodenal


contents or by assays of Giardia antigen in stool. Treatment is with
metronidazole, tinidazole, or nitazoxanide.

Which one of the following is true regarding the use of alendronate


(Fosamax) for the treatment of osteoporosis in men?

a) Its effectiveness is similar to that seen in women


b) It is ineffective in patients with Paget‟s disease
c) It is contraindicated in patients taking NSAIDs
d) It causes a decrease in height
The correct answer is A

Explanation
In men with hypogonadism, testosterone therapy has limited efficacy, and
the efficacy of other therapies for osteoporosis in men has not been
evaluated. Bisphosphonates, including alendronate, are indicated for
treatment of Paget‟s disease. They decrease bone pain, decrease bone
resorption, and increase new bone formation. The proportion of men whose
height decreased by at least 10 mm over a 2-year period was 13% in the
placebo group and 3% in the alendronate group. The frequency of adverse
gastrointestinal effects in the two groups was similar despite the fact
that 36% of the men in the placebo group and 41% of those in the
alendronate group reported taking NSAIDs during the study.
The most common cause of chronic paronychia is

a) Straphylococcus aureus
b) Streptococcus
c) Candida albicans
d) Pseudomonas
The correct answer is C

Explanation
Candida causes 95% of cases of chronic paronychia. The other organisms
are more likely to cause acute paronychia.
A 62 year old female presents to the emergency department complaining of
right leg and knee pain. The pain has progressed over the past 2 weeks
so that now she is unable to bear weight on her right leg. She has a
history of a recent pulmonary embolus and is on warfarin (Coumadin). Her
INR is therapeutic. She is allergic to aspirin, NSAIDs and penicillin.
On examination she is afebrile. Her pulse rate is 102 beats/min, her
blood pressure is 158/96 mm Hg, and her respiratory rate is 14/min. Her
right knee is erythematous, warm, and tender on palpation over the
entire knee joint, and a large effusion is noted. Arthrocentesis reveals
numerous WBCs and negatively birefringent crystals.

What is the best initial treatment for the patient‟s knee problem?

a) Ceftriaxone (Rocephin) intravenously, 1.0 g every 24 hr


b) Prednisone orally, 10 mg/day
c) Colchicine, 0.6 mg every 1?2 hr, up to 4.8 mg total
d) Indomethacin (Indocin) orally, 50 mg every 8 hr
e) Allopurinol (Zyloprim) orally, 100 mg/day
The correct answer is C

Explanation
This patient is having an acute gout attack. While indomethacin is a
good choice for treating an acute gout attack, this patient should not
take indomethacin because of the increased risk of bleeding created by
warfarin. Colchicine would be the best agent in this case. The dosage is
titrated up until symptoms are relieved, side effects occur, or the
maximum dose of 4.8 mg in 24 hours is reached. There is no evidence that
corticosteroids are an effective treatment for acute gout. Allopurinol
should not be used in an acute gout attack. Ceftriaxone is not an
appropriate treatment for gout.
A 35 year old male consults you about a vague chest pain he developed
while sitting at his desk earlier in the day. The pain is right-sided
and was sharp for a brief time when it began, but it rapidly subsided.
There was no hemoptysis and the pain does not seem pleuritic. His
physical examination, EKG, and oxygen saturation are unremarkable. A
chest film shows a 10% right pneumothorax. Which one of the following is
true in this situation?

a) He is likely to be an overweight smoker with a chronic cough


b) Rupture of subpleural bullae would be an unlikely cause of his problem
c) Outpatient observation with a repeat chest radiograph in 24 hours is
indicated
d) A chest tube should be placed expeditiously
e) After treatment his probability of recurrence is less than 15%
The correct answer is C

Explanation
The majority of patients presenting with spontaneous pneumothorax are
tall, thin individuals under 40 years of age. Most do not have
clinically apparent lung disease, and the chest pain is sometimes
minimal at onset and may resolve within 24 hours even if untreated.
Patients with small pneumothorax involving less than 15% of the
hemithorax may have a normal physical examination, although tachycardia
is occasionally noted. The diagnosis is confirmed by chest radiographs.
Studies of recurrence have found that an average of 30% of patients will
have a recurrence within 6 months to 2 years. The treatment of an
initial pneumothorax of less than 20% may be monitored if a patient has
few symptoms. Follow-up should include a chest radiograph to assess
stability at 24-48 hours. Indications for treatment include progression,
delayed expansion, or the development of symptoms. The majority of
patients with spontaneous pneumothoraces, and perhaps almost all of
them, will have subcutaneous bullae on a CT scan.
A man develops the following rash. See picture:

You diagnose him with an intertriginous candida infection. All of the


following treatments are appropriate, except

a) Topical clotrimazole
b) Topical ketoconazole
c) Topical miconazole
d) Systemic amphotericin
The correct answer is D

Explanation
Candidiasis of intertriginous skin occurs as poorly marginated, bright
red plaques with satellite papules and pustules scattered around the
periphery of the main lesion. Candidiasis is most commonly seen in the
groin but also occasionally occurs in the axillary folds and
interdigital web spaces.

In men, groin lesions are located in the inguinal-scrotal fold with


later involvement of the inner thighs, gluteal cleft, and scrotum. In
women, Candida vaginitis usually precedes involvement of the labia
minora and majora. Spread subsequently occurs to the inner thighs and
gluteal cleft. Pruritus is generally present.

A clinical diagnosis can be confirmed by KOH preparations or cultures.


These studies are more likely to be positive if they are performed on
material recovered from intact pustules.

Topical imidazoles (clotrimazole, ketoconazole, miconazole) applied


twice a day are effective in the treatment of candidiasis. In those
patients for whom inflammation is prominent or pruritus is severe, the
addition of a topically applied steroid such as hydrocortisone reduces
the discomfort and shortens the time for healing.

Orally administered amphotericin or ketoconazole are only rarely


indicated in uncomplicated cases.
A 21 year old bisexual man has a 4 week history of intermittent
diarrhea, urethral discharge, and pain in the right knee and left second
toe. He has several oral ulcers, a clear urethral discharge, a scaly
papular rash on palms and soles, onycholysis, sausage-like swelling of
the left second toe, and heat and swelling of the right knee. The
results of Gram stains and cultures of urethral discharge are negative.
Rheumatoid factor is not present. The most likely diagnosis is

a) Reiter‟s syndrome
b) Gonococcal arthritis
c) Behcet disease
d) Psoriatic arthritis
e) Acquired immune deficiency syndrome
The correct answer is A

Explanation
Reiter's syndrome (reactive arthritis) is inflammation of the joints and
tendon attachments at the joints, often accompanied by inflammation of
the eye's conjunctiva and the mucous membranes, such as those of the
mouth and genitourinary tract, and by a distinctive rash. Reiter's
syndrome is also called reactive arthritis because the joint
inflammation appears to be a reaction to an infection originating in the
intestine or genital tract. This syndrome is most common in men aged 20
to 40.

Typically, symptoms begin 7 to 14 days after the infection. Inflammation


of the urethra result either directly from infection of the urethra or
even from a reaction to the intestinal infection. In men, inflammation
of the urethra causes moderate pain and a discharge from the penis or a
rash on the glans of the penis (balanitis circinata). The prostate gland
may be inflamed and painful. The genital and urinary symptoms in women,
if any occur, are usually mild, consisting of a slight vaginal discharge
or uncomfortable urination.

The conjunctiva become red and inflamed, causing itching or burning and
excessive tearing. Joint pain and inflammation may be mild or severe.
Several joints are usually affected at once, especially the knees, toe
joints, and areas where tendons are attached to bones, such as at the
heels. Small, painless or tender sores can develop in the mouth.

Which of the following pulmonary function tests most reliably


discriminates ?pure? chronic bronchitis from emphysema?

a) Total lung capacity


b) Functional residual capacity
c) Residual volume
d) Single breath diffusing capacity
e) Flow at 50% vital capacity
The correct answer is D

Explanation
The single breath diffusing capacity, or DLCO, is a measure of the
ability of the lungs to diffuse oxygen into, and carbon dioxide from,
the bloodstream. Measurement of diffusing capacity of the lung for
carbon monoxide (DLCO) should be considered a routine initial test in
evaluation of chronic airflow obstruction, particularly in advanced
disease. DLCO has been established as sensitive in detection of
emphysema that is associated with loss of alveolar surface area and
pulmonary circulation.
The most common cause of sensorineural hearing loss in the elderly is

a) Meniere's disease
b) Physical trauma
c) Ototoxic drugs
d) Presbycusis
The correct answer is D

Explanation
The most common cause of hearing impairment in the elderly is
presbycusis which is hearing loss that occurs in the high frequency
range (4000Hz to 8000Hz).

Meneiere‟s disease is a combination of vertigo, tinnitus and hearing


loss. Ototoxic drugs such as aminoglycosides and loop diuretics can
cause hearing loss in elderly also.
A 40 year old man complains to his doctor of impotence. Which one of the
following medication could be implicated?

a) Tadalafil
b) Sertraline
c) Clarithromycin
d) Enalapril
e) Clozapine
The correct answer is B

Explanation
Sertraline is an SSRI used in the treatment of depression. Some of the
various side effects of selective serotonin re-uptake inhibitors
include: nausea, loss of appetite, diarrhea, anxiety or irritability,
problems sleeping or drowsiness, loss of sexual desire or ability
(impotence), headaches or dizziness, weight gain.
A 47 year old female presents to your office complaining of hot flashes
and cold sweats of several months‟ duration. She is premenopausal. Which
of the following is accurate advice for this patient regarding vasomotor
symptoms‟

a) They usually peak at around the time of menopause, then decline


after menopause
b) Without treatment, they usually get worse each year after menopause
c) They are always caused by estrogen deficiency
d) Estrogen alone is recommended for therapy
The correct answer is A

Explanation
Vasomotor symptoms slowly increase until perimenopause, at which time
they peak. The symptoms then tend to diminish after menopause. Numerous
other pathologic and functional vasomotor etiologies may mimic hot
flashes. Estrogen is effective in treating hot flashes but generally
should not be given alone as it increases the risk for endometrial cancer.
Cilostazol (Pletal) has been found to be a useful drug for the treatment
of intermittent claudication. This drug is contraindicated in patients with

a) Congestive heart failure


b) A past history of stroke
c) Diabetes mellitus
d) Third degree heart block
e) Hyperlipidemia
The correct answer is A

Explanation
Cilostazol is a drug with phosphodiesterase inhibitor activity
introduced for the symptomatic treatment of arterial occlusive disease
and intermittent claudication. Cilostazol should be avoided in patients
with congestive heart failure, because it can increase the heart rate.

There are no limitations on its use in patients with previous stroke or


a history of diabetes. It has been found to have beneficial effects on
HDL cholesterol levels and in the treatment of third degree heart block.
A contraindication to the use of metformin (Glucophage) in a patient
with type 2 diabetes mellitus is

a) obesity
b) insulin resistance
c) renal insufficiency
d) concurrent use of glyburide (DiaBeta, Micronase)
The correct answer is C

Explanation
Metformin decreases glucose production in the liver and increases
glucose uptake. It has no effect on pancreatic insulin secretion. It is
useful to help overcome insulin resistance, and can be used concurrently
with a sulfonylurea. Studies have proven its safety and usefulness in
the obese diabetic. It is contraindicated in renal insufficiency, as
increased concentrations of the drug can cause lactic acidosis. Other
contraindications include hepatic disease, a previous history of lactic
acidosis, heart failure treated with drugs, and chronic lung disease.
A patient with history of occupational exposure (worked in a silica
mine) 20 years back, now presents with 6 month history of cough and
weight loss. CXR shows fibrosis of upper lobes. What is the most likely
diagnosis‟

a) Pneumonia
b) Tuberculosis
c) Mesothelioma
d) Rapidly progressive silicosis
The correct answer is B

Explanation
Pulmonary tuberculosis is associated with silica dust exposure as an
occupational disease. It is caused by Mycobacterium tuberculosis in
employees who have been exposed to crystalline silica dust in the workplace.

Symptoms include cough, hemoptysis, fever, chills and weight loss.


Diagnosis is made by sputum cultures and CXR. Treatment is usually with
a combination of medicines such as Isoniazid, Rifampin, Ethambutol and
Pyrazinamide.

Pneumonia would not be this long of a duration and would not present
with weight loss. Mesothelioma usually affects the lower lobes with
pleural thickening. Rapidly progressive silicosis develops within 6
months of acute exposure.
A 63 year old male with a history of arrhythmias who is on
antiarrhythmic drug therapy was identified with twisted QRS complex and
QT interval prolongation. What is this event identified as‟

a) Ventricular fibrillation
b) Torsades de Pointes
c) Supraventricular arrhythmias
d) Ventricular arrhythmias
e) Ventricular blockade
The correct answer is B

Explanation
Torsades de Pointes is the event identified by twisted QRS complex and
QT interval prolongation. Twisted QRS complex and QT interval
prolongation occur due usage of the drugs that prolong ventricular
repolarization. Antiarrhythmic drugs (like quinidine) and some
antibiotics (like erythromycin, clarithromycin, azithromycin) also
result in elevation of QT interval. Other causes for Torsades de Pointes
are diarrhea, hypomagnesia and hypokalemia. This is also witnessed in
the people with chronic alcoholism.
Which one of the following is shown in the EKG?

a) Atrial fibrillation
b) Atrial flutter
c) Multifocal atrial tachycardia
d) Complete AV block
e) Normal sinus rhythm
The correct answer is E

Explanation
Baseline artifact from tremors, body shakes, etc., can be confused with
atrial fibrillation or atrial flutter. Normal EKGs should have a normal
P wave before each QRS. The rate and QRS complex should be normal unless
sinus tachycardia is present.

Atrial fibrillation (AF) is characterized by disorganized atrial


activity without discrete P waves. Ventricular response is irregularly
irregular. A slow, regular rate in a patient with AF suggests a
manifestation of digitalis toxicity.

Atrial flutter is characterized by an atrial rate of 250?350 beats/min.


Typically the ventricular rate is one-half of the atrial rate.
Classically, atrial flutter waves are seen as regular saw-tooth-like
atrial activity, most prominent in the inferior leads. When the
ventricular rate is regular and not a simple fraction of the atrial
rate, complete heart block should be suspected, which may be a
manifestation of digitalis toxicity.

Multifocal atrial tachycardia (MAT) requires three or more consecutive P


waves of different morphologies at a rate of >100 beats/min. There may
be an irregular ventricular rate. This is common in digitalis toxicity,
hypokalemia, and severe cardiac and pulmonary diseases, or with certain
drugs (theophylline or adrenergic drugs).

Complete AV block would have a significantly slower ventricular rate,


and P waves would occur at irregular intervals.
A 41 year old man is seen for hematuria. He states that he has had this
on two previous occasions, both in relation to an upper respiratory
tract infection. On both previous occasions the urine cleared
spontaneously over a period of five to seven days.

At the time of the present visit he states that he has been feeling
lethargic with a sore throat for the past five days. Blood pressure is
170/95, urine analysis shows specific gravity 1020, nitrites negative, +
blood and + protein. Urine microscopy reveals oxalate crystals,
dysmorphic red blood cells and red cell casts. Which one of the
following is the most likely diagnosis‟

a) IgA nephropathy
b) Ingestion of beets
c) Renal calculi
d) Bladder carcinoma
e) Post-streptococcal glomerulonephritis
The correct answer is A

Explanation
Immunoglobulin A (IgA) nephropathy is the most common form of
glomerulonephritis in the world; caused by immune complexes
(combinations of antigens and antibodies) deposited in the kidney.
Usually the disorder progresses slowly; end-stage kidney failure
develops in about 20% to 40% of people after 5 to 25 years; progresses
more slowly in children. IgA nephropathy is a kidney disorder that
occurs when IgA?a protein that helps us fight infections‟settles in the
kidneys. After many years, the IgA deposits may cause the kidneys to
leak blood and sometimes protein in the urine.

Many people with IgA nephropathy leak blood in the urine, but this
leakage does not mean they will have long-term problems. Others leak
both blood and protein in the urine. If too much protein leaks into the
urine, the hands and feet can swell.
In the early stages, IgA nephropathy has no symptoms. This disease can
be silent for years, even decades. The first sign of IgA nephropathy may
be blood in the urine. The blood may appear during a cold, sore throat,
or other infection. At times, blood in the urine can only be detected by
your doctor or nurse using special tests. If the amount of blood
increases, you may notice that your urine is pink or the color of tea or
cola.

All of the following are present in irritable bowel syndrome (IBS), except

a) Recurrent type of nature


b) Bloody diarrhea
c) Bloating
d) Constipation
e) Cramping
The correct answer is B

Explanation
The Rome criteria are standardized symptom-based criteria for diagnosing
irritable bowel syndrome (IBS). The Rome criteria require the presence
of abdominal pain or discomfort for at least 3 days per month in the
last 3 months along with 2 of the following: (1) improvement with
defecation; (2) onset (of each episode of discomfort) associated with a
change in frequency of defecation, or (3) change in consistency of stool.

Other symptoms of IBS include abdominal pain or cramping, a bloated


feeling, gas (flatulence), diarrhea or constipation, with sometimes
alternating bouts of constipation and diarrhea and mucus in the stool.
The initial step in the management of hypercalcemia of malignancy should be

a) fluid restriction
b) bisphosphonates intravenously
c) phosphate depletion
d) aggressive rehydration
e) diuresis with furosemide (Lasix)
The correct answer is D

Explanation
Hypercalcemia of malignancy should be treated initially with aggressive
rehydration, followed by diuresis with furosemide, phosphorus
replacement if hypophosphatemia is present, and intravenous
bisphosphonates. Adjunctive therapies include dialysis, glucocorticoids,
calcitonin, plicamycin, and gallium nitrate.
A 22 year old woman presents with fever, malaise, generalized
arthralgias, and a skin rash, over the nose and malar eminences. Which
one of the following possible findings has the greatest relative
significance in the overall prognosis for the patient?

a) Immune complexes at the dermal-epidermal junction in skin


b) Pleuritis
c) Atypical verrucous vegetations of the mitral valve
d) Perivascular fibrosis in the spleen
e) Glomerular subendothelial immune complex deposition
The correct answer is E
Explanation
Systemic lupus erythematosus is a chronic, multisystem, inflammatory
disorder of probable autoimmune etiology, occurring predominantly in
young women.

Many of the clinical manifestations of SLE are caused by the effects of


circulating immune complexes on various tissues or to the direct effects
of antibodies to cell surface components.

Common manifestations include arthralgias and arthritis; malar and other


skin rashes; pleuritis or pericarditis; renal or CNS involvement; and
hematologic cytopenia.

Diagnosis requires clinical and serologic criteria. Treatment of severe


ongoing active disease requires corticosteroids, often
hydroxychloroquine, and sometimes immunosuppressants.
Which of the following syndromes is not associated with HLA-B27?

a) Reiter syndrome
b) Psoriatic Arthritis
c) Ankylosing Spondylitis
d) Behcet syndrome
The correct answer is D

Explanation
Spondyloarthropathies share certain clinical characteristics (eg, back
pain, uveitis, GI symptoms, rashes). Some are strongly associated with
the HLA-B27 allele. Clinical and genetic similarities suggest that they
also share similar causes or pathophysiologies.

Rheumatoid factor is negative in the spondyloarthropathies (hence, they


are also called the seronegative spondyloarthropathies). They include
ankylosing spondylitis, reactive arthritis, psoriatic arthritis and
Reiter‟s syndrome.
An 83-year-old female presents with pain, swelling, and erythema of her
left knee, as well as some swelling and pain in her right wrist. She
first noticed this problem last night before going to bed. She is
generally healthy and takes no medications. She has not been sexually
active since being widowed 15 years ago, and she currently lives with
her sister. She states that she developed pain and swelling in her left
ankle 2 years ago that lasted only a couple of days and resolved
spontaneously. Blood testing shows a very elevated erythrocyte
sedimentation rate but a normal rheumatoid factor and uric acid level.
Which one of the following is the most likely diagnosis‟

a) Gonococcal arthritis
b) Gout
c) Pseudogout (calcium pyrophosphate disease)
d) Rheumatoid arthritis
e) Rupture of the anterior cruciate ligament
The correct answer is C

Explanation
Acute monoarthritis in adults is most commonly caused by infection,
trauma, or crystal deposition. Rheumatoid arthritis infrequently
presents as monoarthritis, and more often has a subacute course with
multiple, symmetric joints involved. Although osteoporosis may result in
a fracture of the knee joint without trauma, there is no reason to
believe that this patient has a torn anterior cruciate ligament.
Gonococcal arthritis is one of the most common causes of septic
arthritis, but is highly unlikely in this elderly, sexually inactive
patient. Nongonococcal septic arthritis (especially due to
staphylococcal and streptococcal bacteria) is still a consideration and
should be ruled out by aspiration of fluid to be sent for culture.

This patient‟s presentation is most consistent with pseudogout. Having a


normal uric acid level suggests against gout, but does not rule it out.
Also, gout is seven times more likely to be seen in males, whereas
pseudogout is 1.5 times more frequent in females. Pseudogout most often
affects the elderly, and usually affects the knee, wrist, and ankle.
Gout presents most commonly in the first metatarsophalangeal joint and
insteps of the feet, but also can occur in the knee, wrist, finger, and
olecranon bursa.

Differentiating between gout and pseudogout can be difficult and is best


done by analysis of joint fluid. In patients with gout, this fluid
contains highly negative birefringent, needle-shaped urate crystals,
whereas in pseudogout the fluid contains rhomboid-shaped, weakly
positive birefringent calcium pyrophosphate crystals.

A 53 year old female presents with a fingernail infection. See picture:

This should be treated with

a) Griseofulvin for 2 weeks


b) Terbinafine for 6 weeks
c) Itraconazole for 2 weeks
d) Nizoral for 1 year
The correct answer is B

Explanation
Onychomycosis (tinea unguium) is a fungal infection of the nail. In such
infections you can see nail plate separation from nail bed and
thickened, discolored (white, yellow, brown, black), broken, and
dystrophic nails.

Terbinafine (Lamisil) is the first-line therapy for these infections 250


mg per day for 6 weeks to treat fingernails and for 12 weeks to treat
toenails. Complete blood count and ALT and AST levels are obtained at
baseline, then every 4 to 6 weeks during therapy.

Itraconazole (Sporanox) is alternative first-line therapy for nail


infections. Given 200 mg per day for 6 weeks to treat fingernails and
for 12 weeks to treat toenails. You need to monitor ALT and AST levels
at baseline, then every 4 to 6 weeks during therapy.
Which one of the following oils has the highest content of omega-3 fatty
acid per tablespoon?

a) Olive
b) Soybean
c) Canola
d) Flaxseed
e) Walnut
The correct answer is D

Explanation
Many health-minded individuals are cooking with flaxseed oil to increase
their intake of omega-3 fatty acids. It has the highest content of
omega-3 fatty acids of the oils listed (8.5 g per tablespoon). Canola
and walnut oils are also good source, with 1.3 and 1.4 g of ?-linoleic
acid per tablespoon.
A 34 year old man is found to have a blood pressure of 180/105 mmHg. He
admits to stress at work and states that his father was diagnosed as
hypertensive at the age of 60 but, thus far, has not required treatment.
He denies taking any medications. He smokes 25 cigarettes per day but
takes no alcohol. On physical examination he is not obese, has no signs
of cardiovascular disease, and no hypertensive retinopathy. The results
of investigations are given below:

Urine analysis negative

Na 146 mmol/L
K 2.8 mmol/L
Cl 102 mmol/L
HCO3 33 mmol/L
Urea 4.2 mmol/L
Creatinine 92 mmol/L
Glucose 6.2 mmol/L

Which of the following investigations is/are indicated?

a) Dietary intake assessment


b) Urine metanephrines
c) CT of chest
d) Plasma Renin Activity and 24 hour urine collection for aldosterone
estimation
The correct answer is D

Explanation
In primary hyperaldosteronism (Conn‟s disease), overproduction of
aldosterone leads to fluid retention and increased blood pressure,
weakness, and, rarely, periods of paralysis. Hyperaldosteronism can be
caused by a tumor in the adrenal gland or may be a response to some
diseases. High aldosterone levels can cause high blood pressure and low
potassium levels; low potassium levels may cause weakness, tingling,
muscle spasms, and periods of temporary paralysis. Doctors measure the
levels of sodium, potassium, and aldosterone in the blood. Sometimes, a
tumor is removed, or people take drugs that block the action of aldosterone.

Doctors who suspects hyperaldosteronism first tests the levels of sodium


and potassium in the blood. Doctors may also measure aldosterone levels.
If they are high, spironolactone or eplerenone, drugs that block the
action of aldosterone, may be given to see if the levels of sodium and
potassium return to normal. In Conn's syndrome, the levels of renin are
also very low.

When too much aldosterone is being produced, doctors examine the adrenal
glands for a noncancerous tumor (adenoma). Computed tomography (CT) or
magnetic resonance imaging (MRI) can be helpful, but sometimes blood
samples from each of the adrenals must be tested to determine the source
of the hormone.
A 30 year old woman presents with complaints of heat intolerance,
sleeplessness, nervousness and weight loss despite an excellent
appetite. What changes in TSH and free T4 would you expect?

a) Decreased TSH, increased free T4


b) Increased TSH, increased free T4
c) Decreased TSH, normal free T4
d) Increased TSH, decreased free T4
e) Normal TSH, increased free T4
The correct answer is A

Explanation
Hyperthyroidism is characterized by hypermetabolism and elevated serum
levels of free thyroid hormones. Symptoms are many but include
tachycardia, fatigue, weight loss, and tremor.

Diagnosis is based on history, physical examination, and thyroid


function tests. Serum TSH is the best test, because TSH is suppressed in
hyperthyroid patients except when the etiology is a TSH-secreting
pituitary adenoma or pituitary resistance to thyroid hormone. Free T4 is
increased.
Which one of the following is a cause of thyrotoxicosis characterized by
a decreased radioactive iodine uptake?

a) Graves‟ disease
b) Subacute thyroiditis
c) Toxic multinodular goiter
d) Solitary toxic thyroid nodule
The correct answer is B

Explanation
Thyrotoxicosis with a high 24-hours radioactive iodine uptake (RAIU) is
caused by Graves‟ disease, toxic multinodular goiter, a solitary hot
nodule, a TSH-secreting pituitary tumor, molar pregnancy, and
choriocarcinoma. Thyrotoxicosis with a low 24-hour RAIU may be the
result of subacute thyroiditis, sporadic silent thyroiditis, postpartum
lymphocyctic thyroiditis, radiation-induced thyroiditis, iodine-induced
thyroiditis, thyrotoxicosis factita, metastatic follicular thyroid
cancer, and struma ovarii.
Which finding is not frequently found in Chronic Myelogenous Leukemia (CML)?
a) Elevated WBCs
b) Elevated vitamin B12 level
c) Elevated LDH
d) Translocation between chromosomes 9 and 14
e) Increased uric acid level
The correct answer is D

Explanation
The diagnosis of CML is suspected based on the results of a simple blood
test. The test may show an abnormally high white blood cell count. In
blood samples examined under a microscope, less mature white blood
cells, normally found only in bone marrow, are seen.

Tests that analyze chromosomes (cytogenetics or molecular genetics) are


needed to confirm the diagnosis. Chromosomal analysis of the leukemia
cells always shows a rearrangement of two particular chromosomes into
what is called the Philadelphia chromosome. The Philadelphia chromosome
produces an abnormal enzyme (tyrosine kinase), which is responsible for
the abnormal growth pattern of the white blood cells in CML.

Philadelphia chromosome or Philadelphia translocation is a specific


chromosomal abnormality that is associated with chronic myelogenous
leukemia (CML). It is due to a reciprocal translocation designated as
t(9;22) which means an exchange of genetic material between chromosome 9
and chromosome 22.

Celiac sprue is associated with which one of the following test


abnormalities‟

a) Low serum ferritin


b) Hypercalcemia
c) Low blood glucose
d) Low alkaline phosphatase
e) Elevated total serum protein
The correct answer is A

Explanation
Iron-deficiency anemia, which causes low serum ferritin, is the most
common clinical presentation of celiac sprue in adults. Osteopenia is
common in untreated celiac sprue due to vitamin D deficiency, which also
results in hypocalcemia and elevated alkaline phosphatase. Celiac sprue
is associated with type I diabetes. Patients with unmanaged celiac sprue
also have low protein levels from malabsorption.

A 45 year old man comes to the emergency department because of the


sudden onset of shortness of breath, light-headedness, diaphoresis and
weakness. He is afebrile. On auscultation of the lungs, bibasilar rales
are heard. Electrocardiogram is shown.
The most likely diagnosis is

a) Acute pericarditis
b) Hyperventilation syndrome
c) Myocardial infarction
d) Pulmonary embolism
e) Spontaneous pneumothorax
The correct answer is C

Explanation
The electrocardiogram shows ST segment elevation in leads V1-V4.
Furthermore, there are Q-waves forming in these leads. Although he is
younger than most patients with a myocardial infarction, the symptoms
and EKG are classic. As expected with an infarction of this size, he is
beginning to develop pulmonary congestion as demonstrated by the
bibasilar rales.
A 68 year old woman presents with a band-like burning pain in the right
upper quadrant extending from the epigastrium around to the midline of
the back. On physical examination, there is no abdominal tenderness.
Findings on ultrasonography of the gallbladder are normal, serum amylase
concentration is normal. The most likely diagnosis is

a) Acalculous cholecystitis
b) Chronic relapsing pancreatitis
c) Diverticulitis of the cecum
d) Herpes zoster
e) Penetrating duodenal ulcer
The correct answer is D

Explanation
Herpes zoster (shingles) is characterized by a vesicular rash that is
preceded by the bandlike burning pain described here. It usually
involves only one or two dermatomes. It is always a
reactivation/flare-up of a previous Varicella infection. In this
patient, there is no abdominal tenderness, which decreases the
likelihood of an intra-abdominal organ being involved.

Acalculous cholecystitis is a diagnosis made more commonly in the ICU.


The patient is usually post-op, in shock, or critically ill. Chronic
relapsing pancreatitis is characterized by a boring or cramping
epigastric pain radiating to the back. It is more commonly seen in
alcoholics, who will have some abdominal tenderness. Amylase and lipase
levels may support the diagnosis if they come back high.
Which of the following is associated with thyroid disease?

a) Neurofibromatosis
b) Vitiligo
c) Erythema nodosum
d) Pemphigus vulgaris
e) Icthyosis vulgaris
The correct answer is B

Explanation
Vitiligo is a loss of skin melanocytes that causes areas of skin
depigmentation of varying sizes. Cause is unknown, but the condition may
be autoimmune, as up to 1/3 of patients have evidence of other
autoimmune disease. Vitiligo affects 0.5 to 2% of the population.
Etiology is unknown, but melanocytes are lacking in affected areas; some
patients have antibodies to melanin. Up to 30% have other autoimmune
antibodies (to thyroglobulin, adrenal cells, and parietal cells) or
clinical autoimmune endocrinopathies (Addison's disease, diabetes
mellitus, pernicious anemia, and thyroid dysfunction), leading to
speculation that vitiligo is an autoimmune disease. However, the
relationship is unclear and may be coincidental. The strongest
association is with hyperthyroidism (Graves' disease) and hypothyroidism
(Hashimoto's thyroiditis).
An elderly alcoholic male is brought to the hospital by his grandson,
who found him in poor condition. The grandson reports that his family
has not seen the patient in months.

The patient denies complaints, but is obviously malnourished, poorly


washed, and mildly intoxicated. You admit the patient to the hospital
and begin providing supportive care, including intravenous fluids with
dextrose, a regular diet, and physical therapy evaluation. On the
evening of the second day he becomes weak and more confused. His blood
pressure is 88/56 mm Hg, and he has a seizure. Your evaluation includes
the following laboratory findings:

Glucose----------------60 mg/dL (N 70-110)


BUN----------------9 mg/dL (N 6-20)
Creatinine----------------2.6 mg/dL(N 0.8-1.3)
Creatine kinase----------------480 U/L (N 38-174)
Troponin I----------------<0.1 ng/mL (N<0.6)
Albumin----------------2.7 g/dL (N 3.4-4.8)
Calcium----------------8.2 mg/dL (N 8.6-10.0)
Phosphate----------------0.7 mg/dL (N 2.7-4.5)
ALT (SGPT)----------------68 U/L (N 10-40)
AST (SGOT)----------------88 U/L (N 10-30)

This episode is most likely related to abnormal levels of which one of


the following?

a) Glucose
b) Creatinine
c) Creatine kinase
d) Calcium
e) Phosphate
The correct answer is E

Explanation
Severe hypophosphatemia is a medical emergency. In poorly nourished
patients, refeeding syndrome can occur. Symptoms usually occur by the
second or third day of improved nutrition, and are often multisystemic.
Findings may include weakness, confusion, dysrhythmias, respiratory
failure, congestive heart failure, hypotension, ileus, metabolic
acidosis, seizures, coma, and sudden death. This constellation of
problems results from decreased insulin secretion as stores of
intracellular phosphate become depleted. Providing carbohydrates through
intravenous fluids or refeeding increases insulin secretion, which
stimulates cells to take up phosphate, causing severe hypophosphatemia.
In this setting cells are unable to produce enough 2,3
diphosphoglycerate and adenosine triphosphate to meet metabolic demands

While hypoglycemia is another medical emergency, this patient‟s glucose


level is not low enough to cause these symptoms. Similarly, renal
failure of some type is present, as is an elevated creatinine kinase
suggesting rhabdomyolysis; however, neither of these problems would be
expected to cause this patient‟s symptoms. Hypocalcemia can cause
multisystemic problems, including weakness and seizures, but this level
is not critically low and is not associated with hypotension.
The development of azotemia suggests the presence of underlying renal
artery stenosis in a patient taking which one of the following?

a) Amlodipine (Norvasc)
b) Doxazosin (Cardura)
c) Lisinopril (Zestril)
d) Metoprolol (Toprol)
e) Hydrochlorothiazide
The correct answer is C

Explanation
ACE inhibitor-induced renal insufficiency is potentiated by sodium
depletion and preexisting renal dysfunction, is usually reversible if
detected promptly, and often provides an indication that critical
renovascular disease is present.
Which is more often associated with hospital acquired pneumonia than
community acquired pneumonia?

a) Streptococcus pneumoniae
b) Hemophilus influenza
c) Legionella
d) Chlamydia pneumoniae
e) Mycoplasma pneumoniae
The correct answer is C

Explanation
Legionella pneumophila most often causes pneumonia, with extrapulmonary
features. Diagnosis requires specific growth media, serologic testing,
or PCR analysis. Treatment is with doxycycline, macrolides, or
fluoroquinolones. The 1st appearance of this organism was in 1976 at a
convention of the American Legion, thus the name Legionnaires' disease.
Nonpneumonic infection is called Pontiac fever. The organisms can be
found in soil and freshwater. Manufactured water-storage containers,
including water-cooled air-conditioning units, enhance its growth.
Spread is most likely by aerosols of potable water.

Legionnaires' disease is a flu-like syndrome with acute fever, chills,


malaise, myalgias, headache, or confusion. Frequently nausea, loose
stools/watery diarrhea, abdominal pain, cough, and arthralgias also
occur. Pneumonic manifestations may include dyspnea, pleuritic pain, and
hemoptysis.
A 40 year old woman presents with a self-detected hard breast mass.
Which one of the following is the procedure of choice for confirming the
diagnosis‟

a) Mammography
b) Thermography
c) Ultrasonography
d) Aspiration cytology
e) Incisional biopsy histology
The correct answer is E

Explanation
In the evaluation of a breast mass the following points are important to
consider:

Painful, tender, rubbery lumps in a younger woman with previous history


of similar findings suggest fibrocystic changes. A stony hard, irregular
lump with skin dimpling suggests cancer.

Initially, physicians try to differentiate solid from cystic lumps


because cysts are rarely cancerous. Typically, ultrasonography is done.
Lesions that appear cystic are sometimes aspirated, and solid lumps are
evaluated with mammography followed by imaging-guided biopsy to confirm
the diagnosis.
You are asked to assess a 65 year old woman with a lung tumor. Routine
blood test reveal that the patient is hyponatremic. You suspect the
cause of hyponatremia is antidiuretic hormone (ADH) secretion by the
tumor. Which one of the following lung tumors does this patient have?

a) Squamous cell carcinoma


b) Adenocarcinoma
c) Large cell carcinoma
d) Small cell carcinoma
e) Bronchioalveolar carcinoma
The correct answer is D

Explanation
Small cell lung cancer can present with a paraneoplastic syndrome called
SIADH (syndrome of inappropriate anti-diuretic hormone). It is caused by
ectopic production of ADH. Abnormalities in water and electrolyte
balance, including hyponatremia and calcium disturbances, may result
from production of ADH and parathyroid hormone?like hormones from small
cell lung cancer.
You see a 90 year old male with a 5-year history of progressive hearing
loss. The most common type of hearing loss at this age affects

a) Predominately high frequencies


b) Predominately mid-frequencies
c) Predominately low frequencies
d) All frequencies roughly the same
The correct answer is A
Explanation
In the geriatric population, presbycusis is the most common cause of
hearing loss. Patients typically have the most difficulty hearing higher
frequencies such as consonants. Lower-frequency sounds such as vowels
are preserved.
Which of the following diabetes drugs is contraindicated in liver and
kidney failure?

a) Insulin
b) Metformin
c) Glyburide
d) Acarbose
The correct answer is B

Explanation
Insulin works by directly pushing glucose from the bloodstream into the
cells. Metformin works by inhibiting hepatic gluconeogenesis. Glyburide
is an oral sulfonyl-urea hypoglycemic drug that increases pancreatic
insulin secretion.

Acarbose inhibits enzymes needed to digest carbohydrates. Because the


carbohydrates are not broken down into glucose molecules, less glucose
is absorbed into the bloodstream.

Metformin is contraindicated in people with any condition that could


increase the risk of lactic acidosis, including liver and kidney
disorders (creatinine levels over 132 micromol/l).
A 60 year old male presents to the office complaining of a productive
cough for the past 3 months.

He mentions that the sputum is plentiful and foul smelling. He is a


chain smoker. Vitals are within normal limits except for a mild fever of
37.60C. Physical exam shows finger clubbing. Auscultation of the lungs
shows coarse crepitations in both lungs.

Chest x-ray reveals increased vascular markings and peribronchial


thickening.

What is the diagnostic test of choice for this patient?

a) Bronchography
b) Bronchoscopy
c) High resolution CT Scan of the lung
d) Sputum for Acid Fast bacilli
e) Sputum gram stain and culture
The correct answer is C

Explanation
High resolution CT scan of the lung is the diagnostic modality of choice
for bronchiectasis. Copious foul smelling sputum is a clue. The chest
x-ray of our patient is characteristic of bronchiectasis: peribronchial
thickening (Tram track appearance).

Bronchography is an old method of diagnosis and it has been replaced


nowadays by HRCT.

Bronchoscopy and biopsy would be warranted if a bronchial lesion were


seen on chest X ray or CT scan.
Sputum examination & Culture and sputum smear for AFB should be done in
clinical practice; however the question is clear and asks for the
diagnostic modality of choice.
A 52 year old white male is being considered for pharmacologic treatment
of hyperlipidemia because of an LDL cholesterol level of 180 mg/dL.
Before beginning medication for his hyperlipidemia, he should be
screened for

a) Hyperthyroidism
b) Hypothyroidism
c) Addison‟s disease
d) Cushing‟s disease
e) Pernicious anemia
The correct answer is B

Explanation
According to the Summary of the National Cholesterol Education Program
(NCEP) Adult Treatment Panel III Report, any person with elevated LDL
cholesterol or any other form of hyperlipidemia should undergo clinical
or laboratory assessment to rule out secondary dyslipidemia before
initiation of lipid-lowering therapy. Causes of secondary dyslipidemia
include diabetes mellitus, hypothyroidism, obstructive liver disease,
chronic renal failure, and some medications.
A 54-year-old female takes levothyroxine (Synthroid), 0.125 mg/day, for
central hypothyroidism secondary to a pituitary adenoma. The nurse
practitioner in your office orders a TSH level, which is found to be 0.1
mIU/mL (N 0.5-5.0). Which one of the following would you recommend?

a) Decrease the dosage of levothyroxine


b) Increase the dosage of levothyroxine
c) Order a free T4 level
d) Order a TRH stimulation test
e) Repeat the TSH level in 3 months
The correct answer is C

Explanation
Although uncommon, pituitary disease can cause secondary hypothyroidism.
The characteristic laboratory findings are a low serum free T4 and a low
TSH. A free T4 level is needed to evaluate the proper dosage of
replacement therapy in secondary hypothyroidism. The TSH level is not
useful for determining the adequacy of thyroid replacement in this case,
and the low level would prevent the physician from determining whether
the dosage of levothyroxine is appropriate. In the initial evaluation of
secondary hypothyroidism, a TRH stimulation test would be useful if TSH
failed to rise in response to stimulation. It is not necessary in this
case, since the diagnosis has already been made.
A 26 year old man developed hemoptysis and dyspnea over the course of 3
months. His physician suspected tuberculosis and started him on triple
therapy with isoniazid, rifampin, and ethambutol. After 2 months, the
patient complained of pins-and-needles sensations in his feet.
Neurologic examination was otherwise unremarkable. Strength was good in
all limbs. Which one of the following nutritional deficiencies is most
likely responsible?

a) Vitamin B12 deficiency


b) Vitamin B6 (pyridoxine) deficiency
c) Vitamin E (alphatocopherol) deficiency
d) Vitamin D deficiency
e) Vitamin B1 (thiamine) deficiency
The correct answer is B

Explanation
There are many drugs that interfere with the metabolism of vitamin B6.
Isoniazid, which is used to treat tuberculosis alters the activity of
vitamin B6.

Acute isoniazid toxicity can result in coma and seizures that are
reversed by vitamin B6. It is recommend to take a supplement that
provides B6 when isoniazid is prescribed, which is usually enough to
prevent symptoms of vitamin B6 deficiency.
The nephrotic syndrome is associated with all of the following, except

a) Diabetic glomerulosclerosis
b) Chronic glomerulonephritis
c) Thrombosis of the renal veins
d) Chronic pyelonephritis
e) Renal amyloidosis
The correct answer is D

Explanation
Nephrotic syndrome has numerous causes which include amyloidosis,
diabetes mellitus, sarcoidosis, carcinoma (bronchus, breast, colon,
stomach, kidney), leukemia, lymphomas, infective endocarditis, leprosy,
postinfectious glomerulonephritis, syphilis, vascular prosthetic
nephritis and thrombosis of the renal veins.
A 25 year old woman has intermittent diplopia. She says she chokes on
her food and regurgitates it, sometimes through her nose. Physical
examination reveals drooping eyelids and bilateral facial muscle
weakness without atrophy, deep tendon reflexes are normal. Which one of
the following is the most likely diagnosis‟

a) Familial periodic paralysis


b) Muscular dystrophy
c) Polymyositis
d) Myasthenia gravis
e) Multiple sclerosis
The correct answer is D

Explanation
Myasthenia gravis is an autoimmune disorder in which communication
between nerves and muscles is impaired, resulting in episodes of muscle
weakness. Myasthenia gravis is more common among women. It usually
develops in women between the ages of 20 and 40. However, the disorder
may affect men or women at any age.

The most common symptoms are weak, drooping eyelids; weak eye muscles,
which cause double vision; and excessive fatigue of specific muscles
after exercise. In 40% of people with myasthenia gravis, the eye muscles
are affected first, but 85% eventually have this problem. In 15% of
people, only the eye muscles are affected, but in most people, the whole
body is affected. Difficulty speaking and swallowing and weakness of the
arms and legs are common. Hand grip may alternate between weak and
normal; this fluctuating grip is called milkmaid's grip. The neck
muscles may become weak. Sensation is not affected.

When a person with myasthenia gravis uses a muscle repetitively, the


muscle usually becomes weak. For example, a person who once could use a
hammer well becomes weak after hammering for several minutes. However,
muscle weakness varies in intensity from hour to hour and from day to
day, and the course of the disease varies widely.
A 62 year old white male comes to your office with pain and swelling of
the left great toe at the metatarsophalangeal joint. Examination shows
it is erythematous, warm, swollen, and tender to touch. The patient has
a history of diabetes mellitus controlled by diet, and hypertension. His
medications include hydrochlorothiazide, 25 mg/day. A CBC and blood
chemistry profile are normal, except for a uric acid level of 9.2 mg/dL
(N 3.6-8.5). Which one of the following is true in this situation?

a) This attack should resolve spontaneously in 3-4 days


b) Allopurinol (Zyloprim) therapy should be started
c) The elevated uric acid level establishes the diagnosis of gout
d) Intra-articular steroid injection should be avoided
e) Stopping the hydrochlorothiazide may control the hyperuricemia
The correct answer is E

Explanation
This is a typical presentation for gout. Elevated uric acid levels are
not necessary for the diagnosis, as there are some patients with normal
uric acid levels who still have gout. Conversely, hyperuricemia does not
establish the diagnosis of gout. Risk factors for the development of
gout include several enzyme deficiencies, renal insufficiency,
hypertension, obesity, moonshine ingestion (causing lead exposure), and
alcohol abuse. There are several medications that elevate uric acid,
including diuretics, low-dose salicylates, niacin, cyclosporine,
ethambutol, and pyrazinamide.

A typical gout attack such as the one described will resolve


spontaneously within 2 weeks without treatment. In patients who have an
acute monoarticular arthritis in addition to gout, other diagnoses such
as osteoarthritis, pseudogout, and infection must be considered. The
diagnosis of gout is established by aspiration of synovial fluid or
tophi, with characteristic uric acid crystals detected by polarized
light microscopy. Treatment can consist of NSAIDs in healthy
individuals. Indomethacin is considered the drug of choice.
Corticosteroids can also be used, and are particularly helpful when the
patient has renal insufficiency. Intra-articular injections of a
corticosteroid such as triamcinolone are useful, and intramuscular
corticosteroids may be especially useful in patients with polyarticular
gout. Colchicine may be used, but may cause diarrhea. Life style changes
such as weight loss, discontinuing alcohol use, and changing
antihypertensive therapy is often all that is needed to control the
hyperuricemia and thus prevent further attacks. Colchicine can be used
for prophylaxis as well, although it does not alter hyperuricemia or
prevent tophi from forming. If a patient has more than two attacks per
year, urate chlorine therapy is indicated. A 24-hour urine collection to
identify whether the patient is an under-excretor or an overproducer of
uric acid would indicate the correct medication. Overproducers are
treated with allopurinol, while under-excretors benefit from probenecid
if renal function is normal and there is no history of kidney stones.
A 72 year old man has suffered a fall. His daughter reports that, for
several months, she has noticed that he walks with smaller, shuffling
footsteps and that his spine and arms seem always to be flexed when
walking. All of the following may be found, except

a) Intention tremor of the hand


b) A fine rapid tremor in the right foot
c) Pill rolling tremor in both hands
d) Micrography
e) Mild dementia
The correct answer is A

Explanation
Parkinson's disease is an idiopathic, slowly progressive, degenerative
CNS disorder characterized by resting tremor, muscular rigidity, slow
and decreased movement, and postural instability. Diagnosis is clinical.
Treatment is with levodopa plus carbidopa, other drugs, and, for
refractory symptoms, surgery.

A resting tremor of one hand is often the first symptom. The tremor is
characterized as follows: Often involving the wrist and fingers in
movements similar to those used to manipulate small objects or pills
(pill-rolling tremor) Usually, the hands, arms, and legs are most
affected, in that order.

Dementia can occur. Hypokinesia and impaired control of distal


musculature cause micrographia (writing in very small letters) and make
activities of daily living increasingly difficult.

Intention tremor is a dyskinetic disorder consisting of wide tremor


during voluntary movements. A tremor that gets worse when a person is
moving or reaching for an object is called an intention tremor. It is
the result of dysfunction of the cerebellum.
A 35 year old man suddenly develops a generalized pruritic macular
erythematous rash and fever. Four days ago he saw his physician because
of a sore throat that started the day before and was started on
penicillin after a throat culture grew beta-hemolytic streptococci.
Laboratory studies now reveal a urea of 14 mmol/L. Urinalysis shows no
casts, 2 protein, 50 RBC/high power field, and 50 WBC/high power field.
Complete blood cell count is normal except for eosinophilia (12%). Urine
protein concentration is 650 mg/24 h. Which one of the following is the
most likely diagnosis‟

a) Acute poststreptococcal glomerulonephritis


b) Membranous glomerulonephritis
c) Polyarteritis nodosa
d) Schonlein-Henoch purpura
e) Drug-induced interstitial nephritis
The correct answer is E

Explanation
Drug-induced interstitial nephritis (DIN) is characterized by a sudden
impairment of renal function and is mainly a result of an
immune-mediated reaction after intake of a drug. Many different drugs,
such as antibiotics, anticonvulsants, diuretics, proton pump inhibitors,
non-steroidal anti-inflammatory drugs and many others, are known to
cause DIN.
The clinical manifestations are characterized by arthralgias, macular or
maculopapular exanthema and fever, together with mild proteinuria,
sterile pyuria, and eosinophilia. In many cases the only sign is an
asymptomatic increase in serum creatinine.

The diagnosis of DIN is confirmed with certainty only by biopsy. The


mainstay of treatment is drug discontinuation; the role of steroids is
controversial.
A 60 year old man develops typical symptoms of angina. He also complains
of occasional lightheadedness during exercise. On physical examination,
blood pressure is 140/80 mm Hg, pulse is 80/minute and regular. There is
a left ventricular heave and a grade 4/6 systolic ejection murmur heard
at the second right interspace and radiating to the neck. The chest
x-ray shows cardiac enlargement and the electrocardiogram shows left
ventricular hypertrophy. Which one of the following courses of
management is correct?

a) Begin digitalis, a diuretic and nitroglycerin and reassess in 1 month


b) Begin isosorbide dinitrate
c) Refer for cardiac catheterization and consideration for cardiac surgery
d) Begin digitalis and nitroglycerin and reassess in 1 month
e) Begin nitroglycerin and propranolol and reassess in 1 month
The correct answer is C

Explanation
The classic symptom triad of aortic stenosis includes angina pectoris,
syncope, and heart failure, which most commonly manifest after the sixth
decade of life. Some patients remain asymptomatic, but others develop
exertional chest pain, effort dizziness or lightheadedness, easy
fatigueability, and progressive inability to exercise. Exertional
dyspnea is the most common initial complaint, even with normal LV
systolic function, and it often relates to abnormal LV diastolic function.

Table. Recommendations for Cardiac Catheterization in Aortic Stenosis


Indication:
-Coronary angiography before AVR in patients at risk for CAD
-Assessment of severity of AS in symptomatic patients when AVR is
planned or when noninvasive tests are inconclusive or a discrepancy
exists in the clinical findings regarding severity of AS or the need for
surgery
-Assessment of severity of AS before AVR when noninvasive tests are
adequate and concordant with clinical findings and coronary angiography
is not needed
-Assessment of LV function and severity of AS in asymptomatic patients
when noninvasive tests are adequate

The primary management of symptomatic patients with valvular aortic


stenosis is interventional. Medical treatment essentially is reserved
for patients who have complications of aortic stenosis such as heart
failure, infective endocarditis, hypertension or arrhythmias.

A 63 year old man presents with intermittent neurological abnormalities


associated with hypoglycemia. The patient experiences an attack in the
office, and the physician draws blood for serum insulin, C peptide, and
glucose. The attack is relieved by giving the patient glucose. One week
later, laboratory results reveal low serum glucose, high serum insulin
and low serum C peptide. Which one of the following would result in
these clinical and laboratory findings‟

a) Insulinoma
b) Reactive hypoglycemia from eating
c) Glucagonoma
d) Insulin taken surreptitiously
The correct answer is D

Explanation
C-peptide is measured to differentiate insulin produced by the body from
insulin injected into the body. When insulin is synthesized by the beta
cells of the pancreas, it is produced as a large molecule (a
propeptide). This molecule is then split into two pieces: insulin and
C-peptide. The function of C-peptide is not known.

The C-peptide level may be measured in a patient with type 1 diabetes to


see if any insulin is still being produced by the body. It may also be
measured in the evaluation of hypoglycemia to see if the person's body
is producing too much insulin.

Normal values in a patient requiring insulin injections indicate that


the person's body is still producing some insulin. Normal values in a
patient who has low blood sugar indicate that the patient is making too
much insulin.

Low values (or no C-peptide) indicate that the person's pancreas is


producing little or no insulin.
A 41 year old man is seen for hematuria. He states that he has had this
on two previous occasions, both in relation to an upper respiratory
tract infection. On both previous occasions the urine cleared
spontaneously over a period of five to seven days.

At the time of the present visit he states that he has been feeling
lethargic with a sore throat for the past five days. Blood pressure is
170/95, urine analysis shows specific gravity 1020, nitrites negative, +
blood and + protein. Urine microscopy reveals oxalate crystals,
dysmorphic red blood cells and red cell casts. The presence of
dysmorphic red blood cells is indicative of

a) Urine infection
b) Delay in analysis of the urine sample
c) Glomerular bleeding
d) Urothelial malignancy
e) Urinary tract calculus
The correct answer is C

Explanation
Immunoglobulin A (IgA) nephropathy is the most common form of
glomerulonephritis in the world; caused by immune complexes
(combinations of antigens and antibodies) deposited in the kidney.
Usually the disorder progresses slowly; end-stage kidney failure
develops in about 20% to 40% of people after 5 to 25 years; progresses
more slowly in children. IgA nephropathy is a kidney disorder that
occurs when IgA settles in the kidneys. After many years, the IgA
deposits may cause the kidneys to leak blood and sometimes protein in
the urine.

A urine test called urinalysis usually provides the first clues. In a


urinalysis, the doctor or nurse dips a special strip with chemicals into
the urine sample. The strip changes color when blood or protein is
present in the urine. If the test strip is positive, the urine will then
be examined with a microscope to look for red blood cells. The red blood
cells may be clumped together to form little tubes. These tubes are
called casts because they are formed or molded inside the kidneys‟ tiny
draining structures. If casts are found, it usually means the kidney
filters are damaged.

A 74 year old man presents with bilateral symmetrical sensorineural


hearing loss. Physical exam is otherwise normal. What is the most likely
diagnosis‟

a) Autosclerosis
b) Acoustic neuroma
c) Presbycusis
d) Circulatory deficit
The correct answer is C

Explanation
Presbycusis, or age-related hearing loss, is the cumulative effect of
aging on hearing. Also known as presbyacusis, it is defined as a
progressive bilateral symmetrical age-related sensorineural hearing
loss. The hearing loss is confined to higher frequencies. Presbycusis is
the most common cause of hearing losing in elderly.

A 38 year old white male wants to go to the mountains to ski. In the


past he has experienced moderate symptoms of acute mountain sickness
(AMS), including headache, nausea, shortness of breath, and sleep
disturbance. He has been otherwise healthy. Which one of the following
has been shown to be helpful in minimizing or preventing the symptoms of
AMS‟

a) Furosemide (Lasix)
b) Erythromycin
c) Acetazolamide (Diamox)
d) Beta-Blockers
e) Fluid restriction
The correct answer is C

Explanation
Acute mountain sickness (AMS) is a clinical syndrome which may affect as
many as 12%-67% of persons ascending to altitudes of 8000 feet or
greater. While a standard definition of AMS does not exist, persons
having three or more of the following symptoms may be considered to have
AMS: headache, nausea, vomiting, sleep disturbance, anorexia, fatigue,
or dyspnea. Gradual ascent is recommended to allow acclimatization.

Measures which may help minimize symptoms include avoidance of alcohol,


increased fluid intake, and a high-carbohydrate diet. Fluid restriction
and diuretics should be avoided because of the diuresis associated with
acclimatization, which may by itself cause dehydration. Antibiotics are
of no benefit. B-Blockers would be harmful, by interfering with the
physiologic responses of tachycardia and increased cardiac output at
higher altitudes.

Acetazolamide speeds the process of acclimatization. The drug is a


carbonic anhydrase inhibitor which results in a renal bicarbonate
diuresis and metabolic acidosis, thereby increasing ventilation and
arterial oxygenation. The respiratory stimulation is particularly
important during sleep, when it reduces the severe hypoxemia caused by
periodic breathing. The drug also lowers cerebrospinal fluid volume and
pressure, which may play an additional role in it s therapeutic and
prophylactic utility.
A 35-year-old white male with known long QT syndrome has a brief episode
of syncope requiring cardiopulmonary resuscitation. Which one of the
following is most likely responsible for this episode?

a) Sinus tachycardia
b) Atrial flutter with third degree block
c) Asystole
d) Torsades de pointes
The correct answer is D

Explanation
Patients with long QT syndrome that have sudden arrhythmia death
syndrome usually have either torsades de pointes or ventricular
fibrillation. Sinus tachycardia would not explain the syncope, and
atrial flutter and asystole are not usual in long QT syndrome.
A patient on lithium therapy develops weakness, lethargy and intolerance
to heat. What investigation must be done?

a) Lithium level
b) TSH level
c) Reassurance
d) CBC
e) CMP
The correct answer is B

Explanation
It is well established that lithium therapy can cause hypothyroidism in
about 2-3 % of patients. Once hypothyroidism is diagnosed, treatment of
the condition should be investigated rapidly with thyroxine.

Discontinuation of lithium therapy is an option but in most cases the


psychiatric indication for continuing it is strong, as in this patient.
Once treatment is begun, regular monitoring of TSH and FT4 will be required.

A 52 year old male with a history of seizure attacks suffered from loss
of consciousness followed by tonic and clonic muscular contractions. His
tongue fell back into his throat and he choked.
What is the best and most prior medication for the treatment of this
condition?

a) Phenytoin
b) Valproic acid
c) Tiagabine
d) Phenobarbital
e) Carbamazepine
The correct answer is B

Explanation
Answer: B ? Valproic acid is considered the agent of first choice for
the treatment of grandmal (tonic-clonic) seizures. It increases the
levels of GABA by affecting the potassium channels and also creates
direct membrane stabilizing effect. Since it acts through GABA it is the
best medication for grandmal seizures with fewer side effects. It is
administered orally with a dose of 1000-3000 mg in divided doses.
Medication should be taken with food to reduce the GI upset.

Phenytoin and carbamazepine are reasonable second options among the


older group of medications, but the newer medications tend to work
equally well if not better and have better side effect profiles,
especially long-term side effect.
Tiagabine is a weak medication for the treatment of grandmal seizure and
it is effective for the treatment of petitmal seizures.
Phenobarbital is also not the choice of medication for grandmal seizure
but it is a good choice for status epilepticus.
A 75-year-old white male presents with a nonhealing 2.5-cm ulcer on his
neck. A 4-mm punch biopsy reveals a squamous cell carcinoma.

Which one of the following would be most appropriate at this time?

a) Cryosurgery
b) Electrodesiccation and curettage
c) Mohs‟ surgery
d) Surgical excision with 6-mm margins
e) Radiotherapy
The correct answer is D

Explanation
For treatment of squamous cell carcinoma greater than or equal to 20 mm,
surgical excision with 6-mm margins is the preferred treatment. This
will clear 95% or more of these tumors, and provides good cosmetic
results. Cryosurgery or electrodessication can be used for tumors <1 cm.
Mohs‟ surgery would be appropriate if the tumor were high-risk (e.g., if
it were in a location associated with a high risk of metastasis.)
In diabetic patients, osteomyelitis most commonly involves bones in the

a) Foot
b) Spine
c) Leg
d) Arm
e) Hand
The correct answer is A

Explanation
Osteomyelitis is the infection of necrotic bone with virulent bacteria.
The most common site in diabetic patients, and for patients with
peripheral vascular disease, is the foot.

A 35 year old male presents with cough, breathlessness, wheeze and


tightness in his chest. He states that all this has started after 15mins
of his afternoon jogging. Which of the following should be obtained as
the primary test to establish this patient's diagnosis‟

a) X-rays
b) Peak flow rate
c) Allergy test
d) Spirometry
e) Pulse oximetry
The correct answer is D

Explanation
Answer: D ? Spirometry assessments should be obtained as the primary
test to establish the asthma diagnosis. Spirometry should be performed
prior to initiating treatment in order to establish the presence and
determine the severity of baseline airway obstruction.
Spirometry gathers many measurements; the primary ones are:
-the forced expiratory volume in the first second (FEV1);
-the forced vital capacity (FVC);
-the ratio of the FEV1 divided by the FVC (the FEV1/FVC or more simply
the FEV1%);
-the peak expiratory flow (PEF); and
-the forced expiratory time (FET100%).

A. In most patients with asthma, chest radiography findings are normal


or may indicate hyperinflation.
B. Peak-flow monitoring is designed for ongoing monitoring of patients
with asthma because the test is simple to perform and the results are a
quantitative and reproducible measure of airflow obstruction. Peak-flow
monitoring should not be used as a substitute for spirometry to
establish the initial diagnosis of asthma.
C. Allergy skin testing is a useful adjunct in individuals with atopy.
Results help guide indoor allergen mitigation or help diagnose allergic
rhinitis symptoms.
E. Pulse oximetry is a non-invasive way to measure oxygenation of blood
or how well oxygen is being exchanged between the lungs and the blood.
It is not diagnostic.

A 40 year old black female presents to your office complaining of a


persistent dry cough for the last 3 months. This seemed to start with a
“bad chest cold”. She went to an urgent-care facility 6 weeks ago and
the physician prescribed albuterol (Proventil, Ventolin) by metered-dose
inhaler empirically for presumed reactive airways disease. This did not
help. Now she also reports dyspnea on exertion that is noticeable when
walking uphill. She has been taking nitrofurantoin (Macrodantin) for
chronic urinary tract infections, but has an otherwise negative history.
She works as a legal secretary.

On examination, she is tachypneic. There is no cyanosis or clubbing. Her


lungs are clear. Her height is 160 cm (63 in) and her weight is 60 kg
(132 lb). Office pulmonary function tests reveal a forced vital capacity
(FVC) of 1.4 L (average 3.3) and a 1-second forced expiratory volume
(FEV1) of 1.6 L (average 2.8). An inhaled bronchodilator produces no
improvement in these numbers. Which one of the following is the most
likely cause of her problem?
a) Chronic asthma
b) Persistent coughing resulting from a viral respiratory illness
c) Cardiac disease
d) Interstitial lung disease
The correct answer is D

Explanation
This patient has a markedly reduced FVC with an FEV1/FVC ratio is 1.14%.
This is consistent with moderately severe pulmonary restriction. Most
likely the patient has chronic interstitial restrictive lung disease.
Nitrofurantoin can cause this picture, usually after continuous
treatment for 6 or more months, and pulmonary function may be impaired
permanently. A wide variety of additional causes have been described
including noxious gases, pulmonary hypersensitivities, neoplasia, and
systemic diseases (e.g., sarcoidosis). Management includes avoidance of
the offending agent or treatment of the underlying condition.
You are evaluating a 68-year-old male with obstructive urinary symptoms.
Which one of the following medications may lead to falsely depressed
levels of prostate-specific antigen (PSA)?

a) Terazosin (Hytrin)
b) Finasteride (Proscar)
c) Tamsulosin (Flomax)
d) Doxazosin (Cardura)
e) Lycopene
The correct answer is B

Explanation
Finasteride has considerably efficacy in treating obstructive symptoms,
but it unfortunately falsely depresses PSA levels. In patients taking
finasteride, this can affect the evaluation for carcinoma of the prostate.
A 70-year-old male with a history of hypertension and type 2 diabetes
mellitus presents with a 2-month history of increasing paroxysmal
nocturnal dyspnea and shortness of breath with minimal exertion. An
echocardiogram shows an ejection fraction of 25%. Which one of the
patient‟s current medications should be discontinued?

a) Lisinopril (Zestril)
b) Pioglitazone (Actos)
c) Glipizide (Glucotrol)
d) Metoprolol (Toprol-XL)
e) Repaglinide (Prandin)
The correct answer is B

Explanation
According to the current guidelines, thiazolidinediones (TZDs) are
associated with fluid retention, and their use can be complicated by the
development of heart failure. Caution is necessary when prescribing TZDs
in patients with known heart failure or other heart diseases, those with
preexisting edema, and those on concurrent insulin therapy (SOR C).

Older patients can be treated with the same drug regimens as younger
patients, but special care is required when prescribing and monitoring
drug therapy. Metformin is often contraindicated because of renal
insufficiency or heart failure. Sulfonylureas and other insulin
secretagogues can cause hypoglycemia. Insulin can also cause
hypoglycemia, and injecting it requires good visual and motor skills and
cognitive ability on the part of the patient or a caregiver. TZDs should
not be used in patients with heart failure.

Which of the following is correct regarding wernicke's encephalopathy?

a) Is not related to korsakoff‟s psychosis


b) It has a mortality rate of 70 % to 80 % if untreated
c) The treatment is thiamine 100 mg PO daily for 5 days
d) The symptoms include nystagmus and ataxia
The correct answer is D

Explanation
Thiamine deficiency (causing beriberi) is most common among people
subsisting on white rice or highly refined carbohydrates in developing
countries and among alcoholics. Symptoms include diffuse polyneuropathy,
high-output heart failure, and Wernicke-Korsakoff syndrome. Thiamine is
given to help diagnose and treat the deficiency.

Wernicke-Korsakoff syndrome, which combines Wernicke's encephalopathy


and Korsakoff's psychosis, occurs in some alcoholics who do not consume
foods fortified with thiamine. Wernicke's encephalopathy consists of
psychomotor slowing or apathy, nystagmus, ataxia, ophthalmoplegia,
impaired consciousness, and, if untreated, coma and death.

Korsakoff's psychosis consists of mental confusion, dysphonia, and


confabulation with impaired memory of recent events. It probably results
from chronic deficiency and may develop after repeated episodes of
Wernicke's encephalopathy.

For Wernicke-Korsakoff syndrome, thiamine 50 to 100 mg IM or IV bid must


usually be given for several days, followed by 10 to 20 mg once/day
until a therapeutic response is obtained. The mortality rate is 10-20%.
A 45 year old woman after an airline flight, complains vertigo, tinnitus
and moderate hearing loss. Her blood pressure is 160/110. What is the
most likely diagnosis‟

a) Hypertensive crisis
b) Meniere's disease
c) Migraine
d) Acoustic neuroma
e) Barotrauma
The correct answer is B

Explanation
Meniere's disease is an inner ear disorder that produces vertigo,
fluctuating sensorineural hearing loss, and tinnitus. There is no
diagnostic test. Vertigo and nausea are treated with anticholinergics or
benzodiazepines. Diuretics and a low-salt diet may decrease frequency
and severity of episodes. For severe cases, the vestibular system can be
ablated with topical gentamicin or surgery.

In Meniere's disease, pressure and volume changes of the labyrinthine


endolymph affect inner ear function. The etiology of endolymphatic fluid
buildup is unknown. Risk factors include a family history of Meniere's
disease, preexisting autoimmune disorders, allergies, trauma to the head
or ear, and, rarely syphilis (even several decades previously). Peak
incidence is between ages 20 and 50.
Which one of the following should be avoided when managing chronic pain
in elderly nursing-home patients‟

a) Acetaminophen
b) Propoxyphene (Darvon)
c) Hydrocodone (Lortab)
d) Oxycodone (OxyContin)
e) Hydromorphone (Dilaudid)
The correct answer is B

Explanation
According to the Geriatrics Society, propoxyphene is not recommended for
persistent mild to moderate pain. Studies suggest that its efficacy is
similar to that of aspirin or acetaminophen alone, but drug
accumulation, neuroexcitatory effects, and ataxia or dizziness may add
unnecessary morbidity in older patients. Other analgesic strategies are
considered more appropriate for patients with persistent mild to
moderate pain. Acetaminophen, hydrocodone, oxycodone, and hydromorphone
are options for treating chronic pain of varying degrees of severity.

A 20 year old university student comes to the student health center


because of marked fatigue. Temperature is 38.3°C (101.0°F). Physical
examination shows striking pallor of skin, nail beds and conjunctivae.
There are petechial hemorrhages in the skin of his legs. A soft, blowing
systolic murmur is present over the precordium. No other abnormalities
are present. The most appropriate study at this time is

a) Complete blood count


b) Determination of bleeding and clotting time
c) Examination of bone marrow aspirate
d) Hemoglobin electrophoresis
e) Serological testing for infectious mononucleosis
The correct answer is A

Explanation
The description of this patient includes a number of signs and symptoms
consistent with anemia; i.e., marked fatigue, striking pallor, and soft
blowing systolic murmur (flow murmur). Petechial hemorrhages may suggest
a platelet disorder or vasculitis. The most important first study at
this time would be a CBC, which would reveal his hematocrit as well as
his platelet count.
You perform a fundoscopic exam on a patient in the ER and see this
finding. See picture:
What is the most likely diagnosis‟

a) Diabetic retinopathy
b) Non-accidental injury
c) Hypertensive retinopathy
d) Retinal hemorrhage
The correct answer is D

Explanation
Retinal hemorrhage is bleeding onto the surface of the retina caused by
rupture of the tiny blood vessels that lie on the surface of the retina.
Retinal hemorrhage indicates increased pressure within the skull. This
may result from head trauma and bleeding, whether accidental or
intentional (child abuse). It was once believed to be pathognomonic of
so-called 'shaken baby syndrome'.
A 67-year-old female presents with progressive weakness, dry skin,
lethargy, slow speech, and eyelid edema. Of the following medications
currently taken by this patient, which one is most likely to be causing
her symptoms‟

a) Donepezil (Aricept)
b) Lithium
c) Lisinopril (Prinivil, Zestril)
d) Alendronate (Fosamax)
e) Glyburide (DiaBeta, Micronase)
The correct answer is B

Explanation
This patient has classic signs of hypothyroidism. Of the drugs listed,
only lithium is associated with the development of hypothyroidism. In
patients taking lithium, it is recommended that in addition to regular
serum lithium levels, thyroid function tests including total free T ,
and TSH be obtained yearly.
Which one of the following is most likely to trigger or exacerbate acne?

a) Amoxicillin
b) Lisinopril (Prinivil, Zestril)
c) Lovastatin (Mevacor)
d) Phenytoin (Dilantin)
e) Rosiglitazone (Avandia)
The correct answer is D

Explanation
Acne may be triggered or worsened by external factors such as mechanical
obstruction (i.e., helmets, shirt collars), occupational exposures, or
medications. Common medications that may cause or affect acne include
phenytoin.

Chest pain associated with cocaine use

a) should be treated with ß-blockers


b) has an onset that has a consistent relationship with time of use
c) is associated with characteristic EKG changes
d) is mainly vasospastic in origin
The correct answer is D

Explanation
While chest pain related to cocaine use and withdrawal may be due to
atherosclerosis, the main mechanism is believed to be a
dopamine-depleted state, resulting in coronary vasospasm. This vasospasm
can occur at even low doses. Coronary artery vasospasm is exacerbated
by ß-blockade, and is likely mediated through alpha-adrenergic
receptors. Therefore, ß-blockers are contraindicated for patients with
chest pain associated with cocaine use, because of concerns about
unopposed alpha-adrenergic stimulation worsening coronary and peripheral
vasoconstriction, and hypertension. The temporal relationship of cocaine
use to the onset of chest pain is highly variable, which may indicate a
chronic abnormality predisposing to myocardial ischemia. The EKG
abnormalities are variable. A significant number of patients with
cocaine-associated chest pain have evidence of persistent ST-segment
elevation rather than the transient changes typical of coronary artery
spasm.
Which one of the following is most likely to be effective in alleviating
symptoms of restless legs syndrome in the elderly?

a) Haloperidol (Haldol)
b) Naloxone (Narcan)
c) Quinine
d) Carbidopa/levodopa (Sinemet)
e) Magnesium replacement
The correct answer is D

Explanation
Studies suggest dopamine agonists and opioids are among the most
effective medications for the treatment of restless legs syndrome. Since
haloperidol is a dopamine antagonist and naloxone antagonizes opiods,
both medications may aggravate symptoms of restless legs syndrome. The
administration of magnesium has not been shown to be of benefit. While
potentially useful for the treatment of nocturnal leg cramps, quinine
has no role in the treatment of restless legs syndrome.

Which one of the following is most accurate regarding the management of


constipation in the elderly?
a) In chronic pain patients, tolerance to the constipating effects of
opioids develops over time
b) Constipation is a physiologic consequence of normal aging
c) Stool softeners should be prescribed to chronically ill older adults
d) Bulk laxatives are most useful in patients with functional
(normal-transit) constipation
The correct answer is D

Explanation
Constipation is not a physiologic consequence of normal aging. In most
cases, a prophylactic laxative should be considered when prescribing
opioid therapy because tolerance to the constipating effects of the
opiods does not develop over time. Despite the belief that a lack of
fluid increases the risk of constipation, few studies have provided
evidence that hydration is associated with the incidence of
constipation. Patients with functional (normal-transit) constipation
benefit most from treatment with bulk laxatives. Stool softeners are
ineffective in chronically ill older adults.
Which of the following is true of Myasthenia Gravis‟

a) Thymic hypoplasia is well recognized in myasthenic patients.


b) Can be associated with autoimmune thyroiditis
c) Antibodies that are produced against acetylcholinesterase
d) Associated with small cell lung carcinoma
The correct answer is B

Explanation
Myasthenia gravis is an autoimmune disorder of episodic muscle weakness
and easy fatigability caused by antibody- and cell-mediated destruction
of acetylcholine receptors. It is more common among young women and
older men but may occur at any age. Symptoms worsen with muscle activity
and lessen with rest. Diagnosis is by IV edrophonium challenge, which
briefly lessens the weakness. Treatment includes anticholinesterase
drugs, immunosuppressants, corticosteroids, thymectomy, and plasmapheresis.

Once myasthenia is diagnosed, CT or MRI of the thorax should be done to


check for a thymoma. Other tests should be done to screen for autoimmune
disorders frequently associated with myasthenia gravis (eg, vitamin B12
deficiency, hyperthyroidism, RA, SLE).

Immunogenic mechanisms play important roles in the pathophysiology of


myasthenia gravis. Supporting clinical observations include the presence
of associated autoimmune disorders in patients suffering from myasthenia
gravis (eg, autoimmune thyroiditis, systemic lupus erythematosus,
rheumatoid arthritis). The importance of T cells in pathogenesis of
myasthenia gravis is becoming increasingly apparent. The thymus is the
central organ in T cell?mediated immunity, and thymic abnormalities such
as thymic hyperplasia or thymoma are well recognized in myasthenic
patients.

A 75-year-old female is admitted to the hospital with a change in mental


status. The initial workup includes a chemistry profile that reveals a
plasma potassium level of 6.4 mEq/L (N 3.7?5.2). Which one of the
following should be given now to rapidly lower the plasma potassium level?
a) Corticosteroids
b) Albuterol
c) Sodium polystyrene sulfonate
d) 0.45% saline
e) Acute hemodialysis
The correct answer is B

Explanation
Severe hyperkalemia (>6.0 mEq/L) requires aggressive treatment. Calcium
gluconate has no effect on the plasma potassium level, but it should be
given first, as it rapidly stabilizes the membranes of cardiac myocytes,
reducing the risk of cardiac dysrhythmias.
Therapies that translocate potassium from the serum to the intracellular
space should be instituted next, as they can quickly (albeit
temporarily) lower the plasma concentration of potassium. These
interventions include sodium bicarbonate, glucose with insulin, and
albuterol.
Total body potassium can be lowered with sodium polystyrene sulfonate,
but this takes longer to affect the plasma potassium level than
translocation methods. In the most severe cases, acute hemodialysis can
be instituted.
An anxious 40 year old patient, under long-term therapy for bronchial
asthma, is admitted to the Emergency Department because of status
asthmaticus. Therapy should include which one of the following?

a) Inhaled beta-2 agonists, hydration, ipratropium bromide


b) Parenteral aminophylline, sodium cromoglycate
c) Hydration, inhaled beta-2 agonists, parenteral corticosteroids
d) Parenteral corticosteroids, sedative
e) Parenteral aminophylline, parenteral corticosteroids, sedative
The correct answer is C

Explanation
The most severe form of asthma is called status asthmaticus. In this
condition, the lungs are no longer able to provide the body with
adequate oxygen or adequately remove carbon dioxide. Without oxygen,
many organs begin to malfunction. The buildup of carbon dioxide leads to
acidosis, an acidic state of the blood that affects the function of
almost every organ. Blood pressure may fall to low levels. The airways
are so narrowed that it is difficult to move air in and out of the lungs.

Patients who have status asthmaticus receive a beta-adrenergic agonist


and corticosteroids. Status asthmaticus may also require intubation and
ventilator support.
A 38 year old alcoholic woman presents with complaints of epigastric
pain radiating to her back with nausea and vomiting. Physical exam
reveals epigastric tenderness. What is the most likely diagnosis‟

a) Cholangitis
b) Pancreatitis
c) Choledocholithiasis
d) Hepatitis
e) Cholecystitis
The correct answer is B

Explanation
Acute pancreatitis is inflammation of the pancreas (and, sometimes,
adjacent tissues) caused by the release of activated pancreatic enzymes.
The most common triggers are biliary tract disease and chronic heavy
alcohol intake. The condition ranges from mild (abdominal pain and
vomiting) to severe (pancreatic necrosis and a systemic inflammatory
process with shock and multiorgan failure).

Diagnosis is based on clinical presentation and serum amylase and lipase


levels. Treatment is supportive, with IV fluids, analgesics, and fasting.
Intracytoplasmic sperm injection (ISCI) is indicated for which of the
following infertility scenarios‟

a) Woman with scarred fallopian tubes


b) Man with sperm with low motility
c) Woman with bicornuate uterus
d) Man with erectile dysfunction
The correct answer is B

Explanation
Intracytoplasmic sperm injection, is a long, fancy way of saying "inject
sperm into the middle of the egg". ICSI is a very effective method to
get fertilization of eggs in the IVF lab after they have been retrieved
from the female partner. This will usually result in normal
fertilization in approximately 70-85% of eggs injected with viable sperm.

Currently recommendations for vitro fertilization (IVF) with ICSI are for:

1. All couples with severe male factor infertility that do not want
donor sperm insemination.

2. All couples with infertility with: Sperm concentrations of less than


15-20 million per milliliter or sperm motility less than 35% or very
poor sperm morphology.

3. All couples having IVF who have had a previous cycle with no
fertilization - or a low rate of fertilization (low percentage of mature
eggs that are normally fertilized).
A 30 year old female comes in with a soft smooth erythematous nodule on
her lower lip. See picture:

She states that a few weeks prior she had some chapped lips with
occasional bleeding. Now, the lips have healed but this lesion arose
suddenly in its place. It is occasionally tender on pressure. The most
likely diagnosis is

a) HSV1
b) Cherry hemangioma
c) Pyogenic granuloma
d) Dermal nevus
The correct answer is C

Explanation
Pyogenic granuloma can occur at any mucosal location of acute or chronic
trauma, or infection. In the mouth the vast majority of these very
common lesions occur on the gingiva, where they may develop as dumb-bell
shaped masses on the facial and lingual surfaces of the dental arch,
connected by a thin isthmus between adjacent teeth. Other sites of
common involvement include the tongue, the lip mucosa and vermilion, and
the buccal mucosa.

All ages and both genders are susceptible to this exuberant inflammatory
response. The lesion is usually a pedunculated, bright red mass with or
without white areas of surface ulceration; some lesions have a normal
coloration. Rarely does pyogenic granuloma exceed 2.5 cm. in size and it
usually reaches its full size within weeks or months, remaining
indefinitely thereafter.

A 78-year-old white female presents with a 3-day history of lower


thoracic back pain. She denies any antecedent fall or trauma, and first
noted the pain upon arising. Her description of the pain indicates that
it is severe, bilateral, and without radiation to the arms or legs.

Her past medical history is positive for hypertension and controlled


diabetes mellitus. Her medications include hydrochlorothiazide,
enalapril (Vasotec), metformin (Glucophage), and a general multivitamin.
She is a previous smoker but does not drink alcohol. She underwent
menopause at age 50 and took estrogen for ?a few months‟ for hot
flashes. Physical examination reveals her to be in moderate pain with a
somewhat stooped posture and mild tenderness over T12-L1. She has
negative straight-leg raising and normal lower extremity sensation,
strength, and reflexes.

Which one of the following is true regarding this patient‟s likely


condition?

a) An MRI or nuclear medicine bone scan should be performed


b) Prolonged (approximately 2 weeks) bed rest will increase the chance
of complete recovery
c) Investigation for an underlying malignancy is indicated
d) Subcutaneous or intranasal calcitonin (Miacalcin) may be very
helpful for pain relief
The correct answer is D

Explanation
The patient described has a classic presentation of an osteoporotic
vertebral compression fracture. The diagnosis should be confirmed with a
plain radiograph. Treatment is basically symptomatic, with a period of
bed rest as short as possible (to avoid complications of immobility),
pain medication, and bracing.

Salmon calcitonin (injectable or intranasal) is often helpful in


providing pain relief. Long-term management of underlying osteoporosis
may help prevent future fractures.
A 66-year-old male is hospitalized for new-onset atrial fibrillation.
His heart rate is controlled, and he is anticoagulated first with low
molecular weight heparin and then with warfarin (Coumadin). His INR at
discharge is 2.3. He presents 3 days later for follow-up and states that
he feels well. His INR is now 10.0.

The most appropriate management at this time would be to withhold


warfarin until his INR is therapeutic and to

a) readmit for monitoring, give vitamin K, and start enoxaparin (Lovenox)


b) administer 2 units of fresh frozen plasma and vitamin K subcutaneously
c) administer vitamin K intravenously
d) administer vitamin K intramuscularly
e) administer vitamin K orally
The correct answer is E

Explanation
The most cost-effective management for excessive anticoagulation is to
administer oral vitamin K and retest the prothrombin time. This patient
is not hemorrhaging, and a recent analysis has shown oral vitamin K to
be as effective as intravenous or subcutaneous vitamin K. An INR greater
than 8.0 does carry a risk of bleeding, so simply withholding warfarin
would not be appropriate, and neither would readmission, given the high
likelihood of correcting the patient‟s excessive anticoagulation with
oral vitamin K alone.
The tumor marker most closely associated with ovarian cancer is

a) CA-125
b) Carcinoembryonic antigen
c) CA 19-9
d) CD25
e) Alpha-fetoprotein
The correct answer is A

Explanation
The workup of a patient with suspected ovarian cancer includes checking
serum levels of the tumor marker CA-125. While it can be elevated in
some other malignancies, and even some nonmalignant gynecologic
conditions, the CA-125 marker is most closely associated with ovarian
cancer. It is not useful as a screening tool because many women with
early (stage I or II) ovarian cancer have normal CA-125 levels.

CEA is most commonly used to follow colon cancer, while AFP is a marker
for hepatocellular carcinoma. CA 19-9 is elevated in pancreatic, colon,
and breast cancer. CD25 is a marker for certain leukemias.
A 34 year old laborer comes to the office because of a 2-kg (5-lb)
weight loss and an increased appetite. He has diabetes mellitus and has
been taking insulin in divided doses. He says that home monitoring of
his serum glucose concentration has shown values from 280 mg/dL to 320
mg/dL. Which of the following is the most appropriate management?

a) Add metformin
b) Change to another type of insulin
c) Increase his caloric intake
d) Increase his insulin dose
e) Redistribute his caloric intake
The correct answer is D
Explanation
This patient has inadequately treated insulin-dependent diabetes. The
goal serum glucose for therapy is < 120 mg/dL. Since there is no
indication that the insulin type is incorrect (hypoglycemia), the
patient simply requires an increased dosage of his current insulin type.

Metformin is an oral antihyperglycemic agent that acts by increasing


peripheral utilization of glucose. The primary purpose of medications
such as this is to delay or avoid the requirement for exogenous insulin.
Once a patient is on insulin, these drugs are of little benefit.
Secondarily, most patients with non-insulin dependent diabetes have
relative ?insulin resistance? and it is for this reason that glucose
utilization drugs are employed as first line management.

This patient‟s symptoms of increased weight loss and appetite are a


function of his poor glucose control, not of inadequate caloric intake.
Poorly controlled insulin dependent diabetics are often thin as they
waste muscle to liberate glucose in an attempt to provide glucose for
the body. The defect is with the inability to utilize the glucose
already present so the end result is hyperglycemia and muscle wasting.
For similar reasons, there is no need to alter his caloric distribution.
A 37 year old male arrives at the Emergency Department unconscious. He
is warm and sweaty. His heart rate is 52 bpm, his BP is 90/60. His
pupils are constricted, his eyes are teary, and he is drooling. You
assume he is suffering from a toxidrome. What antidote will you give him?

a) Flumazenil
b) Naloxone
c) Glucagon
d) Atropine
e) Ethanol
The correct answer is D

Explanation
This patient is presenting with classic signs and symptoms of toxicity
with an cholinergic drug. The antidote for this is with an
anticholinergic drug such as atropine. Atropine will raise the heart
rate and reverse the other effects of this toxidrome.
A 19-year-old female comes to your office with a skin rash that is
mildly pruritic. The rash began on her back with a single 3-cm
salmon-colored oval lesion. This lesion developed a scaly border and
began to clear centrally. A week later, similar smaller lesions
developed on her back, trunk, and proximal extremities. The long axis of
these oval lesions followed the cleavage lines of the skin. The patient
treated these lesions with over-the-counter hydrocortisone and
diphenhydramine (Benadryl), but they did not resolve.

Which one of the following would be the most appropriate management at


this time?

a) Famciclovir (Famvir)
b) Prednisone
c) Cetirizine (Zyrtec)
d) Terbinafine (Lamisil)
e) Reassurance
The correct answer is E
Explanation
Pityriasis rosea is a self-limited, acute exanthematous skin disorder
that is thought to be viral. It begins with a herald patch that is 3-5
cm in size and usually precedes the rash on the trunk and back by a
week. This is followed by central clearing of the lesion, with
development of a scaly border. Smaller, similar lesions usually develop
in the cleavage lines of the skin. The salmon or pink patches may be
mistaken for tinea corporis, especially when the herald patch is
present. Other illnesses to consider include psoriasis, Lyme disease,
secondary syphilis, drug eruptions, and HIV seroconversion illness. No
laboratory markers are present, and a skin biopsy is rarely needed.

Most patients require only reassurance, although some may need topical
corticosteroids to control itching. For more severe cases, phototherapy
can be considered. For unusually severe cases, erythromycin for 2 weeks
may be helpful. There is one placebo-controlled study of 90 patients
that showed complete rash clearance in 73% of patients who received 2
weeks of erythromycin therapy, compared with no clearance of the rash in
patients who received placebo (evidence level B, single controlled
trial). It is thought that the benefit is from anti-inflammatory and
immune-modulating effects rather than antibacterial effects. The rash
may persist for 2-3 months.
As you walk into your office your nurse asks you to see an 80-year-old
white female who has come on an emergency basis. The patient has a long
history of hypertension and has felt very nauseated and lightheaded
since last night. She denies chest pain and dyspnea.

Physical Findings

Bloodpressure--------------not palpable

Temperature--------------36.5 C (97.7 F)

Pulse--------------40 beats/min

Respirations--------------18/min

Appearance--------------generalized pallor

HEENT--------------within normal limits

Chest--------------bibasilar rales

Heart--------------40/min; no gallop, no murmur

Abdomen--------------soft, no masses

Rectal--------------stool negative for occult blood

Extremities--------------no edema

The patient's EKG shows which one of the following?


a) Pericarditis
b) Sinus bradycardia
c) Acute anteroseptal myocardial infarction
d) Acute inferior wall myocardial infarction
e) Idioventricular rhythm
The correct answer is D

Explanation
There is marked ST-T elevation in the inferior leads consistent with an
acute inferior wall myocardial infarction. Pericarditis almost always
presents with severe chest pain, and the ST segment elevation is more
diffuse. With anteroseptal infarction, ST elevation is seen only on
leads V_1 -V_3 .
An elderly female is on Tamoxifen therapy for advanced breast cancer
with bone metastases. She develops increased thirst, increased
urination, disorientation, nausea, vomiting, confusion and agitation.
What is the most likely cause?

a) Tamoxifen side effect


b) Hypercalcemia
c) Brain metastasis
d) SIADH
The correct answer is B

Explanation
Hypercalcemia is the most common paraneoplastic syndrome associated with
cancer. Symptoms and signs of hypercalcemia may be noted when total
serum Ca is > 12 mg/dL (> 3 mmol/L). These signs can include anorexia,
GI reflux, vomiting, lethargy or seizures or generalized irritability,
and hypertension. Other symptoms and signs include constipation,
dehydration, feeding intolerance, and failure to thrive.
A 30-year-old schoolteacher comes to your office with an abrupt onset of
fever, cough, shaking chills, and myalgias during the previous night.
Earlier this week almost half of her fifth-grade students were absent
from school due to respiratory illness, and the school physician has
announced that the school will be closed for the next few days because
of the large number of children with this illness. On examination, she
has a temperature of 39.5?C (103.0?F), appears fatigued and ill, but is
not in respiratory distress. There is no adenopathy or nasal congestion.
In spite of a severe and painful cough, her chest sounds are clear.

Which one of the following should be done to determine therapy?

a) A chest radiograph
b) A CBC and blood culture
c) A cryoglobulin level
d) A urinary antigen test
e) No further testing
The correct answer is E

Explanation
This patient has an influenza-like illness occurring in an epidemic
setting. The likelihood of influenza is so high that treatment does not
need to await diagnostic testing. A positive rapid test for influenza
could be helpful to document the nature of the epidemic, but a negative
test is not reliable to rule out influenza in an individual patient.
Also, culture results come back too late to aid in clinical decision
making. Treatment is helpful only if started within 48 hours of onset.

A chest radiograph, CBC, and blood culture would be useful for


diagnosing pneumococcal pneumonia, which can develop as abruptly as
influenza, but pneumococcal and other bacterial pneumonias are not
epidemic diseases. Cryoglobulins are used for suggesting Mycoplasma
pneumonia, but the test is not very reliable. Mycoplasma is contagious
but does not occur in the dramatic epidemics that characterize
influenza. Urinary antigen testing is useful for Legionella pneumonia,
which can occur in outbreaks in older adults with COPD, but not in
school-age populations. The decision to treat this patient with
antiviral drugs depends on the severity of the patient‟s illness, the
availability of the medication (which may be depleted during outbreaks),
and personal and financial factors, rather than on any laboratory results.

A patient presents with fever, flank pain, frequency, and dysuria. The
urinalysis (urine microscopy) shows 1+ proteinuria, 25 white blood cells
per high power field and a few granular casts. The next step in
investigation should be

a) Intravenous pyelography
b) Plain x-rays of the abdomen
c) A midstream urine culture
d) Cystoscopy
e) A creatinine clearance
The correct answer is C

Explanation
This patients presentation with fever and WBC on his urinalysis is
strongly suggestive of infection. A urine culture should be ordered when
symptoms indicate the possibility of a urinary tract infection, such as
pain and burning when urinating and frequent urge to urinate.
Which one of the following is a relative contraindication to prescribing
rosiglitazone (Avandia) and pioglitazone (Actos)?

a) Pregnancy
b) Nonalcoholic fatty liver disease
c) Use of insulin
d) Lipodystrophy associated with highly active antiretroviral therapy
e) Anemia
The correct answer is A

Explanation
The thiazolidinediones rosiglitazone and pioglitazone are categorized as
pregnancy category C, due to experimental evidence of growth retardation
in mid- to late gestation in animal models. These drugs are approved for
the treatment of type 2 diabetes mellitus, and may be beneficial in
other conditions associated with insulin resistance. Liver enzyme
elevations are improved in patients with nonalcoholic fatty liver
disease. Although not very effective, the drugs are not contraindicated
in patients with lipodystrophy associated with highly active
antiretroviral therapy. Caution must be used when combining the use of
these drugs with insulin therapy, as the increase in plasma volume that
can be associated with this dual therapy may exacerbate heart failure,
but the combination is not contraindicated. The anemia associated with
these drugs is mild and rarely causes clinical consequences.
In taking the history from a patient with pruritus vulvae, all of the
following diagnoses should be considered, except

a) Squamous cell hyperplasia


b) Vitiligo (noninflammatory)

c) Diabetes mellitus
d) Crohn disease
e) Lichen sclerosis
The correct answer is B

Explanation
Pruritus literally means a sensation of itching and vulva is the name
given to the entrance to the vagina. It is a symptoms, not a disease in
itself. At least 10% of women all over the world suffer from this
complaint. Pruritus vulvae is generally relieved through scratching in
the initial stages. At a later stage, the patient develops a burning
sensation in this region. This can intensify to such an extent that
women suffering from this complaint prefer to remain indoors and refuse
to go out. This problem occurs more during the night. The patient may
scratch the area during sleep and wake to find that she has made herself
bleed.

One of the main causes of pruritus vulvae is purulent and mucopurulent


vaginal discharge. Due to this discharge, the vulva region chafes. The
resulting tenderness causes pain. Over 80% of these cases occur due to
this cause. In some cases prorates vulvae may develop due to the
presence of skin diseases not specific to the vulva such as psoriasis,
seborrheic dermatitis and scabies.

Other causes include animal and vegetable parasite infections which tend
to cause pruritus public rather than prutitus vulvae, conditions of the
urinary track like continence of urine and pyuria. Highly acidic urine
sometime causes soreness which subsequently leads to pruritus.
Glycosuria and diabetes, carcinoma, lichen sclerosis and crohn‟s disease
also contribute to this condition. (Crohn disease is a chronic
granulomatous inflammatory bowel disease that may primarily or
secondarily involve the vulvar (2%) and inguinal regions.)
Note: Vitiligo is an acquired pigmentary disorder of the skin and mucous
membranes, and it is characterized by circumscribed depigmented macules
and patches. It often occurs around body orifices such as the lips,
genitals, gingiva, areolas, and nipples.

What is the diagnostic test of choice to confirm Cushing syndrome?

a) CT of adbomen
b) DST (dexamethasone supression test)
c) Random serum cortisol
d) MRI of brain
The correct answer is B

Explanation
Cushing's syndrome is a constellation of clinical abnormalities caused
by chronic high blood levels of cortisol or related corticosteroids.
Cushing's disease is Cushing's syndrome that results from excess
pituitary production of ACTH, usually secondary to a pituitary adenoma.
Typical symptoms include moon facies and truncal obesity with thin arms
and legs. Diagnosis is by history of receiving corticosteroids or by
elevated serum cortisol.

Diagnosis is confirmed with the dexamethasone test, in which 1, 1.5, or


2 mg of dexamethasone is administered po at 11 to 12 pm and plasma
cortisol is measured at 8 to 9 am the next morning. In most normal
patients, this drug suppresses morning plasma cortisol to ? 1.8 ?g/mL (?
50 nmol/L), whereas patients with Cushing's syndrome virtually always
have a higher level. A more specific but equally sensitive test is to
give dexamethasone 0.5 mg po q 6 h for 2 days (low dose). In general, a
clear failure to suppress levels in response to low-dose dexamethasone
establishes the diagnosis.
A 28-year-old male visits your office because he is planning a ski trip.
You practice in a coastal area, and he plans to be at an altitude of
14,500 feet. On a previous ski trip to the same altitude he experienced
symptoms of headache, poor sleep, anorexia, fatigue, nausea, and
vomiting. He asks you what he can do to prevent these symptoms on his
upcoming trip.

Which one of the following would you recommend?

a) Caffeine avoidance
b) Caffeine tablets
c) Furosemide (Lasix)
d) Acetazolamide (Diamox)
e) Fluid restriction
The correct answer is D

Explanation
This patient experienced acute mountain sickness (AMS), which is the
most common altitude illness. It occurs in 40%?50% of persons from low
altitudes who ascend to 14,000 feet. The onset can occur within 8 to 96
hours of arrival at altitudes above 8000 feet, although the altitudes at
which symptoms begin vary significantly. AMS is a clinical diagnosis,
with the most common symptoms consisting of headache, poor sleep,
anorexia, fatigue, nausea, and vomiting. Slow ascent is the best way to
avoid AMS. Adequate hydration may be helpful. Acetazolamide and
dexamethasone help prevent or mitigate the symptoms of AMS. Individuals
who have had AMS in the past should probably be treated prophylactically
with acetazolamide.

Acetazolamide is a carbonic anhydrase inhibitor that causes a


hyperchloremic metabolic acidosis through the loss of bicarbonate,
sodium, and potassium in the urine. Respiration is stimulated by the
acidosis, which leads to a compensatory respiratory alkalosis.
Pretreatment with this agent mimics the acclimated state of acid-base
balance, so that during the first day of altitude exposure, subjects
taking this drug have values for pH, partial pressure of arterial carbon
dioxide, and minute ventilation that are not typically observed until
day 5 in control subjects.
Which one of the following is the best indicator of adequate control in
a patient with diabetes mellitus‟

a) Fasting blood glucose


b) 24-hour glycosuria
c) Blood glucose 2 hours after meals
d) Glycosylated hemoglobin (hemoglobin A1C)
e) Urine free of glucose
The correct answer is D

Explanation
In the normal 120-day life span of the red blood cell, glucose molecules
join hemoglobin, forming glycated hemoglobin. In individuals with poorly
controlled diabetes, increases in the quantities of these glycated
hemoglobins are noted.

Once a hemoglobin molecule is glycated, it remains that way. A buildup


of glycated hemoglobin within the red cell reflects the average level of
glucose to which the cell has been exposed during its life cycle.
Measuring glycated hemoglobin assesses the effectiveness of therapy by
monitoring long-term serum glucose regulation.

The International Diabetes Federation recommend HbA1c values below 6.5%,


while the American Diabetes Association recommends that the HbA1c be
below 7.0% for most patients. A high HbA1c represents poor glucose control.

Persistent elevations in blood sugar (and therefore HbA1c) increase the


risk for the long-term vascular complications of diabetes such as
coronary disease, heart attack, stroke, heart failure, kidney failure,
blindness, erectile dysfunction, neuropathy (loss of sensation,
especially in the feet), gangrene, and gastroparesis (slowed emptying of
the stomach).
A 74 year old black female has moderately severe pain due to
osteoarthritis. However, she is also on medication for a seizure
disorder. When choosing medications to manage her chronic pain, which
one of the following should be used with caution because of her history
of seizures‟

a) Salsalate (Disalcid)
b) Celebrex
c) Hydrocodone (Lortab)
d) Oxycodone (OxyContin)
e) Tramadol (Ultram)
The correct answer is E

Explanation
According to the clinical practice guidelines for management of
persistent pain in older persons, tramadol has efficacy and safety
similar to those of equianalgesic doses of codeine and hydrocodone.
However, because of the threat of seizures (rare but potential),
tramadol should be used with caution in patients with a history of
seizure disorder or those taking other medications that lower seizure
thresholds.
Each of the following are risk factors for colon cancer, except

a) Low fiber diet


b) Severe diverticular disease
c) Familial adenomatous polyposis
d) Ulcerative colitis
e) High fat diet
The correct answer is B

Explanation
People with a family history of colorectal cancer have a higher risk of
developing the cancer themselves. A family history of polyps also
increases the risk of colorectal cancer. People with ulcerative colitis
or Crohn's disease are at greater risk as well. This risk is related to
the person's age when the disease developed and the length of time the
person has had the disease.

People at highest risk tend to consume a high-fat, low-fiber diet.


Greater exposure to air and water pollution, particularly to industrial
cancer-causing substances (carcinogens), may play a role.
A 70-year-old female consults you about osteoporosis treatment. Two
years ago her T score was ?2.6, and she began taking risedronate
(Actonel), 35 mg/week. Her BMI is 24 kg/m , she takes appropriate doses
of calcium and vitamin D, and she takes walks almost every day. Her
current T score is ?2.5, and she is concerned about the minimal change
in spite of therapy. She has never had a fracture, but asks if more
could be done to reduce her fracture risk.

Which one of the following would be the most appropriate recommendation?

a) Continue current treatment


b) Stop risedronate and start alendronate (Fosamax)
c) Stop risedronate and start teriparatide (Forteo)
d) Add raloxifene (Evista)
e) Order a bone biopsy to evaluate bone architecture
The correct answer is A

Explanation
There is not a linear correlation between bone mineral density and
fracture risk. Bone architecture may be changed by bisphosphonate
therapy, which may result in a decreased fracture risk. This patient has
not had a fracture and is on adequate medical therapy that should be
continued.
All the following can be given to improve sexual function in males, except

a) Sildenafil
b) Estrogen
c) Testosterone
d) Gonadotrophin analogues
The correct answer is B

Explanation
Sexual dysfunction in males can be treated with various drugs. Viagra
(sildenafil) is a popular choice for erectile dysfunction (ED).
Testosterone is used as it promotes and maintains secondary sex
characteristics in androgen-deficient males.

Gonadotropin-releasing hormones have been used in patients with


Gonadotropin-Releasing Hormone Deficiency because these patients can
develop hypogonadism leading to infertility and sexual dysfunction.

Estrogen has no role in the treatment of male sexual dysfunction, but in


females estrogen creams are used for atrophic vaginitis in the elderly.
When interpreting a diagnostic test, the term "gold standard" refers to
a definitive diagnosis obtained by some independent means. All of the
following would be examples of gold standards useful in clinical
practice, except

a) Coronary angiography in evaluating a cardiac stress test


b) Bacteriologic culture in testing a rapid diagnostic kit for gonorrhea
c) Bronchoscopy with biopsy in evaluating sputum cytology
d) Autopsy results in evaluating electrocardiography
e) Results of long-term follow-up in a test for early rheumatoid arthritis
The correct answer is D

Explanation
Coronary angiography in evaluating a cardiac stress test, bacteriologic
culture in testing a rapid diagnostic kit for gonorrhea and bronchoscopy
with biopsy in evaluating sputum cytology are all gold standards in
diagnosis.

Diagnostic studies for rheumatoid arthritis (RA) are hampered by the


lack of an independent gold standard for RA. Since the most important
clinical features of RA are the persistence of the arthritis and the
development of erosions, arthritis outcome is a clinically relevant gold
standard. Diagnosis is a phased, multivariable process in which the
probability of the presence of disease is updated continuously when new
diagnostic information is added to the patient profile.
Which one of the following is a risk factor for acute pancreatitis‟

a) Gastroesophageal reflux disease


b) Intravenous drug abuse
c) Angiotensin receptor blocker use
d) Pyelonephritis
e) Gallstones
The correct answer is E

Explanation
Pancreatitis is most closely associated with gallstones and excessive
alcohol use. Gastroesophageal reflux disease, pyelonephritis, drug abuse
(other than alcohol), and angiotensin receptor blocker use are not risk
factors for the development of pancreatitis.
During the morning rounds, a second year resident presents a patient as
"a 58 year old female who suffers from rigidity and tremors. She has
been diagnosed with a neurodegenerative disease involving the
depigmentation of substantia nigra and loss of dopaminergic input to
basal ganglia."
Regarding treatment of this patient, which of the following statements
is true?

a) Studies demonstrate that a patient's quality of life deteriorates


independently of at what stage of the disease the treatment is instituted.
b) Levodopa is the most efficacious symptomatic medication with few
short-term side effects.
c) Dopamine agonists and levodopa are started at a high dose, in order
to reach therapeutic levels as soon as possible
d) Levadopa has more side effects than dopamine agonists.
e) Anticholinergic medications improve bradykinesia or rigidity.
The correct answer is B

Explanation
This patient suffers from Parkinson‟s disease.
>Studies demonstrate that a patient's quality of life deteriorates
quickly if treatment is not instituted at or shortly after diagnosis
>Levodopa is the most efficacious symptomatic medication with few
short-term side effects, but its chronic use is associated with the
development of fluctuations and dyskinesias. Once fluctuations and
dyskinesias become problematic, they are difficult to resolve.
>Dopamine agonists provide moderate symptomatic benefit and rarely cause
fluctuations and dyskinesias by themselves, but they have more side
effects than levodopa, including sleepiness and impulse control
disorders. However, these side effects resolve upon lowering the dose or
discontinuing the medication.
>Dopamine agonists and levodopa are started at a low dose, escalated
slowly, and titrated to control symptoms.
>For patients who have disability due to tremor that is not adequately
controlled with dopaminergic medication, an anticholinergic agent can be
used. Anticholinergic medications provide good tremor relief in
approximately 50% of patients but do not improve bradykinesia or rigidity.
A 25-year-old female presents to your office with a tick embedded in the
skin on her abdomen. It was likely acquired the previous evening when
she was hiking in the woods near her home. She is currently asymptomatic.

Appropriate management at this point would be to

a) apply a hot match to the body of the tick


b) cover the tick with petroleum jelly
c) remove the tick with blunt angled forceps
d) use topical lidocaine on the site
e) prescribe a 3-week course of doxycycline
The correct answer is C

Explanation
Tick-borne illnesses may be prevented by removal of the parasite within
24?48 hours after attachment to the host. Blunt, medium-tipped, angled
forceps or commercially available devices (but not tweezers) should be
used. The tick should be grasped gently and vertical traction applied
until it dislodges. Using improper technique can leave a portion of the
tick behind, resulting in infection or granuloma formation. Other
methods, such as applying a hot match to the tick, applying or injecting
lidocaine, or passing a needle through the tick, are not recommended. If
no symptoms follow the tick bite, antibiotic therapy is not recommended.
A 48 year old man complains of fatigue and shortness of breath.
Peripheral blood smear reveals macrocytosis. Labs are as follows:

Hematocrit: 32% (Normal 42%-52%)


Hemoglobin: 103 g/L (Normal 140-174 g/L)
Serum vitamin B12 level: 66 pmol/L (Normal 150-750 pmol/L)
Serum folate level: 16 nmol/L (Normal 4-22 nmol/L)

Which one of the following causes listed below is most *unlikely*?

a) Vegetarianism
b) Colonic diverticulitis
c) Regional enteritis
d) Pancreatitis
e) Fish tapeworm infection
The correct answer is B

Explanation
This patient has vitamin B12 deficiency related macrocytic anemia.
Pernicious anemia is a chronic illness caused by impaired absorption of
vitamin B12 because of a lack of intrinsic factor (IF) in gastric
secretions.

Causes include inadequate dietary intake (ie, vegetarian diet), chronic


pancreatitis, tapeworm infestation with Diphyllobothrium latum occurs
from eating poorly cooked lake fish, surgical loss of the ileum or
diseases such as tropical sprue, regional enteritis and ulcerative colitis.
A 72-year-old white female is experiencing pain due to a vertebral
compression fracture. Pain control with opioid analgesics and calcitonin
therapy is not adequate. Which one of the following would make
vertebroplasty an appropriate option?

a) Fracture duration <6 months


b) Degree of vertebral collapse 80%
c) Radiologic evidence of destruction of the posterior vertebral wall
d) New-onset bladder dysfunction thought to have a neurologic etiology
e) New-onset bilateral lower-extremity paresis
The correct answer is A

Explanation
Vertebroplasty is a reasonable therapeutic consideration for vertebral
compression fractures if pain is not adequately controlled with
analgesics and conservative therapy. Some studies indicate a better
response with less chronic fractures. Treatment of fractures less than 6
months old is acceptable. More prolonged conservative therapy with an
inadequate response is not appropriate.

Neurologic dysfunction, including bladder dysfunction, paralysis, and


sensory deficits, is a relative contraindication to vertebroplasty.
Spinal cord compression requires other treatment, and high degrees of
compression (>67%) are not amenable to this therapy. Destruction of the
posterior wall is a contraindication to this therapy because the
injected polymethyl methacrylate should not directly contact the spinal
cord. Coagulopathies and infectious processes are also contraindications.
All of the following are features of toxic shock syndrome, except

a) Fever
b) Hypotension
c) Pustular skin lesion
d) Erythematous rash
e) Tampon use
The correct answer is C

Explanation
Toxic shock syndrome is caused by staphylococcal or streptococcal
exotoxins. Symptoms include high fever, hypotension, diffuse
erythematous rash, and multiple organ involvement that may rapidly
progress to severe and intractable shock.

Women with preexisting staphylococcal colonization of the vagina who use


tampons are at highest risk. Mechanical or chemical factors related to
tampon use likely enhance production of the exotoxin or facilitate its
entry into the bloodstream through a mucosal break or via the uterus.

Diagnosis is made clinically and by isolating the organism. Treatment


includes antibiotics, intensive support, and immunoglobulin.
Etiologic factors in female urinary stress incontinence include all of
the following, except

a) Trauma from repeated deliveries


b) Chronic bronchitis
c) Procidentia
d) Weakness of the urethral supports
e) Estrogen deficiency
The correct answer is C

Explanation
Causes of urinary incontinence in women are abundant and include trauma
from repeated deliveries, coughing from chronic bronchitis, weakness of
the urethral supports and estrogen deficiency. Procidentia is not a
known cause.
Which one of the following can contribute to serum calcium elevation?

a) Furosemide (Lasix)
b) Verapamil (Calan, Isoptin)
c) Enalapril (Vasotec)
d) Hydrochlorothiazide
e) Allupurinol (Zyloprim)
The correct answer is D

Explanation
While thiazide diuretics do not cause hypercalcemia by themselves, they
can exacerbate the hypercalcemia associated with primary
hyperparathyroidism. Thiazides decrease the renal clearance of calcium
by increasing distal tubular calcium reabsorbtion. Furosemide tends to
lower serum calcium levels and is used in the treatment of
hypercalcemia. None of the other medications would be expected to
significantly affect the serum calcium level in this patient.
Which of the following is true with respect to diabetes and kidney disease?

a) Primarily affects the tubules


b) Earliest sign is decreased GFR
c) Microalbuminuria is a late sign of DM nephropathy
d) Threshold for dialysis is same as other CRF patients
e) BP control slows progression of DM nephropathy
The correct answer is E

Explanation
In diabetics it is important to test the urine for protein. Blood tests
can tell how much damage has been done to the kidneys. The most
important things a patient can do to control the progression of their
diabetes is to keep blood pressure lower than 130 over 80. Keep HbA1C
level below 7 percent and leading a balance lifestyle with diet and
exercise.

An elderly man with a history of unresected lung cancer and new onset
decreased level of consciousness is found to have a large cerebral
metastasis with some edema and midline shift on an magnetic resonance
imaging MRI of the brain. Which one of the following would be consistent
with the diagnosis‟

a) Unilateral headache
b) Increased heart rate
c) Increased respiratory rate
d) Reduced blood pressure
e) Reduced heart rate
The correct answer is E

Explanation
Metastatic brain tumor commonly occur in patients with primary cancers
such as lung, breast, GI and melanoma. Patient can present with severe
headache and vomiting. Physical exam will show bradycardia, mild
hypertension papilledema due to increased intracranial pressure.

Diagnosis is by CT or MRI.
A 45-year-old female presents to an urgent care center complaining of
left-sided chest pain for the past 2 days. The pain is nonradiating and
sharp in character, and increases with deep inspiration. She has no
associated shortness of breath, cough, nausea, diaphoresis, or
dizziness. She has no significant past medical history or recent travel
history.

On examination she is afebrile, with a pulse rate of 92 beats/min, a


blood pressure of 116/72 mm Hg, and a respiratory rate of 12/min. Her
lungs are clear and her heartbeat is regular with no murmurs. Her lower
extremities have no edema, tenderness, or varicosities.

Which one of the following is the most appropriate next step in her
evaluation?

a) A high-sensitivity D-dimer test


b) A troponin I level
c) Ultrasound examination of the veins of the lower extremities
d) A multidetector helical CT of the chest
e) An antinuclear antibody (ANA) level
The correct answer is A
Explanation
This patient has a low pretest probability of pulmonary embolism based
on the Wells criteria. She would be a good candidate for a
high-sensitivity D-dimer test, with a negative test indicating a low
probability of venous thromboembolism. In patients with a low pretest
probability of venous thromboembolism, an ultrasound or helical CT would
not be the recommended initial evaluation. Neither troponin I nor an ANA
level would be part of the recommended initial evaluation.
Coronary arteriography would be indicated for which one of the following?

a) A 45-year-old policeman with intermittent chest discomfort


suggestive of angina
pectoris who has a negative exercise stress test
b) A 38-year-old asymptomatic bank officer with a family history of
?heart disease? who
is to undergo hernia repair
c) A 72-year-old female with hypertension, asymptomatic PVCs on EKG,
hyperlipidemia
for which she refuses treatment, and worsening of her longstanding
esophageal reflux
pain
d) A 57-year-old anxious female with sharp anterior chest pain and a
mid-systolic click
and murmur
The correct answer is A

Explanation
Patients with careers that involve the safety of others (e.g., police
officers and pilots) who have questionable symptoms and nondiagnostic
stress testing, with reasonable doubt about significant coronary artery
disease, are candidates for arteriography. A young male in otherwise
good health does not need additional cardiac evaluation. Exercise stress
testing might be indicated for the patient with worsening of her typical
pain if her symptoms are unexplained by evaluation of the
gastrointestinal tract. Patients with mitral valve prolapse do not
usually require catheterization for diagnosis of chest pain.

A 45-year-old female is hospitalized with fever and mild confusion. On


examination she has ascites, and the liver edge is firm and irregular.
Blood is drawn, and paracentesis is performed.

*Laboratory Findings*
Serum albumin.. . . . . . . . . . . . 2.4 g/dL (N 3.1?4.3)
Total protein. . . . . . . . . . . . 4.7 g/dL (N 6.0?8.0)
Ascitic fluid. . . . . . . . . . . . . . cloudy yellow
Cytology.. . . . . . . . . . . . . . negative for malignant cells
Albumin. . . . . . . . . . . . . . . 0.6 g/dL
Protein.. . . . . . . . . . . . . . . 0.9 g/dL
WBCs. . . . . . . . . . . . . . . . 525/mm3 with 75% polymorphonuclear
leukocytes
Amylase. . . . . . . . . . . . . . . 50 U/L

Empiric therapy is started. The next day, gram-negative rods are noted
in the ascitic fluid.

Which one of the following is the most likely diagnosis‟


a) Spontaneous bacterial peritonitis
b) Pancreatic ascites
c) Peritoneal carcinomatosis
d) Tuberculous peritonitis
e) Bacterial peritonitis secondary to bowel rupture
The correct answer is A

Explanation
The serum?ascitic fluid albumin gradient >1.1 indicates that this
patient has ascites due to portal hypertension. The negative cytology
and an ascitic fluid amylase level <100 U/L make peritoneal
carcinomatosis and ascites secondary to pancreatitis unlikely. An
ascitic fluid WBC count >500/mm^3 , with >50% polymorphonuclear
leukocytes, points to bacterial peritonitis. Secondary bacterial
peritonitis is usually polymicrobial, with ascitic fluid total protein
levels >1.0 g/dL. With primary bacterial peritonitis a single organism
is usually seen on a culture, with ascitic fluid total protein levels
<1.0 g/dL.
Which one of the following is least associated with Behcet's syndrome?

a) Mouth ulcers
b) Genital ulcers
c) Conjunctivitis
d) Deep vein thrombosis
e) Aseptic meningitis
The correct answer is E

Explanation
Behçet's syndrome is a multisystem, inflammatory, relapsing, chronic
vasculitic disorder. Common manifestations include recurrent oral
ulcers, ocular inflammation, genital ulcers, and skin lesions.
Blindness, neurologic or GI involvement, venous thromboses, and arterial
aneurysms are the most serious manifestations. Diagnosis is clinical
using international criteria. Treatment is mainly symptomatic but may
involve corticosteroids for acute severe ocular or neurologic
involvement or immunosuppressants for severe chronic features.

Almost all patients have recurrent painful oral ulcers resembling those
of aphthous stomatitis; in most, these ulcers are the first
manifestations. Similar ulcers occur on the penis and scrotum, on the
vulva where they are painful, or in the vagina where they may cause
little or no pain.
Chronic excess thyroid hormone replacement over a number of years in
postmenopausal women can lead to

a) Diffuse nontoxic goiter


b) Osteoarthritis
c) Osteoporosis
d) Hyperparathyroidism
The correct answer is C

Explanation
Even mild chronic excess thyroid hormone replacement over many years can
cause bone mineral resorption, increase serum calcium levels, and lead
to osteoporosis. The elevated calcium decreases parathyroid hormone.
Goiter is an indicator, not a cause, for hormone replacement.
Osteoarthritis is not related to thyroid hormone replacement.
Which one of the following statements regarding blood glucose monitoring
in patients with type 1 diabetes is most correct?

a) Blood glucose testing from a fingertip site is preferred


b) Monitoring blood glucose 5 times per day is too often
c) A random blood glucose level of 100 mg/dL is too high
d) Continuous blood glucose monitoring can result in higher hemoglobin
A_1c levels
The correct answer is A

Explanation
According to the Diabetes Control and Complications Trial, patients
should assess fingertip blood glucose levels at least 3 times per day,
including at bedtime, as well as before and after exercising and before
driving. Although testing from other sites is usually reliable, it can
be inaccurate if performed within 60 minutes of meals or after exercise.
If a patient‟s random blood glucose level is less than 100 mg/dL, a
small snack should be eaten. Preprandial results should be 90?130 mg/dL,
and postprandial levels should be <180 mg/dL. Continuous blood glucose
monitoring has been shown to generally lead to tighter control and lower
hemoglobin A_1c levels.
A 23 year old registered nurse comes to the employee health clinic
because she says, "I'm too tired to work." She has had increasing
fatigue, malaise and anorexia during the past several days. Laboratory
studies show:

ALT 1160 U/L, PT 13 sec, Bilirubin 1.8 mg/dL, HBsAg Positive

She is instructed to rest at home and return in 3 days if no new


symptoms develop. Two days after the visit she calls to say that she has
now developed an urticarial rash and swelling of the joints of her
fingers. At this time which of the following is the most correct
statement about her condition?

a) The arthritis and rash are the result of an associated immune


complex disorder
b) The arthritis and rash are unrelated to her liver disease
c) It is unlikely that her blood is infectious
d) She has a 50% risk for developing chronic liver disease
e) She should be given hepatitis B immune globulin
The correct answer is A

Explanation
In approximately 10% of HBV-infected patients, an immune
complex-mediated „serum sickness-like? reaction is seen. Fever,
glomerulonephritis, arthralgias, and a urticarial maculopapular rash may
also be present. An arthritis-like picture involving the joints of the
hands, wrists, and elbows may also be seen.

These symptoms resolve with the onset of jaundice, and hence are related
to the progress of liver disease. The presence of HBsAg in the blood
implies infectivity via both sexual contact and exposure to blood. In
5-10% of cases, HBV infection progresses to chronic hepatitis from there
there's a 1% chance of progressing to fulminant hepatitis. Once the
patient is already infected, administering hepatitis B immunoglobulin is
of no benefit.
When added to a treatment regimen that includes a ?-adrenergic receptor
blocker, which one of the following can produce bradycardia and
conduction disturbances‟

a) Diltiazem (Cardiazem, Dilacor)


b) Nifedipine (Procardia, Adalat)
c) Terazosin (Hytrin)
d) Clonidine (Catapres)
e) Reserpine
The correct answer is A

Explanation
Verapamil and diltiazem slow the heart rate and can affect
atrioventricular conduction. They should be used with caution in
patients also taking a b-blocker.
A 27-year-old male presents with what he thinks is a sinus infection. He
has a 2-day history of right maxillary pain associated with nasal
congestion and clear rhinorrhea. The only significant findings on
examination are a low-grade fever and subjective tenderness with
palpation over the right maxillary sinus. Which one of the following
treatments is most supported by current evidence?

a) Antihistamines
b) Oral decongestants
c) Topical vasoconstrictor sprays
d) Oral analgesics
e) Nasal lavage
The correct answer is D

Explanation
Although oral antibiotics are overwhelmingly prescribed as initial
treatment in acute sinusitis, it has been shown that the majority of
acute illnesses are viral in origin and that 98% of cases will resolve
spontaneously. Analgesics are considered the mainstay of therapy for
acute sinusitis, according to evidence-based recommendations (SOR A).
Other treatments should be considered if symptoms are prolonged (>7
days) or severe (two or more localizing symptoms or signs of serious
bacterial complications).

There is little evidence of effectiveness for antihistamines, oral


decongestants, or vasoconstrictor sprays. There is also little evidence
of effectiveness for nasal lavage in acute sinusitis, although it has an
emerging role in chronic sinusitis.

In the elderly, which one of the following measures of pulmonary


function is characteristically increased?

a) Total lung capacity


b) Vital capacity
c) Resting PaO2 arterial tension
d) Functional residual capacity
e) One-second forced expiratory volume (FEV1)
The correct answer is D

Explanation
Although vital capacity declines with age, total lung capacity remains
constant. The reduction in vital capacity results from an increase in
residual volume. This increase in residual volume and functional
residual capacity results from the collapse of small airways that occurs
at higher lung volumes as age increases. Residual volume increases
nearly 50% between early adulthood and age 70. In addition, standard
spirometric measurements of lung function (i.e., forced expiratory
volume in 1 second, peak expiratory flow rate, and maximal expiratory
flow volume) have been shown to decline with age. Arterial oxygen
tension also slowly declines with age.
A 75 year old man with a history of nocturia has not micturated for the
last 10 hours, and is complaining of severe lower abdominal pain. The
most likely cause is

a) BPH
b) Prostate cancer
c) Renal failure
d) UTI
e) Prostatitis
The correct answer is A

Explanation
Benign prostatic hyperplasia is nonmalignant adenomatous overgrowth of
the periurethral prostate gland. Symptoms are those of bladder outlet
obstruction, urinary frequency, urgency, nocturia, hesitancy, incomplete
emptying, terminal dribbling, overflow incontinence, or complete urinary
retention. Diagnosis is based on digital rectal examination, cystoscopy
or transrectal ultrasonography
43 year old female with a known history of migraines comes to your
office today complaining of unusual chest pains. There is no family
history of any coronary artery disease, and she is neither a smoker nor
does she have hypertension. She describes the discomfort occurring at
times of rest and during exertion. However the episodes vary, and arise
unexpectedly. She is otherwise healthy with no other complaints. Which
of the following is contraindicated in this patient?

a) Nifedipine

b) Sumatriptan

c) Lisinopril

d) Verapamil

e) Flunarizine
The correct answer is B

Explanation
This female has Prinzmetal variant angina, which is caused by coronary
vasospasms. Sumatriptan is a 5HT1D and 1B agonist, which will have a
vasoconstrictive effect. A side effect of this particular drug is that
it induces vasospasms, hence contraindicated in individuals with this
particular angina.
A 68 year old patient presents with an acutely red and swollen right
great toe without history of trauma. Which one of the following findings
is most useful for making a diagnosis in this patient?

a) Joint aspiration
b) Good response to colchicine trial
c) Radiograph showing marginal joint erosion in the first
metatarsophalangeal (MTP) joint
d) An associated right ankle effusion
e) Painless elbow nodule
The correct answer is A

Explanation
Gout is a disorder that results from deposits of sodium urate crystals,
which accumulate in the joints because of high blood levels of uric acid
(hyperuricemia), leading to attacks of painful joint inflammation.

Attacks of gout (acute gouty arthritis) can occur without warning. They
may be triggered by an injury, surgery, consumption of large quantities
of alcohol or purine-rich food, fatigue, emotional stress, or illness.
Typically, severe pain occurs suddenly in one or more joints, often at
night (probably because of the metabolic changes that occur when a
person lies down). The pain becomes progressively worse and is often
excruciating, particularly when the joint is moved or touched. The joint
becomes inflamed, it swells and feels warm, and the skin over the joint
appears red or purplish, tight, and shiny.

A high uric acid level in the blood supports the diagnosis; however,
this level is often normal, especially during an acute attack. A blood
test may show increased numbers of white blood cells due to the
inflammation caused by the urate crystals. The diagnosis is confirmed
when needle-shaped urate crystals are identified in a sample of a tophus
or in joint fluid removed (joint aspiration) with a needle and viewed
under a microscope with polarized light.
A 68 year old woman comes to the office for flexible sigmoidoscopy as
part of a yearly screening. A 3 cm polyp is found in the sigmoid colon
and is removed. She returns now to the office, 6 hours later,
complaining of left lower quadrant pain, fever, nausea and vomiting.
Vital signs are: temperature 38.1°C (100.6°F), pulse 110/min,
respirations 26/min and blood pressure 120/60 mm Hg. Abdominal
examination discloses bowel sounds, tenderness and guarding in the left
lower quadrant. Rectal examination shows no stool and only tenderness
superiorly. The most appropriate next step is to

a) Obtain an angiogram to rule out intestinal ischemia


b) Obtain immediate consultation with a surgeon
c) Pass a soft rubber rectal tube under fluoroscopy
d) Repeat the flexible sigmoidoscopy in order to evaluate the operative
site
e) Start hydrocortisone, intravenously, to decrease any inflammatory
response
The correct answer is B

Explanation
The physical findings described clearly suggest a probable colon
perforation at the site of the polypectomy, and therefore surgical
consultation is warranted. There is no reason to suspect ischemia, and
any further instrumentation of the colon is contraindicated. The patient
should be prepped for probable surgery with antibiotics and not
hydrocortisone since she is already at risk for peritonitis.

A 30 year old menstruating woman presents at the Emergency Department


with a high fever, vomiting and diarrhea of 24 hours' duration. Her
blood pressure is 90/50 mm Hg and there is a diffuse erythematous rash.
All of the following laboratory results are in keeping with a diagnosis
of toxic shock syndrome, *except*

a) Creatine kinase 290 U/L (N 20?170 U/L)


b) Calcium (total) 3.66 mmol/L (N 2.2?2.74 mmol/L)
c) White blood cell count 18 x 10^9/L (N 3.8?9.8 x 10^9/L)
d) Aspartate aminotransferase (AST) 100 U/L (N 11?47 U/L)
e) Thrombocytopenia
The correct answer is B

Explanation
Toxic shock syndrome is caused by staphylococcal or streptococcal
exotoxins. Symptoms include high fever, hypotension, diffuse
erythematous rash, and multiple organ involvement that may rapidly
progress to severe and intractable shock.

Diagnosis is made clinically and by isolating the organism. Treatment


includes antibiotics, intensive support, and immunoglobulin.

Creatine kinase levels may indicate rhabdomyolysis. Electrolyte levels


may indicate hyponatremia, hypokalemia, *hypocalcemia* out of proportion
to hypoalbuminemia, hypophosphatemia, and hypomagnesemia. The CBC may
reveal leukocytosis, Mild anemia with abnormal cells on smear and
thrombocytopenia. Liver function test results may reveal an elevated
aspartate aminotransferase (AST/SGOT) level.
A 27 year old IV drug abuser presents with cough, fever, and an abnormal
chest x-ray showing hilar and mediastinal adenopathy and bilateral lower
lobe infiltrates. His intermediate Mantoux skin test shows a greater
than 5 mm induration. Which one of the following is the most likely
underlying condition associated with this illness‟

a) Sickle cell disease


b) Alcoholism
c) Drug abuse
d) Human immunodeficiency virus (HIV) infection
e) Hodgkin's disease
The correct answer is D

Explanation
Tuberculosis and HIV have been closely linked since the emergence of
AIDS. HIV infection has contributed to a significant increase in the
worldwide incidence of tuberculosis. By producing a progressive decline
in cell-mediated immunity, HIV alters the pathogenesis of tuberculosis,
greatly increasing the risk of developing disease in co-infected
individuals and leading to more frequent extrapulmonary involvement and
atypical radiographic manifestations.

The clinical presentation of pulmonary tuberculosis can vary widely in


both immunocompetent and immunocompromised hosts. In general, the
presentation in the HIV-infected patient is similar to that seen in
HIV-uninfected patients, although the signs and symptoms (such as
fevers, weight loss, and malaise) may be attributed to HIV itself and
the possibility of tuberculosis overlooked. Symptoms are usually present
for weeks to months, and the acute onset of fever and cough is more
suggestive of a nonmycobacterial pulmonary process. If there is no
response to antimicrobial therapy, however, the possibility of
tuberculosis should be considered. In HIV-infected patients, clinical
manifestations of pulmonary tuberculosis reflect different levels of
immunosuppression.

The chest radiograph is the cornerstone of diagnosis for pulmonary


tuberculosis. Upper lobe infiltrates and cavities are the typical
findings in reactivation tuberculosis, whereas intrathoracic
lymphadenopathy and lower lobe disease are seen in primary tuberculosis.
A 70 year old retired farmer complains of difficulty walking because of
poor balance and numbness in his feet for three months. Neurologic
examination shows normal optic fundi, normal strength in all limbs, and
downgoing plantar responses. His gait is wide-based. He is able to stand
with his feet together and his eyes open, but begins to fall almost
immediately when he closes his eyes. Which of the following tests is
most likely to give the correct diagnosis‟

a) Chest x-ray
b) MRI scan of the spinal cord
c) Contrast enhanced brain CT scan
d) Serum Vitamin B12 level
e) Blood glucose level
The correct answer is D

Explanation
Dietary vitamin B12 deficiency usually results from inadequate
absorption, but deficiency can develop in vegans who do not take vitamin
supplements. Deficiency causes megaloblastic anemia, damage to the white
matter of the spinal cord and brain, and peripheral neuropathy.

Anemia usually develops insidiously. It is often more severe than its


symptoms indicate because its slow evolution allows physiologic
adaptation. Occasionally, splenomegaly and hepatomegaly occur.

Neurologic symptoms develop independently from and often without


hematologic abnormalities. In early stages, decreased position and
vibratory sensation in the extremities is accompanied by mild to
moderate weakness and hyporeflexia. In later stages, spasticity,
extensor plantar responses, greater loss of position and vibratory
sensation in the lower extremities, and ataxia emerge.

Diagnosis is usually made by measuring serum vitamin B12 levels. The


Schilling test helps determine etiology.
Which of the following is *not* an extraintestinal manifestation of an
inflammatory bowel disease?

a) Ankylosing spondylitis
b) Sacroiliitis
c) Primary sclerosing cholangitis
d) Erythema nodosum
e) Dermatitis herpetiformis
The correct answer is E

Explanation
Crohn's disease and UC both can affect organs other than the intestines.
Most extraintestinal manifestations are more common in UC and Crohn's
colitis than in Crohn's disease limited to the small bowel.
Extraintestinal manifestations are categorized in three ways:

1. Disorders that usually parallel (ie, wax and wane with) IBD
flare-ups. These include peripheral arthritis, episcleritis, aphthous
stomatitis, erythema nodosum, and pyoderma gangrenosum. One or more of
these parallel disorders develops in > 1/3 of patients hospitalized with
IBD.

2. Disorders that are clearly associated with IBD but appear


independently of IBD activity. These include ankylosing spondylitis,
sacroiliitis, uveitis, and primary sclerosing cholangitis. Ankylosing
spondylitis occurs more commonly in IBD patients with the HLA-B27 antigen.

3. Disorders that are consequences of disrupted bowel physiology. These


occur mainly in severe Crohn's disease of the small bowel. Malabsorption
may result from extensive ileal resection and produce deficiencies, of
fat-soluble vitamins, vitamin B12, or minerals, resulting in anemia,
hypocalcemia, hypomagnesemia, clotting disorders, and bone
demineralization.

Dermatitis herpetiformis is associated with celiac sprue.

An otherwise healthy 50-year-old male presents to the emergency


department with palpitations and is noted to be in atrial fibrillation.
This resolves without treatment. Total duration was less than 2 hours. A
CBC, metabolic profile, thyroid studies, EKG, and echocardiogram were
all normal.

Which one of the following would be most appropriate?

a) Warfarin (Coumadin)
b) Clopidogrel (Plavix)
c) Cilostazol (Pletal)
d) Dipyridamole (Persantine)
e) Aspirin
The correct answer is E

Explanation
For patients with atrial fibrillation who are at low risk for
embolization (age <65 years and no history of hypertension, diabetes, or
heart failure), the risk for embolization is 1%?2% per year. This can be
reduced to 1% or less by using either warfarin or aspirin, but aspirin
would be preferred because the risk of side effects is lower, as is the
cost.
A man with history of his father dying of prostate cancer comes to see
you to discuss what the guidelines are for prostate screening. You tell
him that for him it is appropriate to

a) Screen every year


b) Screen every 5 years
c) Screen 10 years
d) No screening recommended
The correct answer is A

Explanation
The guidelines for prostate cancer screening are as follows:

Annual screening consisting of a PSA test and digital rectal exam (DRE)
starting at age 40 for blacks, and men with a family history of prostate
cancer.

Men with a PSA level below 4.0 ng/mL should consider a biopsy if the PSA
levels have risen 0.5 ng/mL or more in a year. Men with a PSA level that
has risen > 2.5 ng/mL should have a biospy. All men with an abnormal or
positive DRE result should have a biopsy no matter the PSA level.

Which one of the following has activity against influenza A only?

a) Influenza vaccine
b) Rimantadine (Flumadine)
c) Zanamivir (Relenza)
d) Oseltamivir (Tamiflu)
The correct answer is B

Explanation
The specific virus strains contained in influenza vaccine are determined
annually by the FDA; current vaccines include two influenza A subtypes,
H1N1 and H3N2, and influenza B. Rimantadine is used for the prevention
or treatment of influenza A; it is approved for prophylaxis in those
greater than or equal to 1 year of age, and for treatment in
those greater than or equal to 18 years of age. Zanamivir is used for
the treatment of influenza type A and B in those greater than or equal
to 7 years of age. Oseltamivir is used for the treatment of influenza
types A and B in those greater than or equal to 18 years of age.
A 73-year-old white male has severe COPD manifested by repeating
hospital admissions, 30-step dyspnea on exertion, asthenia, and a
resting pO2 of 58 mm Hg. Of the following, which intervention will most
likely positively affect his survival?

a) Beta2-Agonists
b) Inhaled corticosteroids
c) Oxygen supplementation
d) Pulsed antibiotic therapy
The correct answer is C

Explanation
For patients afflicted with severe COPD, as evidenced by an FEV1 of 1.0
or less and PaO2 of 60 mm Hg or less, only supplemental oxygen has been
shown to positively affect survival, reduce dyspnea scores, and reduce
pulmonary artery pressure. Beta-Agonists and inhaled corticosteroids,
especially when used in combination, are associated with a lower rate of
exacerbations, but have no direct effect on mortality. Smoking
cessation, while clearly of benefit to all, is the only therapy to slow
the decline in lung function but does not affect survival. Finally,
there is no compelling study showing any impact on survival with pulsed
antibiotic therapy.
Which of the following is contraindicated with tyramine rich food (eg
cheese)?

a) Imipramine
b) Phenelzine
c) Fluoxetine
d) Lithium
The correct answer is B

Explanation
Phenelzine (sold as Nardil) is a monoamine oxidase inhibitor (MAOI) used
as an antidepressant drug. As with other MAOIs, tyramine-containing
foods cause a hypertensive crisis, so users of phenelzine should adhere
to certain dietary guidelines, primarily consisting of avoiding certain
aged wines and spirits, and certain aged cheeses. The prescribing
physician will advise patients on this issue. The tyramine induced
hypertensive crises is estimated to effect only a few percent of the
population. It is possible to test patients to see if they are at risk.
A 28 year old woman comes to the office because of fatigue, weakness and
palpitations. She is divorced and lives with her 4 year old daughter.
Complete evaluation shows that this patient has hyperthyroidism and mild
ophthalmopathy caused by Graves disease. Before initiating therapy, the
patient wants to know what she can expect in the future. In advising her
about the prognosis, the most accurate statement is

a) Graves ophthalmopathy will resolve as thyroid hormone secretion is


lowered
b) Malignant degeneration of the thyroid is a common complication
c) She will not be able to become pregnant
d) The thyroid will continue to increase in size with any nonsurgical
treatment
e) Untreated patients are at increased risk for cardiac arrhythmias
The correct answer is E

Explanation
Hyperthyroid patients are at increased risk for cardiac arrhythmias,
typically tachyarrhythmias. Graves‟ ophthalmopathy follows a course that
is independent of thyroid disease. Malignant degeneration is not a
complication of Graves disease. Hyperthyroidism does not preclude the
ability to become pregnant. There is no evidence to suggest that every
Graves thyroid increases in size without surgery.

A 55-year-old patient with a known history of chronic alcohol abuse


presents to your office with a several-day history of shortness of
breath and fever. A physical examination reveals a temperature of 38.8?C
(101.8?F). Diffuse rales are present in the lungs. A chest radiograph
shows diffuse, well-defined nodules that are less than 5 mm in diameter.

Which one of the following is the most likely diagnosis‟

a) Bronchiectasis
b) Reactivation pulmonary tuberculosis
c) Primary pulmonary tuberculosis
d) Miliary tuberculosis
The correct answer is D

Explanation
Diffuse small pulmonary nodules are characteristic of miliary
tuberculosis (TB). This disease is seen commonly in those who are
immunosuppressed due to alcohol abuse. Bronchiectasis may be a
complication of endobronchial TB, but occurs late in the course.
Reactivation pulmonary TB may also be a complication, and the radiograph
typically shows localized lesions. Primary TB usually consists of hilar
adenopathy and a focal infiltrate (Ghon complex). Primary TB is usually
a mild process, but if it becomes severe it usually presents with
extensive lobar consolidation rather than diffuse nodules.
In the evaluation of foot ulcerations, a neuropathic etiology is
suggested by which one of the following?

a) Distal foot pain when supine


b) Absence of toe hair
c) Erectile dysfunction
d) Thickened toenails
e) Abnormal monofilament testing
The correct answer is E

Explanation
Arterial insufficiency is suggested by a history of underlying cardiac
or cerebrovascular disease, leg claudication, impotence, or pain in the
distal foot when the patient is supine (the ?rest pain? syndrome).
Findings of diminished or absent pulses, pallor on elevation, redness of
the foot on lowering of the leg (dependent rubor), sluggish refilling of
toe capillaries, and thickened nails or absence of toe hair are
consistent with impaired arterial perfusions to the foot. Measurement of
cutaneous pressure perception with the use of Semmes-Weinstein
monofilaments has been widely considered an ideal method of screening
because of its simplicity, sensitivity, and low cost.

A 55-year-old male with a 5-year history of type 2 diabetes mellitus


comes to your office for follow-up. He has lost 9 kg (20 lb) in the past
year and reports good control of his glucose levels. His examination is
unremarkable, and his blood pressure is 130/80 mm Hg. Laboratory results
reveal a normal CBC and blood chemistry, and his urine is negative for
microalbumin. His hemoglobin A_1c is 6.5%, while a lipid profile reveals
an HDL level of 42 mg/dL, an LDL level of 98 mg/dL, and a triglyceride
level of 148 mg/dL.

This patient‟s risk of sudden death, myocardial infarction, and stroke


can be decreased by adding

a) niacin
b) lisinopril (Prinivil, Zestril)
c) metoprolol (Lopressor)
d) hydrochlorothiazide
e) simvastatin (Zocor)
The correct answer is E

Explanation
The Heart Protection Study demonstrated a significant decrease in the
rates of major vascular events (myocardial infarction, stroke, and
death) when simvastatin was added to the medical regimen of individuals
at high risk. Benefits for those with coronary artery disease,
peripheral vascular disease, cerebrovascular disease, or diabetes
mellitus, and for males older than 65 with hypertension, were
significant regardless of lipid levels.
A 54-year-old female diabetic who takes fenofibrate (Tricor) has the
following fasting lipid levels: total cholesterol 200 mg/dL, LDL
cholesterol 64 mg/dL, HDL cholesterol 40 mg/dL, triglycerides 525 mg/dL.
Her weight has increased by 3 kg (7 lb). She occasionally checks her
blood glucose and says it is ?okay.? She admits to occasional diarrhea.

In order to improve this patient‟s lipid status, the most appropriate


next step is to

a) add ezetimibe (Zetia)


b) check the patient for celiac disease
c) order TSH and hemoglobin A_1c levels
d) discontinue fenofibrate and prescribe rosuvastatin (Crestor)
The correct answer is C

Explanation
Poorly controlled diabetes mellitus and hypothyroidism are often
associated with a metabolic syndrome that can include a mixed
dyslipidemia. Ezetimibe and rosuvastatin will lower the LDL-cholesterol
level but have little or no effect on triglycerides. Celiac disease is
not a cause of hypertriglyceridemia, but it may be associated with
malabsorption, leading to hypocholesterolemia.

A 65-year-old female presents with a complaint of slowly increasing


discomfort in her knees of 3 years‟ duration. An examination and
radiograph are consistent with noninflammatory osteoarthritis. She says
that the pain is well-controlled with acetaminophen, but she wants to
know what can be done to prevent further damage to the joint.

You recommend

a) referral to a rheumatologist for disease-modifying agents such as


methotrexate
b) hyaluranon injections to preserve cartilage
c) corticosteroid injections
d) symptomatic measures only
The correct answer is D

Explanation
Osteoarthritis is a common finding in older people; some studies show
that 25% of patients over age 65 have osteoarthritic changes.
Unfortunately, no pharmacologic treatments have been found to prevent
the progression of joint destruction. Maintaining ideal weight and
avoiding excessive use of the knees, including deep knee bends, running,
and stair climbing, does lessen destructive forces on the joint. A
reasonable walking program can improve both pain and joint function.

Acetaminophen is the first choice for joint pain in someone with


noninflammatory osteoarthritis. NSAIDs provide better pain relief but
can cause renal damage, fluid retention, and GI bleeding, and are
therefore reserved as a second-line treatment. Narcotics usually are
reserved for short-term use during flares of arthritis. Studies show
that injections of corticosteroids or hyaluranons improve symptoms for
some, but have not been shown to lessen joint destruction.
Disease-modifying agents, such as methotrexate, can help inflammatory
arthritic joints, as in psoriatic arthritis and rheumatoid arthritis,
but have not been shown to be of benefit in osteoarthritis.

About a month after returning from the Middle East, an soldier develop a
papule on his forearm that subsequently ulcerates to form a shallow
annular lesion with a raised margin. The lesion shows no signs of
healing 3 months after is first appeared. He has no systemic symptoms.
The most likely diagnosis is

a) Leishmaniasis
b) Schistosomiasis
c) Malaria
d) Trypanosomiasis
e) Syphilis
The correct answer is A

Explanation
The indolent course of the sore described favors the diagnosis of
cutaneous leishmaniasis. Neither malaria nor schistosomiasis produces
such sores. The chancres of syphilis and trypanosomniasis are more
fleeting in duration.
A 67 year old male comes to your office with severe periumbilical
abdominal pain, vomiting, and diarrhea which began suddenly several
hours ago. His temperature is 37.0°C (98.6° F), blood pressure 110/76 mm
Hg, and respirations 28/min. His abdomen is slightly distended, soft,
and diffusely tender; bowel sounds are normal. Other findings include
clear lungs, a rapid and irregularly irregular heartbeat, and a pale
left forearm and hand with no palpable left brachial pulse. Right arm
and lower extremity pulses are normal. Urine and stool are both positive
for blood on chemical testing. His hemoglobin level is 16.4 g/dL (N
13.0-18.0) and his WBC count is 25,300/mm3 (n 4300-10,800). The
diagnostic imaging procedure most likely to produce a specific diagnosis
of his abdominal pain is

a) Intravenous pyelography (IVP)


b) Sonography of the abdominal aorta
c) A barium enema
d) Celiac and mesenteric arteriography
e) Contrast venography
The correct answer is D

Explanation
The sudden onset of severe abdominal pain, vomiting, and diarrhea in a
patient with a cardiac source of emboli and evidence of a separate
embolic event makes superior mesenteric artery embolization likely. In
this case, evidence of a brachial artery embolus and a cardiac rhythm
indicating atrial fibrillation suggest the diagnosis. Some patients may
have a surprisingly normal abdominal examination in spite of severe
pain. Microscopic hematuria and blood in the stool may both occur with
embolization. Severe leukocytosis is present in more than two-thirds of
patients with this problem. Diagnostic confirmation by angiography is
recommended. Immediate embolectomy with removal of the propogated clot
can then be accomplished and a decision made regarding whether or not
the intestine should be resected. A second procedure may be scheduled to
reevaluate intestinal viability.
A 20 year old man with abrupt onset of hematuria and proteinuria,
accompanied by azotemia and salt and water retention, most likely has
which one of the following?

a) Nephrotic syndrome
b) Multiple myeloma
c) Diabetic nephropathy
d) Nephrolithiasis
e) UTI
The correct answer is A

Explanation
Focal segmental glomerulosclerosis is scattered (segmental) mesangial
sclerosis in some but not all (focal) glomeruli. Focal segmental
glomerulosclerosis (FSGS) is now the most common cause of idiopathic
nephrotic syndrome among adults. It is most often idiopathic but may be
secondary to heroin use, HIV infection, obesity, or nephron loss (eg, in
reflux nephropathy or subtotal nephrectomy). Manifestations are
insidious onset of proteinuria, mild hematuria, hypertension, and
azotemia, mainly in adolescents but also in young and middle-aged adults.

Diagnosis is indicated by history, physical examination, and urinalysis;


it is confirmed by renal biopsy. Treatment is with corticosteroids and
occasionally cytotoxic drugs.
A 42 year old man presents with fatigue and joint pain. On physical exam
he is found to have dark skin and a palpable liver. His urine shows
glucosuria. Which of the following will help you in the diagnosis‟

a) Ferritin level
b) Ceruloplasmin
c) Alpha trypsine
d) Hb Electrophoresis
The correct answer is A

Explanation
Primary hemochromatosis is an inherited disorder characterized by
excessive iron accumulation causing tissue damage. Symptoms do not
develop until organ damage, often irreversible, develops. Symptoms
include fatigue, hepatomegaly, bronze skin pigmentation, loss of libido,
arthalgias, and manifestations of cirrhosis, diabetes, or
cardiomyopathy. Diagnosis is based on serum iron studies and gene assay.
Treatment is with serial phlebotomies.

Because iron accumulates in multiple sites, symptoms can develop


referable to many possible organs or systemically. In women, fatigue and
nonspecific constitutional symptoms develop early; in men, cirrhosis or
diabetes is often the initial presentation.

Serum iron is increased (> 300 mg/dL). Serum transferrin saturation is


usually > 50% and often > 90%. Serum ferritin is increased.
A 59-year-old female with type 2 diabetes develops a 2 x 1-cm ulcer on
the plantar aspect of her right foot. The ulcer is very deep and there
is surrounding cellulites. A plain film is normal.
Which one of the following would be the imaging study of choice to rule
of osteomyelitis in this patient?

a) Angiography
b) A CT scan
c) An MRI scan
d) A PET scan
e) A leukocyte scan
The correct answer is C

Explanation
Although leukocyte scans are sensitive for the diagnosis of foot ulcers,
MRI is now considered the imaging study of choice when osteomyelitis is
suspected; the sensitivity and specificity of MRI in diabetic patients
are 90% or greater.
A 40 year old sedentary white male with a family history of stroke sees
you for a health maintenance visit. His blood pressure averages 150/100
mm Hg over several visits. His LDL cholesterol level is 170 mg/dL. Which
one of the following would have the greatest impact on decreasing his
future risk of stroke?

a) Reducing his blood pressure to normal levels


b) Reducing his LDL cholesterol level to <130 mg/dL
c) Aspirin, 81 mg daily
d) A program of regular exercise
The correct answer is A

Explanation
Stroke is the third leading cause of death in Canada, and hypertension
is the most consistently powerful predictor of stroke. There is strong
and consistent evidence supporting the efficacy of antihypertensive
therapy in reducing stroke risk. Several other interventions have been
shown to lower stroke risk to a lesser extent, including statin
treatment of high cholesterol, daily aspirin, and regular exercise.

Which one of the following screening intervals is most consistent


with Canada Preventive Services Task Force recommendations regarding
screening for abdominal aortic aneurysm?

a) Every 5 years in all patients age 65?75 who have ever smoked
b) Every 5 years in females age 65?75 who have ever smoked
c) One time in males age 65?75 who have ever smoked
d) One time in all patients age 65?75 who have ever smoked
e) No screening is recommended
The correct answer is C

Explanation
In 2005 the Canada Preventive Services Task Force recommended one-time
screening for abdominal aortic aneurysm (AAA) by ultrasonography in men
age 65?75 who have ever smoked. The USPSTF made no recommendation for or
against screening for AAA in men age 65?75 who have never smoked,
because of its lower prevalence in this group, and recommended against
routine screening for AAA in women, because of the lower prevalence of
large AAAs in women.
A 72-year-old white male has new-onset hypertension with a current blood
pressure of 190/110 mm Hg. Which one of the following agents can be used
as part of a test for diagnosing renovascular hypertension, but would
also increase the risk for azotemia if used for treatment?

a) Captopril (Capoten)
b) Metoprolol (Lopressor)
c) Clonidine (catapres)
d) Furosemide (Lasix)
e) Amlodipine (Norvasc)
The correct answer is A

Explanation
ACE inhibitors can significantly worsen renal failure in patients with
hypertension caused by renovascular disease. Hyperkalemia is an
associated problem. Captopril renography is a useful diagnostic
screening test.

The other agents are useful for lowering blood pressure but may cause
mild creatinine elevations. They do not, however, cause the significant
elevations of creatinine seen with ACE inhibitors in cases of
significant renovascular disease.

Which one of the following is true regarding medication dosage


adjustments for patients with chronic kidney disease?

a) Loading doses should usually be adjusted


b) Adjustments typically are not necessary until the glomerular
filtration rate is <20 mL/min/1.73m2
c) A normal serum creatinine value indicates that no adjustment is
necessary
d) Serum drug levels are usually required for making adjustments
e) A reduction of dose, an increase in dosing interval, or both may be
necessary
The correct answer is E

Explanation
Many medications require dosage adjustments in patients with chronic
kidney disease. Medications are adjusted based on the estimated
glomerular filtration rate (GFR) or creatinine clearance. Most
medication adjustments require a reduction in the dose, lengthening of
the dosing interval, or both. Loading doses of medications usually do
not need to be adjusted.

Medication adjustments are divided into three groups, based on whether


the GFR is >50 mL/min/1.73m ,10?50 mL/min/1.73m , or <10 mL/min/1.73m .
The production and excretion of creatinine decreases in older patients,
so a normal serum creatinine level does not always correlate with normal
kidney function. Serum drug levels typically are not required for
adjusting medications in patients with chronic kidney disease.
Intestinal complications more common to Crohn‟s disease than ulcerative
colitis include each of the following, except

a) Fistula formation
b) Perianal disease
c) Intestinal obstruction
d) Toxic megacolon
e) Post-surgical recurrence
The correct answer is D

Explanation
Toxic megacolon (megacolon toxicum) is a life-threatening complication
of other intestinal conditions. It is characterized by a very dilated
colon (megacolon), accompanied by abdominal distension (bloating), and
sometimes fever, abdominal pain, or shock. Toxic megacolon is most
usually a complication of inflammatory bowel disease, such as ulcerative
colitis and, more rarely, Crohn‟s disease, and some infections of the
colon. Other forms of megacolon exist and can be congenital (present
since birth, such as Hirschsprung's disease) or associated with some
forms of constipation.

A 55 year old man who is a smoker, comes in with a complaint of pain in


his right calf. He says that it comes and goes and happens when he walks
up a flight of stairs. What is the next step to diagnose your suspicion
of intermittent claudication?

a) MRI of leg
b) V/Q Scan
c) CT of lumbar spine
d) Ankle Brachial index
The correct answer is D

Explanation
Intermittent claudication, the most common symptom of peripheral
arterial disease, results from gradual narrowing of a leg artery. It is
a painful, aching, cramping, or tired feeling in the muscles of the leg,
not in the joints. Intermittent claudication occurs regularly and
predictably during physical activity but is always relieved promptly by
rest. The muscles ache when a person walks, and the pain begins more
quickly and is more severe when the person walks quickly or uphill.

Usually, after 1 to 5 minutes of rest (sitting is not necessary), the


person can walk the same distance already covered, although continued
walking will again provoke the pain at a comparable distance. Most
commonly, the pain occurs in the calf, but it can also occur in the
thigh, hip, or buttock, depending on the location of the blockage.

The ankle-brachial index is an effective screening tool. The


ankle-brachial index is calculated by dividing the ankle pressure (the
higher of the posterior tibial artery pressures) by the brachial
systolic pressure (the higher of the two arm pressures).

An ankle-brachial index below 0.95 at rest or following exercise is


considered abnormal. An ankle-brachial index between 0.8 and 0.5 is
consistent with intermittent claudication, and an index of less than 0.5
indicates severe disease.

On examining a 51 year old man with chest pain who was just involved in
a motor vehicle accident, you find that at the end of expiration his
blood pressure is 130/90 mmHg and at the end of inspiration it is 110/92
mmHg. Which one of the following conditions is most likely?
a) Cardiac tamponade
b) Pneumothorax
c) Multiple rib fractures
d) Acute myocardial infarction
e) Aortic rupture
The correct answer is A

Explanation
Beck's triad is a collection of three medical signs associated with
acute cardiac tamponade, an emergency condition wherein fluid
accumulates around the heart and impairs its ability to pump blood. The
result is the triad of low arterial blood pressure (hypotension),
increased central venous pressure (JVD), and distant (or muffled) heart
sounds.

This patient also exhibits pulsus paradoxus. This is an exaggeration


(>12 mm Hg or 9%) of the normal inspiratory decrease in systemic blood
pressure.
A 70 year old black male who has been hospitalized for 2 1/2 weeks for
congestive heart failure develops severe, persistent diarrhea. For the
past 3 days he has had abdominal cramps and profuse semi-formed stools
without mucus or blood.

The patient‟s current medications include captopril (Capoten), digoxin,


furosemide (Lasix), subcutaneous heparin, sprionolactone (Aldactone),
and loperamide (Imodium). He has coronary artery disease, but has been
relatively pain free since undergoing coronary artery bypass surgery 4
years ago. An appendectomy and cholecystectomy were performed in the
past, and the patient has since been free of gastrointestinal disease.

On physical examination his blood pressure is 100/80 mm Hg, pulse 100


beats/min and regular, and temperature 37.0°C (98.6°F). He has mild
jugular venous distension and crackles at both lung bases. Examination
of his heart is unremarkable, although there is 1+ dependent edema. His
abdomen is diffusely tender without masses or organomegaly. Rectal
examination is normal.

The results of routine laboratory tests, including a CBC, chemistry


profile, EKG, and urinalysis, are al normal. The stool examination shows
numerous white blood cells. Of the following, the most likely diagnosis is

a) Viral gastroenteritis
b) Clostridium difficile colitis
c) Ulcerative colitis
d) Gluten-sensitive enteropathy (celiac sprue)
e) Digoxin toxicity
The correct answer is B

Explanation
This patient most likely has Clostridium difficile colitis, suggested by
semiformed rather than watery stool, fecal leukocytes (not seen in viral
gastroenteritis or sprue), and a hospital stay greater than 2 weeks.
While this disease has traditionally been associated with antibiotic
use, it is posing an increasing threat to patients in hospitals and
chronic-care facilities who have not been given antibiotics. The primary
sources for infection in such cases have been toilets, bedpans, floors,
and the hands of hospital personnel. Prompt recognition and treatment is
essential to prevent patient relapse and to minimize intramural
epidemics. The diarrhea of ulcerative colitis usually contains blood and
occurs intermittently over a protracted course. Digoxin toxicity is
likely to be accompanied by electrocardiographic and laboratory
abnormalities, particularly hyper- or hypokalemia.
What is the best method of diagnosing an aortic dissection?

a) MRI
b) CT scan
c) TEE
d) CXR
The correct answer is A

Explanation
Aortic dissection is the surging of blood through a tear in the aortic
intima with separation of the intima and media and creation of a false
lumen. The intimal tear may be a primary event or secondary to
hemorrhage within the media. The dissection may occur anywhere along the
aorta and extend proximally or distally into other arteries.

Hypertension is an important contributor. Symptoms and signs include


abrupt onset of tearing chest or back pain, and dissection may result in
aortic regurgitation and compromised circulation in branch arteries.

Diagnosis is by imaging tests (eg, transesophageal echocardiography, CT


angiography, MRI, contrast aortography). MRI is the best test for diagnosis.

Treatment always involves aggressive BP control and serial imaging to


monitor progression of dissection, surgical repair of the aorta and
placement of a synthetic graft is needed for ascending aortic dissection
and for certain descending aortic dissections.
You suspect a secondary cause in one of your patients who has developed
hypertension. Which of the following tests would not be part of your
diagnostic work-up?

a) CT of abdomen
b) 5-hydroxyindoleacetic acid
c) Urine catecholamines
d) Renal ultrasound
e) Cortisol level
The correct answer is B

Explanation
Only 5% of causes of hypertension are from a secondary cause. Remember
the mnemonic C.H.A.P.S. for the causes of secondary hypertension. Those
are Cushings, Hyperaldosteronism, Aortic coarctation, Pheochromocytoma,
Stenosis of renal artery.

The diagnostic tests used to work up the above causes of secondary


hypertension are cortisol level, basic metabolic panel, CT of abdomen,
urinary catecholamines, renal ultrasound for Cushings,
Hyperaldosteronism, Aortic coarctation, Pheochromocytoma and Stenosis of
renal artery, respectively.

5-hydroxyindoleacetic acid (5-HIAA) is a break-down product of the


chemical messenger serotonin in the urine. 5-HIAA levels to detect
tumors in the digestive tract (carcinoid tumors).
Which of the following is not a cause of sensorineural hearing loss

a) Ossicular discontinuity
b) Ototoxicity
c) Méniére disease
d) Noise
e) Presbycusis
The correct answer is A

Explanation
Hearing loss also may be caused by damage to the sensory structures
(hair cells) of the inner ear, auditory nerve, or auditory nerve
pathways in the brain (sensorineural hearing loss). These sensory
structures may be damaged by drugs, infections, tumors, and skull injuries.

Causes of sensorineural hearing loss include aging, brain tumors,


certain drugs (ototoxicity), presbycusis, loud noise, Meniere's disease
and sudden pressure changes from flying, diving, and strenuous exercise.
A 28 year old woman presents to you with a three month history of
fatigue, weight gain, menstrual irregularity and declining memory. Which
of the following laboratory results is likely in the diagnosis of the
underlying problem?

a) TSH increased, free T4 increased


b) TSH decreased, free T4 increased
c) TSH increased, free T4 decreased
d) TSH decreased, free T4 decreased
e) TSH within normal range, free T4 within normal range
The correct answer is C

Explanation
Hypothyroidism is common, especially among older people, particularly
women; it affects about 10% of older women. It can, however, occur at
any age.

Insufficient thyroid hormone causes body functions to slow. Symptoms are


subtle and develop gradually. They may be mistaken for depression,
especially among older people. Facial expressions become dull, the voice
is hoarse and speech is slow, eyelids droop, and the eyes and face
become puffy. Many people with hypothyroidism gain weight, become
constipated, and are unable to tolerate cold. The hair becomes sparse,
coarse, and dry, and the skin becomes coarse, dry, scaly, and thick.

Usually hypothyroidism can be diagnosed with one simple blood test: the
measurement of thyroid-stimulating hormone, which will be high due to a
lack of negative feedback. A second blood test is needed to measure the
level of the thyroid hormone T4 that is not bound by protein (free). A
low level confirms the diagnosis of hypothyroidism.
The most effective means of preventing sudden death in high-risk
patients with asymptomatic hypertrophic cardiomyopathy is

a) amiodarone (Cordarone)
b) metoprolol (Lopressor)
c) verapamil (Calan, Isoptin)
d) chronic dual-chamber pacing
e) an implantable cardioverter-defibrillator (ICD)
The correct answer is E

Explanation
Many patients with hypertrophic cardiomyopathy (HCM) never have any
clinical signs or symptoms. The major cause of mortality is sudden
death, which can occur in both asymptomatic and symptomatic patients,
often after physical exertion. Patients with HCM should be counseled
about the risk of competitive sports and dehydration. Medications such
as verapamil, beta-blockers, and diltiazem are used for symptom
management, but do not decrease the risk of sudden death. Because of its
effects on decreasing dysrhythmias, amiodarone may decrease the risk of
sudden death, which is supported by anecdotal data.

For most patients with HCM, the annual risk of dying is similar to that
of the normal adult population, or 1% per year. Patients most at risk
for sudden death include those with ventricular tachycardia on an
ambulatory monitor, marked left ventricular hypertrophy, abnormal blood
pressure response to exercise, syncope, and a family history of sudden
death. At present, the implantable cardioverter-defibrillator (ICD) is
the most effective modality for preventing sudden death in high-risk
patients with asymptomatic HCM. Pacing does not reduce risk significantly.
Which of the following is a feature of secretory diarrhea?

a) Small stool volume (< 1L/day)


b) Increased stool osmotic gap
c) Persistent diarrhea despite fasting
d) Blood and/or pus in stools
e) Malodorous, often floating stools
The correct answer is C

Explanation
Secretory diarrhea means that there is an increase in the active
secretion, or there is an inhibition of absorption. There is little to
no structural damage. The most common cause of this type of diarrhea is
a cholera toxin that stimulates the secretion of anions, especially
chloride ions. Therefore, to maintain a charge balance in the lumen,
sodium is carried with it, along with water. Secretory diarrhea is not
relieved by fasting.
An 80 year old man presents to the emergency room complaining of high
fever and headache.

The condition started yesterday and is worsening progressively. He


vomited thrice this morning. His past medical history is significant for
high blood pressure and diabetes controlled with losartan and insulin
respectively.

Vital signs show: pulse is 100/min, Temp is 40C, BP is 135/85 mm Hg and


RR is 16/min. Confusion is well noted while talking to the patient.
Neuro exam is significant for stiff neck, positive Kernig‟s and
Brudzinsky‟s signs with no focal neurologic deficit. After blood
cultures and LP are performed, what would you do next for this patient?

a) Ceftriaxone
b) Ceftriaxone + Ampicillin
c) Ceftriaxone + Vancomycin
d) Ceftriaxone + Vancomycin + Ampicillin
e) Wait for the results of the culture
The correct answer is D

Explanation
The patient‟s clinical scenario is highly suggestive of bacterial
meningitis. The most common precipitant organisms in the adult age group
are Strep pneumonie, H. influenza and N. meningitidis.

Ceftriaxone alone is not appropriate because, recently, pneumococcus


have become resistant to penicillins and cephalosporins; thus,
vancomycin should be added to ceftriaxone.

In an elderly patient, Listeria monocytogenes should be covered;


Ampicillin should be added. In both extremes of age, ampicillin should
be added to the regimen.

If you want to kill the patient and be sued and finally lose your
license then wait for the results of the culture. Empiric antibiotics
are essential for suspected meningitis!
A 70 year old white female comes to your office for an initial visit.
She has taken levothyroxine (Synthroid), 0.3 mg/day, for the last 20
years. Although a recent screening TSH was fully suppressed at <0.1
microU/mL, she claims that she has felt ?awful? when previous physician
have attempted to lower her dosage. You explain that a serious potential
complication of her current thyroid medication is

a) Adrenal insufficiency
b) Carcinoma of the ovary
c) Carcinoma of the thyroid
d) Hip fracture
e) Renal failure
The correct answer is D

Explanation
Women older than 65 years of age who have low serum TSH levels,
indicating physiologic hyperthyroidism, are at increased risk for new
hip and vertebral fractures. Use of thyroid hormone itself does not
increase the risk of fracture if TSH levels are normal.
At a routine visit, a 50 year old white female with a 10 year history of
type 2 diabetes mellitus has a blood pressure of 145/90 mm Hg and
significant microalbuminuria. Which one of the following would be an
absolute contraindication to use of an ACE inhibitor in this patient?

a) A previous history of angioneurotic edema


b) Renal insufficiency
c) Asthma
d) A history of recent myocardial infarction
e) A cardiac ejection fraction <25%
The correct answer is A

Explanation
Angioneurotic edema can be life-threatening, and ACE inhibitors should
not be given to patients with a history of this condition from any
cause. Elevated creatinine levels are not an absolute contraindication
to ACE inhibitor therapy. Myocardial infarction and a reduced cardiac
ejection fraction are indications for ACE inhibitor therapy. ACE
inhibitors do not affect asthma.
Which of the following beta blockers used to treat hypertension is
contraindicated in an asthma patient?

a) Acebutolol
b) Atenolol
c) Esmolol
d) Metoprolol
e) Labetalol
The correct answer is E

Explanation
Beta blockers are either beta 1 selective where they only block the
receptors on the heart. Or they can be non-selective and block both beta
1 and beta 2 receptors. Blockade of beta 1 receptors leads to a decrease
in heart rate. Blockade of the beta 2 receptor leads to
bronchoconstriction. This should be avoided in asthmatic patients.

Therefore it is best to only given beta 1 selective beta blockers to


asthma patients when treating their hypertension. This group includes
the following four: acebutolol, atenolol, esmolol and metoprolol.
A 32 year old man comes presents with increased shortness of breath and
he has had a cough for the past week. He appears dyspneic and has a
temperature of 38.3 C (101.0 F). On physical examination he has
bibasilar rales and generalized lymphadenopathy. Rectal examination
shows multiple perianal contusions and a small amount of blood oozing
from the anal orifice. Chest x-ray film shows bilateral patchy alveolar
infiltrates. The most appropriate next step is

a) Admit him to the hospital and begin administration of


trimethoprim-sulfamethoxazole, intravenously
b) Admit him to the hospital and begin administration of penicillin and
gentamicin, intravenously
c) Begin administration of erythromycin, orally, and see him again the
next day
d) Prescribe isoniazid and rifampin, orally
e) Recommend aspirin, fluids and rest at home
The correct answer is A

Explanation
The history and physical raises the possibility of HIV infection. There
is a strong suggestion of homosexuality, given that the rectal
examination demonstrates multiple perianal contusions and blood oozing
from the anal orifice. Furthermore, this man has generalized
lymphadenopathy; his chief complaint is of pulmonary distress with an
x-ray consistent with Pneumocystis carinii pneumonia (PCP). Since the
patient has become increasingly short of breath and is febrile, he
should be treated with intravenous therapy. Treatment of choice for PCP
is Bactrim (trimethoprim-sulfamethoxazole).
The treatment of choice for absence seizures is

a) Lamotrigine
b) Ethosuximide
c) Phenobarbital
d) Phenytoin
The correct answer is B

Explanation
Absence seizures (formerly called petit mal) consist of 10 to 30 second
loss of consciousness with eyelid fluttering; axial muscle tone may or
may not be lost. Patients do not fall or convulse; they abruptly stop
activity, then just as abruptly resume it, with no postictal symptoms or
knowledge that a seizure has occurred.

Absence seizures are genetic and occur predominantly in children.


Without treatment, such seizures are likely to occur many times a day.
Seizures often occur when patients are sitting quietly, can be
precipitated by hyperventilation, and rarely occur during exercise.
Atypical absence seizures last longer, are accompanied by more
pronounced jerking or automatic movements, and cause less complete loss
of awareness.

Treatment is with ethosuximide and also with valproic acid.


A 35 year old woman complains of numbness in her face and a pain deep
behind her left eye. She describes weakness in her upper extremities and
a "clumsy right hand." She says she intermittently wets her pants. She
emphasizes that the symptoms are all worse when the weather is hot.
Physical examination discloses increased deep tendon reflexes in both
biceps and triceps. There is temporal pallor and partial atrophy of the
left optic nerve head. Which of the following studies will most likely
confirm the diagnosis‟

a) Cerebrospinal fluid analysis


b) Electromyography
c) MRI of the head
d) Serum protein electrophoresis
The correct answer is C

Explanation
This patient complains of a variety of symptoms, but they are
localizing. She complains of paresthesias and dysesthesias in her right
hand, this in association with increased deep tendon reflexes of her
upper extremities. She also has pain behind her left eye in association
with partial atrophy of her left optic nerve. This patient has classic
symptoms of multiple sclerosis and with inflammation of the orbital
portion of the optic nerve, usually unilateral. Weakness is a very
important symptom in multiple sclerosis, and is particularly important
when it occurs in only one area of the body (localized or focal
weakness). MRI is the diagnostic modality of choice to ascertain the
presence of the characteristic demyelinating lesions.
A 62 year old female with numbness in the lower extremities and
macrocytosis has a normal serum folate level and a serum B12 level of
200 pg/mL (N 150-800). Which one of the following laboratory findings
would confirm the diagnosis of B12 deficiency?
a) Elevated angiotensin converting enzyme
b) Elevated methylmalonic acid
c) Elevated free erythrocyte protoporphyrin
d) Low haptoglobin
e) Low homocysteine
The correct answer is B

Explanation
This patient has several clinical features of vitamin B12 deficiency.
Some patients with significant vitamin B12 deficiency have levels in the
lower range of normal, as this patient does. Vitamin B12 is a cofactor
in the synthesis of both methionin and succinyl coenzyme A, and vitamin
B12 deficiency leads to the accumulation of methylmalonic acid and
homocysteine, which are the precursors of these compounds. An elevated
level of these substances is therefore more sensitive than a low vitamin
B12 level for vitamin B12 deficiency. Homocysteine is also elevated in
folic acid deficiency, however, a methylmalonic acid level is
recommended if vitamin B12 deficiency is a concern and serum vitamin B12
levels are 150-400 pg/mL.

A reduced haptoglobin level is useful to confirm hemolytic anemia. An


elevated free erythrocyte protoporphyrin level may occur in lead
poisoning or iron deficiency. An elevated angiotension converting enzyme
level is found in sarcoidosis.
On an annual screen, a patient with type 2 diabetes mellitus is found to
have microalbuminuria. Which one of the following therapies should be
initiated?

a) A diuretic
b) An b-blocker
c) A b-blocker
d) A calcium channel blocker
e) An ACE inhibitor
The correct answer is E

Explanation
In microalbuminuric patients, blood glucose control should be improved
as much as possible to delay progression to persistent proteinuria. If
glycemic control is adequate and the increased urinary
albumin/creatinine ratio persists, ACE inhibitor therapy should be
started. Initiation of an ACE inhibitor is appropriate in both
normotensive and hypertensive diabetic patients.
A 60-year-old male has moderate anemia, with a suggestion of hemolysis
on a peripheral blood smear. Which one of the following patterns would
be consistent with the presence of hemolysis‟

a) Elevated LDH, decreased haptoglobin, elevated indirect bilirubin


b) Elevated LDH, elevated haptoglobin, decreased indirect bilirubin
c) Decreased LDH, elevated haptoglobin, elevated indirect bilirubin
d) Decreased LDH, decreased haptoglobin, elevated indirect bilirubin
e) Decreased LDH, decreased haptoglobin, decreased indirect bilirubin
The correct answer is A

Explanation
Hemolytic anemia is established by reticulocytosis, increased
unconjugated bilirubin, elevated lactate dehydrogenase (LDH), decreased
haptoglobin, and peripheral blood smear findings.

An elderly female patient has been hospitalized for urosepsis. She was
started on Piperacillin/tazobactam empirically. One week later, the
patient recovered and was discharged on oral antibiotics. 6 days later,
she developed severe watery diarrhea associated with abdominal colic and
fever.

Vitals reveal: pulse is 110/min, Temp is 39C, BP: 120/70mm Hg and RR is


16/min. abdominal exam reveals tenderness in the lower abdomen with no
evidence of peritoneal signs.

CBC shows a WBC count of 16,000/µL. After adequate fluid rehydration,


what is the most appropriate next step in the management of this patient?

a) Give metronidazole and continue the antibiotics


b) Stop the antibiotics
c) Stop the antibiotics and give IV metronidazole
d) Stop the antibiotics and give oral metronidazole
e) Stop the antibiotics and give oral vancomycin
The correct answer is D

Explanation
Pseudomembrenous colitis is a very hot topic for the licensing exams!

Our patient has developed severe watery diarrhea, abdominal pain, and
fever while taking antibiotics for long duration. She is most likely
suffering from pseudomembranous colitis caused by Clostridium difficile.
After proper diagnosis with Cytotoxin assay of the stool, oral
metronidazole is the antibiotics of choice for this condition.

The antibiotics should be always stopped; however, discontinuing the


antibiotics alone is not enough, the patient is complaining of severe
symptoms.

IV metronidazole is not as effective as oral metronidazole; we need the


highest concentration of the drug in the colon!

Oral vancomycin is a very effective medication against C. diff; however,


it is an expensive drug and we do not want to use it too much in order
to decrease the incidence of vancomycin resistant enterococci.
A 72-year-old white male in otherwise good health complains of
generalized pruritus that worsens in the winter. The itching is most
intense after he bathes. He recently noticed a rash on his abdomen and
legs as well. On examination you note poorly defined red, scaly plaques
with fine fissures on the abdomen. No eruption is present at other
pruritic sites. Which one of the following is the most likely cause of
this problem?

a) Stasis dermatitis
b) Lichen simplex chronicus
c) Xerosis
d) Rosacea
e) Candidiasis
The correct answer is C
Explanation
Xerosis is a pathologic dryness of the skin that is especially prominent
in the elderly. It is probably caused by minor abnormalities in
maturation of the epidermis that lead to decreased hydration of the
superficial portion of the stratum corneum. Xerosis often intensifies in
winter, because of the lower humidity and cold temperatures.

Stasis dermatitis, due to chronic venous insufficiency, appears as a


reddish-brown discoloration of the lower leg. Lichen simplex chronicus,
the end result of habitual scratching or rubbing, usually presents as
isolated hyperpigmented, edematous lesions, which become scaly and
thickened in the center. Rosacea is most often seen on the face as an
erythematous, acneiform eruption, which flushes easily and is surrounded
by telangiectasia. Candidiasis is an opportunistic infection favoring
areas that are warm, moist, and macerated, such as the perianal and
inguinal folds, inframammary folds, axillae, interdigital areas, and
corners of the mouth.
A healthy 72 year old female comes to your office for a follow-up visit.
She has hypertension which is well controlled with an ACE inhibitor.
Routine laboratory tests are normal except for a serum calcium level of
10.8 mg/dL (N 8.5-10.5). A repeat calcium level is 11.1 mg/dL. Which one
of the following would be most appropriate at this point?

a) Radiographs of the hands


b) An osteocalcin level
c) An intact parathyroid hormone (PTH) level
d) Bone densitometry
The correct answer is C

Explanation
In primary hyperparathyroidism, hypercalcemia is the result of excessive
PTH secretion by one or more abnormal, enlarged parathyroid glands.
Laboratory findings in most patients with primary hyperparathyroidism
reflect the mild clinical presentation of the disorder. The serum
calcium level is often 1 mg/dL or more above the upper limits of normal.

Bone radiographs may show the classic changes of subperiosteal bone


resorption in the occasional patient with hyperparathyroidism, but in
most cases they are normal or may show osteopenia. Osterocalcin is an
osteroblast-specific protein. It is a marker of increased skeletal
turnover, and it is usually not indicated clinically. The development of
highly sensitive and specific assays for intact, largely active PTH has
simplified the assessment of parathyroid activity. Bone densitometry is
a test to determine the degree of osteoporosis.
A 70 year old man presents with recurrent bilateral upper lobe pneumonia
with mild dysphagia. On exam he is otherwise normal. What is the
procedure of choice to arrive at the diagnosis‟

a) Barium swallow
b) Serum protein electrophoresis
c) CBC
d) ESR
The correct answer is A

Explanation
Swallowing disorders are common, especially in the elderly, and may
cause dehydration, weight loss, aspiration and airway obstruction.

Aspiration is the passage of food or liquid through the vocal folds.


Persons who aspirate are at increased risk for the occurrence of serious
respiratory sequelae, including airway obstruction and aspiration pneumonia.

Diagnostic study of choice would be a barium swallow. The barium will


show any structural defects such as strictures, narrowing or tumors.
A female teenager comes with complaint of acne on her face. Physical
exam shows open and closed comedones with two papules on her chin. You
diagnose her with mild acne vulgaris. What is most appropriate initial
the treatment?

a) Topical benzoyl peroxide


b) Oral clindamycin
c) Oral minocycline
d) Oral accutance
The correct answer is A

Explanation
Mild inflammatory acne should be treated with topical benzoyl peroxide
and/or topical antibiotics (eg, erythromycin, clindamycin).

Moderate acne responds best to oral systemic therapy with antibiotics.


Antibiotics effective for acne include tetracycline, minocycline,
erythromycin, and doxycycline.

Severe acne: Oral isotretinoin (accutane) is the best treatment for


patients with moderate acne in whom antibiotics are unsuccessful and for
those with severe inflammatory acne.
A 55 year old man, with a history compatible with chronic bronchitis,
presents to your office because of progressive shortness of breath on
exertion. In the history, all of the following would be anticipated, except

a) Increased symptoms on exposure to smog


b) An increased incidence of chronic respiratory disease in family members
c) Recurrent episodes of pleurisy
d) A 20 year history of smoking
e) Increased symptoms with acute respiratory infection
The correct answer is C

Explanation
Bronchitis is inflammation of the upper airways, commonly following a
URI. The cause is usually a viral infection though it is sometimes a
bacterial infection; the pathogen is rarely identified. The most common
symptom is cough with or without fever and/or sputum production. In
patients with COPD, hemoptysis, burning chest pain, and hypoxemia may
also occur. Diagnosis is clinical. Treatment is supportive; antibiotics
are necessary only for patients with chronic lung disease. Prognosis is
excellent in patients without lung disease, but in patients with COPD,
acute respiratory failure may result.

Pleurisy develops when something (usually a virus or bacterium)


irritates the pleura, resulting in inflammation. Certain autoimmune
diseases (such as systemic lupus erythematosus) can irritate the pleura.
Pleurisy may also develop when cancer spreads from the lung or another
part of the body to the pleura, irritating it. Inhalation of asbestos
can also cause pleurisy, as can (rarely) the use of certain drugs, such
as nitrofurantoin or procainamide.
A 42 year old woman presents with her second episode of shingles. Which
one of the following is characteristic of recurrent herpes zoster
infections‟

a) The virus is confined to the initial site of infection


b) The recurrences are preceded by acute clinical disease
c) They elicit a poor antibody response
d) They involve infected carrier individuals
e) They involve uninfected carrier individuals
The correct answer is D

Explanation
Shingles (herpes zoster) is infection that results from reactivation of
the varicella-zoster virus in a carrier. This is the same virus that
causes chickenpox. A carrier is someone who is infected with a disease
and shows no clinical symptoms, but who is capable of infecting other
people with the disease. Shingles causes a painful skin eruption of
fluid-filled blisters and sometimes results in chronic pain in the
affected area.
A 36 year old man comes to the office because he has experienced 6 weeks
of gradually increasing fevers, dry cough and shortness of breath.
Records show he has lost 5.4 kg (12 lb) since his last routine visit 6
months ago. Today, on physical examination, there are several large,
nontender, anterior and posterior cervical lymph nodes. His medical
history includes infectious mononucleosis, hepatitis B, impetigo,
prostatitis and tinea versicolor.

Which of the following factors in his history is most suggestive of


pneumonia related to a sexually transmitted disease?

a) Hepatitis B
b) Impetigo
c) Infectious mononucleosis
d) Prostatitis
e) Tinea versicolor
The correct answer is A

Explanation
This man‟s past medical history includes multiple prior infections,
including mononucleosis, hepatitis B, prostatitis, and tinea versicolor.
Furthermore, he has had a subacute illness with weight loss, fevers, and
a nonproductive cough. The history of hepatitis B and prostatitis in a
young man with these symptoms should always suggest the possibility of
HIV infection transmitted through homosexual activity. Of the listed
infections, hepatitis B is the infection most likely to be transmitted
sexually.
A 43 year old housewife presents with several week history of heartburn,
stomach pain and satiety. You order an EGD and a gastric ulcer is
present. You treat the patient for 6 weeks with omeprazole. She returns
and is asymptomatic now. But a repeat gastroscopy done shows that the
ulcer is still present. Her biopsies are negative for malignancy and H.
Pylori.
What is the next step in management?
a) Add ranitidine
b) Extend current treatment
c) Follow up only
d) Partial gastrectomy with excision of ulcer
e) Add bismuth, metronidazole, and tetracycline
The correct answer is B

Explanation
A gastric or Peptic ulcer is essentially damage to the inner lining of
the stomach, causing a 'crater'. This can be caused by /H. pylori/
infections, excessive acid production in the stomach and long term use
of the NSAID class of drugs which include aspirin, ibuprofen and naproxen.

Symptoms of gastric ulcers that patients will present with are pain,
heartburn, early satiety and sometimes bloating and vomiting. Diagnoses
is made by doing an EGD (esophagogastroduodenoscopy) to visualize the
ulcer. A biopsy can be taken at the time also.

Treatment options include medicines that reduce the amount of acid that
your stomach makes. H2 blockers and proton pump inhibitors (PPI) are
commonly used for this. They usually help people start to feel better
within 3 days.

PPIs taken daily or twice-daily doses for 4 weeks heal 80-100% of


gastric ulcers if H pylori infection is not present or has been eradicated.
All H2 blockers are comparable in efficacy and, when used in twice-daily
doses for a period of 8 weeks, have a healing rate of higher than 70%.

If healing of gastric ulcer is not achieved despite repeat treatment,


maintenance antisecretory therapy should be recommended.

Instruct patients to avoid NSAIDs.


Discourage alcohol consumption and cigarette smoking because these
activities impair gastric mucosal protection.
A 20 year old man develops fatigue, fever and sore throat. He has
swollen lymph nodes on physical exam. What is the most appropriate
treatment?
a) Admission to hospital
b) Biopsy of lymph nodes
c) Supportive
d) Antibiotics
The correct answer is C

Explanation
Infectious mononucleosis is caused by Epstein-Barr virus characterized
by fatigue, fever, pharyngitis and lymphadenopathy. Fatigue may persist
weeks or months. Severe complications, including splenic rupture and
neurologic syndromes, occasionally occur. Diagnosis is clinical or with
heterophil antibody testing. Treatment is supportive.
Which one of the following is the best treatment for restless leg syndrome?

a) Ropinirole (Requip)
b) Continuous positive airway pressure (CPAP)
c) Tricyclic antidepressants
d) SSRIs
e) Cyclobenzaprine (Flexeril)
The correct answer is A

Explanation
Dopaminergic agents are considered first-line therapy for restless legs
syndrome (RLS). Of the dopaminergic agents, ropinirole is the only
FDA-approved treatment for RLS. Levodopa, pergolide, and pramipexole are
used for RLS, but these dopaminergic agents have not been approved for
this use.

Long-term treatment with dopaminergic agents can lead to augmentation,


with symptoms occurring earlier in the evening and spreading to the arms
and trunk.Augmentation is more likely with the use of levodopa. Rebound,
the occurrence of RLS early in the morning, may also be seem. Other
treatments for RLS include benzodiazepines, benzodiazepine agonists,
low-dose opoids, anticonvulsants, and supplemental iron.
A 45 year old woman is brought to the Emergency Department after she
fainted in a supermarket. Her husband tells you she was diagnosed as
"hypoglycemic" by a physician 2 years ago. Which one of the following
criteria is most in favor of the diagnosis of hypoglycemia?

a) Abnormal 72 hour fasting test


b) Low blood glucose while symptomatic
c) Typical prodrome of tachycardia
d) History of post-prandial syncope
The correct answer is A

Explanation
Hypoglycemia unrelated to exogenous insulin therapy is an uncommon
clinical syndrome characterized by low plasma glucose level, symptomatic
sympathetic nervous system stimulation, and CNS dysfunction. Many drugs
and disorders cause it.

A 72 hour fast performed in a controlled setting is the standard for


diagnosis. Patients drink only noncaloric, noncaffeinated beverages, and
plasma glucose is measured at baseline, whenever symptoms occur, and q 4
to 6 h or q 1 to 2 h if glucose falls below 60 mg/dL (3.3 mmol/L). Serum
insulin, C-peptide, and proinsulin should be measured at times of
hypoglycemia to distinguish endogenous from exogenous (factitious)
hypoglycemia.

The fast is terminated at 72 hour if the patient has experienced no


symptoms and glucose remains normal, sooner if glucose decreases to < 45
mg/dL (< 2.5 mmol/L) in the presence of hypoglycemic symptoms.
Which one of the following would be most likely to blunt the
antihypertensive effects of an ACE inhibitor?

a) Allopurinol
b) Ibuprofen
c) Chlorpromazine
d) Triamterene
The correct answer is B

Explanation
Many medications interfere with the absorption, metabolism, and
pharmacologic action of ACE inhibitors. Of the choices given, the one
most likely to do this is ibuprofen, a commonly used NSAID.
You are treating a patient with heart failure and have put her on
digoxin. Which one of the following symptoms indicates there is digoxin
toxicity?

a) Pleural effusions
b) Tachycardia
c) Blue nose
d) Diminished hearing
e) Abdominal distention
The correct answer is D

Explanation
Digoxin is used to treat congestive heart failure and the associated
symptoms of shortness of breath when lying flat, wheezing, and ankle
swelling. Digoxin is also used to slow heart rate in rapid atrial rhythm
disturbances such as atrial fibrillation and atrial flutter.

Common adverse effects (?1% of patients) include: loss of appetite,


nausea, vomiting, diarrhea, blurred vision, visual disturbances
(yellow-green halos), confusion, hearing loss, drowsiness, dizziness,
nightmares, agitation, and/or depression.
It is unlikely to see macrocytosis in a patient with anemia in which of
the following?

a) Reticulocytosis
b) Vitamin B12 deficiency
c) Folate deficiency
d) Sideroblastic anemia
The correct answer is D

Explanation
Sideroblastic anemias are iron-utilization anemias that are generally
part of a myelodysplastic syndrome, producing a normocytic-normochromic
anemia with high RBC distribution width or a microcytic-hypochromic
anemia, particularly with increased serum iron, ferritin, and
transferrin saturation.

Macrocytic anemias include Vitamin B12 and folate deficiency.


Reticulocytosis occurs in hemolytic anemia.
A 75 year old man with a history of emphysema presents to his physician
after he develops the acute onset of fevers, rigors, and a cough
productive of green sputum. His temperature is 38.3 C (100.9 F), blood
pressure is 155/90 mm Hg, pulse is 90/min, and respirations are 34/min.
He weighs 80 kg. An arterial blood gas reveals a pH of 7.20, a pCO2 of
60 mm Hg, and a pO2 of 52 mm Hg. Over the next 2 hours, he becomes
increasingly tachypneic, and his pCO2 rises to 74 mm Hg. The decision is
made to intubate him at that point. Which of the following settings
would be most appropriate for his tidal volume on the respirator?

a) 400 mL/breath
b) 800 mL/breath
c) 1200 mL/breath
d) 1600 mL/breath
The correct answer is B

Explanation
The tidal volume for a patient is generally estimated as 10 mL/kg of
weight, which for this patient would be 800 mL/breath. Giving a lower
tidal volume will yield hypoventilation and be insufficient to eliminate
pCO2. Providing a tidal volume greater than 10 mL/kg increases the risk
of pneumothorax, particularly in a patient with longstanding emphysema
who may have thin-walled alveoli.
A 43 year old man presents 2 weeks after you see him for infectious
diarrhea caused by C. Jejuni. He has now developed unilateral proximal
lower limb weakness and bilateral distal parasthesia and decreased ankle
tendon reflex. What is the most likely diagnosis‟

a) Guillain Barre
b) Multiple Sclerosis
c) Myasthenia Gravis
d) Systemic Lupus Erythematosus
The correct answer is A

Explanation
Guillain-Barré syndrome is the most common acquired inflammatory
neuropathy. It is an acute, usually rapidly progressive inflammatory
polyneuropathy characterized by muscular weakness and mild distal
sensory loss.

In most patients, the syndrome begins 1-3 weeks after an infectious


disorder, surgery, or vaccination. Infection is the trigger in > 50% of
patients, common pathogens include Campylobacter jejuni, enteric
viruses, herpesviruses, and Mycoplasma.

Flaccid weakness predominates in most patients, it is always more


prominent than sensory abnormalities and may be most prominent
proximally. Relatively symmetric weakness with paresthesias usually
begins in the legs and progresses to the arms, but it occasionally
begins in the arms or head.

Diagnosis is clinical. If Guillain-Barré syndrome is suspected, patients


should be admitted to a hospital for electromyography (EMG), CSF
analysis, and measurement of forced vital capacity.

Guillain-Barré syndrome is a medical emergency, requiring constant


monitoring and support of vital functions, typically in an ICU.
Plasmapheresis helps when done early in the syndrome and is the
treatment of choice in acutely ill patients. Immune globulin is also
effective when given early.
In the treatment of benign prostate hyperplasia (BPH), which of the
following is true?

a) 5 alpha reductase inhibitors reduce smooth muscle tone


b) Beta reductase agonists reduce smooth muscle tone
c) 5 beta reductase inhibitors are used for BPH treatment
d) Alpha adrenergic antagonists reduce smooth muscle tone
The correct answer is D

Explanation In BPH, the prostate gland cells increase in number and


eventually narrow the urethra that they surround.

This can lead to urinary problems such as frequency, hesitancy and


retention. A digital rectal exam will show an enlarged prostate. A PSA
level, urinalysis and prostate ultrasound are commonly obtained. Two
main types of drugs are used to treat this.

Alpha adrenergic receptor antagonists (alfuzosin, terazosin) block


receptors in the bladder neck and prostate to relax the smooth muscle,
resulting in improvement in urine flow rate and reduction in symptoms of
BPH.

The androgen DHT causes prostatic enlargement. 5 alpha reductase


inhibitors (finasteride and dutasteride) inhibit conversion of
testosterone to DHT, causing serum DHT levels to decrease. This reduces
prostatic size and improves urinary symptoms.

A 35-year-old female jogger presents to your office with leg pain. She
says it began about a month ago as an aching in the lateral aspect of
the knee at the end of a run. She has continued to exercise, and now the
pain is present most of the time and has moved more proximally to the
lateral aspect of the thigh and lateral hip area. She denies back pain
and does not recall sustaining an injury.

On examination she has tenderness 2 cm above the lateral joint line of


the knee, which is worse with standing with the knee flexed at 30?.
Back, hip, knee, and neurologic examinations are otherwise normal.

The most likely diagnosis is


a) Patellofemoral syndrome
b) Facet joint syndrome
c) Iliotibial band syndrome
d) Piriformis syndrome with referral of pain to the knee
The correct answer is C

Explanation
Iliotibial band syndrome is a common cause of lateral knee and thigh
pain. It is an overuse syndrome that begins insidiously, starting with
pain over the lateral aspect of the knee and then moving more into the
lateral thigh and lateral trochanteric area. The patient is tender along
the iliotibial band from the knee to the hip, and the discomfort is made
worse by standing with the knee flexed to 30?, since this stretches the
iliotibial band over the trochanter. Ober‟s test, lying on the
uninvolved side and adducting the involved leg, causes pain in the
lateral knee and thigh area because it causes the iliotibial band to
tighten.

Patellofemoral syndrome causes discomfort mainly on movement of the


patella. Facet joint syndrome usually causes discomfort in the low back
that radiate into the thigh and knee, but there should be no tenderness
of the lateral thigh or knee area. Piriformis syndrome results from
pressure on the sciatic nerve from inflamed piriformis muscle, causing
pain in the leg in the referral pattern. However, this should not cause
tenderness to the lateral thigh or knee, and Ober‟s test should be normal.

A 70 year old man is seen in your office for assessment one week after
experiencing a brief episode of left arm and left facial weakness. Blood
pressure in your office is 140/80 mmHg. A CT scan showed no evidence of
hemorrhage or space-occupying lesion. An EKG shows normal sinus rhythm.
Which one of the following is the most important investigation to order
at this time?

a) A magnetic resonance imaging (MRI) of the brain


b) An echocardiogram
c) Cerebral angiography
d) Chest x-ray
e) Carotid artery ultrasound
The correct answer is E

Explanation
A transient ischemic attack (TIA) is focal brain ischemia producing
sudden neurologic deficits that last < 1 hour.

Most TIAs are caused by emboli, usually from carotid or vertebral


arteries, although most of the causes of ischemic stroke can also result
in TIAs.

The cause of a TIA is sought as for that of ischemic strokes, including


tests for carotid stenosis via a carotid ultrasound, cardiac sources of
emboli, atrial fibrillation, and hematologic abnormalities and screening
for stroke risk factors. Because risk of subsequent ischemic stroke is
high and immediate, evaluation proceeds rapidly, usually on an inpatient
basis. It is not clear which patients, if any, can be safely discharged
from the emergency department.
A 22-year-old competitive cross-country skier presents with a complaint
of not being able to perform as well as she expects. She has been
training hard, but says she seems to get short of breath more quickly
than she should. She also coughs frequently while exercising. A review
of systems is otherwise negative. Her family history is negative for
cardiac or pulmonary diseases. Her physical examination is completely
normal, and pulmonary function tests obtained before and after
bronchodilator use are normal.

After you discuss your findings with the patient, she acknowledges that
her expectations may be too high, but can think of no other cause for
her problem.

Which one of the following would be the next reasonable step?

a) An echocardiogram to look for cardiomyopathy or valvular dysfunction


b) Counseling regarding competition stress and athlete burnout syndrome
c) A sports medicine consultation to evaluate her training regimen
d) A trial of inhaled albuterol (Proventil) for exercise-induced
bronchospasm
The correct answer is D

Explanation
Exercise-induced bronchoconstriction (EIB) is a very common and
underdiagnosed condition in athletes. It is defined as a 10% lowering of
FEV1 when challenged with exercise. The exercise required to cause
bronchoconstriction is 5?8 minutes at 80% of maximal oxygen consumption.

EIB is much more common in high-ventilation sports, such as track and


cross-country skiing. It is also more common in winter sports, because
of the inspiration of cold, dry air. In some studies the incidence among
cross-country skiers is as high as 50%, and 40% of those who have
positive tests for bronchospasm are unaware of the problem. A physical
examination, as well as pulmonary function tests at rest and before and
after bronchodilators, will be normal unless there is underlying asthma.
Among athletes with EIB, 10% will not have asthma.

Bronchoprovocative testing can be ordered, but if it is not available a


trial with an albuterol inhaler is reasonable. Cardiomyopathy or
valvular dysfunction not found during the physical examination is
possible, but much less likely. Psychological stresses are also a
possible etiology, but should not receive undue attention, especially
when simple questioning is not productive and more likely diagnoses have
not been ruled out. Poor training methods are also possible, but in a
competitive athlete this is not the most likely cause.

Thiazolidinedione antidiabetic agents include rosiglitazone (Avandia)


and pioglitazone (Actos). Their mechanism of action is

a) Decreased insulin resistance


b) Decreased carbohydrate absorption in the intestinal tract
c) Increased insulin release by the pancreas
d) Increased insulin production by the pancreas
e) Increased gluconeogenesis
The correct answer is A
Explanation
Thiazolidinediones decrease insulin resistance by binding to nuclear
peroxisome proliferators-activated receptors. They decrease
gluconeogenesis and have no effect on insulin release or production by
the pancreas, or on intestinal carbohydrate absorption.

In differentiating early Parkinson‟s disease from variants such as


drug-induced parkinsonism, progressive supranuclear palsy, and multiple
system atrophy, which one of the following is the most important clue?

a) A resting tremor
b) Bradykinesia
c) Rigidity
d) Gait difficulty
e) Loss of postural reflexes
The correct answer is A

Explanation
Resting tremor is the most common presentation of Parkinson‟s disease.
It is rare in progressive supranuclear palsy and multiple system
atrophy, and less common in drug-induced parkinsonism.

A 36-year-old male presents with pain over the lumbar paraspinal


muscles. He says the pain began suddenly while he was shoveling snow.
Which one of the following is true regarding this patient‟s injury?

a) Systemic corticosteroids speed recovery


b) Exercises specific to low back injuries speed recovery
c) Opioids have significant advantages for symptom relief when compared
with NSAIDs or acetaminophen
d) Continued activity rather than bed rest helps speed recovery
e) Trigger-point injections are superior to placebo in relieving acute
back pain
The correct answer is D

Explanation
Multiple studies have demonstrated that bed rest is detrimental to
recovery from low back pain. Patients should be encouraged to remain as
active as possible. Exercises designed specifically for the treatment of
low back pain have not been shown to be helpful. Neither opioids nor
trigger-point injections have shown superiority over placebo, NSAIDs, or
acetaminophen in relieving acute back pain. There is no good evidence to
suggest that systemic corticosteroids are effective for low back pain
with or without sciatica.
Which one of the following ankle-brachial index (ABI) ranges is
associated with the lowest rate of leg pain and claudication?

a) 1.00-1.39
b) 0.70-0.99
c) 0.40-0.69
d) <0.40
The correct answer is A

Explanation
The average ankle-brachial index (ABI) in patients with nonperipheral
arterial disease has been reported to be approximately 1.15. One study
compared patients whose ABI was 1.10-1.19 with those whose ABI was
1.20-1.29. The percentage of patients who had no pain was similar for
the two groups (85.3% and 87.4%), and the percentage of those with
classic claudication was identical (0.8%).
All of the following are features of necrotizing enterocolitis, except

a) Pneumatosis intestinalis
b) Peritoneal calcification
c) Portal venous gas
d) Pneumoperitonium
The correct answer is B

Explanation
Necrotizing enterocolitis (NEC) is an acquired disease, primarily of
preterm or sick neonates, characterized by mucosal or even deeper
intestinal necrosis. Signs and symptoms include feeding intolerance,
lethargy, temperature instability, ileus, bloating, bilious emesis,
hematochezia, reducing substances in the stool, apnea, and sometimes
signs of sepsis.

Diagnosis is clinical and is confirmed by imaging studies. Screening the


stools of enterally fed premature infants for occult blood or reducing
substances may help diagnose NEC early. Early x-rays may be nonspecific
and reveal only ileus. However, a fixed, dilated intestinal loop that
does not change on repeated x-rays indicates NEC. X-ray signs diagnostic
of NEC are pneumatosis intestinalis and portal vein gas.
Pneumoperitoneum indicates bowel perforation and an urgent need for surgery.

Treatment is primarily supportive and includes nasogastric suction,


parenteral fluids, TPN, antibiotics, isolation in cases of infection,
and, often, surgery.
A symptom of epilepsy that is most commonly associated with a focus in
the temporal lobes is

a) Tingling and numbness in the limbs


b) Blank, staring spells
c) Vertigo and feelings of giddiness
d) Progressive spread of motor involvement
e) Aura of a peculiar smell or taste
The correct answer is E

Explanation
The main symptom of the temporal lobe epilepsy is brief loss of
consciousness. Temporal lobe epilepsy is a disorder with loss of
judgment, uncontrolled behavior, and abnormal acts. During the seizure
the person may appear drowsy, violent or intoxicated. Normal activities,
such as driving a car, typing, or eating, may go on normally.

The person may hallucinate (see things that are not there), have a sense
of unreality and distorted sense of time. Other symptoms may include
chest pain, shortness of breath, rapid heart beat, and abnormal
sensations of smell and taste. Metallic phantogeusia is a hallucinated
taste of metal and has been seen as part of an aura or seizure in
patients with temporal lobe epilepsy.
A 48 year old man complains of a 6 month history of shortness of breath
and cough. Physical examination reveals clubbing with coarse crackles in
both lung bases and enlarged lymph nodes are palpated. Which one of the
following is the most likely diagnosis‟

a) Asthma
b) Reflux-induced cough
c) Chronic bronchitis
d) Idiopathic pulmonary fibrosis
e) Sarcoidosis
The correct answer is D

Explanation
Idiopathic pulmonary fibrosis (IPF), also known as cryptogenic fibrosing
alveolitis, is a chronic, progressive interstitial lung disease with an
unknown cause. It is one of the two classic interstitial lung diseases,
the other being sarcoidosis.

IPF is slightly more common in males and usually presents in patients


greater than 50 years of age. Average survival from time of diagnosis
varies between 2.5 and 3.5 years, depending on severity, although some
patients live greater than 10 years.

Symptoms are gradual in onset. The most common are dyspnea (difficulty
breathing), but also include nonproductive cough, clubbing (a
disfigurement of the fingers), and crackles (crackling sound in lungs
during inhalation).

Sarcoidosis is an immune system disorder characterized by non-caseating


granulomas (small inflammatory nodules). It most commonly arises in
young adults. The cause of the disease is still unknown. Virtually any
organ can be affected; however, granulomas most often appear in the
lungs or the lymph nodes.

Sarcoidosis is a systemic disease that can affect any organ. Common


symptoms are vague, such as fatigue unchanged by sleep, lack of energy,
weight loss, aches and pains, arthralgia, dry eyes, blurry vision,
shortness of breath, a dry hacking cough or skin lesions such as
erythema nodosum.

A 60 year old man with easy fatigability was found to have


hypercalcemia, decreased parathyroid hormone level, normal blood urea
nitrogen (BUN) and creatinine, and increased urinary excretion of
calcium. A chest X-ray showed an irregular density in the right upper
lobe of the lung. Which of the following is the most likely diagnosis‟

a) Multiple myeloma
b) Sarcoidosis
c) Primary hyperparathyroidism
d) Paraneoplastic syndrome
e) Laboratory error
The correct answer is D

Explanation
Paraneoplastic syndromes occur when a cancer produces one or more
substances that circulate in the bloodstream to cause symptoms at sites
distant from the tumor. These substances can affect the function of
other tissues and organs, resulting in a variety of symptoms.
Paraneoplastic syndromes may affect many different organ systems,
including the nervous system and the endocrine (hormone) system, causing
such problems as low blood sugar, diarrhea, or high blood pressure.

Small cell carcinoma of the lung may secrete a substance that stimulates
the adrenal gland to produce increased hormone levels, which can cause
weakness, weight gain, and high blood pressure (Cushing's syndrome).
Small cell carcinoma of the lung may also produce antidiuretic hormone,
causing water retention, decreased sodium levels, weakness, confusion,
and seizures in some people.

Very high calcium levels in the blood (hypercalcemic syndrome) may occur
in people with solid tumors or leukemias. This can occur when the cancer
secretes a hormone-like substance in the blood that causes release of
calcium from bone. High calcium levels may also result if the cancer
directly invades bone, thereby releasing calcium into the bloodstream.
As a result of the high calcium levels in the blood, the person develops
confusion, which can progress to coma and even death.

Which one of the following would be considered first-line therapy for


mild to moderately severe psoriasis confined to the elbows and knees‟

a) Phototherapy using ultraviolet B light


b) Methotrexate
c) Etretinate (Tegison)
d) Betamethasone dipropionate (Diprolene)
The correct answer is D

Explanation
The majority of psoriasis patients can be managed with topical agents
such as betamethasone dipropionate. Systemic treatment is reserved for
patients with disabling psoriasis that does not respond the topical
treatment. This would include phototherapy, methorexate, and etretinate.
During the morning rounds, a second year resident presents a patient as
"a 58 year old female who suffers from rigidity and tremors. She has
been diagnosed with a neurodegenerative disease involving the
depigmentation of substantia nigra and loss of dopaminergic input to
basal ganglia." What is the disease in question?

a) Epilepsy
b) Parkinson‟s disease
c) Mood disorder
d) Schizophrenia
e) Huntington Disease
The correct answer is B

Explanation
Answer: B ? Parkinson‟s disease involves rigidity and tremors followed
by akinesia. It is a neurodegenerative disease involving the the
depigmentation of substantia nigra and loss of dopaminergic input to
basal ganglia. Lack of dopamine levels results in over excitation of
muscles, manifested in the form of rigidity and tremors.

A. Epilepsy involves loss of consciousness with muscular movements.


C. Mood disorder occurs due to the decreased production of not only
dopamine but also nor-epinephrine and serotonin.
D. Schizophrenia is due to the excess production of dopamine,
nor-epinephrine and serotonin.
E. The most striking neuropathology in Huntington Disease occurs within
the neostriatum, in which gross atrophy of the caudate nucleus and
putamen is accompanied by selective neuronal loss and astrogliosis.
A 72-year-old black male with heart failure sees you for follow-up. He
has shortness of breath with minimal exertion. The patient is adherent
to his medication regimen. His current medications include lisinopril
(Prinivil, Zestril), 40 mg twice daily; carvedilol (Coreg), 25 mg twice
daily; and furosemide (Lasix), 80 mg daily. His blood pressure is 100/60
mm Hg, and his pulse rate is 68 beats/min and regular. Findings include
a few scattered bibasilar rales on examination of the lungs, an S gallop
on examination of the heart, and no edema on examination of the legs. An
EKG reveals a left bundle branch block, and echocardiography reveals an
ejection fraction of 25%, but no other abnormalities.

Which one of the following would be most appropriate at this time?

a) Increase the lisinopril dosage to 80 mg twice daily


b) Increase the carvedilol dosage to 50 mg twice daily
c) Increase the furosemide dosage to 160 mg daily
d) Refer for coronary angiography
e) Refer for cardiac resynchronization therapy
The correct answer is E

Explanation
This patient is already receiving maximal medical therapy. The current
guidelines of the North American Society of Pacing and Electrophysiology
endorse the use of cardiac resynchronization therapy (CRT) in patients
with medically refractory, symptomatic, with a QRS interval of at least
130 msec, a left ventricular end-diastolic diameter of at least 55 mm,
and a left ventricular ejection fraction (LVEF) 30%. These guidelines
were refined by an April 2005 AHA Science Advisory, which stated that
“optimal candidates for CRT have a dilated cardiomyopathy on an ischemic
or nonischemic basis, an LVEF 0.35, a QRS complex 120 msec, and sinus
rhythm, and are NYHA functional class III or IV despite maximal medical
therapy for heart failure.”
A 25 year old male present with severe headache, neck pain, photophobia
and fever. After several tests are negative you decide to do a lumbar
puncture, which shows viral meningitis. The CSF analysis would not show
which of the following?

a) Sugar normal
b) Protein elevated
c) Chloride high
d) Cell count < 300
e) Pressure normal
The correct answer is C

Explanation
The CSF analysis in viral meningitis will be as follows:

Opening pressure: will be normal


Cell count: Will be low, in contrast to bacterial meningitis where the
cell count is very high.
Protein: CSF protein level usually is only slightly elevated, but can
range from being normal to as high as 200 mg/dL.
Glucose: Normal in most cases.
Chloride: level in the CSF will be normal.

The measurement of CSF chloride is most useful in the diagnosis of


tuberculous meningitis where it will be low.
For the past 3 days, a 73 year old woman has been complaining of
dizziness, nausea and vomiting. There is no associated tinnitus. These
symptoms are induced by rapid changes in position. Examination of the
eyes shows nystagmus and arcus senilis. Which one of the following is
the most likely cause of this problem?

a) Brain tumor
b) Transient ischemic attacks
c) Meniere's disease
d) Labyrinthitis
e) Acoustic neuroma
The correct answer is D

Explanation
Labyrinthitis is a balance disorder. It is an inflammatory process
affecting the labyrinths that house the vestibular system (which sense
changes in head position) of the inner ear.

In addition to balance control problems, a labyrinthitis patient may


encounter hearing loss and tinnitus. Labyrinthitis is caused by a virus,
but it can also arise from bacterial infection, head injury, an allergy
or as a reaction to a particular medicine. Both bacterial and viral
labyrinthitis can cause permanent hearing loss, although this is rare.

A prominent and debilitating symptom of labyrinthitis is chronic


dizziness. The vestibular system is a set of sensory inputs consisting
of three semicircular canals, sensing changes in rotational motion, and
the otoliths, sensing changes in linear motion. The brain combines
visual cues with sensory input from the vestibular system to determine
adjustments required to retain balance.
Which one of the following is the most common cause of hyperandrogenism
in women?

a) Acromegaly
b) Carcinoma of the ovary
c) Congenital adrenal hyperplasia
d) Cushing‟s syndrome
e) Polycystic ovary syndrome
The correct answer is E

Explanation
Hyperandrogenism is caused by polycystic ovary syndrome in at least 75%
of cases. This diagnosis is made when there is otherwise unexplained
chronic hyperandrogenism and oligo-ovulation or anovulation. Women with
polycystic ovary syndrome should also be evaluated for glucose
intolerance and the metabolic syndrome.

Congenital adrenal hyperplasia is present in 2% of women with


hyperandrogenism; androgen-secreting tumors are seen in 0.2%, with more
than half being malignant. Cushing‟s syndrome and acromegaly are other
infrequent causes of androgen excess.
All of the following are true regarding contact dermatitis, except

a) May be caused by many plants


b) Treatment with systemic steroids could be useful.
c) Treatment with local steroids is frequently unsuccessful.
d) It is diagnosed by exposure history, examination, and sometimes skin
patch testing.
The correct answer is C

Explanation
Contact dermatitis (CD) is acute inflammation of the skin caused by
irritants or allergens. The primary symptom is pruritus. Skin changes
range from erythema to blistering and ulceration, often on or near the
hands but occurring on any exposed skin surface. Diagnosis is by
exposure history, examination, and sometimes skin patch testing.

Treatment entails antipruritics, topical corticosteroids, and avoidance


of causes.
Topical treatment includes cool compresses (saline or Burow's) and
corticosteroids; patients with mild to moderate cases are given
mid-potency topical corticosteroids (eg, triamcinolone 0.1% ointment or
betamethasone valerate cream 0.1%). Oral corticosteroids (eg, prednisone
60 mg once/day for 7 to 14 days) can be used for severe blistering or
extensive disease.
Patients presenting with erythema multiforme often have a prodromal
history of

a) Egg allergy
b) Recent immunization
c) Herpes simplex infection
d) Thermal trauma
e) Streptococcal infection
The correct answer is C

Explanation
Erythema multiforme is often a sequela of herpes simplex infection.
Prophylaxis with acyclovir should be considered in patients with
recurrent herpes-associated erythema multiforme.
A 56 year old man is admitted to the hospital from the emergency
department because of an acute myocardial infarction. He has no
identified risk factors and no history of heart disease. He had no major
complications and is discharged after 8 days. At discharge, the
diagnostic test that should be scheduled for the 2 week follow-up visit
in order to predict most accurately his risk for another infarction is

a) Electrocardiogram
b) Exercise stress test
c) 24 hour ambulatory EKG monitoring
d) Radionuclide ventriculography
e) Ultrasonography of the heart
The correct answer is B

Explanation
The most important test to determine risk for recurrent infarction would
be an exercise stress test. A submaximal exercise test can even be done
before the patient is discharged from the hospital. An electrocardiogram
and a 24 hour ambulatory EKG would demonstrate any predispositions to
arrhythmia, but would not yield any information regarding risk of ischemia.

Radionuclide ventriculography and ultrasonography of the heart can give


useful information regarding ventricular contractility and any wall
motion abnormalities, but would not yield any information regarding
ischemia risk.
Extrinsic allergic alveolitis may be acquired by breathing air
contaminated by which one of the following substances‟

a) Fine silicaceous dust


b) Degreasing solvent
c) Fungal spores
d) Urea formaldehyde foam insulation
e) Chlorine gas
The correct answer is C

Explanation
Extrinsic allergic alveolitis (hypersensitivity pneumonitis) is a type
of inflammation in and around the tiny air sacs (alveoli) and smallest
airways (bronchioles) of the lung caused by an allergic reaction to
inhaled organic dusts or, less commonly, chemicals.

It can be caused by sensitization to many organic dusts mainly fungal


spores. Many types of dust can cause allergic reactions in the lungs.
Organic dusts that contain microorganisms or proteins and chemicals,
such as isocyanates, may cause hypersensitivity pneumonitis. Farmer's
lung, which results from repeated inhalation of heat-loving
(thermophilic) bacteria in moldy hay, is a well-known example of
hypersensitivity pneumonitis.

Moderate-to-severe ophthalmopathy associated with Graves‟ disease


usually responds to

a) Systemic I^131 therapy


b) Propylthiouracil
c) b-blocker therapy
d) Prednisone
e) Subtotal thyroidectomy
The correct answer is D

Explanation
Ophthalmopathy associated with Graves‟ disease usually runs a benign
course independent of the hyperthyroidism. Moderate-to-severe disease
resistant to time and diuretics usually responds to prednisone, 100-200
mg/day. Radiation to the orbits and surgery may occasionally be needed.
The other treatments listed will treat only the hyperthyroidism and not
the ophthalmopathy.
A 35 year old businessman has come to the Emergency Room complaining of
fever and headache for the last six days. He travels extensively for his
company. In the last year alone, he has been to Bangkok, New Delhi,
Jeddah, and Dar es Salaam He returned from Africa only three weeks ago.
Which of the following diagnosis is most likely?
a) Dengue fever
b) Meningococcal meningitis
c) Tuberculosis
d) Malaria
e) HIV
The correct answer is D

Explanation
Malaria is infection of red blood cells with one of four species of
Plasmodium, a protozoan. Malaria causes fever, chills, sweating, an
enlarged spleen, and anemia (due to the breakdown of infected red blood
cells). Although drugs and insecticides have made malaria rare in the
North America and in most industrialized countries, the disease remains
common and deadly in other areas. Malaria may occur in North American
travelers returning from tropical areas.

After infection occurs, symptoms usually appear within a few weeks to


several months, but they may not occur until years later. The initial
symptoms of all forms of malaria are similar. As the infected red blood
cells rupture and release parasites, a person typically develops a
shaking chill followed by a fever that can exceed 104° F (40° C).
Fatigue and vague discomfort (malaise), headache, body aches, and nausea
are common. The fever typically falls after several hours, and heavy
sweating and extreme fatigue follow. Fevers occur unpredictably at
first, but with time, they may become periodic.
A 59-year-old male reports nausea, vomiting, and progressive fatigue for
the past few months. At his last visit, 6 months ago, his blood pressure
was poorly controlled and hydrochlorothiazide was added to his
Beta-blocker therapy. At this visit he appears moderately dehydrated on
examination. Laboratory testing reveals a serum calcium level of 12.5
mg/dL (N 8.0?10.0), a BUN level of 36 mg/dL (N 6?20), and a creatinine
level of 2.2 mg/dL (N 0.6?1.1). A CBC, albumin level, and electrolyte
levels are normal. His intact parathyroid hormone level is reported a
few days later, and is 60 pg/mL (N 10?65). What is the most likely cause
of his hypercalcemia?

a) Renal failure
b) Hyperparathyroidism
c) Milk alkali syndrome
d) Sarcoidosis
The correct answer is B

Explanation
Many patients have mild hyperparathyroidism that becomes evident only
with an added calcium load. Thiazide diuretics reduce calcium excretion
and can cause overt symptoms in a patient whose hyperparathyroidism
would otherwise have remained asymptomatic. The finding of a normal
parathyroid hormone (PTH) level in a patient with hypercalcemia is
diagnostic for hyperparathyroidism, since PTH should be suppressed in
the presence of elevated calcium. Symptomatic hypercalcemia causes
dehydration because of both intestinal symptoms and diuresis. Reversible
renal insufficiency can result, and can become permanent if it is
long-standing and severe.

Conversely, renal failure usually causes hypocalcemia, but can cause


hypercalcemia resulting from tertiary hyperparathyroidism. This develops
after severe hyperphosphatemia and vitamin D deficiency eventually
produce hypersecretion of PTH. This patient‟s renal insufficiency is not
severe enough to cause tertiary hyperparathyroidism. Milk alkali
syndrome is hypercalcemia resulting from a chronic overdose of calcium
carbonate, and is becoming more common as more patients take calcium and
vitamin D supplements. In milk alkali syndrome, and other causes of
hypercalcemia such as sarcoidosis, the PTH level is appropriately
suppressed.

Which one of the following is associated with Kaposi‟s sarcoma?

a) Herpes simplex virus


b) Human papillomavirus
c) Human herpesvirus 8
d) Cytomegalovirus
e) Coxsackievirus
The correct answer is C

Explanation
Patients with HIV infection are at increased risk for a number of
opportunistic infections, including herpes simplex, human papillomavirus
infection, cytomegalovirus infection, and many others. Human herpesvirus
8 (HHV-8) infection is of particular concern because it is associated
with Kaposi‟s sarcoma. The method of transmission is unknown.
A 35-year-old male with a toothache presents to a local clinic for
uninsured patients. On examination you find a decayed left lower molar
that is tender when tapped lightly, and surrounding gingival
inflammation and tenderness. There is no obvious mandibular swelling,
but he does have a tender submandibular lymph node. The earliest
available dental appointment is in 1 week. He has no allergies to
medications. Which one of the following would be the best antibiotic
treatment for this patient?

a) Doxycycline
b) Trimethoprim/sulfamethoxazole (Bactrim, Septra)
c) Clindamycin (Cleocin)
d) Ciprofloxacin (Cipro)
e) Cephalexin (Keflex)
The correct answer is C

Explanation
This patient most likely has periodontitis of the tooth‟s roots with
cellulitis, complicated by an apical abscess. This infection is caused
by anaerobic oral bacteria. Clindamycin or amoxicillin/clavulanate is
preferred for antibiotic treatment. Doxycycline,
trimethoprim/sulfamethoxazole, ciprofloxacin, and cephalexin have
limited effectiveness against anaerobes and would not be indicated.
A 68-year-old female has an average blood pressure of 150/70 mm Hg
despite appropriate lifestyle modification efforts. Her only other
medical problems are osteoporosis and mild depression. The most
appropriate treatment at this time would be

a) Lisinopril (Prinivil, Zestril)


b) Clonidine (Catapres)
c) Propranolol (Inderal)
d) Amlodipine (Norvasc)
e) Hydrochlorothiazide
The correct answer is E
Explanation
Randomized, placebo-controlled trials have shown that isolated systolic
hypertension in the elderly responds best to diuretics, and to a lesser
extent -blockers. Diuretics are preferred, although long-acting
dihydropyridine calcium channel blockers may also be used. In the case
described, -blockers or clonidine may worsen the depression. Thiazide
diuretics may also improve osteoporosis, and would be the most
cost-effective and useful agent in this instance.
A 32-year-old female sees you for a routine examination and is noted to
have a 1-cm thyroid nodule. Her free thyroxine (FT4) and TSH levels are
normal.

Which one of the following would be the most appropriate next step in
managing this patient?

a) Order a thyroid scan and radioactive iodine uptake level


b) Refer for a fine-needle biopsy
c) Refer for thyroidectomy
d) Prescribe levothyroxine (Synthroid) and reevaluate in 4-6 weeks
e) Prescribe propylthiouracil
The correct answer is B

Explanation
A fine-needle biopsy should be the first step in evaluating euthyroid
patients with a nodule. A thyroid scan and measurement of radioactive
iodine uptake would be indicated in the evaluation of a patient with
thyrotoxicosis to determine the functional status of the nodule and the
gland as a whole. Thyroidectomy is not indicated prior to the
establishment of a diagnosis. Prophylthiouracil is a treatment for
hyperthyroidism and is not indicated in a euthyroid patient. The Task
Force does not include the use of levothyroxine in the evaluation or
treatment of thyroid nodules.
A 50 year old man develops hematuria. He is found to have a mass in the
kidney which has an increased blood supply. What is the most likely cause?

a) Medication side effect


b) Infection
c) Nephrolithiasis
d) Renal cell carcinoma
The correct answer is D

Explanation
Renal cell carcinoma is the most common renal cancer. Symptoms appear
late and include hematuria, flank pain, a palpable mass, and fever.
Diagnosis is by CT or MRI and occasionally by biopsy. Treatment is with
surgery. Infection, stones and drugs are all causes of hematuria, but
are unlikely to show a mass with increased blood supply.

A 63 year old woman develops intermittent dizziness. Examination


discloses abnormal corneal light reflex and mild hearing loss in the
right ear. The most likely diagnosis is

a) Cerebellopontine angle tumour


b) Benign paroxysmal positional vertigo
c) Lateral medullary syndrome
d) Méniére disease
The correct answer is A

Explanation
Cerebellopontine angle tumour is a benign tumour of the 8th cranial
nerve that affects approximately 1 out of 100,000 people. This tumour
arises from the myelin forming Schwann cells that coat the 8th cranial
nerve (acoustic nerve). Clinical presentation usually includes
diminished corneal reflex and a hearing deficit.

Diagnosis can be made with auditory evoked potentials and/or MRI


scanning of the brain. Vertigo and tinnitus may be associated symptoms.
Surgical removal of this tumour involves dissection, cautery and
obliteration with laser. Causes: acoustic neuroma (90%), meningioma
(7%), epidermoid (3%), uncommon: metastasis, trigeminal neuroma,
arachnoid cyst and aneurysm.
An 83 year old male has a long history of COPD. His resting oxygen
saturation is 86% on room air. Treatment includes oral bronchodilators,
inhaled corticosteroids, inhaled Beta-agonists, inhaled cholinergics,
and home oxygen. Which one of his treatments has been shown to prolong
survival in cases such as this‟

a) Oral bronchodilators
b) Inhaled corticosteroids
c) Inhaled Beta-agonists
d) Inhaled cholinergics
e) Home oxygen
The correct answer is E

Explanation
Treatment of hypoxemia is critical in the management of COPD and trials
have shown a reduction in mortality with the use of oxygen for 15 or
more hours daily. Inhaled Beta-adrenergic agonists and cholinergic
agents, either alone or in combination, provide symptomatic relief but
do not prolong survival. Theophylline can be used for symptoms
inadequately relieved by bronchodilators. Inhaled corticosteroids do not
appear to alter the rate of decline in lung function in COPD. However,
some evidence shows that these agents alleviate symptoms and reduce
disease exacerbation. Pulmonary rehabilitation improves quality of life
and reduces hospitalizations.
A 44 year old man sustained a comminuted fracture of his left tibia and
fibula 4 months ago. For the past 3 months he has been in the
rehabilitation unit with his leg fully immobilized. Three hours ago he
suddenly developed chest pain and shortness of breath, and he has just
been brought to the emergency department for further evaluation. On
examination he describes an aching discomfort over the right superior
anterior chest and the right scapula posteriorly. The family history is
strongly positive for heart disease. The presence of a right pleural
friction rub in this patient would suggest which of the following?

a) Pericarditis
b) Pneumonia
c) Pneumothorax
d) Pulmonary embolus with infarction
e) Pulmonary embolus without infarction
The correct answer is D

Explanation
Pain and pleural frictional rub is almost diagnostic of infarction of
the overlying pleura. Pericardial rub due to pericarditis is localized
to the lower left side of the heart; there is no evidence for this
diagnosis here. Pneumonia, which is another complication of immobility,
may present with similar signs and symptoms, but should be accompanied
with fever and leukocytosis. A pneumothorax and a pulmonary embolus
without infarction do not typically present with a pleural friction rub.

A 25 year old female, arrives at her family physician‟s office


complaining of easy bruising over the past month. She is not as
concerned about her ?rash-like? skin bruising as she is about her gums
bleeding with brushing her teeth over the past three days. The only
abnormality detected is a platelet count of 70,000/mm. Which of the
following is *least likely* to be the cause?

a) Folate/B12 deficiency
b) Lymphoma/leukemia
c) Factor VIII deficiency
d) SLE
The correct answer is C

Explanation
A vitamin deficiency, lymphoma or leukemia or lupus are in this patients
differential diagnoses.

Factor VIII deficiency (hemophilia A) has an X-linked genetic


transmission. Because these genes are located on the X chromosome,
hemophilia affects males almost exclusively. Daughters of hemophilic
males are obligate carriers, but sons are normal. Each son of a carrier
has a 50% chance of having hemophilia, and each daughter has a 50%
chance of being a carrier.

Patients with hemophilia bleed into tissues (eg, hemarthroses, muscle


hematomas, retroperitoneal hemorrhage), and the bleeding may be delayed
after trauma. Pain often occurs as bleeding commences, sometimes before
other signs of bleeding develop. Chronic or recurrent hemarthroses can
lead to synovitis and arthropathy. Even a trivial blow to the head can
cause intracranial bleeding. Bleeding into the base of the tongue can
cause life-threatening airway compression.
A 25 year old married white female comes to your office for a routine
examination. She expresses some concern because she has not had a
menstrual period in 6 months. She has been trying to get pregnant for
over 2 years without success. Examination shows an obese woman with a
central pattern to her obesity. She has a mild increase in facial hair,
but the examination is otherwise normal, although the pelvic examination
is difficult to perform because of her obesity. Her blood pressure is
138/90 mm Hg. A CBC, metabolic panel, and thyroid studies are all
normal. Other laboratory findings include the following:

Fasting glucose-------------120 mg/dL (N 70-110)


LDL cholesterol-------------136 mg/dL
HDL cholesterol-------------33 mg/dL
Triglycerides-------------220 mg/dL
FSH-------------3.0 mU/mL (N 2.5-10.2)
LH-------------15.0 mU/mL (N 1.9-12.5)

In addition to weight loss and exercise, which one of the following


would improve her chances for conception?

a) Medroxyprogesterone (Provera), 10 mg/day for 14 days each month


b) Conjugated equine estrogens (Premarin) on days 1 through 10 each month
c) Metformin (Glucophage)
d) Niacin
e) Ginseng
The correct answer is C

Explanation
Polycystic ovary disease is probably the result rather than the cause of
amenorrhea. Polycystic ovaries are thought to be the result of
anovulation, which in turn is likely due to a combination of
hyperinsulinemia and insulin resistance which leads to abnormal
hypothalamic secretion of Gn-RH. This results in a high level of
estrogen, causing anovulation. Patients have higher levels of LH and low
to low-normal levels of FSH. Diagnostic criteria include definite or
probable menstrual dysfunction, clinical evidence of increased
androgens, and the exclusion of congenital adrenal hyperplasia. Findings
which support the diagnosis include perimenarchal onset, insulin
resistance, and elevated LH/FSH ratio, and polycystic ovaries on
ultrasonography. As many as 25% of normal women may have polycystic
ovaries when examined by ultrasonography but ultrasonography alone
should not be the sole criterion for diagnosis. The use of metformin
reduces insulin resistance and may normalize menstruation and restore
ovaluation.
In evaluating an adult with anemia, which one of the following findings
most reliably indicates a diagnosis of iron deficiency anemia?

a) Low total iron-binding capacity


b) Low serum iron
c) Low serum ferritin
d) Microcytosis
e) Hypochromia
The correct answer is C

Explanation
The total iron-binding capacity is elevated, not decreased, in iron
deficiency anemia. As an acute-phase reactant, serum iron may be
decreased in response to inflammation even when total body stores of
iron are not decreased. Microcytosis and hypochromia are both features
of iron deficiency anemia occurring late in its development, but both
can also be seen in the thalassemias. Serum ferritin is also an
acute-phase reactant but is normal or elevated in the face of an
inflammatory process. A low serum ferritin, however, is diagnostic for
iron deficiency even in its early stages.
An elderly man presents with hearing loss. On exam, air conduction is
greater than bone conduction (AC>BC) in the both ears. Weber test
lateralizes to right. What is the most appropriate diagnosis‟
a) Sensorineural deafness of the left ear
b) Conduction deafness of the left ear
c) Sensorineural deafness of the right ear
d) Conduction deafness of the right ear
The correct answer is A

Explanation
In the Weber test a tuning fork is struck and the stem of the fork is
placed on the top of the patient's skull - equal distance from the
patient's ears, in the middle of the forehead. The patient is asked to
report in which ear the sound is heard louder.

A patient with a unilateral sensorineural hearing loss would hear the


sound loudest in the unaffected ear. A patient with a unilateral
conductive hearing loss would hear the tuning fork loudest in the
affected ear.

The Rinne test is performed by placing a vibrating tuning fork initially


on the mastoid process (behind the ear) until sound is no longer heard,
the fork is then immediately placed just outside the ear. Normally, the
sound is audible at the ear.

In a normal ear, air conduction (AC) is better than bone conduction


(BC). AC > BC, and this is called a positive Rinne.
In conductive hearing loss, bone conduction is better than air: BC > AC,
a negative Rinne.

In sensorineural hearing loss, bone conduction and air conduction are


both equally depreciated, maintaining the relative difference of: AC >
BC, a positive Rinne.

This patients presentation is consistent with sensorineural hearing loss


of the left ear.
A 57-year-old male presents with a 1-week history of cough and fever.
His WBC count is 13,000/mm^3 (N 4300?10,800) and a chest radiograph
reveals a left lower lobe infiltrate.
The community has recently had multiple cases of infection with the
BI/NAP1 strain of Clostridium difficile.

Which one of the following would be the most appropriate initial therapy
in this patient?

a) A penicillin
b) A macrolide
c) A quinolone
d) A cephalosporin
e) Metronidazole (Flagyl)
The correct answer is B

Explanation
Penicillins, cephalosporins, and quinolones are highly associated with
Clostridium difficile colitis, but macrolides are not. Metronidazole
would not be an appropriate treatment for pneumonia.
In a patient with a mechanical mitral valve replacement who is taking
warfarin (Coumadin), the recommended INR range is

a) 1.0-2.0
b) 1.5-2.5
c) 2.0-3.0
d) 2.5-3.5
e) 3.0-4.0
The correct answer is D

Explanation
Most conditions (DVT or PE, atrial fibrillation) requiring warfarin
therapy are adequately treated at INR values between 2.0 and 3.0;
patients with mechanical heart valves, especially mitral valves, may
benefit from higher values (up to 3.5). Patients with thrombophilia due
to antiphospholipid antibody syndrome may require a higher INR (3.0?4.0)
for optimal therapy.
A previously alert, otherwise healthy 74 year old black male has a
history of slowly developing progressive memory loss and dementia
associated with urinary incontinence and gait disturbance resembling
ataxia. This presentation is most consistent with

a) Normal pressure hydrocephalus


b) Alzheimer‟s disease
c) Subacute sclerosing panencephalitis
d) Multiple sclerosis
The correct answer is A
Explanation
In normal pressure hydrocephalus a mild impairment of memory typically
develops gradually over weeks or months, accompanied by mental and
physical slowness. The condition progresses insidiously to severe
dementia. Patients also develop an unsteady gait and urinary
incontinence, but there are no signs of increased intracranial pressure.

In Alzheimer‟s disease the brain very gradually atrophies. A disturbance


in memory for recent events is usually the first symptom, along with
some disorientation to time and place; otherwise, there are no symptoms
for some period of time. Subacute sclerosing panencephalitis usually
occurs in children and young adults between the ages of 4 and 20 years
and is characterized by deterioration in behavior and work. The most
characteristic neurologic sign is mild clonus.

Multiple sclerosis is characteristically marked by recurrent attacks of


demyelinization. The clinical picture is pelomorphic, but there are
usually sufficient typical features of incoordination, paresthesias, and
visual complaints. Mental changes may occur in the advanced stages of
the disease. About two-thirds of those affected are between the ages of
20 and 40.

An 81-year-old female is brought to your office by her son. He reports a


decrease in his mother‟s memory and prolonged delays in her responses to
questions. She also has developed urinary incontinence. MRI shows
dilated ventricles, but no other pathology, and a lumbar puncture
reveals a normal opening pressure.

Which one of the following would provide additional evidence of normal


pressure hydrocephalus‟

a) Monocular visual loss


b) Internuclear ophthalmoplegia
c) Headache
d) Motor weakness
e) Apraxia of gait
The correct answer is E

Explanation
Gait instability (sometimes referred to as apraxia of gait), urinary
incontinence, and dementia are typical signs and symptoms of normal
pressure hydrocephalus (NPH). A decrease in the absorption of
cerebrospinal fluid (CSF) results in ventricular enlargement, but no
significant increase in CSF pressure. Although it accounts for less than
5% of dementia cases, it is important to recognize NPH because it may be
effectively treated by shunting in some patients. Monocular visual loss
and headache are symptoms of benign intracranial hypertension
(pseudotumor cerebri), and internuclear ophthalmoplegia is a sign of
multiple sclerosis. Motor weakness is not a sign of NPH.
A 53 year old woman is brought to the ER for epistaxis presenting for 45
minutes. Upon arrival, her blood pressure is 210/120 mm Hg, pulse
90/minute and respiratory rate 24/minute. Examination of the anterior
septum shows no active bleeding. Your management will include all of the
following, except

a) Posterior packing
b) Vasopressin IV
c) Oxygen supplement
d) Anterior packing
e) Keep blood in reserve
The correct answer is B

Explanation
Epistaxis is nose bleeding, which occurs from the anterior or posterior
portion of the nasal septum. Diagnosis is by direct visualization.
Treatment varies by site of bleeding but includes cautery and various
types of packing. In this patient vasopressin should not be given as it
would worsen her already high blood pressure.

A 35 year old epileptic presents to your office complaining of a fuzzy


feeling in the head. Examination shows bidirectional horizontal
nystagmus. The most likely cause of this finding is

a) Poorly controlled epilepsy


b) Vitamin B12 deficiency
c) Alcoholism
d) Phenytoin excess
e) Subdural hematoma
The correct answer is D

Explanation
Phenytoin is an anticonvulsant drug which can be useful in the treatment
of epilepsy. The primary site of action appears to be the motor cortex
where spread of seizure activity is inhibited. Possibly by promoting
sodium efflux from neurons, phenytoin tends to stabilize the threshold
against hyperexcitability caused by excessive stimulation or
environmental changes capable of reducing membrane sodium gradient.
Phenytoin reduces the maximal activity of brain stem centers responsible
for the tonic phase of tonic-clonic (grand mal) seizures.

The most common manifestations encountered with phenytoin excess include


nystagmus, ataxia, slurred speech, decreased coordination and mental
confusion. Dizziness, insomnia, transient nervousness, motor twitchings,
and headaches have also been observed.
The elderly patient may process medications differently than a younger
adult. Which one of the following is LEAST affected by aging?

a) Compliance
b) Absorption
c) Distribution
d) Metabolism
e) Excretion
The correct answer is B

Explanation
Drug absorption is slower in the elderly patient, but it is generally as
complete as in younger adults. Confusion, cognitive impairment, and the
number of prescriptions may interfere with compliance. Distribution is
altered by body weight and composition changes that occur with aging,
and metabolism and excretion may be significantly reduced in elderly
patients.
Clinical features of multi-infarct dementia may include all of the
following, except

a) Parkinsonism
b) Deteriorating course
c) Evidence of cerebrovascular disease
d) Focal neurologic signs
e) Cognitive impairment
The correct answer is A

Explanation
Multi-infarct dementia, also known as vascular dementia, is the second
most common form of dementia after Alzheimer disease (AD) in the elderly
(persons over 65 years of age). The term refers to a group of syndromes
caused by different mechanisms all resulting in vascular lesions in the
brain.

Multi-infarct dementia can have a deteriorating course. Patients can


show evidence of cerebrovascular disease, focal neurologic signs and
cognitive impairment.
For older patients with non-valvular atrial fibrillation or recurrent
systemic embolization, maintaining which one of the following
constitutes optimal anticoagulation?

a) An INR of 1.5-2.0
b) An INR of 2.0-3.0
c) An INR of 3.0-4.0
d) A prothrombin time (PT) of 18-24 sec
e) A PT <18 sec
The correct answer is B

Explanation
The optimal method of monitoring warfarin therapy is to use the
international normalized ratio (INR).
The prothrombin time (PT) is not standardized and may vary depending on
the thromboplastins used, which have different sensitivities.
Studies have indicated that low-intensity warfarin (INR 2-3) is
equivalent in efficacy to higher-intensity anticoagulation (INR >3), and
is associated with a lower incidence of major bleeding events.
Patients with mechanical prosthetic valves should have an INR between
2.5 and 3.5.
A 78 year old man lives alone, is a smoker and has a typical ?tea and
toast? diet. He develops bleeding gums and loose teeth. What is the most
likely reason?

a) Vitamin B6 deficiency
b) Vitamin B12 deficiency
c) Vitamin C deficiency
d) Vitamin D deficiency
e) Folic acid deficiency
The correct answer is C

Explanation
Vitamin C (ascorbic acid) is essential for the formation, growth, and
repair of bone, skin, and connective tissue (which binds other tissues
and organs together and includes tendons, ligaments, and blood vessels).
Vitamin C helps maintain healthy teeth and gums. It helps the body
absorb iron, which is needed to make red blood cells. Vitamin C also
helps burns and wounds heal.

In adults, vitamin C deficiency usually results from a diet low in


vitamin C. For example, vitamin C deficiency may result from a diet
deficient in fresh fruits and vegetables. Also, cooking can destroy some
of the vitamin C in food. Pregnancy, breastfeeding, disorders that cause
a high fever or inflammation, surgery, and burns can significantly
increase the body's requirements for vitamin C and the risk of vitamin C
deficiency. Smoking increases the vitamin C requirement by 30%.
Incidence also peaks in elderly populations, who sometimes have
"tea-and-toast" diets deficient in vitamin C.

A 65 year old man comes to you complaining of mild tremor while at rest.
His posture is normal. He notices slight slowness of movement. Which
therapy is the most appropriate in this stage of his Parkinsonism?

a) Carbidopa/Levodopa (Sinemet)
b) Bromocriptine
c) Benztropine
d) Amantidine
The correct answer is D

Explanation
Five stages of Parkinson exist. This patient is in stage 1.

Stage 1: Patient usually experiences mild symptoms. These symptoms may


inconvenience daily tasks the patient would otherwise complete with
ease. Typically these symptoms will include the presence of tremors.

Stage 2: Symptoms are bilateral, affecting both limbs and both sides of
the body. The patient usually encounters problems walking or maintaining
balance, and the inability to complete normal physical tasks becomes
more apparent.

Stage 3: Symptoms can be rather severe and include the inability to walk
straight or to stand. There is a noticeable slowing of physical movements.

Stage 4: Walking may still occur, but it is often limited and rigidity
and bradykinesia are often visible. During this stage, most patients are
unable to complete day-to-day tasks, and usually cannot live on their own.

Stage 5: This stage takes over the patients physical movements. The
patient is usually unable to take care of himself or herself and may not
be able to stand or walk during this stage, usually requiring constant
one-on-one nursing care.

Levodopa and carbidopa have been considered the gold standard drug
therapy for Parkinson's disease. Levodopa is started at the onset of
moderate symptoms.

Amantadine is an antiviral agent that causes increased dopamine in the


brain. It is prescribed to provide relief of mild, early-stage
Parkinson's disease. Amantadine may also be added to carbidopa-levodopa
therapy for people in the latter stages of Parkinson's disease,
especially if they have problems with involuntary movements. Amantadine
by itself has very few side effects. At the onset of mild symptoms
Amantadine is used.
Which one of the following palliative measures is appropriate for a
patient dying of end-stage Alzheimer‟s disease who is experiencing the
?death rattle? in the final hours of life?

a) Subcutaneous administration of fluids


b) Rectal administration of fluids
c) Glycopyrrolate (Robinul)
d) Bethanechol (Urecholine)
e) Rivastigmine (Exelon)
The correct answer is C

Explanation
Anticholinergic medications, such as glycopyrrolate, may be used
effectively to dry the secretions that cause the ?death rattle? in
terminally ill patients. Bethanechol and rivastigmine are both
cholinergic, which has the opposite effect. Administration of fluids by
hypodermoclysis or proctoclysis would increase respiratory secretions,
thereby increasing the symptoms.
A 45 year old man with a history of polycystic kidney disease presents
with painless gross hematuria. You order

a) No investigation is required since the hematuria is most likely due


to the rupture of renal cyst(s)
b) No investigation at this time. Investigate if gross hematuria persists
c) U/S
d) U/S, urine C&S, cystoscopy
The correct answer is D

Explanation
Polycystic kidney disease is a hereditary disorder of renal cyst
formation causing gradual enlargement of both kidneys, sometimes with
progression to renal failure. Almost all forms are caused by a familial
genetic mutation. Signs are nonspecific and include hematuria (64%),
hypertension (50%), and proteinuria (20%). Urinalysis detects mild
proteinuria and microscopic or macroscopic hematuria. Ultrasonography or
CT is the imaging test of choice, showing extensive cystic changes
throughout the kidneys and a moth-eaten appearance due to cysts that
displace functional tissue. Cystoscopy is necessary to look into the
bladder to check for a possible cause for the hematuria.
A 20 year old man has acute fever, malaise and pain in and just below
the left knee. There has not been any injury. Which one of the following
physical findings is most consistent with acute osteomyelitis‟

a) An effusion into the knee joint


b) Marked localized tenderness over the tibial metaphysis
c) Inability to flex the knee because of pain
d) Swelling of the prepatellar bursa
e) Relief of pain by rest and elevation of the leg
The correct answer is B

Explanation
Osteomyelitis is inflammation and destruction of bone caused by
bacteria, mycobacteria, or fungi. Common symptoms are localized bone
pain and tenderness with constitutional symptoms (in acute
osteomyelitis) or without constitutional symptoms (in chronic
osteomyelitis).

Patients with acute osteomyelitis of peripheral bones usually experience


weight loss, fatigue, fever, and localized warmth, swelling, erythema,
and tenderness. Diagnosis is by radiography and cultures. Treatment is
with antibiotics and sometimes surgery.
A 35 year old female volleyball coach consults you about a pruritic,
vesicular eruption on the medial aspect of both feet, which she says has
spread to her hands, causing a vesicular eruption resembling
dyshidrosis. She is not using any drugs internally or topically. The
most likely diagnosis is

a) Psoriasis
b) Erythema multiforme
c) Tinea pedis
d) Neurodermatitis
e) Contact dermatitis
The correct answer is C

Explanation
This presentation is typical of tinea pedis, with the associated
dermatophytid reaction. Treatment of the fungal illness will usually
control both conditions.
Which one of the following is true regarding osteoporosis‟

a) One-third of hip fractures in persons older than 65 occur in men

b) All women should have a bone density measurement at the time of


menopause
c) Multiparity increases the risk for osteoporosis
d) Controlled trials have shown that bone density screening reduces
rates of osteoporotic fracture
The correct answer is A

Explanation
Osteoporosis is becoming an increasingly important health concern in
men. While women are more susceptible to osteoporosis than men,
approximately one-third of hip fractures in persons older than age 65
occur in men. Recommendations for bone density screening in
postmenopausal women include testing women below age 65 who have one or
more risk factors for osteoporosis other than menopause. Women age 65
and older should have bone density measurement regardless of other risk
factors. Risk factors for osteoporosis include petite frame, white
ancestry, sedentary life style, nulliparity, smoking, high caffeine
intake, alcohol use, postmenopausal status, and low calcium intake.
Controlled trials have yet to prove that women who are screening for
osteoporosis with bone densitometry have better outcomes than women who
are not screened. In fact, having multiple risk factors for osteoporosis
is a stronger predictor of hip fracture than low measured density.
Which one of the following is most effective in limiting the
complications of COPD?

a) Pneumococcal vaccination
b) Smoking cessation
c) Oral corticosteroids
d) Albuterol
e) Theophylline
The correct answer is B

Explanation
Smoking cessation is the only intervention that has been shown to slow
the progression of COPD and limit complications. Albuterol and
theophylline will improve acute problems, but will not slow disease
advancement. Corticosteroids are not indicated for chronic management.
Although pneumococcal vaccine is administered frequently, there is very
little evidence to support a direct benefit in preventing complications
of COPD.

Routine monitoring of patients taking amiodarone (Cordarone) should


include which one of the following?

a) Thyroid function tests


b) Amiodarone levels
c) Chest radiographs
d) Stools for occult blood
e) Ophthalmologic examinations
The correct answer is A

Explanation
Liver and thyroid function testing should be performed at least every 6
months in patients taking amiodarone. The most serious potential adverse
effect of amiodarone therapy is pulmonary toxicity, which may result
from direct drug-induced phospholipidosis or immune-mediated
hypersensitivity. The most common clinical presentation is subacute
cough and progressive dyspnea, with associated patchy interstitial
infiltrates on chest radiographs and reduced diffusing capacity on
pulmonary function testing. A much less common presentation is
adult respiratory distress syndrome (ARDS). Routine screening for ARDS
is of limited value, because pulmonary toxicity can develop rapidly with
no antecedent abnormalities on chest radiographs or pulmonary function
tests.

Thyroid toxicity is the most common complication that requires


intervention. Thyroid abnormalities have been described in up to 10% of
patients receiving long-term amiodarone therapy. Hyperthyroidism may
result from an excess of iodine or acute thyroiditis. Hypothyroidism is
2-4 times more common than hyperthyroidism. Corneal microdeposits are
visible on slit-lamp examination in nearly all patients treated with
amiodarone. These deposits seldom affect vision and rarely necessitate
discontinuation of the drug. Routine monitoring of the plasma amiodarone
level is not recommended.

A 58 year old man with a past history of a parathyroidectomy for primary


hyperparathyroidism is now in your office complaining of headaches worse
in the AM, made worse by a small MVA he credits to a loss of peripheral
vision. You plan to

a) Send to the Emergency Department for an immediate CT head


b) Check his calcium to ensure there‟s no remaining parathyroid tissue
c) Check for a pheochromocytoma because you are concerned he has MEN I
syndrome
d) Check for a homonymous hemianopia because you are worried about a
pituitary tumor
e) Check for a bitemporal hemianopia because you are worried about a
pituitary tumor
The correct answer is E

Explanation
A pituitary tumour can cause bitemporal hemianopsia. This is a condition
where one cannot see out of the periphery. Damage to the medial aspect
of the optic chiasm, as is often seen with a pituitary gland tumor, may
compromise the decussating fibers from both nasal hemiretinas. The loss
of peripheral vision in both eyes is called bitemporal hemianopia.

Keep in mind MEN (Multiple endocrine neoplasia). In particular Multiple


endocrine neoplasia, type I (MEN-I) is a hereditary syndrome
characterized by tumors of the parathyroid glands, pancreatic islet
cells, and pituitary gland.

A man comes to see you. His past medical history includes hypertension.
He tells you that he has experienced impotence for the past 6 months
with his wife. But he is able to develop an erection in the morning and
while reading an adult magazine in the washroom. What should you do?

a) Change his blood pressure medicine


b) Prescribe and ED medicine like Cialis or Levitra
c) Reassurance
d) Check a testosterone level
e) Discuss couples counseling
The correct answer is E

Explanation
Erectile dysfunction (ED) is a common problem among males. It can be
caused by medicine side effects. Beta blockers used to treat
hypertension are common causes of male ED. However this patients history
suggests he has non-organic impotence. Since he is able to achieve an
erection. At this time is appropriate to discuss counseling for him and
his wife to help treat this partner-specific ED.

A 75-year-old male is brought to your office 1 month after a stroke that


involved the left anterior cerebral artery, manifested by leg weakness,
initial incontinence, and slowness in mentation. He experienced seizure
activity on the second day after his stroke, but this was controlled by
phenytoin (Dilantin). He has improved significantly and is now
ambulatory. His family states that he now has episodic confusion,
sleepiness, and clumsiness, which is preceded by paresthesias and
dizziness, although no tonic-clonic activity has been noted. He remains
very drowsy for several hours after these episodes. He was wearing a
cardiac monitor during one episode, but it showed nothing remarkable.
His phenytoin level is therapeutic, and a CBC, metabolic profile, and
magnesium level are all normal.
Which one of the following would be the most appropriate next step?

a) Discontinue the phenytoin


b) Add phenobarbital to the phenytoin
c) Begin bupropion (Wellbutrin)
d) Begin modafinil (Provigil)
e) Begin lamotrigine (Lamictal)
The correct answer is E

Explanation
Up to 50% of cases of epilepsy in the geriatric population result from
cerebrovascular disease. Risk factors for post-stroke epilepsy include
cortical involvement, hemorrhage, and large size. Approximately 35% of
those who experience an acute stroke-related seizure develop post-stroke
epilepsy, compared to only 9% of those who do not have acute seizure
activity.

The most common seizures in the elderly are complex partial seizures,
but they do not have the typical presentation seen in younger people
(aura, déjà vu, olfactory hallucinations). Geriatric patients are more
likely to have nonspecific preceding symptoms, such as vaguely localized
paresthesias, dizziness, and muscle cramps. Those present may note
episodic confusion, drowsiness, or clumsiness more than tonic-clonic
movements. The postictal state is likely to be prolonged in the elderly.
Misdiagnosis of these seizures is very common, with the diagnosis often
delayed as much as 2 years from the time of the stroke. The most
valuable diagnostic tool is a reliable history from those who witness
the event.

Treating seizures in the elderly by using anti-epileptic drugs (AEDs) is


complicated by a number of factors. Pharmacokinetics are influenced by
decreases in hepatic metabolism, renal elimination, plasma proteins, and
protein binding. Many elderly patients are on a multiple-medication
regimen that increases the risk of drug interactions, and many AEDs are
enzyme inducing, which increases the risk of osteoporosis. The elderly
also are more sensitive to side effects.

The choice of AEDs should be individualized, although many experts


suggest early use of newer AEDs such as topiramate, gabapentin,
lamotrigine, or levetiracetam because of fewer side effects and better
pharmacokinetics. Monotherapy is preferred, if possible, and older drugs
are less expensive, but limited by side effects.

In the case described here, the patient is having breakthrough seizures


despite therapeutic levels of phenytoin. Switching to lamotrigine would
be preferable and is less likely to cause side effects. When
transitioning from one agent to another it would be best to gradually
decrease the phenytoin rather than abruptly discontinuing it. The
addition of phenobarbital would further increase drowsiness. Bupropion
and modafinil may actually increase seizures.
A 53 year old presents to your office with pain and stiffness in both
hands and knees of 6 months duration. You suspect rheumatoid arthritis.
Which of the following findings on your history and physical examination
is most distinctive with a diagnosis‟

a) Unilateral joint tenderness/effusions


b) Maculopapular rash
c) Iridocyclitis
d) Hepatosplenomegaly
e) Joint stiffness worse in the morning
The correct answer is E

Explanation
Rheumatoid arthritis usually affects the joints on both sides of the
body. So, if the right hip is affected, so will the left. Some people
will have only one attack of rheumatoid arthritis, but most will have
one, then a period where they are arthritis-free, and then another
attack. These flare-ups usually continue until the time between
flare-ups is briefer and briefer, and the symptoms are worse each time.
Some flare-ups last only a few days; some last for weeks. The joints
affected by rheumatoid arthritis have a membrane, called the synovial
membrane, that produces synovial fluid to keep the joints lubricated and
to help the bones move smoothly around the joint. When the synovial
membrane becomes stiff and thick, it becomes difficult to move the joint.

Joint pain and stiffness is usually worse first thing in the morning.
Other symptoms include redness and warmth around the affected joint, low
fever, appetite and weight loss, and fatigue. Eventually, the joint
becomes deformed, the cartilage is destroyed, and the unprotected bone
begins to wear away. The joints can become 'stuck' in one position,
called contractures. Rheumatoid arthritis can cause other problems in
the body, such as vasculitis (inflammation of the blood vessels),
osteoporosis (loss of bone density), lung disease, rheumatoid nodules
(small bumps) under the skin, and blood disorders.
Which one of the following causes rhinitis medicamentosa with prolonged
use in the treatment of rhinitis‟

a) Intranasal antihistamines
b) Intranasal decongestants
c) Intranasal anticholinergics
d) Intranasal mast cell stabilizers
e) Leukotriene antagonists
The correct answer is B

Explanation
Intranasal decongestants such as phenylephrine should not be used for
more than 3 days, as they cause rebound congestion on drug withdrawal.
When used for several months or more, these agents can cause a form of
rhinitis, rhinitis medicamentosa, that can be extremely difficult to treat.

A 72-year-old Asian-Canadian female is brought to your office by her


husband because he thinks she might have Alzheimer‟s disease. For the
past 3 months she has complained of confusion, poor appetite, and lack
of energy. She has been unable to do routine housework. On brief
questioning, her short-term recall seems to be impaired, but a more
detailed examination indicates that her memory is fine.

Which one of the following is the most likely diagnosis‟

a) Alzheimer‟s disease
b) Lewy body dementia
c) Frontotemporal dementia
d) Pseudodementia
e) Mild cognitive impairment
Correct Answer:* d)
Explanation
Instead of dementia, this patient has signs of pseudodementia or
depression, which usually has an abrupt onset. Memory usually is intact
when adequate time is taken to carefully evaluate the patient. The onset
of Alzheimer‟s disease, however, is gradual and includes memory loss.
Lewy body dementia is associated with hallucinations, and the onset is
gradual. Frontotemporal dementia generally occurs before age 60. The
onset of mild cognitive impairment is gradual and includes memory loss.

Which one of the following is a side effect of sibutramine (Meridia)?

a) Hyponatremia
b) Decreased calcium absorption
c) Elevated basal temperature
d) Elevated TSH
e) Elevated blood pressure
The correct answer is E

Explanation
Side effects of sibutramine include increases in blood pressure and
pulse; although these increases are usually mild, they lead to
discontinuation of the medication in up to 5% of patients. Adverse
reactions also include dry mouth, headache, insomnia, and constipation.
A 55 year old man complains of extremely severe, sharp, shooting pain in
his face. He describes the episodes as being "like a bolt of
electricity" that are brought about by touching a specific area, last
about 60 seconds, and occur many times during the day. Neurologic
examination is completely normal, but it is noted that part of his face
is unshaven because he fears to touch that area. Gadolinium-enhanced MRI
shows no abnormalities of the trigeminal nerve. Which of the following
is the most appropriate initial treatment?

a) Anticonvulsants
b) Aspirin
c) Nonsteroidal anti-inflammatory drugs
d) Vasoconstrictors
e) Vasodilators
The correct answer is A

Explanation
The clinical description is that of trigeminal neuralgia ("tic
douloureux"), which is treated with anticonvulsants. Carbamazepine is
usually the first choice, but phenytoin has also been used.
Antidepressants, such as amitriptyline, have also been tried. Surgical
decompression of the nerve or stereotactic ablation are used in
recalcitrant cases.
A 57-year-old male with severe renal disease presents with acute
coronary syndrome. Which one of the following would most likely require
a significant dosage adjustment from the standard protocol?

a) Enoxaparin (Lovenox)
b) Metoprolol (Lopressor)
c) Carvedilol (Coreg)
d) Clopidogrel (Plavix)
e) Tenecteplase (TNKase)
The correct answer is A

Explanation
Enoxaparin is eliminated mostly by the kidneys. When it is used in
patients with severe renal impairment the dosage must be significantly
reduced. For some indications the dose normally given every 12 hours is
given only every 24 hours.

Although some -blockers require a dosage adjustment, metoprolol and


carvedilol are metabolized by the liver and do not require dosage
adjustment in patients with renal failure. Clopidogrel is currently
recommended at the standard dosage for patients with renal failure and
acute coronary syndrome. Thrombolytics are given at the standard dosage
in renal failure, although hemorrhagic complications are increased.

Which one of the following is the best initial screening test for
hemochromatosis‟

a) Total iron binding capacity


b) Hemoglobin electrophoresis
c) Serum iron concentration
d) Serum ferritin concentration
e) Serum transferrin saturation
The correct answer is E

Explanation
The diagnosis of hereditary hemochromatosis is based on a combination of
clinical, laboratory, and pathologic criteria, including elevated serum
transferrin saturation and elevated serum ferritin concentration.
Elevated serum transferrin saturation is the earliest phenotypic
abnormality. While this is the best initial screening test, results may
be normal early in the course of the disease. In addition, because serum
iron concentrations vary throughout the day and measurements may be
affected by the ingestion of food, a test showing elevated serum
transferrin saturation should be repeated as a fasting early-morning
determination. Furthermore, the serum ferritin concentration and serum
transferrin saturation may be elevated in 30%?50% of patients with acute
or chronic viral hepatitis or alcoholic liver disease.

Serum ferritin concentration is a sensitive measure of iron overload,


but it is also an acute-phase reactant and is therefore elevated in a
variety of infectious and inflammatory conditions in the absence of iron
overload. Consequently, it should not be used as the initial screening
test to detect hereditary hemochromatosis.
Influenza A has been isolated in your community. Yesterday, a nurse in
the medical-surgical unit developed significant symptoms of influenza.
She did not receive the annual influenza immunization, and she asks for
your advice.

Which one of the following would be most appropriate at this time?

a) Amantadine (Symmetrel)
b) Atorvastatin (Lipitor)
c) Rimantadine (Flumadine)
d) Oseltamivir (Tamiflu)
e) Acyclovir (Zovirax)
The correct answer is D
Explanation
This health care worker should be treated with oseltamivir because it is
effective for those who have been symptomatic for less than 48 hours.
Amantadines have activity against influenza A but not influenza B, and
recent studies show that 91% of isolates tested between 2004 and January
2006 are resistant to this class of drugs. Interestingly, the idea has
been raised to use statin drugs to calm the immune system‟s cytokine
storm, which results in respiratory failure from the influenza
infection. Acyclovir is not indicated for this patient.
A 78 year old male comes to see you after attending a health fair. He is
concerned because he had a prostate-specific antigen (PSA) level of 5.0
ng/mL (N 0.0-4.0). He has never had his PSA checked before.

His medical history is significant for class IV heart failure treated


with furosemide (Lasix), enalapril (Vasotec), carvedilol (Coreg),
digoxin, and spironolactone (Aldactone). His review of systems is
positive for longstanding nocturia and gradually worsening weakness of
the urinary stream. His physical examination is noteworthy for bibasilar
rales, an S 3 gallop, and moderate lower extremity edema. His prostate
is diffusely large and smooth. His urinalysis is unremarkable. Which of
the following is the most appropriate management for his elevated PSA?

a) No intervention
b) Repeat testing after a course of antibiotics
c) Referral for a CT scan or MRI of the pelvis
d) Referral for prostate ultrasonography and biopsy
The correct answer is A

Explanation
The patient described has a life expectancy that makes the risk-benefit
ratio for the detection of asymptomatic prostate cancer extremely
unfavorable. In addition, a mildly elevated PSA in a 78-year-old with a
large prostrate is most likely due to benign prostatic hypertrophy.

A 72 year old white male diabetic presents with a 2-day history of


dyspnea and decreased urine output. Five days ago coronary angiopathy
revealed the absence of coronary artery obstruction and an ejection
fraction of 40%. There has been no chest discomfort since that
procedure. He underwent abdominoperineal resection for anal cancer 3
years ago. On examination his blood pressure is 180/92 mm Hg. Chest
auscultation reveals basilar crackles. His abdomen is round and nontender.

Laboratory Findings
Plasma creatinine----------------------3.3 mg/dL (N 0.6-1.5)
BUN----------------------42 mg/dL (N 8-25)
Glucose----------------------184 mg/dL
Potassium----------------------5.6 mEq/L (N 3.5-5.0)
Sodium----------------------136 mEq/L (N 135-145)
EKG----------------------sinus rhythm without change

All current abnormal laboratory values were normal 7 days ago. Which one
of the following diagnostic tests would carry the greatest risk for this
patient?

a) Intravenous pyelography (IVP)


b) Catheterized urine for residual volume
c) A spiral CT scan
d) Ultrasonography of the kidneys and bladder
e) Fractional excretion of sodium
The correct answer is A

Explanation
IVP would be a high-risk procedure in this patient because of his
elevated creatinine, diabetes, and recent contrast exposure. Contrast
exposure for coronary arteriography is one possible cause of his renal
failure.

Sonography or spiral CT could be done without contrast to rule out an


obstruction or stone. Bladder catheterization carries a risk of
infection, but would be useful in ruling out bladder outlet obstruction
in an elderly male. It might also prove therapeutic. Fractional
excretion of sodium is a useful test for differentiating prerenal
azotemia from acute tubular necrosis, and should be done prior to
diuretic administration.
A 45 year old white male presents with severe intermittent right flank
pain that radiates into his right groin area. You suspect a ureteral
stone. Which one of the following would most reliably confirm your
suspected diagnosis‟

a) A helical CT scan of the abdomen and pelvis without contrast


b) Intravenous pyelography
c) Abdominal ultrasonography
d) A KUB plain film of the abdomen
e) A urinalysis
The correct answer is A

Explanation
An unenhanced helical CT scan of the abdomen and pelvis is the best
study for confirming the diagnosis of a urinary tract stone in a patient
with acute flank pain, supplanting the former gold standard, intravenous
pyelography. A CT scan may also reveal other pathology, such as
appendicitis, diverticulitis, or abdominal aortic aneurysm. Although
abdominal ultrasonography has a very high specificity, it is still not
better than CT, and its sensitivity is much lower; thus, its use is
usually confined to pregnant patients with a suspected stone. Plain
abdominal radiographs may show the stone if it is radiopaque, and are
useful for following patients with radiopaque stones. CT will reveal a
radiopaque stone. While most patients with stones will have hematuria,
its absence does not rule out a stone.
Which one of the following should be withheld before the administration
of intravenous contrast?

a) Glyburide (Micronase, DiaBeta)


b) Metformin (Glucophage)
c) Rosiglitazone (Avandia)
d) Insulin
The correct answer is B

Explanation
Metformin, an oral hyperglycaemic agent used in the treatment of type 2
diabetes, is eliminated primarily by the kidneys. Because of potential
exacerbation of acute renal failure and lactic acidosis, metformin
should be used with caution in patients undergoing radiographic
procedures that require the parenteral administration of iodinated
contrast media. Glyburide, rosiglitazone, and insulin do not have to be
adjusted in the face of radiologic procedures.
A 41 year old man presents to the clinic complaining of a chronic cough
over the past 4 months, which has now been accompanied by hemoptysis.
His lungs have diffuse bilateral rales. Laboratory findings reveal a
sodium of 142 mEq/L, a potassium of 4.3 mEq/L, a chloride of 110 mEq/L,
a bicarbonate of 24 mEq/L, a BUN of 39 mg/dL, and a creatinine of 2.9
mg/dL. Urinalysis reveals microscopic hematuria and 4+ proteinuria.
Which of the following serologic blood tests would most help confirm the
suspected diagnosis‟

a) Anti-glomerular basement membrane antibodies


b) Anti-mitochondrial antibodies
c) Anti-neutrophilic antibodies
d) Anti-parietal cell antibodies
e) Anti-smooth muscle antibodies
The correct answer is A

Explanation
The combination of hematuria and hemoptysis should always raise the
possibility of Goodpasture syndrome. Anti-glomerular basement membrane
antibodies are pathognomonic for this diagnosis.

Anti-mitochondrial antibodies are found in patients with primary biliary


cirrhosis. The anti-neutrophilic cytoplasmic antibodies are found in
patients with Wegener granulomatosis. Wegener granulomatosis may also
present with pulmonary and renal involvement but will have associated
upper respiratory tract findings, e.g, sinusitis and sinus abscesses.

Anti-parietal cell antibodies are found in patients with the autoimmune


disease known as pernicious anemia. Anti-smooth muscle antibodies are
found in patients with autoimmune hepatitis.
Red eye due to acute glaucoma is often associated with all of the
following, except

a) Shallow anterior chamber


b) Fixed pupil
c) Purulent discharge
d) Pain in the eye
e) Decreased vision
The correct answer is C

Explanation
Open-angle glaucoma is painless and causes no early symptoms. The most
important symptom is the development of blind spots, or areas of vision
loss, over months to years. The blind spots slowly grow larger and
merge. Vision loss occurs so gradually that it is often not noticed
until much of it is lost. Side (peripheral) vision is usually lost
first; central vision is lost last. The changes may progress to the
point where only a small central island of vision remains, in which the
person can see straight ahead perfectly but becomes blind in all other
directions (sometimes referred to as tunnel vision). If glaucoma is left
untreated, eventually even central vision can be lost, resulting in
total blindness.
Closed-angle glaucoma causes abrupt onset of severe pain in and around
the eye, headache, redness, blurred vision, rainbow-colored halos around
lights, and sudden vision loss. Nausea and vomiting may occur in
response to the increased eye pressure. Closed-angle glaucoma is
considered a medical emergency, because if left untreated, blindness can
occur as quickly as 2 to 3 hours after the start of symptoms.
A 71 year old man who has been a patient for several years calls the
office to report an episode of bloody urine. He is instructed to come to
the office, where urinalysis confirms gross hematuria without
proteinuria or casts. The patient denies any pain and is anxious for an
explanation. Physical examination is normal. The most appropriate next
step is to

a) Do a transrectal prostatic biopsy


b) Prescribe a 1 month course of trimethoprim-sulfamethoxazole
c) Schedule bilateral renal angiography
d) Schedule cystoscopy
e) Schedule infusion of the renal pelvis with silver nitrate
The correct answer is D

Explanation
Painless hematuria is an important symptom of bladder cancer. When a
urinalysis reveals no active sediment indicative of renal damage, the
nest most important diagnosis to exclude is bladder cancer. For this
reason, a direct visualization of the bladder via cystoscopy is
appropriate.

A transurethral prostate biopsy is used when prostate enlargement is


suspected on the basis of physical examination (enlarged gland via
digital exam) or symptoms (hesitancy, post-void dribbling, frequent
urinations). The biopsy helps to distinguish between benign hypertrophy
and cancer.
The most common cause of fainting is

a) Cardiac dysrhythmia
b) Medications
c) Orthostatic hypotension
d) Psychiatric disorders
e) Vasovagal syncope
The correct answer is E

Explanation
Neurally mediated syncope (also termed neurocardiogenic or vasovagal
syncope) comprises the largest group of disorders causing syncope. These
disorders result from reflex-mediated changes in vascular tone or heart
rate.
A man complains that recently when he shaves he has a shooting type of
pain in his face. It happens once in a while and then goes away. You
suspect trigeminal neuralgia. What is the treatment of choice?

a) Fluoxetine
b) Prednisone
c) Acyclovir
d) Carbamazepine
The correct answer is D
Explanation
Trigeminal neuralgia is severe paroxysmal facial pain due to a disorder
of the 5th cranial nerve. Pain occurs along the distribution of one or
more sensory divisions of the trigeminal nerve, most often the
maxillary. The pain is paroxysmal, lasting seconds up to 2 min, but
attacks may recur rapidly. It is lancinating, excruciating, and
sometimes incapacitating. Pain is often precipitated by stimulating a
facial trigger point (eg, by chewing, brushing the teeth, or smiling).
Sleeping on that side of the face is often intolerable.

Diagnosis is clinical. Treatment is usually with carbamazepine or


gabapentin; sometimes surgery is required.
For chronic kidney disease patients whose protein excretion exceeds 1
g/24 hr, the National Kidney Foundation‟s Kidney Disease Outcomes
Quality Initiative (KDOQI) recommends a blood pressure goal of

a) Less than or equal to 150/100 mm Hg


b) Less than or equal to 140/90 mm Hg
c) Less than or equal to 130/80 mm Hg
d) Less than or equal to 125/75 mm Hg
e) Less than or equal to 110/60 mm Hg
The correct answer is D

Explanation
The Kidney Disease Outcomes Quality Initiative guideline recommends a
blood pressure goal of less than or equal to 130/80 mm Hg for patients
with normal urinary albumin concentrations, and a blood pressure goal of
less than or equal to 125/75 mm Hg for patients whose protein excretion
exceeds 1 g/24 hr.
A 72 year old man with a history of recently diagnosed small cell cancer
of the lung, presents to the emergency room following a witnessed,
single, tonic-clonic seizure. His serum sodium is 106 mmol/L. Which one
of the following is the best immediate approach to the management of his
electrolyte disturbance?

a) Fluid restriction
b) Intravenous normal saline at 25 mL/hr
c) Intravenous normal saline at 100 mL/hr
d) Intravenous 3% saline at 25 mL/hr
e) Intravenous 3% saline at 100 mL/hr
The correct answer is E

Explanation
Given the presentation, this patient should be treated with Intravenous
3% saline at 100 mL/hr with frequent (q 2 to 4 h) electrolyte
determinations.

Severe hyponatremia (plasma Na < 109 mEq/L; effective osmolality < 238
mOsm/kg) in *asymptomatic* patients can be treated safely with stringent
restriction of water intake. However, treatment is more aggressive when
neurologic symptoms (eg, confusion, lethargy, seizures, coma) are
present. The debate primarily concerns the pace and degree of
hyponatremia correction.

Many experts recommend that plasma Na be raised no faster than 1


mEq/L/h, but replacement rates of up to 2 mEq/L/h for the first 2 to 3 h
have been suggested for patients with seizures. Regardless, the rise
should be > 10 mEq/L over the first 24 h. More vigorous correction risks
precipitation of osmotic demyelination syndrome.

Hypertonic (3%) saline (containing 513 mEq Na/L) may be used, but only
with frequent (q 2 to 4 h) electrolyte determinations. For patients with
seizures or coma, ? 100 mL/h may be administered over 4 to 6 h in
amounts sufficient to raise the serum Na 4 to 6 mEq/L.
A 68 year old patient comes to ER complaining of dyspnea. On exam his
vitals show HR 160, BP 110/60. His EKG shows supraventricular
tachycardia (SVT). What is the treatment of choice?

a) Digoxin PO
b) Digoxin IV
c) Digoxin and Furosemide
d) Digoxin and Warfarin
e) Adenosine
The correct answer is E

Explanation
In the treatment of SVT, adenosine, a short-acting medication is used to
decrease the heart rate. This medication is given by IV to act quickly.
Adenosine has some temporary side effects, including facial flushing,
chest pain, shortness of breath, nausea and dizziness. If a single dose
does not stop supraventricular tachycardia, then you can give higher
doses. Adenosine successfully stops paroxysmal supraventricular
tachycardia (PSVT) in more than 90% of cases.

If adenosine is unsuccessful, other medications can be used, such as


calcium channel blockers, digoxin (Lanoxin), or beta-blockers.

A 39 year old white male comes to your office with severe pain,
fluctuance, erythema, and tenderness localized over the tip of the
distal middle finger. The most likely diagnosis is

a) A paronychia
b) A felon (whitlow)
c) An eponychial abscess
d) Bacterial tenosynovitis
e) Digital cellulitis
The correct answer is B

Explanation
A felon, also called a whitlow, is an infection of the digital pulp of
the terminal phalanx. The anatomic characteristics of this region
compartmentalize infection, resulting in exquisite pressure and pain in
the pulp spaces. As with all abscesses, incision and drainage is the
single most important therapy.

Paronychial and eponychial infections occur in the dorsal surface of the


finger about the nail. Bacterial tenosynovitis is an extension of a deep
space infection along the tendons. Digital cellulitis is a nonspecific
term and does not adequately describe the infection depicted. The
presence of fluctuance with this infection suggests abscess formation
rather than a simple cellulitis.
A 50 year old man with a history of hemochromatosis presents to the
emergency room vomiting up bright red blood. He had his most recent
phlebotomy yesterday. His blood pressure is 110/85 mmHg, his pulse 115
beats per minute; his face is flushed, and he is diaphoretic. During the
physical examination, splenomegaly and a venous pattern on his chest and
abdomen are noted. He seems somewhat drowsy and confused but has no
focal neurologic signs. What is the most likely cause of this patient's
bleeding?

a) Portal hypertension
b) Hemoglobin deficiency
c) Gastric ulcer
d) Bronchogenic carcinoma
e) Factor VIII deficiency
The correct answer is A

Explanation
Long term complications of hemochromatosis include liver cirrhosis.
Portal hypertension is caused most often by cirrhosis (in developed
countries), schistosomiasis (in endemic areas), or hepatic vascular
abnormalities. Consequences include esophageal varices and
portal-systemic encephalopathy.

Portal hypertension is asymptomatic; symptoms and signs result from its


complications. The most dangerous is acute esophageal variceal bleeding.
Patients typically present with sudden painless upper GI bleeding, often
massive. Bleeding from portal hypertensive gastropathy is often subacute
or chronic. Ascites, splenomegaly, or portal-systemic encephalopathy may
be present.

Portal hypertension is inferred in a patient with chronic liver disease


by the presence of collateral circulation, splenomegaly, ascites, or
portal-systemic encephalopathy. Proof requires direct portal pressure
measurement by a transjugular catheter, which is invasive and usually
not performed. Imaging may help when cirrhosis is suspected. Ultrasound
or CT often reveals dilated intra-abdominal collaterals, and Doppler
ultrasound can determine portal vein patency and flow.

Diagnosis is based on clinical criteria, often in conjunction with


imaging studies and endoscopy. Treatment involves prevention of GI
bleeding with endoscopy, drugs, or both, and sometimes with portocaval
shunting.
A 53 year old woman with history of ?wetting herself? presents with
dysuria and lower abdominal pain. She is noticed to have a 12cm x 12cm
mass, that is palpable in her lower abdomen. What is the most likely
cause of her presentation?

a) Ovarian tumor
b) Urinary retention
c) UTI
d) Uterine fibroids
The correct answer is B

Explanation
Urinary retention is incomplete emptying of the bladder or cessation of
urination; it may be acute or chronic. Causes include impaired bladder
contractility, bladder outlet obstruction, detrusor-sphincter
dyssynergia (lack of coordination between bladder contraction and
sphincter relaxation), or a combination.

Retention is most common among men, in whom prostate abnormalities or


urethral strictures cause outlet obstruction. In either sex, retention
may be due to drugs (particularly those with anticholinergic effects,
including many OTC drugs), severe fecal impaction (which increases
pressure on the bladder trigone), or neurogenic bladder in patients with
diabetes, multiple sclerosis, Parkinson's disease, or prior pelvic
surgery resulting in bladder denervation.

Urinary retention can cause urinary frequency and urge or overflow


incontinence. It may cause abdominal distention and pain. When retention
develops slowly, pain may be absent. Long-standing retention predisposes
to UTI and can increase bladder pressure, causing obstructive uropathy.

Diagnosis is obvious in patients who cannot void. In those who can void,
diagnosis is by postvoid catheterization showing a residual urine volume
> 100 mL.

Relief of acute urinary retention requires urethral catheterization.


Subsequent treatment depends on cause. In men with benign prostatic
hypertrophy, drugs (usually ?-adrenergic blockers or 5?-reductase
inhibitors) or surgery may help decrease bladder outlet resistance.

Which one of the following is more effective than epinephrine for the
treatment of asystole?

a) Amiodarone (Cordarone)
b) Atropine
c) Lidocaine (Xylocaine)
d) Sodium bicarbonate
e) Vasopressin (Pitressin)
The correct answer is E

Explanation
The effects of vasopressin have been shown to be similar to those of
epinephrine in the management of ventricular fibrillation and pulseless
electrical activity, but vasopressin has been shown to be superior to
epinephrine in patients with asystole. The use of vasopressin followed
by epinephrine may be more effective than the use of epinephrine alone
in patients with refractory cardiac arrest.
This EKG shows which one of the following patterns‟
a) Left anterior fascicular block
b) Left bundle-branch block
c) Right bundle-branch block
d) Wenckebach block
e) Wolff-Parkinson-White syndrome
The correct answer is E

Explanation
The QRS complex is widened (0.12 sec), but this is due to prominent
delta waves, best seen here in the lateral leads; this feature plus the
short PR interval of 0.10 seconds establishes this as the pseudo-left
bundle-block pattern of Wolff-Parkinson-White (WPW) syndrome. WPW is one
of the preexcitation syndromes, so-called because of the existence of
one or more bypass tracts or accessory bundles through which accelerated
conduction can occur. This EKG illustrates type B WPW with a QS in V1
and V2 and an R wave in leads V5 and V6.

WPW syndrome is associated with paroxysmal tachyarrhythmias. On


electrophyiologic testing, the individual whose EKG is shown proved to
have two accessory bypass tracts and four different types of inducible
supraventricular tachycardias ? three reciprocating and the other an
antegrade atrial flutter/atrial fibrillation at a high rate of
conduction. If he were to present to the emergency department in atrial
fibrillation, digoxin and verapamil would be contraindicated since they
will block AV conduction but not slow down bypass tract conduction;
these agents could, in fact, foment degeneration into ventricular
tachycardia and fibrillation.
An elderly man traveled to North America from Australia. After 5 days he
develops dyspnea, fever, tachycardia and is rushed to the hospital. What
is the most likely diagnosis‟

a) Pulmonary embolism
b) Pneumonia
c) Myocardial infarction
d) Aortic dissection
The correct answer is A

Explanation
Pulmonary embolism (PE) is the occlusion of one or more pulmonary
arteries by thrombi that originate elsewhere, typically in the large
veins of the lower extremities or pelvis. Nearly all PEs arise from
thrombi in the lower extremity or pelvic veins (deep venous thrombosis
(DVT).

'Virchow's triad' of stasis, hypercoagulability and endotheloal damage


are risk factors for developing DVT.

This man has sat on a very long airplane flight from Australia to North
America. Meeting the criteria for stasis. His symptoms are classic for
PE which can include: acute dyspnea, pleuritic chest pain, tachycardia
and tachypnea. The first symptom in an older patient may be altered
mental status.
A 50-year-old female reports a 1-month history of pain in her wrists.
She does not recall any injury. On examination both wrists are warm but
not red. They feel boggy to touch and lack 30 degrees of both flexion
and extension. No other joints are affected. She feels fatigued and
unwell, but attributes this to her busy schedule.

Radiographs of her wrists are normal. Laboratory findings are


unremarkable except for a mildly elevated erythrocyte sedimentation rate
and a negative rheumatoid factor.

What is the most likely diagnosis‟

a) Rheumatoid arthritis
b) Osteoarthritis
c) Inapparent injury
d) Fibromyalgia
e) Lyme disease
The correct answer is A

Explanation
Rheumatoid arthritis is most often symmetric at presentation and
particularly affects the wrists and other extremity joints that have a
high ratio of synovium to articular cartilage. In the early months of
the disease, rheumatoid factor is often negative, even in cases where it
will be positive later. Although radiographs and laboratory tests are
helpful, the diagnosis is primarily clinical. Osteoarthritis of the
wrists usually involves the carpal-metacarpal joint of the thumb
primarily, and the joint would be red only if there were an injury.
Fibromyalgia usually involves the soft tissue of the trunk, and does not
produce visible inflammation. Lyme disease can cause a variety of joint
diseases, but not chronic symmetric arthritis.
Which one of the following is considered a normal change in sexual
functioning as men age?

a) The inability to maintain an erection to ejaculation


b) An increased need for direct tactile sensation to obtain or maintain
an erection
c) The development of retrograde ejaculation
d) Decreased time between erections
e) Greater forcefulness of ejaculation
The correct answer is B

Explanation
As men age, there are normal changes in sexual functioning, including an
increase in the time required for sexual arousal, decreased penile
sensitivity (leading to an increased need for direct tactile sensation
to obtain or maintain an erection), increased time between erections,
and less forcefulness of ejaculation. Retrograde ejaculation and the
inability to sustain an erection to ejaculation are not considered
normal accompaniments of aging.
An 18-year-old male seen in your office is found to be overweight and to
have acanthosis nigricans. Both of his parents have a history of
diabetes mellitus. His fasting plasma glucose level is 111 mg/dL (N
<100). Which one of the following is the correct diagnosis‟

a) Prediabetes
b) Type 1 diabetes mellitus
c) Type 2 diabetes mellitus
d) Maturity-onset diabetes of the young
The correct answer is A

Explanation
This patient has prediabetes, which is defined as having a fasting
plasma glucose level of 101?125 mg/dL.

These patients are at high risk for developing diabetes mellitus later
in life. Prediabetes is associated with metabolic syndrome, and weight
loss, exercise, and certain pharmacologic agents have been shown to
prevent or delay the subsequent development of diabetes mellitus.

Diabetes mellitus is diagnosed in three ways: symptoms of diabetes


(polyuria, polydipsia, unexplained weight loss) plus a random plasma
glucose level 200 mg/dL; a fasting plasma glucose level 126 mg/dL; or a
glucose level 200 mg/dL on a 2-hour 75-g oral glucose tolerance test. It
is important to note that in the absence of unequivocal hyperglycemia
the diagnosis must be confirmed by repeat testing on a subsequent day.
Once the diagnosis of diabetes is confirmed, further testing is needed
to differentiate between type 1, type 2, and maturity-onset diabetes of
youth.

A 30 year old female presents to her physician's office for an increase


in fatigue over the last 2 months. On physical examination, the
physician notes delayed capillary refill in the nailbeds and a pale
appearance of the inner lining of the eyelids. A peripheral smear
reveals hypochromic erythrocytes. Which one of the following tests would
be most useful in confirming the probable diagnosis‟

a) Serum ferritin and total iron-binding capacity (TIBC)


b) Schilling's test
c) Glucose 6-phosphate dehydrogenase assay
d) Hemoglobin electrophoresis
e) Serum folate and cobalamin (vitamin B12)
The correct answer is A

Explanation
Hypochromic anemia is a form of anemia characterized by a
disproportionate reduction of red cell hemoglobin in proportion to the
volume of the erythrocyte and an increased area of central pallor in the
red cells. Hypochromic anemia is most commonly caused by iron deficiency
from a low iron intake, diminished iron absorption, or excessive iron
loss. It can also be caused by infections or other diseases, therapeutic
drugs, and lead poisoning. Diagnosis of iron deficiency anemia is made
by obtaining levels of serum iron, ferritin and TIBC.
When considering therapy for the obese patient, one should know that the
major component of caloric expenditure is

a) Dietary thermogenesis (the energy required for metabolism of food)


b) The basal metabolic rate (the amount of energy required to maintain
metabolic homeostasis)
c) Physical activity
d) The negative-feedback response of growth hormone and cortisol to the
ingestion of food
The correct answer is B

Explanation
When caloric intake is greater than energy expenditure, obesity results.
The major components of caloric expenditure are the basal metabolic
rate, dietary thermogenesis, and physical activity. The basal metabolic
rate (the amount of energy required to maintain metabolic homeostasis)
accounts for 60%-70% of caloric expenditure. Dietary thermogenesis (the
energy required for digestion and metabolism of food) accounts for
5%-10% of caloric expenditure and depends upon the amount and type of
food eaten and, to an extent, on the degree of obesity. Physical
activity accounts for about 25%-35% of caloric expenditure in the
average person.
A patient has a surgery for a pituitary tumor. Post-operatively he
develops polyuria. Which continues with signs of dehydration despite
receiving 4 liters of IV fluids. What is the most appropriate management?

a) Give more IV fluids


b) Start oral rehydration
c) Give vasopressin (DDAVP)
d) Give hydrochlorothiazide
The correct answer is C

Explanation
This patient likely has developed damage to the pituitary gland which
has affected the release of anti-diuretic hormone (ADH).

Diabetes insipidus (DI) results from a deficiency of ADH due to a


hypothalamic-pituitary disorder (central DI) or from resistance of the
kidney to ADH (nephrogenic DI). Polyuria and polydipsia develop.

Diagnosis is by water deprivation test showing failure to maximally


concentrate urine. ADH levels and response to exogenous ADH help
distinguish CDI from NDI. Treatment for central DI is with desmopressin
or vasopressin. Treatment for nephrogenic DI includes use of diuretics
(mainly thiazides).
A 62 year old man with a history of obesity, hypertension and GERD,
suffered from “squeezing” sensation in his chest which he described as
"suffocating and heavy." The pain was also felt in his jaw, neck,
shoulders and arms. It sustained for several minutes and then went away.
Right now, his blood pressure is 140/90 with a regular pulse of 98 and
his EKG shows no abnormalities.
What would be the most likely diagnosis of this presentation?

a) Myocardial infraction
b) TIA
c) Angina pectoris
d) Bradyarrhythmia
e) GERD
The correct answer is C

Explanation
Angina pectoris is characterized by „squeezing? or ?gripping? pain which
is suffocating and heavy, sustained for few minutes radiating to jaw,
neck, shoulders and arms. Angina pectoris occurs due to coronary
blockade either through coronary stenosis or atherosclerosis. If
coronary artery, the only artery that supplies blood to myocardium, is
blocked due to stenosis or atherosclerosis, blood supply to myocardium
would be stopped. Angina is one of the ischemic heart diseases that if
left untreated, it would lead to myocardial infarction.
Which one of the following is most consistent with a diagnosis of asthma?

a) Reduced FEV_1 and reduced FEV_1 /FVC ratio


b) Reduced FEV_1 and normal FEV_1 /FVC ratio
c) Reduced FEV_1 and increased FEV_1 /FVC ratio
d) Reduced FVC and normal FEV_1 /FVC ratio
e) Reduced FVC and increased FEV_1 /FVC ratio
The correct answer is A

Explanation
Asthma is typically associated with an obstructive impairment that is
reversible with short-acting bronchodilators. A reduced FEV_1 and FEV_1
/FVC ratio indicates airflow obstruction. A reduced FVC with a normal or
increased FEV_1 /FVC ratio is consistent with a restrictive pattern of
lung function.
A visibly distraught 34-year-old female presents to your office. She is
a housekeeper at a local motel, and while cleaning a room she sustained
a needle stick from a syringe that had been left in a trash can. She
produces the syringe, which is an insulin syringe with an uncapped
small-bore needle.

At this time, it would be appropriate to

a) perform DNA polymerase testing on the needle


b) reassure the patient that the risk of developing HIV is much less
than 1%
c) begin two-drug HIV prophylaxis
d) begin three-drug HIV prophylaxis
The correct answer is B

Explanation
The risk of developing HIV after a large-bore needle stick from a known
HIV positive patient is less than 0.3%. Current guidelines for patients
exposed to small-bore or solid needle sticks from unknown sources do not
recommend prophylaxis, unless there is a high likelihood of the source
being HIV positive.
A 35 year old man has been on life support systems for the past 48 hours
following blunt head trauma. Which of the following is the most
important criterion to declare the patient brain dead and to permit
removal of life support systems‟

a) Bedside EEG showing no electrical activity


b) Decorticate and decerebrate posturing
c) Failure to respond to electroconvulsive stimuli
d) Glasgow coma score of 3 or less
e) Unenhanced MRI of the brain showing dilated ventricles
The correct answer is A

Explanation
The definition of brain death is constantly being revised. However, if
there is no electrical activity on EEG, and if there is total loss of
all brain stem reflexes, then it is safe to label an already
irreversibly comatose patient as brain dead. The most definitive measure
of absent brain activity is an EEG. However, remember that before
interpreting a flat-line EEG, one must ascertain that the patient is not
hypothermic or suffering from barbiturate overdose, since these can give
the appearance of no electrical activity on an EEG.
Which of the following is false with regard to falls in the elderly?

a) Falls are the leading cause of injury-related visits to emergency


departments
b) The mortality rate for falls increases dramatically with age in both
sexes
c) In persons over age 75, fractures of the upper extremity are about
twice as common as fractures of the lower extremity.
d) Falls can be markers of poor health and declining function
e) Risk factors for falls in the elderly include increasing age,
medication use, cognitive impairment and sensory deficits
The correct answer is C

Explanation
Falls are the leading cause of injury-related visits to emergency
departments and the primary etiology of accidental deaths in persons
over the age of 65 years. The mortality rate for falls increases
dramatically with age in both sexes and in all racial and ethnic groups,
with falls accounting for 70% of accidental deaths in persons 75 years
of age and older. Falls can be markers of poor health and declining
function, and they are often associated with significant morbidity.

More than 90% of hip fractures occur as a result of falls, with most of
these fractures occurring in persons over 70 years of age. In persons
over age 75, fractures of the lower extremity are about twice as common
as fractures of the upper extremity. One third of community-dwelling
elderly persons and 60% of nursing home residents fall each year. Risk
factors for falls in the elderly include increasing age, medication use,
cognitive impairment and sensory deficits. Outpatient evaluation of a
patient who has fallen includes a focused history with an emphasis on
medications, a directed physical examination and simple tests of
postural control and overall physical function.

A 45-year-old white male presents with a 3-day history of new-onset


acute upper abdominal pain radiating to the back. His pulse rate is 110
beats/min and other vital signs are normal. Bowel sounds are hypoactive
and epigastric tenderness without guarding is present.

Which one of the following is most accurate regarding laboratory testing


for possible pancreatitis in this situation?
a) Elevated serum triglyceride levels can cause falsely elevated serum
amylase levels
b) There is a direct correlation between serum amylase levels and the
severity of the pancreatitis
c) Serum amylase and lipase levels may be falsely low in renal failure
d) A serum amylase level that is three times normal is highly specific
for pancreatitis
The correct answer is D

Explanation
A serum lipase level is a more specific test for pancreatitis than an
amylase level. Amylase is more likely to be elevated in other acute
abdominal conditions. Amylase levels may also be elevated with salivary
gland abnormalities, chronic sialadenitis, renal failure, and liver disease.

Although elevated triglyceride levels are associated with pancreatitis,


patients with hypertriglyceridemia and proven pancreatitis have been
found to have deceptively low levels of amylase, and sometimes of lipase
activity. There is no significant correlation between the magnitude of
serum amylase elevation and the severity of pancreatitis. In fact, a
milder form of acute pancreatitis is often associated with higher levels
of serum amylase than those seen in a more severe form of the disease.
Serum trypsinogen is secreted specifically by the pancreas and is
helpful in determining when elevated amylase levels are coming from
non-pancreatic sources. Because amylase, lipase, and trypsinogen are all
cleared by the kidneys, renal failure, particularly with a creatinine
clearance < 50 mL/min, may result in delayed clearance and false
elevations of enzymes.

A 72-year-old male presents with unintentional weight loss of 25 lb over


the last 6 months. His history, including a nutritional assessment, is
unremarkable, as is his physical examination. His current medications
include mirtazapine (Remeron) for depression and hydrochlorothiazide for
hypertension. Which one of the following would be the most appropriate
next step?

a) Order a CBC, chemistry panel, stool for occult blood, and TSH
b) Refer for immediate colonoscopy and esophagogastroduodenoscopy
c) Schedule CT of the chest, abdomen, and pelvis
d) Start megestrol (Megace) to promote weight gain
e) Discontinue mirtazapine
The correct answer is A

Explanation
There should be a rational approach to evaluating weight loss in an
elderly patient. The workup should be directed by findings in the
history and physical examination, with special emphasis given to
neurologic and psychosocial aspects. Unless the history or physical
examination point in a specific direction, standard tests should be
performed first, including a CBC, chemistry panel, stool for occult
blood, and TSH level. Although the etiology of unintentional weight loss
in the elderly is malignancy in 16%?36% of such cases, specific tests
are not indicated before CT.

Medications, including SSRIs, NSAIDs, bupropion, digoxin, and metformin


can cause weight loss; however, amitriptyline often leads to weight
gain. Mirtazapine has been shown to increase appetite and promote weight
gain. Megestrol has been used successfully to treat cachexia in patients
with AIDS or cancer. When given in a dosage of at least 320 mg/day,
megestrol has produced weight gain, but side effects of edema,
constipation, and delirium may limit its usefulness. Lower dosages may
be effective for stimulating weight gain in frail elderly patients,
although this approach needs to be tested in randomized, controlled
trials. In the patient described, a workup seeking the etiology of the
weight loss should begin promptly.

A 38 year old woman presents to you convinced that she has chronic
fatigue syndrome. The history reveals no evidence of fever or swollen
glands, and no other symptoms other than fatigue and difficulty
sleeping. Her physical examination is completely normal. She has a high
thyroid-stimulating hormone level. You would then suspect and
subsequently confirm

a) Hyperthyroidism secondary to pituitary adenoma


b) Hypothyroidism
c) Grave disease
d) Thyroiditis
e) Chronic fatigue syndrome
The correct answer is B

Explanation
Hypothyroidism is underactivity of the thyroid gland that leads to
inadequate production of thyroid hormone and a slowing of vital body
functions.

Insufficient thyroid hormone causes body functions to slow. Symptoms are


subtle and develop gradually. They may be mistaken for depression,
especially among older people. Facial expressions become dull, the voice
is hoarse and speech is slow, eyelids droop, and the eyes and face
become puffy. Many people with hypothyroidism gain weight, become
constipated, and are unable to tolerate cold.

Hypothyroidism is common, especially among older people, particularly


women; it affects about 10% of older women. It can, however, occur at
any age. Very severe hypothyroidism is called myxedema.

Diagnosis is made by obtaining TSH and FreeT4 levels. Treatment is with


thyroid replacement with synthroid.
A 26-year-old female presents with a 2-month history of amenorrhea,
nausea, and fluttering in her chest. The fluttering feels similar to
what she experienced 3 years ago when diagnosed with Graves‟ disease. At
that time, she was successfully treated with medication, which she
discontinued after 18 months. Current laboratory tests reveal a positive
hCG, a TSH of 0.03 micro U/mL (N 0.4?5.0), and a free T4 of 4.0 micro
g/dL (N 0.8?2.0). Which one of the following would be the most
appropriate treatment in this situation?

a) 131 Iodine ablation


b) Propylthiouracil
c) Subtotal thyroidectomy
d) Methimazole (Tapazole)
The correct answer is B

Explanation
Overt hyperthyroidism causes an increase in neonatal morbidity from
preterm birth and low birth weight. Propylthiouracil should be
considered the treatment of choice because methimazole may be associated
with congenital anomalies. 131I is contraindicated in pregnancy because
of radiation dangers to the fetus, as well as thyroid destruction.
Although subtotal thyroidectomy is a viable treatment option, it is
recommended only if medical therapy is unsuccessful.
In patients with eating disorders, chronic laxative use is associated
with which one of the following?

a) Salivary gland enlargement


b) Hypokalemia
c) Dental enamel erosion
d) Esophagitis
The correct answer is B

Explanation
Complications of laxative use include electrolyte acid-base disorders
(acute acidosis, chronic hypokalemic alkalosis), dehydration,
constipation, cathartic colon, and edema.

Complications of vomiting include electrolyte acid-base disorders


(hypokalemic, hypochloremic metabolic alkalosis), dehydration, dental
enamel erosion, salivary gland enlargement, esophageal rupture,
bleeding, esophagitis, irritable bowel symptoms (chronic), edema, and
hypokalemic nephropathy.
Which of the following is *least* likely to cause avascular necrosis‟

a) Sickle cell disease


b) Hip dislocation
c) Steroid use
d) Constrictive dressings
e) Post-traumatic fracture
The correct answer is D

Explanation
Avascular necrosis (AVN) is focal bone infarction, which may be
secondary to various conditions or idiopathic. Severe osteoarthritis can
result. Symptoms include bone or joint pain. Early diagnosis is best
made by MRI.

AVN involves ischemic death of osteocytes and other bone marrow


components, producing subchondral bone infarction. Of the many
conditions associated with AVN, some are more clearly causative.

Posttraumatic AVN develops when blood supply is impaired. Susceptible


bone is usually intra-articular and has a limited attachment of soft
tissue and accompanying vasculature. The hips, shoulders, body of the
talus, and carpal scaphoid are commonly affected.

Causes of avascular necrosis include alcohol abuse, coagulation


disorders, corticosteroids (high dose), decompression sickness, fatty
liver, fracture of the femoral neck, hip dislocation and sickle cell
disease.
A 45-year-old female with rheumatoid arthritis has a hemoglobin level of
9.5 g/dL (N 11.5?16.0). Her arthritis is well controlled with
methotrexate (Trexall).

Further evaluation reveals the following:


Hematocrit. ........... 29.0% (N 35.0-47.0)
Mean corpuscular volume.. . 78 micro m3 (N 80-98)
Platelets. . . . . . . . . . . . . . 230,000/mm (N 150,000-400,000)
WBCs. .............. 6900/mm (N 4000-11,000)
Differential. ......... normal
Serum iron. ........... 15 micro g/dL (N 50-170)
Total iron binding capacity. . 150 micro g/dL (N 45-70)
Iron saturation. ......... 10% (N 15-50)
Serum ferritin. ......... 7 ng/mL (N 12-150)
Reticulocyte count.. ...... 8×10 /L (N 10-100)
Stool guaiac............ negative ×3

Which one of the following would be the most appropriate next step?

a) Evaluation for a source of blood loss


b) Hemoglobin electrophoresis to screen for thalassemia
c) Stopping the methotrexate and beginning an alternative treatment for
rheumatoid arthritis
d) No further evaluation
The correct answer is A

Explanation
Anemia of chronic disease is characterized by the underproduction of red
cells, due to hypoferremia caused by the uptake of iron by the
reticuloendothelial system. Total-body iron stores are increased but the
iron in storage is not available for red cell production. This anemia is
normochromic and normocytic, and is associated with a reduction in iron,
transferrin, and transferrin saturation. Ferritin is either normal or
increased, reflecting both the increased iron within the
reticuloendothelial system and increases due to immune activation (acute
phase reactant). In iron deficiency anemia, total-body iron levels are
low, leading to hypochromia and microcytosis, low iron levels, increased
transferrin levels, and reduced ferritin levels. This patient‟s anemia
is most likely multifactorial, with anemia of chronic disease and drug
effects playing a role. However, she also has iron deficiency, and
searching for a source of blood loss would be important. With
thalassemia, marked microcytosis is seen, and with hemolysis of any
standing, slight macrocytosis and an increased reticulocyte count would
be expected.
A 62 year old male comes in for high blood pressure follow up. Patient
has history of obesity, diabetes mellitus and angina pectoris. Last
visit's blood pressure readings have shown 160/100. Today, his blood
pressure is 140/90. You decide to start him on a medication to treat his
hypertension. Given his past medical history and diagnosis, what would
be the best choice of the following?

a) Propranolol
b) Lisinopril
c) Amlodipine
d) Hydralazine
e) Telmisartan
The correct answer is C

Explanation
Answer: C ? Amlodipine is the right medication for hypertension in
patients with angina pectoris. Calcium channel blockers inhibit the
influx of the calcium ions through the calcium channels. This results in
the relaxation of the vascular smooth muscle. Calcium channel blockers
also produce systemic and coronary vasodilation by blocking the calcium
ions influx. Since the calcium channel blockers produce relaxation of
the smooth muscle of coronary arteries it is a good medication for the
patients with angina pectoris.

A. Propranolol can worsen this patient's diabetes.


B. Lisinopril is not as effective as calcium channel blockers in this case.
D. Hydralazine is not recommended for this patient because Hydralazine
can cause tachycardia, potentially leading to myocardial ischemia and
angina attacks
E. Telmisartan is not used in patients with angina pectoris.
A 39 year old man is brought to the emergency department by his same sex
partner because of confusion, diplopia and mild right arm weakness. On
examination the patient is somewhat agitated and shows confusion for
recent events. There is decreased pupillary response on the left with
some paresis of lateral gaze on the right. Temperature is 38.3°C
(101.0°F). Peripheral leukocyte count is increased. The most appropriate
next step in evaluation of his neurologic signs and symptoms is

a) Bilateral carotid arteriography


b) CT scan of the head
c) Electroencephalography
d) Lumbar puncture
e) Serum test for HIV antibodies
The correct answer is B

Explanation
The patient is described as a homosexual male with both sebacate and
acute neurologic abnormalities. Furthermore, the neurologic signs and
symptoms described clearly indicate focal findings as would be seen with
a mass lesion. In a homosexual man, one would suspect the possibility of
toxoplasmosis, and a CT scan of the head should be performed.
Toxoplasmosis would be revealed as multiple ring-enhancing lesions.

Bilateral carotid angiography and an electroencephalogram are not


indicated at this time. A lumbar puncture would in fact be
contraindicated given the possibility of a mass lesion and possible
increased intracranial pressure. While order a serum test for HIV
antibodies may be appropriate it will not be most useful in evaluating
his current signs and symptoms. intubation and intravenous antibiotic
therapy.
A 40 year old woman has episodic headaches that last 30 minutes and are
associated with sweating, palpitations, and feelings of apprehension.
She has lost 6.8 kg over the past three months. Physical examination
reveals a thin woman with a pulse of 112 beats/minute and a blood
pressure of 150/100 mmHg lying and 130/80 mmHg standing. Which one of
the following is the most helpful diagnostic test?

a) CT scan of the brain


b) Captopril renal nuclear medicine scan
c) 24 hour urinary 5-hydroxyindoleacetic acid (5-HIAA) levels
d) 24 hour urinary aldosterone levels
e) 24 hour urinary vanillylmandelic acid (VMA) and catecholamine levels
The correct answer is E
Explanation
A pheochromocytoma is a catecholamine-secreting tumor of chromaffin
cells typically located in the adrenals. It causes persistent or
paroxysmal hypertension. Diagnosis is by measuring catecholamine
products in blood or urine. Imaging tests, especially CT or MRI, help
localize tumors. Treatment involves removal of the tumor when possible.
Drug therapy for control of BP includes b-blockade, possibly combined
with b-blockade.
All of the following are treatments for physiological non-scarring
alopecia, except

a) Spironolactone
b) Minoxidil
c) Hair transplantation
d) Intralesional triamcinalone
e) Finasteride
The correct answer is D

Explanation
Most treatments for hair loss have been developed for male-pattern
baldness because it is so prevalent. Minoxidil prolongs the anagen phase
and may increase blood flow to the follicle; 1 mL of 2% or 5% topical
drug applied bid to the scalp is most effective for vertex alopecia in
male-pattern baldness affecting men or women. However, at most 30 to 40%
of patients experience significant hair growth, and minoxidil is
generally not effective or indicated for other causes except possibly
alopecia areata.

Finasteride inhibits 5-alfa reductase enzyme, blocking conversion of


testosterone to dihydrotestosterone, and is useful for male-pattern
baldness. Finasteride 1 mg po once/day stimulates scalp hair growth.

Surgical options include follicle transplant, scalp flaps, and alopecia


reduction. Few procedures have been subjected to scientific scrutiny,
but patients who are self-conscious about their hair loss may consider them.

A 50 year old hypertensive patient presents to the ER complaining of


palpitations and shortness of breath. While taking a brief history, he
becomes confused and unresponsive. A STAT EKG shows the tracing below:

Which of the following is the most appropriate next step of management


of this patient?

a) Amiodarone
b) Asynchronized cardioversion
c) CPR
d) Propranolol
e) Synchronized cardioversion
The correct answer is E

Explanation
A rule of thumb for the boards and the wards: Any Arrhythmia + Unstable
Patient = Cardioversion! The EKG shows irregular QRS complexes with
absent P waves; which is diagnostic of atrial fibrillation. There is no
time to try any medications. The patient is unstable thus synchronized
cardioversion is warranted.

Asynchronized cardioversion is indicated for ventricular fibrillation.

CPR is indicated for asystole and pulseless electrical activity (PEA).

Beta Blockers are excellent rate controlling drugs; however they will
not save the patient‟s life at the moment.

Medical cardioversion could be tried in the acute atrial fibrillation in


a stable patient or in the chronic atrial fibrillation (>48hrs) provided
that the patient is adequately anticoagulated with a therapeutic INR (2-3).
A 43-year-old male complains of difficulty washing his face and combing
his hair with his right hand. On examination a nodule, band, and slight
contracture are noted in the palm proximal to the fourth finger. This
patient‟s symptoms are associated with which one of the following?

a) Hyperparathyroidism
b) Diabetes mellitus
c) Hyperthyroidism
d) Hypothyroidism
e) Adrenal insufficiency
The correct answer is B

Explanation
The patient has Dupuytren‟s disease, which is most common in men over 40
years of age. It is a progressive condition that causes the fibrous
fascia of the palmar surface to shorten and thicken. It initially can be
managed with observation, but corticosteroid injection and surgery may
be needed. The condition will regress in 10% of patients. There is a
3%-33% prevalence of Dupuytren‟s contracture in patients with diabetes
mellitus; however, these patients tend to have a mild form of the
disease with slow progression.
A 71 year old man is admitted to the hospital because of fever, malaise
and weight loss for the past 6 weeks. His temperature has been as high
as 38.3°C (101.0°F) daily. Today, vital signs are: temperature is 38.8°C
(101.8°F), pulse 100/min and respirations 14/min.

The patient appears chronically ill but is in no acute distress. Other


physical findings include a cotton-wool exudate in the right eye,
crackling rales at the lung bases and a moderate blowing grade 2/6
pansystolic murmur. Leukocyte count is 11,500/mm3 with normal
differential and hematocrit is 35%. Urinalysis shows 4 WBC/hpf and 10
RBC/hpf. At this point which of the following is the most likely diagnosis‟

a) Bacterial endocarditis
b) Diverticulitis
c) Hodgkin disease
d) Pyelonephritis
e) Vivax malaria
The correct answer is A

Explanation
The patient in question most likely has endocarditis. He has had
constitutional symptoms of fever, malaise, weight loss, and night sweats
for the last four to six weeks. This interval of time indicates that
this is not an acute illness, such as would be caused by diverticulitis
or pyelonephritis . There is no risk factor described for the patient to
have developed malaria. Although patients with Hodgkin disease may
present with these constitutional symptoms over four to six weeks, there
is no description in the physical examination of adenopathy or
hepatosplenomegaly. Furthermore, there is also a murmur present in
association with fever.
A 25 year old male has a dental infection associated with facial
swelling and lymphadenopathy. Which one of the following is the most
appropriate antibiotic?

a) Cephalexin (Keflex)
b) Tetracycline
c) Penicillin
d) Ciprofloxacin (Cipro)
e) Azithromycin (Zithromax)
The correct answer is C

Explanation
Dental infections complicated by the development of cellulites should be
treated with oral antibiotic therapy. The antibiotic of choice is
penicillin. Clindamycin should be used if a patient is allergic to
penicillin.
A 25 year old female, arrives at her family physician‟s office
complaining of easy bruising over the past month. She is not as
concerned about her ?rash-like? skin bruising as she is about her gums
bleeding with brushing her teeth over the past three days. What
investigations would be unnecessary to order?

a) CBC and differential


b) Serum electrolytes & creatinine
c) Fibrin split products
d) PT & PTT (includes INR)
The correct answer is C

Explanation
This patient presents with symptoms that are consistent with a diagnosis
of either vitamin deficiency, lymphoma or leukemia or lupus. She has
been health before.

Appropriate tests to order include a complete blood count to assess if


she is anemic. A basic metabolic panel and tests to evaluate her
coagulation system such as PT and PTT.

Fibrin split products is a test used to diagnose DIC (disseminated


intravascular coagulation). DIC that evolves slowly (over weeks or
months) causes primarily venous thrombotic and embolic manifestations;
DIC that evolves rapidly (over hours or days) causes primarily bleeding.
DIC occurs in during complications of obstetrics, infection
(particularly with gram-negative organisms) and malignancy.
Risk factors for hepatocellular carcinoma include all of the following,
except

a) Hepatitis A
b) Cirrhosis
c) Exogenous steroid use
d) Hemochromatosis
e) Smoking
The correct answer is A

Explanation
Hepatocellular carcinoma (hepatoma) usually occurs in patients with
cirrhosis and is common in areas where infection with hepatitis B and C
viruses is prevalent. Hepatocellular carcinoma is usually a complication
of cirrhosis.

The presence of HBV increases risk of hepatocellular carcinoma by >


100-fold among HBV carriers. Incorporation of HBV DNA into the host's
genome may initiate malignant transformation, even in the absence of
chronic hepatitis or cirrhosis.

Other disorders that cause hepatocellular carcinoma include cirrhosis


due to chronic hepatitis C virus (HCV) infection, hemochromatosis, and
alcoholic cirrhosis. Patients with cirrhosis due to other conditions are
also at increased risk.

Environmental carcinogens may play a role; eg, ingestion of food


contaminated with fungal aflatoxins is believed to contribute to the
high incidence of hepatocellular carcinoma in subtropical regions.

Diagnosis is based on Alfa-fetoprotein (AFP) levels, imaging tests, and


sometimes liver biopsy.
Following a myocardial infarction, a 65 year old man is noted to have a
wide physiological splitting of S2 on cardiac auscultation. Which one of
the following conditions is most likely to be causing this finding?

a) Left bundle-branch block


b) Right bundle-branch block
c) Right ventricular ectopic beats
d) Atrial septal defect
e) Aortic stenosis
The correct answer is B

Explanation
Wide physiological splitting of the 2nd heart sound is seen in the
following cardiac conditions: Delayed pulmonic closure, RBBB (right
bundle Branch block, pulmonary stenosis, atrial septal defect (ASD),
early aortic closure and ventricular septal defect and mitral insufficiency.

This patient is too old to have ASD diagnosed for the first time.

A 42 year old man presents with an acute onset of severe right ocular
pain and blurry vision. Physical examination reveals the pupil to be
mid-dilated and fixed. No discharge is present. Which one of the
following should the examining physician expect?

a) Blood in the anterior chamber


b) An increased intraocular pressure
c) A corneal erosion on slit-lip examination
d) Papilledema
e) Bloodless arterioles and a cherry red fovea
The correct answer is B

Explanation
In acute angle closure glaucoma the patient will have severe ocular pain
and decreased vision, and the patient sees coloured haloes around
lights. Alternatively, there may be little ocular pain but severe
headache and nausea or vomiting. Examination shows a fixed, mid-dilated
pupil and cloudy cornea, and tonometry shows raised intraocular pressure.
A 38-year-old nurse presents with a 6-week history of nervousness,
fatigue, palpitations, and reduced appetite. The physical examination is
notable only for lid lag and a slight diffuse enlargement of her thyroid
gland. A laboratory evaluation is notable for a free T4 level of 8.0
ng/dL (N 0.8-2.7) and a TSH level of 11.4 ?U/mL (N 0.3-4.7).

Which one of the following is the most likely diagnosis‟

a) Graves‟ disease
b) Thyroiditis
c) Struma ovarii
d) Thyrotropin-secreting pituitary adenoma
e) Factitious hyperthyroidism
The correct answer is D

Explanation
A normal or elevated serum thyrotropin level associated with an elevated
free serum thyroxine level is most consistent with a
thyrotropin-secreting pituitary adenoma or a thyroid hormone resistance
syndrome. All of the other conditions listed are associated with an
elevated free thyroxine level, but would result in a suppressed serum
thyrotropin level.
A 25 year old woman with family history of kidney disease is found to be
hypertensive and is started on a beta blocker. What is your next step in
the work up of her condition?

a) Urinary metanephrines
b) Renal doppler ultrasound
c) CT of abdomen
d) Serum cortisol
The correct answer is B

Explanation
This woman, being so young, most likely has high blood pressure from a
secondary cause of renal artery stenosis.

Renovascular hypertension is blood pressure elevation due to partial or


complete occlusion of one or more renal arteries or their branches. It
is usually asymptomatic unless long-standing. A bruit can be heard over
one or both renal arteries in < 50% of patients.

Diagnosis is by physical examination and renal imaging with duplex


ultrasonography, radionuclide imaging, or magnetic resonance
angiography. Angiography is done before definitive treatment with
surgery or angioplasty.
A 35 year old man presents to the ER complaining of retrosternal chest
pain that began 2 days ago.

The pain is sharp and pleuritic in nature and increases when the patient
lies down. He recalls flu like illness one week ago. The EKG shows the
following tracing below:

What is the most appropriate next step of management of this patient?

a) Azithromycin
b) Heparin
c) Ibuprofen
d) Prednisone
e) Streptokinase
The correct answer is C

Explanation
Acute pericarditis is usually viral in etiology and presents by a sharp,
pleuritic chest pain that is decreased when leaning forward. The
patient‟s EKG is characteristic for acute pericarditis: diffuse
ST-segment elevation in all leads with PR-segment depression.

This is not MI! Don?t be fouled, always screen the ST-segment with your
eyes in all leads.

Acute pericarditis is best treated with NSAIDS.

-Prednisone would be indicated when the acute pericarditis symptoms are


resistant to NSAIDs.
-Streptokinase would be appropriate for an STEMI within 12 hours of
presentation.
-Heparin is appropriate for acute coronary syndromes.
-Antibiotics have no role in the treatment of this viral pericarditis.

A 75 year old white male with well-controlled type 2 diabetes mellitus


is scheduled for an abdominal CT scan with oral and intravenous
iodinated contrast. Which one of the following medications should be
withheld 48 hours before and after the procedure?

a) Glyburide (Micronase, DiaBeta)


b) Glipizide (Glucotrol)
c) Acarbose (Precose)
d) Metformin (Glucophage)
e) Rosiglitazone (Avandia)
The correct answer is D

Explanation
Metformin should be withheld before and after radiographic procedures
with contrast, due to its interaction with iodinated contrast materials.
This interaction may cause impaired renal function or lactic acidosis.
The other drugs listed do not carry this risk.
A 25 year old woman returns to the office for a follow-up visit. She was
last seen 2 weeks ago because of a facial rash that worsens with sun
exposure. Today she says she has joint pain. She says, "It just hurts
everywhere. Sometimes my knees hurt, and sometimes it‟s my elbows or my
ankles. And I have been feeling tired all the time. I just don't have
any energy." She denies any joint swelling.

An office urinalysis shows 3+ protein. You consider the diagnosis of


systemic lupus erythematosus and plan additional diagnostic testing. If
she has systemic lupus erythematosus, studies are most likely to show
which of the following?

a) Decreased numbers of helper T cells


b) Decreased serum concentrations of C3 and C4
c) Increased numbers of plasma cells
d) Serum antimicrosomal antibodies
e) Serum antiplatelet antibodies
The correct answer is B

Explanation
Lupus is a chronic inflammatory disease that can affect various parts of
the body, especially the skin, joints, blood, and kidneys. Arthralgia
and skin rashes are very common findings in these patients. The ANA
antibody test is often used as a first test in the diagnostic evaluation
of lupus. Laboratory tests which measure complement levels in the blood
are also of some value. Complement is a blood protein that, with
antibodies, destroys bacteria. It is an “amplifier” of immune function.
If the total blood complement level is low, or the C3 or C4 complement
values are low, and the person also has a positive ANA, some weight is
added to the diagnosis of lupus. Low C3 and C4 complement levels in
individuals with positive ANA test results may also be indicative of
lupus kidney disease.

Decreased helper (CD4) T cells are characteristic of certain hereditary


T-cell deficiencies as well as acquired ones such as AIDS. Increased B
cells are characteristic of certain leukemias, lymphomas, or disorders
such as multiple myeloma. Serum antimicrosomal antibodies are
characteristic of subacute lymphocytic thyroiditis. This occurs most
often in the postpartum period but may also occur sporadically.
Antimicrosomal antibodies are present in 50 to 80% of patients, while
antithyroid peroxidase antibodies are present in nearly all patients.
Serum antiplatelet antibodies are found in disorders such as
heparin-induced thrombocytopenia and certain idiopathic thrombocytopenias.
This picture shows a 1-5-cm lesion on a man‟s upper back. This lesion is
most typical of which one of the following?

a) Junction nevus
b) Superficial spreading malignant melanoma
c) Senile hemangioma
d) Carbon tattoo
e) Kaposi‟s sarcoma
The correct answer is B

Explanation
The upper back of men and women and the shins of women are common
locations for superficial spreading malignant melanoma. The lesion is
characterized by different shades of tan to black, and a notched
irregular margin. Its appearance is more ominous and irregular than a
junction nevus, and it is too brown and irregular to be the new variant
of Kaposi‟s sarcoma. Carbon tattoos result from traumatic or
self-induced imbedding of carbon particles in the skin. This produces
isolated circumscribed areas of blue-black coloration. Senile
hemangiomata are slightly raised and ruby red in color.
Which one of the following is considered first-line treatment for
shock-resistant ventricular tachycardia?

a) Amiodarone (Cordarone)
b) Vasopressin (Pitressin)
c) Lidocaine (Xylocaine)
d) Procainamide (Pronestyl)
e) Magnesium
The correct answer is A

Explanation
The treatment of shock-resistant ventricular tachycardia (VT) has
changed in recent years. Amiodarone is now the first-line drug, rather
than vasopressin or epinephrine. Lidocaine is less effective than other
agents for terminating VT, but has a use in patients with stable VT with
preserved ventricular function. Procainamide and lidocaine may be used
in similar situations, but procainamide is also useful for atrial
dysrhythmias.
A 48 year old alcoholic man has had a deep venous thrombosis (DVT) and
the INR is well-controlled on warfarin. He is noted to have an INR of
6.0 following a weekend binge. Which one of the following is the most
likely pharmacological explanation of this finding?

a) Alcohol intake decreases the volume of distribution of warfarin


b) Alcohol intake increases the absorption of warfarin from the gut
c) Alcohol competes for and reduces the hepatic metabolism of warfarin
d) Alcohol reduces the plasma protein binding of warfarin
e) Alcohol reduces the renal excretion of warfarin
The correct answer is C

Explanation
Intermediate use (2-3 drinks per day) probably does not alter the INR at
all.
Intermittent large amount of alcohol drinking leads to an increase in
INR, because the alcohol interferes with warfarin metabolism, i.e.
warfarin is metabolized less rapidly.
Chronic heavy alcohol intake results in a decreased INR, because the
alcohol actually increases the metabolism of warfarin.
Routine blood tests frequently reveal elevated calcium levels. When this
evaluation is associated with elevated parathyroid hormone levels, which
one of the following is an indication for parathyroid surgery?

a) Age > 50
b) Kidney stones
c) Serum calcium 0.5 mg/dL above the upper limit of normal
d) Concurrent hyperthyroidism
e) Increased bone density
The correct answer is B

Explanation
Indications for parathyroid surgery include kidney stones, age less than
50, a serum calcium level greater than 1 mg/dL above the upper limit of
normal, and reduced bone density. Hyperthyroidism is not a factor in
deciding to perform parathyroid surgery.
Ischemic pain of the heart develops when the oxygen demand by the
myocardium exceeds the supply to it by coronary arteries either due to
coronary artery disease or blockade due to atherosclerosis or clot
formation. Which of the following does NOT increase myocardial oxygen
demand?

a) Increased myocardial contractility


b) Increased left ventricular diastolic volume
c) Aortic valve stenosis
d) Decrease in systemic vascular resistance
e) Ventricular dilation
The correct answer is D

Explanation
Oxygen demand is a concept that is closely related to the oxygen
consumption of an organ. The two terms are often used interchangeably
although demand is related to need, whereas consumption is the actual
amount of oxygen consumed per minute. Under some conditions, demand may
exceed consumption because the latter may be limited by the delivery of
oxygen to the myocardium.
Major determinants of myocardial oxygen consumption are: Tension,
Preload, Afterload, Contractility, Heart Rate.

>Doubling heart rate approximately doubles MVO2 because ventricular


myocytes are generating twice the number of tension cycles per minute.
>Increasing inotropy (contractility) also increases MVO2 because the
rate of tension development is increased as well as the magnitude of
tension, both of which result in increased ATP hydrolysis and oxygen
consumption.
>Increasing preload (ventricular end-diastolic volume) also increases MVO2;
>Afterload is increased when aortic pressure and systemic vascular
resistance are increased, by aortic valve stenosis, and by ventricular
dilation. When afterload increases, there is an increase in end-systolic
volume and a decrease in stroke volume.
A 56 year old black male with long standing hypertension and a
30-pack-year smoking history has a 2-day history of dyspnea on exertion.
Physical examination is unremarkable except for rare crackles at the
bases. Which one of the following serologic tests would be most helpful
for detecting left ventricular dysfunction?

a) Brain Natriuretic peptide (BNP)


b) Troponin-T
c) C-reactive protein (CRP)
d) D-dimer
e) Cardiac interleukin-2
The correct answer is A

Explanation
Brain Natriuretic peptide (BNP) is a 32-amino acid polypeptide secreted
from the cardiac ventricles in response to ventricular volume expansion
and pressure overload. The major source of BNP is the cardiac
ventricles, and because of its minimal presence in storage granules, its
release is directly proportional to ventricular dysfunction. It is a
simple and rapid test that reliable predicts the presence or absence of
left ventricular dysfunction on an echocardiogram.
Which one of the following most favors a diagnosis of cardiogenic shock
in a patient with acute myocardial infarction?

a) Acute pulmonary edema refractory to standard treatment


b) Ventricular arrhythmia and hypotension
c) Hypotension and low central venous pressure
d) Hypotension and high central venous pressure
e) Hypotension and sinus bradycardia
The correct answer is D

Explanation
Cardiogenic shock is based upon an inadequate circulation of blood due
to primary failure of the ventricles of the heart to function effectively.

Since this is a category of shock there is insufficient perfusion of


tissue (i.e. the heart) to meet the required demand for oxygen and
nutrients. This leads to cell death from oxygen starvation, hypoxia.
Because of this it may lead to cardiac arrest (or circulatory arrest)
which is an acute cessation of cardiac pump function.

Cardiogenic shock is defined by sustained hypotension with tissue


hypoperfusion despite adequate left ventricular filling pressure. Signs
of tissue hypoperfusion include oliguria (<30 mL/h), cool extremities,
and altered mentation. Hypotension due to decrease in cardiac output. A
rapid, weak, thready pulse due to decreased circulation combined with
tachycardia. Distended jugular veins due to increased jugular venous
pressure.
A 57 year old black male presents to your office 2 days after evaluation
in the emergency department for acute difficulty urinating. His acute
symptoms have improved. He requests screening for prostate cancer. Your
are concerned that his visit to the emergency department may affect the
results of prostate cancer screening, and you decide to obtain more
information. Which one of the following possible occurrences 2 days
earlier would NOT preclude prostate-specific antigen (PSA) screening?

a) Performance of a rectal examination


b) Performance of prostate massage
c) Performance of cystoscopy
d) A diagnosis of urinary tract infection
e) A diagnosis of urinary retention
The correct answer is A

Explanation
Elevated results on prostrate-specific antigen (PSA) screening are
associated with a variety of factors. A simple rectal examination,
however, does not elevate the level.

Benign prostatic examination is a frequent cause of mild elevations in


PSA. Interventions such as prostatic massage or cystoscopy will also
elevate the level. Other conditions that can elevate PSA include urinary
tract infections and urinary retention.
The most common cause of hemoptysis seen in the primary care setting is

a) pulmonary embolism
b) lower respiratory tract infection
c) lung cancer
d) heart failure
e) asthma
The correct answer is B

Explanation
Infection of the lower respiratory tract causes well over half of all
cases of hemoptysis, with bronchitis implicated more often than
pneumonia. Lung cancer should always be considered, as it may account
for up to a quarter of hemoptysis cases. Pulmonary embolism is a much
less common cause.
A 69-year-old white male with severe chronic low back pain is treated
with oxycodone, 80 mg twice daily and 15 mg every 4 hours for
breakthrough pain. Which one of the following is the best initial
regimen to prevent associated constipation?

a) Docusate sodium/senna
b) Sorbitol
c) Lactulose
d) Weekly enemas
e) No prophylactic treatment
The correct answer is A

Explanation
Patients taking opioids should receive prophylactic laxatives consisting
of a stool softener (docusate sodium) and a bowel stimulant (senna), as
long as diarrhea is not present. If this measure fails, an osmotic
agent, such as sorbitol or lactulose, should be added.
A 63 year old man develops hemoptysis, weight loss and chest pain. His
initial chest x-ray shows a mass which is further confirmed by CT of the
chest. Biopsy is positive for malignancy. Which of the following would
NOT be a cause of his lung cancer?

a) Asbestos
b) Coal mining
c) Nickel mining
d) Marijuana
e) Tobacco
The correct answer is D

Explanation
Lung cancer is now the number one cause of cancer deaths in both men and
women. Initial symptoms include hemoptysis, chest pain and weight loss.
Imaging studies such as chest x-ray and chest CT are done. A
bronchoscopy guided biopsy is needed to obtain a tissue sample which
will be looked at by the pathologist to determine if the mass is malignant.

Smoking causes 87% of all lung cancers. Other causes include exposure to
the following: asbestos, radon, arsenic, chromium, coal dust, uranium,
nickel, aromatic hydrocarbons and ethers.

Marijuana, unlike tobacco and alcohol, does not appear to cause head,
neck, or lung cancer.
A 79-year-old male presents with left-sided hemiparesis. His previous
medical history is significant for long-standing hypertension and type 2
diabetes mellitus. On examination his blood pressure is 220/130 mm Hg
and his pulse rate is 96 beats/min. CT of the head shows no acute
bleeding. An EKG shows left ventricular hypertrophy with diffuse
nonspecific changes. Which one of the following would be most
appropriate with regard to his blood pressure at this time?

a) Watchful waiting
b) Reduction of systolic blood pressure (SBP) to 190 mm Hg
c) Reduction of SBP to 170 mm Hg
d) Reduction of SBP to 150 mm Hg
e) Reduction of SBP to 130 mm Hg
The correct answer is B

Explanation
Cautious reduction of systolic blood pressure by 10%?15% while
monitoring neurologic status seems to be the safest treatment goal in
the setting of acute ischemic stroke when the systolic blood pressure is
>220 mm Hg or the diastolic blood pressure is 120?140 mm Hg. According
to JNC-7, more aggressive blood pressure reduction may increase
cerebrovascular complications.
A 60-year-old male presents with back pain. The pain began approximately
2 months ago and is located in the mid-thoracic area. The patient says
he has been taking over-the-counter ibuprofen, 600 mg 4 times daily, but
the pain has been increasing in intensity. It does not radiate and is
worse at night after he goes to bed. He has had no chills or fever, but
reports losing about 10-15 lb, which he attributes to side effects of
the analgesic. He has had no bowel or bladder dysfunction.

Other than well-controlled hypertension, the patient has been well. Both
parents died in their mid-40s in an automobile accident. The patient
does not smoke, rarely drinks alcohol, and has never used illegal drugs.
For the last 20 years, the patient has worked for the city sanitation
department as a manual laborer. Recently, he has found it more difficult
to do his job because of the pain.

Based on history alone, which one of the following is the most likely
diagnosis‟

a) Spinal epidural abscess


b) Self-limited musculoskeletal back strain
c) Descending aortic dissection
d) Abdominal aortic aneurysm
e) Neoplastic disease
The correct answer is E

Explanation
Back pain is second only to upper respiratory problems as a
symptom-related reason to visit a primary care physician. About 5%-10%
of patients with acute back pain have manifestations of serious
pathology. This patient has several red flags that a serious problem is
causing his back pain. He is over 50 years old. The pain has been
gradual in onset, has been present over 6 weeks, and is unrelenting
despite analgesic use. It is in the thoracic spine, and is worse with
recumbency, as well as at night. The patient has also lost weight. All
of these findings are red flags for malignancy. The only finding present
in many patients with spinal malignancy, but not in this case, is a
previous history of cancer.

With this history, the other diagnoses listed are much less likely.
Spinal epidural abscess presents with back pain, fever, neurologic
deficits, and bowel or bladder dysfunction. Injection drug use is the
most common risk factor for hematogenous seeding of the spine.
Musculoskeletal strain is possible, but all the red flags, especially
persistence for 2 months, make this diagnosis unlikely.

The textbook description of pain in aortic dissection is the


instantaneous onset of chest pain that is maximal at the onset and is
described as knifelike, ripping, or tearing. This is present in only 50%
of patients. Dissection of the descending aorta is commonly reported as
back pain. However, many patients present with symptoms attributable to
secondary organ involvement (neurologic symptoms, abdominal pain,
gastrointestinal bleeding, or hoarseness). Common misdiagnoses for
aortic dissection include musculoskeletal chest or back pain, for example.

Abdominal aortic aneurysm (AAA) presents with a history of findings such


as hypertension, smoking, and/or diabetes. Symptoms include syncope or
lower extremity paresthesias. Patients with AAA commonly present with
back pain and are misdiagnosed as having renal colic or other
nonvascular causes of back pain.
Which one of the following is most predictive of the progression from
mild cognitive impairment to frank Alzheimer‟s dementia?
a) Self-reported memory deficits
b) Memory deficits reported by a family member
c) A normal MRI scan of the brain
d) The absence of the apolipoprotein E4 allele
The correct answer is B

Explanation
Informant-reported deficits in memory and cognitive function correlate
best with progression from mild cognitive impairment to Alzheimer‟s
dementia. Self-reported deficits confer a much smaller likelihood of
progression. Hippocampal atrophy and the presence of the E4 allele also
correlate with progression.
A 30 year old woman presents with a headache, gradual development of a
partial bitemporal hemianopia, and a history of cessation of menses
several years ago. Which one of the following is the *most likely *cause?

a) Dilated third ventricle


b) Craniopharyngioma
c) Saccular aneurysm of the circle of willis
d) Pituitary adenoma
e) Meningioma of the tuberculum sellae
The correct answer is D

Explanation
Enlargement of the pituitary gland is usually due to a tumor but may be
due to bleeding into the gland or involvement by some other disease,
such as tuberculosis or sarcoidosis. An enlarged pituitary gland may
produce symptoms such as headaches.

Because the growing gland often presses on the optic nerve, which passes
above the pituitary gland, loss of vision may occur. Vision loss often
initially affects only the upper, outermost fields of vision in both
eyes (bitemporal hemianopia). Underproduction or overproduction of
pituitary hormones may also occur. Treatment depends on the cause of the
enlargement.
For long-term therapy, the most effective control of heart rate in
atrial fibrillation, both at rest and with exercise, occurs with which
one of the following?

a) Digitalis
b) Beta-Adrenergic blockers
c) Calcium channel blockers
d) Class 1 A antiarrhythmics
The correct answer is B

Explanation
For long-term therapy, Beta-adrenergic antagonist drugs provide the most
effective control of heart rate in atrial fibrillation, both at rest and
during exercise. Although calcium channel blockers also lower heart rate
both at rest and with exercise, they are not as effective as
Beta-blockers. Digitalis is primarily effective in controlling the heart
rate at rest, and often does not adequately control heart rate with
exercise. The Class 1 antiarrhythmics are most useful in maintaining
sinus rhythm and, in fact, may paradoxically increase heart rate.
A 72 year old male presents to your clinic in atrial fibrillation with a
rate of 132 beats/min. He has hypertension, but no history of congestive
heart failure or structural heart disease. He is otherwise healthy and
active. The best initial approach to his atrial fibrillation would be

a) Rhythm control with antiarrythmics and warfarin (Coumadin) only if


he cannot be consistently maintained in sinus rhythm
b) Rhythm control with antiarrythmics and warfarin regardless of
maintenance of sinus rhythm
c) Ventricular rate control with digoxin, and warfarin for anticoagulation
d) Ventricular rate control with digoxin and aspirin for anticoagulation
e) Ventricular rate control with a calcium channel blocker or
Beta-blocker, and warfarin for anticoagulation
The correct answer is E

Explanation
Five recent randomized, controlled trials have indicated that in most
patients with atrial fibrillation, an initial approach of rate control
is best. Patients who were stratified to the rhythm control arm of the
trials did NOT have a morbidity or mortality benefit and were more
likely to suffer from adverse drug effects and increased
hospitalizations. The most efficacious drugs for rate control are
calcium channel blockers and Beta-blockers. Digoxin is less effective
for rate control and should be reserved as an add-on option for those
not controlled with a Beta-blocker or calcium channel blocker, or for
patients with significant left ventricular systolic dysfunction.

In patients 65 years of age or older or with one of more risk factors


for stroke, the best choice for anticoagulation to prevent
thromboembolic disease is warfarin. Of note, in patients who are
successfully rhythm controlled and maintained in sinus rhythm, the
thromboembolic rate is equivalent to those managed with a rate control
strategy. Thus, the data suggest that patients who choose a rhythm
control strategy should be maintained on anticoagulation regardless of
whether they are consistently in sinus rhythm.
A 32-year-old female with Crohn‟s disease presents to your office.
Despite optimal doses of mesalamine (Pentasa), she is still experiencing
mild to moderate activity of her disease. You decide to add antibiotic
therapy to her regimen.

Which one of the following would be most appropriate?

a) Amoxicillin
b) Azithromycin (Zithromax)
c) Metronidazole (Flagyl)
d) Doxycycline
e) Trimethoprim/sulfamethoxazole (Bactrim, Septra)
The correct answer is C

Explanation
This patient has mild to moderate Crohn‟s disease. Initial treatment
should be a salicylate such as mesalamine or sulfasalazine. If her
disease is not adequately controlled, metronidazole has been shown to be
an effective treatment (SORT evidence level A: randomized controlled
trial). Ciprofloxacin has also been shown to be effective (SORT evidence
level B: lower quality randomized controlled trial).

A sickle cell patient presents with severe abdominal pain and fever. He
is given IV fluids. Blood cultures are obtained. What is your next step?

a) Urine gram stain and culture


b) Hydroxyurea
c) Transfusion
d) Parenteral opiate
e) Oxygen supplementation
The correct answer is D

Explanation
Failure to treat acute pain aggressively and promptly may lead to
chronic pain syndrome.
Treatment of pain crises is primarily pharmacologic in nature, and
opioids represent the mainstay of therapy. Patients with severe pain
should be given a parenteral opiate in full therapeutic doses at fixed
intervals (and not as needed) until pain diminishes at which time the
opiate is tapered and then stopped and oral analgesic therapy is
instituted.
Hydration is another mainstay of treatment.
For all types of pain, incentive spirometry is recommended. For frequent
and severe pain, long-term hydroxyurea (HU) is presently the accepted
treatment. For HU nonresponders, chronic transfusions for a limited
period may be an option. Management of constant pain is extremely
difficult, and expert advice should be obtained.

>Perform urinalysis if the patient has fever or signs of urinary tract


infection (UTI). Patients with sickle cell anemia often have hematuria
and isosthenuria. If signs of urinary tract infection are present,
obtain a urine Gram stain and culture.
>Transfusions are not needed for the usual anemia or episodes of pain
associated with sickle cell disease. Urgent replacement of blood is
often required for sudden severe anemia due to ASS, parvovirus B19
infection, or in hyperhemolytic crises.
>Oxygen supplementation is only beneficial if the patient has hypoxia.

A 57 year old man is brought to the emergency department by the rescue


squad after he was found lying unconscious in the street. No other
history is available. On physical examination he has a temperature of
40.0C (104.0F) and marked nuchal rigidity. While awaiting the results of
a lumbar puncture, which of the following is the most appropriate
intravenous pharmacotherapy?

a) Ceftriaxone
b) Ciprofloxacin
c) Glucocorticoids
d) Penicillin
e) Ticarcillin
The correct answer is A

Explanation
This patient has signs (nuchal rigidity) and symptoms (fever) of
meningitis. The standard of care is to initiate antibiotic therapy
immediately, even before a diagnostic lumbar puncture is taken. The
antibiotic of choice must both cover the likely offending organism and
have good penetration into the CSF. Ceftriaxone, a third generation
cephalosporin meets both of these requirements.

A 39 year old white male is seen in your office with a history of sudden
painful swelling of the right parotid gland. The patient has a
temperature of 38.2°C (100.8°F). The parotid gland is tender on
examination. Which one of the following would be most appropriate at
this point?

a) Observation only
b) Asking about pets in the household
c) Tuberculin test
d) A CT scan
e) Antibiotics
The correct answer is E

Explanation
The patient has acute suppurative sialadenitis which is caused by
coagulase-positive Staphylococcus aureus, Streptococcus pneumoniae, and
other bacteria. The patient therefore needs treatment with antibiotics.
Tuberculosis is a rare cause. Cat scratch disease involves the lymph
glands, not the salivary glands. A CT scan may be indicated if there is
no improvement, or if a tumor is suspected.
Which of the following foods are ok for a patient with Celiac disease to
eat?

a) Wheat
b) Barley
c) Oats
d) Rice and corn
The correct answer is D

Explanation
Celiac disease (nontropical sprue, gluten enteropathy, celiac sprue) is
a hereditary intolerance to gluten, a protein found in wheats, barley,
and oats, which causes characteristic changes in the lining of the small
intestine, resulting in malabsorption.
People with celiac disease must exclude all gluten from their diet,
since eating even small amounts may cause symptoms. The response to a
gluten-free diet is usually rapid. Once gluten is avoided, the brushlike
surface of the small intestine and its absorptive function return to
normal. Gluten is so widely used in food products that people with
celiac disease need detailed lists of foods to be avoided and expert
advice from a dietitian. Gluten is found, for example, in commercial
soups, sauces, ice cream, and hot dogs.

Several grains and starch sources are considered acceptable for a


gluten-free diet. The most frequently used are corn, potatoes and rice.

A 75 year old white female presents with back pain of several months‟
duration, which is worsened by movement. Her examination is unremarkable
except for mild pallor. She takes furosemide (Lasix) for hypertension.
Laboratory Findings

Hemoglobin--------------------------------10.0 g/dL (N 12.0-16.0)


Serum creatinine--------------------------------2.0 mg/dL (N 0.6-1.5)
BUN--------------------------------40 mg/dL (N 8.5-25)
Serum uric acid--------------------------------8.0 mg/dL (N 3.0-7.0)
Serum calcium--------------------------------12.0 mg/dL (N 8.5-10.5)
Total serum protein--------------------------------9.8 g/dL (N 6.0-8.4)
Globulin--------------------------------6.1 g/dL (N 2.3-3.5)
Albumin--------------------------------3.7 g/dL (N 3.5-5.0)
Serum IgG--------------------------------3700 mg/dL (N 639-1349)
Urine--------------------------------positive for Bence-Jones protein

Which one of the following would be most appropriate at this point?

a) Repeat the physical examination and laboratory evaluation every 6 months


b) Take the patient off the diuretic and repeat the laboratory
evaluation in 1 month
c) Obtain a bone scan
d) Obtain a bone marrow examination
e) Begin therapy with tamoxifen, 20 mg daily
The correct answer is D

Explanation
This patient has typical symptoms and laboratory findings of multiple
myeloma, which accounts for 1% of all malignant diseases and has a mean
age at diagnosis of 61 years. The diagnosis is confirmed by a bone
marrow examination showing > 10% plasma cells in the marrow. The serum
level of M-protein is typically > 3 g/dL. A bone scan is inferior to
conventional radiography and should not be used. Tamoxifen is indicated
for the treatment of breast cancer, which is unlikely given the physical
and laboratory findings in this case.
Positron emission tomography (PET) scans used to detect cancer most
commonly use a radioactive tracer tagged to a molecule that is an
analogue of

a) Oxygen
b) Glucose
c) Hemoglobin
d) Nitrate
e) Phosphate
The correct answer is B

Explanation
The fluorine-18-labeled tracer fluorodeoxyglucose (FDG) is a glucose
analogue taken up by metabolically active cells using glucose as a
substrate for their metabolism. This enables the PET scanner to detect
metabolically active tissues such as cancer metastases
A healthy 45-year-old male comes to your office for a routine visit.
Findings are normal on a review of systems and physical examination.
After you discuss the findings and provide routine counseling about a
healthy lifestyle, the patient asks if he should have a full-body CT
scan just to make sure he is healthy.

Which one of the following would be appropriate advice?


a) This is a reasonable option, but most insurance companies will not
pay for it
b) The chance of finding an abnormality is very low, but for a person
anxious about his
or her health the reassurance is worth the expense
c) The amount of radiation exposure from low-dose full-body CT is minimal
d) In a healthy patient, incidental findings will lead to unnecessary
biopsies, anxiety, and
significant radiation exposure
The correct answer is D

Explanation
With financial incentives driving the health care industry, full-body CT
has been promoted by private scanning facilities. However, the
Preventive Services Task Force, the College of Radiology, and many other
organizations that have studied full-body CT screening are opposed to
it. No study has shown the final outcome of such screening to be in the
best interest of the patient, and it may actually be detrimental.

Although the initial cost of $1000 may seem high by itself, the final
cost also includes the expense of further testing due to false-positive
findings, the time taken off work for the initial test, and the time
required for any further tests and consultations that might take place.
Various studies have shown that significant disease is found in
1.9%?2.0% of those scanned, yet 87%?91% of all patients scanned will
have positive findings. Of these, 37% will be asked to undergo further
studies to prove the findings are benign. Findings such as noncalcified
pulmonary nodules will either necessitate procedures such as lung needle
biopsies or wedge resections, or will be left alone, causing increased
anxiety to the patient and also affecting the patient‟s future insurability.

The radiation exposure from a full-body CT scan is estimated to be


equivalent to 500 chest radiographs, or 100 chest radiographs for
low-dose scanning.
A hunter develops an inflamed papule on one finger. Several days later,
the patient develops severe illness with atypical pneumonia. On physical
exam his lymph nodes of the axilla of the affected arm are enlarged.
Reduced breath sounds and occasional rales are heard. Splenomegaly is
noted. He tells you he has handled rabbits recently. Which of the
following is the most likely diagnosis‟

a) Actinomycosis
b) Brucellosis
c) Plague
d) Tularemia
The correct answer is D

Explanation
This is tularemia, the causative organism of which is Francisella
tularensis. The classic clue in test questions is exposure to rabbits,
especially in the wild. Untreated cases tend to last 3 to 4 weeks before
resolving. Streptomycin is the antibiotic of choice; gentamicin and
chloramphenicol can alternatively be used. Deaths occur in about 6% of
untreated cases and are very rare in treated cases.
A 20-year-old female presents with a sudden onset of fever, chills, and
headache of 2 days‟ duration, and now has a pink blanching rash. The
rash covers most of her body, including the palms of her hands and the
soles of her feet, but not including her face. She recently returned
from a camping trip, but has had no recent contact with anyone who has
been ill. Which one of the following would be the most appropriate
treatment for this patient‟s symptoms‟

a) Doxycycline (Adoxa), 100 mg twice daily for 10 days


b) Azithromycin (Zithromax), 500 mg daily for 3 days
c) Cephalexin (Keflex), 500 mg twice daily for 10 days
d) Penicillin, 500 mg twice daily for 10 days
e) Reassurance
The correct answer is A

Explanation
This is a classic description of rickettsial illness: a history of
outdoor activity, the sudden onset of fever, chills, and rash on the
palms of the hands and the soles of the feet. Penicillin, cephalexin,
and azithromycin do not cover rickettsia. Reassurance would be
inappropriate because this condition can be life threatening and should
always be treated.

Which of the following is not an aggravating factor of congestive heart


failure?

a) Hypertension
b) Thyrotoxicosis
c) Alcohol
d) Inactivity
e) Arrhythmia
The correct answer is D

Explanation
Congestive heart failure can be caused by a number of conditions that
damage the heart muscle, including a heart attack or heart infection;
coronary artery disease; excessive alcohol consumption; an over- or
under-active thyroid gland; and untreated high blood pressure.

Congestive heart failure can also be caused by damage to or


abnormalities in the valves inside the heart, which regulate blood flow
out of the heart, and genetic conditions, such as hypertrophic
cardiomyopathy, in which the heart muscle becomes thickened due to
enlarged heart muscle cells.

Factors such as high fever, anemia, lung infections, pulmonary embolism,


high salt or fluid intake, overexertion, obesity, stress, and rapid
heartbeat can worsen or trigger congestive heart failure in people with
already weakened hearts.
A 24 year old woman with a seizure disorder treated with phenytoin is
noted to have the following findings at 15 weeks of pregnancy:

Hemoglobin (Hgb) 93 g/L (Normal 123-157 g/L)


Hematocrit (HCT) 29% (Normal 37-46%)
Mean corpuscular volume (MCV) 105 fL (Normal 80-100)

Which one of the following is the most likely cause for these findings‟
a) Sickle cell trait
b) Iron deficiency
c) Physiologic anemia
d) Folate deficiency
e) Thalassemia
The correct answer is D

Explanation
This patient has developed a macrocytic anemia. Causes include vitamin
B12 and folate deficiency.

Phenytoin (Dilantin) can reduce serum folate levels, occasionally


leading to megaloblastic anemia, and possibly contributing to
neurological side effects and mental status changes. Folic acid
supplements may reduce phenytoin side effects. Pregnant women taking
phenytoin may be especially at risk from reduced folate levels.

Which one of the following would be a contraindication to initiating


beta-blocker therapy in a patient with congestive heart failure?

a) Class III heart failure (symptoms with less than ordinary exertion)
b) Left ventricular systolic dysfunction (ejection fraction < 40%)
c) Current digoxin use
d) Recent hospital admission for decompensated heart failure
The correct answer is D

Explanation
In general, it is advisable to avoid initiating Beta-blocker therapy
during or immediately after admission for decompensated heart failure.
None of the other situations would contraindicate its initiation.
A 42 year old man presents with dark skin and a palpable liver. His
father died of cirrhosis. What is the most likely diagnoses‟

a) Wilson disease
b) Hemochromatosis
c) Diabetes mellitus
d) Hepatic cirrhosis
The correct answer is B

Explanation
Hemochromatosis is an inherited disorder characterized by excessive iron
accumulation causing tissue damage. Symptoms do not develop until organ
damage, often irreversible, develops. Symptoms include fatigue,
hepatomegaly, bronze skin pigmentation, loss of libido, arthalgias, and
manifestations of cirrhosis, diabetes, or cardiomyopathy.

90% of patients will present with excessive skin pigmentation.


Diagnostic testing will reveal that the serum iron is increased (> 300
mg/dL). Phlebotomy is the simplest method of excess iron removal in most
cases.

The other choices would not typically present with dark skin pigmentation.
A 22 year old male has acute lower back pain without paresthesias or
other neurologic signs. There is no lower extremity weakness. Which
treatment has been shown to be of the most benefit initially?
a) Complete bed rest for 2 weeks
b) Bed rest plus local injection of steroids
c) A low back strengthening program
d) Resumption of physical activity as tolerated
The correct answer is D

Explanation
Recent studies have shown superior benefits to allowing patients with
acute low back pain, without sciatic involvement, to return to normal
activities as tolerated. This was better than either bed rest or a back
exercise program. Injections would be considered only if conservative
therapy fails.
A 44-year-old female comes to your office with chest pain of several
days‟ duration. She describes the pain as sharp and stabbing, and
indicates that it is located at the left sternal border; it is increased
by coughing and palpation. There is no family history of heart disease,
nor is there a personal history of diabetes, hypertension, smoking, or
hyperlipidemia. A physical examination, an EKG, and chest radiographs
are all normal.

Further diagnostic studies should include

a) a treadmill exercise test with EKG and blood pressure monitoring


b) a stress echocardiogram
c) stress myocardial perfusion imaging
d) referral for cardiac catheterization
e) no additional tests
The correct answer is E

Explanation

This patient exhibits atypical (noncardiac) chest pain and has no risk
factors for coronary artery disease (CAD). Since the likelihood of
coronary disease is very low, stress testing is not indicated. For
patients with typical chest pain and risk factors, the probability of
CAD is high and it is usually best to proceed directly to cardiac
catheterization. In individuals with an intermediate probability of CAD
who are able to exercise, the choice is treadmill testing if they have
an interpretable EKG, no evidence of left ventricular dysfunction, and
no history of previous coronary artery bypass surgery. If the baseline
EKG is not interpretable (due to left bundle branch block, early
repolarization, left ventricular hypertrophy, or digoxin use) then an
exercise test with imaging (nuclear or echocardiographic) is indicated.
Those unable to exercise can have pharmacologic stress testing with imaging.

Which one of the following is most effective in slowing lung function


decline in COPD?

a) Theophylline
b) Inhaled b-agonists
c) Inhaled corticosteroids
d) Antibiotics
e) Smoking cessation
The correct answer is E
Explanation
In patients with COPD, smoking cessation has been shown to slow the
decline of lung function and even improve lung function. Theophylline,
inhaled b-agonists, inhaled corticosteroids, and antibiotics will not
provide this benefit.
A 60-year-old white female presents for her annual physical examination.
She tells you that since her last visit she has begun taking ginkgo
biloba to improve her memory. She takes no other medications.

The evidence supporting a benefit from ginkgo biloba is best for which
one of the following?

a) Tinnitus
b) Benign prostatic hyperplasia
c) Dementia
d) Intermittent claudication
The correct answer is C

Explanation
Studies support the use of ginkgo biloba for tinnitus, dementia, and
intermittent claudication. Those examining its use in patients with
dementia are of the highest quality, and consistently demonstrate that
ginkgo is more effective than placebo in delaying cognitive decline. The
studies examining its use in tinnitus, claudication, and cognitive
improvement in people with normal mentation are limited by bias and
small numbers of studies with inconsistent results. Ginkgo is not
appropriate for the treatment of benign prostatic hyperplasia.
A 60-year-old white female is in the intensive-care unit for respiratory
failure secondary to community-acquired pneumonia. She smokes 2 packs of
cigarettes per day, but has no other significant medical or personal
history. A medical student is making rounds with you and asks if the
patient‟s thyroid function should be tested.

Which one of the following is true regarding thyroid testing in this


situation?

a) Most patients who are critically ill should be tested for occult
hypothyroidism
b) Serum TSH is a good screening test for thyroid disease in critically
ill patients
c) low TSH level in this patient would indicate hyperthyroidism
d) A low T_4 level is a frequent finding in patients hospitalized in
the intensive-care unit
e) A low T_3 level should be treated with thyroid replacement in
hospitalized critically ill
patients
The correct answer is D

Explanation
Many patients hospitalized with nonthyroid illness in the intensive care
unit have abnormalities on thyroid testing. Low serum concentrations of
both thyroxine (T_4 ) and triiodothyronine (T_3 ) are common, and TSH
may be decreased as well. Thyroid function should not be assessed in
seriously ill patients unless a thyroid problem is strongly suspected.
Measurement of serum TSH alone is inadequate for the evaluation of
thyroid function in critically ill patients.
A 37 year old male presents with new-onset low back pain after starting
a new job in a meat packing plant. He denies trauma, fever, weight loss,
saddle anesthesia, or radicular symptoms. His examination is
unremarkable except that there is pain with forward flexion of the
lumbar spine.

Initial management should include

a) NSAIDs alone
b) NSAIDs and bed rest
c) NSAIDs and corticosteroid injection
d) Corticosteroid injection alone
e) Corticosteroid injection and bed rest
The correct answer is A

Explanation
Nonspecific low back pain is a very common problem encountered in
primary care. Patients should be evaluated for red flags such as trauma,
weight loss, loss of sensation in the perineal area, bowel or bladder
dysfunction, and fever, which may indicate a more serious underlying
etiology. A 2006 Cochrane review showed that NSAIDs are effective for
treating nonspecific, acute low back pain. An earlier Cochrane review
showed that bed rest provides no pain relief for patients with low back
pain. Corticosteroid injection is not indicated for the initial
treatment of low back pain.
Patients often use echinacea for the prevention and treatment of

a) Memory loss
b) Upper respiratory symptoms
c) Gastrointestinal illnesses
d) Depression
e) Fatigue
The correct answer is B

Explanation
Echinacea is a genus of native North American plants commonly known as
purple coneflower. It has been recommended as a prophylactic treatment
for upper respiratory infection, and is widely used for this indication,
although it appears to be relatively ineffective. The research is
difficult to evaluate because of the hererogeneity of the products used
in various studies.
A 70-year-old male sees you because of slowly increasing problems with
COPD. He has had frequent exacerbations requiring emergency department
visits. He currently uses a tiotropium (Spiriva) inhaler once a day, as
well as an albuterol (Proventil) inhaler, 2 puffs 4 times a day as
needed. An examination shows decreased breath sounds throughout, and an
oxygen saturation of 92%. 1. Spirometry shows he has severe COPD (stage
III); his FEV /FVC ratio is 65% of predicted and his FEV1 is 45% of
predicted.

The most reasonable change in treatment would be to add

a) oxygen, 2 L/min while sleeping


b) inhaled fluticasone (Flovent), 2 puffs twice daily
c) oral low-dose prednisone daily
d) oral theophylline (Uniphyl) twice daily
The correct answer is B

Explanation
This patient is suffering from severe COPD (stage III) and has a history
of frequent exacerbations. The addition of a corticosteroid inhaler for
patients with severe disease has been found to significantly decrease
the number of exacerbations, but has no effect on overall mortality.
Side effects of oral candidiasis and easy bruising of the skin are
increased.

Continuous oxygen has been shown to improve overall mortality and


endurance in patients with an oxygen saturation of 88% or less, but has
not been shown to improve quality of life in those with mild hypoxemia
or if used only at night.

Oral prednisone has been shown to be effective when used to treat acute
exacerbations, but when used on a chronic basis it is no more effective
than corticosteroid inhalers. Chronic oral prednisone is also associated
with significant side effects, and therefore is not generally recommended.

Oral theophylline has not been shown to be of benefit in either


preventing exacerbations or improving quality of life, and has
significant side effects of gastrointestinal toxicity, seizures, and
arrhythmias. It should be reserved for carefully selected patients only.

Raloxifene (Evista)

a) Is used to manage hot flashes


b) Increases bone density
c) Stimulates breast tissue
d) Stimulates endometrial proliferation
e) Raises LDL and total cholesterol levels
The correct answer is B

Explanation
Raloxifene is a selective estrogen-receptor modulator. It was initially
approved by the FDA just to treat osteoporosis because it increases bone
density. It does not stimulate either endometrium or breast tissue. It
does lower both LDL and total cholesterol. However, it can actually
cause hot flashes, and is therefore not useful for managing menopausal
symptoms.

A 62 year old woman has demonstrated increasing loss of recent memory


over a 5 year period, now associated with reduced affect and nominal
dysphasia. She dresses neatly and social amenities are preserved. There
are no localizing motor or sensory findings. Which one of the following
is the most likely diagnosis‟

a) Cerebral vascular disease


b) Hydrocephalus
c) Huntington's disease
d) Alzheimer's disease
e) Creutzfeldt-Jakob disease
The correct answer is D
Explanation
In Alzheimer's disease, the ability to remember, think, understand,
communicate, and control behavior progressively declines because brain
tissue degenerates. The symptoms of Alzheimer's disease are similar to
those of other dementias. They include memory loss, changes in
personality, problems using language, disorientation, difficulty doing
daily activities, and disruptive behavior. A person with Alzheimer's
disease may not have all the symptoms.

Symptoms usually begin subtly. People may not notice any changes at
first, depending on what activities they are involved in. At some point,
people with Alzheimer's disease may notice they are not doing their job
or activities quite as well as in the past.

In most people with Alzheimer's disease, the first sign is forgetting


recent events. But the disease may begin with changes in personality.
For example, people may become emotionally unresponsive, depressed, or
unusually fearful or anxious. Or emotions may rapidly and unpredictably
change from one extreme to another. Early in the disease, people have
difficulty using language. They may use a general word or many words
rather than a specific word, use words incorrectly, or be unable to find
the right word. They become less able to use good judgment and think
abstractly.
A 67 year old male patient with history of chronic hypertension, obesity
and cigarette smoking experienced fluid retention along with symptoms of
heart failure. Which of the following medications could worsen his heart
failure?

a) Digitalis
b) Furosemide
c) Vasopressin
d) Lisinopril
e) Metoprolol
The correct answer is C

Explanation
In heart failure, Vasopressin worsens heart failure by
causing vasoconstriction of arteries and veins, potentially contributing
to remodeling of the left ventricle and causing fluid retention and
worsening of hyponatremia.

A. Digitalis reduces the force of contraction and cardiac output and


helps in reduction of the risk of heart failure.
B. Furosemide is decreases the uptake of the water or fluids from the
kidney and decreases the retention there by reducing the risk of heart
failure.
D. Lisinopril reduces the risk of heart failure and one of the best
medications used for heart failure. It is an ACE inhibitor, which
inhibits the conversion of angiotensin I to angiotensin II, which is a
potent vasoconstrictor.
E. Metoprolol is a beta-blocker which decreases the risk of heart
failure by blocking the beta-receptors.

A 38 year old man, previously in good health, suddenly develops severe


abdominal pain radiating to the left groin and associated with nausea,
perspiration, and the need for frequent urination. He is restless,
tossing in bed, but has no abnormal findings. Which one of the following
is the most likely diagnosis‟
a) Herpes zoster
b) Left ureteral calculus
c) Sigmoid diverticulitis
d) Torsion of the left testicle
e) Retroperitoneal hemorrhage
The correct answer is B

Explanation
Urinary calculi are solid particles in the urinary system. Symptoms,
such as severe pain, often accompanied by nausea and vomiting, and
sometimes gross hematuria, usually occur when calculi pass into the
ureter, cause obstruction, or both. Pain (renal colic) is of variable
intensity but is typically excruciating and intermittent, often occurs
cyclically, and lasts 20 to 60 min. Nausea and vomiting are common. Pain
in the flank or kidney area that radiates across the abdomen suggests
upper ureteral or renal pelvic obstruction. Pain that radiates along the
course of the ureter into the genital region suggests lower ureteral
obstruction.

On examination, patients may be in obvious extreme discomfort, often


ashen and diaphoretic. Patients with renal colic may be unable to lie
still and may pace, writhe, or constantly shift position. The abdomen
may be somewhat tender on the affected side as palpation increases
pressure in the already-distended ureter, but peritoneal signs
(guarding, rebound, rigidity) are lacking. For some patients, the first
symptom is hematuria or either gravel or a calculus in the urine. Other
patients may have symptoms of a UTI, such as fever, dysuria, or cloudy
or foul-smelling urine.
Most cases of chronic urticaria are

a) caused by infection
b) caused by an allergic reaction
c) caused by an autoimmune response
d) idiopathic
e) associated with an anxiety disorder
The correct answer is D

Explanation
Chronic urticaria was once considered to be a manifestation of an
anxiety disorder or an allergic or idiosyncratic reaction to foods, food
additives, or food dyes. There is no good evidence to support these
suppositions. The likelihood of there being infectious causes of chronic
urticaria, such as Helicobacter pylori, is still being debated, but is
unlikely. An autoimmune mechanism appears to be the likely cause in a
subpopulation of patients, but 60% of cases appear to be idiopathic.
A 45-year-old male sees you for follow-up after a pre-employment
physical examination reveals blood in his urine. He brings a copy of a
urinalysis report that shows 3?5 RBCs/hpf. He has not seen any gross
blood himself. He is asymptomatic, is on no medications, and does not
smoke. You perform a physical examination, with normal findings. A
repeat urinalysis confirms the presence of red blood cells but is
otherwise normal. Which one of the following would be most appropriate
at this point?

a) Observation and reassurance


b) A repeat urinalysis in 6 months
c) Urine cytology only
d) Ultrasonography of the kidneys and urine cytology only
e) Ultrasonography of the kidneys, urine cytology, and cystoscopy
The correct answer is E

Explanation
The current guidelines define clinically significant microscopic
hematuria as 3 RBCs/hpf. Microscopic hematuria is frequently an
incidental finding, but may be associated with urologic malignancy in up
to 10% of adults.

The upper urinary tract should be evaluated in this patient. There are
no clear evidence-based imaging guidelines for upper tract evaluation;
therefore, intravenous urography, ultrasonography, or CT can be
considered. Ultrasonography is the least expensive and safest choice
because it does not expose the patient to intravenous radiographic
contrast media.

Urine cytology and cystoscopy are used routinely to evaluate the lower
urinary tract. The AUA recommends that patients with microscopic
hematuria have radiographic assessment of the upper urinary tract,
followed by urine cytology studies. The AUA also recommends that all
patients older than 40 and those who are younger but have risk factors
for bladder cancer undergo cystoscopy to complete the evaluation.
Cystoscopy is the only reliable method of detecting transitional cell
carcinoma of the bladder and urethra.

A 35 year old man who is a new city employee was driving a city-owned
car when he was struck from behind by another car. He experienced
immediate neck and shoulder pain and was seen and examined by you in the
emergency department. Physical examination showed slight cervical muscle
spasm. Cervical spine and shoulder x-ray films at that time were normal.
You reassured him that no structural damage was evident on the studies.
He was treated with nonsteroidal anti-inflammatory drugs (NSAIDs) and
was scheduled for a return visit in the office in 2 weeks.

Today, at the 4-week return visit, he insists his condition has not
improved and demands everything be done to "find out why I still hurt."
Physical examination including neurologic examination, is normal. Repeat
x-ray films of the cervical spine are normal. At this time it is most
appropriate to

a) Inform the patient that no additional studies are needed


b) Order a CT scan of the cervical spine
c) Order an MRI of the cervical spine
d) Order studies the patient requests
e) Refer him to a neurologist
The correct answer is A

Explanation
Despite the patient requests, there is nothing at this time to suggest
any additional work-up. It is important to communicate clearly to the
patient that you are not giving up, and that it is your assessment that
no further testing is needed at this time. Assure him that you will
continue to follow up this problem with him.

The other choices, any further testing or consults would have a low
yield result, and would cause great expense in a situation where there
is low suspicion and a possibility of false positives.
Which one of the following should be considered in geriatric patients
when a long airline flight is planned?

a) Hypoxia with desaturation


b) Temporal disorientation
c) Barotitis
d) Dehydration
e) Deep venous thrombophlebitis
The correct answer is E

Explanation
“Coach class thrombosis”, deep venous thrombosis or pulmonary embolism
associated with cramped conditions on long airline flights, is in fact a
real phenomenon. However, the risk is small and only those already at
increased risk of venous thromboembolism need to be concerned about it.

The known complications of venous stasis must be avoided by the


prophylactic use of ambulation and exercises during long flights.
Patients at increased risk or presently on antithrombotic medications
must be carefully monitored prior to their trip.
Which one of the following is a risk factor for perioperative
ventricular arrhythmias‟

a) Supraventricular tachycardia
b) Congestive heart failure
c) Age > 60
d) Premature atrial contractions
e) Past history of hyperthyroidism
The correct answer is B

Explanation
Significant predictors of intra-operative and peri-operative ventricular
arrhythmias include preoperative ventricular (not supraventricular)
ectopy, a history of congestive heart failure, and a history of
cigarette smoking. Age and a history of hyperthyroidism are not
significant predictors of peri-operative ventricular arrhythmias.
Within a 2-week period you see an unusually large number of patients of
all ages who complain of painless loss of central vision. You refer them
appropriately to an ophthalmologist. The feedback you get is that most
of these persons have bilateral central scotomas caused by an
ophthalmoscopically visible macular defect. Which one of the following
events would most likely explain this problem?

a) A solar eclipse
b) A factory accident exposing persons over a wide area to dangerous
levels of carbon monoxide
c) A severe dust storm
d) The recent winner-take-all boxing tournament
e) The annual community picnic where lots of potato salad was consumed
The correct answer is A

Explanation
Direct observation of the sun without an adequate filter, which often
occurs during a solar eclipse, results in a specific type of radiation
injury termed solar (eclipse) retinopathy. The lens system of the eye
focuses the sun‟s light onto a small sport on the macula, usually in one
eye only, producing a thermal burn. The resulting retinal edema may
clear spontaneously with minor functional loss, or it may cause
significant tissue atrophy, leaving a defect seen with an opthalmoscope
as a macular hole. This macular injury produces a permanent central
scotoma. Visual blurring and difficulty with light perception are
reversible manifestations of the tissue hypoxia associated with carbon
monoxide poisoning. Dust and other particulate matter can produce injury
to the cornea and conjunctiva. These lesions are painful, usually
prompting medical attention and appropriate treatment. While severe
contusions to the globe and periorbital structures can produce retinal
detachment, the clinical vignette does not support a boxing-type injury.
Although potatoes have eyes, there is no other significant relationship
between potato salad and ophthalmologic disease.
A female patient with positive ANA, presents with ankle edema,
arthralgia, protein in her urine and a butterfly rash on her face. What
is the most likely diagnosis‟

a) Lupus
b) Scleroderma
c) Multiple sclerosis
d) Rheumatoid arthritis
The correct answer is A

Explanation
Systemic lupus erythematosus is a chronic, multisystem, inflammatory
disorder of probable autoimmune etiology, occurring predominantly in
young women. Common manifestations include arthralgias and arthritis;
malar and other skin rashes; pleuritis or pericarditis; renal or CNS
involvement; and hematologic cytopenia.

Diagnosis requires clinical and serologic criteria. The fluorescent test


for ANA is the best screen for SLE. Treatment of severe ongoing active
disease requires corticosteroids, often hydroxychloroquine, and
sometimes immunosuppressants.
An asymptomatic 60-year-old male who has a history of hypertension comes
to your office for the first time. He has not taken antihypertensive
medication in over a year and cannot recall the name of the drug
prescribed. His examination is unremarkable. A CBC, urinalysis, and
chemistry profile are all normal. His blood pressure is 159/92 mm Hg. An
EKG shows a sinus rhythm, but is compatible with left ventricular
hypertrophy. An echocardiogram confirms this finding.

Which one of the following would be most appropriate for the management
of this patient‟s hypertension?

a) Hydralazine
b) Minoxidil
c) Doxazosin (Cardura)
d) Amlodipine (Norvasc)
e) Lisinopril (Prinivil, Zestril)
The correct answer is E

Explanation
Regression of left ventricular hypertrophy is an important therapeutic
target, as successful treatment significantly reduces cardiovascular
events. Optimal blood pressure control is the goal, but it appears that
the most helpful drugs in this setting would be an ACE inhibitor, a
thiazide diuretic, a ß-blocker, or an angiotensin receptor blocker (ARB).
The goal LDL level for patients with diabetes mellitus is

a) <100 mg/dL
b) <130 mg/dL
c) <160 mg/dL
d) <200 mg/dL
The correct answer is A

Explanation
<100 mg/dL has been set as the LDL goal in patients with coronary heart
disease and coronary heart disease equivalents, including diabetes
mellitus.
A positive spot urine test for homovanillic acid (HVA) and
vanillylmandelic acid (VMA) is a marker for which one of the following?

a) Hepatoblastoma
b) Wilms‟ tumor
c) Lymphoma
d) Malignant teratoma
e) Neuroblastoma
The correct answer is E

Explanation
Tumor markers are useful in determining the diagnosis and sometimes the
prognosis of certain tumors. They can aid in assessing response to
therapy and detecting tumor recurrence. Serum neuron-specific enolase
(NSE) testing, as well as spot urine testing for homovanillic acid (HVA)
and vanillylmandelic acid (VMA), should be obtained if neuroblastoma or
pheochromocytoma is suspected; both should be collected before surgical
intervention. Quantitative beta-human chorlonic gonadotropin (hCG)
levels can be elevated in liver tumors and germ cells tumors.
alpha-Fetoprotein is excreted by many malignant teratomas and by liver
and germ cell tumors.
Which one of the following is more likely to occur with glipizide
(Glucotrol) than with metformin (Glucophage)?

a) Lactic acidosis
b) Hypoglycemia
c) Weight loss
d) Gastrointestinal distress
The correct answer is B

Explanation
Metofrmin is a biguanide used as an oral antidiabetic agent. One of its
main advantages over some other oral agents is that it does not cause
hypoglycemia. Lactic acidosis, while rare, can occur in patients with
renal impairment. In contrast to most other agents for the control of
elevated glucose, which often cause weight gain, metformin reduces
insulin levels and more frequently has a weight-maintaining or even a
weight loss effect. Gastrointestinal distress is a common side-effect of
metformin, particularly early in therapy.

You might also like